You are on page 1of 456

Downloaded From : www.EasyEngineering.

net

ww
w.E
a syE
ngi
nee
rin
g.n
et

**Note: Other Websites/Blogs Owners Please do not Copy (or) Republish


this Materials, Students & Graduates if You Find the Same Materials with
EasyEngineering.net Watermarks or Logo, Kindly report us to
easyengineeringnet@gmail.com

Downloaded From : www.EasyEngineering.net


y
Downloaded From : www.EasyEngineering.net

o
u
rs
m
a
h
b
o
o
b
.w
o
rd
p
re
s
s
.c
SSC
o
m
ww
w.E
asy
En
gin
English Language eer
i n g.n
Guide for CGL/CHSL/MTS/GD Constable/Stenographer
et

Fully
Solved

Downloaded From : www.EasyEngineering.net


Downloaded From : www.EasyEngineering.net

• Head Office : B-32, Shivalik Main Road, Malviya Nagar, New Delhi-110017

• Sales Office : B-48, Shivalik Main Road, Malviya Nagar, New Delhi-110017
Tel. : 011-26691021 / 26691713

ww
w.E
Typeset by Disha DTP Team
asy
En
gin
eer
ing
.ne
DISHA PUBLICATION
t
All RightS Reserved

© Copyright Publisher

No part of this publication may be reproduced in any form without prior permission of the publisher. The author and the
publisher do not take any legal responsibility for any errors or misrepresentations that might have crept in. We have
tried and made our best efforts to provide accurate up-to-date information in this book.

For further information about the books from DISHA,


Log on to www.dishapublication.com or email to info@dishapublication.com

Downloaded From : www.EasyEngineering.net


Downloaded From : www.EasyEngineering.net

1. Grammar— Basic Concepts & Common Mistakes 1-26

2. Active & Passive Voice 27-36

3. Direct & Indirect Speech 37-44

4. ww
Clause Analysis 45-54

5.
w.E
Synthesis of Sentences 55-64

6.

7. Idioms and Phrases


asy
Transformation of Sentences 65-74

75-88

8. Spotting Errors En 89-108

9. Sentence Correction
gin 109-130

10. Choosing the Correct Sentence


eer 131-148

11. Vocabulary — Word Bank


ing 149-174

12 Word Power .ne 175-208

13. Relationship Based Problems


t
209-218

14. Fill in the Blanks (FIB) 219-244

15. Cloze Test 245-260

16. Parajumbles 261-306

17. Paragraph Completion 307-334

18. Reading Comprehension 335-451

Downloaded From : www.EasyEngineering.net


Downloaded From : www.EasyEngineering.net

ww
w.E
asy
En
gin
eer
ing
.ne
t

Downloaded From : www.EasyEngineering.net


Downloaded From : www.EasyEngineering.net

1
PTER
CHA

Grammar— Basic Concepts


& Common Mistakes

INTRODUCTION
Grammar is a Latin word which refers to the study of the form and arrangement of word, sentences and phrases. This is why, first of

ww
all, we’ll describe what a word, sentence or phrase really means:
(1) WORD – Word is the basic unit of a language, and blocks of words join together to form a sentence. A word can be divided into

w.E
its ‘stem’ (the basic part which contains the meaning of the word) and its ‘inflection’ (the ending of a word which could identify
whether it is singular or plural, or which tense is it in)
For example: In the words ‘Cats’ and ‘Talked’ the stem is ‘cat’ and ‘talk’; whereas infinitives are ‘s’ (showing that there are more
than one cats) and ‘ed’ (indicating the past tense).

(A) asy
(2) SENTENCE – A sentence is a group of words which communicates a complete thought. The various parts of a sentence are:
Subject: If we were to say in layman terms, the subject of a sentence is a name, person or thing about which the sentence

En
is speaking. To know the subject of a sentence, ask who or what before the verb used in the sentence. For example, in
the sentence: ‘Sheela is singing a song’, if we have to determine the subject, then we must ask, ‘Who is singing the song?’

(B)

The answer is Sheela and that is our subject.

gin
Predicate: In simple words, we can say that a predicate is what remains in the sentence after removing the subject, so
Sentence – Subject = Predicate

eer
Predicate is basically everything that’s been written about the subject. For example: our predicate in “Sheela is singing a song”
will be: is singing a song.

(C)
As you can see, every sentence contains a subject and a predicate.

ing
Object: It is a person or a thing on which the action of the verb takes place in the sentence. Thus, it is something upon which
the subject acts.

.ne
We can find out the object in a sentence by asking who or what before the subject of the sentence. For example: in ‘Sheela is
singing a song’, if we ask, ‘What is Sheela singing?’, we get the answer as ‘a song’. Thus, the object of the sentence is ‘a song’.
(3) PHRASES AND CLAUSES
PHRASES
A phrase is a small group of words that forms a meaningful unit within a clause. There are several different types, as follows:
(A) Noun Phrase- A noun phrase is built around a single noun, for example:
t
l A vase of roses stood on the table.

l She was reading a book about the emancipation of women.


(B) Verb Phrase- A verb phrase is the verbal part of a clause, for example:
l She had been living in London.
l I will be going to college next year.
(C) Adjective Phrase
l An adjective phrase is built around an adjective, for example:
l He’s led a very interesting life.
l A lot of the kids are really keen on football.
(D) Adverbial Phrase- An adverbial phrase is built round an adverb by adding words before and/or after it, for example:
l The economy recovered very slowly.
l They wanted to leave the country as fast as possible.
Prepositional Phrase- In a prepositional phrase the preposition always comes at the beginning, for example:
l I longed to live near the sea.
l The dog was hiding under the kitchen table.

Downloaded From : www.EasyEngineering.net


Downloaded From : www.EasyEngineering.net

2  l  Grammar— Basic Concepts & Common Mistakes

Of course, we also use the word phrase to refer to a short group of words that have a particular meaning when they are used
together, such as rain cats and dogs, play for time, or a square meal. This type of phrase is often referred to as an idiom.
CLAUSES

A clause is a group of words that contains a verb (and usually other components too). A clause may form part of a sentence or it
may be a complete sentence in itself. For example:

l  He was eating a bacon sandwich.

  [clause]

l  She had a long career   but she is remembered mainly for one early work.

[clause]  [clause]
(A) Main clause- Every sentence contains at least one main clause. A main clause may form part of a compound sentence or
acomplex sentence, but it also makes sense on its own, as in this example:
l He was eating a bacon sandwich.
[main clause]
Compound sentences are made up of two or more main clauses linked by a conjunction such as and, but, orso, as in the
following examples:
l  I love sport    and      I’m captain of the local football team.

ww
l
[main clause]   [conjunction]  [main clause]
  She was born in Spain  but her mother is Polish.

w.E [main clause]      [conjunction]  [main clause]


(B) Subordinate clause- A subordinate clause depends on a main clause for its meaning. Together with a main clause, a
subordinate clause forms part of a complex sentence. Here are two examples of sentences containing subordinate clauses:
l  After we had had lunch,   we went back to work.

l
[subordinate clause]   
I first saw her in Paris,   asy [main clause]
where I lived in the early nineties.
[main clause]      

En
[subordinate clause]
There are two main types of subordinate clause: conditional clauses and relative clauses.
(i)  Conditional clause

If it looks like rain    gin


A conditional clause is one that usually begins with if orunless and describes something that is possible or probable:
a simple shelter can be made out of a plastic sheet
[conditional clause]   [main clause]
I'll be home tomorrow  unless the plane's delayed for hours.
[main clause] [conditional clause]
eer
(ii)  Relative clause
ing
A relative clause is one connected to a main clause by a word such as which, that, whom, whose, when, where, orwho:
I first saw her in Paris,  where I lived in the early nineties.
[main clause]     [relative clause]
She wants to be with Thomas,  who is best suited to take care of her. .ne
[main clause] [relative clause]
I was wearing the dress   that I bought to wear to Jo's party.
[main clause] [relative clause]
Using Relative Clauses- Have you ever wondered about when to use that and when to use which or who in this type of
t
sentence? In fact, for much of the time that is interchangeable with either of these words. For example:
l You’re the only person who has ever listened to me.
l You’re the only person that has ever listened to me.
l It’s a film that should be seen by everyone.
l It’s a film which should be seen by everyone
When referring to something, rather than someone, thattends to be the usual choice in everyday writing and conversation
in British English. However, there is one main case when you should not use that to introduce a relative clause. This is
related to the fact that there are two types of relative clause: a restrictive relative clause and a non-restrictive relative clause.
(C) Restrictive relative clause A restrictive relative clause (also known as a defining relative clause) gives essential information
about a noun that comes before it: without this clause the sentence wouldn’t make much sense. A restrictive relative clause
can be introduced by that, which, whose, who, or whom. You should not place a comma in front of a restrictive relative
clause:
l She held out the hand   which was hurt.
l She held out the hand  that was hurt.
[main clause]   [restrictive relative clause]

Downloaded From : www.EasyEngineering.net


Downloaded From : www.EasyEngineering.net

Grammar— Basic Concepts & Common Mistakes  l 3

You can also leave out that or which in some restrictive relative clauses:
• It reminded him of the house that he used to rent in Oxford.
• It reminded him of the house which he used to rent in Oxford.
• It reminded him of the house he used to rent in Oxford.
[main clause] [restrictive relative clause]
(D) Non-restrictive relative clause- A non-restrictive relative clause (also called a non-defining relative clause) provides extra
information that could be left out without affecting the meaning or structure of the sentence. Non-restrictive relative
clauses are normally introduced by which, whose, who, or whom, but never by that. You should place a comma in front of
them:
She held out her hand,   which Rob shook.
[main clause]    [non-restrictive relative clause]
If a non-restrictive relative clause is in the middle of a sentence, you should put commas before and after it:
Bill,   who had fallen asleep on the sofa,   suddenly roused himself.
[non-restrictive relative clause]
(4) PHRASAL VERBS- A phrasal verb is a verb formed from two (or sometimes three) parts: a verb and an adverb or preposition.
These adverbs and prepositions are often called particles when they are used in a phrasal verb. Most phrasal verbs are formed

ww
from a small number of verbs (for example, get, go, come, put and set) and a small number of particles (for example, away, out,
off, up and in). Phrasal verbs sometimes have meanings that you can easily guess (for example, sit down or look for). However,
in most cases their meanings are quite different from the meanings of the verb they are formed from. For example, hold up can

w.E
mean 'to cause a delay' or 'to try to rob someone'. The original meaning of hold (for example, to hold something in your hands) no
longer applies.
There are five main types of phrasal verb. These are:

asy
(A) Intransitive phrasal verbs (= phrasal verbs which do not need an object).
For example: You're driving too fast. You ought to slow down.
(B) Transitive phrasal verbs (= phrasal verbs which must have an object) where the object can come in one of two positions:
(1)  Between the verb and the particle(s).
For example: I think I'll put my jacket on. En
or
(2) After the particle. gin
For example: I think I'll put on my jacket

eer
(C) Some transitive phrasal verbs are reparable. The object is placed between the verb and the preposition.
For example : She looked the phone number up.
(D) Transitive phrasal verbs with fixed object after the verb:
ing
For example : Rita ran into Sheela the other day at her local residence, she had not seen her since leaving school.
(E) Transitive with two objects, reparable.
For example : Rita's parents were really pleased and put her result down to plenty of revising.
.ne
PARTS OF SPEECH
It is now time to understand grammar in more detail by looking at the different parts of speech:
(1) NOUN:  It is a word which is used to name a place, person, an abstract idea or a thing. For example: in the sentence,
‘Abhishek is a great person.
t
There are two nouns being used: ‘Abhishek’ that names a person and ‘person’ which identifies a class of living beings.
There are different types of nouns, but a particular noun can belong to more than one type:
(a) Proper nouns: A proper noun always starts with a capital letter, and it always refers to a specific person, place or thing. For
example: days of the week, historical places, name of a university or a person etc. So proper nouns are always specific and
indicate or refer to a particular thing or person. (The italicized words below are proper nouns)
I do not work on Sundays.
Travelling to India was indeed a great experience. My favourite place has to be the Taj Mahal in Agra.
(b) Common nouns: In definition, it is completely opposite to a proper noun: it refers to a place, thing or person in general and
not in a specific sense. As these nouns are not specific, therefore the initial letter will not be capitalized (unless of course, it is
the first word of a sentence). Some examples:
The nearest town from here is 32 miles away.
I find it tough to understand why some people hate smokers.
(c) Concrete nouns: These nouns identify things which can be perceived or felt through our senses: touch, sight, taste, hearing
or smell. Thus, these nouns refer to objects that can be sensed. For example:
Whenever I take my dog for a walk, he always tries to run away.

Downloaded From : www.EasyEngineering.net


Downloaded From : www.EasyEngineering.net

4  l  Grammar— Basic Concepts & Common Mistakes

(d) Abstract nouns: These nouns can be defined as being contrary to concrete nouns because these do not refer to what can be
experienced through our physical senses. These refer to abstract thoughts like truth, justice etc. For example:
• Buying sweets for the party was an afterthought.
• Thinking about my childhood always makes me nostalgic.
(e) Countable nouns: These are the nouns which can be counted. So these refer to things which can be counted by us that can
be either singular or plural. For example,
• The table in my room looks terrific.
• After leaving the job, he spent most of his weekdays reading books.
(f) Uncountable nouns: As the name suggests, these are opposites to countable nouns. These nouns cannot be counted. They
can be measured in some units but not counted. For example:
• Oxygen is essential for human life.
• I need some water.
(g) Collective nouns: These nouns refer to a group of entities like things, animals or people. For example:
• He was the smartest in his class.
• The f lock of sheep destroyed the garden.
(2) PRONOUNS:  These are those parts of speech which can be used in place of the nouns. Use of pronouns makes the sentences
less repetitive or cumbersome. For example: Dheeraj went to the market to buy the chocolates which he liked the most.

ww
If we used ‘Dheeraj’ instead of the pronoun ‘he’ then our statement would have been a bit awkward and redundant. Take a look
at this example:
• Soumitra wants his assets to be divided between his sons and their wives.

w.E
Here, if we didn’t use any of the pronouns, then the sentence would have looked something like this:
• Soumitra wants Soumitra’s assets to be divided between Soumitra’s sons and Soumitra’s sons’ wives.
Pronouns are a great way of expressing ourselves without complicating the sentence with excessive repetition of the nouns.
There are several types of pronouns:
(a) Personal pronouns
asy
These are used to refer to a specific thing or person. These are of two types:
(i) Subjective personal pronoun

En
Those personal pronouns which act as the subject of a sentence are known as subjective personal pronouns (I, you, he,
she, it, they, we). For example:
I didn’t expect this from you.
My birthday? Oh, it was indeed special! gin
(ii) Objective personal pronoun

eer
These pronouns act as the object of an infinitive phrase (a phrase which begins with ‘to’), a verb, a compound verb (more
than one verbs) or a preposition. Me, you, her, him, it, us, you, them, are examples of objective personal pronouns. For


example:
l  After reading the book, Ram threw it away.
Here, it is acting as the direct object for the verb threw. ing

verb: ‘will meet’.) .ne
l  I have been told that Mickey will meet us in the cafeteria. (Here, ‘us’ is acting as the direct object of the compound


l Come on! Give that pen to me! (Here, ‘me’ is acting as the direct object for the preposition ‘to’)
(iii) Possessive personal pronouns
Pronouns which indicate possession, or identify ownership, or tell who owns something or someone, are known as t
possessive personal pronouns. Mine, your, hers, its, ours, theirs, his, etc. are possessive personal pronouns. For example:
l  Is that your purse?
Here, a question is asked to know if that purse belongs to someone.
l  Ours is the one parked outside the garage.
Here, ‘ours’ is acting as the subject of the sentence and is indicating that the speaker owns the car.
l  That Ferrari is mine.
Here, the pronoun ‘mine’ is indicating that a particular Ferrari is owned by the one who spoke this statement.
Note: In the above example, the possessive personal pronoun mine is also acting as a subject complement. Subject
complements are those words which follow the linking verb. A subject complement could be a noun, pronoun or an
adjective.
(b) Demonstrative pronouns
These pronouns basically point to thing(s) or identify them. For example, ‘this’ points to something which is nearby, either in
time or space. This, that, those, these are some of the common demonstrative pronouns (this and that refer to singular nouns
or noun phrases; whereas, these and those are used to refer to plural nouns or noun phrases.) For example:
l  This is something which I have never seen before!
Here, ‘this’ is the subject and it is referring to a noun which is nearby to the speaker.
l  I really want to visit that place.
The demonstrative pronoun, ‘that’, points to some place which is not nearby to the speaker.

Downloaded From : www.EasyEngineering.net


Downloaded From : www.EasyEngineering.net

Grammar— Basic Concepts & Common Mistakes  l 5

(c) Interrogative pronouns


The pronouns which are used to ask questions are called as interrogative pronouns. Who, whom, which, what are some of
the interrogative pronouns. The compounds formed by adding the suffix ‘ever’ to these are also interrogative pronouns, viz.
whoever, whomever, whichever, whatever.
Usage of interrogative pronouns: who, whom and sometimes which is used to refer to people. Whereas, what and which are
normally used in cases of things and animals. (refer to the common errors section). For example:
l  Who else is coming for tonight’s party?
Here, the interrogative pronoun ‘who’ is the subject of the sentence
l  To whom does this belong?
Here, ‘whom’ is acting as the object.
l  What is there?
Here, ‘what’ is the object of the verb: ‘is’.
(d) Relative pronouns
Relative pronouns are those pronouns which are used to link or join a phrase or clause, to another phrase or clause. Who,
whom, that, which are used as relative pronouns. Adding the suffix ‘ever’ leads to other relative pronouns, i.e. whoever,
whomever, whichever, etc.
Again, who and whoever are used to refer to subjects; whereas, whom and whomever are used in cases of objects.

ww
l  Whoever broke this vase will surely be in trouble soon.
‘Whoever’ acts as the subject here.
l  You may select whatever you like.


w.E
Here, ‘whatever’ is the object of the compound verb ‘may select’.
l  The player who plays the best, wins the ‘man of the match’ award.
‘Who’ is acting as the subject of the verb and as the relative pronoun that also introduces the subordinate clause, ‘who plays
the best’.

asy
Note: A subordinate clause is also known as a dependent clause. They start with a subordinate conjunction or a relative
pronoun, and contain a subject and a verb but still cannot act as standalone sentences: an additional thought or information
is required to complete or finish the thought.
(e) Indefinite pronouns
En
These pronouns do not refer to something or someone specific, but still some entities that are identifiable. Following are

gin
some of the indefinite pronouns: all, another, any, anybody, anyone, anything, each, everyone, everyone, everything, few,
many, nobody, none, one, several, somebody, someone, etc. For example:
l  Many came to the party, but still it wasn’t great to be a part of it.

l  He donated everything he found in his old basement.
eer
Many acts as the subject, and we can see that it is not referring to someone specific, not even in number.

As you can see, everything isn’t referring to something specific and is neither pointing to something in particular.
(f) Reflexive pronouns
ing
The subject of a clause or sentence is referred to, by using reflexive pronouns. Following are some reflexive pronouns: myself,
yourself, himself, herself, itself, ourselves, yourselves, themselves. For example:

.ne
l  Even though alcoholics know that drinking is harmful, but still they are unable to stop themselves from consuming
alcohol.
l  I was angry at myself for insulting an old man.
(g) Intensive pronouns
t
These pronouns are used to intensify their antecedents. The reflexive pronouns mentioned above can also be used as intensive
pronouns. For example:
l  The chief minister himself said that sincere efforts will be made to develop the state.
The intensive pronoun himself is intensifying its antecedent, i.e. the chief minister.
l  They themselves promised to fulfil the prophesy.
The antecedent here is ‘they’.
(3) ADJECTIVES
The words which modify nouns or pronouns are known as adjectives. They identify, quantify or describe nouns or pronouns.
Normally, these adjectives precede the noun or pronoun they modify. For example:
l  His mansion is wonderful and massive.


Wow! That blue dress is beautiful!
There are many different types of adjectives:
(a) Possessive adjectives
These are similar to possessive pronouns (my, your, hers, its, ours, theirs, his). The only difference is that their use modifies a
noun or pronoun. For example:
l  I could not attend my class.
Here, the word ‘my’ describes the noun ‘class’.
Where is your pen?
‘Your’ describes or tells something about the noun ‘pen’.

Downloaded From : www.EasyEngineering.net


Downloaded From : www.EasyEngineering.net

6  l  Grammar— Basic Concepts & Common Mistakes

(b) Demonstrative adjectives


Again, these are similar to demonstrative pronouns (this, that, these, those) but they work as adjectives by modifying the
noun or noun phrase. For example:
l  While running, I tripped over that cord.

In the given sentence, the demonstrative adjective ‘that’ modifies the noun ‘cord’.
l  This painting is indeed spectacular!

‘This’ modified the noun ‘painting’.


(c) Interrogative adjective
These are identical to interrogative pronouns (which, what) but here, they work as adjectives by modifying the noun or noun
phrase instead of just standing on their own. For example -
l  Which paintings do you want? (Here which modified the noun paintings by defining or informing about the paintings
referred to in the sentence)
l  What luggage are you carrying? (what modifies the luggage)
(d) Indefinite adjective
Again, these are similar to indefinite pronoun except that they modify the noun, pronoun or noun phrase like in the following sentences:
l Many Indians go abroad for higher studies. (Here many modifies the noun Indians)
I will not listen to any advice you have for me (any modifies advice)

ww
l
(4) VERB
Verb is arguably considered to be the most important part of a sentence. It asserts or tells something about the subject of the
sentence, and also depicts the state of being, events or actions. Verb is a critical element of the predicate of a sentence.

w.E
Given below are some of the examples of verbs:
l He is drinking orange juice.

Informs the action the subject of the sentence is doing.
l I will go there soon.

asy
It is a compound verb which describes an action taking place in the future.
Verbs can be divided into three types –


l The car hit him. (physical action)
En
(i) Action Verb – these verbs represent an action, which could either be physical or mental. Example:

l He was singing all day long. (compound verb showing physical action)


l I am imagining something. (mental action)
gin
(ii) Verbs of being (forms of be) – These verbs show a state of existence:

l His paintings are a work of art.
l He was in his room some time back.

l I have been there.
eer
(iii) Linking verbs – these verbs link the subject with its complement.
(5) ADVERB ing
verbs, adjectives, clauses, phrases or even another adverb. An adverb can do the following:
1. Indicate manner, time, place, cause, or degree of a noun, pronoun or an action. .ne
We have seen in the previous section, how adjectives modify nouns or pronouns. Similarly, adverbs also work as modifiers to

2. Questions like how, when, where and how much, can also be answered by an adverb.
For example:
l  Our cook quickly made the food, when we told her that we were very hungry.

The adverb ‘quickly’ modifies the verb ‘made’ and also tells how fast the food was made.
t
l  Johnson wasn’t getting picked for the team, but he was determined to patiently wait for his chance.

The adverb ‘patiently’ modifies the verb ‘wait’ and also shows the manner in which Johnson decided to wait.
l  We urged him to undertake rock climbing more carefully, otherwise he could be paralyzed for life.

Here, the adverb ‘carefully’ is in fact modifying another adverb (more).
l  Luckily, I reached on time.

The adverb ‘luckily’, modifies the whole sentence.
The verbs which join two clauses together are known as conjunctive adverbs (consequently, finally, furthermore, hence, however,
incidentally, indeed, instead, likewise, meanwhile, nevertheless, next, nonetheless, otherwise, still, then, therefore, thus). But a
conjunctive adverb requires a semicolon to join two independent clauses. For example:
l He could not prepare his car; therefore, he took a taxi.

l It took us two days; finally, we got the tickets for show.

(6) CONJUNCTIONS
These words are used to link words, phrases and clauses; therefore, it is essential to know their use in order to form logical and
coherent sentences. For example:

l I bought apples and oranges.

l Do it when you are free.

Downloaded From : www.EasyEngineering.net


Downloaded From : www.EasyEngineering.net

Grammar— Basic Concepts & Common Mistakes  l 7

These are just some basic examples, but now will look at conjunctions in more depth.
There are following types of conjunctions –
(a) Co-ordinating Conjunctions
These words (and, but, or, nor, for, so, or yet) are used to join individual words, phrases and independent clauses. For example
(all of the italicized words are co-ordinating conjunctions):
(a) Eggs and fishes are rich in proteins. (Two nouns are linked with ‘and’)
(b) He was sad, for he had suffered a lot. (‘for’ is linking two independent clauses)
(b) Subordinating conjunctions
A dependent clause is introduced with a subordinating clause, and it also indicated the kind of relationship that exists
between the dependent and independent clause. The most common subordinating conjunctions are:
after, although, as, because, before, how, if, once, since, than, that, though, till, until, when, where, whether, and while.
We will now give some examples of its use; the italicized words indicate the subordinating conjunction.
i) After he talked with his friends, he felt better. (‘After he talked with his friends’ is a dependent clause)
ii) If I call you, please pick up the phone. (‘If I call you’ is the dependent clause)
iii) He realized that he needs to work harder when he saw his exam results. (‘when he saw his exam results’ is the dependent clause)
(c) Correlative conjunctions

ww
These always come in pairs and are used to link equivalent sentence elements both...and, either...or, neither...nor, not only...
but also , so...as, and whether...or'. The examples are given below, as the italicized words show correlative conjunctions:
(i) Both my father and mother are doctors. (‘both’ ‘and’ conjunction pair is used to link two noun phrases which also act as

w.E
the compound noun of the sentence)
(ii) I will either go to Greece or Canada in my holidays. (‘either’ ‘or’ conjunction pair is linking two nouns here)
(7) PREPOSITIONS

word which it introduces.


asy
Nouns, pronouns and phrases are linked to the other words of the sentence via prepositions. The object of the prepositions is the

Spatial or temporal relationship the object has to the rest of the sentences is indicated by prepositions; the following examples
depict that:
(i) The laptop is on the table
En
(ii) The laptop is beneath the table.
(iii) He was leaning against the lamp.
(iv) His house is beside the regional park. gin
(v) I saw him over the bridge.
(vi) He talks a lot during the class. eer
ing
A prepositional phrase consists of a preposition, the object of preposition and any associated adjective or adverb. There are
around 150 prepositions, but the most common ones are about, above, across, after, against, along, among, around, at, before,
behind, below, beneath, beside, between, beyond, but ,by, despite, down, during, except, for, from, in, inside, into, like, near, of,

.ne
off, on, onto, out, outside, over, past, since, through, throughout, till, to, toward, under, underneath, until, up, upon, with, within,
and without.
Examples showing use of prepositions:
i) Excessive pollution is a cause of concern throughout the world. (‘throughout’ introduces the noun phrase ‘the world’; this
prepositional phrase acts as the adverb because it describes the location of problem)
ii) He always did his work with enthusiasm and dedication. (here the preposition ‘with’ introduces the nouns enthusiasm and
t
dedication which act as the compound noun in the given sentence; the prepositional phrase here is an adverb because it describes
how he did the work)
iii) The teacher was searching for his book in the cabin. (in introduces the noun ‘in’ the cabin and the prepositional phrase acts
as an adverb because it describes where the teacher was searching)
Some of the prepositions are:-
(i) In is used with the names or countries and large towns; at is used when speaking of small towns and villages.
For example :
l  I live in Delhi.
l  I live at Rohini in Delhi.
(ii) In and at are used in speaking of things at rest; to and into are used in speaking of things in motion.
For example :
l  He is in bed.
l  He is at the top of the class.
l  He ran to school
l  He jumped into the river.
l  The snake crawled into its hole.

Downloaded From : www.EasyEngineering.net


Downloaded From : www.EasyEngineering.net

8  l  Grammar— Basic Concepts & Common Mistakes

(iii) On is often used in speaking of things at rest; and upon for the things in motion. For example:
(a) He sat on a chair.
(b) The cat sprang upon the table.
(iv) Till is used of time and to is used for place.
For example :
l He slept till eight o’clock.
l He walked to the end of the street.
(v) With often denotes the instrument and by the agent.
For example :
l He killed two birds with one shot.
l He was stabbed by a lunatic with a dagger.
(vi) Since is used before a noun or phrase denoting some point of time and is preceded by a verb in the perfect tense.
For example :
l I have eaten nothing since yesterday.
l He has been ill since Monday last.
(vii) From is also used before a noun or phrase denoting some point of time but is used with non-perfect tense.

ww
For example :
l I commenced work from 1st January.
He will join school from tomorrow.

w.E
l

(viii) For is used with a period of time.


For example :
He has been ill for five days.

asy
l

l He lived in Bombay for five years.


(ix) Use of in before a period of time means at the end of period, but use of within before a period of time means before the
end of period.
For example :
En
I shall return in an hour. (means I shall return at the end of an hour).

gin
l

l I shall return within an hour. (means I shall return before the end of an hour)..
(x) Scarcely should be followed by when and not by but.

(xi) The phrase ‘seldom or ever’ is wrong ‘Seldom or never’ is right.


eer
For example : Scarcely had he gone, when (not than) a policeman knocked at the door.

For example : Such goods are made for export, and are seldom or never used in this country.
(xii) Examine the following sentence
l This is as good, if not better than that. (Wrong) ing
l

l
This is as good as, if not better than, that. (Right)
This is as good as that, if not better. (Right)
.ne
(xiii)

(xiv)
Beside means at the side of while besides means in addition to. For example :
l

l
Beside the ungathered rice he lay.
Besides being fined, he was sentenced to a term of imprisonment.
Above and Below merely denote position While over and under also carry a sense of covering or movement.
t
l The bird flew above the lake. (Wrong)
l The bird flew over the lake. (Correct)
Here over is used to denote upward position and movement also.
(xv) During is used when reference is made to the time within which something happens. For is used when we are talking
about how long something lasts.
l There are few incidents of irregularity for the emergency years. (Wrong)
l There are few incidents of irregularity during the emergency years. (Correct)
(xvi) Compare is followed by to when it shows that two things are alike. It is followed by with when we look at the ways in which
two things are like and unlike each other. For example :
l Sanath Jayasuria’s balling may be compared to the sales of a useful book, they score right from the beginning. (Wrong)
l Sanath Jayasuria’s batting may be compared with the sales of a useful book; they score right from the beginning. (Right)
l If we compare Delhi University with the regional ones, we find the former to be much more efficient. (Wrong)
l If we compare Delhi University to the regional ones, we find the former to be much more efficient. (Right)
(8) INTERJECTIONS
Some of the more interesting parts of speech are the interjections. These are used to express emotions or feelings and are mostly
the easiest to identify. The examples include, greetings like Hi, Hello, Goodbye, expressions like Great, Oh, Wow!, Bravo!, etc.

Downloaded From : www.EasyEngineering.net


Downloaded From : www.EasyEngineering.net

Grammar— Basic Concepts & Common Mistakes  l 9

Interjections are normally used as part of active speech where the writer has to express immediate feelings.
l Hi, my name is Gaurav.
l Oh dear! That would hurt.
l Excuse me! Please make way for the pregnant lady.
(9) ARTICLES
An article is word used to modify a noun, which is a person, place, object or idea. Technically, an article is an adjective, which is
any word that modifies a noun.
1. A or an does not refer to a particular person or thing. It leaves indefinite the person or thing spoken of.
For example : I saw a doctor. (means I saw any doctor)
2. An is used before a word beginning with vowel sound (please note a word beginning with vowel sound and not necessarily
a vowel itself).
For example : an ass, an enemy, an inkstand, an orange, an umbrella, an hour.
3. An is placed before an abbreviation if the first letter of an abbreviation is F, H. L, M, N, R, S or X.
For example :
l An MBA was required for the post.

ww l An SAO is an officer of high rank


4. A is used before a word beginning with a consonant sound.
For example : a boy, a woman a horse, a one-rupee note, a university, a European (both university and European begin with

w.E
a consonant sound of ‘yu’ )
5. A and an are used with words ‘few’ and ‘little’ if they refer to a small number or a small amount. Words ‘few’ and ‘little’
without the articles means almost none.
For example:
l asy
We have little time to spare. (means almost no time)
l

l
En
We have a little time to spare. (means some time)
Few persons were present at the meeting. (means almost no one was present)
l

6. A is used in the following senses :


(A) In its original numerical sense of one. gin
A few persons were present at the meeting. (means some were present)

For example:
  l Not a word was said. eer

  l A word to the wise is sufficient.
(B) In the vague sense of a certain time. ing

For example : One evening a beggar came to my door.
(C) In the sense of any, to single out an individual as the representative of a class.
For example : A pupil should obey his teacher. .ne
(D) To make a common noun of a proper noun.
For example : A Daniel came to judgement. (A Daniel = A very wise man)
7. The points out a particular person or thing or someone or something already referred to.
t
For example :
l I saw the doctor. (means I saw some particular doctor)
l The book you want is out of print.
8. The is used with names of gulfs, rivers, seas, oceans, groups of islands and mountain ranges.
For example :
The Persian Gulf, The Red Sea, The Indian Ocean, The British Isles, The Alps.
9. The is used before the name of certain books.
For example : The Vedas, The Puranas, The Ramayana.
But we never say ‘The Valmiki’s Ramayana’. The is not used when the name of a book is mentioned along with the author’s
name. So, ‘Valmiki’s Ramayana’ is correct.
10. The is used before the names of things unique of their kind.
For example : the sun, the sky, the ocean, the sea.
11. The is used before a plural common noun if it refers to a particular group among the class and not the whole class.
For example : Drive away the cows from the field.
12. The is used before a proper noun only when it is qualified by an adjective.
For example : The great Rani of Jhansi, the immortal Kalidas.

Downloaded From : www.EasyEngineering.net


Downloaded From : www.EasyEngineering.net

10  l  Grammar— Basic Concepts & Common Mistakes

13. The is used before the superlatives.


For example :
l Sachin is the best batsman in the world today.
l The best person should win.
14. The noun if emphasis is laid on the use of such a noun. Here, noun can be proper or abstract noun
l the time for doing it.
l occasion to help the distressed.
15. The is used with ordinals.
For example :
l He was the first student to finish his homework.
l The second chapter of the book is very interesting.
16. The is used before an adjective when the noun is understood.
For example :
l The poor are always with us. (Here poor mean poor people, which is understood.)
l The weak and the strong. (Here weak means weak people and strong means strong people.)
17. No article is used before a common noun when it refers to all the members of the class.

ww
For example :
l

l
Man is mortal.
Fish has high protein content.
l

w.E What kind of flower is it?


18. The is used before a common noun to give it the meaning of an abstract noun.
For example : The devil in him begins its misdeeds now and then.
19. No article is used before the names of materials such as gold, stone, wine, iron, wheat, wood, cloth.
For example :
l Gold is a precious metal. asy
l

l Iron is a useful metal.


Note: But it is correct to say En
Wheat grows in Uttar Pardesh, Haryana and Madhya Pardesh.

For example : An iron is a useful gadget.


gin
Because here we are not taking about material iron, but the object which is used to make clothes smooth.
20. No article is used before proper nouns.
For example :
Delhi is the capital of India. eer
ing
l

l Newton was a great philosopher.


But consider the following examples where an article is used before a proper noun.
l

l
This man is a second Newton.
Bombay is the Manchester of India. .ne
Here Newton and Manchester is not used as a proper noun but a common noun. The first sentence means that this man is
as great as Newton and the second sentence means that Bombay is a great manufacturing City like Manchester.
21. No articles are used before a common noun used in its widest sense.
For example :
t
l The science has developed much in the past hundred years. (Incorrect)
l Science has developed much in the past hundred years. (Correct).
22. No article is used before the noun following ‘Kind of ’:
For example :
l What kind of a hobby is this? (Incorrect)
l What kind of hobby is this? (Correct)
23. No article is used before abstract nouns.
For example :
l Wisdom is the gift of heaven.
l Honesty is the best policy.
But consider the following examples where an article is used before an abstract noun.
l The wisdom of Solomon is famous.
l I cannot forget the kindness with which he treated me.
Here the article is used before the abstract noun as the abstract noun has been qualified by an adjective or adjectival clause.

Downloaded From : www.EasyEngineering.net


Downloaded From : www.EasyEngineering.net

Grammar— Basic Concepts & Common Mistakes  l 11

24. No article is used before languages, subject of arts and science.


For example :

l We are studying English.
l Geometry is the toughest subject I have ever studied.
25. No article is used before words such as school, college, church, bed, table, hospital, market, prison.
For example :
l I went to school till last year.
l I have never been to hospital.
But an article is used before these words when reference is made to a definite place.
26. No article is used before the name of relations like father, mother, aunt, uncle.
For example : Mother would like to see you.
But If someone else’s mother is being talked about then the should be used.
For example : The mother would like to see you.
27. Article should not be used before positions that are held at one time by one person only.
For example :
l S D Sharma was elected the president of the country. (Incorrect)

ww l S D Sharma was elected president of the country. (Correct)


28. Please consider this sentence
(a) I have a black and white cat.

w.E
Here I mean that I have one cat that is partly black and partly white.
Now, consider this sentence
For example : I have a black and a white cat.

asy
Here I mean that I have two cats one is black and the other white. Hence the rule is that when two or more adjectives
qualify the same noun, the article is used before the first adjective only. But when they qualify different nouns, the article
is used before each adjective separately.


Consider one more example.
l The President and Chairman is absent. En
l The President and the Chairman are present.
gin
Sentence a means that only one person is acting as president as well as chairman. Sentence b means that two different persons
are acting as the President and the Chairman and both the persons are absent.
(10) DETERMINERS
eer
These words introduce a noun with words like a/an, the, this, every, those, many, etc. These are placed before the noun so that it
is clear what the noun refers to. For example:
ing
‘People’ just means a group of human beings, but ‘these people’ indicates the people referred to in the phrase/clause and hence,
‘these’ is used as a determiner here.
There are several classes of determiners:
(I) Definite articles .ne
This is a definite article which comes under determiners, and is amongst the most frequently used words in English. We will
list some of the cases or situations in which it is used:
(a) It is used to refer to something which is unique, or only one of such a type exists.
For example:
t
l The Pope gave a great speech yesterday.
There is only one Pope, and this is signified or indicated by the definite article ‘the’ used before it.
l The Sun is hiding in the clouds today!
As we know, there is only one Sun, so it is appropriate to introduce, or mention it by having a definite article before it to
convey this oneness.
l The king of India was well known for his generosity.
It indicates that there was only one king at that particular time.
(b) A definite article is also used before superlative adjectives, again indicating that only one of such kind exists.
For example:
l He is the smartest student in our class
There can only be one who would be deemed the smartest.
l He is the tallest person in our family
Again, there can be only one who can be considered the tallest.
(c) To refer to something we have already mentioned.
This is similar to the only one thing, because we are referring to something or someone unique, i.e. just the one we have
already mentioned. Thereby, we are being very particular and we intend to refer to only that entity.

Downloaded From : www.EasyEngineering.net


Downloaded From : www.EasyEngineering.net

12  l  Grammar— Basic Concepts & Common Mistakes

For example:
l A boy was running down the streets and he suddenly fell as he tried to avoid a collision.
l The boy was lucky that he did not suffer any serious injuries.
You could argue the necessity of this ‘the’, but with this determiner, it is indicated that we are not referring to just any boy, we
refer to that one boy we just mentioned. Thus, it helps in removing this small ambiguity.
(d) To imply one trait or just something about all the entities referred to by the noun.
For example:
l  The dogs are often reliable.
‘The’ helps convey a trait which all the dogs have.
l  The moms are always very emotional about their kids.
The definite article helps say something about ‘all the moms’.
(e) When we are referring to a system or service. For example:
l The fire brigade took just five minutes to arrive at the site.
l Did you listen to the radio last night?
l The police did a commendable job.
(f) When referring to a specific group of people. For example:



(g)
ww
l
l
l
The rich should do more charity.
It’s heartrending to see the poor suffer so much.
She works to make lives of the orphans better.
Before naming a country (the United Kingdom), some unique place (the Bermuda triangle), organization (the WHO), well

w.E
known buildings or works of art (the Taj Mahal), hotels or pubs or restaurants (the Oberoi hotel), or geographical features (the
Mountain) and even families (the Thakurs).
(II) Indefinite articles

asy
Let us now move on to the indefinite articles: A and An.
Definite articles are used before stating something that is very specific or unique. On the other hand, indefinite articles are used
to refer to non-specific nouns. The use or significance of both these indefinite articles is the same, but ‘a’ is used before a noun

l Are you reading a novel? En


which has a consonant sound. Whereas, ‘an’ is used before a noun which has a vowel sound. For example:

l I saw an elephant while going to school!


gin
The situations or cases in which these two indefinite articles are used:
(1) Before a countable noun when the user does not exactly know what we are referring to. For example:
l  I am looking for a red pen.
Now which pen is specifically talked about, is not known. eer
l  I need a cigarette.
Which cigarette? The reader or hearer does not know that.
(2) Use it to indicate that something is a part of the group, or is a constituent. ing
For example:
l  She is a student in the Bangalore University.
She is one of the students who studies in this university. .ne
(3) Never use an indefinite article with an uncountable or plural noun.
For example:
l
l
I am eating a grapes. (Incorrect)
I am eating grapes. (Correct)
t
l I am drinking a water. (Incorrect)
l I am drinking water. (Correct)
(4) Use it while conveying what someone is, or what they do.
For example:
l She is a teacher.
l My mother is a housewife.
(5) Use indefinite articles with a singular noun to say something that represents all things of that kind.
For example:
l  A dog likes drinking milk.
All the dogs like drinking milk.
(III) Demonstratives
These determiners show where an object or person is in with relation to the speaker. There are four demonstrative determiners:
this, that, these, those.
This (for singular nouns) and these (for plural nouns) are used to refer to something or someone near to the speaker in distance
or time; whereas, that (for singular nouns) and those (for plural nouns) are used to refer to something or someone which is
relatively further away, or behind in time. For example:

Downloaded From : www.EasyEngineering.net


Downloaded From : www.EasyEngineering.net

Grammar— Basic Concepts & Common Mistakes  l 13

l  Whose is this pen?

l  These pictures are really beautiful.

l  Those days are still fresh in my memory.

l  Give me that pen.


The relative positioning indicated could also be psychological like:
l  I know nothing about that. (far away)

l  This is bound to be joyful. (near)


(IV) Possessive determiners
These show who the thing belongs to. The possessive determiners are: my, your, his, her, its, our, their. For example:
l  This is my pen.

l  Their homes were burned in the protest.


(V) Quantifiers
These can be classified under adjectives and adjectival phrases, as they inform or tell something about the noun. Quantifiers
answer simple questions like ‘How much?’, or ‘How many?’ Some of the most common quantifiers are: a few, a little, much, many,
a lot of, most, some, any, enough, etc.
l  I have many pens.

ww
l  Do you have some water?

(VI) Question words


While asking a question, these words indicate which thing or person is being referred to. There are three question words: which,


w.E
what, and whose.
l  What are you talking about?

l  Which movie are you watching?

l  Whose is this house?

(VII) Defining words


asy
These indicate which thing or person is referred to in the sentence. The words that come under the category of defining words


are: which, and whose.
l  I do not remember which car is this.
En
MODALS
l  The chef whose recipes are marvelous has just moved out.

gin
verbs to express see meanings such as possibility, permission, certainly etc.
(1) Can usually expresses ability or capacity eer
The verbs like can, could, may, might, would, shall, should and ought are called modal verbs or modals. They are used with ordinary

I can swim across the river


Can you lift this table?
(2) Can is also use to empress permission
ing
You can go now.
(3) May is a more formal modal used to express permission .ne
You may come in.
May I leave the room now?
(4) May is also used to suggest possibility in an affirmative sentence.
He may be at home
It may rain tomorrow
t
(5) Can is used to suggest possibility in negative/interrogative sentence.
Can this be true ?
It cannot be so.
(6) May when used in a negative sentence suggests an improbability whereas can suggests impossibility.
He may not come today.
She cannot sing.
(7) Could and might are used as past forms of can and ‘may’.
I could swim across the river when I was young.
I thought he might be at home.
(8) Might suggests less possibility or probability than may.
I might go to Bangalore next week suggests the probability of going is less than a sentence with ‘may’ will suggest.
(9) Could is used as a polite form of seeking permission or making a request.
Could you pass me the plate ?
Could I please talk to Mr. Grover?
(10) Shall is used with first person and will in all the persons to denote future action.
I shall need the money tomorrow.
When will you come next?

Downloaded From : www.EasyEngineering.net


Downloaded From : www.EasyEngineering.net

14  l  Grammar— Basic Concepts & Common Mistakes

(11) Shall is used with the second and third person to express command, promise or threat.
You shall never come near my child.
You shall be punished for this.
We shall go for a picnic this Sunday.
(12) Will You ? indicates an invitation or request.
Will you dine with us tonight ?
Will you lend me your car for a week ?
(13) Should and would are used as past forms of shall and will.
I expected that I would get a first class.
She would sit for hours listening to the radio.
(14) Should is used to express duty or obligation.
We should obey the laws.
You should keep your promise.
(15) Should is used to express a supposition
If it should rain, they will not come.
(16) Should can also be used to express probability.
He should be in the library.
(17) Must is used to express necessity.

ww
You must improve your spelling.
(18) Must is also need to express obligation, and is a stronger word than should.
We must follow the law.

w.E
(19) Must is also used to express logical certainty.
Living alone in such a big city must be difficult.
(20) Ought is used to express moral obligation and is stronger than both should and must.
We ought to love our parents.

asy
(21) Ought is also used to express probability sometime when the probability is very strong.
The book ought to be very useful.

VERB TENSES
En
Tenses are very important to learn and understand in order to form grammatically correct sentences. Not many would know terms

gin
like ‘present perfect continuous tense’ but would still be able to speak or write correct English. We have already studied that verbs are
‘action words’, but these verbs can also depict the time of action, event or condition by changing their form. This is done by the use
of verb tenses.

eer
There are many ways of categorising tenses in order to study and analyse them. We will categorise the tenses in the simplest manner:
(Note: the examples given below are based on the simple phrase ‘I go’)
(I) Present tense:
(a) Simple Present – I go
(b) Present Progressive – I am going
ing
(c) Present Perfect – I have gone
(d) Present Perfect Progressive – I have been going .ne
( II) Past Tense
(a) Simple Past – I went
(b) Past Progressive – I was going
(c) Past Perfect – I had gone
t
(d) Past Perfect Progressive – I had been going
( III) Future Tense
(a) Simple Future – I will go
(b) Future Progressive – I will be going
(c) Future Perfect – I will have gone
(d) Future Perfect Progressive – I will have been going
We will now discuss these tenses in detail:
(1) Simple present tense
This tense is used to describe an action or event which is occurring at the time of speaking or writing. Hence, it is used to describe
the action of the present time. We will now give a few examples of sentences in the present tense:
(a) Describe the event taking place in the present.
l  She has a resolve in her eyes.

l  They do not find comfort in peace now.

(b) Simple Present Tense is also used while saying or writing facts, or general truths.
l  There are seven days in a week.

l  A triangle has three sides.

Downloaded From : www.EasyEngineering.net


Downloaded From : www.EasyEngineering.net

Grammar— Basic Concepts & Common Mistakes  l 15

(c) The simple tense is also used while stating something that generally happens, or is a habitual event.
l  He wakes up early every morning.

l  I never go to that restaurant because the service is really poor there.


(d) Simple tense is also used to describe works of arts, like
l  Bible is the most famous book in the history of literature.

l  Paintings of Picasso are a treat for the eyes.


(e) Simple Present tense can even be used while referring to a future event, if it is used with an adverb or adverbial phrase.
l  The exam will start in 15 minutes.

l  That movie will be shown on Tuesday.


(2) Present Progressive tense
Sometimes, present progressive and simple present are used interchangeably, but present progressive is used to emphasize that
the event or condition is continuing, or we can say that it used to describe a continuous present event.
l Sentences revealing the on-going state of actions:
(i) She is cleaning her room right now.
(ii) I think Sky Sports is broadcasting this match in England.
l Like simple present, present progressive when used with an adverb or adverbial phrase can be used to refer to a future event.
(i) People are going to queue up in 15 minutes.

ww
(ii) My course is finishing next week.
(3) Present Perfect Tense
This tense is used to describe an event or action which began in the past and still exists or still continues in the present, or has

w.E
just finished in the present. Present perfect tense can also be used to describe an event or action that took place in the past, but
which still has an effect on the present.
l He has not given us the money yet.

l
speaking either.
asy
The above statement says that the money was not given in the past and the money has still not been given at the time of

Ron has decided to wake up early in the morning every day.

En
The above statement says that Ron decided something in the past, but that decision taken in the past is still influencing or
affecting the present.
l

gin
The government has decided to increase the price of diesel; consequently, the cost of transportation has also increased.
Both the events talked about in the above sentence refer to something which took place in the past, but still has an effect on
the present.
(4) Present Perfect Progressive
eer
It is similar to present perfect tense in the sense that it is used to refer to an event which begun in the past and continues to
exist or be there in the present. The only difference between both these tenses is that, present perfect progressive tense is used to
emphasize the fact that the event still continues in the present.
The following sentences are examples of present perfect progressive tense: ing


l I have been hoping to get results.
l We have been shopping all day.
(5) Simple Past Tense .ne


l Sam called me yesterday.

l He parked his car in a no-parking zone.


t
This tense is used to express an event or action that took place in the past, or sometime before the moment of speaking or writing.
Each of the following sentences refers to an event which happened at some point in the past.

(6) Past Progressive Tense


This tense is used to describe an event or action which was ongoing in the past. Often, these actions take place within a specific
time in the past. We have seen that the present progressive tense is used to describe an event which took place in the past and still
has an effect or influence in the present; but, in case of the past progressive tense, the event of the past has no evident or obvious
effect in the present. Actions described in the past progressive tense began and finished at different times in the past. Hence, the
events of past which lasted for some time are expressed with the past progressive tense.
The following examples show how the past progressive tense is used:
l Yesterday, I saw her walking on the streets.
The above sentence describes an event that took place for a continuous period of time in the past. Barring anything out of
ordinary, the woman’s walk on the streets yesterday has no obvious or evident effect on the present.
l Rohit was telling us stories of the times he spent working for the Indian army.
The above statement says that Rohit was telling stories in the past, and he stopped telling them in the past too, i.e. he is not telling
those stories in the present. But that event, which is story-telling, took place for a continuous period of time in the past.
(7) Past Perfect Tense
The past perfect tense is used to express an event which was completed in the past; sounds similar to the simple past tense? The
difference between these tenses is that the past perfect tense is used to emphasize that the event or action ended before some

Downloaded From : www.EasyEngineering.net


Downloaded From : www.EasyEngineering.net

16  l  Grammar— Basic Concepts & Common Mistakes

another event or action begun. (all in the past). So, basically the two events are mentioned in such a way that one of them ended
before the other one started.
The following examples show how the past perfect tense is used:
l Mithun had finished eating the lunch before Josh came.
The above sentence describes two events: ‘eating’ and ‘came’, but the information which is inferred from this sentence is: the event
of ‘eating’ ended before another event ‘came’ took place.
l The drought had lasted for 2 months.
We discussed a similar statement in the present perfect tense –‘the drought has lasted for 2 months’ this statement indicated that
the drought was declared 2 months ago, and it still exists, i.e. the drought is there even now. So, this event took place in the past
and it is there in the present too. But, in the past perfect tense – ‘The drought had lasted for 2 months’ says that the drought started
and also finished at their respective points in the past, and this has no evident connection to any present event.
l After Hitesh had started earning, he felt more independent.
Both the events or conditions took place at the same point in the past. The speaker could have also said, ‘After Hitesh started
earning, he felt more independent’ (simple past). This statement expressed in the past perfect tense emphasizes that he felt
more independent after he had started earning. So, we can see that the difference of just one word alters the meaning or the
information conveyed.
(8) Past perfect Progressive Tense

ww
This tense is also used to refer to two events of the past. The tense shows that a continuous or progressive event began before
another event had begun, or it interrupted the first event. So, it is essentially used to stress that a continuous event took place
before some other event started.


w.E
Examples of sentences in the past perfect progressive tense are shown below:
l I had been watching the movie before he came.
The above sentence says that the continuous event ‘watching’ was interrupted or stopped by some other event: ‘he came’.

asy
l I had been looking for my pen drive for 2 hours before I found it.
The above sentence says that the event of the past: ‘had been searching’ took place for a period of time in the past, i.e. 2 hours.
This continuous event was followed by another event: ‘found’.
(9) Simple Future Tense
En
This tense is used to express future events, i.e., the events which will take place after the act of writing or expressing.



The following examples show the use of the simple future tense:
l They will come tomorrow.
l Will you go out tonight? gin
(10) Future Progressive Tense
eer
This tense is used to describe continuous events which will take place in the future. Therefore, ongoing future events are expressed
via this tense.


The following examples will reveal how the future progressive tense is used:
l Nehal will be singing songs in my party tomorrow. ing

The above sentence states that a continuous event ‘singing’ will take place in the future.
l They will be performing the classical dance in tomorrow’s show.
.ne
The event which will exist for a continuous period of time is ‘performing classical dance’, and this event will happen sometime in
the future, i.e. ‘tomorrow’.
(11) Future Perfect Tense t
This tense is used to refer to an event or action that will be completed at some point in the future before another future event
starts. So, two events of the future are expressed in such a way that it is conveyed that one will take place after the other.
The following examples will show how this tense is used:
l We will have completed shifting all the furniture before we move into our new house.

The above sentence says that the event of shifting the furniture precedes the event of moving into the new house.
l He will have bought all the gifts before the Christmas Eve.

Two future events are mentioned in the above sentences in such a way that it is evident that the presents will all be bought before
the arrival of the Christmas eve.
(12) Future Perfect Progressive Tense
This is probably the least used tense, but it still has its own different uses. This tense is used to express an event that will take place
for a continuous period of time in the future, and this event will be completed at some specified time in the future.
The following are some of the examples of sentences in future perfect progressive tense:-
l I will have been working for six years by the time I become thirty years old.

The continuous event in the above sentence is ‘being in the job’; the above sentence informs that the six years being in the job will
be completed when the speaker becomes thirty years old.
l When he comes back, I will have been watching the movies for four hours.

In this sentence too, the ongoing action of ‘watching the movies’ will precede the act of coming back.

Downloaded From : www.EasyEngineering.net


Downloaded From : www.EasyEngineering.net

Grammar— Basic Concepts & Common Mistakes  l 17

PRESENT PAST FUTURE


Go/Goes Went Will Go
(i) Normal/Daily activity (i) Past event (i) An event that will take
(ii) Historical present place sometime in future
Simple
(iii) Expressing works of art
(iv) Future events (in conjunction with
adverb/adverbial phrase)
Am/Is/Are going Was/Were going Will be going
(i) An ongoing present event (i) continuous event of the past (i) continuous event of the
Progress
(ii) Referring to a future event (in conjunction future
with adverb/adverbial phrase)
Have gone Had gone Will have gone
(i) An event that started in the past and still (i) An event that stated in the (i) An event that will be
Perfect
continues in the present past and finished in a specified completed in the future be-
time in past fore another event starts
(ii) Used to show that a past event has an (ii) An event that finished in the

ww effect on the present

Have been going


past before some another event
began
Had been going Will have been going
Perfect
w.E
Progressive
(i) An ongoing event that started in the past
(used to stress the ongoing nature of the
event)
(i) A continuous action of the
past before another event of the
past began
(i) continuous action of
future will be completed
before some other time in

PUNCTUATIONS
asy the future

En
Punctuations are necessary for the production of effective and clear writing. Some of the questions on the different tests are based
solely on your knowledge and use of punctuations. We will take up all the different punctuation marks which we can say are most
important to our study:
(1) Comma gin
eer
It is probably the most frequently used punctuation mark. It is used to separate different parts of sentences, and to tell the readers
to pause between words, or group of words or clauses and also helps in clarifying the meaning of a sentence. Use of comma is
optional, i.e. it depends on the writer how much he/she wants to use it. Some writers use it a lot, while others use them sparingly.

all very subjective. ing


Thus, it depends on the individual, and maybe you will find that not using them frequently leads to a more effective writing; it is

Every comma represents a slight pause, hence a small break in the sentence. The pauses are necessary and hence the use of
comma is optional only if the meaning of the sentence will not change if the comma is not used.
We will now try to describe all the different uses of comma: .ne


l Exams will take place on the Sunday, so be prepared.
l I went there to buy some chocolates, but I didn’t find any.
t
(a) Commas are used to separate phrases, clauses or groups of three of more words. You can use these before the coordinating
conjunctions which join the independent clauses.

Let us also take an example of how comma is optional – In the example below, we will not use the comma but you can choose
to use it before the conjunction ‘and’:
l He asked for more money and he got it.
(b) Use comma before an introductory phrase or word.
l Used after an adverbial introductory phrase:
After searching all day long, he finally found his purse.
l Used after an introductory adverb:
Fortunately, I reached there on time.
l Introductory prepositional phrase:
In the bottom drawer, you will find a pen.
(c) Using a comma to separate items in a list, or separate the entities –
l I need to go to the supermarket to buy some vegetables, oil, eggs and flour.

l The countries I would like to visit: Croatia, Italy, Spain and Brazil.

(d) Some writers use comma after nonessential phrases; this usage is optional.
l That place, I reckon, is really haunted.

l Well yes, I hate it.

Downloaded From : www.EasyEngineering.net


Downloaded From : www.EasyEngineering.net

18  l  Grammar— Basic Concepts & Common Mistakes

(e) Use a comma to set off an appositive (a word or a phrase which renames a noun)
l Mike, Ron’s father, was a brilliant mathematician.

End of sentence punctuation marks:


These punctuation marks let the reader know that the sentence is over. The punctuation marks which indicate the end of a
sentence are:
(2) Period
It is the most frequently used end punctuation marks; it is just used to terminate a sentence.
l Go to the kitchen, and bring me something to eat.
l It is quite windy today.
(3) Exclamation mark.
This punctuation mark is used after an exclamatory sentence. Use it in a sentence which represents a sense of urgency, excitement
and some strong emotions. Exclamation marks are rarely, if at all, used in formal writing.
l This fish is really beautiful!
l Go there now!
The exclamation mark should not be used to unnecessary exaggerate sentences, like:–
He is the best player ever!

ww
The word best has already emphasized and stressed how good a player he was, so there is no need of an exclamation mark in the
end.
(4) Question mark

l
w.E
This punctuation mark is used after a sentence which refers to a direct question:-
Where are you going?
What is his name?

l
asy
As already stated, the question marks should be used only after a direct question. We will give some examples of indirect
questions. See that no question mark is required in such sentences:–
I am not sure who broke this window.


It is not a direct question to anyone.
l He then asked some tough questions. En
Again, no question is asked to the listener (s).
(5) Semi-colons
gin
These are used to join two independent clauses which are not joined by coordinating conjunctions; you may consider a semi-

eer
colon as a replacement for coordinating conjunctions. Note that the independent clauses joined by a semi-colon must always be
related to each other. For example, the semi-colon is wrongly used in the sentence given below.
l  The weather is nice; food is delicious.

ing
There is no relation or obvious connection between the independent clauses of this sentence, so the semi-colon should not be
used here.


You should also not use semi-colon in case of a dependent clause, like –
l  After having a good dinner; we went for a walk. .ne

A comma should be used in place of this semi-colon.
We will now give a couple of examples showing an appropriate use of the semi-colon:-
l  I like all the subjects we are taught in school; however, science is my favourite subject.
t
The word ‘however’ in this sentence is known as a conjunctive adverb or just a transition word as it connects two clauses of a
compound sentence.
l  Eating good food keeps one healthy; exercising is also important.

Semi-colons can be used to avoid confusion in case a string of commas already exists, like –
I have been to quite a few famous places: London, England; Paris, France; Rome, Italy; and Athens, Greece.
A semi-colon should not be used where a coordinating conjunction exists, like:-
I would love to buy this car; but, I do not have enough money for it.
A comma should be used instead of this semi-colon.
In some particular cases, you can use a semi-colon when a coordinating conjunction already exists: if independent clauses are
very long, and quite a few number of commas have been used already.
(6) Colons
A lot of people confuse the colon with the semi-colon, and vice versa, but the use of these punctuation marks is very different.
Basically, a colon is used to call the attention of the readers to what is to follow. We will now describe the uses of a colon and when
it should be used:-
(a) Use a colon to introduce a list –
l The cities I have visited: London, Istanbul, Athens and Rome.

Downloaded From : www.EasyEngineering.net


Downloaded From : www.EasyEngineering.net

Grammar— Basic Concepts & Common Mistakes  l 19

(b) Colons can also be used after an independent clause, but only if the clause that follows this colon explains or reasons the
previous clause, or just somehow completes it. For example:
l My presentation went really well: everyone said that I presented my ideas nicely.
l All his hard work resulted in nothing: he failed to acquire the passing marks.
l I learned a great lesson today: never give up.
Now, let us see how a colon should not be used:–
(a) Do not use a colon between a preposition and its object.
I went to: London, Rome, Athens and Paris. (Incorrect)
I went to London, Rome, Athens and Paris. (Correct)
(b) A colon should also not be used between a verb and its object.
My favourite food cuisines are: Indian, Chinese and Mexican. (Incorrect)
My favourite food cuisines are Indian, Chinese and Mexican. (Correct)
(c) A colon should not be used after explanatory or introductory words.
It is common knowledge that the bureaucrats in the department, such as: Mr. Prasad, Mr. Das and Mr. Choudhary are corrupt.
Incorrect)
It is common knowledge that the bureaucrats in the department, such as Mr. Prasad, Mr. Das and Mr. Choudhary are corrupt.

ww
(Correct)
(7) Hyphens
Prefixes, suffixes and letters are joined to words by hyphens. They are also used to join word units or form compound words. We


l
w.E
will now see some of the examples of how they are used:
One-thirds of the people say they do not smoke.
Fifty-five people have applied for this job.
(8) Parenthesis

asy
The elements which are inside the parentheses are related to the sentence, but cannot be deemed essential. Let us see how these
are used –

En
(a) They add or show information that is not essential to the sentence; they are also used to de-emphasize what is mentioned
before them. For example –

gin
l  We saw quite a few European countries (Italy, England, Spain) last summer.

(b) These can also be used to enclose numbers, i.e. a list in the sentence:
l  I want to see (1) Delhi, (2) Bombay and (3) Srinagar

eer
(c) A sentence can also be enclosed in parenthesis which would help explain or clarify the information conveyed in the statement.
Note that when a sentence is enclosed in brackets, no period is needed, but exclamation and question marks can be used, if


necessary, like

ing
l  The clothes (I bought them two weeks back) were all there in the suitcase.

(d) An independent sentence can also be enclosed in parenthesis. In that case, the end punctuation will be inside the parentheses:

.ne
l  Chetan said that the movie was great (I had seen that movie too.) He said it is one of the best movies he has ever seen.
(9) Apostrophe


l  Ron’s car is one of a kind.

The use of ‘Ron’s’ shows the possessive case.


t
The possessive case of a noun is shown by an apostrophe, it can also be used to show the letters you have left out in contraction.
Let us see some of the examples:


l  Reviews of this movie are good, but I haven’t seen it yet.

The use of ‘haven’t’ is a contraction of have not.


The possessive case of a noun is shown by putting an apostrophe just after the s.

l  I am very concerned about my friends’ future.
Possessive pronouns do not need an apostrophe, like hers, theirs, and yours. The same applies to the possessive pronoun its (it’s
on the other hand is just a contraction of it is)

l Its colours are magnificent.

l I brought my id card, but she forgot to bring hers.
COMMON MISTAKES
(1) Overlooking the Subject and Verb
It is a very basic rule in grammar, but sometimes students just seem to skip it: a sentence must have a verb and a subject;
otherwise, it cannot be called a proper sentence. So, while looking for the correctness of a sentence, check out the verb and
subject of that sentence. Knowing them, also helps in checking errors like subject-verb agreement, sentence fragmentation etc.
Students sometimes directly go hunting for some of the complicated errors that might be in a sentence, but it is always advisable
to check the subject and verb of the sentence first.

Downloaded From : www.EasyEngineering.net


Downloaded From : www.EasyEngineering.net

20  l  Grammar— Basic Concepts & Common Mistakes

(2) Ignoring redundancy


An appropriate sentence is one which is concise and clear. A sentence which is wordy, or stretched, although could be
grammatically correct, but it cannot be preferred over a sentence which conveys the same message but is much more concise.
Such errors are redundancy errors, which incidentally, the students easily miss out on. To understand what we are talking about
here, let’s take an example:
l  The inflation rose by a 10 percent increase.

l  The inflation rose by 10 percent.

As you can see, ‘b’ is much more precise and concise; whereas, ‘a’ has a redundancy error.
(3) Relying on what you hear
Adjudging a sentence correct just because ‘it sounds right’ is risky. It might be that you have read/heard something like that
before. But it is possible that either what you heard was lexically wrong, or was grammatically incorrect. Always base your
judgment of a sentence on grammar rules.
(4) Finding answers by looking at options.
If you are relying on options to help you find the answer, then you are making it tougher for yourself. Following this approach
could also mean that you have not really understood what is wrong with the given statement. It is better to spend time on

ww
understanding the possible error rather than eliminate options. Eliminating options could also lead you to comparing the
options, which would again waste your time.
(5) Logic of the sentence

w.E
A statement has to logically make sense. This is of prime importance too. Make sure that a statement you think is right, makes
logical sense too. Understanding the sentence that follows or precedes the given underlined part, can really help.
(6) While replacing the underlined part with an option, ensure that you have not ended up adding some information which cannot

asy
be derived from the given sentences. Our job is to form a grammatically correct statement, and not infer information.

SOME MORE GRAMMAR MISTAKES


(1) ‘Into’ OR ‘in to’
En
‘Into’ is a preposition or a linking verb which means “expressing movement or action with the result that someone or something

gin
becomes enclosed or surrounded by something else” (source: Oxford dictionary). For example:
Deepam got into her car.
She put the books into her bag.

eer
There are cases where the word ‘in’ and ‘to’ just happen to come together. For example:
He walked in to see if his friend was there.
(2) Its OR it’s
ing
‘Its’ is a possessive noun, indicating that something belongs to someone, for example: I took away its tires. ‘It’s is the short of ‘it
is’. For example: Find out the keys, it’s surely in the house.
(3) Who OR whom
.ne
the book?” ‘Whom’ is used when it works as the object of the verb, for example: “Whom did you see in the corridor?” “From
whom did you buy that?”
(4) Me / myself / I
t
Both of these are pronouns, but ‘who’ is used when it is the subject of a verb, for example: “Who is he?” “Do you know who stole

Deciding between ‘me’ and ‘I’ is similar to the choice between ‘who’ and ‘whom’. ‘Me’ works as the object, and ‘I’ serves as the
subject of the sentence.
For example:
I did it.
He did it for me.
(5) Lie OR Lay
The word ‘lay’ is often incorrectly used. While using ‘lay’, there must be an object, i.e., someone would ‘lay’ something somewhere.
This is why, when mentioning rest, you lie down (not lay down).
(6) Irregular verbs
These are verbs with which we do not use the normal conjugations to create the different forms. For example, we use –ed with
‘play’ to create ‘played’. But the same past simple verb form for ‘know’ will be ‘known’. There are a number of irregular verbs and
you must know them all to not make a mistake. For example, the word ‘broadcasted’ doesn’t exist, it is just ‘broadcast’, so you
have no option but to say something like, “that show was broadcast late night”; ‘impactful’ is not a word, ‘impact’ is a noun which
refers to one body striking another or just some strong influence.
Some of the common irregular verbs are mentioned in the table below:

Downloaded From : www.EasyEngineering.net


Downloaded From : www.EasyEngineering.net

Grammar— Basic Concepts & Common Mistakes  l 21

V1 V2 V3 V1 V2 V3
Base Form Past Simple Past Participle Base Form Past Simple Past Participle
Awake Awoke Awoken Hear Heard Heard
Be Was, were Been Hide Hid Hidden
Beat Beat Beaten Hit Hit Hit
Become Became Become Hold Held Held
Begin Began Begun Hurt Hurt Hurt
Bend Bent Bent Keep Kept Kept
Bet Bet Bet Know Knew Known
Bid Bid Bid Lay Laid Laid
Bite Bit Bitten Lead Led Led
Blow Blew Blown Learn Learned/learnt Learned/learnt
Break Broke Broken Leave Left Left
Bring
Broadcast wwBrought
Broadcast
Brought
Broadcast
Lend
Let
Lent
Let
Lent
Let
Build
Burn
Buy
w.E
Built
Burned/burnt
Bought
Built
Burned/burnt
Bought
Lie
Lose
Make
Lay
Lost
Made
Lain
Lost
Made
Catch
Choose
Caught
Chose asy
Caught
Chosen
Mean
Meet
Meant
Met
Meant
Met
Come
Cost
Came
Cost
Come
Cost En Pay
Put
Paid
Put
Paid
Put
Cut
Dig
Cut
Dug
Cut
Dug gin
Read
Ride
Read
Rode
Read
Ridden
Do
Draw
Did
Drew
Done
Drawn
Ring
Rise eer Rang
Rose
Rung
Risen
Dream
Drive
Dreamed/dreamt
Drove
Dreamed/dreamt
Driven
Run
Say
ing
Ran
Said
Run
Said
Drink
Eat
Drank
Ate
Drunk
Eaten
See
Sell
Saw
Sold
.ne
Seen
Sold
Fall
Feel
Fight
Fell
Felt
Fought
Fallen
Felt
Fought
Send
Show
Shut
Sent
Showed
Shut
Sent
Showed/shown
Shut
t
Find Found Found Sing Sang Sung
Fly Flew Flown Sit Sat Sat
Forget Forgot Forgotten Sleep Slept Slept
Forgive Forgave Forgiven Speak Spoke Spoken
Freeze Froze Frozen Spend Spent Spent
Get Got Got (sometimes Stand Stood Stood
gotten)
Swim Swam Swum
Give Gave Given
Take Took Taken
Go Went Gone
Teach Taught Taught
Grow Grew Grown
Tear Tore Torn
Hang Hung Hung
Tell Told Told
Have Had Had
Think Thought Thought

Downloaded From : www.EasyEngineering.net


Downloaded From : www.EasyEngineering.net

22  l  Grammar— Basic Concepts & Common Mistakes

V1 V2 V3
Base Form Past Simple Past Participle
Throw Threw Thrown
Understand Understood Understood
Wake Woke Woken
Wear Wore Worn
Win Won Won
Write Wrote Written

(7) Affect OR Effect


These are often mistakenly used in the same way, but they actually differ. ‘Affect’ is used as a verb, for example:
Your laziness is affecting your performance.
Money affects your standard of living.
The word ‘effect’ is used more like a noun, for example:

ww
There was no effect of scolding on the delinquent.
Please remove your personal effects from the office.
(8) Bring OR Take

w.E
Its correct usage depends on whether the object is moving towards or away from the subject being talked about in the sentence.
If it is moving away, then use ‘take’ whereas, use ‘bring’ in case of motion towards the subject. For example:
Tell your maid to take the clothes to the cleaners.
Bring some chocolates from the market.
(9) Since OR Because
asy
Both these words are normally used interchangeably, but there’s a subtle difference in what they refer to. ‘Since’ normally refers

En
to time, whereas ‘because’ refers to cause. In those cases, where ‘since’ is used to mean causation, it can be differed from because
by the fact that ‘because’ puts relatively more emphasis on the cause. For example:
I haven’t had food since the morning.
I am feeling hungry because I haven’t eaten anything.
(10) Neither … Nor OR Either … Or
gin
Neither is always used with ‘nor’, whereas either with ‘or’. For example:
Neither Carl nor Jackson is fond of music.
I’ll either listen to some songs or go out. eer
AND YET MORE… AVOIDABLE MISTAKES
ing
(1) Sentence is a set of words that is complete in itself, so a sentence must stand alone and be meaningful. We often carelessly use


meaningful. For example:
.ne
full stops and forget their importance. A full stop (or period) should signal a complete and stand-alone sentence which is also

Mitesh is a very rich businessman, but he has never done charity works. Although, he’s a very good and king-hearted person. (Wrong)

(2) Redundant sentences: A redundant sentence is not grammatically incorrect, but precise and concise sentences are always


preferred. Therefore, we should make sure that the sentences are not ‘tiresome’: t
Mitesh is a very rich businessman, but he has never done charity works, although, he’s a very good and king-hearted person. (Right)

The meeting was scheduled for Friday, but very few were able to attend it, so it was then rescheduled and the new meeting would be
held the following Monday. (Wrong)
The meeting which was scheduled for Friday, was eventually rescheduled for the following Tuesday so everyone could attend it. (Right)
(3) Omitted Commas: Sentences in which commas aren’t used properly can fail to make sense.
For example:
When it comes to dancing, people differ in their choices. (Wrong)
When it comes to dancing, people differ in their choices. (Right)
(4) Subject-Verb Agreement: This is amongst the most commonly committed mistakes in Grammar. The basic rule is: the subject
and verb must agree in number, i.e., singular subjects should have singular verbs, whereas plural subjects should come with
plural verbs. For example:
‘He talk.’
This sentence is grammatically wrong. Here, ‘he’ is a singular subject, so it should have a singular verb, i.e. ‘talks’.
To find this error in a sentence, first find the subject and verb in it, and then see if they agree in ‘number’, by saying the subject
and verb together.
Finding Verb: Verbs are ‘doing words’ which represent the action done by the subject. Finding verbs is not as easy as it sounds.
There can be cases or situations when a word which looks like a verb would actually be serving as a noun:
l  Hippies sing in their own unique style.
l  Hippies love to sing.

Downloaded From : www.EasyEngineering.net


Downloaded From : www.EasyEngineering.net

Grammar— Basic Concepts & Common Mistakes  l 23

Now, let’s look at both the sentences. In the first one, ‘Hippies’ are the subject and ‘sing’ is the verb. Verb is indicating what the
subject does, or an action. The second sentence has the same subject or noun, but now ‘sing’ is not a verb, in fact, it is part of the
noun phrase because singing is something which they love.
Verbs form a crucial part of a sentence. They convey the essential meaning of the clause or a noun phrase. The first statement
basically tells that something sings, and the second one says that something loves something, so the verb in this sentence is ‘love’.
One more important thing to note in the second sentence is that a verb may need an object too (an object is what receives the
verb). ‘Singing’ is what is being loved, thus, the object is ‘sing.’
The following are some rules that can help you in checking whether the subject and verb agree in number:
I. Phrases like ‘Ricky and Mike’ are plural, whereas the following would be singular:
l  Ricky, in addition to Mike
l  Ricky, as well as Mike
l  Ricky, along with Mike
l  Ricky, together with Mike
II. The following pronouns are singular: each, anyone, anything, someone, something, somebody, everything, no one, another,
everything, etc.
I II. The following conjunctions are singular too: neither, either.
Some tricky singulars and plurals:

ww
Plural: phenomena; singular: phenomenon
Plural: data; singular: datum
Plural: criteria; singular: criterion
Plural: bacteria; singular: bacterium


w.E
(5) Parallelism: “The same grammatical form should be maintained while comparing or listing things in a sentence.”
Before we explain what is actually meant by this, let’s understand what is meant by infinitives and gerunds.
Infinitives – These are verb-like phrases which begin with the preposition ‘to’. They can also act as nouns in a sentence. For


example:
I like to sing.
I like to travel. asy

En
In the above example, you can easily see that ‘to sing’ and ‘to travel’ are acting as nouns.
Gerunds – These are words formed by adding ‘-ing’ to the verb. They can also act as nouns.



For example:
l  I like singing.
l  I like travelling. gin
eer
Gerunds in the above sentence are acting as nouns because ‘singing’ and ‘travelling’ are something which the subject likes.
Gerunds and Infinitives are used interchangeably, but it is not always preferred. Infinitives are more specific than are gerunds.
Therefore, to provide a more clear or specific information or idea, infinitives are preferred.



For example:
l  Rocky loved singing.
l  Rocky loved to sing.
ing
Now, which one would you prefer, or say, which one gives a better information about Rocky?
.ne
The first sentence uses the gerund form and talks about ‘singing’. Now, it is not clear if Rocky loves to hear people singing songs,
or if he himself likes to sing. This distinction is cleared by using the infinitive form as in the second sentence.


For example:
l  I like singing more than I love to dance. (Wrong)
t
So, let’s come back to the grammar rule that we previously talked about. While we are comparing things or events in a sentence,
we should either use infinitives or gerunds. Using both of them in a sentence is deemed grammatically incorrect.

In the above statement, a person is comparing entities he likes, but he used both, a gerund (singing) and the infinitive (to dance)
form. The corrected and preferred form would be:
I like to sing more than I like to dance.
Parallel construction of sentences:
If you are getting confused by what is being said here, i.e., in what terms the sentence should have used the same grammatical
form, the example below should help:
(Both X and Y, should be of the same grammatical form)
(a) x is like y e.g. singing is like dancing, etc. (b) x more than y (c) Prefer x to y
(d) Neither x nor y (e) Either x or y (f) Both x and y
(g) The more x, the less y (h) The better x, the better y (i) Not only x, but also y
(j) Not x, but y (k) Less y than y (l) More x than y
(6) Comparison Problems: This error is committed a lot of times: two non-comparable things are compared in a sentence.
For example:
l  His chances of playing well tomorrow are less than lottery. (Wrong)

In the above statement, ‘chances of playing well’ are compared to ‘lottery’.


l  His chances of playing well tomorrow are less than his chances of winning the lottery. (Right)

Downloaded From : www.EasyEngineering.net


Downloaded From : www.EasyEngineering.net

24  l  Grammar— Basic Concepts & Common Mistakes

In the above statement, a logical comparison is made by contrasting the chances of the two events. Comparison is not made
between chances of an event, and the event itself, like in the grammatically incorrect statement mentioned above.
So make sure you do not commit such silly mistakes.
Apart from this, the things or items compared in a sentence should always be same in number, i.e., either singular or plural.
For example:
l Both of them were sore loser. (Wrong)
l Both of them were sore losers. (Right)
Usage of comparing words:
(a) Words like fewer, many or number are used only when countable things are talked about.
For example:
l He has many interesting books in shelf.
l He has a number of hobbies.
l Fewer people came to this week’s concert.
(b) Words like less, much or amount are used only when uncountable things are talked about.
For example:
l There is less traffic on the outskirts of a town.
l There is much to worry about the country’s economic state.

wwl Huge amount of water was there on the field.
(c) Words like between, more or any adjective with (–er) like stronger is used while comparing exactly two things.
For example:


w.E
l
l
Carl is stronger than Mohan.
Carl is more powerful than Mohan.
(d) Words like among, most and adjectives with (-est) are used while comparison is made between more than two things.
For example:



l
l asy
Carl is the strongest in his class.
Carl is the most powerful guy in this class.
Carl is the best among his classmates.
l
(7) Pronoun-Antecedent disagreement
En
Pronouns are those words which replace a noun or noun phrase, and their usage removes the redundancy in statement. Pronouns

neither (not referring to a specific thing, place, idea or person).


gin
can be definite like it, she, he, you and I (referring to a specified thing, place, idea or person), or indefinite, like anyone, those,

The literal meaning of ‘antecedent’ is something or someone from which something descends. (for example, you are your
grandparents’ descendants)
Now, let’s understand what is meant by pronoun-antecedent disagreement: eer
ing
Each and every definite pronoun takes the place of or replaces a noun in the sentence, which is called as ‘antecedent’. The
grammatical rule is that both the antecedent and this pronoun must agree in number and kind (personal or impersonal).
For example:
l  Chinu was the one that stood out in that crowd.

.ne
In the above sentence, the pronoun (that) doesn’t agree with its antecedent (Chinu) in the kind as of course. Carl is a person but,
‘that’ is an impersonal pronoun. The correct form would be:
l  Chinu was the one who stood out in that crowd.
Some other rules related to antecedent and pronoun usage are mentioned below:
(a) Antecedent of a pronoun should be clear and not ambiguous. For example:
l  Mita went shopping with Renu and she bought a football kit. (Wrong)
t
In the above statement, does the pronoun refer to Mita or Renu? It is not clear. Hence, the above statement is grammatically wrong.
l  Mita bought a football kit when she went to shopping with Renu. (Right)
(b) Correct usage of interrogative pronouns
We have listed the correct usage of pronouns below:
l Who - referring to a person
l What - referring to a thing
l Where - referring to a place
l When - referring to a time
l Why - referring to a reason
l How - referring to an explanation
(c) Pronoun Consistency
You should use the same pronoun while referring to a same thing, place, person or idea. For example:
l  ‘One’ should be careful about ‘themselves’ while visiting places which are under some regional conflicts. (Wrong)

In the above statement, two different pronouns are used to refer to the same noun.
l  ‘One’ should be careful about ‘oneself ’ while visiting places which are under some regional conflicts. (Right)

(d) Adjectives: These words modify the nouns and pronouns by describing something about them.

Downloaded From : www.EasyEngineering.net


Downloaded From : www.EasyEngineering.net

Grammar— Basic Concepts & Common Mistakes  l 25

(8) Coordinating sentences.


Sentences can be of two types: Compound or complex.
(a) Compound sentences are those which contain more than one clauses, or complete ideas. Here, the individual clauses can
also stand alone as sentences. Hence, there are no dependent clauses. For example:
l  Weather was awful yesterday with heavy rains and strong winds; it was a struggle to even reach home safely.
The above sentence contains two clauses which are separated by a semicolon. Each of these can be stand-alone sentences.
Thus, these two are independent clauses. We can say that the above sentence is a compound sentence.
(b) Complex sentences contain dependent clauses, i.e. at least one clause of the sentence cannot stand alone as a sentence
because it would not represent a complete idea. For example:
l  As soon as I entered the stadium in our derby match, I was blown away by the atmosphere and the sheer enthusiasm of
fans cheering for their team.
Now, the above sentence is complex, because it contains a dependent clause: ‘As soon as I entered the stadium in our derby
match’, this clause cannot stand alone as a sentence, because it’s not representing a complete idea. It is dependent on the
subsequent clause to complete the information or idea.
Now, let’s look at the kind of errors made in this section:
Run-on sentences


ww
If two independent clauses are joined by a comma, then it leads to a grammatical error known as a run-on sentence. In such
cases, either a colon or semicolon is used. For example:
l I went to bed early that night, I was very tired as it had been a long day. (Wrong)

w.E
l I went to bed early that night: I was very tired as it had been a long day. (Right)
We can see that both the clauses in the above sentences are independent, so here we have a compound sentence. A colon or
semicolon should not be used interchangeably, as they refer or indicate different things:

l
asy
(a) A colon implies that an explanation will follow, i.e. the clause before the colon will be explained or reasoned by the clause that
follows it. For example:
His performance went smoothly and everyone in the crowd loved his act. (Wrong)
l

En
His performance went smoothly: everyone in the crowd loved his act. (Right)
The first statement is a bit unclear, because we cannot be sure of why that performance went well. Was it due to the fact that the
crowd loved it or for any other reason?
gin
This ambiguity is removed by the use of colon which indicates that the first clause has been explained or reasoned by the one
following it.
(b) A semicolon is used to join two independent clauses which are closely related.
For example: eer


l

l
I have been to a lot of countries, Greece is my favourite place. (Wrong)
I have been to lots of countries; Greece is my favourite place. (Right)
ing
The first one is a run-on sentence. As the sentence contains two related independent clauses, hence the right way to join them is
via a semicolon.
.ne
Note: You should not use both the semicolon and conjunction to join clauses in a sentence. Pick either a conjunction or a
semicolon to join independent clauses.
(9) Modifier problems t
It is another common mistake to use adjectives as adverbs. It is important to remember that adjectives only modify the nouns;
whereas, adverbs modify verbs and even adverbs, or adjectives. Let’s explain this error through an example:
l  As he is British and someone who always lived in Britain, I was surprised at how fluent he represented his ideas in Hindi. (Wrong)

This statement is wrong, because the verb talked about in the sentence is ‘representing’ the ideas, and how was that done?
Fluently. Thus, an adverb should be used there instead of an adjective. Just saying that ‘his Hindi was fluent’ would have been
right.
Note: Some words can work as both adverbs and adjectives, depending on how they are used. For example: Ferraris are fast cars.
Here ‘fast’ is an adjective, describing how Ferraris (a noun) are like.
l  Usain Bolt runs fast.

Here, ‘fast’ is an adverb describing how the verb, i.e. ‘running’ is done.
Comparative Adjectives and adverbs:
While using adjectives or adverbs for comparing nouns or actions, we should be careful of the form we use:
Let’s talk about adjectives first. There are two ways through which adjectives are made comparative, and there’s no rule as such to
determine which form is used when. You just have to know the right word. For example: Fast can be made comparative by using
the form: faster; whereas, beautiful is made comparative by using the form: more beautiful. We can see that words like ‘more fast’
or ‘beautiful-er’ just sound wrong.

Downloaded From : www.EasyEngineering.net


Downloaded From : www.EasyEngineering.net

26  l  Grammar— Basic Concepts & Common Mistakes

Mostly, adverbs are made comparative by adding more. For example:


l  Jess performed ‘more gracefully’ than Alex did.

But, there are some irregular adverbs too, like fast (it is made comparative in faster, not more fast)
Note: Some modifiers should only be used in their absolute form, as using them in their comparative form makes them
illogical. For example:
l  I adore both the players, but Steve is ‘more perfect’ than Michael. (Wrong)

Perfect is itself some sense of completeness, and there’s nothing like more complete.

TYPES OF GRAMMAR QUESTIONS


Spotting error (Discussed later)
Sentence Correction (Discussed later)
Choosing the correct sentence (Discussed later)
Identifying the part of sentence which is grammatically wrong
In such questions, a sentence will be presented to you in four parts, namely: A, B, C, & D. Together, these four form a meaningful
sentence: but one of these has a grammatical error and your job is to identify it.
Example:
ww
(a) The Mumbai police have found

w.E
(b) the body of a man
(c) who they believe to be
(d) the prime suspect in the murder case

asy
(c); The option that is grammatically incorrect is option (c). It incorrectly uses ‘who’ in place of ‘whom’. The difference between
‘who’ and ‘whom’ is like the one between ‘I’ and ‘Me’. ‘Who’ acts as a subject, whereas ‘whom’ acts as the object in the sentence. Here
the Subject (Police) performed an action (found) on the person they found out. So, ‘whom’ is the right pronoun here.

En
Identifying which of the given statements are incorrect/ inappropriate
l This is just a variation of the major question types, in which you have to pick the incorrect sentence instead of the correct one.

gin
In these questions, pay attention to the directions given alongside.

SKILLS REQUIRED
(1) Knowledge of Grammar Rules
eer
This is the most important attribute: knowing all the important grammatical rules, and their proper usage. You cannot argue

ing
against this. If you don’t know the rules, or if you cannot spot something fishy, then you cannot solve these questions. We have
discussed basic grammar in the beginning of this section, along with the common mistakes, so, it will be good for you to recap

.ne
that before we head to the questions. Also, if you want, you can try the internet for detailed understanding of the topics that you
feel, you need more practice for.
(2) Vocabulary
t
If you know a lot of words, and also know their appropriate usage, then not only understanding, but also tackling questions
would be easier for you. It will make you understand the meaning and context of sentences better, thereby aiding you in solving
the question.
(Of course, some questions are just based on checking your vocabulary, but a good vocabulary can also help you in tackling other
questions in the verbal section.)
(3) Clarity of thought
You need to be clear in your thoughts. Any ambiguity or opacity could lead to picking the wrong answer. If you are confused
about any options, remember that with grammar questions, there is always only one correct way. Because of this, making an
educated guess may work in some other sections, but trying something like that with grammar could land you in a tough spot.
Staying on top of your nerves and keeping a clear head is especially required in this section, because you will be have to remember
and bear in your mind all the rules of grammar, or the all the meanings a word might have. So, you need to be clear and consistent
in your conclusions and thoughts.
(4) Intellectual capacity
It refers to the ability to understand something. It is quintessential that you are able to properly comprehend what each of the
given sentence, clause or phrase is trying to convey. This is one fundamental step in solving a question. If you do not understand
the meaning, and are just trying to solve the question based on the grammatical correctness part, you might end up choosing the
wrong answer. The sentence must always make proper sense.

Downloaded From : www.EasyEngineering.net


Downloaded From : www.EasyEngineering.net

2
PTER
CHA

Active & Passive Voice

ww
INTRODUCTION
Compare the following sentences
To change a sentence from active to passive voice, do the
following:
1. Move the active sentence’s direct object into the sentence’s

w.E
Sentence I : Ram helps Hari.
Sentence II : Hari is helped by Ram.
While both sentences express the same meaning, there is a
subject slot
ActiveVoice →
Marilyn mailed the letter →
Passive Voice
The letter . . .

asy
difference in their construction, the difference of voice.
In sentence I, the subject Ram is the doer of the action and thus
it is in Active voice, the verb is in the Active voice.

direct object

subject

thus the verb is in the Passive Voice. En


In sentence II, the subject is Hari to whom the action is done and 2. Place the active sentence’s subject into a phrase beginning
with the preposition by
Following are some examples of Active and passive voice
sentences gin Active Voice →
Mailyn mailed the letter. →
Passive Voice
The . . . by Marilyn.
Active Voice
The peon opened the gate.
Passive Voice
The gate was opened by the
peon.

eer
subject

Some boys were helping the


old man.
The old man was being
helped by some boys.
3.
ing
Add a form of the auxiliary verb be to the main verb and
change the main verb’s form

He will finish the work in a


fortnight.
The work will be finished by
him in a fortnight.
Active Voice →
Mailyn mailed the letter. → T
.ne
Passive Voice
he letter was mailed by Marilyn.
Why did your brother write
such a letter?
He handed her a chair.
Why was such a letter written
by your brother ?
A chair was handed to her by
him.

verb
Because passive voice sentences necessarily add words and
t
change the normal doer-action-receiver of action direction, they
Following are the rules for transformation of voice. may make the reader work harder to understand the intended
meaning.
Tense Active voice Passive Voice
As the examples below illustrate, a sentence in active voice
Simple present take/takes is/am/are taken flows more smoothly and is easier to understand than the same
Present is/am/are taking is/am/are being sentence in passive voice.
continuous taken
Active Voice
Present perfect has/have taken has/have been At each concert the soprano sang at least one tune from a
taken well-known opera. ↑ ↑ ↑
Simple past took was/were taken doer of action receiver
action of action
Past continuous was/were taking was/were being
Passive Voice
taken
At each concert, at least one tune from a well-known opera
Past perfect had taken had been taken was sung by the soprano. ↑
Simple future will/shall take will/shall be ↑ ↑ receiver
taken action doer of action of action

Downloaded From : www.EasyEngineering.net


Downloaded From : www.EasyEngineering.net

28  l  Active & Passive Voice

Active Voice
Asbestos abatement teams will remove large chunks of
asbestos-laden material from the hallways on the second and
third floors.
doer of action
receiver of action
action
Passive Voice
Large chunks of asbestos-laden material will be removed
from the hallways on the second and third floors by asbestos
abatement teams.
action
receiver of doer of action
action
It is generally preferable to use the ACTIVE VOICE.
To change a passive voice sentence into an active voice sentence, simply reverse the steps shown above.
1. Move the passive sentence’s subject into the active sentence’s direct object slot

ww
Passive Voice
The letter was mailed →
by Marilyn.
→ Active Voice
. . .(mailed) the letter.

w.E ↑
subject

direct object
2.

The letter was mailed . . . → asy
Remove the auxiliary verb be from the main verb and change main verb’s form if needed
Passive Voice Active Voice
mailed the letter

be auxiliary En
3.
Passive Voice → Active Voice gin
Place the passive sentence’s object of the preposition by into the subject slot.

. . .by Marilyn.

→ * Marilyn mailed .

eer
object of preposition by subject

ing
Because it is more direct, most writers prefer to use the active voice whenever possible. The passive voice may be a better choice,
however, when
• the doer of the action is unknown, unwanted, or unneeded in the sentence
Examples: .ne

• The ballots have been counted.
• Sometimes our efforts are not fully appreciated.
the writer wishes to emphasize the action of the sentence rather than the doer of the action
Examples:
t
• The high-jump record was finally broken last Saturday.
• A suspect was questioned for sixteen hours by the police.
• The writer wishes to use passive voice for sentence variety.

Downloaded From : www.EasyEngineering.net


Downloaded From : www.EasyEngineering.net

Active & Passive Voice  l 29

Practice Exercise
LEVEL-I
DIRECTIONS (Qs. 1-40) : In the following questions, the 7. People speak English all over the word.
sentences have been given in Active / Passive Voice. From the (a) English is spoken all over the world.
given alternatives, choose the one which best expresses the given (b) English was spoken all over the world.
sentence in Passive / Active Voice. (c) English was spoken by people.
1. They first sun-dried the garbage for one to three days to bring (d) English is spoken by people.
down the moisture level. 8. Who gave you permission to enter?
(a) The moisture level was brought down by sun-drying (a) By whom were you given permission to enter?
the garbage for one to three days
(b) By whom was you given permission to enter?
(b) One to three days of sun-drying brought down the
(c) By whom you were given permission to enter?

ww
moisture level of the garbage.
(c) The moisture level of the garbage came down when it
was sun-dried for one to three days.
9.
(d) By whom given you permission to enter?
The Principal has granted him a scholarship.

w.E
(d) The garbage was first sundried for one to three days to
bring down the moisture level.
2. Women like men to flatter them.
(a) A scholarship has granted to him by the Principal.
(b) He has been granted a scholarship by the Principal.
(c) He has granted a scholarship by the Principal.

asy
(a) Men are liked by women to flatter them.
(b) Women like to be flattered by men. 10.
(d) A scholarship was granted to him by the Principal.
Before festivals the shops are thronged with men, women
(c) Women like that men should flatter them.
(d) Women are liked to be flattered by men. En and children making various purchases.
(a) During festivals people throng the shops
3. What one must do, one must do properly.
(a) What must be done, must be done properly. gin (b) Men, women and children throng the shops before
festivals making various purchases.
(b) It must be done properly what one must do.
(c) It must be done what one must do properly.
(d) One must do properly what has to be done.
eer
(c) Men, women and children make purchases during
festivals.

4. Look at the poll results-do they inspire hope?


(a) Let the poll results be looked-is hope inspired by them?
11.
ing
(d) The shops are thronged by people making purchases.
We all know that there is only one God.
(a) We are all known that there is only one God.
(b) Let the poll results be looked at-has hope been inspired
by them? .ne
(b) It is known to us all that there is only one God.
(c) We have all known that there is only one God.
(c) Let the poll results be looked at-is hope being inspired
by them?
(d) Let the poll results be looked at-is hope inspired by
them?
12.
(d) Only one God is known by us all.
The people elected him Mayor.
(a) Him was elected Mayor the people.
t
5. It is your duty to make tea at eleven O’clock. (b) He was elected Mayor by the people.
(a) You are asked to make tea at eleven O’clock. (c) Mayor is elected by the people.
(b) Your are required to make tea at eleven O’clock. (d) He is elected by the people Mayor.
(c) You ar esupposed to make tea at eleven O’clock. 13. Don’t laugh at me.
(d) Tea is to be made by you at eleven O’clock. (a) Let me be laughed at.
6. He was congratulated by his teacher on his brilliant success (b) Let me be not laughed at.
in the recent examination. (c) I am laughed at.
(a) His teacher congratulated him on his brillant success (d) Let me be not laughed.
in the recent examination. 14. I saw him leaving the house.
(b) His teacher congratulated him for his success in the (a) Leaving the house he was seen by me.
examination. (b) He was seen leaving the house by me.
(c) His teacher congraulated him on his success. (c) He had been seen leaving the house.
(d) His teacher congratulated him. (d) He was seen to be leaving the house.

Downloaded From : www.EasyEngineering.net


Downloaded From : www.EasyEngineering.net

30  l  Active & Passive Voice

15. Someone pulled the bull violently. 24. The telegraph wires have been cut.
(a) The bull had been pulled violently by someone. (a) Someone has been cut the telegraph wires
(b) The bull was to be pulled violently by someone. (b) No one has cut he telegraph wires.
(c) The bull had been pulled violently. (c) The telegraph wires have cut someone.
(d) The bull was pulled violently. (d) Someone has cut the telegraph wires.
16. This shirt cannot be worn by me any longer. 25. Will she tell us the truth?
(a) I cannot wear this shirt any longer. (a) Is the truth told to us by her?
(b) Wearing of this shirt any longer is not possible.
(b) The truth will be told to us by her.
(c) This shirt is too worn out to be worn any longer.
(c) Will the truth be told to us by her?
(d) This worn out shirt cannot be worn any longer
(d) Will the truth be told us by her?
17. A lion does not eat grass, however hungry he may be.
(a) Grass is not eaten by a lion, however hungry he may 26. Mr. Sen asked him a question.
be (a) He was asked a question Mr. Sen.
(b) Grass is not being eaten by a lion, however hungry he (b) He was asked a question to Mr. Sen.

ww
may be
(c) Grass is eaten not by a lion, however hungry he may
be
(c) He was asked a question by Mr. Sen.
(d) A question was being asked by Mr. Sen.
27. The teacher punished the boys who had not done their home

w.E
(d) Grass is being not eaten by a lion, however hungry he
may be
18. Someone saw him picking up a gun.
work.
(a) The boys who had not done their homework had been

asy
(a) He was seen pick up a gun by someone
(b) He was seen picking up a gun by someone
punished by their teacher.
(b) The boys were punished by their teacher who had not
done their homework.
(c) he was seen when he was picking up a gun
(d) He was seen by someone pick a gun En (c) The boys who had not done their homework were

19. He was obliged to resign.


(a) He was made to resign gin punished by the teacher.
(d) The boys who had not done their homework were being
(b) To resign was his obligation
(c) Circumstances obliged him to resign eer
punished by the teacher.
28. Somebody told me that there had been an explosion in the
(d) Resignation obliged him
20. Why did you not agree to my proposal?
(a) Why was my proposal not agreed to?
Town Hall.

ing
(a) I was told by somebody about the explosion in the Town
Hall.
(b) Why was my proposal not agreed by you?
(c) Why my proposal was not agreed to by you? .ne
(b) I was told about the explosion in the Town Hall.

(d) Why was my proposal not agreed to by you?


21. The boy has rung the bell
(a) The bell has been rung by the boy.
Hall.
t
(c) I was informed that there was an explosion in the Town

(d) I was told by somebody that there had been an explosion


in the Town Hall.
(b) The bell was being rung by the boy.
29. The doctor advised the patient not to eat rice.
(c) The bell was rung by the boy.
(a) The patient was advised by the doctor not to eat rice.
(d) The bell has been being rung by the boy.
(b) The patient was advised by the doctor that he should
22. He likes people to call him Sir.
(a) He likes to be called Sir by people. not eat rice.
(b) He likes to be call Sir by people. (c) The patient was being advised by the doctor that he
(c) He likes people who call him Sir. should not rice by the doctor.
(d) To call him Sir is liked by people. (d) The patient has been advised not to eat rice by the doctor.
23. We added up the money and found that it was correct. 30. I cannot accept your offer.
(a) The money was added up and found to be correct. (a) Your offer cannot be accepted by me.
(b) Correct it was found and the money was added up. (b) I cannot be accepted by your offer.
(c) The money added up by us and it was correctly found. (c) The offer cannot be accepted by me.
(d) The money added up by us found it was correct. (d) Your offer cannot be accepted.

Downloaded From : www.EasyEngineering.net


Downloaded From : www.EasyEngineering.net

Active & Passive Voice  l 31

31. You should open the wine about three hours before you use 39. You need to clean your shoes properly.
it. (a) Your shoes are needed to clean properly.
(a) Wine should be opened about three hours before use.
(b) You are needed to clean your shoes properly.
(b) Wine should be opened by you three hours before use.
(c) Your shoes need to be cleaned properly.
(c) Wine should be opened about three hours before you
(d) Your shoes are needed by you to clean properly.
use it.
40. He is said to be very rich.
(d) Wine should be opened about three hours before it is
(a) He said he is very rich.
used.
(b) People say he is very rich.
32. They will inform the police.
(c) He said it is very rich.
(a) The police will be informed by them.
(d) People say it is very rich.
(b) The police will inform them.
(c) The police are informed by them. DIRECTIONS (Qs. 41-45) : In the following questions, a

(d) Informed will be the police by them. sentence has been given in Active / Passive Voice. Out of the

ww
33. You can play with these kittens quite safely.
(a) These kittens can played with quite safely.
four alternatives suggested, select the one which best expresses
the same sentence in Passive /Active Voice and give the anuswer.

w.E
(b) These kittens can play with you quite safely.
(c) These kittens can be played with you quite safely.
(d) These kittens can be played with quite safely.
41. She took the dog for a walk.
(a) The dog was took for a walk.
[SSC CHSL 2012]

asy
34. A child could not have done this mischief.
(a) This mischief could not be done by a child.
(b) The dog took a walk by her.
(c) The dog was taken for a walk by her.
(b) This mischief could not been done by a child.
En (d) The dog took her for a walk.
(c) This mischief could not have been done by a child.
(d) This mischief a child could not have been done. gin
42. The waiter filled the glasses with water.
(a) The waiter was filled the glasses with water.
35. James Watt discovered the energy of steam.
(a) The energy of steam discovered James Watt. eer
(b) The water were filled in the glasses by waiter.
(c) The glasses filled with water by the waiter.
(b) The energy of steam was discovered by James Watt.
(c) James Watt was discovered by the energy of steam. 43. ing
(d) The glasses were filled with water by the waiter.
They will laugh at you
(d) James Watt had discovered energy by the steam.
36. She makes cakes every Sunday.
(a) You will be laughed at by them.
.ne
(b) You will have been laughed at by them.
(a) Every Sunday cakes made by her.
(b) Cakes are made by her every Sunday.
(c) Cakes make her every Sunday.
(c) You can be laughed at by them.
(d) You may be laughed at by them.
44. Trespassers shall be prosecuted.
(a) The authorities may prosecute trespassers.
t
(d) Cakes were made by her every Sunday.
(b) The authorities might prosecute trespassers.
37. She spoke to the official on duty. (c) The authorities can prosecute trespassers.
(a) The official on duty was spoken to by her (d) The authorities shall prosecute trespassers.
(b) The official was spoken to by her on duty. 45. He was driving the car so fast that it skidded on the snowy
(c) She was spoken to by the official on duty. road.
(d) She was the official to be spoken to on duty. (a) The car had been driven by him so fast that it skidded
on the snowy road.
38. They have built a perfect dam across the river.
(b) The car has been driven by him so fast that it skidded
(a) Across the river a perfect dam was built.
on the snowy road.
(b) A perfect dam has been built by them across the river. (c) The car was driven by him so fast that it skidded on
(c) A perfect dam should have been built by them. the snowy road.
(d) Across the river was a perfect dam. (d) The car was being driven by him so fast that it skidded
on the snowy road.

Downloaded From : www.EasyEngineering.net


Downloaded From : www.EasyEngineering.net

32  l  Active & Passive Voice

LEVEL-II
DIRECTIONS (Qs. 1- 50): In the following questions, a sentence 10. The principal will announce the results.
has been given in Active/Passive Voice. Out of the four alternative (a) The results will have announced by the principal.
suggested, select the one which best expresses the same sentence in (b) The results were announced by the principal.
Passive/Active Voice and mark your answer in the Answer Sheet. (c) The results will be announced by the principal.
1. The servant will execute all orders promptly. (d) The results are announced by the principal
(a) All orders will be executed promptly by the servant. 11. She is putting in many hours of work.
(b) All orders will have to be executed promptly by the (a) Many hours of work will be put in by her. .
servant. (b) Many hours of work is being put by her.
(c) All orders should be executed promptly by the servant. (c) Many hours of work would be put in by her.
(d) All orders would be executed promptly by the servant. (d) Many hours of work are being put in by her.
12. Open the door.
2. The gardener ought to water the plants daily.
(a) Let the door be opened.
(a) The plants have been watered daily by the gardener.
(b) The door shall be opened.
(b) The plants are ought to water daily by the gardener.

ww
(c) The plants ought to water daily by the gardener.
(d) The plants ought to be watered daily by the gardener.
13.
(c) The door was opened.
(d) Let the door shall be opened.
My mother bakes cakes.
3.

w.E
The convict was found guilty by the jury.
(a) The jury found that the convict was guilty.
(b) The jury finds the convict guilty.
(a) My mother is baking a cake.
(b) A-cake is being baked by my mother.
(c) Cakes are baked by my mother.

4.
(c) The jury found the convict guilty.
(d) The jury convicted him.
My father will write a letter. asy 14.
(d) A cake was baked by my mother.
The lady of the house was furnishing the mansion.
(a) The mansion has been furnished by the lady of the
(a) A letter is written by my father.
(b) A letter was written by my father. En house.
(b) The mansion was being furnished by the lady of the
(c) A letter will be written by my father.
(d) A letter will have been written by my father. gin house.
(c) The mansion is being furnished by the lady of the house.
5. They established this club in 2000.
(a) This club had been established in 2000.
(b) This club was being established in 2000.
15.
eer
(d) The mansion is furnished by the lady of the house.
My father has promised me a bicycle.
(a) I have promising a bicycle by my father.
(c) This club is being established in 2000.
(d) This club was established in 2000. ing
(b) I have been promised by my father a bicycle.
(c) have been promised a bicycle by my father.
6. For a long time people believed the earth to be flat.
(a) The earth is believed to be flat for a long time. 16. Everyone praises good men.
.ne
(d) I promised a bicycle by my father.

(a) Everyone give praises to good men.


(b) The earth was being believed to be flat for a long
time.
(c) The earth had been believed to be flat for a long-time.
(d) The earth was believed to be flat for a long time.
17.
(b) Good men are given praises by everyone.
(c) Good men are being given praises by everyone.
(b) Good men are praised by everyone.
I have lost my book.
t
7. My brilliant niece is speaking in Italian.
(a) my book have been lost.
(a) Italian was being spoken by my brilliant niece.
(b) my book must have been lost.
(b) Itailan is spoken by my brilliant niece.
(c) my book had been lost.
(c) Italian has been spoken by my brilliant niece
(d) rny book has been lost.
(d) Italian is being spoken by my brilliant niece.
8. Some people were helping the wounded woman. 18. I was obliged to go.
(a) The wounded woman was helped by some people? (a) Circumstances obliges me to go.
(b) The wounded woman was being helped by some people. (b) Circumstances oblige me to go.
(c) The wounded woman is being helped by some people. (c) Ctrcumstances obliged me to go.
(d) The wounded is helped by some people. (d) Circumstances oblige I should go.
9. They fund schools for girls. 19. Who wrote it?
(a) Schools for girls will be funded by them. (a) By who might it be written?
(b) Schools for girls are funded by them. (b) By whom was it written?
(c) Schools for girls was funded by them. (c) By who would it be written?
(d) Schools for girls are being funded by them. (d) By who will it be written?

Downloaded From : www.EasyEngineering.net


Downloaded From : www.EasyEngineering.net

Active & Passive Voice  l 33

20. The boy did not break the glass. 28. They drew a circle in the morning.
(a) The glass was not broken by the boy. (a) A circle has been drawing since morning.
(b) The glass is not broken by the boy. (b) In the morning a circle have been drawn by them.
(c) The glass had not been broken by the boy. (c) A circle was being drawn by them in the morning.
(d) The glass has not been broken by the boy. (d) A circle was drawn by them in the morning
21. Our task had been completed before sunset. 29. We must respect the elders.
(a) We complete our task before sunset. (a) The elders deserve respect from us.
(b) We had completed our task before sunset. (b) The elders must be respected.
(c) We completed our task before sunset. (c) The elders must be respected by all.
(d) We have completed our task before sunset. (d) Respect the elders we must.
22. The burglar destroyed several items in the room. Even the 30. Has anybody answered your question?
carpet has been torn. (a) Your question has been answered?
(a) Several items destroyed in the room by the burglar. (b) Anybody has answered your question?
Even the carpet he has torn. (c) Have you answered your question?
(b) The burglar, being destroyed several items in the room, (d) Has your question been answered?

wwalso carpet has torn.


(c) Including the carpet, several items in the room have
31. After driving professor Kumar to the museum she dropped
him at his hotel.

w.E
been torn by the burglar.
(d) Several items in the room were destroyed by the burglar.
He has even torn the carpet.
(a) After being driven to the museum, Professor Kumar
was dropped at his hotel.
(b) Professor Kumar was being driven dropped at his hotel.
23. The boys were playing cricket.
asy
(a) Cricket has been played by the boys.
(c) After she had driven Professor Kumar to the museum
she had dropped him at his hotel.
(d) After she was driven Professor Kumar to the museum
(b) Cricket was played by the boys.
(c) Cricket was being played by the boys.
En 32.
she had dropped him at his hotel.
I remember my sister taking me to the museum.

24.
(d) Cricket had been played by the boys.
The government has launched a massive tribal welfare
gin (a) I remember I was taken to the museum by my sister.
(b) I remember being taken to the museum by my sister.
programme in Jharkhand.
(a) The Government in Jharkhand has launched a massive
tribal welfare programme. eer
(c) I remember myself being taken to the museum by my
sister.

(b) Jharkhand government has launched a massive tribal


welfare programme.
33.
ing
(d) I remember taken to the museum by my sister.
Who is creating this mess?

(c) A massive tribal welfare programme is launched by the


government in Jharkhand. .ne
(a) Who has been created this mess?
(b) By whom has this mess been created?
(c) By whom this mess is being created?

25.
(d) A massive tribal welfare programme has been launched
by the government in Jharkhand.
We have warned you.
34.
(d) By whom is this mess being created?
They greet me cheerfully every morning.
(a) Every morning I was greeted cheerfully.
t
(a) You have been warned. (b) I am greeted cheerfully by them every morning.
(b) Warned you have been. (c) I am being greeted cheerfully by them every morning.
(c) We have you warned. (d) Cheerful greeting is done by them every morning to
(d) Have you been warned. me.
26. The boy laughed at the beggar. 35. Darjeeling grows tea.
(a) The beggar was laughed by the boy. (a) Tea is being grown in Darjeeling.
(b) The beggar was laughed at by the boy. (b) Let the tea be grown in Darjeeling.
(c) The beggar was being laughed by the boy. (c) Tea is grown in Darjeeling.
(d) The beggar was being laughed at by the boy. (d) Tea grows in Darjeeling.
27. They will demolish the entire block. 36. They have built a perfect dam across the river.
(a) The entire block will have to be demolished by them. (a) Across the river a perfect dam was built.
(b) The block may be demolished entirely. (b) A perfect dam has been built by them across the river.
(c) The entire block will be demolished by them. (c) A perfect dam should have been built by them.
(d) The entire block is being demolished. (d) Across the river was a perfect dam.

Downloaded From : www.EasyEngineering.net


Downloaded From : www.EasyEngineering.net

34  l  Active & Passive Voice

37. Do you imitate others? (c) He delivers the packet to your house.
(a) Are others being imitated by you? (d) The packet would be delivered to your house.
(b) Are others imitated by you? 44. I don’t know his whereabouts.
(c) Have others being imitated by you? (a) His whereabouts are not know to me.
(d) Were others being imitated by you? (b) No one knows his whereabouts.
38. You need to clean your shoes properly. (c) His whereabouts is not know to me.
(a) Your shoes are needed to clean properly. (d) His whereabouts had not been know to me.
(b) You are needed to clean your shoes properly. 45. Who can count the stars in the sky?
(c) Your shoes need to be cleaned properly. (a) By whom can the stars in the sky be counted?
(d) Your shoes are needed by you to clean properly. (b) By whom can be stars in the sky counted?
39. The invigilator was reading out the instructions. (c) By whom could the stars be counted in the sky?
(a) The instructions were read by the invigilator. (d) By whom could the stars in the sky be counted?
(b) The instructions were being read out by the invigilator.
46. His comment hurt me deeply.
(c) The instructions had been read out by the invigilator.
(a) I was hurt deeply by his comment.
(d) The instructions had been read by the invigilator.
40.
ww
The packet has not been opened by anyone.
(a) Anyone has not opened the packet.
(b) I could be hurt deeply by his comment.
(c) His comment was hurtful.
(d) I should not be hurt by his comment.

w.E
(b) No one has opened the packet.
(c) No one has not opened the packet.
(d) No one has been opening the packet.
47. He stole that watch.
(a) That watch was stolen by him.
(b) That watch stolen by him
41.
asy
Two motorcycle mechanics were arrested for their alleged
involvement in innumerable vehicle thefts.
(a) The police has arrested two motorcycle mechanics for
(c) That watch stole by him.
(d) That watch had stolen by him.

En
their alleged involvement in innumerable vehicle thefts.
(b) The police arrested two motorcycle mechanics for their
48. Sita was reading the Gita.
(a) The Gita was being read by Sita.
alleged involvement in innumerable vehicle thefts.
(c) For their alleged involvement in innumerable vehicle gin (b) The Gita were being read by Sita.
(c) Gita was being read by Sita.
thefts, police had arrested two motorcycle mechanics.
(d) For their alleged involvement in innumerable vehicle 49. eer
(d) he Gita was read by Sita.
Rahul will pass the message.

42.
thefts, two motorcycle mechanics have been arrested.
He was disgusted with the flattery of his assistant.
ing
(a) The message will passed by Rahul.
(b) The message would be passed by Rahul.
(a) The flattery of his assistant disgusted him.
(b) His assistant’s flattery had disgusted him.
(c) He was disgusted with his assistant for his flattery. .ne
C. The message will pass by Rahul.
D. The message will be passed by Rahul.

43.
(d) The flattery of this assistant will disgust him.
The packet will be delivered to your house.
(a) your packet is delivered to your house.
50. His hard work satisfied me.
(a) I was satisfied for his hard work.
(b) I was satisfied by his hard work.
(c) I was satisfied with his hard work.
t
(b) Someone will deliver the packet to your house. (d) I was satisfied at his hard work.

Downloaded From : www.EasyEngineering.net


Downloaded From : www.EasyEngineering.net

Active & Passive Voice  l 35

Hints & Solutions


LEVEL- I shold be chosen as the answer. Here also, we should not
1. (d) The garbage was first sundried for one to three days to miss any information supplied in the original sentence.
bring down the moisture level. 11. (b) It is known to us all that there is only one God.

2. (b) Women like to be flattered by men. 12. (b) He was elected Mayor by the people.

3. (a) What must be done, must be done properly. 13. (b) Let me be not laughed at.
14. (b) He was seen leaving the house by me.
4. (d) Let the poll results be looked at-is hope inspired by
them? 15. (d) The bull was pulled violently.
16. (a) I cannot wear this shirt any longer.
5. (c) You are supposed to make tea at eleven O’clock.
17. (a) Grass is not eaten by a lion, however hungry he may
6.
ww
(a) In Voice change, one can transform the sentence from
Active to Passive or Vice versa, but can’t change
the sense. Taht is, one can’t remove the information
be.
18. (b) He was seen picking up a gun by someone.

w.E
conveyed through the sentence or add any additional
information. That is the transformation must be done
19. (b) To resign was his obligation.
20. (d) Why was my proposal not agreed to by you?

asy
keeping the information intact. That is the reason behind
selecting (a) as the answer.
21. (a) The bell has been rung by the boy.
22. (a) He likes to be called Sir by people.
7.
En
(a) “People speak English” can be transformed into Passive
form thus-
23. (a) The money was added up and found to be correct.
24. (d) Someone has cut the telegraph wires.
“English is spoken by the people”
gin
25. (d) Will the truth be told us by her?
26. (c) 27. (c) 28. (d) 29. (a)
So, in Passive from, the sentence stands thus-
“English is spoken by the people all over the world.
30. (a)
eer
31. (d) 32. (a) 33. (d)

In this type of sentence, we need not use “by the people”.


For example
34. (c)
38. (b)
ing
35.
39.
(b)
(c)
36. (b)
40 (b)
37. (a)

People call him Mahatma. (Active)


.ne
41. (c) The sentence is in active voice and to convert it in
passive voice the predicate of the sentence becomes
the subject.

8.
He is called Mahatma. (Passive)
So the answer is (a).
(a) The Auxiliary Verb that should be used with you is 43. (a)
t
42. (d) The predicate of the sentence has become the subject
of a sentence while converting it into passive voice.

were and not was. Again, as the given sentence is 44. (d) For converting this sentence into passive voice and to
an Interrogative sentence, in Passive form also, the give it a logical meaning the phrase ‘the authority’ has
sentence will be Interrogative, that is inn Verb + Subject to be added.
form. Keeping these two conditions in mind, if we judge 45. (d) The sentence is in active voice with past continuous
tense in it. When it is converted into passive voice, it
the given options we find option (a) correct.
still remains in past continuous tense but with a little
9. (b) The transformation follows the simple rules related to variance due to its grammatical rule. In the sentence
the transformation of an Assertive Sentence. ‘was driving the car’ gets converted into ‘was being
driven’ in case of passive voice.
10. (b) The given sentence is in Passive form. Here subject is
“the shops” and object is “Men, women and children”. LEVEL- II
So, a sentence where “the shops” is used as the object 1. (c) 2. (d) 3. (a) 4. (c)
5. (d) 6. (d) 7. (d) 8. (b)
and “men, women and children” is used as the Subject
9. (b) 10. (c) 11. (b) 12. (a)

Downloaded From : www.EasyEngineering.net


Downloaded From : www.EasyEngineering.net

36  l  Active & Passive Voice

13. (c) 14. (b) 15. (c) 16. (d) 47. (a) Note : Rule to make active voice of Simple Past Tense
17. (d) 18. (c) 19. (b) 20. (a) 48. (a) Note: Assume if we had “Someone was reading the Gita”,
21. (b) 22. (d) 23. (c) 24. (d) then active voice would be,
25. (a) 26. (b) 27. (c) 28. (d) The Gita was being read.
29. (b) 30. (d) 31. (a) 32. (b)
49. (d) Note : Rule to make active voice of Simple Future Tense :
33. (d) 34. (b) 35. (c) 36. (b)
Passive subject + shall/will + be + third form of verb + by
37. (b) 38. (c) 39. (b) 40. (b)
+ passive object
41. (b) 42. (a) 43. (b) 44. (a)
50. (c) Note: We use “with” preposition with word “satisfied”
45. (a) 46. (a)

ww
w.E
asy
En
gin
eer
ing
.ne
t

Downloaded From : www.EasyEngineering.net


Downloaded From : www.EasyEngineering.net

3
PTER
CHA

Direct & Indirect Speech

ww
DIRECT SPEECH
Quoting the exact words of the speaker is called “The Direct Speech”.
Dhruv said, “I am writing a letter now”.

w.E
INDIRECT SPEECH
Reporting of what a speaker said without quoting his exact words is called ‘Indirect Speech’.

asy
Dhruv said that he was writing a letter then.
Rule 1: The adverbs of nearness should be put into those of distance.
Direct Speech -
En
Indirect Speech
now
here
this
-
-
-
then
there
that gin
these
ago
-
-
those
before eer
thus
today
-
-
so
that day ing
tonight
yesterday
the previous day tomorrow
-
-
-
that night
the day before (or)
the next day (or) the following day .ne
last week
next week
Rule 2: Tenses
-
-
the week before
the week after t
If the reporting verb is in the Present or Future tense (e.g., say, will say) there is no change in the tense of the verb in the
Indirect speech.
• Aman says, “I eat a mango”. (Direct Speech)
• Aman says, that he eats a mango”. (Indirect Speech)
If Reporting Verb is in the Past Tense, the tense of the verbs in the reported speech or Indirect Speech must be generally changed.
1. Present Tense in the Direct becomes past tense.
• John said, “I write a letter”. (Direct Speech)
• John said that he wrote a letter. (Indirect Speech)
2. Past Tense in the direct becomes past perfect or remains unchanged.
• Angel said, “I bought a pen yesterday”. (Direct Speech)
• Angel said that she had bought a pen the day before. (Indirect Speech)
3. Present Continuous in the direct becomes past continuous.
• Julie said, “I am going to Church”. (Direct Speech)
• Julie said that she was going to Church. (Indirect Speech)

Downloaded From : www.EasyEngineering.net


Downloaded From : www.EasyEngineering.net

38  l  Direct & Indirect Speech

4. Past Continuous in the direct becomes past perfect continuous.


• Neel said, “I was playing cricket”. (Direct Speech)
• Neel said that he had been playing cricket. (Indirect Speech)
5. Present Perfect in the direct becomes past perfect.
• Kamal said, “I have done my home work”. (Direct Speech)
• Kamal said that he had done his home work. (Indirect Speech)
6. Present Perfect Continuous in the direct becomes past perfect continuous.
• He said, “I have been reading a novel”. (Direct Speech)
• He said that he had been reading a novel. (Indirect Speech)
7. ‘Will’ and ‘Shall’ are changed to ‘would’.
• He said, “I will go to London tomorrow”. (Direct Speech)
• He said that he would go to London the next day. (Indirect Speech)
8. may - might
can - could
must - had to (or) must
• Johnsi said, “I must go now”. (Direct Speech)

ww
• Johnsi said that she must (or) had to go then. (Indirect Speech)
Exception to the above rule:
If the direct speech contains the Universal Truth, the tense of the direct speech remains unchanged even if the reporting verb

w.E
is in the past.
• The teacher said, “The sun rises in the East”. (Direct Speech)
• The teacher said that the sun rises in the East. (Indirect Speech)
Rule 3: Statement or Assertive Sentence
asy
• Remove the quotation marks in the statement
• Use the conjunction ‘that’
• Change the reporting verb ‘say to’ into ‘tell’
• Change the reporting verb ‘said to’ into ‘told’ En
Note:
• He said that (correct) gin
• He told me that (correct)
• He told that (Incorrect)
1. “I will work hard to get first class” said Lazar (Direct Speech)
eer
Lazar said he would work hard to get first class. (Indirect Speech)
2. “You can do this work” said Nelson to Johnsi (Direct Speech) ing
Nelson told Johnsi that he could do that work. (Indirect Speech)
3. He says, “I am glad to be here this evening” (Direct Speech.)
.ne
He says that he is glad to be there that evening. (Indirect Speech)
4. “I’m going to the library now” said David (Direct Speech)
David said that he was going to the library then. (Indirect Speech)
Rule 4: Imperative Sentence (Order or Request)
t
• Remove the quotation mark in an Imperative sentence.
• Use ‘to’ if it is an affirmative sentence. (without don’t)
• Use ‘not to’ if the sentence begins without Don’t.
• Don’t use ‘that’
• Omit the word ‘please’. Use the word ‘request’ instead of ‘say’.
• If the direct speech contains a request or a command, the reporting verb (say, said) change to tell, request, order, command
etc., in its correct tense.
1. “Don’t talk in the class” said the teacher to the boys. (Direct Speech)
The teacher advised the boys not to talk in the class. (Indirect Speech)
2. “Please give me something to eat. I am hungry” the old man said to them. (Direct Speech)
The old man requested them to give him something to eat and said that he was hungry (Indirect Speech)
3. “Be careful” said he to her. (Direct Speech)
He ordered her to be careful. (Indirect Speech)
4. “Bring me a cup of tea” said Nelson to Andriya. (Direct Speech)
Nelson asked Andriya to bring him a cup of tea. (Indirect Speech)

Downloaded From : www.EasyEngineering.net


Downloaded From : www.EasyEngineering.net

Direct & Indirect Speech  l 39

Rule 5: Interrogative Sentence (Questions)


• Remove the quotation marks and question mark in the interrogative sentence.
• Use ‘if’ or ‘whether’ if the sentence inside the quotation marks begins with a helping verb (Auxiliary verb).
• Use the given interrogative word (what, when, where, why, who, whom, whose, which, now etc.) if it does not begin with
the helping verb.
• Don’t use ‘that’.
• Changing the reporting verb (say, said) into ‘ask’ or ‘enquire’ in its correct tense.
• Omit helping verb like ‘do, does, did’. But don’t omit them when they are with ‘not’.
1. “Won’t you help me to carry this box?” said I to my friend. (Direct Speech)
I asked my friend if he would not help me to carry that box. (Indirect Speech)
2. Mohan said to Stalin, “Why didn’t you attend the meeting yesterday”? (Direct Speech)
Mohan asked Stalin why he had not attended the meeting the day before. (Indirect Speech)
3. “How often do you go to the theatre?” said David to John. (Direct Speech)
David asked John how often he went to the theatre. (Indirect Speech)
4. Mohamed said to Sultan, “Do you like mangoes?” (Direct Speech)

ww
Mohamed asked Sultan if he liked mangoes. (Indirect Speech)
Rule 6: Exclamatory Sentence
• Change the exclamatory sentence into statement or Assertive.

w.E
• Remove the quotation marks and exclamatory mark.
• Use the conjunction ‘that’
• Omit the interjections such as Oh, O, Alas, how, what, hurrah.

asy
• Add the word ‘very’ to the adjective or adverb if necessary.
• If the verb is not given, use ‘Be’ form verb (is, was, are, were, am) in its correct tense according to the subject.

• Use ‘exclaim’ sorrowfully for sorrowful incidents.


En
• Change the reporting verb (say, said) to ‘exclaim joyfully’

1. “O, what a beautiful flower that is!” said she. (Direct Speech)

gin
She exclaimed joyfully that that was a very beautiful flower. (Indirect Speech)
2. “What a horrible sight!” we all exclaimed. (Direct Speech)
We all exclaimed that it was a very horrible sight. (Indirect Speech)
3. “Alas! I have broken my brother’s watch” said he. (Direct Speech) eer
4. “How beautiful she is!” said Boon. (Direct Speech)
Boon exclaimed joyfully that she was very beautiful. (Indirect Speech)
ing
He exclaimed sorrowfully that he had broken his brother’s watch. (Indirect Speech)

.ne
t

Downloaded From : www.EasyEngineering.net


Downloaded From : www.EasyEngineering.net

40  l  Direct & Indirect Speech

Practice Exercise
LEVEL-I
DIECTIONS (Qs. No. 1-35): In the following questions, a (c) He contemplated where shall he be that time the
sentence has been given in Direct/Indirect Speech. Out of the four following year.
alternatives suggested select the one which best expresses the same (d) He wondered where he would be that time the following
sentence in Indirect/Direct Speech year.
1. He said to her, “Are you coming to the party? 9. Rajesh said, “I bought a car yesterday.”
(a) He asked her whether she was coming to the party. (a) Rajesh said that I have bought a car the previous day.
(b) He told her if she was coming to the party. (b) Rajesh told that he had bought a car yesterday.
(c) He asked her if she will be coming to the party. (c) Rajesh said that he had bought a car the previous day.
(d) He asked her if she will be coming to the party. (d) Rajesh said that he bought a car the previous day.
2. The sage said, “God helps those who help themselves” 10. The employer said to the workman, “I cannot pay you higher

ww
(a) The sage said that God helps those who help themselves.
(b) The sage said that God helped those whose helped
themselves.
wages.”
(a) The employer told the workman that he could not be
paid higher wages.

w.E
(c) The sage said that God helps those who helped
themselves.
(d) The rage said God will help those who will help
themselves.
(b) The employer forbade the workman that he could not
pay him higher wages.
(c) The employer forbade the workman to pay higher
wages.
3. “Please don’t go away”, she said.
asy
(a) She said to please her and not go away.
(d) The employer warned the workman that he cannot pay
him higher wages.
(b) She told me not to go away.
(c) She begged that I not go away.
(d) She begged me not to go away. En 11. My cousin said, “My room-mate snored throughout the
night.”
(a) My cousin said that her room-mate had snored
4. He said, “I clean my teeth twice a day.”
(a) He said that he cleaned his teeth twice a day. gin throughout the night.
(b) My cousin told me that her room-mate snored
(b) He said that he cleans his teeth twice a day.
(c) He said that he used to clean his teeth twice a day.
(d) He said that he is used to cleaning his teeth twice a day. eer
throughout the right.
(c) My cousin complained to me that her room-mate is
snoring throughout the night.
5. He said to them, “ Don’t make a noise”.
(a) He told them that don’t make a noise.
12.
ing
(d) My cousin felt that her room-mate may be snoring
throughout the night.
He asked his teacher, “Need I read this chapter?”
(b) He told them not to make noise.
(c) He told them not to make a noise.
(d) He asked them not to make a noise. that chapter. .ne
(a) He asked his teacher whether there was a need to read

6. The teacher said, ‘Be quiet, boys.”


(a) The teacher said that the boys should be quiet.
(b) The teacher called the boys and ordered them to the
quiet.
chapter.
t
(b) He asked his teacher whether he needed to read this

(c) He asked his teacher if it was necessary to read this


chapter.
(c) The teacher urged the boys to be quiet. (d) He asked his teacher if he had to read that chapter.
(d) The teacher commanded the boys that they be quiet. 13. He said, “What a beautiful scene!”
7. My friend said to me, “Has your father returned from (a) He said that what a beautiful scene it was.
Calcutta?” (b) He wondered that it was a beautiful scene.
(a) My friend said to me that my father has returned form (c) He exclaimed what a beautiful scene it was.
Calcutta. (d) He exclaimed that it was a very beautiful scene.
(b) My friend asked me if my father had returned from 14. He said, “I saw a book here.”
Calcutta. (a) He said that he saw a book here.
(c) My friend told me that his father has returned from (b) He said that he saw a book there.
Calcutta. (c) He said that he had seen a book here.
(d) My friend enquired me if his father has returned from (d) He said that he had seen a book there.
Calcutta. 15. He said to me, “What time do the offices close?”
8. He said, “Where shall I be this time next year?” (a) He wanted to know what time the offices close.
(a) He asked that where should he be that time next year. (b) He asked me what time did the offices close.
(b) He wondered where he should be that time the following (c) He asked me what time the offices close .
year. (d) He asked me what time the offices did close.

Downloaded From : www.EasyEngineering.net


Downloaded From : www.EasyEngineering.net

Direct & Indirect Speech  l 41

16. Pinki said to Gaurav, “will you help me in my work just now” 26. “Are you alone, my son?” asked a soft voice close behind
(a) Pinki asked Gaurav if he would help her in her work me.
just then. (a) A soft voice from my back asked if I was alone.
(b) Pinki questioned to Gaurav that will you help me in my (b) A soft voice said to me are you alone son.
work just now. (c) A soft voice asked that what I was doing there alone.
(c) Pinki told Gaurav whether he will help her in her work (d) A soft voice behind me asked if I was alone.
just now. 27. He said, “I must go next week”.
(d) Pinki asked to Gaurav that will he help her in her work (a) He said that he must go next week.
just now. (b) He said that he must go the following week.
17. Sarita said to me, “I will do it now or never.” (c) He said that he would have to go the following week.
(a) Sarita told me that I would do it then or never. (d) He said that he was to go the following week.
(b) Sarita told me that she would do it now or never. 28. He said to her, “Don’t read so fast.”
(c) Sarita told me tht she will do that now or never. (a) He told her not to read so fast.
(d) Sarita told me that she would do it then or never.
(b) He advised her don’t read so fast.
18. She said to him, “Why don’t you go today?”
(c) He requested her not to read so fast.
(a) She said to him that why he don’t go today.
(d) He ordered her not to read so fast.
(b) She asked him if he was going that day.
29. “I don’t know the way. Do you?” he asked.”

ww
(c) She asked him why he did not go today.
(d) She asked him why he did not go that day.
19. Gavaskar said, “Bravo! Azhar, you have done well.”
(a) He said that he didn’t know the way and did I know it.
(b) He told that he was not knowing the way, but wondered

w.E
(a) Gavaskar exclaimed with joy that Azhar had done well.
(b) Gavaskar called Azhar and exclaimed that he had done
well.
if I knew.
(c) He said that he didn’t know the way and asked me if I
did.
(d) He asked me if I knew the way which he didn’t.
well.
asy
(c) Gavaskar congratulated Azhar, saying that he had done

(d) Gavaskar praised Azhar for his having done well.


30. He said, “Will you listen to such a man?”
(a) He asked them will you listen to such a man.
20. He said to me, “Where is the post office?”
(a) He wanted to know where the post office was.
(b) He asked me that where the post office was. En (b) He asked them are you listening to such a man.
(c) He asked them whether they would listen to such a man.
(d) He asked them whether they will listen to such a man.
(c) He asked me where the post office was.
(d) He asked me where was the post office. gin
31. Mary said to Robert, “Let him come, then we shall see.”
(a) Mary said to Robert that if he came, they would see
21. He said, “The mice will play, when the cat is away.”
(a) He said that the mice will play when the cat is away.
(b) He said that the mice would play when the cat was away.
him.
eer
(b) Mary told Robert that they shall see him if he came.
(c) Mary told Robert that once he came, they would see
(c) He said that the mice would play when the cat would
be away.
(d) He said that the mice shall play, when the cat is away.
him.
ing
(d) Mary told Robert that they would see him if he might
22. He said to his servant, “Why are you so lazy today?”
(a) He asked his servant why he was so lazy that day.
come.

.ne
32. He said, “I have often told you not to waste your time.”
(a) He said that he had often told not to waste your time.
(b) He asked his servant why he had been so lazy that day.
(c) He asked his servant why he was being so lazy that day.
(d) He asked his servant why was he so lazy that day.
23. He said, “Can you sing?” And I said, “No”.
t
(b) He said that he had often told him not to waste his time.
(c) He said that he had often suggested to him not to waste
him time.
(d) He told that he has often told him not to waste his time.
(a) He asked me that could I sing and I refused.
(b) He asked me if I could sing and I said that I couldn’t. 33. Rajan said, “O that I were a child again !”
(c) I denied, when he asked me if I could sing. (a) Rajan exclaimed with wonder that he was a child again.
(d) He asked me if I could sing and I said no. (b) Rajan wondered that were be a child again.
24. He said to her, “May you succeed!” (c) Rajan strongly wished that he had been a child again.
(a) He told her that she might succeed. (d) Rajan prayed that he were a child again
(b) He prayed to God that she may succeed. 34. I said to my brother, “Let us go to some hill station for a
(c) He wished her success. change.”
(d) He said to her that she might succeed. (a) I asked my brother to go to some hill station for a
25. He said, “May God grant peace to the departed soul” change.
(a) He wished by God to grant peace to the departed soul. (b) I asked my brother if he would go to some hill station
(b) He wished that God may grant peace to the departed soul. for a change.
(c) He prayed that might God grant peace to the departed (c) I permitted my brother to go to some hill station for a
soul. change.
(d) He prayed that God would grant peace to the departed (d) I suggested to my brother that we should go to some
soul. hill station for a change.

Downloaded From : www.EasyEngineering.net


Downloaded From : www.EasyEngineering.net

42  l  Direct & Indirect Speech

35. The Manager said, “Well, what can I do for you? 38. “Who now,” they had asked, “will listen to our troubles and
(a) The Manager asked what he could do for him. protect us from the crocodiles ?”
(b) The Manager woundered what he could do for him. (a) They had wanted to know who will now listen to their
(c) The Manager wanted to know what he could do for him. troubles and protect them from the crocodiles.
(d) The Manager said that he couldn’t do any for him. (b) They wanted to know who will listen to their troubles
DIRECTIONS (Qs. 36 - 40): a sentence has been given in Direct and protect them from the crocodiles.
/Indirect. Out of the fouralternatives suggested, select the one (c) They had wanted to know who would listen to their
which best expresses the same sentence in Indirect/Direct and troubles and protect them from the crocodiles.
mark your answer in the Answer Sheet. [SSC CHSL 2013] (d) They had wanted to know who then would listen to
36. He said to me, “I grew these carrots myself.” their troubles and protect them from the crocodiles.
(a) He told me that he had grown those carrots himself. 39. Moti asked Gangu whether the latter was in his senses.
(b) He told me that he grew those carrots himself. (a) “Gangu, are you in your senses ?” asked Moti.
(c) He told me I grew these carrots myself.
(b) “Are you senseless, Gangu ?” asked Moti.
(d) He told me that he grew these carrots himself.
37. Mrs. Shankar said, “I know what it is to be depressed. (c) “Hey Gangu, are you in your senses now ?” asked Moti.
(a) Mrs. Shankar is depressed she said. (d) “Gangu, have you lost your senses ?” asked Moti.
40. He said that we are all born to die.

ww
(b) Mrs. Shankar said that she was knowing what it was
to be depressed.
(c) Mrs. Shankar said that she knew what it was to be
(a) He said, “We are all born to die.”
(b) He said, “We have all been born to die.”

w.E
depressed.
(d) Mrs. Shankar knows what it is to be depressed.
(c) He exclaimed, “We were all born to die.”
(d) He said, “We were all born to die.”

asy LEVEL-II

En
DIRECTIONS: (Qs. 1-40): In the following questions, a sentence
has been given in Direct/Indirect. Out of the four alternatives
suggested, select the one which best expresses the same sentence in
6. I said, ‘Water is essential for life.’
(a) Water is essential for life was said by me.
(b) I told that water was essential for life.
Direct/Indirect and mark your answer in the Answer Sheet.
1. She said to the children, ‘You mustn’t play with fire.’ gin(c) I exclaimed that water was essential for life.
(d) I said that water is essential for life.
(a) She told the children they mustn’t play with ‘ fire.
(b) She told the children they are not to play with fire.
(c) She told the children they were not to play with fire. eer
7. Tania said to her friend, ‘Can you lend me an umbrella?’
(a) Tania asked her friend to give her an umbrella.
(d) She told the children not to play with fire.
2. He bade his friends goodbye. ing
(b) Tania requested her friend to lend her an umbrella
(c) Please give me an umbrella Tania requested her friend.
(d) Will you lend me your umbrella, Tania asked her friend.
(a) ‘I am bidding you Goodbye’
(b) ‘Goodbye, my friends’, he was saying to them.
(c) ‘I will see you later’, he told his friends. .ne
8. He said, ‘If I had the tools I could mend the car.’
(a) He said that if he had the tools he could mend the car.
(d) He said, ‘Goodbye, my friends.’
3. He exclaimed with joy that their team had won the
tournament.
(c) He told he could mend the car. t
(b) He tells that he had the tools he could mend the car.

(d) He said I could mend the car if I have the tools.


(a) He exclaimed, ‘Hurrah ! Our team has won the 9. He said, ‘I was doing it.’
tournament !’ (a) He said that he did it.
(b) He said, ‘Our team has won the tournament.’ (b) He said that he has done it.
(c) He said, ‘Wow ! Our team won the tournament.’ (c) He said that he had done it.
(d) He said, ‘Our team won the tournament.’ (d) He said that he had been doing it.
4. Iba said that she might go home the next day with her sister. 10. Daman says, ‘Priya is working in Ahmedabad.’
(a) Iba said, ‘I can go home tomorrow with my sister.’ (a) Daman says that Priya is working in Ahmedabad.
(b) Iba said, ‘I may just go home tomorrow with my sister.’ (b) Daman say that Priya was working in Ahmedabad.
(c) Iba said, ‘I could go home tomorrow with my sister.’ (c) Daman said that Priya was working in Ahmedabad.
(d) Iba said, ‘I will go home tomorrow with my sister.’ (d) Daman said that Priya works in Ahmedabad.
5. She asked me if I was going to college. 11. I said to him, ‘Will you go to Delhi?’
(a) She asked to me, ‘Was I going to college?’ (a) I asked him will he go to Delhi.
(b) She asked me, ‘Will you go to college?’ (b) I asked him if he would go to Delhi.
(c) Said, ‘Am I going to college?’ (c) I said to him would you go to Delhi.
(d) She said to me. ‘Are you going to college? (d) I said to him would he go to Delhi.

Downloaded From : www.EasyEngineering.net


Downloaded From : www.EasyEngineering.net

Direct & Indirect Speech  l 43

12. You will say, “I am ill.’ 22. Sita said. ‘I may go there.”
(a) You would say that you are ill. (a) Sita said that she might go there.
(b) You would say that you were ill. (b) Sita says that she may go there.
(c) You will say that you are ill. (c) Sita said that she will go there.
(d) You will say that you were ill. (d) Sita says that she is going there.
13. The old man said that he would soon be leaving the world. 23. His father said that it was time he visited his uncle.
(a) The old man said, ‘I am going to leave the world soon.’ (a) It is time you visit your uncle’, said his father.
(b) The old man said, ‘I shall soon be leaving the world.’ (b) ‘It was time you visited your uncle’, said his father.
(c) The old man said, ‘I would soon leave the world.’ (c) It is time you visited my uncle’, said his father
(d) The old man said,’ I am soon leaving the world.’ (d) ‘It is time you visited your uncle’, said his father.
14. My friend requested me to bring him a sandwich. 24. The prisoner enquired if a lawyer had been — arranged for him.
(a) Please bring me a sandwich’, said my friend. (a) The prisoner enquired, ‘Had a lawyer been arranged for
(b) He said, ‘My friend, please bring me a sandwich.’ me?’
(c) Please bring my friend a sandwich.’, said he. (b) The prisoner asked, ‘Has a lawyer been arranged for me?’
(d) My friend said, ‘Will you bring me a sandwich.’ (c) The prisoner enquired, ‘Is there a lawyer for me?’
15. Anne said, ‘It is time to leave for the meeting.’ (d) The prisoner said, ‘Have you arranged a lawyer for me?’
(a) Anne said that it was time to leave for the meeting.

ww
(b) Anne said its time to leave for the meeting.
(c) Anne told that it is time to leave for the meeting.
25. He says that he is very sorry.
(a) He said. ‘He is very sorry’
(b) He told me. ‘I felt sorry’

w.E
(d) Anne said that it was time for the meeting.
16. He said that he intended to leave for Surat that night.
(a) ‘I intend to leave for Surat that night’, he said.
(b) ‘I intend to left for Surat that night’, he said.
(c) He says, ‘I am very sorry’
(d) He said. ‘He was very sorry’
26. Everybody’ says, ‘How well she sings !’

asy
(c) ‘I intended to leave for Surat tonight’, he said.
(d) ‘I intend to leave for Surat tonight”, he said.
(a) Everybody comments that she sings well.
(b) Everybody says that she sang well.
(c) Everybody says how well she sings.
17. He said, ‘I have read this novel.’
(a) He said that he had read that novel.
(b) He said that he had read this novel. En (d) Everybody tells that she sang very well.
27. The lady asked me how my uncle was.

(c) He said that he read that novel.


(d) He said that he has read this novel. gin(a) The lady said to me, ‘How is your uncle?’
(b) The lady said to me, ‘How was your uncle?
(c) The lady asked me, ‘How is your uncle doing?’
18. Phila ordered her brother to turn down the television and let
her work in peace.
(a) Phila said to her brother, ‘Turned down the television eer
(d) The lady asked me, ‘How has your uncle been?’
28. He said, “Be quiet and listen to my words.”
and let me do my work in peace.’
(b) Phila said to her brother, ‘Turn down the television to ing
(a) He urged them to be quiet and listen to his words
(b) He urged them and said be quiet and listen to words
(c) He said they should be quiet and listen to his words
let me do my work in peace.’
(c) Phila said to her brother, ‘Turn down the television that
let me do my work in peace.’ .ne
(d) He said you should be quiet and listen to my words
29. He said to me, “I have often told you not to play with fire.”
(d) Phila said to her brother, ‘Turn down the television and
let me work in peace.’
19. He said, ‘Honesty is the best policy.’
(a) He said that honesty has been the best policy.
with fire
t
(a) He said that he has often been telling me not to play

(b) He told me that he had often told me not to play with


fire
(c) He remained me that he had often told me not to play
(b) He said that honesty is the best policy.
with fire
(c) He said that honesty had been the best policy.
(d) He said to me that he often told me not to play with fire
(d) He said that honesty was the best policy.
30. The Captain said to his men, “Stand at ease.”
20. The teacher said to him, ‘Is not poverty a curse?’
(a) The Captain urged his men to stand at ease
(a) The teacher asked him was poverty not a curse.
(b) The teacher asked him whether poverty is not a curse. (b) The Captain wanted his men to stand at ease
(c) The teacher asked him is poverty not a curse (c) The Captain told his men that they should stand at ease
(d) The teacher asked him if poverty was not a curse (d) The Captain commanded his men to stand at ease
21. ‘Come home early’, said Mr Mathew to his clerk. 31. The poor examine said, “O God, take pity on me.”
(a) Mr Mathew asked his clerk to be home early. (a) The poor examinee prayed God to take pity on him
(b) Mr Mathew told his clerk to come home as early as (b) The poor examinee, invoking God, improve him to take
possible. pity on him
(c) Mr Mathew requested his clerk to come home a little (c) The poor examinee exclaimed that God take pity on
early. him
(d) Mr Mathew ordered his clerk to come home early. (d) The poor examine asked God to take pity on him

Downloaded From : www.EasyEngineering.net


Downloaded From : www.EasyEngineering.net

44  l  Direct & Indirect Speech

32. Ram says to me “You are smart.” 37. I said to him,”who teaches you English?”
(a) Ram tells me that I am smart. (a) I asked him who was taught him English
(b) Ram tells me that he is smart. (b) I asked him who had taught him English
(c) Ram tells me that he was smart.
(c) I asked him who taught me English
(d) Ram tells me that I shall be smart.
33. He said, ”May God Grant her peace!” (d) I asked him who taught him English
(a) He prayed that God might grant her peace 38. I said to him, “Why are you working so hard”?
(b) He ordered that God might grant her peace (a) I asked him why was he working so hard
(c) He prayed that God may grant me peace. (b) I asked him why he had been working so hard
(d) He told that God may grant me peace. (c) I asked him why he was working so hard
34. Mohan said to me, “Do not tell a lie.”
(d) I asked him why had he been working so hard
(a) Mohan forbade me not to tell a lie.
(b) Mohan advised me not to tell a lie. 39. Manna asked Rohan, “Have you sat in a trolley bus before?”
(c) Mohan ordered me to tell a lie. (a) Manna asked Rohan whether he had sat in a trolley bus
(d) Mohan advised me to tell a lie. earlier.
35. She said, ”Good bye Ramesh”. (b) Manna asked Rohan had he sat in a trolley bus before.
(a) She bade Ramesh good bye. (c) Manna asked Rohan if he sat on a trolley bus before.

ww
(b) She bade to Ramesh good bye.
(c) She bids Ramesh good bye.
(d) She had bade Ramesh good bye.
(d) Manna asked Rohan if he has ever sat in a trolley bus
40. He said to the interviewer, “Could you please repeat the

w.E
36. The student said, “Alas! I wasted my time last year.”
(a) The student exclaimed with surprise that he had wasted
his time the previous year.
(b) The student exclaimed with sorrow that he had wasted
question?”
(a) He requested the interviewer if he could please repeat
the question

his time the previous year.


asy
(c) The student exclaimed with joy that he had wasted his
(b) He requested the interviewer to repeat the question
(c) He requested the interviewer to please repeat the
question
time the previous year.
(d) The student exclaims with sorrow that he had wasted
his time the previous year. En (d) He requested the interviewer the interviewer if he could
repeat the question

Hints & Solutions gin


eer
LEVEL- I 39.
ing
(a) While converting a sentence from indirect speech to
direct speech, certain changes have to be made as we
1.
6.
(a)
(c)
2. (a)
7. (b)
3. (d)
8. (d)
4. (b)
9. (d)
5. (d)
10. (b) .ne
use inverted commas to mark off the exact words of the
speaker. And the speech is addressed in first person and
11.
16.
21.
26.
(a)
(a)
(a)
(d)
12. (d)
17. (d)
22. (a)
27. (c)
13. (d)
18. (d)
23. (b)
28. (a)
14. (d)
19. (c)
24. (c)
29. (c)
15. (c)
20. (c)
25. (d)
30. (c)
40.
\
second person mode.
(a) Omit ‘that’ from the direct speech. t
31. (d) 32. (b) 33. (c) 34. (d) 35. (c) LEVEL- II
36. (a) When the sentence is converted from direct speech to
indirect speech, the pronoun ‘I’ is converted to ‘he’ and 1. (a) 2. (d) 3. (a) 4. (b) 5. (d)
the simple past tense gets itself converted into past perfect
tense. 6. (d) 7. (b) 8. (a) 9. (d) 10. (a)
37. (c) The pronoun ‘I’ of the sentence is converted to ‘she’ and 11. (b) 12. (b) 13. (b) 14. (a) 15. (a)
the direct speech which is in present tense gets converted 16. (d) 17. (a) 18. (d) 19. (b) 20. (d)
into past tense. 21. (d) 22. (a) 23. (d) 24. (b) 25. (c)
38. (d) While converting a direct speech into indirect speech the
26. (c) 27. (a) 28. (a) 29. (b) 30. (d)
adverb ‘now’ is changed to ‘then’ and the tense of the
sentence also changes with respect to the time period of 31. (a) 32. (a) 33. (a) 34. (b) 35. (a)
the speech. 36. (b) 37. (d) 38. (c) 39. (a) 40. (d)

Downloaded From : www.EasyEngineering.net


Downloaded From : www.EasyEngineering.net

4
PTER
CHA

Clause Analysis

INTRODUCTION

ww
A Clause is a part of a sentence having its own subject and predicate. Clause Analysis is the complete course in mastering the art of
breaking up simple or complex or compound sentences into their different components for better understanding. It contains various
rules for analysing different kinds of clauses with several examples.

w.E
The exact nature of each clause should be figured out from the kind of function it serves in the sentence. A COMPLEX sentence is
made up of a MAIN CLAUSE and a SUBORDINATE CLAUSE, which starts with a subordinator. The main clause is the one that can
stand on its own. The subordinate clause depends upon the main clause and starts with a subordinator.
• Simple (consisting of one clause)

asy
• Compound (consisting of two co-ordinate main clauses)
• Complex (containing at least one subordinate clause)
Simple: I was reading the newspaper.
En
Compound: I read the newspaper, but nothing caught my interest.
Complex: I was reading the newspaper that I subscribe to.
I was reading the newspaper while I was having breakfast.
I was reading the newspaper while having breakfast. gin
The structure of a complex sentence (main clause)
{-------Matrix clause--------}{-------------Subordinate clause-------------------} eer
I read in the newspaper that the president is facing further criticism.
Matrix clause = main clause minus subordinate clause.
ing
The structure of a compound sentence
{-------Main clause------} C {-------main clause-------------}
I read the newspaper, but nothing caught my interest. .ne
C = Connector

TYPES OF CLAUSES
t
There are different types of Clauses. Some of them are as follows:

A. INDEPENDENT CLAUSE
An independent clause is a clause that can stand alone as a sentence (i.e., it expresses a complete thought). An independent clause,
like all clauses, has a subject and verb.
When there is no dependent clause in the same sentence and only independent clauses, the independent clause is a simple sentence.
For example:
l I like coconut macaroons.

  (This is an independent clause and simple sentence)


l I like coconut macaroons even though I dislike coconut.

(This is an independent clause and a dependent clause in this sentence. This is a complexsentence)
Independent Clauses have three components :
1. They have a subject - they tell the reader what the sentence is about.
2. They have an action or predicate - they tell the reader what the subject is doing.
3. They express a complete thought - something happened or was said.

Downloaded From : www.EasyEngineering.net


Downloaded From : www.EasyEngineering.net

46  l  Clause Analysis

n independent clause can be simply formed with a subject and a verb:


A
• Jim reads.
Jim is the subject. Reads is the action or verb. A complete thought was expressed--something was said, and the reader now knows
that Jim likes to read.
Independent clauses can also be joined to other independent clauses, if the independent clauses are related. However, they MUST be
joined using the proper punctuation.
• Jim read a book; he really enjoyed the book.
The first clause is an independent clause. Jim is the subject, read is the action, book is the object.
The second clause is an independent clause. He is the subject, enjoyed is the action and the book is the object.
The independent clauses are related, so they can be joined to create a complex sentence. They are correctly joined by a semicolon.
Independent clause can be connected in a variety of ways
1. By a comma and little conjunction (and, but, or, nor, for, yet, and sometimes so.)
2. By a semicolon, by itself.
3. By a semicolon accompanied by a conjunctive adverb (such as however, moreover, nevertheless as a result, consequently etc.)
4. And of course, independent clause are often not connected by punctuation at all but are separated by a period.
Independent clauses can be quite complex, but the important thing to remember is that they stand on their own and make sense
alone.
ww TYPES OF INDEPENDENT CLAUSE

Meaning

w.E
To add an Idea
Coordinators

And
Conjunctive Adverb

in addition, furthermore,
moreover, also
Examples

Women follow more healthful diets, and they go to doctors


more often.
To show time or
sequence asy
first, second, then, next, later, First, robots can perform repetitive tasks without becoming
meanwhile, finally tired or bored. Second, they can fulfill tasks requiring

To contrast But, yet En


however, nevertheless, in
pinpoint accuracy.
Increasing the size of airports is one solution to traffic
contrast, on the other hand
gin
congestion; however, this is a long-term solution whose
benefits may not be seen for many years into the future.
To show result So
eer
therefore, thus, consequently, Native and non-native English speakers have different needs;
as a result therefore, most schools provide separate classes for each

To introduce an
alternative
Or otherwise
group.

ing
Students must take final exams; otherwise, they receive a
grade of incomplete.
To emphasise in fact, of course, indeed,
certainly .ne
The little girl hated spiders; in fact, she was terrified by them.

To provide an
example
To generalise or
for example, for instance

in general, overall, in short


In the operating room, robotic equipment can assist the
surgeon. For instance, a robot can kill a braintumor.
Hermes is not only the messenger of Zeus, but the patron
t
summarise of thieves. In general, he is the god of authorised and
unauthorised transfers.

B. DEPENDENT CLAUSE
A dependent clause is a clause that does not express a complete thought.
A clause can be dependent because of the presence of:
• Marker Word : (Before, after, because, since, in order to, although, though, whenever, wherever, whether, while, even though,
even if)
• Conjunction (And, or, nor, but, yet)
Dependent clause can be identified and classified according to their role in the sentence.
Steps to identify and analyse the clause:
 Identify the separate subordinator and the finite verb.
 Separate the main clause and subordinate clause.
(Start with the subordinator and go on till the end until you come across another finite verb).

Downloaded From : www.EasyEngineering.net


Downloaded From : www.EasyEngineering.net

Clause Analysis  l 47

 ask questions to the main clause so that the answer is the subordinate clause
What – noun
Which – adjective
When, where, how – adverb
C. NOUN CLAUSE
Noun clauses name a person, place, thing or idea. Since it acts as a noun, it can be a subject, object, a subject complement, an object
complement or an appositive.
 A noun clause does the function of noun in the sentence.
 Asking the question “what” to the main clause can identify it.
 It can also substitute with the pronoun.
 The noun clause may either be in the subject or object position in a sentence.
Example: I know that the students have gone for a picnic.
I know – Main clause
that the students have gone for a picnic – Subordinate Noun Clause
Other examples of Noun Clauses are:

ww
• Why she said that
• Whomever you like
• How they would get there

w.E
• Who let the cat out of the bag
• What she anticipated
• Whatever makes you happy
• That you are listening
• Whether he can drive that far
• If the dress is on sale
asy
• Whoever shows up on time
En
D. ADVERB CLAUSE

gin
The adverb clause tends to tell us something about the sentence’s main verb, when, why, under what conditions. It modifies verbs and
begins with subordinating conjunctions. Adjectives can often be changed into adverbs if "-ly" is added to them. Adverbs can be found
on all three levels.
Word level: I'm leaving later. (When am I leaving?) eer
Phrase level: Put the book on the table. (Where should you put it?)
Clause level: Because she felt sick. Betty went home. (Why did she go home?)
Adverbs can modify verbs: Bill felt asleep quickly. (How did he fall asleep?) ing
Adverbs also modify adjectives: Our cat has bright green eyes. (What degree of green?)
They modify other adverbs, as well: I cannot run very quickly. (How quickly?) .ne
Note : Qualifiers such as very, often, always, not, and never are adverbs
Examples:
• When the President arrives
t
• Because I can’t wait for the bus
• As if he knew what was going to happen
• Than his sister can
• If you can work on Sundays
• Until the sun sets
• While flowers continue to bloom
• Whenever you come to visit
• Since I don’t have enough money
• Although I had never considered it
• Unless you have the right size
• As the lights were dimming
• No matter how you look at it
• How he got elected
• Before the food gets cold
• Supposing that she really wanted to go

Downloaded From : www.EasyEngineering.net


Downloaded From : www.EasyEngineering.net

48  l  Clause Analysis

E. ADJECTIVE CLAUSE
Adjective clauses modify nouns and usually begin with a relative pronoun and sometimes with a subordinating conjunction.They
function as "modifiers" (change agents) in a sentence and can be labeled on all three levels.
Word level: Tommy pulled the red wagon down the street. (What kind of wagon?)
Phrase level: A man with a beard came into the room. (Which man?)
Clause level: All students whose cars are illegally parked will be ticketed. (Which students?)
Some examples of dependent clauses that are adjective clauses are:
• That I sold him
• Which is located in Italy
• Who is intelligent
• Whom we met after the movie
• Whose writing is always intriguing
• When the leaves turn colours and fall
• Where I went to elementary school
• Why the movie was a flop
• That was a bestseller

ww
Who live by the ocean
Punctuation Marking

w.E
With noun clauses, no commas are used. Adjective restrictive clauses are not separated by commas, but with adjective descriptive
clauses commas are used. Adverb clauses that come before the independent clause are followed by a comma, but if they come after
the independent clause, no comma is used.

Types of clauses asy


Subordinators Examples
1. Noun clauses
which, whose, whether, that, if
En
what, where, why, how, where, when, who whom, He knows that his business will be successful.
That there is a hole in the ozone layer of the earth’s
atmosphere is well known.
2. Adjective
clauses
who, whom, which, whose, that, where, when
gin
Men who are not married are called bachelors.
Last year we traveled to San Francisco, which is famous

3. Adverb clauses a. time when, before, after, until, since, eer


for its architecture.
When he won the money, he decided to buy a car.

b. place
as soon as
where, wherever ing
She drove wherever she wanted.
c. cause
d. purpose
because, as, since
so that, in order that .ne
She got a parking ticket because she parked illegally.
He drove fast so that he could get to work on time.
e. result
f. condition

g. concession
so ... that, such ... that
if, unless

although, even though


He drove so fast that he got a speeding ticket.
t
If she hadn’t won the lottery, she would have been very
unhappy.
Although she thought she was a good driver, she got a lot
of tickets for speeding.

Downloaded From : www.EasyEngineering.net


Downloaded From : www.EasyEngineering.net

Clause Analysis  l 49

Practice Exercise
LEVEL-I
DIRECTIONS (Qs. 1-10) : In the followiong questions, combine DIRECTIONS (Qs. 23-27) : Choose the correct adjective clause
each of the following pairs of simple sentences into one complex subordinators.
sentence containing an adjective clause 23. After a month, we finally got to India, where my dad worked/
1. The theft was committed last night. The police has caught works/working.
the man. 24. My friend Seema, who/whom/whose car is black, is nineteen
2. The French language is different from the Latin language. years old.
25. My mom would like to sky in Alaska in which/where/that is
Latin was once spoken throughout Europe.
cold must part of the year.
3. You are looking upset. Can you tell me the reason?
26. She has two cats whose/which/that names are kitty and
4. He had several plans for making money quickly. All of them chococat.

ww
have failed.
5. The landlord was proud of his strength. He despised the
weakness of his tenants.
27. I have a house which/where/in which is in a Maharashtra.
DIRECTIONS (Qs. 28-37): Combine each set of simple sentence
into one complex sentence containing an adverb clause.

w.E
6. This is the village. I was born here.
7. You put the keys somewhere. Show me the place.
28. I waited for my friend. I waited till he arrived.
29. He hid somewhere. His pursuers could not find him.
30. You are intelligent. I am intelligent.

asy
8. Paul was an old gentleman. He was my travelling companion.
9. A fox once met a crane. The fox had never seen a crane
before.
31. He was not there. I left a message with his mother.
32. We wish to live. We eat for that purpose.

En
10. The shop keeper keeps his money in a wooden case. This is
the wooden case.
33. She was very tired. She could barely stand.
34. Don’t eat too much. You may fall ill.

DIRECTIONS (Qs. 11-22) : Find out the noun clauses in the


following sentences and state what purpose they serve. gin
35. He started early. He finished late.
36. You must tell me everything. Otherwise I will not be able to
help you.
11.
12.
The king ordered that the traitor should be put to death.
He said that he would not go. eer
37. I will get ready. Do not go till then.
DIRECTIONS (Qs. 38-45): Find the adverb clauses in these
13.
14.
That he is not interested in the offer is known to us.
He said that he was not feeling well. ing
sentences. What are their meanings (time, place...)? If it is a
reduced adverb clause, add the missing words.
15.
16.
I cannot rely on what he says.
I don’t know where he has gone. .ne
38. You seem very happy when you help other people.
39. While you wait, we will detail your car.
17.
18.
19.
20.
He asked whether the servant had polished his shoes.
The news that he is alive has been confirmed.
The belief that the soul is immortal is almost universal.
It is certain that we will have to admit defeat.
40. I am happier than I ever was before.
41. That horse is more obstinate than a mule.
t
42. The woman took notes while being taught to cook with
broccoli.
43. Ben fields baseballs better than he hits.
21. It was fortunate that he was present. 44. As the lions approached the carcass, the cheetahs retreated
22. The report that only ten persons were killed in the riots is once more.
not true. 45. While eating, I choked on a bone.

Downloaded From : www.EasyEngineering.net


Downloaded From : www.EasyEngineering.net

50  l  Clause Analysis

LEVEL-II
DIRECTIONS: (Qs. 1-23) : Choose the right kind of clauses from 13. Do you know which is house is.
the following. (a) Adverb clause
1. They told us why they are doing that. (b) Adjunct clause (relative clause)
(a) Noun clause (b) Adjective clause (c) Noun clause
(c) Adverb clause (d) wh-clause (d) Relative clause
2. They left early. 14. I know that he will come.
(a) Noun clause (b) Declarative clause (a) Noun clause
(c) Adjective clause (d) Adverb clause (b) Adverb clause
3. THe time when they left early (b) Adjunct clause (relative clause)
(a) Noun clause (b) Relative clause (d) Relative clause
(c) Adjective clause (d) Adverb clause 15. We come here thatwe may study.
4. THe claim that he expressed (a) Noun clause
(a) Adjunct clause (relative clause) (b) Adverb clause
(b) Relative clause
(b) Adjunct clause (relative clause)

ww
(c) Adjective clause
(d) Adverb clause
5. We like the music that you brought.
(d) Relative clause
16. He is so weak that he cannot run.
(a) Noun clause

w.E
(a) Adjunct clause (relative clause)
(b) Relative clause
(c) Adjective clause
(d) Adverb clause
(b) Adjunct clause (relative clause)
(c) Relative clause
(d) Adverb clause
6. That was when they laughed
asy
(a) Adjunct clause (relative clause)
17. He is such a weak boy that he cannot run.
(a) Noun clause
(b) Relative clause
(c) Adjective clause
(d) Predicative SV-clause En (b) Adjunct clause (relative clause)
(c) Relative clause
(d) Adverb clause
7. Bill stopping the project was a big disappointment.
(a) Adjunct clause (relative clause) gin
18. He is such a boy as does not help anybody.
(a) Adjective clause
(b) Relative clause
(c) Adjective clause
(d) Non-finite gerund clause eer
(b) Adjunct clause (relative clause)
(c) Relative clause
(d) Adverb clause
8. I asked him whenhe would go there.
(a) Noun clause
(b) Adjunct clause (relative clause)
ing
19. Since my brother came, he has been teaching.
(a) Adjective clause
(c) Relative clause
(d) Adjective clause
(b) Adjunct clause (relative clause)
(c) Relative clause
(d) Adverb clause
.ne
9. I shall not tell you where he lives.
(a) Adjunct clause (relative clause)
(b) Noun clause
(c) Relative clause
20. I shall do whatever he says.
(a) Adjective clause
(c) Relative clause
(b) Noun clause
(d) Adverb clause
t
(d) Adjective clause 21. I do not know whether he will come tomorrow.
10. I do not know if he will come. (a) Adjective clause (b) Noun clause
(a) Adverb clause (c) Relative clause (d) Adverb clause
(b) Adjunct clause (relative clause) 22. While it was raining, nobody went out.
(c) Noun clause (a) Adjective clause (b) Noun clause
(d) Relative clause (c) Relative clause (d) Adverb clause
11. I do not know whocame here last night. 23. It was dark, however we went out.
(a) Adverb clause (a) Adjective clause (b) Noun clause
(b) Adjunct clause (relative clause) (c) Relative clause (d) Adverb clause
(c) Noun clause
(d) Relative clause DIRECTIONS (Qs. 24 & 25): Choose the appropriate option to
12. I cannot say whose book is this? complete the sentences and that will be a relative clause.
(a) Adverb clause 24. The term track and field refers to athletic events ____ include
(b) Adjunct clause (relative clause) foot races and jumping and throwing events.
(c) Noun clause (a) where (b) that
(d) Relative clause (c) whom (d) in which

Downloaded From : www.EasyEngineering.net


Downloaded From : www.EasyEngineering.net

Clause Analysis  l 51

25. The moment _____ I learned the result of the art competition 40. It is important to ask whether the wedding is formal or
was one of the worst times in my life. semi-formal.
(a) which (b) when (a) Noun clause (b) Adjective clause
(c) where (d) why (c) Adverb clause (d) None of the above
DIRECTIONS (Qs. 26-30): Choose the clause type of the DIRECTIONS (Qs. 41-55): Choose the correct alternatives.
following sentences.
41. Murat decided to give up smoking _________he had had a
26. Get ready now.
heart attack two times.
(a) declarative (b) interrogative
(a) in case (b) although
(c) imperative (d) exclamative
(c) because (d) even if
27. Where did you leave the keys?
(a) declarative (b) interrogative 42. ____________ she was walking along the garden, she found
(c) imperative (d) exclamative a very rare flower.
28. You should brush your teeth. (a) because (b) provided that
(a) declarative (b) interrogative (c) so that (d) while
(c) imperative (d) exclamative 43. ____________ my father woke up, he realised that he had
29. Rita wants to know where to buy cheap gifts. forgotten to turn off the lights.

ww
(a) declarative
(c) imperative
(b) interrogative
(d) exclamative
30. Come in, sit down, and tell me why you were late.
(a) when
(c) while
(b) unless
(d) even though
44. __________ he behaves well, Mary will never talk to him.

w.E
(a) declarative
(c) imperative
(b) interrogative
(d) exclamative
DIRECTIONS (Qs. 31-40): Determine the function of the
(a) although
(c) if
(b) unless
(d) in the even that
45. __________ my best friend Hasan five years older than me,

asy
hilighted dependent clause in each of the followings.
31. Some people buy expensive carssimply because they can.
he looks very young.
(a) until (b) when
(a) Noun clause
(c) Adverb clause
(b) Adjective clause
En
(d) None of the above
(c) although (d) because
46. __________ they have passed the stamina test, they can get
their scuba diving license.
32. Some people buy expensive cars simply because they can.
(a) Noun clause
(c) Adverb clause
(b) Adjective clause
(d) None of the above gin (a) Now that
(c) Unless
(b) Even though
(d) When
33. The bankers need to know what they should do.
(a) Noun clause (b) Adjective clause easily.
eer
47. I have decided to buy a car ____________ I can go to work

(a) in case (b) so that


(c) Adverb clause (d) None of the above
34. Which one is the person who stole your car?
(a) Noun clause (b) Adjective clause ing
(c) provided that (d) although
48. Meera looks at her husband _________ she hasn’t seen him
(c) Adverb clause (d) None of the above
35. Wherever there is a large India city, there will be poverty.
before.
(a) as if .ne
(b) when
(a) Noun clause
(c) Adverb clause

(a) Noun clause


(b) Adjective clause
(d) None of the above
36. The books which the professor assigned were very expensive.
(b) Adjective clause
(c) although

they didn’t obey the rules.


(a) although
(d) because

(b) because
t
49. The soldiers in the camp will be punished ___________

(c) Adverb clause (d) None of the above (c) even if (d) however
37. Canada might give up its marketing boards if the European 50. You must wear your helmet while riding a motorbike,
Community gives up its grain subsidies. __________ you may hurt seriously in case of an accident.
(a) Noun clause (b) Adjective clause (a) but (b) although
(c) Adverb clause (d) None of the above (c) even though (d) otherwise
38. That is the place where American's and Japanese's armies 51. __________ you are in top form, your coach always shows
fought. you respect.
(a) Noun clause (b) Adjective clause (a) Unless (b) As long as
(c) Adverb clause (d) None of the above (c) Although (d) As if
39. Unless the crown can make a better case, the accused 52. __________ Jinni did the ironing, _____________ she had
murderer will not be convicte(d) also prepared the breakfast.
(a) Noun clause (b) Adjective clause (a) not only / but (b) not had only / but
(c) Adverb clause (d) None of the above (c) not only had / but (d) not only have / but

Downloaded From : www.EasyEngineering.net


Downloaded From : www.EasyEngineering.net

52  l  Clause Analysis

53. I don’t want to invite Kamal to the party ___________ he 64. The dog that Sam chose from the litter seems to be healthy.
always talks about nonsense things. (a) prepositional phrase (b) participial phrase
(a) although (b) even if (c) gerund phrase (d) infinitive phrase
(c) despite (d) because 65. Sam Smith, who recently spoke to the youth group, excels at
54. I’m going to work harder __________ I can be promoted. motivating young people.
(a) when (b) so that (a) prepositional phrase (b) participial phrase
(c) because (d) in spite of (c) gerund phrase (d) infinitive phrase
55. She was very exhausted ____________ she didn’t stop 66. Pushed beyond endurance, the runner dropped the baton.
working. (a) prepositional phrase (b) participial phrase
(a) but (b) unless (c) gerund phrase (d) infinitive phrase
67. The shoes he saw in the catalogue are available down the
(c) although (d) because
street.
DIRECTIONS (Qs. 56-75): Identify the underlined phrase or (a) prepositional phrase (b) participial phrase
clause. (c) gerund phrase (d) infinitive phrase
56. Steven's book, which made Oprah's Book Club this month, 68. The reporter crouched behind that tree got the best picture
is not in any stores. of the arrest.

ww
(a) prepositional phrase
(c) gerund phrase
(b) participial phrase
(d) infinitive phrase
57. While preparing for the speech, Joe couldn't help but worry
(a) prepositional phrase (b) participial phrase
(c) gerund phrase (d) infinitive phrase
69. Keith tried supporting both teams, but his heart was with

w.E
about his entrance.
(a) prepositional phrase
(c) gerund phrase
(b) participial phrase
(d) infinitive phrase
Oregon.
(a) prepositional phrase (b) participial phrase
(c) gerund phrase (d) infinitive phrase

asy
58. Ahmad wants to visit Quebec, but he will need to wait for
his next vacation.
70. Katrina, who resented being left at home, drew on the walls
with her crayons.
(a) prepositional phrase
(b) gerund phrase (d) infinitive phraseEn
(b) participial phrase
(a) prepositional phrase (b) participial phrase
(c) gerund phrase (d) infinitive phrase
71. Arnold hoped to find an answer to the funding shortfall.
59. Hoping for a miracle, the doctors continued the surgery.
(a) prepositional phrase (b) participial phrase gin (a) prepositional phrase (b) participial phrase
(c) gerund phrase (d) infinitive phrase
(c) gerund phrase (d) infinitive phrase
60. Our boss supports donating time to charity.
(a) prepositional phrase (b) participial phrase eer
72. Pressed for time, the agent ran the red light.
(a) prepositional phrase (b) participial phrase

(c) gerund phrase (d) infinitive phrase


61. Melanie hoped to find a cure for the disease, but she tried to ing
(c) gerund phrase (d) infinitive phrase
73. His uncle thinks that working for the government is the key
to stability.
be realistic.
(a) prepositional phrase (b) participial phrase
(c) gerund phrase (d) infinitive phrase
(c) gerund phrase .ne
(a) prepositional phrase (b) participial phrase
(d) infinitive phrase

62. After the banquet, the cooks will take a well-deserved break.
(a) prepositional phrase (b) participial phrase
(c) gerund phrase (d) infinitive phrase
74. Richard's chance to make his point slipped away.

(c) gerund phrase t


(a) prepositional phrase (b) participial phrase
(d) infinitive phrase
75. Is it possible that Joshua will compete against that man?
63. Joey is hoping for a change to play pool with his uncle. (a) prepositional phrase (b) participial phrase
(a) prepositional phrase (b) participial phrase (c) gerund phrase (d) infinitive phrase
(c) gerund phrase (d) infinitive phrase

Downloaded From : www.EasyEngineering.net


Downloaded From : www.EasyEngineering.net

Clause Analysis  l 53

Hints & Solutions


LEVEL- I 33. She was so tired that she could barely stand.
34. If you eat too much, you may fall ill.
1. The police has caught the man who committed the theft last 35. Though he started early, he finished late.
night. 36. If you do not tell me everything, I will not be able to help you.
2. The French language is different from the Latin language which 37. Do not go until I get ready.
was once spoken throughout Europe.
3. Can you tell me the reason why you are looking upset?
38. when you help other people (time)
4. All the plans which he had for making money quickly have
39. While you wait (time)
failed.
40. than I ever was (happy) before (comparison)
5. The landlord who was proud of his strength despised the
41. than a mule (is obstinate) (comparison)
weakness of his tenants.
6. This is the village where I was born. 42. while (she was) being taught to cook with broccoli (time)
7.
8.
9.
ww
Show me the place where you put the keys.
Paul who was an old gentleman was my travelling companion.
A fox which had never seen a crane before once met a crane.
43.
44.
45.
than he hits (baseballs well) (comparison)
As the lions approached the carcass (time)
While (I was) eating (time)
10.
11. w.E
This is the wooden case where the shopkeeper keeps his money.
Here the noun clause ‘that the traitor should be put to death’ is
that object of the verb ordered.
LEVEL- II
12.
asy
Here the noun clause ‘that he would not go’ is the object of the
verb said.
1.

2.
(d) Subject-auxiliary inversion is absent in embedded wh-
clause.
(b) Standard declarative clause
13.
subject of the verb is.
En
Here the noun clause ‘that he is not interested in the offer’ is the
3. (b) Relative clause introduced by the relative form when and
modifying the noun time
14.

15.
Here the noun clause ‘that he was not feeling well’ is the object
of the verb said.
Here the noun clause ‘what he says’ is the object of the gin
4.
5.
(a) Adjunct clause (relative clause) that modifies a noun
(a) Relative clause functioning as an adjunct that modifies the

16.
preposition on.
Here the noun clause ‘where he has gone’ is the object of the
6.
noun music

eer
(d) Predicative SV-clause, i.e., a clause that functions as (part
of) the main predicate

17.
verb know.
Here the noun clause ‘whether the servant had polished his
7.
8. ing
(d) The gerund stopping constitutes a non-finite clause
(a) Conjunction when denotes he

18.
shoes’ is the object of the verb asked.
Here the noun clause ‘that he is alive’ is in opposition to the
noun news.
9.
10.
(b) Conjunction where denotes he
(c) Conjunction if denotes he
.ne
19.

20.
Here the noun clause ‘that the soul is immortal’ is in opposition
to the noun belief.
Here the noun clause ‘that we will have to admit defeat’ is in
11.
12.
13.
14.
(c) Conjunction denotes who came
(c) Conjunction whose denotes book
(c) Here the conjunction denotes house
(a) Here the conjunction denotes he
t
opposition to the pronoun it. 15. (b) Here the conjunction that denotes study
21. Here the noun clause ‘that he was present’ is in opposition to 16. (d) Here the conjunction that denotes run
the pronoun it. 17. (d) Here the conjunction that denotes run
22. Here the noun clause ‘that only ten persons were killed in the 18. (a) Here the conjunction as denotes anybody.
riots’ is in opposition to the noun report. 19. (d) Here the conjunction since denotes came
23. Works 20. (b) Here the conjunction whatever denotes he
24. whose 21. (b) Here the conjunction whether denotes he
25. which 22. (d) Here the conjunction while denotes raining
26. whose 23. (a) Here the conjunction however denotes we
27. which. 24. (b) We use relative clauses to give additional information
28. I waited for my friend until he arrived. about something without starting another sentence. Here that
29. He hid where his pursuers could not find him. gives additional information about athletic events.
30. I am as intelligent as you are. 25. (b) Here when gives extra information
31. As he was not there, I left a message with his mother. 26. (c) This is an imperative clause. The second person subject you
32. We eat that we may live. has been omitted.

Downloaded From : www.EasyEngineering.net


Downloaded From : www.EasyEngineering.net

54  l  Clause Analysis

27. (b) This is an interrogative clause. Note the use of subject question "which place?" This is an adjective clause, modifying
auxiliary inversion (did you leave... rather than you did leave...). the noun "place."
28. (a) This is a declarative clause. (Although it may function as a 39. (c) This clause provides the conditions under which the accused
directive it has the form of a declarative - note that the second murderer will not be convicted, so it must be an adverb clause.
person subject you is not omitted.) 40. (a) This clause is the direct object of the infinitive "to ask,"
29. (a) This is a declarative clause. answering the question "what is it important to ask?"
30. (c) This is a series of imperative clauses. The second person 41. (c) 42. (d) 43. (a)
subject you has been omitted. 44. (b) 45. (c) 46. (a)
31. (a) This clause answers the question "why," showing cause, so it
47. (b) 48. (a 49. (b)
is an adverb clause. It does not act as a subject or object, and it
50. (d) 51. (b) 52. (a)
does not modify a noun or pronoun.
32. (a) The clause answers the question "what?", and acts as the 53. (d) 54. (b 55. (a)
direct object of the verb "hope." 56. (a) Prepositional phrase
33. (a) This clause does not tell you which bankers need to know, 57. (c) Gerund phrase
but rather, it tells you what they need to know -- since it 58. (d) Infinitive phrase
answers the question "what?" (and acts as the direct object of

ww
"to know"), it is a noun clause.
34. (b) The relative pronoun "who" might have confused you here;
59. (b) Participial phrase
60. (c) Gerund phrase
61. (d) Infinitive phrase

w.E
however, the clause itself does not answer the question "who?",
but the question "which person?", showing that it modifies the
noun "person" and is acting as an adjective clause.
62. (a) Prepositional phrase
63. (d) Infinitive phrase

asy
35. (c) This clause tells where poverty will exist, and specifying
a location is the function of an adverb or (in this case) of an
adverb clause.
64. (a) Prepositional phrase
65. (c) Gerund phrase
66. (b) Participial phrase
36. (b) This clause modifies the noun "books," and modifying a
En
noun or pronoun is the function of an adjective or (in this case)
67. (a) Prepositional phrase
68. (b) Participial phrase
of an adjective clause.
37. (c) This clause provides the conditions under which Canada
might give up its marketing boards, and it is an adverb or
gin
69. (c) Gerund phrase
70. (a) Prepositional phrase
an adverb clause which answers the question "under what
conditions?" 72. (b) eer
71. (d) Infinitive phrase
Participial phrase
38. (b) This is a very tricky example, and the subordinating
conjunction "where" could have fooled you. In fact, the clause
73.
74.
(a)
(d) ing
Prepositional phrase
Infinitive phrase
does not answer the adverb question "where?", but the adjective 75. (a) Prepositional phrase

.ne
t

Downloaded From : www.EasyEngineering.net


Downloaded From : www.EasyEngineering.net

5
PTER
CHA

Synthesis of Sentences

INTRODUCTION

ww
A sentence is a group of grammatically linked words that expresses a complete thought.
The process by which two or more simple sentences are joined to form one-simple, compound or complex sentence is called synthesis.

w.E
The process of breaking a sentence into phrases and clauses is called analysis. Synthesis of Sentences is the opposite of analysis of
sentences and involves combining number of simple sentences into one new sentence. The new sentence formed might be either
simple, compound or a complex sentence.
A simple sentence consists of — a single finite verb, a subject and a predicate. When two simple sentences are combined to form a

asy
new sentence, it should contain a finite verb.
The process of synthesis works in three ways:


• En
Making a new simple sentence by adding two simple sentences
Making a new compound sentence with two simple sentences
Making a new complex sentence with two simple sentences

MAKING OF SIMPLE SENTENCE gin


eer
A simple sentence can be made by combining two or more simple sentences by the following given ways:
1. By using a Participle


The two sentences have a common subject.
And the two actions are done simultaneously. ing
Two simple sentences can be combined into one simple sentence by using a participle when:-

A participle cannot be used when the simple sentences have different subjects.
For example: .ne
• Separate: He saw a tiger. He fled away. [‘He’ is the common subject.]
• Combined: Seeing a tiger he fled away.
• Separate: She tired of reading. She retired to bed. [‘She’ is the common subject.]
• Combined: Being tired of reading, she retired to bed.
t
When in two simple sentences, one action is completed before the other starts, we use ‘having’+ ‘past participle’ or ‘being’+ ‘past
participle’ to join the sentences.
Example: I was exhausted. I went to sleep.
Being exhausted, I went to sleep.
2. By using a noun or a phrase in apposition
Two simple sentences can be combined into a single simple sentence by using a noun or phrase in apposition. An apposition is the
use of a noun or a phrase immediately following a noun in the sentence and making special reference to it.
For example:
• Separate: Neha spent two days in Rome. It is one of the most attractive places in Italy.
• Combined: Neha spent two days in Rome, one of the most attractive places in Italy.
• Separate: My cousin was killed in a plane crash. He was one of my greatest supporters in my bad time.
• Combined: My cousin, one of my greatest supporters in my bad time, was killed in a plane crash.
3. By using a preposition with noun or gerund
In some cases, a preposition/ preposition phrase may be used to form a new sentence from few simple sentences. Preposition is used
with a noun or a gerund.

Downloaded From : www.EasyEngineering.net


Downloaded From : www.EasyEngineering.net

56  l  Synthesis of Sentences

For example:
• Separate: The moon rose. Their journey was not ended.
• Combined: Before their journey was ended, the moon rose. (preposition +gerund)
• Separate : The king has lot of power. He does not have any friends.
• Combined: In spite of all his powers, the king does not have any friend. (preposition + noun).
4. By using, Nominative Absolute Construction
A nominative absolute is the subject that does not affect the number and/or person of the verb in the sentence.
This is used when:
• The two simple sentences to be joined appear to be related.
• They have different subjects.
• The new simple sentence formed is as followed:
‘subject + having + past participle’ OR ‘there+ being+ subject’.
• If the sentence is in passive voice, it should be changed into an absolute phrase retaining its passive form.
For example:
• Separate: The police arrived. The mob dispersed.

ww


Combined: The police having arrived, the mob dispersed.
Separate: Rains have been plentiful this year. The crop of rice has been rich.

w.ECombine: Rains having been plentiful this year, the crop of rice has been rich.
5. By using an Infinitive
Two simple sentences can be joined into one by using an infinitive when a sentence expresses a purpose or cause.

For example
asy
Separate: I am going to Delhi. I have to purchase a house. [Purpose: purchase of house]

• Combined: I am going to Delhi to purchase a house.

En
• Separate: Ajay is very weak. Ajay cannot pass this year. [Cause: weak]
• Combined: Ajay is too weak to pass this year.
6. By using an Adverb or an Adverbial Phrase
gin
To combine two simple sentences into one simple sentence, an adjective is converted into an adverb or an adverbial phrase.
For Example:
• Separate: It was evening. The flight had not reached by the time. eer



Combined: The flight had not reached by evening.
Separate: Ajay was dismissed from service. His dismissal was undeserved.
Combined: Ajay was undeservedly dismissed from service.
ing
Several of these methods can be combined in the same sentence also for the formation of a new sentence.
• The sun rose. The fog dispersed. The general determined to delay no longer. He gave order to advance. .ne
These four simple sentences may be combined to form a single simple sentence.
• At sunrise, the fog having dispersed, the general, determined to delay no longer, gave the order to advance.
By using too+ Adjective / Adverb
t
• Separate: He is Poor. He cannot buy this.
• Combined: He is too poor to buy this.
• Separate: She is weak. She cannot run.
• Combined: She is too weak to run.
Adjective / Adverb + Enough
• Separate: This hall is large. Five hundred persons can sit in it.
• Combined: This hall is large enough for five hundred persons to sit in.
• Separate: He has much time. He can enjoy this big mall.
• Combined: He has time enough to enjoy this big mall.

MAKING OF COMPLEX SENTENCE


Complex sentence are made by adding two or more than two simple sentences. A complex sentence has a principal clause and one
or more subordinate clauses. The principal clause can be considered a complete sentence in itself, but the subordinate clause is
dependent on the main clause for its meaning. When we join two or more simple sentences to form a complex sentence, we retain
one sentence as the principle clause and convert the other sentence into a subordinate clause which can be a noun, adjective or a
adverb clause.

Downloaded From : www.EasyEngineering.net


Downloaded From : www.EasyEngineering.net

Synthesis of Sentences  l 57

1. By using noun clause


Noun clause works in different ways in a complex sentence:
• Subject of verb,
• Object of transitive verb or preposition,
• Verb of incomplete predication ( is, are, am, was, were) complement and
• in case of opposition.
• In synthesis also noun form can be used by using noun clause.
The conjunction used for introducing a noun clause is ‘that’.
For example:
• Separate: He is speaking true. It is known to all.
• Combined: That he is speaking true it is known to all.
• Separate: She is saint. Everybody knows it.
• Combined: Everybody knows that she is a saint.
2. By using Adjective Clause
Synthesis of adjective clause can be done by relative pronoun (who, which, that) or relative adverb (where, when, why). An adjective
clause does the work of an adjective in a complex sentence and modifies a noun in the principal clause. When two simple sentences

ww
are joined to form one complex sentence, usually the second sentence is changed into an adjective clause.
The relative pronoun or relative adverb is placed nearest to its antecedent.
For example:

w.E
Separate- Aditi met an old man. He was very weak.
Combined- Aditi met an old man who was very weak.
Separate- I have purchased a big house, it has a big guest room. My guestroom is well- decorated.

asy
Combined- The house that I have purchased has a big guest room which is well decorated.
3. By using Adverb clause

En
The adverb clause functions as an adverb in a complex sentence. Synthesis can be done by using Adverb clause in a way of
Adverb clause of time , reason, place, condition, comparison, contrast, result, manner. The adverb clause is introduced using the

For example:
• gin
appropriate subordinate conjunction (although, so that, if, while, when, until etc.)

Separate: Tim committed the theft. He has been caught by the police.


Combined: Tim committed the theft, so he has been caught by the police.
Separate: Tell me the truth. I shall pardon you. eer
• Combined: I shall pardon you if you tell me the truth.

MAKING OF COMPOUND SENTENCE ing


.ne
Compound sentences are made by adding two or more than two simple sentences. Synthesis also can be done by making co- ordinate
conjunctions like (either-or, neither-nor, also, likewise, so, therefore, and, but) sometimes comma (,) and Semi colon (;) also work like
co-ordinate conjunction and compound sentence can be made by that. Coordinating conjunctions can be used to join grammatical
units of the same class, rank or pattern.
t
Coordinating conjunctions can be classified as: cumulative conjunctions, adversative conjunctions, alternative conjunctions and
illative conjunctions.
1. Cumulative conjunction adds one statement to another.
This includes conjunctions like–and, as well as, not only-but also, both, and.
Compound Example: Pratap is a hard worker. He is a perfectionist.
: Pratap is a hard worker and a perfectionist.
It can also be written as: Pratap is not only a hard worker but also a perfectionist.
The conjunctions not only-but also, both-and are used for greater emphasis.
2. Adversative conjunctions express opposition and contrast.
It includes the following conjunctions–but, still, yet, whereas, nevertheless, however.
Compound Example: Amit was sick. He came for meeting.
: Amit was sick but came for meeting.
3. An alternative conjunction expresses a choice between two alternatives.
It includes conjunctions like– or, either-or, neither-nor, else, otherwise.
Compound Example: Sit down. Leave the class.
: Sit down or leave the class.
4. Illative conjunction expresses inference. It includes conjunctions like–so, therefore, for.
Compound Example: The court has given the verdict. We must accept it.
: The court has given the verdict, therefore, we must accept it.

Downloaded From : www.EasyEngineering.net


Downloaded From : www.EasyEngineering.net

58  l  Synthesis of Sentences

IMPORTANT NOTE: Model-5:


While connecting two sentences by using the words or “ As soon as ”, “ No sooner ” :
phrases given in brackets- • I saw my brother. At once I ran to meet him. (As soon as)
[If], [Unless], [ As if], [As though], [Although], [Through], [Too As soon as I saw my brother, I ran to meet him.
..to], [So ... that ... not], [ As soon as], [No sooner], [Since], • As soon as she saw a snake she ran away. (No sooner)
[As], [Because], [Participles], [In the event of], [In spite of], No sooner did she see a snake that she ran away.
[Would rather], [If only].The following conductions should be Model-6 :
omitted– “ Since ”, “ As ”, “ Because ” :
Yet, still, but, really, at once, so, thus, however, nevertheless, • I did not study well. So I failed in the examination. (Since)
only and therefore, and hence, then, for, also immediately, Since I did not study well, I failed in the examination.
soon, now sometimes, ever and so, and or, exactly at that • Kamala is blind. So she cannot see me. (As)
time, otherwise etc. As Kamala is blind she cannot see me.
• I like you. I shall help you. (because)
FOLLOWING MODELS ARE: Because I like you, I shall help you.
Model-1 : Model-7 :
“ If ” and “ unless ” clause : Participles :
• She saw the tiger and ran away. (seeing)

• ww
Work hard, you will pass. (If)
If you work hard, you will pass.
You must work hard. You will fail. (Unless) •
Seeing the tiger, she ran away.
I saw him walk along the road. (walking)

Model-2 :
w.E
Unless you work hard, you will fail.

“ As if ”, “ As though ” :
I saw him walking along the road.
Model-8 :
“ In the event of ”
• If you study well, you will pass the examination. (In the

• asy
Mahesh is not a rich man. But he talks like one. (as though)
Mahesh talks as though he were a rich man.
Ajay spoke like a great orator. (as if)
event of)
In the event of you studying well, you will pass the
Ajay spoke as if he were a great orator.
Model-3 :
En examination.
Model-9 :
“ In spite of ”
“ Although ”, “ Though ” :
• She is poor. But she is generous. (Though)
Though she is poor, she is generous.

gin He worked hard. But he failed. (In spite of)
In spite of him working hard, he failed.


He danced well. But he did not get the prize. (Although)
Although he danced well, he did not get the prize.
Model-10 :

eer
“ Would rather ”
• He prefer to write. He don’t want to speak. (would rather)
Model-4 :
“ Too...to ”, “ So...that...not ” :
• She is very weak. So she cannot walk. (too...to)

Model-11 : ing
He would rather write than speak.


She is too weak to walk.
Raman is too weak to walk. (So...that...not...)
“ If only ”

.ne
The dog barked loudly. She did not like it. (if only)
If only the dog had not barked, she would have liked it.
Raman is so weak that he cannot walk.

Downloaded From : www.EasyEngineering.net


Downloaded From : www.EasyEngineering.net

Synthesis of Sentences  l 59

Practice Exercise
LEVEL-I
DIRECTIONS (Qs. 1-9): Combine the following simple sentences 6. Ashoka was a great conqueror. But he failed to unite the
into one simple sentence. various conquered territories into a well-knit empire.
1. Mahatma Gandhi was a great visionary. He realised the (a) Ashoka was a great conqueror, but he failed to unite
danger of communal conflicts. He sacrificed his life to bring the various conquered territories into a well-knit
about unity.
empire.
(a) A great visionary, Mahatma Gandhi realised the
(b) Though Ashoka was a great conqueror, but he failed
danger of communal conflicts and sacrificed his life
to unite the various conquered territories into a well-
to bring about unity.
(b) Mahatma Gandhi, a great visionary, He sacrificed his knit empire.
life to bring about unity on realising the danger of (c) Despite being a great conqueror, Ashoka failed to
unite the various conquered territories into a well-

wwcommunal conflicts.
(c) Mahatma Gandhi sacrificed his life to bring about
unity when he realised the danger of communal
knit empire.
(d) Ashoka failed to unite the various conquered territories

w.E
conflicts.
(d) Having realised the danger of communal conflicts
Mahatma Gandhi, a great visionary, sacrificed his life 7.
into a well-knit empire, in spite of being a great
conqueror.
I must finish this novel. There are still few chapters.

2.
to bring about unity.

asy
She stood on tip-toe. She reached for the mangoes on tree.
(a) She reached for the mangoes on tree by standing tip-
(a) I have still few chapters in this novel to finish.
(b) I still have few chapters in the novel before I finish it.
(c) Few chapters are still left for me to finish the novel.
toed.
En
(b) She reached for the mangoes on tree when she stood 8.
(d) There are still few chapters for me to finish this novel.
They decided to start a travel group. All agreed to this.
on tip-toe.
(c) Standing on tip-toe she reached for the mangoes on
tree.
gin (a) They all agreed to start a travel group.
(b) They jointly decided to start a travel group.
(c) They agreed to jointly start a travel group.
(d) She reached the mangoes on tree standing on tip toe.
3. India will win the World cup. That is certain. 9. eer
(d) They unanimously decided to start a travel group.
Rabindranath Tagore was a great poet. He was a good
(a) It is certain that India will win the World Cup.
(b) India will certainly win the World Cup.
(c) Certainly, India will win the World Cup.
painter.
ing
(a) Rabindranath Tagore was not only a great poet, but

(d) India will win the World Cup certainly.


4. The US Trade Centre caught fire. The building was brunt to
also a good painter.

.ne
(b) Besides being a great poet, Rabindranath Tagore was
a good painter.
ashes.
(a) The US Trade Centre having caught fire, the building
was brunt to ashes.
(b) The building of US Trade Centre caught fire and burnt
good painter. t
(c) Rabindranath Tagore was a great poet as well as a

(d) Rabindranath Tagore was a great poet and painter.


to ashes. DIRECTIONS: (Qs. 10-15): Combine the following simple
(c) The building of US Trade Centre was burnt to ashes sentences into a compound sentence.
when it caught fire. 10. There was little hope of success. She tried hard.
(d) The US Trade Centre caught fire and the building was (a) There was little hope of success, still she tried hard.
brunt to ashes. (b) There was little hope of success, but she tried hard.
5. The servant mopped the room; she found a wallet lying in (c) She tried hard but there was little scope of success.
the corner. (d) There was little hope of success, nevertheless she tried
(a) The servant found a wallet while mopping the corner hard.
of the room. 11. Sheena will not spend her money. She will not invest it.
(b) The servant founded a wallet lying in the corner while (a) Sheena will neither spend her money nor invest it.
mopping the room. (b) Sheena will not spend her money and will not invest
(c) The servant mopped the room and found a wallet it.
lying in the corner. (c) Sheena will either spend her money or invest it.
(d) While mopping the room, the servant found a wallet (d) Sheena will not spend her money or will not invest it
lying in a corner. either.

Downloaded From : www.EasyEngineering.net


Downloaded From : www.EasyEngineering.net

60  l  Synthesis of Sentences

12. Ani speaks English. She also speaks French. 19. Her younger brother is in Canada. He is a lecturer at a
(a) Ani speaks English as well as French. prestigious University.
(b) Ani speaks English apart from French. (a) Her younger brother is a lecturer at a prestigious
(c) Ani speaks both English and French. University in Canada.
(d) Ani speaks English and French. (b) Her younger brother, who is in Canada, is a lecturer at
13. Get inside. You may catch cold. a prestigious University.
(a) Get inside else you may catch cold. (c) Her younger brother is a lecturer at a prestigious
(b) You may catch cold, get inside. Canadian University.
(c) Get inside, otherwise you may catch cold. (d) Her Canada based younger brother is a lecturer at a
(d) Get inside or you may catch cold. prestigious University.
14. He went to the market. He saw the latest gadget. He could 20. Which of the following statement about synthesis of
not, however purchase it. sentence is incorrect?
(a) Synthesis means combining two or more simple
(a) He went to the market and could not purchase the
sentences in one sentence.
latest gadget that he saw.
(b) Synthesis is opposite of transformation of sentences.
(b) He went to the market and saw the latest gadget but he
(c) Synthesis leads to the formation of either aa simple,
could not purchase it. complex or compound sentence.

ww
(c) He saw the latest gadget in the market and could not
purchase it.
(d) He could not purchase the latest gadget he saw in the
(d) All are correct.
21. Which of the following statement is incorrect?
(a) When we combine two simple sentences by using an

w.E
market.
15. Mohan is industrious. He is very particular in his work.
(a) Mohan is very particular in his work as he is
adverb or an adverbial phrase, we change the adverb
of the sentence into adjective.
(b) Two simple sentences are joined using a participle
industrious.
asy
(b) Mohan is not only industrious but also very particular
in his work.
when they have a common subject.
(c) The conjunction for noun clause in synthesis of
complex sentences is ‘that’.

En
(c) Mohan is industrious and particular in his work.
(d) Mohan is industrious as well as particular in his work.
(d) Compound sentences are joined using coordinating
conjunctions.

DIRECTIONS: (Qs. 16-19): Combine the following simple


sentences into a complex sentence.
gin
DIRECTIONS: (Qs. 22- 26): Study the given simple sentences
that have been combined to form a simple sentence. Decipher how

16. India will have a good monsoon season this year. The
meteorologists expect this. London. eer
they have been combined.
22. Separate: She wanted to educate her son. She sent him to

(a) The meteorologists expect a good monsoon season


this year in India.
(b) The meteorologists expect that India will have a good
ing
Combined: She sent her son to London to educate him.
(a) By using an adverb or an Adverbial Phrase
(b) By using a Participle
monsoon season this year.
(c) As expected by the meteorologists, India will have a
(c) By using an infinitive
.ne
(d) By using, Nominative Absolute Construction
good monsoon season this year.
(d) The meteorologists expect India to have a good
monsoon season this year.
17. We visited the Golden Temple. Gurunanak Devji preached
Combined: Anita was badly scolded by her uncle
(a) By using a preposition with noun or gerund
(b) By using, Nominative Absolute Construction
t
23. Separate: Anita was scolded. Her uncle scolded badly.

(c) By using a noun or a phrase in apposition


his first sermon here.
(d) By using an adverb or an Adverbial Phrase
(a) We visited the Golden Temple wherein Gurunanak
24. Separate: She has failed many times. She still hopes to get
Devji preached his first sermon.
success at last.
(b) We visited the Golden Temple because Gurunanak
Combined: In spite of many failures she still hopes to get
Devji preached his first sermon. success at least.
(c) We visited the Golden Temple where Gurunanak (a) By using a preposition with noun or gerund
Devji preached his first sermon. (b) By using a Participle
(d) Gurunanak Devji preached his first sermon at the (c) By using a noun or a phrase in apposition
Golden Temple. (d) By using an infinitive
18. She was honest. This was his statement. It was right. 25. Separate: I love Sanchit very much. He is my brother.
(a) He rightly said in his statement that she was honest. Combined: I love my brother Sanchit very much.
(b) She was rightly said as honest in his statement. (a) By using a noun or a phrase in apposition
(c) His statement was right in which he said that she was (b) By using a preposition with noun or gerund
honest. (c) By using an adverb or an Adverbial Phrase
(d) His statement that she was honest was right. (d) By using an infinitive

Downloaded From : www.EasyEngineering.net


Downloaded From : www.EasyEngineering.net

Synthesis of Sentences  l 61

26. Separate: The monkey sat on a branch. He gibbered. (c) I met your friend whose wallet was stolen in the
Combined : Sitting on a branch the monkey gibbered. market.
(a) By using an infinitive (d) I met your friend because his wallet was stolen in the
(b) By using a noun or a phrase in apposition market.
(c) By using a pParticiple 29. Given are two simple sentences, decipher which of the
(d) By using a preposition with noun or gerund following is correctly combined complex sentence by using an
27. Given are two simple sentences, decipher which of the adverb clause?
following is correctly combined complex sentence by using a Separate: The burglar saw the police. He took his heels.
noun clause? (a) Seeing the police, the burglar took his heels.
Separate: She has failed. The reason is her illness.
(b) On seeing the police, the burglar took his heels.
(a) The reason of her failure is that she is ill.
(c) As soon as the burglar saw the police, he took his
(b) She was ill and thus failed.
heels.
(c) She failed because of her illness.
(d) The reason behind her failure is her illness. (d) As the burglar saw the police, he took his heels.
28. Given are two simple sentences, decipher which of the 30. Given are two simple sentences, decipher which of the
following is correctly combined complex sentence by using an following is correctly combined complex sentence by using an
adjective clause? adjective clause?

market.ww
Separate: I met your friend. His wallet was stolen in the Separate: I saw a car. It was black in colour.
(a) I saw a car in black colour.
(b) I saw a black coloured car.

w.E
(a) I met your friend and his wallet was stolen in the
market.
(b) As soon as I met your friend, his wallet was stolen in
the market.
(c) I saw a black car.
(d) I saw a car which was black in colour.

asy LEVEL-II
DIRECTIONS (Qs. 1-10): Combine the following sentences using
En 6. The prisoner fell down on his knees. He begged for mercy.
an appropriate conjunction from the given options.
1. He is snobbish. I like him.
(a) Though he is snobbish, I like him. gin(a) The prisoner fell down on his knees, he begged for
mercy
(b) The prisoner fell down on his knees and begged for
(b) He is snobbish and I like him
(c) He is snobbish but I like him eer
mercy.
(c) The prisoner begged for mercy felling down on his
knees.
(d) I like him because he is snobbish
2. My mother attended the wedding. My father attended the
wedding.
knees. ing
(d) He begged mercy the prisoner felling down on his

(a) My mother and father attended the wedding.


(b) My mother attended the wedding also my father. .ne
7. I left home early. I could not get to work on time.
(a) Because I left home early I could not get to work on
time.
(c) Both my mother and father attended the wedding.
(d) My father, mother attended the wedding.
3. He has walked five miles. It is possible that he is tired.
(a) He has walked five miles, it is possible that he is tired.
t
(b) But I left home early could not get to work on time,
(c) Though / although I left home early, I could not get to
work on time.
(d) I could not get to work on time as I left home early.
(b) He has walked five miles, so he must be tired. 8. Azim works hard Kabir works harder.
(c) He has walked five miles therefore it is possible that he (a) Azim works hard but Kabir works harder.
is tired. (b) Both Azim and Kabir work harder.
(d) He has walked five miles and possible that he is tired. (c) Kabir and Azim work harder.
4. I went to the shop. I bought a watch. (d) Kabir, Azim work harder.
(a) I went to the shop bought a watch. 9. You must start at once. You will be late.
(a) You will be late must start at once.
(b) I went to the shop to bought a watch.
(b) You must start at once or you will be late.
(c) I went to the shop and bought a watch.
(c) You must start at once because you will be late.
(d) To buy a watch I went to the shop.
(d) You must start at once for you will be late.
5. You must be quiet. You must leave the room.
10. You are not interested in this offer. I know it.
(a) You must be quiet. You must leave the room. (a) I know it you are not interested in this offer.
(b) You must leave the room and quiet. (b) You are not interested in this offer this I know it
(c) Must you leave and quiet the room (c) I know that you are not interested in this offer.
(d) You must be quiet or you must leave the room. (d) I know as you are not interested in this offer.

Downloaded From : www.EasyEngineering.net


Downloaded From : www.EasyEngineering.net

62  l  Synthesis of Sentences

DIRECTIONS (Qs. 11-32 ): Choose the right combined sentence 21. It is true. My brother has left for America.
from the following options. (a) It is true my brother has left for America
11. I say it. He is a good man. (b) My brother has left for America, it is true.
(a) I say he is a good man. (c) It is true that my brother has left for America.
(b) He is good man I say (d) It is my brother left for America is true.
(c) I say that he is a good man. 22. It was not known. What is she doing in Indore.
(d) I say he is good man it. (a) It was not known what she was doing in Indore.
12. I cannot say. Will he come tomorrow? (b) It was known she is doing what in Indore.
(a) I cannot say he will come tomorrow. (c) What she was doing in Indore is not known.
(b) I cannot say if/whether he will come tomorrow. (d) Not known she was doing what in Indore.
(c) He will come tomorrow I cannot say. 23. I know the man. He is a doctor.
(d) Will he come tomorrow I cannot say. (a) Who is a doctor I know the man
13. I don’t know. What is he doing? (b) He is a doctor I know the man
(a) I don’t know he is doing what? (c) I know the man who is a doctor.
(b) What is he doing I don’t know? (d) He is the man I know the doctor.
(c) He is doing what I don’t know. 24. I don’t know any of the men. They live here.

ww
(d) I don’t know what he is doing.
14. Everybody was pleased to know. She was absent.
(a) Everybody was pleased to know that she was absent.
(a) They live here I don’t know any of them
(b) I don’t know any of the men that live here.
(c) I don’t know they live here any of the men.

w.E
(b) She was absent everybody was pleased.
(c) Everybody was pleased to know she was absent.
(d) Everybody was pleased she was absent to know.
(d) Any of the men I don’t know live here.
25. The monster was proud of his power. He was defeated by
Hercules.

asy
15. She kept on asking. When will her mother return?
(a) She kept on asking when her mother would return.
(a) The monster who was proud of his power was defeated
by Hercules.
(b) When will her mother return kept on asking she.
(c) She kept on asking when her mother retun would.
(d) Would her mother return she kept on asking. En (b) The monster was defeated by Hercules was proud of
his power.
(c) He was defeated by Hercules was proud of his power.
16. I don’t believe. She said something.
(a) I don’t believe what she said. gin(d) The monster proud of his power was defeated by
Hercules.
(b) What she said I don’t believe.
(c) I don’t believe in what she said.
(d) She said something don’t believe I.
eer
26. It was midnight. The trains collided then.
(a) The trains collided then it was midnight

17. Please listen. Your teacher is telling something.


(a) Please listen what your teacher is telling you. ing
(b) In the midnight collided then the trains.
(c) It was midnight when the trains collided.
(d) Midnight trains collided then.
(b) Please listen to what your teacher is telling.
(c) Your teacher is telling something to listen.
.ne
27. He settled in Nagpur. He was born there.
(a) He was born there settled in Nagpur.
(d) Please listen your teacher to telling something.
18. The hope is. She will return tomorrow.
(a) The hope is that she will return tomorrow.
(b) The hope is that she will return tomorrow.
(b) He settled in Nagpur was born there.
(c) Settled in Nagpur was born there he.
(d) He settled in Nagpur where he was born.
28. I will get money for you. Don’t go till then.
t
(c) She will return tomorrow is the hope. (a) Don’t go till then I will get money for you.
(d) That she will return is the hope. (b) I will get money for you, don’t go.
19. The fact is. Nisha has not qualified the test. (c) Don’t go I will get money for you.
(a) Nisha has not qualified the test is the fact. (d) Don’t go until I get money for you.
(b) The fact, Nisha has not qualified the test. 29. He was going to school. He was caught in the train.
(c) The fact is that Nisha has not qualified the test. (a) Going to school he was in the train was caught.
(d) Nisha is the fact not qualified the test. (b) He was caught in the train going to school.
20. She never believed in his statement. His father is a doctor. (c) Caught in the train he was going to school.
(a) She never believed in his statement that his father was (d) While he was going to school, he was caught in train.
a doctor. 30. His father retired last year. He has been idle from that time.
(b) His father is a doctor she never believed statement. (a) He has been idle from that time his father retired.
(c) In his statement she never believed his father is a (b) Since his father retired last year he has been idle.
doctor. (c) His father retired last year, he has been idle from that
(d) His father is a doctor she never believed in his time.
statement. (d) Idle from that time his father retired last year.

Downloaded From : www.EasyEngineering.net


Downloaded From : www.EasyEngineering.net

Synthesis of Sentences  l 63

31. He will win the first prize. That is certain. (c) You look for it if you will find it.
(a) He will certainly win the first prize. (d) You will find it if you look for it.
(b) He will win the first prize that is certain. 36. I told you not to do it. You have done it again.
(c) That is certain the first prize he will win. (a) You have done it again when I told you not to do it.
(d) The first prize that is certain he will win. (b) I told you not to do it  but  you have done it again.
32. She had many shortcomings. But she was a nice woman. (c) I told you not to do it, you have done it again.
(a) She had many shortcomings but she was a nice (d) Despite I told you not to do it you have done it again.
woman. 37. She was really upset with him. She said nothing.
(b) In spite of having many shortcomings, she was a nice (a) She was really upset with him she said nothing.
woman. (b) Being she was really upset with him she said nothing
(c) She was a nice woman had many shortcomings. (c) As she was really upset with him, she said nothing.
(d) She was a nice woman have many shortcomings. (d) She said nothing was really upset with him.
DIRECTIONS (Qs. 33-40): Combine the following pairs of sen- 38. You heat ice. It melts.
tences with suitable conjunctions. (a) If/when you heat ice, it melts.
33. He didn’t get admission. His grades were not very good. (b) You heat ice it melts.
(a) He didn’t get admission, his grades were not very good. (c) It melts heating ice.

ww
(b) He didn’t get admission because his grades were not
very good.
(c) His grades were not very good and he didn’t get
39.
(d) Heating ice it melts.
It will start working. You give it a kick.
(a) It will start working you give it a kick.

w.E
admission.
(d) As his grades were not very good he didn’t get
admission.
(b) It will start working by giving it a kick.
(c) It will start working  when/if  you give it a kick.
(d) You give it a kick it will start working.

asy
34. The teacher enters the class room. You should get up.
(a) When the teacher enters the class room, you should
get up.
40. He paid the subscription fee. His membership was renewed.
(a) He paid the subscription fee his membership was
renewed.

En
(b) You should get up as the teacher enters the class room.
(c) With the teacher enters the class room you should get
(b) His membership was renewed as he paid the
subscription fee.
up.
(d) As soon as the teacher enters should get up
35. You will find it. You look for it. gin (c) He paid the subscription fee so his membership was
renewed.
(d) His membership was renewed by paying subscription
(a) If you look for it will find it
(b) When you look for it you will find it. eer fee.

Hints & Solutions


i ng .ne
LEVEL- I
1. (d) Having realised the danger of communal conflicts
10.
11.
12.
(a) Sheena will neither spend her money nor invest it.
(a) Ani speaks English as well as French.
t
(d) There was little hope of success, nevertheless she tried hard.

Mahatma Gandhi, a great visionary, sacrificed his life to bring 13. (d) Get inside or you may catch cold.
about unity. 14. (b) He went to the market and saw the latest gadget but he
2. (c) Standing on tip-toe she reached for the mangoes on tree. could not purchase it.
3. (b) India will certainly win the World cup. 15. (b) Mohan is not only industrious but also very particular in
4. (a) The US Trade Centre having caught fire, the building was his work.
16. (b) The meteorologists expect that India will have a good
brunt to ashes.
monsoon season this year.
5. (d) While mopping the room, the servant found a wallet lying
17. (c) We visited the Golden Temple where Gurunanak Devji
in a corner.
preached his first sermon.
6. (c) Despite being a great conqueror, Ashoka failed to unite the 18. (d) His statement that she was honest was right.
various conquered territories into a well-knit empire. 19. (b) Her younger brother, who is in Canada, is a lecturer at a
7. (a) I have still few chapters in this novel to finish. prestigious University.
8. (d) They unanimously decided to start a travel group. 20. (b) Synthesis is opposite of analysis of sentences. Synthesis
9. (b) Besides being a great poet, Rabindranath Tagore was a means joingjoining of two or more sentences and analysis means
good painter. fragmenting a sentence into words/phrases. Transformation is

Downloaded From : www.EasyEngineering.net


Downloaded From : www.EasyEngineering.net

64  l  Synthesis of Sentences

a process of transforming sentences retaining the meaning of 18. (a) Subordinate conjunction that becomes the complement of
the sentence. the verb and thus combines two clauses.
21. (a) When we combine two simple sentences by using an adverb 19. (c) Subordinate conjunction that that becomes the complement
or an adverbial phrase, we change the adjective of the sentence of the verb and combines two noun clauses.
into adverb. 20. (a) Subordinate conjunction that here acts as the case in
22. (c) By using an infinitive apposition to the noun and combines the two simple sentences.
23. (d) By using an adverb or an Adverbial Phrase 21. (c) Subordinate conjunction that here acts as the case in
24. (a) By using a preposition with noun or gerund apposition to the pronoun
25. (a) By using a noun or a phrase in apposition 22. (a) Subordinate conjunction what here acts in apposition to
26. (c) By using a Participle the pronoun and combines two sentences.
27. (a) The reason of her failure is that she is ill. 23. (c) Subordinate conjunction of the Adjective clause who
28. (c) I met your friend whose wallet was stolen in the market. combines the two sentences.
29. (c) As soon as the burglar saw the police, he took his heels. 24. (b) Subordinate conjunction of the Adjective clause that
30. (d) I saw a car which was black in colour. combines the two sentences.
25. (a) Subordinate conjunction of the Adjective clause who
combines the two sentences.
LEVEL- II
1. ww
(a) Subordinating conjunction though joins the two sentences
26. (c) Subordinate conjunction of the Adjective clause when joins
the sentences in a right way.
2.

3.
w.E
(c)  Correlative Conjunctions both, and combines the two
sentences in a perfect manner.
(b) Both the conjunctions so and because can be used to bring
27. (d) Subordinate conjunction of the Adjective clause where
combines the two sentences in a correct way.
28. (d) Time clause conjunction until combines the two sentences

4.
out meaningful combination.
asy
(c) Correlative conjunction and combines two simple
in a correct way.
29. (d) Time clause conjunction while combines the two sentences
in a correct way.

5.
sentences.

En
(d) Correlative conjunction or combines both the sentences.
30. (b) Time clause conjunction since combines two simple
sentences in a correct way.
6.

7.
(b) Coordinating conjunction and is used to combine the two
sentences.
(c) Subordinating conjunction though/although combines two gin
31. (a) By using an adverb certainly the two simple sentences are
combined in a correct grammatical form.
32. (b) By using a preposition + noun / gerund this sentence is

8.
simple sentences.
(a) Both but the coordinating and than the subordinating
combined.
eer
33. (b) Subordinating conjunction because is used to join the two

9.
conjunction can be used
(b) Coordinating conjunction or is used
sentences.
ing
34. (a) When a coordinating conjunction connects two independent
10.
11.
(c) Subordinating conjunction that is used
(c) Subordinate conjunction that combines the two noun
clauses

.ne
35. (d) Subordinating Conjunctions if is used to join the two
simple sentences.
12.

13.
cluses.
(b) Subordinate conjunction if/whether combines the two
noun cluses.
(d) Subordinate conjunction what combines the two noun
t
36. (b) Some conjunctions combine with other words to form
what are called correlative conjunctions. They always travel in
pairs, joining various sentence elements that should be treated
as grammatically equal. Here but is used to combine the two
cluses.
sentences.
14. (a) Subordinate conjunction that becomes object of infinitive
37. (c) Correlative conjunction as is used to combine the two
and combine the two noun clauses.
simple sentences.
15. (a) Subordinate conjunction when becomes the object of 38. (a) Subordinating conjunction if is used to combine the two
gerund and participle and combine the two noun clauses. sentences.
1 6. (c) Subordinate conjunction what becomes the object to 39. (c) When a coordinating conjunction connects two independent
preposition and combines the two clauses in a simple way. clauses,
17. (b) Subordinate conjunction what becomes the object to 40. (c) Be careful of the conjunction so. Sometimes it can connect
preposition and combines the two clauses two independent clauses along with a comma, but sometimes
it can’t.  Here so is used to combine the two sentences.

Downloaded From : www.EasyEngineering.net


Downloaded From : www.EasyEngineering.net

6
PTER
CHA

TRANSFORMATION OF
SENTENCES

ww
INTRODUCTION
There are three types of sentences in English–simple, complex
AFFIRMATIVE TO NEGATIVE
Rule 1: Only/alone/merely → Replaced by → None but (person)/

w.E
and compound sentences. The transformation of a sentence
means changing its form without altering its sense. We can
change a simple sentence into a compound sentence or a
nothing but (things)/ not more than or not less than (number)
Example:
Affirmative: Only God can help us.

asy
complex sentence. This is done by expanding a word or phrase
into a clause. Similarly, we can change a complex or compound
Negative: None but God can help us.
Rule 2: Must →Replaced by → Cannot but/ Cannot help+ (v+ing)

into a word or phrase.


En
sentence into a simple sentence. This is done by reducing a clause Example:
Affirmative: We must obey our elders.
• A simple sentence has just one clause.
• A complex sentence has one main clause and one or more gin
Negative: We cannot but obey our elders. /We cannot
help obeying our elders.
subordinate clauses.
• A compound sentence has more than one main clause.
Study the examples given below:
eer
Rule 3: Both----and → Replaced by → not only ---- but also.
Affirmative: Both Amit and Ajanta were dancing.

• It is too late to start a new chapter.


This sentence has just one clause and therefore it is a simple ing
Negative: Not only Amit but also Ajanta were dancing.
Rule 4: and (if join two words) → Replaced by → Not only -----

sentence. The number of clauses in a sentence is equal to the


number of finite verbs in it.
but also
Example:
.ne
Note: to-infinitives and –ing forms are not finite verbs.
The structure too…to can be replaced by the structure so…that.
t
Affirmative: She was beautiful and soft spoken.
Negative: She was not only beautiful but also soft
spoken.
Rule 5: Everyone/everybody/every person/ (every + common
• It is so late that we cannot start a new chapter.
noun)/all → Replaced by → There is no + attached word + but.
The given sentence has two clauses: one main clause (It is so late)
Example:
and one subordinate clause (That we cannot start a new lesson.)
Affirmative: Everyone loves sweet.
Therefore it is a complex sentence.
Transformation of sentences can be of the following types: Negative: There is no one but loves sweet.
• Affirmative to Negative Rule 6: As soon as → Replaced by → No sooner had ----- Than
• Negative to Affirmative Example:
• Assertive to Interrogative Affirmative: As soon as the students saw the teacher,
• Interrogative to Assertive they ran away.
• Exclamatory to Assertive Negative: No sooner had the students saw the teacher,
• Assertive to Exclamatory they ran away.
• Imperative to Assertive Rule 7: Absolute Superlative degree → Replaced by → No other +
• Assertive to Imperative attached word +so + positive form + as + subject

Downloaded From : www.EasyEngineering.net


Downloaded From : www.EasyEngineering.net

66  l  Transformation of Sentences

Example: Example:
Affirmative: Rajasthan is the biggest state in India. Assertive: She was very honest.
Negative: No other state is as big as Rajasthan in India. Interrogative: Wasn’t she very honest?
Rule 8: Sometimes affirmative sentences are changed into Affirmative: He is not a good human being.
Interrogative: Is he a good human being?
negative by using opposite words. Before the word, off course
Rule 2: No auxiliary verb in sentence →→ Change it by using →→
‘not’ is used.
Do/does/did Or Don’t/doesn’t/didn’t
Example:
Example:
Affirmative: I shall remember you. Assertive: He plays Basketball.
Negative: I shall not forget you. Interrogative: Does he play basketball?
Rule 9: Always → Replaced by → Never Rule 3: Never → Replaced by → Ever
Example: Example:
Affirmative: Rama always attends the class. Assertive: She never drinks tea.
Negative: Rama never misses the class. Interrogative: Does she ever drink tea?
Rule 10: Too ---- to → Replaced by → so ---that+ cannot/could Rule 4: Everybody/everyone/All → Replaced by → Who + Don’t/
Doesn’t/ Didn’t

ww
not (in past)
Example:
Affirmative: She is too weak to walk.
Example:
Assertive: Everyone wishes to be happy.

w.E
Negative: She is so weak that she cannot walk.
Rule 11: As – as → Replaced by → Not less – than
Example:
Interrogative: Who doesn’t wish to be happy?
Rule 5: Every + noun → Replaced by → is there any + noun+ Who
don’t/doesn’t/didn’t

asy
Affirmative: Sonam was as wise as Rhea.
Negative: Sonam was not less wise than Rhea.
Rule 12: Universal truths are change by making them negative
Example:
Assertive: Every person wishes to be happy.
Interrogative: Is there any person who doesn’t wish to be
interrogative.
Example: En happy?
Rule 6: No body/ no one / None → Replaced by → Who.
Affirmative: The Sun rises in the east.
Negative: Doesn’t the Sun rise in the east. gin Example:
Assertive: Nobody could count my love for you.
Rule 13: Sometimes → Replaced by → Not + always
Example: eer
Interrogative: Who could ever count my love for you?
Rule 7: There is no → Replaced by → Is there any/ Who (person)/
What (thing)
Affirmative: My son sometimes visits me.
Negative: My son doesn’t always visit me.
Rule 14: Many → Replaced by → Not a few
Example:
ing
Assertive: There is no use of this activity.
Example:
Affirmative: Aruna have many friends. .ne
Interrogative: What is the use of this activity?
Rule 8: It is no → Replaced by → Is there any/Why
Example:
Negative: Aruna do not have few friends.
Rule 15: A few → Replaced by → not many
Example:
Affirmative: Bhutan has a few scholars.
exam. t
Assertive: It is no use of taking unfair means in the

Interrogative: Why take unfair means in the exam?


Rule 9: It doesn’t matter → Replaced by → what though/ Does it
Negative: Bhutan doesn’t have many scholars. matter
Rule 16: Much → Replaced by → A little Example:
Example: Assertive: It does not matter if you fail in school.
Affirmative: He has much money. Interrogative: What though if you fail in school?
Negative: He doesn’t have a little money. Interrogative to assertive is to be done doing Vice versa.
Rule 17: A little → Replaced by → not much
Example: EXCLAMATORY TO ASSERTIVE
Affirmative: Drek has a little riches. Rule1: Subject and Verb of exclamatory sentence are to be used
Negative: Drek doesn’t have much riches. as the subject and verb of assertive sentence at the outset of the
sentence.
ASSERTIVE TO INTERROGATIVE How/what → Replace by → Very (before adjective)/ Great (before
Rule 1: If the sentence is in the affirmative you have to change noun)
it into negative interrogative. If it is in negative then you have to Exclamatory: How fortunate Aman is!
change it into bare interrogative. Assertive: Aman is very fortunate.

Downloaded From : www.EasyEngineering.net


Downloaded From : www.EasyEngineering.net

Transformation of Sentences  l 67

Rule 2: Sometimes the subject and verb may be eclipsed For positive use- No other + rest part after supr. Degree + verb +
Exclamatory: What a pity! so/as + positive form of adj/adv + as + sub.
Assertive: It is a great pity. Example:
Rule 3: Hurrah/ Bravo → Replace by → I/we rejoice that/ It is a Superlative: Simar is the tallest girl in the class.
matter of joy that Comparative: Simar is taller than any other girl in the
Exclamatory: Hurrah! India has won the game. class.
Assertive: It is a matter of joy that India has won the Positive: No other girl in the class is as tall as Simar.
game. Rule 2: If In superlative degree ‘One of the’ is transformed in
Rule 4: Alas → Replace by → I/we Mourn that/It is a matter of this way:
sorrow or grief that Comparative: Sub+verb+comp. form +than most other+ Rest
Exclamatory: Alas! She has failed in the examination. part.
Assertive: We mourn that she has failed in the examination. Positive: Very few+ rest part after supr. Degree + verb + so/as +
Rule 5: Had/were/If /would that (at the outset) → Replaced by → positive form of adj/adv + as + sub.
I wish + subject again + were/ had+ rest part. Example:
Example: Superlative: Tansen was one of the greatest poets in

ww
Exclamatory: Had I the wings of a peacock!
Assertive: I wish I had the wings of a peacock.
Assertive to exclamatory is to be done doing Vice versa.
Akbar’s court.
Comparative: Tansen was greater than most other poets
in Akbar’s court.

w.E
IMPERATIVE TO ASSERTIVE
Positive: Very few poets in Akbar’s court were so great
as Tansen.
Note: Superlative: Of all/ of any

Example: asy
Rule 1: Add subject + should in doing assertive Comparative: Than all other/than any other
Positive: It does not exist
Imperative: Do the homework.
Assertive: You should do the homework.
En Example:
Superlative: Mr. Wan is the oldest of all men in the
Rule 2: Please/kindly → Replaced by → you are requested to
Example:
Imperative: Kindly, grant me a leave.
gin society.
Comparative: Mr. Wan is older than all other men in the
society.
Assertive: You are requested to grant me a leave.
Rule 3: Do not → Replaced by → You should not eer
Positive: No other man is as old as Mr. Wan.
Rule 3: Simple comparative is transformed into positive by using
Example:
Imperative: Do not run in the afternoon sun. ing
(not so + adj/adv+as)/ (so+adj/adv+as) if negative. Second noun
or pronoun is used first.
Example:
Assertive: You should not run in the afternoon sun.
Rule 4: Never → Replaced by → you should never
.ne
Comparative: Rita is wiser than Mita.
Positive: Mita is not so wise as Rita.
Example:
Imperative: Never tell a lie.
Assertive: You should never tell a lie.
Rule 5: Let us → Replaced by → We should
as +adj/adv+ as
Example:
t
Rule 4: No/not less --- than is transformed into positive by using

Comparative: Karan is not less hard working than


Example: Suman.
Imperative: Let us go out for a picnic. Positive: Karan is as hard working as Suman.
Assertive: We should go out for a picnic.
Rule 6: Let + noun/pronoun → Replaced by → Subject + might
COMPLEX →→→ SIMPLE →→→ COMPOUND
Example: Rule 1: S ince/As/When Change is to be made in the subordinate
Imperative: Let him play video game. clause
Assertive: He might play video game. When subjects are same.
Simple: 1) Omit since/as/when
CHANGE OF DEGREE (2) (Verb+ing) of the subordinate clause
(3) Then write the rest part
Rule1: If the superlative degree says about the best thing then
(4) Use subject with the principal clause
the rule is: (5) Principal clause remains unchanged
For comparative use – subject +verb + adjective/adverb (comp. Example: Since the man worked hard, he made a good result.
form) + Than any other + rest part Simple: Working hard, the man made a good result.

Downloaded From : www.EasyEngineering.net


Downloaded From : www.EasyEngineering.net

68  l  Transformation of Sentences

Compound: 1) Omit since /as /when Rule 7: Simple: So that is replaced by to/in order to.
2) Write down the rest part. Example:
3) Join clauses by using and, and so, and therefore Compound: Ankit works hard so that he may prosper in
4) Write the main clause unchanged.
life.
Example: The man worked hard and made a good result.
Simple: Ankit works hard to/in order to prosper in life.
Rule 2: In case of ‘Be’ verb in subordinate clause:
Compound: “and want/wants to” is used to join two
(1) Use being/ Because of + Pronoun/noun (possessive form) +
being. clauses.
Example: Ankit works hard and wants to prosper in life.
Example: Since she was unwell, she could not work hard.
Rule 8: Simple: ‘so + adjective + that’ is replaced by ‘Too +
Simple: Because of her being unwell, she could not work hard.
adjective + to’
Compound: She was unwell and therefore could not work hard.
Example: The girl is so dumb that she cannot understand
Note: and therefore, is used for showing reasons.
the joke.
Rule 3: When the subjects of clauses are different:
Simple: The girl is too dumb to understand the joke.
Simple:
Compound: Use ‘And Therefore’ to make it a compound

ww
(1) Subject of subordinate clause
(2) Verb+ing (be verb → being; Have verb → having)
Example:
sentence.
Example: She is so dumb and therefore cannot

w.E
Since the weather was rainy, he did not go out.
Simple: The weather being rainy, he did not go out.
understand the joke.
Rule 9: When (if mentions time) is replaced by:
For short time – At
Compound: use ‘and therefore’ to join two clauses.
Example: asy For month or Season – In
For age--- at the age of.
The weather was rainy and therefore he did not go out.

Rule 4: If, is Replaced by – ‘by + (verb+ing) En Example: He woke up when it was early morning.
Simple: He woke up at early morning.
Example:
If you play safe, you will win the game. gin Complex: When it is spring, the flower blossoms.
Simple: In Spring the flower blossoms.
Simple: By playing safe, you will win the game.
Compound: 1.) Omit if+subject eer
Compound: When Sam was one he went to pre-school.
Simple: At the age of one, Sam went to pre-school.

2.) use ‘and’ to join two clauses.


Example: Compound: Play safe and you will win the
ing
Compound: Use and to join clauses.
Example: She woke up and it was midnight.

game.
Rule 5: Simple: If-not/unless, is replaced by, without+ (verb+ing)
use----
.ne
Rule 10: Simple: If the clause says about a continuous fact then

Example:
Complex: If you do not work hard, you will fail in life.
Simple: Without working hard, you will fail in life.
At the time of instead of ‘When’
Example: When I was studying the bell rang.
t
Simple: At the time of my studying, the bell rang.
Compound: Use ‘And’.
Example: I was studying and the bell rang.
Compound: Use or/otherwise to join two clauses. Rule 11: Simple: Noun clause can be replaced by noun.
Example: Work hard or you will fail in life. Example:
Rule 6: Simple: ‘Though’ is replaced by in spite of + Possessive Compound: She admitted that she was disloyal.
Simple: She admitted her disloyalty.
form of the subject + (verb+ing)
Compound: That she is disloyal is known to all.
Example:
Simple: Her disloyalty is known to all.
Complex: Though she tried her best, she could not pass Compound: Use ‘And’
in examination. Example: He is honest and it is known to all.
Simple: In spite of trying her best she could not pass in Rule 12: Simple: If Complex sentence is made with relative
pronoun (who, what, which, that), omit it and make (verb+ing).
examination.
Compound: The architect who is working in the site is
Compound: Use ‘but’ to join two clauses. known to all.
Example: She tried but could not pass in examination. Simple: The architect working in the site is known to all.

Downloaded From : www.EasyEngineering.net


Downloaded From : www.EasyEngineering.net

Transformation of Sentences  l 69

Note: If the verb is in the past participle it remains unchanged. Compound: He founded the solution and it was
Example: The canvas which was drawn by Hussain is incorrect.
very creative. Gerundial Infinitive
Simple: The canvas drawn by Hussain is very creative. Example: Aditya have no money that he can lend you.
Rule 13: Simple: Adjective Clause is changed into ---- Adjective, Simple: Aditya have no money to lend you.
Past participle Phrase, Noun in apposition, infinitive. Compound: Aditya have no money and he cannot lend
Adjective you.
Example: A woman who is drowning catches at a straw. Rule 14: In the compound “not only---- but also” is Changed by
Simple: A drowning woman catches at a straw.
“Besides + (Verb +ing)”
Compound: A woman is drowning and so catches at a
straw. Example: Mrs Rafia not only teaches us Science but also
Past Participle Phrase writes journals.
Example: The solution that he found was incorrect. Simple: Besides teaching us Science, Mrs. Rafia writes
Simple: The solution founded by him was incorrect. journals.

ww
w.E
asy
En
gin
eer
ing
.ne
t

Downloaded From : www.EasyEngineering.net


Downloaded From : www.EasyEngineering.net

70  l  Transformation of Sentences

Practice Exercise
LEVEL-I
DIRECTIONS (Qs. 1-30): Answer the following questions by (a) The chocolates vary in their sweetness.
choosing the most appropriate option. (b) This chocolate is sweeter than that chocolate.
1. Transform the given sentence into assertive. (c) This chocolate is not sweeter than that chocolate.
‘How gorgeous is the sunset!’ (d) This chocolate is sweeter than that one.
(a) How is the sunset so gorgeous? 9. Transform the given assertive sentence into interrogative.
(b) The sunset is gorgeous, isn’t it? ‘Everybody has heard of Amitabh Bachchan.’
(c) The sunset is very gorgeous. (a) Has everybody heard of Amitabh Bachchan?
(d) How gorgeous the sunset is! (b) Isn’t that everybody has heard of Amitabh Bachchan?
2. Transform the given sentence into superlative. (c) Everybody has heard of Amitabh Bachchan, isn’t it?
(d) Amitabh Bachchan is known to all, isn’t it?

ww
‘No other metal is as expensive as gold’.
(a) No other metal except gold is expensive.
(b) Gold is expensive of all the metals.
10. Transform the given sentence into complex.
‘Her looks proclaim her innocence’.

w.E
(c) Isn’t gold the most expensive of all metals.
(d) Gold is the most expensive of all metals.
3. Transform the given sentence into simple.
(a) Her looks proclaim that she is innocent.
(b) She is innocent, her looks proclaim.
(c) Her innocence is seen on her looks.

‘She said that she was coming’.


(a) She will come today. asy 11.
(d) She is innocent, proclaims her looks.
Transform the given sentence and change into noun.
‘Parul sympathises with the needy and poor’.
(b) She is coming.
(c) She told of her coming.
En (a) Parul is sympathetic towards needy and poor.
(b) Parul treats needy and poor with sympathy.
(d) She told that she was coming.
4. Transform the given sentence into adjective.
‘She treated me respectfully’.
gin
12.
(c) Parul has sympathy for the needy and poor.
(d) Needy and poor are sympathised by Parul.
Transform the given sentence by changing into verb.
(a) She treated me with respect.
(b) She respectfully treats me. eer
‘The examination result gave her pleasure’.
(a) The examination result pleased her.
(c) She was respectful to me.
(d) She treats me respect. ing
(b) Her examination result gave her pleasure.
(c) She was pleased by her examination result.
5. Transform the given sentence into negative.
‘Human is mortal’. 13.
.ne
(d) She was pleased to see her examination result.
Transform the given sentence into interrogative.
‘A great deal of research has been done on the Science
(a) Human is immortal.
(b) Human is not immortal.
(c) Not all humans are mortal.
(d) Not all humans are immortal.
project’.
t
(a) Has much research been done on the science project?
(b) Has the Science project undergone research?
6. Transform the given sentence into complex. (c) The Science project is thoroughly researched, isn’t it?
‘My ambition is to serve the country’. (d) Much research has been done on the science project,
(a) My ambition is that I should serve my country. isn’t it?
14. Transform the given sentence into complex sentences.
(b) My ambition is that I shall serve my country.
‘We have informed her of her success’.
(c) To serve my country is my ambition.
(a) We have informed her of her being successful.
(d) I serve my country is my great ambition.
(b) She has been informed by us of her success.
7. Transform the given sentence into negative.
(c) We have informed her that she has succeeded.
‘Many people want to travel the world’.
(d) She has been informed that she has succeeded.
(a) Many people want to travel the world, isn’t it? 15. Transform the given sentence into compound sentence.
(b) Many people don’t want to travel the world. ‘Everyone knows that the earth is round.’
(c) Don’t many people want to travel the world? (a) The earth is round and everyone knows it.
(d) Not many people want to travel the world. (b) Everyone knows that earth is circular in shape.
8. Transform the given sentence into comparative. (c) The Earth is round is a known fact.
‘That chocolate is not as sweet as this one’ (d) The Earth is round is known to everyone.

Downloaded From : www.EasyEngineering.net


Downloaded From : www.EasyEngineering.net

Transformation of Sentences  l 71

16. Which of the transformations of the sentence –‘Rajdhani 24. Transform the given compound sentence into complex
Express is the fastest train’ is correctly changed into positive sentence.
sentence? ‘Cancer is curable and everybody knows this’.
(a) There is no other train as fast as Rajdhani Express.
(a) Cancer is curable is known to all.
(b) No other train is as fast as Rajdhani Express.
(b) Cancer is curable is a known fact.
(c) Both (a) and (b)
(c) Everybody knows that cancer is curable.
(d) None of these.
17. Transform the given sentence by removing ‘too’. (d) Everybody knows that cancer can be cured.
‘This tea is too hot for me’. 25. Transform the given complex sentence into compound
(a) This tea is so hot that I cannot take it. sentence.
(b) This tea is very hot for me. ‘As soon as we heard the shout, we rushed to the spot’.
(c) This tea is hot for me. (a) We heard the shout and rushed to the spot.
(d) This tea is hot that I cannot take it (b) Hearing the shout, we rushed to the spot.
18. Transform the given sentence into compound. (c) We heard the shout and rushed.
‘Coming to me, Anita delivered the letter’.

ww
(a) Anita came to me and delivered the message.
(b) Anita delivered the letter while she was coming to me.
26.
(d) We rushed to the spot as soon as we heard the shout.
Transform the given compound sentence into complex
sentence.

w.E
(c) Anita came to deliver the message.
(d) Anita came to me to deliver the message.
19. Transform the given sentence into complex.
‘You have to be 18 years old or you can’t drive’.
(a) You can’t drive because you are under 18.

‘Buy one shirt and get one free’.


asy
(a) If you buy one shirt, you get another shirt free.
(b) To drive, you have to be 18 years old.
(c) If you are under 18, you can’t drive.
(d) If you are 18 year old, you can’t drive.
(b) If you buy one shirt, you get one free.
(c) A shirt free on purchase of a shirt.
En 27. Transform the given exclamatory sentence into assertive
sentence:
(d) Buy one shirt to get one free.
20. Transform the given sentence into complex.
‘Listen and I will give you all’.
gin ‘How kind of you to help her like that’.
(a) How kind of you that you helped her like that.
(a) If you listen, I will tell you all.
(b) Listen and know all. eer
(b) It is very kind of you to help her.
(c) You are very kind to help her like that.
(c) Listen carefully to know all.
(d) If you listen to me, I will tell you everything. 28. ing
(d) It is very kind of you to help her like that.
Transform the given complex sentence into compound
21. Transform the given sentence by removing ‘too’.
‘The oranges are too cheap to be good.’
sentence.
.ne
‘Although he is rice, he is not happy’.
(a) The oranges are cheap and good.
(b) The oranges are cheap as well as good.
(c) The oranges are very cheap good.
(d) The oranges are so cheap that they cannot be good.
(a) He isn’t happy but he is rich.
(b) He is rich and still not happy.
(c) He is rich but he is not happy.
t
(d) He is rich but not happy.
22. Transform the given complex sentence into compound
29. Transform the given sentence into negative.
sentence.
‘If you cannot obey me you have to leave’. ‘Ajay loved his parents’.
(a) If you don’t obey me, you will have to leave. (a) Ajay hates his parents.
(b) Obey or leave. (b) Ajay did not love his parents.
(c) Either obey me or leave. (c) Ajay does not love his parents.
(d) Obey me or leave. (d) Ajay did not hate his parents.
23. Transform the given sentence into interrogative. 30. Transform the given sentence into positive.
‘Stop it’. ‘Samay writes more neatly than any other boy in the class’.
(a) Will you stop it? (a) No other boy of Samay’s class writes as neatly as he.
(b) Please stop it. (b) Samay writes neatly than all his classmates.
(c) Can you stop it? (c) Samay’s writing is very neat.
(d) Please stop. (d) Samay’s writing is neater than all the boys in his class.

Downloaded From : www.EasyEngineering.net


Downloaded From : www.EasyEngineering.net

72  l  Transformation of Sentences

LEVEL-II
DIRECTIONS (Qs. 1-10): Combine each of the following pairs of DIRECTIONS (Qs. 11-20): Identify the following sentences or
sentences using infinitives. clauses from the given options.
1. They wanted to win. They worked hard for that reason. 11. We can wait here until Sourav calls us.
(a) They worked hard to win. (a) compound (b) simple
(b) They worked hard for that reason to win. (c) complex (d) none of the above
(c) To win they worked hard. 12. Either the engine starts, or we will remain here all night.
(d) They wanted to win and worked hard (a) compound (b) simple
2. She was sensible. She did the right thing. (c) complex (d) none of the above
(a) She was sensible to did the right thing. 13. A group of my classmates studied the problem and solved it
(b) She did the right thing to be sensible
in a few minutes.
(c) She was sensible enough to do the right thing.
(a) compound (b) simple
(d) To be sensible she did right thing.
3. The students were not able to solve the problem. It was too (c) complex (d) none of the above
difficult for them. 14. The old car was in fairly good operating condition, but the
condition of the body was poor.

ww
(a) It was too difficult to solve the problem.
(b) To the students it was too difficult to solve the problem.
(c) The problem for the student was too difficult to solve.
(a) compound
(c) complex
(b) simple
(d) none of the above

w.E
(d) The problem was too difficult for the students to solve.
4. It is too late. We can’t start a new lesson now.
(a) It is too late for us to start a new lesson.
(b) It is too late that can’t start a new lesson now.
15. The small streams that run through the area are loaded with
fish.
(a) adverb clause (b) adjective clause

asy
(c) It is getting late to start a new lesson.
(d) We are too late starting a new lesson.
(c) noun clause (d) none of the above
16. When the leaves begin to fall, we will harvest the last of our
crops.
5. The manager appointed John. He will do the accounts.

En
(a) He will do the accounts the manager appointed John.
(b) The manager appointed James to do the accounts.
(a) adverb clause
(c) noun clause
(b) adjective clause
(d) none of the above
(c) The manager appointed to John he will do the accounts
(d) Being appointed by manager John will do the accounts. gin
17. Your sister said you haven’t slept in two or three nights.
(a) adverb clause
(c) noun clause
(b) adjective clause
(d) none of the above
6. We went to the theater. We saw a movie there.
(a) We went to the theater seeing a movie.
(b) We saw a movie there went to the theater. eer
18. People who have poor diets are likely to catch colds.
(a) adverb clause (b) adjective clause
(c) To seeing a movie we went to the theatre.
(d) We went to the theater to see a movie.
7. I have no interest in politics. I must tell it frankly.
ing
(c) noun clause (d) none of the above
19. The crew checked the ship and prepared it for sea.
(a) compound (b) `simple
(a) I have no interest in politics to tell it frankly
(b) I must tell it frankly to no interest in politics
(c) complex
.ne
(d) none of the above
20. We must prepare the dam, or the water will rush through.
(c) To be frank, I have no interest in politics.
(d) I must tell it frankly to having no interest in politics.
8. The man took out a knife. He intended to frighten all of us.
(a) The man took out a knife to frighten all of us
(a) compound
(c) complex
(b) simple
t
(d) none of the above
DIRECTIONS: (Qs. 21-24): Transform the simple sentence into
(b) He intended to frighten all of us took out a knife a compound sentence.
(c) To frighten all of us he took knife. 21. He must work hard to make up for the lost time.
(d) The man took out a knife to intend to frighten all of us. (a) He should make up the lost of time working hard.
9. Mary collects old stamps. It is her hobby. (b) He worked hard and make up the lost time.
(a) It is her hobby to collect stamps. (c) Working hard he can make up the lost time
(b) It is Mary’s hobby to collect old stamps. (d) He has to make up the lost time working hard.
(c) Mary collects old stamps it is her hobby. 22. To his eternal disgrace, he betrayed his country.
(d) Marry’s old habit is collecting stamps. (a) He betrayed his country is an eternal disgrace.
10. The men held a meeting. They wanted to elect a manager for
(b) For him betraying his country was an eternal disgrace
the company.
(a) They wanted to elect a manager for the company, the (c) He betrayed his country and this was to his eternal
men held a meeting. disgrace.
(b) They wanted a manager to held a meeting. (d) Eternal disgrace was his betraying the country.
(c) The men held a meeting to elect a manager for the 23. Besides robbing the poor child, he also murdered the child.
company. (a) He not only robbed the poor child and also murdered
(d) A meeting was held to elect a meeting. the child.

Downloaded From : www.EasyEngineering.net


Downloaded From : www.EasyEngineering.net

Transformation of Sentences  l 73

(b) He robbed the child and murdered the child. DIRECTIONS (Qs. No. 27-30): Transform the following without
(c) The poor child was robbed and murdered by him. changing the meaning.
(d) After robbing the poor child he murdered him. 27. He gave a curt reply.
24. The teacher punished the children for disobedience. (a) His reply was curtly.
(a) For disobedience the teacher punished the children. (b) He always gives replies in a curtly manner.
(b) The teacher punished the children for their (c) Curtly replies are his habbit.
disobedience. (d) He replied curtly.
(c) The children were disobedience so the teacher 28. I see him everyday.
punished them. (a) I see him always.
(d) Being disobedient the teacher punished the students. (b) I see him daily.
(c) I see him all the time.
DIRECTIONS (Qs. No. 25-26): Change one part of a sentence (d) I see him every time.
for another part without changing the meaning. 29. This scene is surpassingly beautiful.
25. This kind of jokes never amuses me. (a) Surpassingly beautiful is the scene.
(a) This kind of jokes are always amusing. (b) The beauty of this scene is surpassing.
(c) How surpassingly the beautiful scene.

ww
(b) I am not amused by this kind of jokes.
(c) This kind of joke never gives me any amusement.
(d) Always amusing this kind of jokes.
(d) Surpassing beautiful the scene is.
30. He is admittedly the greatest general of this country.

w.E
26. It costs twelve dollars.
(a) Its cost is twelve dollars.
(b) The cost is twelve dollars.
(a) This is admitted he is the greatest general of this
country
(b) This country’s greatest general he is.
(c) It has been admitted that he is the greatest general of
(c) Twelve dollar is the cost of it.
asy
(d) The cost stands at twelve dollars. this country.
(d) Admittedly the greatest general of this country he is.

En
gin
eer
ing
.ne
t

Downloaded From : www.EasyEngineering.net


Downloaded From : www.EasyEngineering.net

74  l  Transformation of Sentences

Hints & Solutions


LEVEL- I 7. (c) An infinitive phrase will begin with an infinitive [to +
Simple form of the verb]. It will include objects and/or
1. (c) The sunset is very gorgeous. modifiers.
2. (d) Gold is the most expensive of all metals. 8. (a) to infinitive +verb is the correct way to combine the two
3. (c) She told of her coming. sentences.
4. (c) She was respectful to me. 9. (b) to infinitive + verb combines the sentences.
5. (b) Human is not immortal. 10. (c) to + verb infinitive form in a right way combines the two
6. (a) My ambition is that I should serve my country. sentences.
7. (d) Not many people want to travel the world. 11. (c) This sentence is complex because it contains one
8. (d) This chocolate is sweeter than that one. independent clause, we can wait here, and at least one
9. (a) Has everybody heard of Amitabh Bachchan. dependent clause, until Sourav calls us.
10.
11. ww
(a) Her looks proclaim that she is innocent.
(c) Parul has sympathy for the needy and poor.
12. (a) This sentence is compound because it contains two
independent clauses-Either the engine starts and we will
remain here all night and no dependent clauses.
12.
13.
14.
w.E
(a) The examination result pleased her.
(a) Has much research been done on the science project?
(c) We have informed her that she has succeeded.
13.

14.
(b) This sentence contains one independent clause and no
dependent clause, so it is a simple sentence.
(a) This sentence is compound because it contains two
15.
16. asy
(a) The earth is round and everyone knows it.
(c) Both a and b are correctly transformed simple sentence 15.
independent clauses and no dependent clauses.
(b) The dependent clause that run through the area works as

17.
into positive sentence.
(a) This tea is so hot that I cannot take it.
En 16.
an adjective, describing the small streams.
(a) The dependent clause here is When the leaves begin to
18.
19.
(a) Anita came to me and delivered the message.
(b) If you buy one shirt you get one free. gin
17.
fall. It works as an adverb, modifying the verb harvest.
(c) The dependent clause in two or three nights works as a
noun.
20.
21.
(a) If you listen I will tell you all.
(d) The oranges are so cheap that they cannot be good. 18.
eer
(b) The dependent clause who have poor diets works as an
adjective. It modifies the noun People.
22.
23.
24.
(d) Obey me or leave.
(a) Will you stop it?
(c) Everybody knows that cancer is curable.
19.
ing
(b) This sentence contains one independent clause and no
dependent clauses, so it is simple.

25.
26.
(a) We heard the shout and rushed to the spot
(c) If you are under 18 you can’t drive
20.

21.
.ne
(a) This is a compound sentence because it contains two
independent clauses and no dependent clauses.
(b) This sentence can be made into two parts and those two
27.
28.
29.
(d) It is very kind of you to help her like that.
(c) He is rich but he is not happy.
(d) Ajay did not hate his parents.
22.
parts can be joined by a conjunction ‘and’.
t
(c) By enlarging phrase or word into a co-ordinate clause
this sentence has been transformed.
30. (a) No other boy of Samay’s class writes as neatly as he. 23. (a) not only but also is used to enlarge the sentence and
makes a compound sentence.
LEVEL- II 24. (c) were and so are added to the coordinating clause.
25. (c) In this sentence the verb has been changed into its noun
1. (a) to + infinitive (win-verb)
form.
2. (c) to +infinitive verb form
26. (a) Here also the verb has been changed into its noun form.
3. (d) to solve is used as to+ solve (verb) in the infinitive form. 27. (d) Here the adjective has been changed into an adverb.
4. (a) ‘for us to start’ the infinitive clause rightly combines the 28. (b) Everyday correctly replaces daily.
two sentences. 29. (b) Here the adjective has been changed into its noun form.
5. (b) to + verb in infinitive form is the correct use 30. (c) Here, the adverb has been changed into a verb form.
6. (d) to + verb clause in infinitive form combines the two And the simple sentence itself has been changed into a
sentences in a right way. compound sentence.

Downloaded From : www.EasyEngineering.net


Downloaded From : www.EasyEngineering.net

7
PTER
CHA

Idioms and Phrases



ww
Some Commonly Used Idioms & Phrases
Beat back (to compel to retire) : The firemen were beaten back by angry flames and the building was reduced to ashes.
Boil down to (to amount to) : His entire argument boiled down to this that he would not join the movement unless he saw some



w.E
monetary gain in it.
Cast aside (to reject, to throw aside) : Men will cast aside truth and honesty for immediate gains.
Cry down (to deprecate) : Some of the Western powers did their best to cry down India’s success in the war.


that he cut him off with a shilling.
asy
To cut off with a shilling (to give someone a mere trifle in the will) : The father was so angry with the son over his marriage

Egg on (to urge on) : Who egged you on to fight a professional boxer and get your nose knocked off?

• En
Gloss over (explain away) : Even if you are an important person your faults cannot be glossed over.
To laugh in one’s sleeves (to be secretly amused) : While I was solemnly reading my research paper to the audience, my friends


were laughing in their sleeves for they knew what it was worth.
gin
Play off (to set one party against another for one’s own advantage) : It best serves the interests of the super powers to play off



one poor nation against another.

eer
Pull one through (to recover, to help one recover) : Armed with the latest medicines, the doctor will pull him through.
Cost a slur upon (by word or act to cast a slight reproach on someone) : Many a man casts a slur on his own good name with


some mean act.
ing
To catch a Tartar (to encounter a strong adversary) : When Hitler marched in to Russia he little knew that he would catch a


Tartar in the tough people of that country.

.ne
To come off with flying colours (to come out of a conflict with brilliant success) : The 1971 election outcome was uncertain but
finally the congress came off with flying colours.


best. t
To come off second best (to be defeated in every contest) : Be it an election or a tambola, I have always come off the second

To cut the Gordian knot (to remove a difficulty by bold or unusual measures) : The Parliament threw out the Bill for Abolition
of Privy Purses. The Government cut the Gordian knot by abolishing the privy purses through an ordinance.
• To fall to one’s lot (to become one’s fate) : It fell to the lot of Mujib and his colleagues to reconstruct the shattered economy of
their nation.
• To get into hot water (to get into difficulty) : The businessman got into hot water with the Income-tax authorities for concealing
his income from ancestral property.
• To give someone the slip (to dodge someone who is looking for you) : The police had nearly got the dacoits when the latter
gave them the slip in the Chambal ravines.
• To go on a fool’s errand (to go on an expedition which leads to a foolish end) : Many people earlier believed that going to the
moon was like going on a fool’s errand
• To go to the wall (to get the worst in a competition) : In the struggle of life, the weakest goes to the wall.
• To go to rack and ruin, to go to the dogs (to be ruined) : If a big war comes, our economy will go to the dogs.
• To have one’s hands full (to be very busy) : Pakistan could hardly expect active help from the U.S.A. as her hands were already
full with Vietnam, Laos and West Asia problems.
• To have a bone to pick with one (to have a difference with a person which has not yet been fully expressed) : The extreme leftists
have a bone to pick with the police and if ever they come to power there may be unpleasantness between the two.

Downloaded From : www.EasyEngineering.net


Downloaded From : www.EasyEngineering.net

76  l  Idioms and Phrases

• To have the whip hand of (to have mastery over) : After the split in the party Mrs. Gandhi has the whip hand of the Congress.
• To have too many irons in the fire (to have so much work in hand that some part of it is left undone or is done very badly) : Let
the Government not go in for nationalisation so fast. If they have too many irons in the fire they are bound to fare badly.
• To have the tree or right ring (To be genuine) : Nixon’s pronouncements on world peace do not have the right ring.
• To have two strings to one’s bow (to have an alternative means of achieving one’s purpose) : A wife always has two strings to
her bow if coaxing fails to achieve the desired end; tears succeed.
• To have an axe to grind (have personal interests to serve) : Bigger nations supply arms to the smaller ones primarily because
they (the bigger nations) have their own axe to grind
• To keep the wolf from the door (to keep away extreme poverty and hunger) : Lakhs in India have to struggle everyday to keep
the wolf from the door.
• To make short work of (to bring to sudden end) : The locusts made short work of the ripe standing corn.
• To make amends for (to compensate for damage) : By his kindness today he has made amends pr his past insolence.
• To make common cause with (to unite, to co-operate with) : During the last elections the princes made a common cause with
the rightist parties. Both went down.
• To make a virtue of necessity (to do a very disagreeable thing as though from duty but really because you must do it) : When a
minister knows that he is going to be booted out of the cabinet he makes a virtue of necessity and resigns on health grounds.

ww
To make much ado about nothing (make a great fuss about a trifle) : Demonstrations and protests over the change in the timing
of news bulletins over AIR was making much ado about nothing.


w.E
To make a cat’s paw or a tool of someone (to use someone as a means of attaining your object) : The super-powers have made
a cat’s paw of the smaller nations of Asia in their game of power politics.
To play into the hands of someone (to act as to be of advantage to another) : By raising the slogan ‘Indira Hatao’ the opposition


asy
played into her hands and Mrs. Gandhi won the elections hands down (easily).
To play second fiddle’ (to take a subordinate part) : With Mrs. Gandhi as the undisputed leader of the Congress and the nation,
everyone else is content to play second fiddle to her.

En
To put the cart before the horse (to begin at the wrong end to do a thing) : Preparing the blue print of a project without the
provision of funds is like putting the cart before the horse.

morass; we have to put our own shoulders to the wheel. gin
To put one’s shoulder to the wheel (to make great efforts ourselves) : No amount of foreign aid will pull us out of the economic



• eer
To set store by (to value highly) : India, surely sets much store by the Indo Soviet Treaty of Friendship.
To set the Thames on fire (to do something extraordinary) : He is a steady worker but never likely to set the Thames on fire.
To set one’s house in order (to arrange one’s affairs) : Let Pakistan set her own house in order before talking of the welfare of


the Kashmiris.
ing
To take into one’s head (to occur to someone) : The Manager look it into his head that by shutting off the electricity for a few


hours daily he could save on refrigeration costs.
.ne
To take the bull by the horns (to grapple with a problem courageously instead of avoiding it) : There is no short cut to prosperity.


We have to take the bull by the horns and make people work like slaves.

going into partnership with that man. t


To take a leap in the dark (to do a hazardous thing without any idea of what it may result in) : You took a leap in the dark in

To throw cold water upon (to discourage something) : The doctor threw cold water upon my plans for a world tour by declaring
that I could never stand the strain of it.
• To throw up the sponge (to give up a contest) : Faced with stiff competition from big companies, many a small company will
throw up the sponge.
• To turn over a new leaf (to change one’s course of action completely) : After a long career of crime the convict suddenly turned
over a new leaf and became a model citizen.
• To turn tail (to retreat ignominiously) : The enemy turned tail in the face of heavy onslaughts on its key positions.
• To turn the tables (to reverse someone’s success or superiority) : Pakistan started war with a blitz on our positions but the
superior tactics of our Armed Forces soon turned the tables on them.
• To cook or doctor an account (to tamper with or falsify the account) : From the balance sheet presented to the shareholders,
the company seemed to be flourishing, but it afterwards turned out that the Secretary had cooked the accounts.
• To bear the brunt of (to endure the main force or shock of) : The infantry has to bear the brunt of a battle.
• To beard the lion in his den (to oppose someone, in his stronghold) : The Indian Army broke through strong Pakistani fortifications
in the Shakargarh area and bearded the lion in his own den.
• To bid fair to (to give fair prospect of) : His health is so good that he bids fair to live till he is sixty.

Downloaded From : www.EasyEngineering.net


Downloaded From : www.EasyEngineering.net

Idioms and Phrases  l 77

• To blow one’s own trumpet (to parade one’s own good deeds) : Modesty does not pay. Only if you blow your own trumpet, you
can succeed.
• To blunt the edge of (to make something less effective) : Time blunts the edge of grief.
• To build castles in the air (to indulge in reveries or visionary schemes) : There is nothing wrong if you build castles in the air;
now put foundations under them.
• To burn the candle at both ends (to use too much energy) : Our resources are limited. Let us use them judiciously and not burn
the candle at both ends.
• To buy a pig in a poke (to purchase a thing without previously examining it) : Buying shares in a new Company started by
unknown entrepreneurs is like buying a pig in a poke.
• To cross or pass the Rubicon (to take a decisive step forward) : The Government will have to think of many things before
nationalising the textile industry for once they cross the Rubicon there will be no going back.
• To cry over spilt milk (to nurse unnecessary regrets) : We have failed to build up a sizeable total against England’s meagre first
innings total. It is no use crying over spilt milk now.
• To err on the safe side (to choose a course which may in fact be inaccurate, but which will keep you safe from risk or harm) :
In going in for mixed economy rather than wholesale nationalisation the Government were erring on the safe side.

• ww
To flog a dead horse (waste one’s energies) : We are flogging a dead horse if we are trying to make Sanskrit the national language
of India.
To feather one’s nest (to provide for oneself through dishonest means) : Many tax collectors make a point of feathering their



w.E
own nests well while they have opportunity.
To Eat one’s heart out (to brood over one’s sorrows or disappointments) : Don’t eat your heart out over failure in this competition.
To eat humble pie (to have to humiliate oneself) : Since none came to his support he had to eat humble pie and give in to their



demands.
asy
To eat one’s words (to retract one’s assertions under compulsion) : It is hard for a haughty man to have to eat his words.
To throw down the gauntlet, to take up the gauntlet (to offer or give a challenge, to accept a challenge) : It is not for a small

• En
country to throw down the gauntlet to the right and the left.
To run the gauntlet (to undergo severe criticism or ill treatment) : Most trend-setting books have to run the gauntlet of the


literary critics.
gin
To burn one’s fingers (to get oneself into unexpected trouble) : They were happily placed in the woollen industry. But they went


in for cosmetics and burnt their fingers.
eer
To force one’s hands (to compel one to do something unwillingly or earlier than he wished to do it) : The Government wanted


ing
to do all that they could to meet the workers’ demands. But the violence by the strikers forced their hands to declare a lockout.
To haul over the coals (to scold a man, reprove him) : If your bad habits become known, you will get hauled over the coals and
richly deserve it.

.ne
To let the grass grow under your feet (to be inert and passive to things around) : The authorities should listen to students’
grievances. By being indifferent they would only let the grass grow under their feet till it will be too late to turn these young


people take away from the path of violence.

in a nutshell be is insane. The explanation of his conduct can be put in a nutshell - he is insane.
t
To put in a nutshell (this is said of a thing which is capable, of, or presented in, brief expression) : His conduct is weird. To put

To let loose the dogs of war (to set in motion the destructive forces of war) : Pakistan has let loose the dogs of war in Kashmir,
through organised terrorism.
• To lord it over someone (to domineer over someone, to act as a lord) : The love of power is so strong in human nature, that
when a man becomes popular he seeks to lord it over his fellows.
• To mind one’s Ps and Qs (to be punctilious) : The manager suspects his chief clerk of dishonesty, and if the clerk does not mind
his Ps and Qs, he will soon find himself without a job.
• To muster in force (to assemble in large numbers) : The citizens mustered in force to welcome their beloved leader.
• To pay one back in one’s own coin (to give tit for tat, to retaliate) : Howsoever revengeful you may be, unless you are strong
enough you cannot pay him back in his own coin.
• To plough a lonely furrow (to work without help or support) : In the organised society of today no individual or nation can plough
a lonely furrow.
• To poison the ears or mind (to prejudice another person) : A judge must not allow anyone to poison his mind against either the
plaintiff or the defendant.
• To rest on one’s laurels (to rest satisfied with honours already won, and to make no attempt to gain further distinction) : Even
if he wins the biggest award, a film star will never rest on his laurels. He will try to rise higher and higher.

Downloaded From : www.EasyEngineering.net


Downloaded From : www.EasyEngineering.net

78  l  Idioms and Phrases

• To rest on one’s oars (to suspend efforts after something has been attained) : The agitators have been vigorously at work during
the winter, but at present they seem to be resting on their oars.
• To harp on the same string (to keep repeating the same sentiment over and again) : This gentleman keeps harping on the same
string: he is from Oxford and deserves this and deserves that etc.
• To rise like a phoenix from its ashes (the phoenix was a fabulous Arabian bird. It had no mate but when about to die, made a
funeral pile of wood and aromatic gums and on it burned itself to ashes. From the ashes a young phoenix was believed to rise)
: Germany was completely decimated in the Second World War. But she has risen like a phoenix from its ashes.
• To rule the roast or roost (to lord it over others in a party or group) : In almost every party there is some overbearing person
who tries to rule the roost.
• To run in the same groove (to move forward on the same path, to advance in harmony) : It is clear that the ideas of both reformers
run in the same groove.
• To run in the blood (a peculiarity which clings to certain families) : Snobbery runs in the blood of the Englishmen.
• To scatter to the winds (to waste, to scatter abroad) : We have scattered to the winds what we had gained by our independence.
• To be on the right scent (to be on the right track) : The customs have decided to patrol the Kerala seas to nab smugglers from
Dubai. They are on the right scent (Its opposite is to be on the wrong scent or wrong track)
• To see how the wind blows (to observe what influence, favourable or adverse, is likely to affect the existing state of things) : In

• ww
party-politics people sitting on the fence keep on watching how the wind is blowing before deciding on their options.
To see a thing through coloured glasses (to regard something favourably because of one’s prejudice) : Pakistan has for long

• w.E
looked at India through coloured glasses and never trusted even the most genuine gestures for peace. (The world is a place of
strife and one should not see it through coloured glasses.)
To show the white feather (to show signs of cowardice) : The agitators shouted and gesticulated but the moment the police
appeared on the scene they seemed to show the white feather.

• asy
To sow broadcast (to scatter widely or without stint) : The emissaries of the banished king were sowing sedition broadcast.
To split hairs (to make subtle and useless distinctions) : As the drought played havoc in Bihar, the authorities were busy splitting


En
hairs trying to decide whether it was ‘scarcity conditions’ or famine.
To steal a march (to gain an advantage over another stealthily) : While we were still debating the desirability of joint ventures


gin
with foreign concerns, Singapore and Malaysia stole a march over us and opened their gates to foreign investment in a big way.
To steer clear of (to avoid) : India decided on non-alignment to steer clear of the hazards of alignment with one block or the
other.

will stick at nothing if he can only serve himself. eer
To stick at nothing (the phrase implies readiness to stoop to baseness or deception to reach one’s end) : An ambitious politician


• ing
To strain every nerve (to use one’s utmost efforts) : We have to strain every nerve to get over the poverty line.
To strike while the iron is hot (to take advantage of the opportunity when it arises) : If you want to succeed in life, you must


hot.
.ne
strike the iron while it is hot. In going in for general elections immediately after the war, the Congress struck while the iron was

To swallow the bait (to catch others by guile, by offering them large promises) : The candidate offered the people everything


on earth and in the heavens if selected. The people swallowed the bait and elected him.

always talk shop at parties.


t
To talk shop (to use the phrases peculiar to one’s circumstances) : Except for the undertakers, people of the same professions

To tie one’s hands (to restrain one from action) : The Government’s hands are already tied with problem plants. It would not
like to go in for nationalisation in a big way.
• To tread on the heels of (follow close behind) : Famine treads on the heels of drought.
• To fish in troubled waters (to make personal profit out of a disturbance) : The super powers are there in West Asia to fish in
troubled waters.
• To pour oil on troubled waters (to say or do anything which soothes and calms angry passions) : The government poured oil
on troubled waters by announcing a judicial enquiry into the firing.
• To win or gain laurels or to bear away palm (to achieve success in a contest) : The Indian Cricket Team won laurels on two
successive occasions once in West Indies and then in England.
• To worship the rising sun (to pay respect to the man who is rising in power the influence) : The newly appointed manager has
taken over and his clerks worship the rising sun.
• Argus-eyed (jealously watchful) : The husband of a pretty wife has got to be Argus-eyed.
• Aegean stables: (to clean Aegean stables, To correct a great abuse, from the stables of king Agues of Greece, whose stables
had not been cleaned for thirty years) : The law against prostitution has cleaned no Aegean stables; it has merely pushed it
underground.

Downloaded From : www.EasyEngineering.net


Downloaded From : www.EasyEngineering.net

Idioms and Phrases  l 79

• Backstairs influence (influence exerted secretly and in a fashion not legitimate) : The moneyed people do exercise backstairs
influence on Parliament.
• Bad blood : (active enmity) : There has been bad blood between India and Pakistan since 1947.
• A bone of contention : (subject of dispute) : Kashmir continues to be a bone of contention between India and Pakistan since
1947.
• A bosom friend (A very intimate and trusted friend) : Bosom friends never betray one another.
• A bull in a China shop: (Someone who destroys everything at the same time he happens to be in) : The plainsmen proved to
be a bull in a China shop in the hills, ruining the hill people in all ways.
• A close shave : (a narrow escape from collision accident): The bus had a close shave as its driver swerved to the right a split
second before the on-coming truck could run into it.
• A cold comfort: (something calculated to cause pain or irritation) : The promise of a better future is only cold comfort to the
frustrated youth of today.
• A dog in the manger policy: (said of a person who cannot himself use what another wants, and yet will not let that other have
it) : The affluent nations are a dog-in-the manger, destroying what they can’t use themselves than giving it to the poor nations
of Asia and Africa.
• Elbow room: (opportunity for freedom of action) : Only give him elbowroom and he will succeed.

• ww
A fair-weather Friend: (one who deserts you in difficulties) : A fair-weather friend disappears the moment your money disappears.
French leave: (absence without permission.) : He went on a french leave and was summoned by the direction the next day he
went to office.

• w.E
Good offices: (recommendation) : One can get a good job only through the good offices of some one in power.
A good Samaritan: (one who be-friends a stranger or a friendless person) : Centuries ago, India played a good Samaritan to
the hapless Parsees fleeing their native land.

pretty woman. asy
The green-eyed monster: (jealousy) : The green-eyed monster strikes a woman the moment she sees her husband talking to a


the entire country.
En
A Herculean task (a job requiring great efforts) : Eradication of poverty is a Herculean task requiring the collective efforts of

• gin
Lynch Law: (the practice of punishing people where the punishment is inflicted by unauthorised persons and without judicial
trial) : Mob law denotes the same thing when carried out by a mob. In African countries they often resort to lynch laws.
A maiden speech (the first speech of a new member in a public body as in Town Hall or in Parliament) : Amitabh’s maiden


speech was very impressive.
eer
A nine day’s wonder (a fascinating but temporary phenomenon): Beauty is, proverbially, a nine day’s wonder.


question.
ing
An open question : (a matter for discussion and not yet decided): As far as India is concerned, Kashmir is no longer an open

A red-letter day: (an auspicious, fortunate or important day): The 26th January, 1950 is a red-letter day in India’s history.

• .ne
Scot-free: (exempt from payment, unhurt, safe): Because he had influential connections, the culprit went scot-free.
A sheet anchor: (the chief safety, the last refuge for safety): One’s faith in God is one’s sheet anchor in times of stress and strain.




A white elephant (an unprofitable possession) : The upper Houses are white elephants and should be abolished.
A white lie: (an evasion, a harmless and non-malicious untruth) : Professional members often indulge in white lies.
t
Tall Talk: (boastful language): If we have no real accomplishments, we indulge in tall talk to delude ourselves and others too.

A wild goose chase (a foolish, wild, unprofitable adventure) : Attempts towards stabilisation of prices in a developing economy,
is a wild goose chase.
• An apple of discord: (a subject of envy and strife) : Kashmir continues to be the apple of discord between India and Pakistan.
• Cock and bull story (a silly improbable story) : That India wanted to break up West Pakistan was a cock and bull story published
by the U.S.A.
• A fish out of water : (a person in uncomfortable surroundings) : An Indian may earn tons of money in the Western countries,
but he will always feel like a fish out of water there.
• The gift of the gab: (fluency of speech) : The gift of the gab combined with a slight cunning makes for a successful politician.
• Lion’s share: (an unfairly large share) : The big nations continue to have the lion’s share of world trade.
• A mare’s nest : (a discovery that turns out to be false or worthless) : There was much fanfare about the solar cooker. Later it
turned out to be a mare’s nest.
• The milk of human kindness: (kindly feelings a phrase used by Shakespeare.) : With all their poverty, Indians do not lack the
milk of human kindness.
• Penelope’s web : (a work which seems to be going on and yet never comes to an end.) : A housewife’s chores are a penelope’s
web.

Downloaded From : www.EasyEngineering.net


Downloaded From : www.EasyEngineering.net

80  l  Idioms and Phrases

• The pros and cons of a question: (arguments for and against a thing) : They discussed the pros and cons of the matter before
taking a decision.
• The skin of one’s teeth: (a phrase used when one escapes losing everything except life.) : The storm broke up the ship but the
sailors escaped by the skin of their teeth.
• A snake in the grass: (a secret foe.) : China has certainly been a snake in the grass for India. Even in the heyday of Hindi
Chini bhai-bhai, she was quietly devouring bits of our territory.
• A stone’s throw: (very near.) : The Taj Hotel is at a stone’s throw from the Gateway of India.
• All moonshine: (foolish, idle, untrue statement.) : The talk about welfare of the poor is all moonshine.
• Behind the scenes : (of a person having secret or private information and influence) : The dismissed Secretary, having been
behind the scenes, has made some strange revelations as to the way in which the business is managed.
• Between two fires : (assailed or shot at from two sides) : A man, arbitrating between the mother and wife, is to be between the
two fires, for his decisions can rarely please both.
• In a body: (together) : The striking workers went in a body to the Manager to present their demands.
• Wide off the mark or beside the mark: (irrelevant) : ‘Beside the mark reasoning or argument’.
• Cheek by jowl: (in the same position) : There was a lawyer who never had a client cheek by jowl with a doctor who never had


• ww
a patient.
Out at elbows: (destitute) : The rising prices and the new taxes may soon see most of us out at elbows.
Part and Parcel : (integral part of a society, community etc.) Some customs and traditions are a part and parcel of Indian

• w.E
culture.
A storm in a tea cup: (a great fuss about a trifle) : The crackers fired by Diwali revellers caused a storm in the tea cup when
minority communities thought it to be a bomb attack by the other community.


asy
A fly in the ointment : (a trifling circumstance which mars enjoyment) : It was a wonderful picnic, the only fly in the ointment
being the absence of shady trees at the picnic spot.
Not worth his salt: (good for nothing) : A soldier who shivers at the boom of guns is not worth his salt.

En
With a pinch of salt: (to take a statement with a grain of salt is to feel some doubt whether it is altogether true) : Shaw’s claim
of having remained a celibate even after marriage has to be taken with a pinch of salt.

• gin
Null and void: (Invalid, valueless, no longer in force) : The court declared the appointment to be null and void.
To be posted up: (well acquainted with) : I want to be posted up in Indian History.

• eer
To be worth its weight in gold: (extremely valuable) : In the desert a bottle of water is often worth its weight in gold.
To be Greek or double Dutch to one: (unintelligible) : He spoke so fast that all he said was double Dutch to the audience.


To be with in an ace of (to be very nearly) : He was within an ace of being shot.

ing
To be at the back and call: (to be always ready to serve) : You must not expect me to be at your back and call, I have my own
business to attend to.

more bitter. They have always been at daggers drawn. .ne
To be at daggers drawn : (in bitter enmity) : With every passing year the hostility between the Arabs and the Israelis has grown



To be at sea: (contused, uncertain of mind) : I am quite at sea in Mathematics.

was at his wits end as to whom to attend first.


t
To be at one’s wits end: (perplexed) : With the master shouting from the bathroom and the mistress from the kitchen the servant

To be in one’s element: (to be in agreeable company or work) : Shaw is in his element when he is writing about the social ills
of his time.
• To be on wane: (to be on the decline) : After the second World War, the British Empire was on the wane.
• To be on the carpet: (to be summoned to one’s employer’s room for reprimand) : The unpunctual clerk was repeatedly on the
carpet.
• To be on the last legs: (about to collapse) : With science dominating life more and more, religion seems to be on its last legs.
• Chip of the old block (a son who is very like his father) : The younger Nawab of Pataudi has proved to be a chip of the old
block. He is as good a batsman as his father.
• To bring under the hammer: (to sell it by auction) : If a person goes insolvent, his creditors will bring everything that he owns
under the hammer to recover their money.
• To pay one’s way:( not get into debt) : While at college, he paid his way by working as a newspaper vendor.
• To weather the storm: (to come out of a crisis successfully) : In a crisis it is unity which helps a nation to weather the storm.
• To sail before the wind: (to go in the direction towards in which the wind is blowing) : An opportunist is he who sails before
the wind (Its opposite is to sail close to the wind i.e., to break a law or principle)

Downloaded From : www.EasyEngineering.net


Downloaded From : www.EasyEngineering.net

Idioms and Phrases  l 81

• To be in the same boat (To be equally exposed with a person to risk or misfortune) : In a nuclear war, the rich and the poor
nations will be in the same boat. None will be able to protect themselves.
• To sail under false colours: (To pretend to be what one is not, to try to deceive) : In our blessed country, a smuggler sailing
under the false colours of a socialist will never be exposed.
• To take the wind out of one’s sails: (Frustrating him by anticipating his arguments, take away his advantage suddenly) : Before
the U.S. could spread the canard about India’s intention to destroy West Pakistan after “capturing” Bangladesh, India took the
wind out of their sails by declaring a unilateral cease-fire.
• Game is not worth the candle : (The advantage or enjoyment to be gained is not worth the time spent in gaining it) : Journey
to the moon is an elaborate and costly affair and some people with a pragmatic approach feel the game is not worth the candle.
• Not fit to hold a candle to: (One is inferior) : For all his pious platitudes and political stunts, Mr. Nixon is not fit to hold a candle
to Lincoln or Roosevelt.
• Hope springs eternal in the human breast : (one never loses hope).
• Fools rush in where angels fear to tread : (said of reckless persons)
• He who pays the piper calls the tune : (one has to act according to the wishes of one’s master)
• You cannot make a silk purse out of a sow’s ear : (said of something impossible)


• ww
A bird in hand is worth two in the bush : (right use of the present opportunity)
One man’s meat is another man’s poison : (what is good for one may he harmful for another person)
Out of the frying pan into the fire : (from one trouble to another)



w.E
The last straw breaks the camel’s back : (the smallest addition to an already heavy task makes it intolerable)
Distance lends enchantment to the old : (things look nice and beautiful when they are not within reach)
Render unto Caesar what is Caesar’s : (to be wise)



asy
Look before you leap : (don’t be reckless and impulsive)
Make hay while the sun shines : (to make/ill use of the given opportunity)
Never look a gift horse in the mouth : (there can be no choice about things given in charity)


Beggars can’t be choosers : (no choice in scarcity)
En
Nearer the Church, farther from heaven : (the more opportunity you have, the less you benefit from it)

• A rolling stone gathers no moss : (an aimless person cannot succeed)gin
Every cock fights best on his own dung hill : (one is very brave and confident in one’s own place)



Rome was not built in a day : (things take time to complete and to mature)
One swallow does not make a summer : (one person can ‘t do everything) eer



Apparel proclaims the man : (you judge a man’s worth by his clothes)
To run with the hare, to hunt with the hound : (to be insincere to someone)
Sweet are the uses of adversity : (sufferings are to be welcomed)
ing
• Uneasy lies the head that wears the crown : (with power and authority come worries and responsibilities)
.ne
t

Downloaded From : www.EasyEngineering.net


Downloaded From : www.EasyEngineering.net

82  l  Idioms and Phrases

Practice Exercise
LEVEL-I
DIRECTIONS (Qs. 1-50) : Choose the alternative which best 12. Harp on :
expresses the meaning of the idiom/ phrase. (a) To comment (b) To criticise
1. To turn over a new leaf : (c) To keeep on talking (d) To keep on insulting
(a) To change completely one’s course of action 13. To bring one’s eggs to a bad market :
(b) To shift attention to new problems (a) To face on humiliating situation
(c) To cover up one’s faults by wearing new marks (b) To bring one’s commodities to a market where there is
(d) To change the old habits and adopt new ones no demand for them
2. To wrangle over an ass’s shadow : (c) To show one’s talents before audience which is incapable
(a) To act in a foolish way of appreciating them
(b) To quarrel over trifles (d) To fail in one’s plans because one goes to the wrong
(c) To waste time on petty things people for help

3.
ww
(d) To do something funny
All agog :
(a) Everybody (b) All ready
14. To give/get the bird :
(a) To get the awaited (b) To have good luck
(c) To send away (d) To get the impossible
4.
w.E
(c) Restless (d) Almighty
To take with a grain of salt :
(a) To take with some reservation
15. To save one’s face :
(a) To hide oneself (b) To oppose
(c) To avade disgrace (d) To say plainly
(b) To take with total disbelief
(c) To take whole heartedly
(d) To take seriously asy 16. To split hours :
(a) To sidetrack the issue
(b) To quarrel over trifles
5. Hobson’s choice :
(a) Feeling of insecurity
(b) Accept or leave the other En (c) To indulge in over-refined arguments
(d) To find faults with other
(c) Feeling of strength
(d) Excellent choice gin
17. Will o’ the wisp :
(a) Anything which eludes or deceives
(b) To act in a childish way
6. To talk through one’s hat :
(a) To speak fluently
(c) To talk wisdom
(b) To talk nonsense
(d) To speak at random eer
(c) To act in a foolish way
(d) To have desires unbacked by efforts
7. To snap one’s fingers :
(a) To speak abruptly
(b) To accept immediately
18.
ing
To read between the lines :
(a) To concentrate
(b) To read carefully
(c) To grasp eagerly
(d) To become contemptuous of
(c) To suspect
(d) To grasp the hidden meaning .ne
8. To take the bull by the horns :
(a) To punish a person severly for his arrogance
(b) To grapple courageously with difficulty that lies in our
way
19. To flog a dead horse :
(a) To act in a foolish way
(b) To waste one’s efforts
(c) To revive interest in an old subject
t
(c) To handle it by fierce attack (d) To revive old memories
(d) To bypass the legal process and take action according 20. A tall order :
to one’s own whims (a) A task difficult to perform
9. To be in abeyance : (b) A bid problem
(a) To be in trouble (b) Dual minded
(c) A royal summon
(c) In a fighting mood (d) Insuspense
(d) A big demand
10. To cast pearls before a swine :
21. To turn the tables :
(a) To spend recklessly
(b) To spend a lot of money on the unkeep of domestic hogs (a) To defeat (b) To oppose
(c) To waste monkey over trifles (c) To create chaos (d) To change the sorry scheme
(d) To offer to a person a thing which he cannot appreciate (e) To change completely the position of disadvantage
11. To take people by storm : 22. To keep the ball rolling :
(a) To put people in utter surprise (a) To earn more and more
(b) To captivate them unexpectedly (b) To work constantly
(c) To exploit people’s agitation (c) To keep the conversation going
(d) To bring out something sensational attracting people’s (d) To make the best use of
attention

Downloaded From : www.EasyEngineering.net


Downloaded From : www.EasyEngineering.net

Idioms and Phrases  l 83

23. All and sundry : (c) To die fighting


(a) Greater share (d) To carry on a contest regardless of consequences
(b) All of a sudden 35. To give a false colouring :
(c) Completion of work (a) To misrepresent (b) To submit the false report
(d) Everyone without distinction (c) To be dishonest (d) To conceal the facts
24. To worship the rising sun : 36. To show the white feather :
(a) To honour a man who is coming into office (a) To show signs to cowardice
(b) To honour to promising people (b) To seek peace
(c) To indulge in flattery (c) To show arrogance
(d) To welcome the coming events (d) To become polite
25. To play fast and loose : 37. Spick and span :
(a) To beguile others (a) Neat and clean (b) Outspoken
(b) To be winning sometimes and losing at other times (c) A ready-made thing (d) Garrulous
(c) To play with someone’s feeling 38. To take the wind out of another’s sails :
(d) To play tricks (a) To manouevre ot mislead another on the high seas
26. To have brush with : (b) To cause harm to another

ww
(a) To start painting
(b) To have good and pleasing terms
(c) To be impressed 39.
(c) To defeat the motives of another
(d) To anticipate another and to gain advantage over him
To carry the coal to newcastle :
27.
w.E
(d) To have a slight encounter
To pull one’s socks up :
(a) To prepare
(c) To get ready
(b) To try hard
(d) To depart
(a) To work hard
(b) To finish a jab
(c) To do unnecessary things
28. Dog in the manger :
asy
(a) An undersized bull almost the shape of a dog
40.
(d) To do menial jobs
The pros and cons :
(a) For and aginst a thing (b) Foul and fair
(b) A dog that has no kennel of its own

En
(c) A person who puts himself in difficulties on account
of other people
41.
(c) Good and evil
A baker’s dozen :
(d) Former and latter

(d) A person who prevents others from enjoying something


useless to himself gin
42.
(a) Twelve
(c) Thirteen
A bull in a china shop :
(b) Charity
(d) Allowance
29. To set the people by ears :
(a) To box the people
eer
(a) A person who is very ugly but loves the beautiful things
of life
(b) To insult and disgrace the people
(c) To punish heavily
(d) To excite people to a quarrel ing
(b) A person who takes a sadistic delight in harming
innocient people
(c) A person who becomes too excited where no excitment
30. To give chapter and verse for a thing :
(a) To produce the proof of something
is warranted
.ne
(d) A person who is rought and clumsy where skill and

31.
(b) To eulogize the qualities of a thing
(c) To make publicity of a thing
(d) To attach artificial value to a thing
To plough the sands :
43.
care are required
Hard-pressed :
(a) Bewildered
(c) Hard discipline
(b) Insulted
(d) In difficulties
t
(a) To work hard to achieve one’s aim
44. To be at one’s finger’s end :
(b) To afford hope or ground for expecting a good result
(a) To be hopeless
(c) To busy oneself in a way which cannot lead to any
(b) To be highly perplexed
profitable result
(c) To be completely conversant with
(d) To advance one’s position in life
(d) To count things
32. To take umbrage :
45. To pull strings :
(a) To feel depressed (b) To be offended
(c) To be satisfied (d) To be pleased (a) To exert hidden influence
33. To drink like a fish : (b) To tease someone
(a) To drink little (c) To speed up
(b) To drink alone (d) To start something
(c) To be a drunkard 46. A green horn :
(d) To drink in the company of others (a) An envious lady
34. To fight to the bitter end : (b) A trainee
(a) To fight with poison-tipped arrows (c) An inexperienced man
(b) To fight to the last paint of enemy position (d) A soft-hearted man

Downloaded From : www.EasyEngineering.net


Downloaded From : www.EasyEngineering.net

84  l  Idioms and Phrases

47. To be old as the hills : 57. For this act of indifference he will be taken to task by the
(a) To be very ancient (b) To be wise tand learned authority.
(c) To be old but foolish (d) Not being worth the age (a) he will get an offical reprimand from the authority.
48. To pour oil in troubled water : (b) he will be rewarded by the authority.
(a) To forment trouble (c) he will tender his resignation to the authority.
(b) To add to the trouble (d) he will be entrusted with an official job.
(c) To instigate 58. You need to have something up your sleeve if the present
(d) To calm a quarrel with sooting words plan does not work.
49. To mind one’s P’s and Q’s : (a) have some honest means
(a) To be cautious (b) have some hidden sources of money
(b) To be accurate and precise (c) have a secret pocket in the sleeve
(c) To be careful of one’s accounts (d) have an alternative plan
(d) To be careful of one’s personality 59. The new manager ruled the roost to every one.
50. To break the ice :
(a) exercised authority
(a) To start quarreling
(b) rushed through work
(b) To end the hostility
(c) got paid very handsomely

ww
(c) To start to conversation
(d) To end up partnership
DIRECTIONS (Qs. 51-55) : In the following questions, four
(d) created good impression
60. Despite his initial arrogance he had to eat humble pie.

w.E
alternatives are given for the idiom/phrase underlined in the
sentence. Choose the alternative which best expresses the meaning
of the idiom/phrase
(a) he had to yield under pressure
(b) he maintained composure
(c) he failed to protest eventually

51. asy
[SSC Sub Insp. 2012]
For some people, writing verse is as duck takes to water.
(d) he accepted the food oflfered

DIRECTIONS (Qs. 61-65) : In the following questions, four


(a) like taking the duck to water
(b) like bursting out suddenly
En alternatives are given for the Idiom/phrase underlined in the
sentence. Choose the alternative which best expresses the meaning
of the Idiom/ Phrase and mark it in the Answer Sheet.

52.
(c) like dropping the duck in the water
(d) like easily and naturally speaking
He made my day by telling me how important I was to him. gin [SSC Sub Insp. 2014]

(a) gave me great pleasure


(b) displeased me
61.

eer
We must work with all our might and main, otherwise we
cannot succeed.
(a) full force (b) complete trust
(c) spoiled my day
(d) made me resentful 62. ing
(c) exceptional skill (d) full unity
The sailors nailed their colours to their mast.
53. He made away with ten thousand rupees in the course of
three months.
(a) earned (b) ran away with
(a) put up a colourful mast
(b) refused to climb down
.ne
54.
(c) squandered (d) saved
The students of that group have assured their project guide
that they will all work against the clock.
(a) work while keeping patience
63.
(c) took over the ship
(d) decided to abandon the ship
t
We had (had) better batten down the hatches. The weather
is unpredictable.
(a) stay in-door
(b) work with vigour to finish in limited time (b) prepare for a difficult situation
(c) work with enthusiasm (c) go somewhere safe
(d) work hard to go against the time (d) face the obstacles
55. To let off steam, my friend started murmuring. 64. It is difficult to have a sensible discussion with her as she
(a) to release his tension (b) to show his anger flies off at a tangent.
(c) to show his approval (d) to show his displeasure (a) gets carried away
DIRECTIONS: (Qs. 56-60) : In the following questions, four (b) starts discussing something irrelevant
alternatives are given for the Idiom/Phrase printed in bold in the (c) loses her temper easily
sentence. Choose the alternative which best expresses the meaning (d) does not really understand anything
of the Idiom/ Phrase. 65. The students found it hard to go at equal speed with the
[SSC Sub Insp. 2013] professor.
56. I tried to feel his pulse on the issue, but in vain. (a) get away from (b) put up with
(a) find his views (b) enlighten him (c) keep up with (d) race against
(c) argue with him (d) guide him

Downloaded From : www.EasyEngineering.net


Downloaded From : www.EasyEngineering.net

Idioms and Phrases  l 85

DIRECTIONS: (Qs. 66-68) : In the following questions, four 67. We cannot depend on him for this assignment as it needs
alternatives are given for the idiom / phrase underlined in the careful handling and he is like a bull in a china shop.
sentence. Choose the alternative which best expresses the meaning (a) a felicitous person (b) a clumsy person
of the idiom / phrase and mark it in the Answer Sheet. (c) a tactful person (d) a no-nonsense person
[SSC Multi tasking 2013] 68. The mother always insists on keeping the house spick and
66. The man changed colours when I questioned him on the span.
allocation of funds. (a) open (b) locked
(a) turned pale (b) got numbed
(c) safe (d) tidy
(c) turned happy (d) get motivated

LEVEL-II
DIRECTIONS (Qs. 1-27) : Choose the alternative which best 10. To take a leaf out of somebody’s book :
expresses the meaning of the idiom / phrase. (a) To take him as a model
1. To cool one’s heels : (b) To steal something valuable

ww
(a) To rest for sometime
(b) To give no importance to someone
(c) To remain in a comfortable position 11.
(c) To follow the dictates of someone
(d) To conform to other’s standard
To set the thames on fire :

2. w.E
(d) To be kept waiting for sometime
A fool’s errand :
(a) A blunder (b) An impossible task
(a) To do something remarkable
(b) To try to do the impossible
(c) To burn something to ashes

3. Swan-song :
asy
(c) A useless undertaking (d) None of these

(a) Music as sweet as a song of birds


12.
(d) To fling inonical remarks
To cast pearls before a swine :
(a) To offer some a thing which he cannot appreciate
(b) A melodious song in praise of someone
(c) Praise of a woman by her lover
En (b) To bring something good before the eyes of a greedy
person

4.
(d) Last work of a poet or musician before death
To oil the knocker :
(a) To instigate a person to do a job gin
13.
(c) To spend racklessly on a useless fellow
(d) To indulge in fruitless endeavours
To bear the palm :
(b) To do a work with increased pace
(c) To tip the office-boy
(d) To revive old enmity
(a) To win
eer
(c) To accept defeat
(b) To accept a challenge
(d) To endure something

5. By the rule of thumb :


(a) By the use of force
14.

ing
To change colour :
(a) To change appearance
(b) To shift allegiance to
(b) By the use of trickery
(c) By cheating and deception
(d) By practical experience which is rather rough
(c) To favour the wrong person
(d) None of these .ne
6. To live fast :
(a) To lead a life of dissipation
(b) To accomplish a purpose
(c) to do a task hurriedly
15. To cut the gordian knot :
(a) To solve a difficult problem
(b) To be victorious
(c) To break ties with someone
t
(d) To use up one’s income as fast as it comes in (d) To defeat to powerful person
7. To hold somebody to ransom : 16. To wrangle over an ass’s shadow :
(a) To keep captive and demand concession (a) To act in a foolish way
(b) To humiliate somebody (b) To do something funny
(c) To offer bribe (c) To quarrel over trifles
(d) To blackmail and extract money (d) To quarrel over the possession of an ass
8. To hit the jackpot : 17. To have one’s heart in one’s boots :
(a) To gamble (a) To be deeply depressed
(b) To get an unexpected victory (b) To be frightened
(c) To be wealthy (c) To get angry
(d) To make money unexpectedly (d) To keep a secret
9. To pay off old scores : 18. To strike one’s colours :
(a) To have one’s revenge (a) To fight vigorously (b) To work hard
(b) To settle a dispute (c) To surrender (d) To make a rude gesture
(c) To repay the old loan
(d) None of these

Downloaded From : www.EasyEngineering.net


Downloaded From : www.EasyEngineering.net

86  l  Idioms and Phrases

19. To ride hell for Leather : 30. Adolescence is a period of halcyon days.
(a) To ride with furious speed (a) hard days (b) of mental pressure
(b) To adopt flase means to succeed (c) happy days (d) days of preparation
(c) To work hard for a small accomplishment 31. My sincere advice to my maidservant fell on stony ground.
(d) To earn money by all means (a) was counter productive
20. To speak daggers : (b) had a strong impact
(c) made on stubborn
(a) To abuse someone
(d) had little success
(b) To indulge in varacious talks
32. He has all his ducks in a row; he is complacent.
(c) To speak to a person with hostility
(a) has everything ready (b) is well organised
(d) None of these (c) always scores a zero (d) never gets confused
21. Be in the mire :
33. With great difficulty, he was able to carve out a niche for
(a) Be under dept (b) Be in love
(c) Be in difficulties (d) Be uneasy himself.
(a) became a sculptor
22. To give a rap on the knuckles :
(a) To eulogise (b) To chatise (b) did the best he could do
(c) To condemn (d) To rebuke (c) destroyed his career

ww
23. To be above board.
(a) To have a good height
(b) To be honest in any business deal
34.
(d) developed a specific position for himself
You will succeed if you follow my advice to the letter.
(a) about writing letters (b) written in the letter

w.E
(c) Having no debts
(d) To try to be beautiful
35.
(c) in every detail (d) very thoughtfully
A critic’s work is to read between the lines.
(a) to comprehend the meaning
(b) to appreciate the inner beauty
24. To cry wolf.
(a) To listen eagerly
(c) To turn pale asy
(b) To give false alarm
(d) To keep off starvation
(c) to understand the inner meaning
(d) to read carefully
25. He is on the wrong side of seventy.
(a) more than seventy years old En 36. Where discipline is concerned I put my foot down.
(a) take a firm stand (b) take a light stand
(b) less than seventy years old
(c) seventy years old gin
37.
(c) take a heavy stand (d) take a shaky stand
The convict claimed innocence and stood his ground in spite
of the repeated accusations.
(d) eighty years old
26. To have an axe to grind.
(a) knelt
eer
(b) surrendered
(a) a private end to serve (b) to fail to arouse interest
(c) to have no result
27. To drive home.
(d) to work for both sides
38.
ing
(c) kept standing
(d) refused to yield
We must work with all our might and main, otherwise we
(a) To find one’s root
(b) To return to place of rest
cannot succeed.
(a) full force .ne
(b) complete trust
(c) Back to original position
(d) To emphasise
DIRECTIONS (Qs. 28-42) : In the following questions, four
alternatives are given for the idiom / phrase and bold italicised in the
39.
(c) exceptional skill (d) full unity
The sailors nailed their colours to their mast.
(a) put up a colourful mast
(b) refused to climb down
t
sentence. Choose the alternative which best expresses the meaning (c) took over the ship
of the idiom / phrase and mark it in the Answer-Sheet. (d) decided to abandon the ship
40. We had (had) better batten down the hatches. The weather
28. Once the case reached the court, the police washed their is unpredictable.
hands off it. (a) stay in-door
(a) waited for a response to (b) prepare for a difficult situation
(b) claimed credit for (c) go somewhere safe
(c) disassociated themselves from (d) face the obstacles
(d) seemed eager to continue 41. It is difficult to have a sensible discussion with her as she
29. She wanted to go hitch-hiking but her mother put her foot flies off at a tangent.
down and now she’s going by bus. (a) gets carried away
(a) took a firm stand (b) expressed her displeasure (b) starts discussing something irrelevant
(c) scolded her badly (d) got irritated (c) loses her temper easily
(d) does not really understand anything

Downloaded From : www.EasyEngineering.net


Downloaded From : www.EasyEngineering.net

Idioms and Phrases  l 87

42. The students found it hard to go at equal speed with the 49. To cry wolf.
professor. (a) To listen eagerly (b) To give false alarm
(a) get away from (b) put up with (c) To turn pale (d) To keep off starvation
(c) keep up with (d) race against 50. He is on the wrong side of seventy.
DIRECTIONS (Qs. 43-47) : In the following questions, four (a) more than seventy years old
alternatives are given for the idiom / phrase and bold italicised in the (b) less than seventy years old
sentence. Choose the alternative which best expresses the meaning (c) seventy years old
of the idiom / phrase and mark it in the Answer-Sheet. (d) eighty years old
51. To have an axe to grind.
[SSC CGL 2012]
(a) a private end to serve
43. Once the case reached the court, the police washed their (b) to fail to arouse interest
hands off it. (c) to have no result
(a) waited for a response to (d) to work for both sides
(b) claimed credit for 52. To drive home.
(c) disassociated themselves from (a) To find one’s root

44. ww
(d) seemed eager to continue
She wanted to go hitch-hiking but her mother put her foot
(b) To return to place of rest
(c) Back to original position

w.E
down and now she’s going by bus.
(a) took a firm stand
(b) expressed her displeasure
(d) To emphasise
DIRECTIONS (Qs. 53-57) : In the following questions, four
alternatives are given for the Idiom/Phrase underlined in the
(c) scolded her badly
(d) got irritated asy sentense. Choose the alternative which best expresses the meaning
of the Idiom\Phrase and mark it in the Answer Sheet.
45. Adolescence is a period of halcyon days.
(a) hard days (b) of mental pressure
En 53.
[SSC CGL 2014]
With great difficulty, he was able to carve out a niche for

46.
(c) happy days (d) days of preparation
My sincere advice to my maidservant fell on stony ground.
gin himself.
(a) became a sculptor
(b) did the best he could do

47.
(a) was counter productive (b) had a strong impact
(c) made on stubborn (d) had little success
He has all his ducks in a row; he is complacent. eer
(c) destroyed his career
(d) developed a specific position for himself

(a) has everything ready


(c) always scores a zero
(b) is well organised
(d) never gets confused
54.

ing
You will succeed if you follow my advice to the letter.
(a) about writing letters
(c) in every detail
(b) written in the letter
(d) very thoughtfully
DIRECTIONS (Qs. 48-52) : In the following questions, four
alternatives are given for the Idiom/Phrase underlined. Choose
55.
(a) to comprehend the meaning .ne
A critic’s work is to read between the lines.

the alternative which best expresses the meaning of the Idiom/


Phrase and mark it in the Answer Sheet
[SSC CGL 2013] 56.
(b) to appreciate the inner beauty
(c) to understand the inner meaning
(d) to read carefully
Where discipline is concerned I put my foot down.
t
48. To be above board. (a) take a firm stand (b) take a light stand
(a) To have a good height (c) take a heavy stand (d) take a shaky stand
57. The convict claimed innocence and stood his ground in spite
(b) To be honest in any business deal
of the repeated accusations.
(c) Having no debts
(a) knelt (b) surrendered
(d) To try to be beautiful (c) kept standing (d) refused to yield

Downloaded From : www.EasyEngineering.net


Downloaded From : www.EasyEngineering.net

88  l  Idioms and Phrases

Hints & Solutions


LEVEL- I LEVEL- II
1. (d) 2. (b) 3. (c) 4. (a) 1. (d) 2. (c) 3. (d) 4. (c)
5. (b) 6. (b) 7. (d) 8. (b) 5. (d) 6. (a) 7. (a) 8. (b)
9. (d) 10. (d) 11. (b) 12. (c) 9. (b) 10. (a) 11. (b) 12. (a)
13. (d) 14. (c) 15. (c) 16. (c)
17. (a) 18. (d) 19. (b) 20. (a) 13. (a) 14. (b) 15. (a) 16. (c)
21. (d) 22. (c) 23. (d) 24. (a) 17. (a) 18. (c) 19. (a) 20. (c)
25. (d) 26. (d) 27. (b) 28. (d) 21. (c) 22. (d) 23. (b) 24. (b)
29. (d) 30. (a) 31. (c) 32. (b) 25. (a) 26. (a) 27. (d) 28. (c)
33. (c) 34. (d) 35. (a) 36. (a) 29. (a) 30. (c) 31. (d) 32. (b)
37. (a) 38. (d) 39. (c) 40. (a) 33. (d) 34. (c) 35. (c) 36. (a)
41. (c) 42. (d) 43. (d) 44. (c) 37. (d) 38. (a) 39. (b) 40. (b)
45. (a) 46. (c) 47. (a) 48. (d)
49.
51.
(b)
ww 50. (c)
(d) The meaning of as duck takes to water easily and
naturally.
41.
43.
(b) 42. (b)
(c) Idiom washed their hands off means : to absolve oneself
of responsibility or future blame.
52.
53.
54.
(a)
w.E
(c) Squandered means use up, cash out.
(b) The meaning of against the clock : in a great hurry
44. (a) Idiom put her foot down means : to tell someone in a
strong way that they must do something or that they
must stop doing something.
55.
asy
to get something done before a particular time.
(b) The meaning of let off steam : to release one’s pent-
up emotions, such as anger, usually verbally.
45.

46.
(c) Idiom Halcyon days means : a very happy or successful
period in the past.
(d) Idiom fell on stony ground means : It a request, a
56.
57.
(a) Feel pulse = to try to know someone’s views.
(a) Take somebody to task = to criticize somebody
strongly for something they have done. En warning, or advice falls on stony ground, people ignore
it.
58.

59.
(d) Have/keep something up your sleeve = to keep a
plan or an idea secret until you need to use it.
(a) Rule the roost = to be the most powerful member of
gin
47. (b) Idiom has all his ducks in a row means : to organize
things well.

60.
a group.
(a) Eat humble pie = to say and show that you are sorry
48.

49. eer
(b) If somebody is above board, he/she is honest in any
business deal.
(b) To cry wolf means that someone is giving false alarm.
61.
for a mistake that you made.
(a) The idiom ‘with all our might and main’ means with
great physical strength; great force.
50.

51.
ing
(a) If somebody is on the right/ wrong side of 30/ 40 etc
that means he/she is younger/ older than 30/ 40 etc.
(a) If you have an axe to grind; that means you have a
62. (b) THe idiom ‘nailed their colours to their mast’ means
to defiantly display one’s opinions and beliefs. Also,
to show one’s intention to hold on to those beliefs 52.
private end to serve.
.ne
(d) If you drive something home, that means you are
until the end. Its origin dates back to 17th century.
In 17th century nautical battles colours (flags)
were struck (lowered) as a mark of submission.
It was also the custom in naval warfare to direct 53.
done that.
t
making something completely clear to someone. She
didn’t have to drive the point home. The movie had

(d) In the sentence, ‘to carve out a niche’ means ‘to develop
one’s cannon fire at the opponent’s ship’s mast, thus a specific position for him’. The word ‘niche’ means ‘a
disabling it. If all of a ship’s masts were broken the specialized area or sector’.
captain usually had no alternative but to surrender.
If the captain decided to fight on this was marked 54. (c) The alternative meaning of the phrase ‘to the letter’ is
by hoisting the colours on the remnants of the ship’s ‘in every detail’.
rigging, that is, by ‘nailing his colours to the mast’. 55. (c) The meaning of the idiom ‘to read between the lines’ is
63. (b) THe idiom ‘batten down the hatches’ means to ‘to understand the inner meaning’. So, the critic’s work
prepare for difficult times. is to understand the inner meanings.
64. (b) THe idiom ‘ she flies off at a tangent’ means to 56. (a) The meaning of the idiom ‘put my foot down’ is ‘to take
pursue a somewhat related or irrelevant course a firm stand’. So, as the meaning of the sentence implies
while neglecting the main subject.
that where discipline matters, one should take a firm
65. (b) THe idiom ‘ go at equal speed’ means to go neck-to-
stand.
neck and put up with the teacher.
57. (d) The idiom ‘stood his ground’ means ‘refuse to yield’.
66. (a) Turned pale means colourless
So, as the meaning of the sentence implies, the convict
67. (b) a clumsy person means plump claimed his innocence and refused to yield for the
68. (d) tidy means clean blames that were laid upon him.

Downloaded From : www.EasyEngineering.net


Downloaded From : www.EasyEngineering.net

8
PTER
CHA

Spotting Error

Tw
hese questions are asked on various competitive exams. They are considered a great style of questions to test one’s grammar
skills. We are given sentences from which we have to pick out the one which is grammatically incorrect (or correct,
depending on the directions). These questions are basically aimed at testing your grammar usage and not to see if you know

ww
the complicated and/or some uncommon grammar rules. Spotting errors questions would not pose great problems to you if your
grammar knowledge is sound.

HOW TO SOLVE
.E asy
(1) Four or five options are given to you, out of which one is grammatically incorrect. So, first of all read out all the options quickly,
but clearly (If the given four options will all be related, as if it is a short paragraph or information on a subject in four/five
sentences. You should be able to grasp the verb tense of the event being talked about and pick your answer accordingly.)

En
(2) If a sentence sounds awkward, or ambiguous then it might be your answer; verify it, and you would have completed the question.
(3) In case you are not able to find the answer this way, then read each statement individually and see if you can find any grammatical

TIPS
mistake.

gin
eer
(1) Sometimes, the error could be just of the verb tense. But, how will you be able to spot the verb tense when any of the given part(s)
of the sentence, or paragraph, could be wrong? You have to spot all those words which inform you about the verb tense, and then

your answer, as the given option will have the verb–tense error.
ing
see which word is not complying with the verb tense the other words represent. If you can spot such a word, then you also have

(2) Keep all the common grammatical mistakes (which we have discussed previously) in mind while reading the options. These are the

.ne
most commonly asked types and hence, a lot of the times, you can spot these errors before you even read the complete sentence.

COMMON MISTAKES
(1) Pronoun-consistency is one error which gets easily ignored, so make sure the options represent pronoun consistency.
t
(2) The given options are given in a sequential order, even if they are just one sentence all together. So, reading the option individually
and not in the sequence would lead to a mistake. Students sometimes see the options and just look at them individually in order
to spot an error.

FROM BEGINNER TO MASTERY


Example 1.
“Meeting will take place at the Rancozy Hotel, hence I’m going to the Rancozy Hotel.”
What is wrong with this sentence?
Explanation  It is redundant, because using the same noun more than once in a sentence sounds unnecessary. In such situa-
tions, we make use of pronouns: Meeting will take place at the Rancozy Hotel, hence I’m going there.
Example 2.
“I went there; because I wanted to.”
Find the flaw in the given sentence.
Explanation  The given sentence has incorrectly used the semi-colon. Instead, there should be a colon there.
Example 3.
“One is always conscious about how they look.”
Can you spot an error in this sentence?

Downloaded From : www.EasyEngineering.net


Downloaded From : www.EasyEngineering.net

90  l  Spotting Error

Explanation It suffers from the pronoun-antecedent error. The correct sentence would be:
One is always conscious about how he/she looks.
Example 4.
(a) In the forthcoming elections (b) every man or woman
(c) must vote for the candidate (d) of their choice
Explanation  (d); Option (d) is incorrect, because it has a pronoun antecedent error. The word ‘every’ is a singular pronoun,
so it mentions every man and every woman singularly, not collectively. But, the pronoun of antecedent of ‘every man and woman’ is
‘their’, so it refers to them collectively. The correction would be – ‘of his or her choice’.
Example 5.
(a) Almost all school teachers insist that (b) a student’s mother
(c) is responsible for their student’s conduct (d) as well as his dress
Explanation  (c); The correct answer is option (c), because the pronoun ‘their’ used in the statement makes it look like the
students belong to the mother instead to the teachers. The correct statement would be: “… for the student’s conduct”.
Example 6.
(a) it is essential that diseases like tuberculosis (b) are detected and treated
(c) as early as possible in order to (d) assure a successful cure.

ww
Explanation  (d); Option (d) has an error because of the word ‘assure’. Assure (verb) means ‘to inform positively, with cer-
tainty and confidence’. For example: He assured me that this drink is safe to drink. Hence, to assure, is to remove the doubts and
tell someone something confidently. The appropriate word for this context is ‘ensure’. It basically means ‘to make sure’, so it refers to

w.E
making sure that something will or will not happen. If used in the sentence, it would mean making sure there is a successful cure
for the disease.
Example 7.
(a) If one has to decide
(c) you should choose that option asy (b) about the choice of a career
(d) which is really beneficial.

En
Explanation (c); Option (c) is grammatically incorrect, because of the pronoun inconsistency. The pronoun used in (a) is
‘one’ whereas, the pronoun used in (c) is ‘you’. The correction would be – one should choose that option.
Example 8.
(a) He is the sort of person
(c) would be capable of gin
(b) who I feel
(d) making these kind of mistakes
Explanation 
eer
(d); This part of the sentence makes the pronoun antecedent error. The pronoun: ‘these’ used here is for refer-
encing the noun: ‘kind’. Thus, as you can see, it should either be ‘this kind’ or ‘these kinds’.
Example 9.
(a) Sumit found the new job
(c) he had left ing
(b) more preferable to the one
(d) so he decided to continue for a while.

.ne
Explanation (b); We need to choose (b), because ‘more preferable’ is an incorrect phrase. Preferable itself means more desir-
able than the other, and hence, ‘more preferable’ is a redundant phrase.
Example 10.
(a) Prakash said that,
(c) and that if funds were available,
Explanation 
(b) if he were elected president
(d) he would create a national theatre.
t
(c); Option (c) is wrong, because of the use of ‘that’. ‘That’ can be used as a relative pronoun to connect two
clauses (but here there is no connection required). It can also be used to introduce the subject of the sentence, but here we have a
dependent and difference conditional clauses in which the use of that is not required.
Example 11.
(a) Her acceptance of speech (b) was well received
(c) eliciting thunderous applause (d) in several points.
Explanation  (d); The correct usage would be ‘on several points’.
Example 12.
(a) An oppressive solemnity (b) and not the festive mood
(c) one might have expected (d) characterised the mood by the gathering
Explanation  (d); Option (d) is the answer because the correct preposition is ‘of ’ not ‘by’: … mood of the gathering.
Example 13.
(a) All aspiring artists must (b) struggle by the conflict
(c) between faith in their own talent (d) and knowledge that very few are great enough to succeed
Explanation  (b); Option (b) is the incorrect part of the sentence because it has used the wrong preposition. The correct
form is ‘struggle with’ and not ‘struggle by’.

Downloaded From : www.EasyEngineering.net


Downloaded From : www.EasyEngineering.net

Spotting Error  l 91

Example 14.
(a) Despite some bad news (b) Michel’s stature was not diminished
(c) and her fans or critics (d) were unanimous in appreciating her work
Explanation  (c); Option (c) is the correct choice because the word should be and in place of or. ‘Or’ is used to introduce an
alternative or a similar word, but fans and critics are clearly not similar words. Also, the sentence means that both the parties agreed
and hence the conjunction should be ‘and’.
Example 15.
(a) Jazz is an American art form (b) which was now flourishing in Europe
(c) through the efforts of expatriates (d) in France, Scandinavia and Germany
Explanation  (b); Option (b) is the answer, because the given sentence is in the present tense, but the ‘was’ is used to denote
past tense. ‘Is’ should be the replacement, to make it a grammatically correct sentence.
Example 16.
(a) Character, and (b) not riches (c) win us (d) respect
Explanation  (c); The correct sentence is: Character, and not riches, wins us respect.
Direction:  Each sentence below has four underlined words or phrases, marked A, B, C and D. Identify the underlined part that must
be change to make the sentence correct.
Example 17.
ww
Neither the examiner (A) nor his assistant (B) were informed (C) about the cancellation of the examination. No Error (D).
(a) A
Explanation 
Example 18. w.E (b) B (c) C (d) D
(c); Neither … nor would take a singular verb: ‘was’ and not the plural one: ‘were’.

(a) A
Explanation 
(b) B
asy
Being (A) a short holiday (B) we had to return (C) without visiting many of the places (D)
(c) C (d) D
(a); The structure of the sentence is awkward and hence creates an impression that the people were the holiday.
This makes no sense. This is a modifier error.
En
gin
eer
ing
.ne
t

Downloaded From : www.EasyEngineering.net


Downloaded From : www.EasyEngineering.net

92  l  Spotting Error

Practice Exercise
LEVEL-I
DIRECTIONS (Qs. 1-70):  In the following questions, a sentence 15. (a) Did he say he will
has been divided into four parts and marked a, b, c and d. One of (b) bring the book tomorrow?
these parts contains a mistake in grammar Idiom or syntax. Identify (c) He has been promising so
that part and mark it as the answer. (d) for the past seven days
16. (a) The cost of food grains have increased
1. (a) They appointed him (b) as a manager (b) so rapidly during the last two years
(c) as he (d) is efficient (c) that salaried classes find it difficult
2. (a) Owing to illness (b) he was unable (d) to save any part of their income
(c) to go (d) for his holiday 17. (a) The train is unusually late today
3. (a) Pickpocketers are (b) sometimes spotted (b) for it has been announced just now

ww
(c) by policemen
4. (a) His both hands
(c) so he
(d) at bus stops
(b) have been injured
(d) cannot work
(c) that it will arrive in a hour
(d) on platform number three
18. (a) Fluosol is a transparency liquid

w.E
5. (a) Several guests noticed Mr. Peter
(b) fall back
(c) in his chair
(b) that very much resembles water
(c) but in fact is twice
(d) as dense as water
19. (a) Along the northern border of India
(d) and gasping for breath
6. (a) The short story asy
(b) should not exceed
(b) is seen the Himalayas
(c) tall, mighty and majestic
(c) more than
7. (a) If one reads the newspaper regularly
(b) you will be surprised at the improvement En
(d) two hundred words (d) in their unique splendour
20. (a) The next generation of commercial airplanes
(b) are expected to be faster
(c) in your overall reading skills
(d) day by day gin (c) less fuel-thirsty and
(d) more fully computerized than the present one
8. (a) Preetam asked her sister
(b) why had she not gone to the school
(c) the previous day eer
21. (a) Each of the speakers whom I invited
(b) to participate in the debate
(c) has indicated their unwillingness
(d) or applied for leave
9. (a) In tropical climate, it is necessary ing
(d) because of the short notice
22. (a) The creature on Mars, if any
(b) are bound to be very different from us
(b) that a person drink
(c) several cups of water daily
(d) if he wishes to remain healthy .ne
(c) not only in shape but also in size
(d) because of different gravitational conditions
10. (a) Pollution effects more people
(b) today than it ever did in the past
(c) because more people live near industrial units
(d) and inhale noxious gases from the atmosphere
23. (a) In order to identify a bird
(b) you must note it’s peculiar marking
(c) and then refer to a guide book
(d) and match them with those of different birds
t
11. (a) Sita claimed that she had 24. (a) My friend who returned from Florida
(b) not only gone to Delhi (b) after a stay of many years there said
(c) but also to Agra (c) that the climate of Florida is
(d) and had seen the Taj Mahal (d) very much like Madras in summer
12. (a) The advertisement for the new detergent 25. (a) The set of enactments passed by the assembly
(b) claimed that it was as effective (b) have been sent to the Council
(c) if not more than (c) for consideration by its members
(d) washing soap in removing dirt from clothes (d) before being notified in the gazette
13. (a) If you would have come earlier 26. (a) Much of the students in the class
(b) there would have been enough time (b) are weak in Mathematics and therefore
(c) for us to go to the movie (c) the Headmaster has arranged for special tuition
(d) which has been running to full-houses (d) in the evening after class-hours
14. (a) Nalini found it difficult 27. (a) The speaker gave a lucid exposition of
(b) to manage single-handedly the three children (b) the butterfly, its evolution
(c) who were always quarrelling (c) its growth, its varieties
(d) between themselves (d) and how it is used to eat

Downloaded From : www.EasyEngineering.net


Downloaded From : www.EasyEngineering.net

Spotting Error  l 93

28. (a) He is so versatile 42. (a) I do not think that


(b) that he is capable of beating (b) neither of the two proposals of yours
(c) all his other classmates (c) will be acceptable to him, and so
(d) both in studies, public speaking and sports (d) the stalemate is likely to continue
29. (a) It was the drug and not the disease 43. (a) Since banks usually give gifts
(b) that killed him (b) to customers who deposited large amounts,
(c) he would have been alive today (c) it is worthwhile that you bargain with them
(d) if he did not take the drug (d) before you choose a particular bank
30. (a) Neither the warning of his father 44. (a) Everyone who saw the movie ‘Star Wars’
(b) nor the punishment by his teacher (b) is unanimous in their view
(c) have had any effect on Ramu, who (c) that it is the best science fiction movie
(d) continues to be absent from classes frequently (d) ever produced
31. (a) Which of the three puppies 45. (a) We consider him the best candidate for the post
(b) do you like better (b) because he understands the subject,
(c) the white, the black (c) gets along well with his colleagues
(d) or the brown one? (d) and hard working

ww
32. (a) We have given up trying to reform him
(b) since we found that the more we advised him
(c) the least he was inclined
46. (a) The Minister said that he could not
(b) inaugurate the conference but promised
(c) that he would come a bit lately

w.E
(d) even to listen to us
33. (a) India was the country whom
(b) everyone thought would win the hockey cup
(d) and address the delegates
47. (a) Though his choice was neither economics or politics
(b) he had to study one of them,
(c) but, to the surprise of all,

asy
(d) it was eliminated in the first round
34. (a) Since neither my wife nor me
(c) because the science class had already been filled
(d) and there was no other hope for him
48. (a) He hanged his head in shame
(b) was willing to go to the movie with her
(c) my daughter had to sit at home
En (b) when he came
(c) to know of his
(d) and just watch the TV
35. (a) The Prime Minister declared emphatically
(b) in Parliament last week that gin (d) son’s failure.
49. (a) I shall always be
(c) wherever
(b) thinking of you
(d) I will go.
(c) his Government will not tolerate indiscipline
(d) in any public sector undertaking eer
50. (a) Excessive eating
(c) for health.
(b) is injurious
(d) No error
36. (a) The old movie shown today on the TV
(b) was better than any movie
(c) that had been shown
(c) were
ing
51. (a) John as well as

52. (a) Both the brothers


(b) his brother
(d) present.
(b) are extremely
(d) during the last eight months
37. (a) Is it true, I would like to know
(c) fond of
53. (a) Of the two schemes .ne
(d) one another.

(b) that less persons die of snake-bites


(c) than of traffic accidents
(d) in this country at present
38. (a) The student, who the friends have nicknamed
(b) put forward by the Government
(c) I think this is the one
(d) most likely to succeed.
54. (a) The teacher asked
t
(b) the students
(b) Jolly John, bunks classes often (c) that why many of them (d) had come without book.
(c) and goes to movies even after 55. (a) My choice (b) is quite
(d) repeated warnings by the teacher (c) different from (d) yours.
39. (a) Whenever I see a movie that 56. (a) Boys study in order (b) that they could
(b) has a tragic ending, (c) earn their (d) livelihood.
(c) I feel badly for at least two more days 57. (a) The man (b) who was
(d) and so I avoid them (c) killed he was (d) my cousin.
40. (a) Though I take coffee or tea, 58. (a) I was promoted
(b) whichever is offered, (b) to the post of principal though
(c) I consider coffee more preferable (c) I was junior than
(d) if I am given a choice (d) all the other members of the staff.
41. (a) After the option to study French was abolished, 59. (a) The manager of the bank
(b) the college attracted less students (b) together with his
(c) and the strength of our class (c) staff have
(d) is just thirty now (d) resigned.

Downloaded From : www.EasyEngineering.net


Downloaded From : www.EasyEngineering.net

94  l  Spotting Error

60. (a) He was reading (b) very hard for 78. Credit cards have (a) / brought about a revolutions (b) / in
(c) the last six months (d) still he failed. people’s spending habits. (c) / No error. (d)
61. (a) Every one of those 79. I informed the principal (a)/ that I was running temperature
(b) who came here are (b)/ and therefore could not attend the meeting. (c) / No
(c) foolish and error (d).
(d) cannot he relied. 80. The farmer is irrigating (a)/ his fields (b)/ since morning.
62. (a) We are not (b) to abuse (c)/ No error (d).
(c) our hardly won (d) liberty. 81. I am learning English (a)/ for ten years (b)/ without much
63. (a) My boss is (b) much angry effect (c)/ No error (d).
(c) with me (d) these days.
82. Inflation and shortages (a)/ have made it very difficult for
64. (a) I want a (b) better and
him (b)/ to make his both ends meet (c)/ No error (d).
(c) efficient (d) servant
65. (a) He walks (b) as though DIRECTIONS(Qs. 83-87) : Read each sentence to find out whether
(c) he was drink. (d) No error. there is any grammatical error in it. The error, if any, will be in one
66. (a) Transport has been arranged part of the sentence. The number of that part is the answer. If there
(b) to facilitate is no error, the answer is (e) i.e. ‘No Error”, (Ignore the errors of

ww
(c) the students
(d) in visting the exhibition.
67. (a) I have seen
punctuation, if any).
83.
[IBPS Clerk 2012]
If tomorrow is (a)/ declared a holiday, (b)/ we shall go (c)/
to a picnic. (d)/ No Error (e)

w.E
(b) my friend outside
(c) the cinema house
(d) last night.
68. (a) Unless
84.

85.
My grandfather used (a)/ to go (b)/ for a walk (c)/ every
morning. (d)/ No Error (e)
The blast from (a)/ the explosion (b)/ knocked the factory
(b) you have no objection
(c) I will come asy 86.
worker (c)/ to unconsciousness. (d)/ No Error (e)
Raju found it difficult (a)/ to explain (b)/ his final exam
(d) tomorrow.
69. (a) No less than
(b) four thousand People En 87.
marks (c)/ to his parents. (d)/ No Error (e)
My friend become (a)/terribly upset (b)/ after losing her
purse (c)/ at the supermarket. (d)/ No Error (e)
(c) lost their lives
(d) in the recent earthquakes. gin
DIRECTIONS(Qs. 88-92): Read this sentence to find out whether
there is any grammatical mistake/error in it. The error, if any, will
70. (a) It is amazing
(b) that how people
(c) are aware of the value eer
be in one part of the sentence. Mark that part with the error as your
answer. If the sentence is correct as it is, mark ‘No error’ as your
(d) of solitude and contemplation.
DIRECTIONS(Qs. 71–82):   Read each sentence to find out 88. ing
answer. (Ignore the errors of punctuation if any.) [IBPS Clerk 2013]
A red and sore tongue/is an indicator from/lack of iron
whether there is any grammatical mistake / error in it. The error if
any, will be any part of the sentence. Make the number of that part
Vitamin-B 12/in the body.
(a) a red and sore tongue .ne
with error as your answer. If there is 'No error' , mark (d).
71. An unit is an abstract idea, (a) / defined either by reference
to (b) / a randomly chosen material standard or to a natural
phenomenon. (c) / No error (d).
(b) is an indicator from
(c) lack of iron and Vitamin-B12
(d) in the body
(e) No error
t
72. A major contribution of Mathura sculptors (a) / of that 89. In the high-strung life/of over- crowded metros/there a
period were the creation and popularization (b) / of the constantly tug of war/over space and resources.
Buddha’s image in human form. (c) / No error (d). (a) in the high-strung life
73. The two books are the same (a) / except for the fact that his (b) of over-crowded metros
(b) / has an answer in the back. (c) / No error (d). (c) there a constantly tug of war
74. The Prime Minister’s good looks won him (a) / the election (d) over space and resources
but he has still to prove (b) / that he’s not a just pretty face. (e) No error
(c) / No error (d). 90. The foremost criterion of selection we adopted/were the
75. There is a beautiful moon out tonight (a) / and Neeta and I number of years of training/a singer had received/under a
are going for a stroll (b) / would you like to come along with particular guru.
she and I ? (c) / No error (d). (a) The foremost criterion of selection we adopted
76. Some women admit that (a) / their principle goal in life (b) (b) were the number of years of training
/ is to marry a wealthy man. (c) / No error (d). (c) a singer had received
77. Take two spoonsful (a) / of this medicine (b) / every three (d) under a particular guru
hours. (c) / No error (d). (e) No error

Downloaded From : www.EasyEngineering.net


Downloaded From : www.EasyEngineering.net

Spotting Error  l 95

91. Excess weight is the result of/unhealthy eating habits/which 100. My father gave me (a) / a pair of binocular (b) / on my
are inherent risk factors/responsible for many diseases. birthday. (c) / No error. (d)
(a) excess weight is the result of 101. Kalidas is (a) / a Shakespeare (b) /of India. (c) / No error.
(b) unhealthy eating habits (d)
(c) which are inherent risk factors 102. The teacher as well as his students, (a) / all left (b) / for the
(d) responsible for many diseases trip. (c) / No error. (d)
(e) No error 103. More you (a) / think of it, (b) / the worse it becomes.(c) /
92. The therapeutic benefits/at helping others/have long been/ No error. (d)
recognised by people.
(a) the therapeutic benefits (b) at helping others DIRECTIONS (Qs. 104-108) ): Some parts of the sentences have
(c) have long been (d) recognised by people errors and some are correct. Find out which part of a sentence has
(e) No error an error and blacken the oval corresponding to the appropriate letter
(a, b, c). If a sentence is free from errors blacken the oval
DIRECTIONS(Qs. 93 - 98) : Some parts of the sentences have corresponding to (d) in the answer sheet. [SSC CHSL 2014]
errors and some have none. Find out which part of a sentence has
an error and mark the appropriate answer. If a sentence is free from I worked as medical representative for eight months
104.
error, then mark answer (d). [SSC CHSL 2012] (a) (b) (c)

93. ww
It was he who came running in the house
(a )
/
(b)
/
No error
(d)

w.E
with the news about the earthquake. No Error.
(c)
/
(d)
105. Shakespeare has written
(a)
many plays
(b)

94.
(a )
/
asy
Her mother does not approve of her to go to the party
(b)
/
as well as some poetries
(c)
No error
(d)
without dressing formally. / No Error.
(c) (d)
En 106.
Neither of the girls were willing to
(a)
,
(b)
,

95. Riding across the battle field / the famous Bhishm /


(a ) (b) gin accept the proposal
(c)
No error
(d)
saw a large number of dead warriors. No Error.
(c)
/
(d)
107.
eer
A interesting book A Tale of two
(a)
,
(b)

96.
My Aunt was first to get a degree No Error.
(a )
/
(b)
/
(c)
/
(d)
,
ing
cities was written by Alexander Dumas No error
(c)
,
(d)

97.
Padmini had not rarely missed
(a )
/
(a) (b) (c) .ne
108. In India , there are , many poors , No error
(d)
a dance performance or festival since
(b)
she was eight years old. No Error.
/
/ DIRECTIONS (Qs. 109-113) : Some parts of the sentences have
t
errors and some have none. Find out which part of a sentence has
an error. If there is no error, your answer is (d).
[SSC Sub Insp. 2012]
(c) (d)
109. Having lived / in Kerala for ten years, /
Krupa and Kavya studied in the Delhi Public School (a) (b)
98. / /
(a ) (b) my friend is used to speak Malayalam with his friends. /
and so does Kamya. No Error. (c)
/ No Error
(c) (d)
(d)
DIRECTIONS(Qs. 99 - 103) : Some parts of the sentences have 110. Much water / has flown / under this bridge / No error
errors and some are correct. Find out which part of a sentence has (a) (b) (c) (d)
an error and blacken the oval – corresponding to the appropriate 111. The law should specifically / provide a clause /
letter (a, b, c). If a sentence is free from error, blacken the oval (a) (b)
corresponding to (d) in the Answer Sheet. [SSC CHSL 2013] to protect animals from poachers / No error
99. Hasan plays (a) / both-cricket and billiards (b) /at the national (c) (d)
level. (c) / No error. (d) 112. What kind / of a man / are you? / No error.
(a) (b) (c) (d)

Downloaded From : www.EasyEngineering.net


Downloaded From : www.EasyEngineering.net

96  l  Spotting Error

113. The lady approached me timidly /and trembling slightly / DIRECTIONS (Qs. 124-133): In the following questions, some
(a) (b) parts of the sentences have errors and some are correct. Find
she sat down besides me. / No error out which part of a sentence has an error and blacken the oval
(c) (d) corresponding to the appropriate letter (a, b, c). If a sentence is free
from error, blacken the oval corresponding to (d) in the Answer Sheet.
DIRECTIONS (Qs. 114-118): In the following questions, some
parts of the sentences have errors and some are correct. Find out [SSC Multi tasking 2013]
which part of a sentence has an error. The number of that part is the 124. I and him / are / very good friends. / No error.
answer. If a sentence is free from error, then your answer is (d). i.e., (a) (b) (c) (d)
No error. [SSC Sub Insp. 2013]
125. One should / look after / their parents. / No error.
114. When one hears of the incident (a)/ about the plane crash (a) (b) (c) (d)
(b)/ he feels very sorry. (c)/ No error (d) 126. She placed / the offering / to God in the altar. / No error.
115. I went there (a)/ with a view to survey (b)/ the entire (a) (b) (c) (d)
procedure. (c)/ No error (d)
127. Teachers were instructed / to follow an uniform method / of
116. It had laid (a)/ in the closet (b)/ for a week before we found
it. (c)/ No error (d) (a) (b)
117. He was present (a)/ in the court (b)/ to give witness. (c)/ No evaluation. / No error.

ww
error (d)
118. He laughed (a)/ her (b)/ as she fell off the tree. (c) / No error
(d)
(c) (d)
128. The newspapers they admit that / advertising sometimes /
(a) (b)

w.E
DIRECTIONS (Qs. 119-123): In the following questions, some
parts of the sentences have errors and some are correct. Find
out which part of a sentence has an error and blacken the oval
influences their editorial policy. / No error.
(c) (d)
129. No sooner did I finish / my speech, I was subjected / to a

asy
corresponding to the appropriate letter (a, b, c). If a sentence is free
from error, blacken the oval corresponding to (d) in the Answer Sheet.
(a)
barrage of questions. / No error.
(c) (d)
(b)

In 1906 a earthquake destroyed much En


[SSC Sub Insp. 2014] 130. I saw him / coming out of the hotel / on 10 o’clock. / No error.
(a) (b) (c) (d)
119.
(a)
/

of San Francisco. No error.


(b)
/

gin
131. One of my friend / is returning / to India from the U.S.A. /
(a) (b) (c)

(c)
/
(d)

His parents does not approve of


No error.
(d)
eer
132. He knows / that your muscles / are not same as his. /
120.
(a)
/ (b) (a)
No error. ing
(b) (c)

his business No error.


(c) / (d)
(d)

.ne
133. We shall wait / till you / will finish your lunch. / No error.

121.
The college library is not only equipped with
(a)
/
(b)
/

very good books but also with the latest journals.


/
(a) (b) (c)

t
DIRECTIONS(Qs. 134 - 135) : There are four words out of which
one is correctly spelt. Find the correctly spelt word :
(d)

(c) [SSC CHSL, 2012]


No error. 134. (a) Manoeuvre (b) Manueover
(d) (c) Manuovere (d) Maneouvre
besides each other 135. (a) Venerable (b) Vanerable
122. The lovers walked / / (c) Veneruble (d) Venarable
(a) (b)
DIRECTIONS (Qs. 136-137): Four words are given in each
in silence. No error. question, out of which only one word is correctly spelt. Find the
/
(c) (d)
correctly spelt word and mark your answer in the Answer Sheet.
for the army,
123. Men are wanted / /
(a) (b) [SSC CHSL 2013]
and the navy, and the air force, No error. 136. (a) Dysentary (b) Dysantery
/ (d)
(c) (c) Dysentry (d) Dysentery

Downloaded From : www.EasyEngineering.net


Downloaded From : www.EasyEngineering.net

Spotting Error  l 97

137. (a) Rejevanation (b) Rejuvenation 148. (a) Resillient (b) Persuade
(c) Rejvenation (d) Rejuenation (c) Dubious (d) Depplete
149. (a) Ineffectual (b) Iniffectual
DIRECTIONS (Qs. 138-142): Four words are given ineach (c) Inefecttual (d) Inefictual
question, out of which only one word is correctly spelt. Find the 150. (a) Massacer (b) Massecre
correctly spelt word and mark your answer in the Answer Sheet. (c) Masacre (d) Massacre
151. (a) Conciance (b) Consience
[SSC CHSL, 2014]
(c) Conscience (d) Connscience
138. (a) Mountainer (b) Mountaineer
152. (a) Nirvana (b) Nirvena
(c) Mounteener (d) Mountineer (c) Nirvanna (d) Nyrvana
139. (a) Happened (b) Happenned
DIRECTIONS (Qs. 153-158): In the following questions, there
(c) Hapened (d) Hapenned
are four different words out of which one is correctly spelt. Find the
140. (a) Sentimantalist (b) Sentimentelist correctly spelt word. [SSC Multi tasking 2013]
(c) Sentimentalist (d) Santimentalist
153. (a) exellence (b) excellence

ww
141. (a) Laibertarian
(c) Liebertarian
(b) Libertarian
(d) Liberterian
(c) excellencce
154. (a) grammar
(d) exillance
(b) grammer

w.E
142. (a) Emphetic
(c) Emphatick
(b) Emphattic
(d) Emphatic

DIRECTIONS (Qs.143-147): In the followoing questions, there


(c) gramer
155. (a) ommited
(c) omitted
(d) gramar
(b) ommitted
(d) omited

asy
are four different words out of which one is correctly spelt. Find
the correctly spelt word.
156. (a) calender
(c) colendar
(b) calandar
(d) calendar

En
[SSC Sub Insp. 2012]
157. (a) objectionable
(c) objecktionable
(b) objectioneble
(d) objectionablle

143. (a) pursuasive


(c) persuesive
(b) persuasive
(d) persuasieve gin
158. (a) apollogy
(c) apalogy
(b) appology
(d) apology

144. (a) assendency


(c) ascendancy
(b) ascendency
(d) ascendensy eer
DIRECTIONS (Qs. 159-163) : In the following questions, four
words are given. In each group, out of which only one word is

145. (a) anathema


(c) anathemaa
(b) annathema
(d) anathima
ing
correctly spelt. Find the correctly spelt word.
[SSC Sub Insp. 2013]

146. (a) quaint (b) qauint


159. (a) rhythym
(c) rhythim .ne
(b) rhithim
(d) rhythm
(c) quiant
147. (a) effervesent
(c) effervescent
(d) quaaint
(b) efervescent
(d) efferescent
160. (a) indeganeous
(c) indegenous
161. (a) saccarine
(b) indigenous
(d) indigeneous
(b) sacarine
t
(c) sachharine (d) saccharine
DIRECTIONS (Qs. 148-152): Four words are given in each
question, out of which only one word is correctly pelt. Find the 162. (a) revolutionize (b) revoulutionize
correctly spelt word and mark your answer in the Answer Sheet. (c) revvolutionize (d) revollutionize
163. (a) disentry (b) dysentry
[SSC Sub Insp. 2014]
(c) diesentry (d) dysentery

Downloaded From : www.EasyEngineering.net


Downloaded From : www.EasyEngineering.net

98  l  Spotting Error

LEVEL-II

12. (a) The chairman reviewed the many details


DIRECTIONS (Qs. 1-40):  In each of these questions, a sentence
(b) connecting with the profitability
has been divided into four parts and marked a. b, c and d. One of
these parts contains a mistake in grammar Idiom or syntax. Identify (c) of the Company and then decided that
that part and mark it as the answer. (d) further expansion was not desirable
13. (a) On entering the meeting hall
1. (a) The only persons in the theatre (b) loud cheers greeted the Prime Minister
(b) on that stormy night (c) who acknowledged them with a smile
(c) were the staff of the theatre (d) and waved back happily at the gathering
(d) and me 14. (a) I am sure that if you were me
2. (a) Sunita is more talkative than (b) and had been talked to in a similar manner
(b) anybody in the class because (c) you would also have lost your temper
(c) he is not afraid of the teacher (d) and talked back as I did

3. ww
(d) who is his own brother
(a) There is only the banana
(b) and one apple in the refrigerator
15. (a) He would not listen to us at all
(b) and it was quite apparent that
(c) he had other different sources of information

4.
w.E
(c) so let us go to the market
(d) and buy some more fruits
(a) Like his brother who did not wear his helmet
(d) than what we were relying on
16. (a) We were shown two houses yesterday
(b) by the broker who is helping us in this regard

(b) and was injured in the accident


(c) Rajan was always careful asy (c) but we found that none of them was suitable
(d) for our specific requirements
17. (a) Although politicians may be dishonest

5.
(d) and wore his helmet without fail
(a) We were not worried about being late En (b) in their professional life
(c) it is probably unfair to brand them
(b) since we knew that our other friends
(c) would have been caught in a worse traffic jam
gin (d) as dishonest in their private dealings too
18. (a) I had no particular fancy for tea
(b) or coffee; so, when given a choice
6.
(d) than us
(a) The part of Madras that interested us the most
(b) were the beach and the museum eer
(c) I opted for a cold drink which
(d) I am fond of
(c) which we recommend to all friends
(d) who plan to visit that city ing
19. (a) If only I would have studied that one chapter
(b) just on the eve of the examination
(c) I could have answered all the questions
7. (a) Although Greek and Latin were
(b) of extreme important during their day .ne
(d) and obtained over eighty percent marks
20. (a) Despite all the threatening and cajoling

8.
(c) they had become dead languages
(d) by the beginning of the fifteenth century
(a) Amphibians are creatures which
(b) live equally effortlessly
(b) the accused denied to disclose
(c) who his accomplices were t
(d) in the bank robbery (theft of the bank’s cash).
21. (a) What other steps can be taken
(c) in water and land are found (b) to expedite the recovery of loans
(d) in all the continents of the world (c) are being discussed by the Bank Chairman
9. (a) Near the pond was standing (d) at their meeting in Bombay
(b) a dog, a donkey and a cow 22. (a) If I knew all the facts,
(c) but when I threw a stone at them (b) I would not have stood bail for him
(d) it was only the dog that ran away (c) and defended him to my friends who
10. (a) Psychiatrists claim that the dream process (d) had warned me to keep away from him
23. (a) Though the elephant, because of its size
(b) can offer insights into
(b) appears to be a rather slow animal
(c) how the brain has worked, though (c) it can in fact run faster than
(d) it cannot be taken as the final evidence (d) any other man
11. (a) Collecting money for the new school 24. (a) The University has announced that
(b) may not be very easy (b) everyone who takes the examination now
(c) but if everyone does their best (c) will receive their results within
(d) we can still reach the target (d) eight weeks at the latest

Downloaded From : www.EasyEngineering.net


Downloaded From : www.EasyEngineering.net

Spotting Error  l 99

25. (a) Most labour disputes can be solved amicably 43. I will try to put over (a) / some feelers to gauge (b) / people’s
(b) if it is only realized reactions to our proposal. (c) / No error (d).
(c) that the long term interest of both management and 44. She stood off (a) / from the crowd (b) / because of her height
labour and flaming red hair. (c) / No error (d).
(d) are really the same
45. The data on (a) / the divorce case is (b) / on the judge’s desk.
26. (a) He declined the invitation
(c) / No error (d).
(b) to participate in the meeting this evening
(c) saying that he shall have an urgent business 46. Your husband doesn’t (a) / believe that you are older (b) /
(d) he has to personally attend to than I. (c) / No error (d).
27. (a) Though he claimed he knew French, 47. You should be cautious (a) / and make a few discrete
(b) I was sure that he only could follow it enquires about (b) / the firm before you sign anything. (c) /
(c) but could not either write No error (d).
(d) or speak it with fluency 48. I will need several weeks (a) / to invent the lie of the land
28. (a) He always took (b) Pride of before (b) / I can make any decision about the future of the
(c) the high standard (d) of his work. business. (c) / No error (d).
29. (a) The Socialist party is (b) as good if not better 49. The salesman gave us (a) / a big spiel about why (b) / we
(c) than any other (d) political party. should buy his product. (c) / No error (d).

ww
30. (a) The reason for the train being late
(b) was because
(c) the train was involved
50. It’s stupid to go (a)/ to the expense of taking (b)/ music
lessons if you never practice (c)/ No error. (d)
51. You will find it difficult (a)/ to explain of your use (b)/ of

w.E
(d) in an accident.
31. (a) They found themselves
(b) in competition with
such offensive language (c)/ No error. (d)
52. Because of the (a)/ extenuating circumstances, (b)/ the court
acquitted him out of the crime (c)/ No error (d).
(c) men who were as good
asy
(d) if not better than themselves. No error
32. (a) He never has (b) and never
53. The company has (a)/ set off itself some stiff production (b)/
goals for this year (c)/ No error. (d)
(c) will play
33. (a) I think
(d) at cards.
(b) every one of En 54. The music was so loud (a) / that we had bellow over each (b)
/ other to be heard. (c) / No error (d).
(c) these men are
34. (a) If I fail
(c) I shall give
(d) incompetent.
(b) in this examination,
(d) the next examination. gin
55. The children are (a) / really in their element (b) / playing on
the beach. (c) / No error (d).
56. The film was so disjointed (a) / that I could not tell you (b) /
35. (a) He hanged his
(b) head in shame eer
what the story was about. (c) / No error (d)
57. He had been (a) / saved of death as if (b) / by divine
(c) when he came to know of his
(d) son’s mischief.
36. (a) I will ing
intervention. (c) / No error (d).
58. A cogent remark (a) / compels acceptance because (b) / of
their sense and logic. (c) / No error (d)
(b) never forget the kindness
(c) you have shown

.ne
59. In financial matters (a) / it is important to (b) / get
disinterested advice. (c) / No error. (d)
(d) to me.
37. (a) He is such a fool
(c) nonsense.
38. (a) He started early
(b) who will talk any
(d) No error.
(b) because
error (d)
t
60. I could not (a)/ answer to (b)/ the question. (c) No error (d)
61. Two years passed (a)/ since (b)/ my cousin died. (c)/ No

62. Have you gone through (a)/ either of these three chapters
(c) he may (d) not get late.
39. (a) It is easy (b) distinguishing (b)/ that have been included in this volume? (c)/ No error
(c) this pen (d) from that. (d).
40. (a) Every flower and (b) every leaf 63. Ramesh has agreed (a)/ to marry with the girl (b)/ of his
(c) proclaim the (d) glory of God. parent's choice. (c)/ No Error.
64. When he was arriving (a)/ the party was (b)/ in full swing.
DIRECTIONS (Qs. 41–77): Read each sentence to find out whether (c)/ No Error. (d)
there is any grammatical mistake / error in it. The error if any, will
65. The most studious b oy (a)/ in the class (b) was made as the
be any part of the sentence. Make the number of that part with error
captain (c)/ No error (d).
as your answer. If there is 'No error', mark (d).
66. I am participating (a) / in the two-miles race (b) / tomorrow
41. Microwaves are the principle carriers (a) / of television, morning (c)/ No error (d).
telephone and data transmissions (b) / between stations on 67. The sum and substance (a)/ of his speech (b)/ were essentially
earth and between the earth and satellites. (c) / No error (d). anti-establishment (c)/ No error (d).
42. Mahavira was an advocate of nonviolence and vegetarianism, 68. It has been such a wonderful evening, (a)/ I look forward to
(a) / who revived and recognized the Jain doctrine (b) / and meet you again (b)/ after the vacations (c)/ No error (d).
established rules for their monastic order. (c) / No error (d).

Downloaded From : www.EasyEngineering.net


Downloaded From : www.EasyEngineering.net

100  l  Spotting Error

69. When the boy committed a mistake, (a)/ the teacher made As the prison will get
him to do (b)/ the sum again (c)/ No error (d). 82. /
(a)
70. Unless the government does not revise its policy of
liberalization (a)/ the growth of the indigenous technology an official telephone facility soon, the prisoners
/
(b)/ will be adversely affected (c)/ No error (d). (b)
71. Whenever a person lost anything (a)/ the poor folk around wont have to make calls in discreet manner
(b)/ are suspected. (c)/ No error (d) (c)
72. Everyday before (a)/ I start work for my livelihood
(b)/ I do my prayer (c)/ No error (d) through smuggled mobile phones No error
73. Pooja went to her friend’s house at the appointed hour; but (d) / (e)
(a)/ she was told (b)/ that her friend left half an hour earlier
83. The area was plunged into / darkness mid a wave of /
(c)/ No error (d)
74. Rekha is (a)/ enough old (b)/ to get married (c)/ No error (d) (a) (b)
75. As far as I am concerned, (a)/ I shall do everything (b)/
cheering and shouting slogans like 'Save The Earth'.
possible to help you (c)/ No error (d)
(c) / /
76. Let us congratulate him (a)/ for his success (b)/ in the (d)
No error
examination (c)/ No error (d)

ww
77. Many people prefer to travel (a)/ by the road (b)/ because it
is less expensive (c)/ No error (d) 84.
(e)
The poll contestants apporached /

w.E
DIRECTIONS: (Qs. 78-87) : Read each sentence to find out
whether there is any grammatical error or idiomatic error in it. The
error,if any, will be in one part of the sentence. The letter of that part
(a)
the commission complaining that the hoardings
(b) /

of punctuation, if any).
asy
is the answer. If there is ‘No error’, the answer is ‘(e)’. (ignore errors
violated the code of conduct
(c)
/

78. The Government has asked individuals/


(a)
[IBPS PO 2012]

En and influenced public perception. No error


(d) / (e)
With income of over 110 lakhs to/
(b)
electronic file tax returns for the year 2011-12/
gin
85.
The country has adequate laws but problems
(a)
/
(b)
/

(c)
Something which was optional till last year. No error eer
arise when these are not /
(c)
79.
(d) (e)
The power tarrif had already/ been increasesd twice in/
ing
implemented in letter and spirit. No error
(d)
/
(e)
the last 15 months and the Electricity Board had also
(c)
/ 86. The Management feels that /
(a) .ne
levied additional monthly charges to consumers.

No error
(e)
(d)
the employees of the organisation are
(b)
/

The poll contestants apporached to work hard.


(a)
/ (d) /
t
80. Despite of curfew / in some areas, minor / No error
(a) (b) (e)
As far the issue of land encroachment
communal incidents were reported 87. /
(a)
(c) / in villages is concerned, people will
from different areas of the walled city. /
(b)
(d) / have to make a start from their villages by
/
81. This comes / at a time / when fund allocation / (c)
(a) (b) (c) sensitising and educating the villagers about this issue. .
No error (d)
is been doubled. No error
/ (e) (e)
(d)

Downloaded From : www.EasyEngineering.net


Downloaded From : www.EasyEngineering.net

Spotting Error  l 101

Directions (Qs.88-92): Read each sentence to find out whether 102. Reetu was a model / (a) and so she would / (b) starve herself
there is any grammatical mistake/error in it. The error if any, will to maintain / (c) his body weight/ (d). No Error (e).
be in any part of the sentence. Mark the number of that part with
error as your answer. If there is ‘No error’, mark (e). DIRECTIONS (Qs. 103-107) : In the following questions some
parts of the sentences have errors and some have none. Find out
[SBI PO 2013] which part of a sentence has an error and blacken the rectangle
88. There cannot be any situation where/(a) somebody makes corresponding to the appropriate letter (a, b, c). If there is no error,
money in an asset/ (b) located in India and does not pay tax/ blacken the rectangle corresponding to (d) in the Answer-Sheet.
(c) either to India or to the country of his origin./ (d) No error
(e). [SSC CGL 2012]
89. India has entered a downward spiral / (a) where the organised, 103. Air pollution, together with littering ,
productive/ (b) and law abide sectors are subjec to / (c) a
sevage amounts of multiple taxes./ (d) No error (e).
90. The bank may have followed/ (a) an aggressive monetary are causing many problems in our cities. No error.
tightening policy/ (b) but its stated aim of / (c) curbin inflation c d
have not been achieved/ (d) No error (e).
b
91. Equal opportunities for advancement/ (a) across the length
and breadth / (b) of an organisation will/ (c) keep many 104. The accused refused to answer to the policeman

ww
problems away. /(d) No error (e).
92. A customised data science degree/ (a) is yet to become/(b)
a standard programme/ (c) to India’s premier educational
a

on duty. No error.
b

w.E
institutes./ (d) No error (e)
DIRECTIONS (Qs. 93-97) : Read each sentence to find out whether
there is any grammatical error or idiomatic error in it. The error, 105.
c d

What is the use of me attending the session? No error.

asy
if any, will be in one part of the sentence. The letter of that part is
the answer. If there is “No Error” the answer is (e). (Ignore errors
of punctuation if any.)
106.
a b c d
We met our prospective employer, for a briefing session
a b

En
[SBI Clerk 2012]
in the Taj Hotel. No error.

93. The shepherd counted (a) / his sheep and found (b) / that
one of (c) / them is missing. (d) / No Error (e)
gin c d
107. Because of the severe snow storm and the road blocks,
a
94. The teacher were (a) / impressed by her performance (b) /
and asked her to (c) / participate in the competition. (d) No eer
the air force dropped food and
Error (e)
95. She asked her (a) / son for help her (b) / find a place to bury
(c) / the gold ornaments (d) No Error (e)
ing b
medical supplies close to the city. No error.
c d
96. The painter was (a) / ask to paint a (b) / picture of the king,
(c) / sitting on his throne (d) No Error (e) .ne
DIRECTIONS (Qs. 108-112) : In the following questions four
97. The story was (a) / about how an (b) / intelligent man had
saving (c) / himself from being robbed (d) No Error (e)
DIRECTIONS (Qs. 98-102) : Read each sentence to find out
Answer-Sheet by blackening the appropriate rectangle t
words are given in each question, out of which only one word is
correctly spelt. Find the correctly spelt word and indicate it in the

[SSC CGL 2012]


.

whether there is any grammatical error in it. The error, if any, will
be in one part of the sentence. The number of that part is the answer, 108. (a) garulous (b) garrulous
if there is no error the answer is (e) i.e., ‘No Error’ (Ignore errors of (c) garullous (d) garrullous
punctuation, if any). 109. (a) marquee (b) markue
(c) marquei (d) marquie
[SBI Clerk 2012] 110. (a) puissant (b) puiscant
98. Reshma was /(a) a daily wage worker/ (b) who gone to work/ (c) puiscent (d) puissent
(c) despite having high fever (d). No Error (e). 111. (a) disconncerting (b) disconserting
99. Radhika was very upset /(a) because she had to goes/ (b) to (c) discuncerting (d) disconcerting
112. (a) exilarate (b) exhilerate
a boarding school / (c) so she cried the entire night. (d). No
Error (e). (c) exsilarate (d) exhilarate
DIRECTIONS (Qs. 113-114) : In the following questions, four
100. Although Shanku was born/ (a) in a rich merchant’s families/
words are given in each question, out of which only one word is
(b) he was very humble and/ (c) was a philanthropist by
nature. (d). No Error (e). correctly spelt. [SSC CGL 2012]
101. Professor Shastri were/ (a) a loved man and/ (b) had students 113. (a) milennium (b) millenium
visiting / (c) him every day. (d). No Error (e). (c) milleneum (d) millennium

Downloaded From : www.EasyEngineering.net


Downloaded From : www.EasyEngineering.net

102  l  Spotting Error

114. (a) ocassion (b) occassion


(c) occasion (d) occation and integrity. /
DIRECTIONS (Qs. 115-119 : In the following questions, some
parts of the sentences have errors and some are correct. Find out 122. / / /
which part of a sentence has an error. If a sentence is free from error,
your answer is (d) i.e. No error.
123. / /
[SSC CGL 2013]

115. / /

/ /

116. / / 124. / / /

/ ww DIRECTIONS (Qs. 125-126) : Four words are given in each


117. w.E
The Headmaster with all his senior teachers
(a)
question, out of which only one word is correctly spelt. Find the
correctly spelt word and mark your answer in the Answer Sheet.

/ /
asy / 125. (a)
(c)
Plebeian
Plebian
[SSC CGL 2014]
(b) Plibeian
(d) Plebiean
118. /
En 126. (a)
(c)
Suroundings
Sarroundings
(b) Surroundings
(d) Surondings

/ / gin
DIRECTIONS (Qs. 127-131) : Read each sentence to find out
whether there is any grammatical error in it. The error if any, will

119. / / / eer
be in one part of the sentence, the number of that part is the answer.
If there is no error, mark (e). (Ignore errors of punctuation. if any)

DIRECTIONS (Qs. 120-124) : In the following questions, some ing [SBI PO 2014]
127. In the first two months of this fiscal, tractor sales has seen
(a)/a drop of about five percent (b)/ however, the industry
parts of the sentences have errors and some are correct. Find
out which part of a sentence has an error and blacken the oval
corresponding to the appropriate letter (a, b, c). If a sentence is free .ne
is waiting for the monsoon (c)/ to really arrive at a firm
conclusion about growth prospects for the current year. (d)/
from error, blacken the oval corresponding to (d) in the Answer Sheet.

120.
[SSC CGL 2014]
No error (e)
t
128. Dolphins are truly out of the ordinary because of their
intelligence (a) / and. among the many creatures that share
the earth form (b)/they come closest to humankind in terms
of (c)/familial traits, emotions and learining. (d)/ No error
(e)
/ / 129. Corruption indulged in by the high and mighty adversely
impacts (a)/ our nation, and in the coming months (b) / we
may see revival of efforts (c)/ to tackle such large scale
corruption. (d)/ No error (e)
/
130. It is notable and welcome that the ministry of (a)/
environmental and forests is to issue approvals online (b)/
in a time bound manner, with clear timelines (c)/in place
for the various sub-steps along the way. (d)/No error (e)
121. / / 131. To portray (a)/ what a fairness cream does without (b)/ any
sort of comparison or visual (c)/ references are very difficult.
(d)/ No error (e)

Downloaded From : www.EasyEngineering.net


Downloaded From : www.EasyEngineering.net

Spotting Error  l 103

Hints & Solutions


LEVEL- I 20. (b) is expected to be faster
21. (c) has indicated his unwillingness
1. (d); There is a verb tense error here. In referring to anybody, everybody, everyone, anyone, each etc.,
2. (a); There is an incorrect use of ‘owing to’. the pronoun of the masculine or the feminine gender is used
3. (a); ‘Pickpocketers’ is not the correct word. The correct one is
according to the context but when the sex is not determined,
‘Pickpockets’ and hence option (a) is right.
we use the pronoun of the masculine gender, as there is no
4. (a); Option (a) is our answer, because ‘his’ is a possessive
singular pronoun of the third person to represent both male
pronoun and it should come next to the noun it is referring to,
i.e. his would come just before hands. and female, as, Each must do his best.
5. (b); As the given sentence is describing a continuous action 22. (a) The creatures on Mars, if any,
in the past, ‘fall’ should be replaced with ‘falling’ back. Hence, The plural of ‘creatures’ is used to agree with the verb ‘are’.
23. (b) .... Note its peculiar

ww
option (b) is correct.
6. (c); Option (c) is correct, because it is redundant and there is
no use of it. The word ‘exceed’ is enough to convey that the
The third person, neuter gender, possessive case is ‘its’. There is
no need for the apostrophe unless one says “the bird’s markings”.

w.E
short story should not have more than 200 words.
7. (a) If you read …..
The indefinite pronoun ‘one’ or the personal pronoun ‘you’
should he used throughout the sentence.
24. (c) ....of Florida was
The sentence is in Indirect Speech (as the reporting verb “said”
indicates) and hence a past tense in the principal clause is

8. (b) why she had not gone...


asy
As per the rules of syntax the subject comes before the verb and
followed by a past tense in the subordinate clause.
25. (b) has been sent..
Some nouns which are plural in form, but singular in meaning,
not after.
9. (a) In a tropical climate....
En
The article ‘a’ is used as “tropical climate” here is being used
take a Singular verb; as, The news is true.
26. (a) Many of the
generically. One should either say “a tropical climate” or
“tropical climates”. gin ‘Many’ and ‘much’ both are comparative adjectives but the
adjective ‘many’ is generally used for number and the adjective
10. (a) Pollution affects
The verb affects is to be used here which means “to produce an
effect on’ eer
‘much’ for quantity.
27. (d) how it eats
The simple present is used to express habitual actions.
11. (b) gone not only to Delhi
‘Not only – but also are correlative conjunctions and when ing
28. (d) in studies, public speaking and sports.
The word ‘both’ is used to refer to two things.
used, should be followed by the same part of speech.
12. (b) as effective as
13. (a) If you had come earlier...
29. (d) if he had not taken the drug

.ne
The action of taking the drug occurred before he died.

The past tense is to be used here.


14. (d) among themselves.
The preposition ‘between’ is used when two things are
considered whereas ‘among’ is used to qualify more than two
30. (c) has had any …….

t
The singular form ‘has’ is to be used as both his father and
his teacher refer to the singular (third person) Two or more
singular subjects connected by or, nor, neither – nor, either—
things or people. or, take a verb in the singular.
15. (a) Did he say he would... 31. (b) do you like best
‘Would’ is used as the past equivalent of ‘will’ when reporting The word ‘better is a Comparative Adjective The Superlative
in Indirect Speech. ‘best’ is used when more than two things (or sets of things) are
16. (a) has increased.... compared.
Often, by what is called the “Error of Proximity”, the verb is 32. (c) the less he was...
made to agree in number with a noun near it instead of with its ‘More’ is a comparative adjective and should be followed by the
proper subject. This should be avoided. comparative form of ‘little’ (i.e. less) and not the superlative.
17. (c) an hour. 33. (a) ……the country which
‘An’ is used before a word beginning with a vowel sound. The relative pronoun ‘which’ is used for things without life and
18. (a)....is a transparent liquid.
for animals. It may refer to a singular or plural noun. Here it is
The adjective ‘transparent’ is to be used here and not the noun
‘transparency’. introducing the defining adjective clause.
19. (d) in its unique splendour. 34. (a) ……my wife nor I
The Himalayas is referred to as a singular noun. 35. (c) his Government would not……
In indirect Speech ‘will’ is changed into ‘would’.

Downloaded From : www.EasyEngineering.net


Downloaded From : www.EasyEngineering.net

104  l  Spotting Error

36. (b) ……better than any other movie 48. (a) It should be ‘hung’; ‘hanged’ means ‘put to death’.
When a comparison is instituted by means of a Comparative 49. (d) It should be ‘wherever I go’
followed by than the thing compared must be always excluded 50. (c) It should be ‘injurious to
from the class of things with which it is compared, by using 51. (c) ‘As well as ‘is followed by a singular verb so it should be was
other or some such words. present
37. (b) that fewer persons die.... 52. (d) In case of ‘both ‘we use ‘each other’ in place of ‘one another’.
She adjective ‘less’ refers to quantity and the adjective ‘few’ is 53. (a) It should be ‘out of the two schemes’. This is the one more
used to denote number. likely.
38. (a) The student, whom the …… 54. (c) Remove ‘that’ before ‘why’
The relative pronoun ‘whom’ is used with the Accusative and the
55. (d) Your’s or that of your.
relative pronoun ‘who’ with the Nominative. When a pronoun
56. (b) It should be 'in order to'
or, noun) is used as the Subject al a verb if a raid to be in the
Nominative case and when it is used as the Object of a verb it 57. (c) ‘he’ is superflous, omit it.
is said to be in the Accusative Case. To find the Nominative, 58. (c) It should be ‘junior to’
put ‘Who/What’ before the verb and to find the Accusative put 59. (c) Use ‘has’ in place of ‘have’
‘Whom/What’ before the verb and its subject. 60. (a) Misuse of Tense
Whom\What have the friends (subject) nicknamed as Jolly It should be ‘had been’ in place of was.

ww
John (object)?
It is the relative pronoun who which has different forms for
Accusative (i.e. whom) and Genitive (i.e. whose).
The case of a relative pronoun depends upon the use oi the
61.
62.
63.
(b) It should be ‘who come’ here ‘is’.
(c) Use adj. form, hard in place of adverbial form ‘hardly’.
(b) Instead of ‘much angry’ it should be ‘very angry

w.E
pronoun in the clause which it Genitive (i.e. whose) introduces
—not upon the case of its antecedent. Compare the following
two sentences in which the antecedent (i.e. the noun to which
64.

65.
(c) It should ‘more efficient’ both the adj. should be in
comparative degree
(c) It should he ‘as though he were drunk’, because he was not

asy
the pronoun refers or relates) is in the Nominative case but
the pronouns are in the nominative and accusative cases,
respectively, because of their use in their own clauses.

66.
drunk in the real sense.
(d) It should be ‘for visiting instead of ‘in visiting’ the
exhibition.

En
This he who came yesterday. (nominative—subject of the verb
came)
67.
68.
(a) It should be either “I had’ or ‘I saw’
(b) ‘No’ should not be used, because the word ‘unless’ itself is
This is he whom you saw. (accusative — object of saw)
The student who is selected must be reliable. (subject of the
verb is selected) gin
69.
70.
giving a Negative sense.
(a) It should be ‘no fewer’, no less is used for quantity
(b) ‘that’ to be avoided as double contraction are not used in a
The student whom you select must be reliable. (object of the
verb select)
Whom do you want? (object of the verb do want)
71.
eer
singular clause.
(a) In the following question the use of article ‘an’ before ‘unit’

Who do you think will be there? (subject of the verb will be)
39. (c) I feel bad for at ……
Some adverbs have two forms, the form ending in - ly and the
ing
is wrong. The letter under this part is (a) ; so (a) is the correct
answer. The reason being ‘an’ is used before singular countable
nouns which begin with a vowel sound (a, e, i, o, u). The vowel
form which is the same as the adjective.
For e.g. He sings very loud.
72. .ne
‘u’ in unit sound like ‘you’ i.e. this word sounds as if it begins
with the consonant ‘y’. So we use ‘a’ before them.
(b) The word ‘were’ is wrongly used. The letter under this part
He sings very loudly.
The word ‘bad’ is appropriate in the sentence given.
40. (c) I consider coffee preferable to tea
The word ‘preferable’ has the force of a Comparative (Hence the
t
is (b), therefore, (b) is the correct answer. The correct form
would be ‘ of that period was the creation and popularization’
the reason being the subject of ‘were’ in the sentence is ‘A major
contribution’, which is singular. So, ‘was’ should be used to
Comparative’ more’ is redundant) and is generally followed by
the preposition ‘to’. make the sentence grammatically correct.
41. (b) attracted fewer students 73. (c) The preposition ‘in’ is wrongly used. The letter under this
42. (b) either of the two.... part is (c), therefore, (c) is the correct answer. The correct form
Either means any one. will be – “has an answer at the back”.
43. (b) ……customers who deposit.... 74. (c) The error lies in part (c) of the sentence, therefore, (c) is
44. (b) is unanimous in his view the correct answer. The correct form will be – that he’s not just
45. (d) and is hard-working/works hard a pretty face”.
The clause here needs the verb ‘is’ to complete it. 75. (c) In this particular question, the phrase ‘she and I’ it wrongly
46. (c) ....would come a bit late used. The letter under this part is (c), therefore, (c) is the
Late means after the due / usual / proper time. Lately is an
correct answer. The correct sentence will be. “There is beautiful
adverb meaning not long ago\recently.
moon out tonight and Neeta and I are going for a stroll would
47. (a) neither economics nor politics
Some conjunctions are always used in pairs. They are called you like to come along with us?”
Correlative Conjunctions, as; 76. (b) Here word ‘principle’ in option (b) need to be replaced
Either……or; Neither……nor; Both……and; Though……yet; by ‘principal’ since, this principal indicates main aim which
Whether— or; only……but also. suggests the right meaning of the sentence.

Downloaded From : www.EasyEngineering.net


Downloaded From : www.EasyEngineering.net

Spotting Error  l 105

77. (a) Here ‘spoonsful’ word need to be replaced with ‘spoonfuls’. 119. (a) In 1906 an earthquake destroyed much of San Francisco.
Since there is no word like spoonsful, it does not make any Instead of ‘a’ before earthquake, article ‘an’ should be used
sense in the sentence. because it is followed by a vowel letter.
78. (b) revolutions should be replaced with revolution 120. (a) His parents don’t approve of his business. In English, don’t
79. (b) It should be ‘that I was running a temperature’. is used when speaking in the first and second person plural
80. (a) It should be ‘has been irrigating’ in present perfect and singular and the third person plural (“I,” “you,” “we,” and
continuous tense.
“they”).Doesn’t, on the other hand, is used when speaking in
81. (a) Being present perfect continuous tense, replace ‘am’ with
the third person singular only (“he,” “she,” and “it”).
have been.
121. (d) No error.
82. (c) In ‘c’, the word ‘his’ is wrong. So, remove this word.
83. (d) Here, for a picnic should be used. 122. (b) The lovers walked beside each other in silence. (the
84. (e) preposition besides means in addition to, whereas beside means
85. (a) Here, The burst from should be used. The words blast and next to)
explosion are synonymous. 123. (c) Men are wanted for the army, the navy and the air force.
86. (b) Here, to reveal should be used. When joining two or more grammatically similar expressions,
87. (a) The event shows past time. Hence, Simple Past i.e. My and is used followed in the end and commas are used to
friend became ...... should be used.
88. (b)
93. (b) ww 89 (c)
94 (b)
90 (b)
95 (d)
91 (e)
96 (b)
92 (b)
97 (a)
distinguish the other expressions
124. (a) If pronouns of different persons are to be used together in a
sentence to provide good and normal sense then the serial order
98. (d)

w.E
99 (d)
100. (b) Delete ‘pair of ’ before binocular because the word
‘binocular’ itself suggests a pair.
101. (b) ‘a’ should be replaced with ‘the’. Here Kalidas is not
should be second, third and first so here it should “He and I are
very good friends.”
125. (c) When a sentence starts with ‘one’ then to in accordance with

asy
Shakespeare but he is compared to Shakespeare.
102. (b) Delete ‘all’ before ‘left’. Here the usage of ‘all’ is superfluous
the singularity of the subject it should be one’s parents.
126. (c) ‘God’ is a universal noun so it should be preceded by ‘the’.

En
as ‘the teacher as well as his students’ itself signifies everyone.
103. (a) Add ‘the’ before ‘more’. Here the sentence consists of two
127. (b) According to the rules of the ‘articles’ the word ‘uniform’
should be preceded by ‘a’ because here the vowel sound of ‘u’
is different
clauses- Principal and Subordinate, where the Principal clause
should be given more stress by adding ‘the’ before ‘more’.
104. (d), No error. gin
128. (a) Here the use of ‘they’ is not needed as ‘the newspaper’ itself
is the subject
105. (c), Shakespeare has written many plays as well as few poetry.
[Poetry is an Uncountable Noun or Mass Noun; which means a eer
129. (b) According to the rules of conjunctions the word ‘no sooner...’
should be followed by ‘than...’
noun that cannot be used freely with numbers or the indefinite
article, and which therefore takes no plural form.]
106. (b), Neither of the girls are willing to accept the proposal. ing
130. (c) The preposition ‘on ‘ is used to indicate a date or place
should be ‘by’.

107. (a), The correct sentence should be- ‘An interesting book ‘ A
tale of two cities’ was written by Alexander Dumas. Article ‘a’
friends...’
.ne
131. (a) Here' it means one among many so it should be ‘one of my

132. (c) Here' due to the sense of comparison it should be ‘the same...’
in the first part should be replaced by ‘an’ because it is followed
by a vowel letter.
108. (c), In India, there are many poor. [Poors is no word. Poor refer
to all the poor people.
109. (c) Speak in Malayalan.
t
133. (c) Here' the word ‘will’ is not needed because ‘shall’ is already
used in the initial part of the sentence that itself signifies the
tense of the sentence.
134. (a)
110. (b) flowed
135. (a)
111. (d)
112. (b) 136 (d)
113. (c) beside 137 (b)
114. (b) Here, indefinite article i.e. about a plane crash should be 138. (b), Mountaineer is the correctly spelt word.
used. No particular incident is evident here. 139. (a), Happened is the correctly spelt word.
140. (c), Sentimentalist is the correctly spelt word.
115. (b) With a View to should be followed by gerund i.e. suveying.
141. (b), Libertarian is the correctly spelt word.
116. (a) Here, time period is given. Hence. Past Perfect Continuous 142. (d), Empathic is the correctly spelt word.
i.e. It had been lying ....should be used. 143. (b)
117. (c) Here, to provide evidence/ as a witness .... should be used. 144. (c)
145. (a)
118. (b) Here, it is a preposition related error. Hence, at her should
146. (a)
be used here.
147. (c)

Downloaded From : www.EasyEngineering.net


Downloaded From : www.EasyEngineering.net

106  l  Spotting Error

148. (c) Dubious is the correctly spelt word. It means hesitating. 4. (a) Unlike his brother
149. (a) Ineffectual is the correctly spelt word. It means not The adverb unlike is to be used here as logical reasoning
producing any significant or desired result. suggests.
150. (d) Massacre is the correctly spelt word. It means the act or an 5. (d) than we had been
instance of killing a large number of humans indiscriminately The past perfect tense ‘had been’ is used here to denote an
and cruelly. action completed before a certain moment in the past.
151. (c) Conscience is the correctly spelt word. Conscience is 6. (a) The parts of Madras…
an aptitude, faculty, intuition or judgment that assists in A verb must agree with its subject in number and person. The
distinguishing right from wrong plural ‘parts’ fits in with the verb ‘were’.
Thus, if the subject is of the Singular Number, First Person, the
152. (a) Nirvana is the correctly spelt word. Nirvana literally means
verb must be of the Singular Number, First Person; as, I am
“blown out”, as in a candle. It is most commonly associated with
here. I was there. I have a bat. I play cricket.
Buddhism
7. (b) of extreme importance...
153. (b) the correct spelling is excellence
‘Importance’ is a noun; ‘important’ is an adjective.
154. (a) the correct spelling is grammar 8. (c) and land and are found
155. (c) the correct spelling is omitted The cumulative conjunction ‘and’ is needed here to add one

ww
156. (d) the correct spelling is calendar
157. (a) the correct spelling is objectionable 9.
10.
statement to another.
(a) were standing
(c) how the brain works. The simple present tense is needed as

w.E
158. (d) the correct spelling is apology
159. (d) Rhythm = a strong regular repeated pattern of sounds or
movements.
11.
12.
part (b) indicates.
(c) everyone does his best
(b) connected with

161. (d) Saccharine = sentimental asy


160. (b) Indigenous = native ; belonging to a particular place.
13.
The past tense is to be used as part (a) indicates.
(a) On his entering the meeting hall

162. (a) Revolutionize = to completely change the way that


something is done. En 14.
15.
(a) that if you were I
(c) other sources of information/different sources of
information.
163. (d) Dysentery = an infection of the bowels that causes severe
diarrhoea with loss of blood.
gin
16.
The tautology is to be removed.
(c) …….them were suitable

LEVEL- II
17.
eer
(b) ….their professional lives
The plural form of the noun is to be used as part (a) indicates.

1. (d) and I
When a noun (or pronoun) is used as the Subject of a verb, it
18.
19.
20. ing
(c) In a cold drink 'a' is redundant.
(a) If only I had studied....
(b) ....accused refused to disclose
is said to be in the Nominative Case and when it is used as the
Object of a verb, it is said to be in the Objective (or Accusative)
case. In the sentence given, the staff and the person speaking
21. (c) is being discussed....
.ne
One denies an accusation and refuses a request or plea.

form the subject of the verb ‘were’ and hence the Nominative
Case of the First Person-Singular i.e. ‘I’ should be used instead
of the Accusative Case i.e. ‘me’.
Note- To find the Nominative Case put Who? or What? before
22.
23.

24.
(a) If I had known....
(d) any man/any other animal.
t
To say any other man presupposes that the elephant too is a
man.
(c) will receive his/her results……
the verb. 25. (d) 'is really the same.'
To find the Accusative Case put whom? or What? Before the 26. (c) that he has an urgent
verb and its subject. For e.g.. Hari broke the window. (Object). 27. (b) ……that he could only follow it
The window was broken. (Subject) As a general rule only should be placed immediately before the
The Nominative generally comes before the verb and the word it is intended to modify.
Accusative after the verb. Hence they are distinguished by the 28. (b) It should be ‘pride in
order of words, or by the sense. 29. (b) It should be ‘as good as’ any other political party. Either it
2. (b) than any other boy in the class. should be in the positive or comparative degree.
When a comparison is instituted by means of a Comparative 30. (b) In this clause, use the correct word ‘that’ in place of
followed by ‘than’, the thing compared must be always excluded ‘because’
from the class of things with which it is compared, by using 31. (c) ‘as good as’ The two degrees, Positive and Comparative are
‘other’ or some such words. confusing, Say ‘as good as’ if not better than.
3. (a) a banana 32. (d) ‘at’ is redundant.
The reference here is to one banana and not a particular one. 33. (c) use the singular verb ‘is’ in place of ‘are

Downloaded From : www.EasyEngineering.net


Downloaded From : www.EasyEngineering.net

Spotting Error  l 107

34. (c) I shall take, because teachers give test and students take 56. (b) Here ‘could’ need to be replaced by ‘can’. Since ‘could’ in
them.. option (b) is grammatically incorrect.
35. (a) It should be ‘hung his head’ because hanged his head means 57. (b) Here ‘of ’ need to be replaced with ‘from’. Since a person can
put to death. be saved from death, not of death.
36. (d) Use ‘towards’ 58. (c) Here ‘their’ needs to be replaced with ‘its’.
37. (b) It should be ‘as can’ instead of ‘who will.’ 59. (d) There is no error in this statement as well. The sentence is
38. (c) It should be ‘he might’ because the Principal Clause is in grammatically correct.
the Past Indefinite Tense. 60. (b) It should be ‘answer’ instead of ‘answer to’.
39. (b) Use the Infinitive, ‘to distinguish’ instead of the participle 61. (a) It should be ‘two years have passed’ in present perfect tense.
‘distinguishing 62. 'Either' refers to each one of a number while 'any' tells us how
40. (c) In case of Present indefinite tense with singular numbers many or denotes order in a series. So, replace either with any.
we use ‘s’ or ‘es’ so it should he proclaims 63. (b) The connector ‘with’ should be remove.
41. (a) In this question the word ‘principle’ is wrongly spelt. The 64. (a) The past continuous tense is used for an action that was
letter under this part is (a), therefore, (a) is the correct answer. still going on at a particular moment in the past. The past
The correct form would be “Microwaves are the principal continuous is often used in combination with the simple past.
So, Replace ‘was arriving’ with arrived.

ww
carriers”, “which means microwaves are the chief or main
carriers”. The word principle is not used because it means – a
basic general truth that is the foundation of something.
65. (c) In ‘c’, the word ‘as’ is wrong and there is no need of it. So,
remove this word.

w.E
42. (b) The word ‘who’ is wrongly used. The letter under this part
is (b), therefore, (b) is the correct answer. The correct form
would be “Mahavira who was an advocate of nonviolence and
66. (b) In ‘b’, the word ‘the two-miles’ is wrong. Replace it with ‘a
two-mile’.
67. (c) In sentence ‘the sum and substance’ signifies one thing and

asy
vegetarianism, revived and reorganized ___”. The reason being,
the position of ‘who’ as a relative pronoun is wrong as it come
just after the antecedenthere Mahavira.
hence followed by a singular verb. So replace ‘were’ with ‘was’.
68. (b) The correct phrase is 'look forward to verb+ing' hence it
should be 'I look forward to meeting you ....'.
43. (c) Reaction should replace with 'reactions'
En
44. (a) In the question, the word ‘off ‘ is wrongly used. The letter
69. (b) The preposition 'to' here is redundant.
70. (a) Since ‘unless’ is already in use to forbid, we should not use
under this part is (a) therefore (a) is the correct answer. The
correct form of sentence will be “the stood apart from the gin‘does not revise’. So, Replace it with ‘revises’.
71. (a) The correct structure of the sentence should be 'whenever
crowd hair.”
45. (b) The word 'data' is used as plural. Hence, the verb should
agree 'are' in place of 'is'. eer
a person loses anything ....'.
72. (c) It should be ‘I say my prayer’. Remove ‘do’ as it has no need
here.
46. (d) No error.
47. (b) The word ‘discrect’ means careful and prodent in one’s ing
73. (c) It should be ‘friend had left’ in the past perfect tense.
74. (b) It should be ‘old enough’ because the adverb ‘enough’
speech or actions’ while discrete means ‘individually separate
or distinct.
comes after the adjective to describe it.

.ne
75. (b) It should be ‘I will do’ because modals ‘shall’ is used for
obligation.
48. (b) The word ‘invent’ is wrongly used. The letter under this
part is (b), therefore, (b) is the correct answers. The correct
sentence will be “I will need several weeks to detect the lie of
the man before I can make any decision ........ business.
76. (b) The proposition 'for' should be replaced with 'on'.
t
77. (b) Road is a mean to reach somewhere. So we reach through
the road. A Road remains at its place as it is. ‘By’ can be used for
49. (b) The word ‘about’ is wrongly used. The letter under this
a vehicle but not roads.
part is (b), therefore, (b) is the correct answer. The correct
sentence will be ‘The salesman gave us a big spiel as to why 78. (e)
......... product”. 79. (d) Here, levied additional monthly charges on consumers is
50. (b) The correct phrase is 'at the expense of doing something'.
used.
Hence 'to' should be replaced with 'at'.
51. (b) In the sentence 'to explain of ' the 'of ' is redundant. 80. (a) Here, Despite curfew is used.
52. (c) The error lies in part (c) of the sentence, so, (c) is the 81. (d) Here, Has been doubled is used.
correct answer. The correct form of sentence will be ‘the court 82. (a)
acquitted him of the crime’ the word ‘out’ is not to be used. The
83. (b)
word ‘Acquit’ means to free from a charge.
53. (b) The error lies in part (b) of the sentence. So, (b) is the correct 84. (c)
answer. The correct form of sentence will be ‘the company has 85. (c) Here, arise when they are not is used.
set itself some production. The word ‘off ‘ is to be removed. 86. (e) No error
54. (b) The correct sentence should be 'we had to bellow'.
55. (d) This is grammatically correct sentence. 87. (b)

Downloaded From : www.EasyEngineering.net


Downloaded From : www.EasyEngineering.net

108  l  Spotting Error

88. (d) it should be ‘either in India or in the country of his origin which is not the case. Hence, it should be as Mohan’s eyes
89. (c) and law abiding .... reflected .....
90. (d) .... has not been achieved instead of have 116. (c) With a view to (doing) something because you are planning
to do something in the future. Ex: We bought the house with a
91. (e) the sentence is correct ( no error)
view to retiring there.
92. (d) in India’s ..... in place of to India’s ...
117. (b) ‘has come’ is the correct option because the verb will agree
93. (d) Since the sentence begins in past tense. It should end in
with the first subject.
past tense also since it the subject is singular the verb will also
118. (b) adjacent always takes the preposition ‘to’.
be singular. Hence “them is missing” should be “them was 119. (b) Grapes cannot be the subject of the sentence; hence it should
missing”. be ‘Grapes cannot be gathered......’
94. (a) The teacher that is the subject is singular so “were” will be 120. (a) Part ‘a’ of the statement is wrong. It will be as ‘If I had
replaced with “was”. realized’. The Past perfect tense suits here and it denotes the
95. (b) “Son for help her” should be “son to help her”. sentence to be in Active voice.
96. (b) The verb “ask” will be in the past tense it will become 121. (b) Part ‘b’ of the statement is wrong. The word ‘enhanced’

ww
“asked”.
97. (c) “Saving” will be replaced with past tense of the verb “Save”
given here is wrong. The correct word that will suit the statement
is ‘topper’. Correct application of word enriches the meaning

w.E
that is saved because it is preceded by had.
98. (c) Replace ‘gone’ with ‘had gone’
of the sentence.
122. (b) Part ‘b’ of the sentence is wrong. The correct form of sen-
tence is ‘It is I am who should be responsible for the delay’.
99. (b) Replace ‘goes’ with ‘go’
100. (b) Replace ‘families’ with ‘family’ asy 123. (b) Part ‘b’ of the sentence is wrong. The preposition ‘of’ is
missing in that part. ‘Of’ is applicable after the word ‘evils’.
101. (a) Replace ‘were’ with ‘was’
102. (d) Replace ‘his’ with ‘her’ En 124. (b) Part ‘b’ of the sentence is wrong. It will be arranged as ‘over
extensively’. In the part ‘b’ ‘extensively over’ is denoting wrong
103. (b) ‘are’ is replaced by ‘is’
104. (b) Remove ‘to’ before the policeman gin application of words which is acting as a breach to the meaning
of the sentence.
105. (b) ‘me’ is replaced by ‘myself’
106. (c) ‘in’ is replaced by ‘at’ eer
125. (a) Among the four options ‘plebeian’ is correctly spelt word.
126. (b) Among the four options ‘surroundings’ is the correctly spelt
107. (d) No error
108. (b) Garrlilous
word.
ing
127. (a) Here subject (tractor sales) is plural. Hence, ..... tractor sales

109. (a) Marquee


have seen ....... should be used.
.ne
128. (b) It is preposition related error. Hence, .......... that share the
110. (a) Puissant
111. (d) Disconcerting
112. (d) Exhilarate
earth with us should be used.
129. (e) t
130. (b) Here, Noun i.e., environment and forests is ....... should be
113 (d) used.
114 (c) 131. (d) Here, Infinitive i.e., To portray is subject. Hence, singular
115. (b) Here Mohan’s eyes reflect means that it was Mohan’s habit verb i.e. references is very difficult should be used.

Downloaded From : www.EasyEngineering.net


Downloaded From : www.EasyEngineering.net

9
PTER
CHA

Sentence Correction

Aw
s the name suggests, these questions ask you to correct a sentence. That is, of course, because it is wrong in the first place (but
not necessarily so). They are asked on a number of tests because they do not only check your ability to spot an error, but also
to correct it. Hence, your grammar abilities are put to some real test in these questions.

ww
Let us first see how these questions are framed. Sentence(s) will be given to you, and a part of a sentence or the entire sentence will
be underlined. Now, from the options given, you have to select the one which you think should replace the underlined part. Normally,
the underlined part of the sentence is the first option. This is because the given statement might be correct and have no error(s).

.E
Thus, there’s the possibility that there might be no error in the statement, so you first have to analyse if there is any error at all. These
questions can end up eating up your time, so make sure that you do not give yourself more than a minute or two to solve a sentence

asy
correction question. Anything beyond that is considered too much for such questions.
Even if grammar is not your strong point, you can still manage to crack these questions by looking at the types of questions asked,

En
and what kind of errors to search for in a question. We have covered some of the errors in the previous sections.

HOW TO SOLVE

gin
(i) Understand the given sentence, including the part that is not underlined. This way, you would be able to grasp the context and
verb tense of the event or information talked about.

as the correct option.


eer
(ii) Reread the underlined part; if you are sure that there is no error, then just select the option which mentions the underlined part

(iii) If you get to know the error (s) in the underlined part, then correct it without looking at options; if you spotted the errors and
corrected them appropriately, your answer would be there in the options.
ing
(iv) If you have not been able to identify the error, but still think that the underlined part is not right, then start checking on the
‘grammar mistakes’ we mentioned in the previous sections.

.ne
(v) After selecting the option you think is correct, it is always advisable to reread the statement(s) given in the question with the
corrected part to see if you have followed the right tense and everything is in line with the context of the sentences before or after

TIPS
the underlined portion.
t
(i) Mostly, the sentence would look logically incorrect if modifiers are incorrectly used, or if the verb tenses are not appropriate or
consistent. You can read more about verb tenses and modifiers in the previous section.
(ii) A redundant sentence might look wrong, but it could still be the answer if the other options are logically or grammatically
incorrect, or if they change the intended meaning.
(iii) Idioms can be tricky, as you may think that the statement sounds awkward, but then it would be a correct idiom. So, make sure
you are in the know when it comes to idioms.
(iv) Words like ‘during’, ‘before’ will help you know the time of the event, and hence the appropriate verb tense.
(v) If you have been unable to spot an error in the underlined part, then you can check how the given options vary to see which
grammatical concept(s) is/are being tested, and then see if the underlined portion is the best one.
(vi) VPIMPS is the acronym for the most commonly tested errors in grammar. So if you think a sentence is not right, but not quite
sure why, you can just check the VPIMPS:
(a) Verbs (b) Parallelism (c) Idioms
(d) Modification (e) Pronouns (f) Sentence structure

SKILLS REQUIRED
Refer Grammar Section.

Downloaded From : www.EasyEngineering.net


Downloaded From : www.EasyEngineering.net

110  l  Sentence Correction

COMMON MISTAKES
(1) Overlooking the rest of the sentence : In order to save time, some students just look at the underlined part, avoid the correct part
of the sentence altogether and try to select the correct option. There is a reason why the underlined section exists: telling you the
context and the tense. Make sure you do not miss out on that.
(2) Not paying attention to the directions: The directions for these type of questions clearly state that you have to choose the option
that would best replace the underlined part. Nowhere is it mentioned that you have to only pay heed to the grammar of the
sentence. It should be a complete sentence that makes sense, for e.g., if you have two options that both seem grammatically
correct, then check for usage errors and find out which part (when in line with the completed sentence) makes the most sense.

FROM BEGINNER TO MASTERY


Directions :  In each of the following examples a part of sentence has been underlines. From the choices given, you are required to choose
the one which would best replaced the underlined part.
Example 1.
I fell up the stairs.
(a) fell up (b) fell down (c) fell at (d) fall off
Explanation  (b) When one trips and falls from the stairs, it is assumed that they will fall down and not ‘up’, or ‘at’ or ‘off ’.
Example 2.
ww
I have been there often, because it had to be a nice place.
(a) had to be a nice place. (b) must be nice going there. (c) is a nice place. (d) I felt so.
Explanation 
w.E (c) The speaker says that he/she has been there often. This means that probably he/she thinks that it is a decent
place. Since, this is experienced in the present tense, therefore the reason should be in the present tense too. Thus, option (c) be-
comes the most appropriate answer.
Example 3.
asy
My chances of winning the lottery is similar to you.
(a) winning the lottery is similar to you
(c) having won the lottery is to yours.
Explanation  En (b) winning the lottery is similar to yours
(d) winning the lottery are similar to yours.
(d) When we are comparing the fate of two people, we have to compare the fates and not the two peoples. If we

gin
say, ‘My chances of winning the lottery is similar to you,’ it means that we are comparing the person and the fate, whereas when we
say ‘My chances of winning the lottery are similar to you,’ we are comparing the two fates.
Example 4.
Yesterday, I will have nothing to do.
(a) I will have nothing to do. eer
(b) I had nothing to do.
(c) was full of boredom.
Explanation  ing
(d) I would have nothing to do.
(b) Here, because of the use of the ‘yesterday’, the sentence is considered to be acting out in the past tense.
Thus, option (b) is correct.
Example 5.
I was a bit surprised to see a call from an unknown number, so first of all I asked “What are you?” .ne
(a) Which are you (b) Who are you (c) Why did you call
Explanation  (b) This one is easy. When we talk of people/person, we use the pronoun: ‘Who’.
Example 6.
(d) How are you
t
Victory is everything in the Indian universe and Tendulkar will be expected to translate his genius to that effect. To contemplate
any other option is to contemplate the risk of failure.
(a) To contemplate any other option is to contemplate the risk of failure.
(b) Failure is not an option that can be contemplated
(c) Any other action has the potential of failure
(d) Failure is not an option.
Explanation  (c); The given statement states that a victory is paramount in India and Tendulkar is expected to perform to his
potential to make sure that India wins. Thinking about any other possibility could lead to a failure. Now, the given statement sounds
redundant, so we have to remove this error and make it simpler and option ‘C’ would do just that.
Example 7.
If you are on a three-month software design project and, in two weeks, you’re put together a programme that solves part of the
problem, show it to your boss without delay.
(a) and, you’ve put together a programme that solves part of the problem in two weeks
(b) and, in two weeks you’ve put together a program that solves part of the problem
(c) and, you’ve put together a programme that has solved part of the problem in two weeks
(d) and, in two weeks you put together a programme that solved only part of the problem

Downloaded From : www.EasyEngineering.net


Downloaded From : www.EasyEngineering.net

Sentence Correction  l 111

Explanation  (b); This one is easy. The position of the phrase: ‘two weeks’ should be in the starting of the clause and not at
the end. This leaves us with two options: (b) and (d). option (d) changes the context of the sentence and makes it sound incorrect.
Example 8.
Many of these environmentalists proclaim to save nothing less than the planet itself.
(a) to save nothing lesser than (b) that they are saving nothing lesser than
(c) to save nothing less than (d) that they save nothing less than
Explanation  (c); ‘Lesser’ is used while we are comparing two things, but there is no comparison done in the sentence. The
word ‘less’ is just used to show the degree or extent of what they are doing or intend to. So, the appropriate word is ‘less.
Example 9.
Bacon believes that the medical profession should be permitted to ease and quicken death where the end would otherwise only
delay for a few days and at the cost of great pain.
(a) be delayed for a few days (b) be delayed for a few days and
(c) be otherwise only delayed for a few days and (d) otherwise only delay for a few days and
Explanation  (c); The given statement points out the opinion of Bacon: medical profession should be allowed to ease and
quicken the death in cases where, not doing so would only delay the death for a few days, and those few days would only bring
more pain and trouble to that patient. This question also tests our usage of the verb ‘be’.
Example 10.

ww
Men's interest in developing a cure for cancer have promoted the rapid advances in the abstruse field now known as Genetic
Engineering.
(a) Men's interest in developing a cure for cancer have promoted

w.E
(b) That men are interested in developing a cure for cancer have promoted
(c) Interest in developing a cure for cancer has promoted
(d) Men's interest in developing cure for cancer has promoted
Explanation 
asy
(c); The given sentence is wrong because verb ‘have’ is not in accordance with the noun ‘interest’, because ‘have’
is plural whereas, ‘interest’ is singular. Therefore, the correct combination would be ‘interest…has..’ Option (c) is right, because it
has the correct noun-verb pair.
Example 11.
En
A little known danger of potent hallucinogens such as lysergic acid deithylamide-25 is that not only is the user immediately

gin
disoriented, but also he will experience significant ego suppression for a period of three weeks as well.
(a) but also he will experience significant ego suppression for a period of three weeks as well
(b) also there will be a three-week period of ego suppression as well
(c) the ego is suppressed for a period of three weeks as well
(d) but the user's ego is suppressed for a period of three weeks in addition eer
Explanation 
ing
(d); The underlined sentence will begin with a coordinating conjunction that links the two clauses and only
one of the given options has a coordinating conjunction i.e. option (d). The given sentence is wrong because it is redundant with
the usage of ‘also’ and ‘as well’, as they both mean the same. Option (a) and option (b) have the same error. Option (c) is not chosen

.ne
because there is no coordinating conjunction for the second clause. Option (d), i.e. the answer, also follows the correct ‘..not only
x…but Y..’ sentence contract as both X and Y are parallel (a grammatically correct sentence would have similar X and Y, i.e. they
both will be verbs , adjectives or nouns)
Example 12.
By the time peace and happiness will have come to the planet, many lives will be wasted.
(a) come to the planet, many lives will have been wasted
t
(b) will have come to the planet, many lives will have been wasted
(c) shall have come to the planet, many lives shall be wasted
(d) would have come to the planet, many lives would have been wasted
Explanation  (a); Both the events discussed in the sentence will take place in the future in such a way that one will take
place before another. Hence, we will use the future perfect tense. The event which will happen earlier, or before the specified event
employs ‘will have’ and only option (a) has used this tense correctly.
Example 13.

The leader of the Neanderthal tribe rarely hunted for food, and because of it was never acknowledged as a great hunter.
(a) Because the leader of the Neanderthal tribe rarely hunted for food, he
(b) In that he rarely hunted for food, the leader of the Neanderthal tribe was
(c) Rarely hunting for food was the reason that the leader of the Neanderthal tribe
(d) Hunts were rare, and because of this the leader of the Neanderthal tribe
Explanation  (a); The main clause of the sentence is the one which is not underlined, and the one which is underlined is the
clause which will reason it, hence our underlined part of the sentence will be the adverbial clause. Option (a) gives us the appropriate
adverbial clause which begins with a subordinate conjunction. Option (b) is wrong because it is ambiguous. Option (c) is redundant
and concise when compared with option (a) and option (d) is ambiguous because it is not easily understandable.

Downloaded From : www.EasyEngineering.net


Downloaded From : www.EasyEngineering.net

112  l  Sentence Correction

Example 14.

Regardless “new modernism” in literature, which produces novels, which often read like the diaries of madmen, most readers still
prefer a conventional plot and simple style.
(a) Regardless of the “new modernism” in literature, which produces
(b) Rega rdless of the “new modernism” literature, which produce
(c) Regardless, the “new modernism” in literature produces
(d) Irregardless of the “new modernism” in literature, which produces
Explanation  (a); ‘Regardless’ is an adverb which usually means ‘without regard or consideration for’. Option (a) is correct, because
it uses the adverb correctly to form an appropriate adverbial clause. Option (b) is wrong because ‘new modernism’ refers to the changes
brought on in literature, so the preposition ‘in’ should be used there to clear ambiguity and it also has the incorrect verb ‘produce’, it
should be ‘produces’ instead. Option (c) is incorrect, because it uses regardless as an adjective which does not make logical sense.
Example 15.
A career in the medical profession, which requires an enormous investment of time and money, do not guarantee success as there
is so much competition.
(a) which requires an enormous investment of time and money, do not guarantee success as there is so much competition
(b) which requires an enormous investment of time and money, does not guarantee success since there is so much competition
(c) requiring an enormous investment of time and money, and cannot guarantee success because there is so much competition

ww
(d) requires that an enormous investment of time and money be made and success cannot be guaranteed due to competition
Explanation  (b); The underlined part ‘which requires an enormous investment of time and money’ forms the adjective clause
as it describes ‘medical profession’ and it does so correctly. The remaining underlined part gives us the main clause of the sentence.

w.E
The error in the given sentence is in subject verb disagreement. ‘Medical profession’ is a singular noun, hence it should have a singular
noun ‘does’ and not ‘do’. Hence, option (b) is correct. Options (c) and (d) are incorrect, because they do not properly use the required
adjective clause.
Example 16.
asy
The more the union stubbornly refused to budge from its original demand for 20% across-the-board salary increase, the more
the company management reiterated its original proposal of a mere 3% raise.

En
(a) the more the company management reiterated its original proposal of a mere 3% raise
(b) the company management’s original proposal for a mere 3% raise was reiterated all the more
(c) the company management kept on reiterating its original proposal of a mere 3% raise

gin
(d) the more the company management’s original proposal of a mere 3% raise was reiterated
Explanation  (a); The given sentence is right. The only other sentence which follows ‘the more the..the more the…’ sentence

reiterated by some other party instead of the management itself.


Example 17. eer
construction is (d), but it is awkwardly worded so that it is giving the impression that the 3% raise proposal of the management was

ing
The president of the resident welfare association tried to convince his neighbours they should join forces prevent crime in the
neighbourhood rather than continuing to be victimized.
(a) they should join forces to prevent crime in the neighbourhood rather than continuing to be victimized

.ne
(b) that they should join forces to prevent crime in the neighbourhood rather than continue to be victimized
(c) about joining forces to prevent crime in the neighbourhood instead of continuing to be victimized
(d) to join forces to prevent crime in the neighbourhood rather than continuing to be victimized
t
Explanation  (b); This question is about sentence construction and parallelism, and it is clearly evident that the underlined part
forms the sub-ordinate sentence and it should be introduced with a conjunction to differentiate, or part the sentence, and only option
(b) does that with the conjunction ‘that’ which introduces the subordinate clause. Also, notice that there are two verbs in the sentence
and both should be used in the same infinitive form.
Example 18.
Lawyers and doctors alike both agree something should be done about the rise in medical malpractice cases which are on the increase.
(a) alike both agree that something should be done about the rise in medical malpractice cases
(b) alike agree that something should be done about the rise in medical malpractice cases
(c) agree that something should be done about the rise in the number of medical malpractices
(d) None of the above
Explanation  (b); Alike means ‘equally or in a similar manner’. The given sentence has three errors: the word ‘both’ is redundant
and not required as the coordinating conjunction ‘and’ has already formed the compound noun for the subject. Another error is that
there is a bit of ambiguity because the sub-ordinate clause has not been introduced as there is no subordinating conjunction: there
must be a subordinating conjunction to make it clear what both doctors and lawyers agree on. Third error of the given sentence is the
adjective clause, ‘which are on the increase’ is redundant because the phrase ‘are on the rise’ has already conveyed that the number of
medical malpractices is increasing. Option (b) has removed both these errors. Option (a) is wrong because it is similar to the given
sentence. Option (c) is wrong, because it is taking an information which the original sentence tried to convey i.e. they both make the
same error.

Downloaded From : www.EasyEngineering.net


Downloaded From : www.EasyEngineering.net

Sentence Correction  l 113

Example 19.
The conductor seemed entirely arbitrary the choice of tempo, because of which each successive movement of the piece seemed
to have no connection to what had come before.
(a) The conductor’s choice of tempo seemed entirely arbitrary
(b) It seemed the conductor chose tempo entirely arbitrary
(c) The conductor was entirely arbitrary in his choice of tempo
(d) The tempo was chosen entirely by the arbitrary conductor
Explanation  (a); Arbitrary is used to describe something which is based on a random choice or personal whim, rather than
any reason or system. A person would not normally be described as arbitrary. Thus, we can easily rule out options (c) and (d). They
state that the conductor was arbitrary, which is illogical. The correct wording would be that the conductor’s decision or choice was
arbitrary and not the conductor himself. Hence, option (a) is correct. Option (b) is incorrect because arbitrarily should be used in-
stead of arbitrary because arbitrarily is an adverb which would describe how he made the choice, thereby describing a mental action
(verb) whereas, arbitrary is an adjective.
Example 20.
Although the conditions in which she lived suggest that she was miserly, her contributions to charities show that she is generous.
(a) her charities showed generous contributions
(b) her generosities made large contributions

ww
(c) her contributions to charities showed that she is not generous
(d) her contributions to charities show that she was generous
Explanation  (d); The given sentence is wrong because of the verb ‘is’. This verb is in the present tense, whereas the given

w.E
sentence is in the past tense as indicated by the verb ‘was’ used in the part of the sentence which is not underlined. Option (d)
corrects this error and is our answer. Option (a) and (b) are wrong, because of the sentence construction with subordinating
conjunction ‘although’ which means ‘even though’ so the main clause will be contradictory with the adverbial clause introduced by

neglects the ‘although’.


Example 21.
asy
‘although’. So, the proper sentence construction would be parallel and this is how option (d) is. Option (c) is incorrect, because it


En
Although he was often incomplete in his work, he was promoted simply because he was with the company longer than anyone
else.
(a) Although work was often incomplete
(c) Although his work was often incomplete
Explanation 
gin
(b) His work was often incomplete although
(d) Although he often completed his work
(c); The given sentence is wrong, because it sounds ambiguous by considering an individual incomplete. Op-

eer
tion (c) corrects this error by stating that his work was often incomplete. Option (a) is wrong, because it is ambiguous and does not
specify whose work or which work. Option (b) is wrong, because the correct placement of the subordinating conjunction ‘although’

the use of although makes no sense.


Example 22. ing
would be in such a way that it introduces the adverbial clause – ‘His work was often incomplete’. Option (d) is wrong, because then

Though the concert had been enjoyable, it was protracted overly.


(a) it was overly protracted. (b) it overly protracted. (c) it protracted overly. .ne
(d) it got protracted overly.
Explanation 
t
(a); Protracted is used to describe something that is relatively long in duration. It is a negative term that is used
to describe something which lasted more than you wanted it to. As a verb, it means ‘lengthen in time’. The given sentence places the
verb before the adverb, which is not always preferred and option (a) gives a more appropriate placement of the adverb. The remain-
ing options are not consistent with the tense. There should be a verb ‘was’ to make the entire sentence consistent with the past tense.

Downloaded From : www.EasyEngineering.net


Downloaded From : www.EasyEngineering.net

114  l  Sentence Correction

Practice Exercise
LEVEL-I
DIRECTIONS (Qs. 1-80): In the following questions, in the 14. Let’s buy a new sari with the annual bonus, can we?
given sentences, a part of the sentence is underlined. Beneath each (a) can’t we (b) don’t we
sentence, four different ways of phrasing the underlined part are (c) shall we (d) No improvement
indicated. Choose the best alternative.
15. Since 1986, there is no earthquakes here.
1. All the allegations levelled against him were found to be
baseless. (a) have been (b) are
(a) levelled for (b) level with (c) were being (d) No improvement
(c) level against (d) No correction required 16. It is time the six year old is learning how to read and write.
2. He has said so out of affection, do not take it to heart. (a) has learned (b) learnt
(a) in heart (b) it in the heart (c) was learning (d) No improvement
(c) by the heart (d) No improvement

ww
3. I would gladly accompany your sister if you had asked him.
(a) would have gladly accompanied
(b) was to have gladly accompanied
17. He plays cricket and tennis also.
(a) both
(c) too
(b) besides
(d) No improvement

w.E
(c) will gladly accompany
(d) No improvement
4. What are needed are not large houses but small cottages.
18. I am used to hard work.
(a) hard working
(c) work hardly
(b) work hard
(d) No improvement
(a) were
(c) is
(b) was
asy
(d) No improvement
5. Sumit would have been looked smart in traditional clothes.
19. Practically every part of the banana tree is used by man.
(a) most part (b) any part
(a) was looked
(c) had looking
(b) would be looked
(d) would have looked En (c) each part (d) No improvement
20. No sooner I saw the tiger than I ran away.
6. Hardly had I finished writing the letter before Anil arrived.
(a) then (b) while gin (a) No sooner I had seen
(c) As soon as I saw
(b) No sooner did I see
(d) No improvement
(c) when (d) as
7. His father won’t be able to leave for Varanasi until they have
arrived.

eer
21. Try to be diplomatic when you refuse any invitation,
so for not to cause bad feeling

(a) until they arrive


(b) until they will arrive ing
(a) as far not
(c) if as far not
(b) so as not
(d) since then not
22. The need of the hour was some fast action on the part of the
(c) until they will have arrived
(d) No improvement
8. He returned with two burning mugs of coffee and set them
leaders.
(a) needed .ne
(b) needing
down on her desk.
(a) heating
(c) blazing
(b) steaming
(d) No improvement
9. People are tiring very soon in an activity which they don’t like.
(c) needs
23. The doctor examined my pulse.
(a) observed (b) saw
t
(d) No correction required

(a) tried (b) trying (c) felt (d) No improvement


(c) tired (d) tyring 24. He doesn’t hesitate to do whatever his brother does.
10. They felt humiliated because they realised that they had (a) will do (b) would do
cheated. (c) shall do (d) would done
(a) had been cheating (b) had been cheated 25. Bad habits must be nipped at the bud.
(c) have been cheated (d) were to be cheated (a) nipped off the bud (b) nipped in the bud
11. The summer has set out and the days are getting warm.
(c) nipped on the bud (d) No improvement
(a) set up (b) set in
(c) set off (d) No improvement 26. It is no good to cry over spilt milk.
12. She could not help but laugh. (a) It is no good crying (b) It is of no good to cry
(a) laughing (b) but laughing (c) It is of no good crying (d) No improvement
(c) laugh (d) No improvement 27. To various practices and norms for bank’s transactions
13. Im sorry, but I don’t believe what you say. are laid down by the Reserve Bank of India.
(a) I will not believe (b) I am not believing (a) are laid up (b) are led down
(c) I will not be believing (d) No improvement (c) are lead up (d) No correction required

Downloaded From : www.EasyEngineering.net


Downloaded From : www.EasyEngineering.net

Sentence Correction  l 115

28. No one needs to worry about me. 44. Seeing the injustice done to them hy the Britishers, the
(a) will worry (b) need worry sepoys broke into open revolt.
(c) shall worry (d) No improvement (a) broke to (b) broke upto
29. When the soldiers realised that they had been fooled, they (c) resorted to (d) No improvement
rush back. 45. The cloud of misfortunes appears to have blown out.
(a) had rush (b) have rush (a) away (b) over
(c) had rushed (d) rushed (c) up (d) No improvement
30. The officer looked on my complaint. 46. Anand has the guts to rise from the occasion and come out
(a) looked of (b) looked into successful.
(c) looked to (d) No improvement (a) to rise against (b) to rising from
31. The actress said that it sometimes took her two hours to put (c) to rise to (d) to rise with
her make -up. 47. A bird in hand is worth two in bush.
(a) put over (b) put up (a) two in the bush (b) two at a bush
(c) put on (d) No improvement (c) two on bush (d) No improvement
32. The matter must be considered in every point of view. 48. Scarcely had he entered the room than the phone rang.
(a) with (b) from (a) when (b) while

ww
(c) at (d) No improvement
33. He enjoys to tell stories to children.
(a) how to tell stories (b) telling stories
(c) as (d) No improvement
49. Five years ago today, I am sitting in a small japanese car,
driving across Poland towards Berlin.

w.E
(c) to narrate stories (d) No improvement
34. Manisha purchased the very good of all the saris kept in the
shop
(a) was sitting
(c) sat
(b) have been sitting
(d) No improvement
50. If you had told your problem yesterday, we might had helped
(a) the better
(c) the most good asy
(b) the very better
(d) the best
you
(a) would have (b) might have been
35. On seeing the lion she felt too much afraid.
(a) very much (b) excessively
En (c) would have been (d) No correction required
51. When he heard the rhetorical speech of the leader, he was
carried along by his enthusiasm.
(c) much (d) No improvement
36. Last evening I sent to the optician and bought spectacles.
(a) a spectacle (b) two spectacles gin (a) was carried away
(c) was carried aloft
(b) was carried down
(d) No improvement
(c) a pair of spectacles (d) No improvement
37. Education is a strong instrument for moulding the character (a) with
(c) on
eer
52. The members of his family are coming in this train.
(b) by
(d) No improvement
of the young.
(a) striking
(c) potent
(b) powerful
(d) No improvement
ing
53. If he had time he will call you.
(a) would have (b) would have had
38. One of the men gave first aid to Hitesh who is injured in a
road accident.
(c) has
.ne
(d) No improvement
54. The intruder stood quietly for few moments.
(a) who had injured
(c) which was injured
(b) who was injured
(d) as he was to injure
39. Mrs. Johnson had staying here since 1954 and has made
India her home
(a) for few time
(c) for the few moments
t
(b) for a few moments
(d) No correction required
55. A lot depends on your early brought up in the family.
(a) bringing up (b) bringing on
(c) upbringing (d) No improvement
(a) will stay (b) was to stay
56. Many believed that girls who received western education
(c) is staying (d) has been staying
would make slaves of their husbands.
40. The problems of translation are still remain.
(a) could receive (b) had received
(a) are remain (b) will remained
(c) have received (d) No improvement
(c) will still remain (d) No improvement
57. I have not written many letter to him since my father had
41. They have stopped from constructing new buildings.
died.
(a) to contruct (b) at constructing (a) died (b) was dead
(c) constructing (d) No improvement (c) has died (d) No improvement
42. All over Russia, Indian films are more popular than those in
58. The manager was unhappy at Gaurav because he did not
any other country.
complete the work in time
(a) in (b) that in
(a) is unhapy at
(c) that of (d) those of
(b) is to be unhappy at
43. Wise men catch time by the forelock.
(c) was unhappy with
(a) hold (b) seize
(d) No correction required
(c) take (d) No improvement

Downloaded From : www.EasyEngineering.net


Downloaded From : www.EasyEngineering.net

116  l  Sentence Correction

59. In the modern world it is difficult to live through one’s 71. As a child, my parents took me to Jammu to visit my
ideals. grandmother.
(a) to live upto (b) to live by (a) When I was a child, my parents took me to Jammu to
(c) to live for (d) No improvement visit my grandmother
60. It is long since I last saw you. (b) My parents took me, as a child, to Jammu to visit my
(a) long time (b) long before grandmother
(c) long ago (d) No improvement (c) My parents took me to Jammu to visit my grandmother
61. She gave most of her time to music as a child.
(a) devoted (b) spend (d) A child, my parents took me to Jammu to visit my
(c) lent (d) No improvement grandmother
62. When I was fourteen, I sat the entrance examination for 72. Anyone interested in the use of computer can learn much if
senior secondary school. you have access to a state-of-the-art microcomputer.
(a) sat for (b) sat in (a) if he has access to
(c) sat at (d) No improvement (b) if access is available to
63. He has been working off and on for several years to compile (c) by access to
a dictionary. (d) from access to

ww
(a) on or off
(c) regularly
(b) on and off
(d) No correction required
73. Start the motor, and then you should remove the blocks.
(a) Start the motor, then removing the blocks
(b) Starting the motor, the blocks should then be removed

w.E
64. Rajni unnecessarily picked up a quarrel with Kashish and
left the party hurriedly.
(a) picking up (b) picked on
(c) Start the motor and then remove the blocks
(d) Starting the motor remove the blocks
74. Eaten in Portugal only, the Indians viewed the potato with
(c) picked
65. She did not like the movie, nor I did.
(a) nor did I asy
(d) has picked up

(b) nor I like it


suspicion for they assumed it had poisonous properties
since only the white skinned people consumed it.
(c) nor I liked it (d) No improvement
66. It will take two hours to walk across the forest.
En (a) Indians viewed the potato with suspicion for they
(b) Indians were suspicious of the potato and they
(a) over
(c) away
(b) between
(d) through
67. The unemployment rate has dropped sharply this month,
gin (c) potato was viewed with suspicion by Indians who
(d) potato was suspicious to Indians, and it was
75. Though he was more faster than his opponent on the field,
though it may only be temporary.
(a) but it may be only temporary eer
his chances of winning the race was low as he lacked the
killer instinct.
(b) but the drop may only be temporary
(c) even though the drop may only be temporary field
ing
(a) Though he was more faster than his opponent on the

(b) As he was more faster than his opponent on the field


(d) but such a drop may only be a temporary one
68. In accordance to your instructions, we have remitted the
amount in the bank. .ne
(c) Though he was more fast from his opponent on the field
(d) Though he was faster than his opponent on the field
(a) by
(c) for
(b) with
(d) No improvement
69. Although he was the most friendly of all present and different
from the others, he hadn’t hardly any friends except me.
opposition from they who maintain that it was an
unauthorised act.
t
76. Ever since the sting operation, there has been much

(a) Ever since the sting operation, there has been much
opposition from they who maintain that it had been an
(a) different from the others, he hardly had any friends
unathorised act.
except I
(b) ever since the sting operation, there has been much
(b) different than the others, he hardly had any friends
opposition from they who maintain that it was an
except me
unauthorised act.
(c) different than the others, he hardly had any friends
(c) Ever since the sting operation, there has been much
except I
opposition from they who maintain that it was an
(d) different from the others, he hardly had any friends
unauthorised act.
except me
(d) Ever since the sting operation, there has been much
70. Since we are living in Bombay for five years, we are reluctant opposition from those maintaining that it was an
to move to another city. unauthorised act.
(a) Being that we living 77. Anita liked to watch television, of which she found the
(b) Since we were living science programme especially fascinating.
(c) Since we have been living (a) television, of which she found the science programme
(d) Being that we have been living especially fascinating.

Downloaded From : www.EasyEngineering.net


Downloaded From : www.EasyEngineering.net

Sentence Correction  l 117

(b) television; she found the science programme especially 85. He had to bear the brunt of his father’s mistakes.
fascinating. (a) bear the burnt (b) bear the burns
(c) television, and it was especially the science programme
(c) bear a brunt (d) bear and burn
that were of fascination.
(d) television; the fascination of the science programme (e) No correction required
especially. DIRECTIONS (Qs. 86-90) :  In the following questions, four
78. Although gale force winds often pass through the Eiffel words printed in bold type are given. These are lettered (a), (b),
Tower, causing it to sway no more than four inches. (c) and (d). One of these words printed in bold might either be
(a) causing it to sway no more wrongly spelt or inappropriate in the context of the sentence. Find
(b) and yet it sways no more out the word that is inappropriate or wrongly spelt, if any. The
(c) they do not cause it to sway more
number of the word is your answer. If the words printed in bold
(d) and they do not cause it to sway
79. Underestimating its value, breakfast is a meal many people are correctly spelt and appropriate in the context of the sentence
then mark (e), i.e., ‘All Correct’ as your answer.
skip.
[SBI Clerk 2012]
(a) Underestimating its value, breakfast is a meal many
people skip. 86. Ronald was very his new car and would

ww
(b) Breakfast is skipped by many people because of their
underestimating its value. be on the lookout for an to it. All

w.E
(c) Many people, underestimate the value of breakfast and
skip it.
(d) Many people skip breakfast because they underestimate
87. Ramesh had an to alcohol and avoid going
its value.

asy
80. Certain shipwrecks have a particular fascination for those
people which have a belief in finding the treasure in them.
to with his All

En
(a) which have a belief in finding the treasure in them.
(b) who believe they hold treasure and that they can find it.
88. Lucy was overtime she had to

(c) who believe that there is treasure to be found in them.


(d) who believe about treasure to be found in them. gin for an inportant All

DIRECTIONS (Qs. 81-85) : Which of the phrases (a), (b),


(c) and (d) given below should replace the phrase given in bold
89. Roshni was
eer of pets and she was a owner of

in the following sentence to make the sentence grammatically


meaningful and correct ? If the sentence is correct as it is and
‘No correction is required’, mark (e) as the answer.
may
ing All

[SBI Clerk 2012] 90. Ahmad was not only very


.ne
but also very
81. They are yet to decided about buying the new furniture.
(a) still decide
(c) yet to decide
(e) No correction required
(b) yet to decision
(d) still decided
and would always first in his

DIRECTIONS (91-95): Which phrase should replace the phrase


All
t
82. Rohan’s mother was feed up of his laziness. given in bold in the sentence to make it grammatically correct?
(a) fed up in (b) fed off If the sentence is correct as it is given, then mark ‘No correction
(c) feeds up of (d) fed up of required’ as your answer. [IBPS Clerk 2013]
(e) No correction required 91. We asked her that how she got time to write all these books.
83. She was just looking outside the window when a beautiful (a) that how did she got
bird caught the eye. (b) that how she was getting
(c) how did she get
(a) catch the eye (b) eye catching (d) how she got
(c) caught her eye (d) catch her eyes (e) No correction required
(e) No correction required 92. Studies in the past have shown that those who limit their
84. They sent out the invitations last evening. activity span during the day in winters are more likely to
(a) send out (b) sending out suffer from depression.
(a) more likely for (b) mostly likely to
(c) sent at (d) sending in
(c) most likely for (d) most likeliest for
(e) No correction required (e) No correction required

Downloaded From : www.EasyEngineering.net


Downloaded From : www.EasyEngineering.net

118  l  Sentence Correction

93. In some cases, factors like low salary, lack of growth 102. The tragedy is reflection of an episode that took place a
prospects and lack of motivation compel all employee to decade ago.
look for a change. (a) rendition
(a) compel those employees (b) reincarnation
(b) compelling all employees
(c) reminiscent
(c) compelling the employee
(d) No improvement
(d) compel employees
(e) No correction required DIRECTIONS (Qs. 103-104) : In the following questions, a part
94. Living with compassion and contributing to others lives of the sentence is underlined. Below are given alternatives to the
would helping us add happiness to our lives as well. underlined part at (a), (b), (c) which may improve the sentence.
(a) will helping us (b) will help us Choose the correct alternative. In case no improvement is needed,
(c) would helped them (d) will helped us your answer is (d).Mark your answer in the Answer Sheet.
(e) No correction required [SSC CHSL 2013]
95. The easiest way for prevent stress caused by work or home
103. The commoners joined the king’s army at crushing the
pressures is to indulge in high levels of physical activity.
rebels.
(a) easily way to (b) easier ways for

ww
(c) easiest way to
(e) No correction required
(d) easier way from
(a) into
(c) without
(b) in
(d) No improvement
104. She is quite well now, except a slight cold.

w.E
DIRECTIONS (Qs. 96 - 102) : In the following questions, part of
the sentence is bolded. Below are given alternatives to the bolded
part at (a), (b) and (c) which may improve the sentence. Choose
(a) except have a slight cold
(b) excepting a slight cold
(c) except for a slight cold

answer is (d).
asy
the correct alternative. In case no improvement is needed your

[SSC CHSL, 2012]


(d) No improvement

DIRECTIONS (Qs. 105-109) : In the following questions, a part

96.
En
The trek is difficult but it is worth well the endeavour.
of the sentence is underlined. Below are given alternatives to the
underlined part at (a), (b),(c) which may improve the sentence.
(a) well worth the endeavour
(b) worth the endeavour well
(c) the endeavour well worth gin
Choose the correct alternative. In case no improvement is needed
your answer is (d). [SSC CHSL 2014]
(d) No improvement
97. The manager was given a holiday and so he decided to go eer
105. Sohan is pleased at the news yesterday.
(a) has been pleased (b) had been pleased
to the hitch-hiking.
(a) for the hitch-hiking ing
(c) was pleased (d) No improvement
106. She did not like the movie, nor I did.
(b) for hitch-hiking
(c) hitch-hiking
(a) nor did I.
(c) nor did I like it.
107. Old habits die hardly. .ne
(b) nor I like it.
(d) No improvement
(d) No improvement
98. Goaded to frenzy, the bull charged its tormentors.
(a) the tormentors were charged by the bull
(b) the tormentors were being charged by the bull
(a) hard
(c) much hardly
(b) too hard
(d) No improvementt
108. One cannot be indifferent to one’s health, can’t one
(a) can’t be ? (b) can one ?
(c) the bull charged on its tormentors
(c) isn’t it ? (d) No improvement
(d) No improvement
109. The mother with her children were expected.
99. The war was a time of tribulations for all of us. (a) was (b) will
(a) intimacy (b) placidity (c) have (d) No improvement
(c) stupidity (d) No improvement
100. The temptations that bestow young people today are ruining DIRECTIONS (Qs. 110-114): In the following questions, a
them. part of the sentence is underlined. Below are given alternatives
(a) appeal (b) beset to the underlined part at (a), (b) and (c) which may improve the
(c) confront (d) No improvement sentence. Choose the correct alternative. In case no improvement
101. We kept all the old paintings in a place where they would is needed, your answer is (d). [SSC Sub Insp. 2012]
remain safe from harm or danger. 110. The ship ran over when it crashed into an iceberg.
(a) will remain safe (b) are safe (a) get over (b) gave up
(c) may remain safe (d) No improvement (c) went down (d) No improvement

Downloaded From : www.EasyEngineering.net


Downloaded From : www.EasyEngineering.net

Sentence Correction  l 119

111. His wife was contentious. the correct alternative. In case no improvement is needed, your
(a) contagious (b) quarrelsome answer is (d). [SSC Sub Insp. 2013]
(c) content (d) No improvement 120. The gentry of the town was invited.
112. Historians feel there is an earnest need for the review of (a) is invited (b) has been invited
history text books every five years and a revision of the (c) were invited (d) No improvement.
same every ten years. 121. After the written exam, you will also have an oral exam.
(a) practical (b) viva voice
(a) imperative (b) indispensable
(c) vocal (d) No improvement
(c) urgent (d) No improvement
122. The regular use of alcohal, only in small quantities, tends
113. My car broke off on my way to the office.
to cause mischief in many ways to various organs of the
(a) out (b) in
body.
(c) down (d) No improvement (a) though in small quantities
114. Freedom is a wonderful thing, for Jimmy was eager to (b) even in a little quantity
experience it. (c) even in small quantities

ww
(a) though
(c) but
(b) and
(d) No improvement
(d) No improvement
123. If you were the Prime Minister of India what steps would

w.E
DIRECTIONS (Qs. 115 - 119) : In the following questions, a
part of the sentence is underlined. Below are given alternatives
to the underlined part at (a), (b) and (c) which may improve the
you have taken to end unemployment?
(a) will you take
(c) would you take
(b) will you be taking
(d) No improvement

is needed, your answer is (d) asy


sentence. Choose the correct alternative. In case no improvement 124. A high school student is not even understanding the basics
of Chemistry and Physics.

En
[SSC Multi tasking, 2013]
115. On receiving his appointment letter, Ravi treated us with a
(a) does not understand even

sumptuous meal.
(a) treated us to (b) treated us for gin(b) was not even understanding
(c) has even not understood

(c) treated us by (d) No improvement


116. She is scrutinising hard for the final examination. eer
(d) No improvement
DIRECTIONS (Qs. 125-129) : In the following questions, a


(a) recollecting
(c) revising
(b) recapitulating
(d) No improvement ing
sentence a part of the sentence is underlined. Below are given
alternatives to the underlined part at (a), (b), (c) which may
improve the sentence. Choose the correct alternative. In case no
117. Since she directing the play for quite some time, she knows
the actors really well. the Answer sheet. .ne
improvement is needed your answer is (d). Mark your answer in
[SSC Sub Insp. 2014]
(a) Since she has directed
(b) Since she has been directing
(c) Since she was directing
125. He will revise it when he is comes back.
(a) when he come back
(c) when he came back
t
(b) on coming back
(d) No improvement
(d) No improvement 126. The members of the student’s union did not give the
118. You can borrow my laptop as long as you promise not to examination in protest.
misuse it. (a) did not write (b) did not sit for
(a) only long as (b) too long as (c) did not show up for (d) No improvement
(c) so long as (d) No improvement 127. She was kept from her assignment because of her loquacious
119. This is the late edition of the Shakespearean play which was room-mate.
originally published in 1603. (a) noisy (b) irritating
(c) talkative (d) No improvement
(a) later (b) latest
128. James epitomizes everything that a leader should be.
(c) latter (d) No improvement
(a) worships (b) loves
DIRECTIONS (Qs. 120-124) : In the following questions, a
(c) adores (d) No improvement
sentence / part of the sentence is printed in bold. Below are
129. The businessman is respectively connected.
given alternatives to the bold sentence / part of the sentence (a) respectfully (b) respectably
at (a), (b) and (c) which may improve the sentence. Choose (c) receptively (d) No improvement

Downloaded From : www.EasyEngineering.net


Downloaded From : www.EasyEngineering.net

120  l  Sentence Correction

LEVEL-II
DIRECTIONS (Qs. 1-54): In the following questions, in the 9. The main point of his speech was well understood.
given sentences, a part of the sentence is underlined. Beneath each (a) that he spoke (b) in the speech of his
sentence, four different ways of phrasing the underlined part are (c) made when he spoke (d) No correction required
indicated. Choose the best alternative. 10. The indecisive man was readily persuaded to change his
1. The government has given subsidies to the Navratnas but mind again.
there is no telling whether the subsequent one will do. (a) was persuaded ready (b) was ready to persuade
(a) whether the subsequent government will do so. (c) was ready persuaded (d) No improvement
(b) if the government to follow will accept the policy 11. The teacher asked the intruder who was he and why was he
(c) if the government to follow will adhere to the policy occupying his chair.
(d) no telling whether the subsequent one will do so (a) who he was and why he was
2. Rahul Bajaj has done a great job of taking the company to its (b) who he was and why was he
present status, but it is time that he let go of the reins. (c) who he had been and why he had been
(a) let go of the reins (d) No correction required
(b) stepped down 12. The custom of many centuries ago origin is slowly

ww
(c) let go off the reins
(d) delegated responsibility
3. With the pick-up in the standard of education, expensive
disappearing.
(a) which was originated ago many centuries
(b) originating for many centuries

w.E
private schools have started blooming up in every corner of
the country.
(a) started blooming in every corner of the country
(c) which orginated many centuries ago
(d) with many centuries of origin
13. He stayed back so that it can look as if he was unaware of the

asy
(b) have started mushrooming all over the country
(c) have mushroomed all over the country
entire incident.
(a) may look
(c) will look
(b) would look
(d) No correction required
(d) have blossomed all over the country

En
4. It is important that whatever else happens, these two factors
14. The local library has recommended that the books put up
for the used book sale should be in good condition and
should not be messed around with.
(a) It is important that
(b) It is a fact that gin should have no writing in them or be underlined.
(a) and should have no writing in them or be underlined
(b) and should not have writing in them or not be underlined
(c) It should be urgently understood that
(d) It should be understood that eer
(c) and contain no writing or underlining
(d) without containing writing or underlining
5. It must be note that under no circumstance should the
company go in for diversification.
(a) It must be noticed that
small town.
ing
15. The news of her employment soon circulated around the

(a) circulated round the small town


(b) It must be noted that
(c) It must be pointed out that
(b) circulated in the small town
.ne
(c) was circulating across the small town
(d) It should be noticed that
6. British Airspace has been focusing on building European
links.
(a) concentrating on creating European links.
(d) was circulating within the small town

responsible for many motor accidents.


(a) the craziness for speeding that is mad
t
16. It is the craziness for speeding that is maddening that is

(b) the mad craze for speed


(b) pursuing ways of building European connectivity.
(c) the mad craze for speeding
(c) stressing on building European links.
(d) the craze for speeding that is maddening
(d) focusing on forging European links.
17. If they cooperate together by dividing up the work, they
7. The appetite of banks for funds was lost under the onslaught
shall be over with the work faster.
of the slowdown, corporate refused to borrow-even as bank
(a) if they cooperate together by dividing the work
deposit flourished.
(b) if they cooperate by dividing up the work
(a) bank deposits flourished (c) if they cooperate by dividing the work
(b) bank deposits swelled
(d) if they cooperate together by division of work
(c) bank deposits were enhanced
18. Knowing the area was prone to earthquakes, all the buildings
(d) bank deposits flummoxed
were reinforced with additional steel and concrete.
8. He did many mischiefs.
(a) Having known that the area was prone to earthquakes
(a) made many a mischiefs
(b) made much mischief (b) Since they knew the area was prone to earthquakes
(c) Committed many mischiefs (c) Since the area was known to be prone to earthquakes
(d) No correction required (d) Being prone earthquakes

Downloaded From : www.EasyEngineering.net


Downloaded From : www.EasyEngineering.net

Sentence Correction  l 121

19. He sailed for New York on Monday, arriving there on 25. Sharon made it to work in the nicks of times, or else she
Saturday for the much-awaited inauguration of the new would have missed the meeting.
hospital. (a) nick of time (b) nicked time
(a) and arrived there on Saturday for the much-awaited
(c) nick of timeliness (d) nick and time
inauguration of the new hospital.
(b) arriving there on Saturday for the inauguration of the 26. Varun was on cloud nine after having stood first in his class.
much-awaited new hospital. (a) in ninth cloud (b) on nine clouds
(c) arriving there for the inauguration of the much awaited (c) a cloudy nine (d) cloud on nine
new hospital on Saturday. 27. Vithal had a habit of pass the buck when it came to important
(d) and arrived here on Saturday for the long awaited issues at work.
inauguration of the new hospital. (a) pass to bucking (b) passing buck
20. After trying to convince him for a long time, I realized that (c) passing the buck (d) pass buck
he was one of those people who never listens to reason. 28. Puneet raked his brains and tried to find an answer to a
(a) he was one of those people who never listen to reason.
tricky question given in the paper but couldn't find one.
(b) he was one of those people who never .listen to reasoning
(c) he is one of those people who never listen to reason. (a) rake his brain (b) racked his brains
(d) he is one of those people who never listens to reason. (c) racked brains (d) raked brain
29. US secretary of state made it clear that time running out

ww
21. Scientists have long recognized the promise of research
involving human embryos and foetuses for the advancement
of basic science as well as for the development of life saving
vaccines and therapies.
for diplomacy over Iran's nuclear programme and said that
talks aimed at preventing Tehran from acquiring a nuclear

w.E
(a) the promise of research involving human embryos and
foetuses for the advancement of
(b) the promise of research involving human embryos and
weapon would resume in April.
(a) runs out
(c) ran out
(b) was running out
(d) run
foetuses to advance
asy
(c) the promising research which involves human embryos
and foetuses for the advancing of
30. While the war of the generals rage on, somewhere in small
town India, wonderful things are happening, quietly and
minus fanfare.

En
(d) the promising research, which involves human embryos
and foetuses in advancing
(a) rage
(c) rages on
(b) raging
(d) raged on
22. By finding, dating, and interpreting the trash and treasures
of ancient generations, our curiosity about our past as a
species is satisfied in archaeology, and shows us how we gin
31. According to WWF, the small island nation of Samoa was
the first in switch off its lights for Earth Hour.
(a) first to switch of (b) the first to switch off
have become what we are.
(a) generations, our curiosity about our past as a species is
satisfied in archaeology, and shows eer
(c) the first of switch off (d) first in switch of
32. The campaign is significant because not just the youths are
(b) generations satisfy, in archaeology, our curiosity about
our past as a species and show ing
directly appealing to the world but because their efforts
challenge the chimera of normalcy in the area.
(a) not just because (b) just not because
(c) generations, our past as a species satisfies our curiosity
in archaeology and shows
(d) generations, archaeology satisfies our curiosity about
(c) not just
.ne
(d) because just
33. The doctor's association has threatened to go on indefinite
our past as a species and shows
23. Full of speculation that the United States was in a ‘housing
bubble’ that was about to burst, the popular press ran feature
stories depicting the irrational run-up in home prices and a
strike support of their teachers.
(a) on supporting to
(c) for support
t
(b) to supporting
(d) in support of
34. If one prays honestly and sincerely, God will listen to one’s
potential crash which was very likely.
(a) stories depicting the irrational run-up in home prices prayer.
and a potential crash which was very likely. (a) If one pray (b) One if prays
(b) stories about the irrational run-up in home prices and (c) If one will praying (d) No correction required
that there would be a potential crash. 35. Not only I but also my parents are looking forward to see
(c) Stories that describe about the irrational run-up in you soon.
home prices and about the potential for a crash. (a) am looking forward to seeing
(d) stories about the irrational run-up in home prices and (a) are looking to see forward
about the potential for crash. (c) are looking forward seeing
24. Seeing that there was an ongoing sale in one of her favourite (d) are looking forward to seeing
stores, Seeta made a bee line for it immediately after entering 36. They are waiting for her since morning.
the mall. (a) are waiting (b) are awaiting to
(a) made a bee’s line for (c) have been a waiting (d) have been waiting for
(b) make bees lined to 37. If he had asked me, I would have helped him.
(c) made a bee line to (a) If he ask (b) had he asked
(d) No correction required (c) He had asked (d) No correction required

Downloaded From : www.EasyEngineering.net


Downloaded From : www.EasyEngineering.net

122  l  Sentence Correction

38. I am sure that scientists must discover a cure for AIDS by (b) that they may have lost out in the race for a seat in good,
the end of next decade. local institution.
(a) will have discovered (b) might discovered (c) for the reason that they may have lost out in the race for
(c) have discovered (d) must discover a seat in a good, local institution.
39. It is important that whatever else happens, these two factors (d) so that they may have lost out in the race for a seat in a
should not be messed around with good, local institution.
(a) It is important that 45. During the literary renaissance of the 1920s, a large number
(b) It is a fact that of new writers — William Faulkner, Ernest Hemingway,
(c) It should be urgently understood that John Dos Passos, and F. Scott Fitzgerald — sought to record
(d) It should be understood that the inner life of Americans and to scrutinize the American
40. The appetite of banks for funds was lost under the onslaught dream, the dream that anyone can earn his own fortune
of the slowdown, corporates refused to borrow- even as and live happily ever after through hard work, which had
bank deposits flourished become tarnished.
(a) bank deposits flourished (a) the dream that anyone can earn his own fortune and
(b) bank deposits swelled live happily ever after through hard work, which had
(c) bank deposits were enhanced become tarnished

ww
(d) bank deposits flummoxed
41. The MP rose up to say that, in her opinion, she thought the
Woman’s Reservation Bill should be passed on unanimously
(b) the tarnished dream that anyone can make his own
fortune and live happily ever after through hard work

w.E
(a) rose to say that she thought the women’s reservation bill
should be passed
(b) rose up to say that, the women’s reservation bill should
(c) the tarnished dream that anyone can, through hard
work, make his own fortune and live happily ever after
(d) the dream that anyone can earn his own fortune and live
be passed on
asy
(c) rose to say that, in her opinion, she thought that the
women’s reservation bill should be passed
happily ever after, though tarnished, through hard work
46. Two recent statements on the tenure of university professors
offer conflicting points of view: those that say that lifetime

En
(d) rose to say that, in her opinion, the women’s reservation
bill should be passed on
tenure ensures academic freedom and those that say
that lifetime tenure encourages professional laziness and
42. Doctors suggest that patients suffering on account of high
blood pressure take regular exercise stay away from nicotine gin irresponsibility.
(a) those that say that lifetime tenure ensures academic
freedom and those that say that lifetime tenure
and eat plenty of vegetables.
(a) who suffer from high blood pressure should be taking
regular exercise, stay away from nicotine and eat plenty eer
encourages professional laziness and irresponsibility
(b) some declare that lifetime tenure ensures academic
of vegetables.
(b) suffering from high blood pressure have to be taking ing
freedom, and others say that it encourages professional
laziness and irresponsibility
regular exercise, stay away from nicotine and eating plenty
of vegetables.
(c) suffering high blood pressure should take regular .ne
(c) saying that lifetime tenure either ensures academic
freedom or encourages irresponsible laziness
(d) one emphasizes the academic freedom that tenure
exercise, stay from nicotine and eat plenty of vegetables.
(d) suffering from high blood pressure take regular exercise,
stay away from nicotine and eat plenty of vegetables.
and irresponsibility it encourages t
ensures, and the other stresses the professional laziness

47. In the conflict between the Israelis and the Palestinians,


43. The irate resident dragged the builder to the court for failing the refusal of each side to acknowledge each other as a
to transfer the title of the property in their names. legitimate national movement is closer to the heart of the
(a) to the court for failing to transfer the title of the property problem than is any other issue.
to their names. (a) the refusal of each side to acknowledge each other as a
(b) to court for failing to transfer the title of the property to legitimate national movement is closer to the heart of
their names. the problem than
(c) to court in failing to transfer the title of the property in (b) that the refusal of each side to acknowledge another as
their names. a legitimate national movement is closer to the heart of
(d) to court for failing to transfer the title to the property in the problem as
their names. (c) the refusal of each side to acknowledge another as a
44. One of the reasons rich students opt for a foreign degree legitimate national movement is closer to the heart of
is because they may have lost out in the race for a seat in a the problem than
good, local institution. (d) that the refusal of each side to acknowledge another as
(a) because they may have lost out in the race for a seat in a a legitimate national movement is closer to the heart of
good, local institution. the problem than

Downloaded From : www.EasyEngineering.net


Downloaded From : www.EasyEngineering.net

Sentence Correction  l 123

48. Agencies studying discrimination in housing have (a) disapproves of every aspect of the opponent’s platform,
experimentally proved that minority clients are often they later conceded that there must be a basis
discouraged as prospective buyers of residential real estate (b) disapproves of every aspect of the opponent’s platform,
and the antidiscrimination legislation of recent decades it later conceded that there must be a basis
were only mitigating, rather than abolishing, inequity in (c) disapproved of every aspect of the opponent’s platform,
housing practices. they later conceded that there had to bo some basis
(a) the antidiscrimination legislation of recent decades (d) had disapproved of every aspect of the opponent’s
were only mitigating, rather than abolishing, inequity in
platform, it later conceded that there must be a basis
housing practices
(b) in recent decades, the antidiscrimination legislation 52. Large and experienced firms are more efficient at acquiring
only mitigated, rather than abolishing, inequity in smaller and distressed firms than are large and inexperienced
housing practices firms, and converting them to profitable ventures.
(c) that antidiscrimination legislation of recent decades (a) Large and experienced firms are more efficient at
has only mitigated, rather than abolished, inequity in acquiring smaller and distressed firms than large and
housing practices inexperienced firms.
(d) that, in recent decades, antidiscrimination legislation (b) Large and experienced firms are more efficient than

ww
has only mitigated, rather than abolishing, housing
practices’ inequity
49. Unlike German Shepherds or Doberman pinchers, there is
large and inexperienced firms at acquiring smaller and
distressed firms

w.E
an unwillingness on the part of many people to believe that
pit bulls might be fully domesticated.
(a) Unlike German Shepherds or Doberman pinchers,
(c) Large and experienced firms, acquire smaller and
distressed firms more efficiently than large and
inexperienced firms

asy
there is an unwillingness on the part of many people to
believe that pit bulls might be fully domesticated.
(d) Large and experienced firms, more efficient than large
and inexperienced firms at acquiring smaller and

and Doberman pinchers might be fully domesticated,


are unwilling to believe the same of pit bulls. En
(b) Many people, willing to believe that German Shepherds distressed firms
53. The economic growth increased from 7 to 9 percent in

(c) Unlike German Shepherds or Doberman pinchers, pit


bulls bring out an unwillingness in many people to gin November 2010, supporting the expectations that industrial
growth rate in October-December quarter more than
believe that they might be fully domesticated.
(d) Many people are unwilling to believe that pit bulls might
eer
doubled that of the 4 per cent growth rate in industrial
growth for the previous quarter.
(a) that industrial growth in the October-December
be fully domesticated even while they are willing to
believe that German Shepherds and Doberman pinchers
might be. ing
quarter, more than the doubled that of
(b) of industrial growth in the October-December quarter,
50. In contrast to accredited universities that can grant
degrees and whose students can be eligible for Federal Aid,
it more than doubled
.ne
(c) of industrial growth in the October-December quarter,
nonaccredited colleges, while still able to enroll students
and to provide instruction, cannot provide their students
with the same types of services.
(a) nonaccredited colleges, while still able to enroll students
that it would more than double that of
t
(d) that industrial growth in October-December quarter
would more than double.
and to provide instruction, cannot provide their students 54. As a result of surging financial greed, the international
with the same types of services rating agencies upgraded the rating of the credit derivative
(b) nonaccredited colleges cannot provide their students instruments, and hence analysts recommended a strong
with the same types of services, though still able to buy, ignoring the advice of Warren Buffett who warned
enroll students and provide instruction that these instruments would prove not only dangerous but
(c) enrolling students and providing instruction, while ineffective in the long-run
unable to provide their students with the same types of
(a) who warned that those instruments would prove to be
services, are nonaccredited colleges
both dangerous and
(d) the services of nonaccredited colleges, while including
the ability to enroll students and provide instruction, do (b) warning that these instruments would prove not only
not include the same types of services dangerous and also
51. Although the losing party disapproves of every aspect of (c) warning that these instruments would prove itself to be
the opponent’s platform, they later conceded that there both dangerous and
must be a basis for a cooperative government and agreed to (d) who was warning that these instruments would prove
compromise. not only dangerous but

Downloaded From : www.EasyEngineering.net


Downloaded From : www.EasyEngineering.net

124  l  Sentence Correction

DIRECTIONS (Qs. 55 - 59) : In the following questions a part 65. As employees, we are accountable for our stakeholders.
of the sentence is underlined. Below are given alternatives to (a) accountable with (b) accountable to
the underlined part at (A), (B) and (C) which may improve the (c) accountable against (d) No improvement
sentence. Choose the correct alternative. In case no improvement 66. Recently he had insured for a mediclaim policy.
is needed your answer is (D). Mark your answer in the Answer- (a) He had recently insured for
Sheet. [SSC CGL 2012]
(b) Recently he insured for
55. You shall have attended if the court had instructed you to
do so. (c) He insured recently for
(a) would have had to attend (d) No improvement
(b) would attend 67. Everyday, we usually had lunch at 1.30 p.m.
(c) would have to (a) we have had usually
(d) No improvement (b) we have usually
56. The relics of Greece over which such a great deal of evidence
(c) we usually have
has been collected should be preserved.
(a) from which (b) on which (d) No improvement
(c) ascent which (d) No improvement 68. All nations must first become agricultural strong.
57. When the beverage was ready, they drank possibly as much (a) become agricultural strong

ww
as they could.
(a) as much as they possibly could
(b) as much as possibly they could
(b) become strong agriculture
(c) become agriculture strong

w.E
(c) as much as they could possibly
(d) No improvement
58. A citizen is expected to give allegiance to his country of
(d) No improvement
69. An orangutan’s intelligence is as superior to that of man.
(a) is more superior to
origin.
(a) homage
(c) obedience
asy
(b) loyalty
(d) No improvement
(b) is superior to
(c) is superior than that of

En
59. We were with daggers drawn despite attempts to understand
each other.
(d) No improvement
DIRECTIONS (Qs. 70-79): In the following questions, a
(a) in
(c) at
(b) on
(d) No improvement gin
sentence/a part of the sentence is underlined. Below are given
alternatives to the underlined part at (a), (b), (c) which may
improve the sentence. Choose the correct alternative. In case no
DIRECTIONS (Qs. 60 - 69) : In the following questions, a
sentence/part of the sentence is underlined. Below are given
alternatives to the underlined sentence/part of the sentence at (a), eer
improvement is needed your answer is (d). Mark your answer in
the Answer Sheet.
(b) and (c) which may improve the sentence. Choose the correct
alternative. In case no improvement is needed, your answer is (d).
70. ing [SSC CGL, 2014]
Why should you be despaired of your success of your
[SSC CGL 2012]
60. The climate of Karnataka is cooler than Tamil Nadu.
undertaking?
.ne
(a) you despair of the success of your undertaking
(a) is cooler to (b) is cooler than of
(c) is cooler than that of (d) No improvement
61. The Tsunami victims suffered of cholera.
(a) suffered from (b) suffered under
(b) you despair of success of undertaking
t
(c) you be despaired of the success of your undertaking
(d) No improvement.
(c) suffered in (d) No improvement 71. As Rees was going to town in the High Street a savage dog
62. I gave to Sana the keys. attacked him and bit him.
(a) I gave (b) I gave to the (a) going to town a savage dog attacked him and bit him
(c) I gave the (d) No improvement in the High Street
63. If he smokes less he might get rid of his cough. (b) in the High Street a savage dog attacked him and bit
(a) If he smoked less he would get rid of his cough. him in the town
(b) If he had smoked less he might get rid of his cough. (c) going to town in the High Street a savage dog bit him
(c) If he smokes less he might have got rid of his cough. and attacked him
(d) No improvement. (d) No improvement
64. He compensated the loss to me. 72. Something is pretty here that Vineeta can wear to the party.
(a) He compensated the loss for me. (a) Something here is pretty
(b) He compensated me to the loss (b) Something is here pretty
(c) He compensated me for the loss. (c) Here is something pretty
(d) No improvement. (d) No improvement

Downloaded From : www.EasyEngineering.net


Downloaded From : www.EasyEngineering.net

Sentence Correction  l 125

73. I have dreamt all my life to own a beautiful maroon coloured 77. I prevailed on him to vote for you.
car. (a) to (b) at
(a) of owning (b) to owning (c) upon (d) No improvement
(c) at owning (d) No improvement 78. Eager to pass his final exams, studying was the students top
74. Sitting on the hill top, the sun went down watching before priority.
him.
(a) the student’s top priority was studying.
(a) he watched the sun go down.
(b) the sun went down with him watching (b) the student made studying his top priority.
(c) the sun went down when he watched (c) the top priority of studying was made by the student.
(d) No improvement (d) No improvement.
75. The office is soon to be closed. 79. Mr. Dev will not go to the wedding reception without being
(a) just to (b) about to called.
(c) immediately to (d) No improvement (a) if he is not invited
76. He has achieved nothing out of his way worth mentioning. (b) till he is invited
(a) out of the way (b) by the way (c) unless he is invited
(c) in a big way (d) No improvement (d) No improvement

ww
w.E
asy
En
gin
eer
ing
.ne
t

Downloaded From : www.EasyEngineering.net


Downloaded From : www.EasyEngineering.net

126  l  Sentence Correction

Hints & Solutions


LEVEL- I watch television’ and ‘she found the...especially fascinating’
are independent of each other. So the usage of the phrase ‘of
1. (d) 2. (d) 3. (a) 4. (c) 5. (d) which’ in option (a), is inappropriate as to whom it refers is
6. (c) 7. (a) 8. (b) 9. (c) 10. (b) unclear and it also depicts the link between the two parts
11. (b) 12. (a) 13. (d) 14. (a) 15. (a) of sentence. Option (c) is wordy. In option (d), the word
16. (b) 17. (c) 18. (d) 19. (d) 20. (b)
‘especially’ in the end is wrong as the sentence does not
21. (b) 22. (d) 23. (c) 24. (d) 25. (b)
continue further. Also, it does not make any sense. Hence,
26. (a) 27. (d) 28. (b) 29. (d) 30. (b)
option (b) is the correct option.
31. (c) 32. (b) 33. (b) 34. (d) 35. (a)

78. (c) Option (c) depicts the best way of presenting the
36. (c) 37. (b) 38. (b) 39. (d) 40. (c)
underlined part. Option (a) is incorrect as the phrase
41. (c) 42. (d) 43. (b) 44. (d) 45. (b)
46.
51. ww
(c) 47. (a)
(a) 52. (b)
48. (d)
53. (c)
49. (a)
54. (b)
50. (a)
55. (c)
‘causing it’ does not goes with the word ‘although’, which
begins the sentence. The word ‘although’ itself means ‘in
spite of the fact’. So the phrase ‘and yet’ in option (b) is
56.
61.
66.
w.E
(b) 57. (a)
(a) 62. (b)
(d) 67. (c)
58. (c)
63. (b)
68. (b)
59. (a)
64. (c)
60. (d)
65. (a)
inappropriate to use with it. Option (d) is incorrect as the
two parts of the sentence, one showing the action and other
the effect, are dependent on each other. But the use of ‘and’
69.
asy
(d) When we use the word ‘hardly’, it implies a negative
meaning, there is then no need to use not, so the most
suitable use is ‘he hardly had any friends’.
shows that they are independent. Hence, option (c) is the
correct option.
70.
En
(c) The tense in this sentence should be the present perfect
continous because it refers to an action that started at some

79. (d) Option (d) depicts the best way of presenting the
underlined part as it is grammatically correct and concise.
time in past and continues till the present time so right use
is ‘have been living in Mumbai.’ gin Option (a) is incorrect as it is not clear that ‘its’ refers to what.
Also, the phrase ‘many people skip’ is related to breakfast.
71. (a) As a child, should be followed by a phrase that has ‘child’
as its subject, i.e., as a child I was taken by my parents to visit eer
In order to show this relation, a relative pronoun ‘which’
should be used before it. Option (b) is incorrect as it depicts
Jammu & Kashmir or other ways, we can change the first
part of the sentence, the second part describes an action so
ing
an unnecessary shift from active voice to passive voice, thus
making it wordy. Underestimation of breakfast serves as a


72.
the first should be an adverb clause. Thus when I was child
is the most suitable alternative.
(a) ‘Anyone’ implies a person involved. So a personal .ne
reason for people skipping breakfast. But in option (c), the
usage of ‘and’ shows no such relationship of cause and effect.
Hence, option (d) is the correct option.
pronoun will be needed to give conditions later in the
sentence, there must be a ‘he’ or ‘one’ in the sentence. The
noun cannot be in the second person but has to be in the

80. (c) Option (c) depicts the best way of presenting the
t
underlined part as it is grammatically correct and concise.
Option (a) is incorrect as the relative pronoun ‘which’ is used
third person because ‘Anyone’ refers to an unknown third for a thing and not for ‘people’. Option (b) is incorrect as it
person. is not clear that ‘they’ refers to ‘people’ or ‘shipwrecks’. Also,
73. (c) The use of the modal ‘should’ is unnecessary in this the emphasis is on the finding of treasure in shipwrecks and
statement. It is a statement giving directions. not simply on the finding of treasure. Option (d) is incorrect
74. (c) The first part of the sentence is directed at a subject, as with the word ‘believe’, the use of ‘that’ is more appropriate
which is ‘eaten in portugal only’, i.e., the potato, therefore than the word ‘about’ in the given context. Hence, option (c)
the comma should be followed by a phrase with potato as its is the correct option.
subject and not Indians. Of (c) & (d), (c) is the right option
81. (c) Since the whole sentence is in the present tense “yet
because ‘suspicious to Indian’ is not an appropriate usage.
to decided” should be replaced with “yet to decide”.
75. (d) The comparative degree for verb ‘fast’ is ‘faster’ and
there is no need to add more to it. 82. (d) Rohan’s mother cannot be feed up. It has no meaning.
76. (d) Replace ‘they’ with ‘those’. This should be replaced with fed up of.
77. (b) Option (b) depicts the best way of presenting the 83. (c) Since the subject is a female “caught the eye” should
underlined part. The two parts of the sentence ‘Anita liked to be replaced with “caught her eye”.

Downloaded From : www.EasyEngineering.net


Downloaded From : www.EasyEngineering.net

Sentence Correction  l 127

84. (e) LEVEL- II


85. (e)
1. (a) This is an easy one because the other choices don’t fit in
86. (e) properly even when we read them along with the sentence.
87. (c) The correct spelling is ‘parties’ 2. (a) The given statement means that Rahul Bajaj, who did
88. (d) The correct spelling is ‘meeting’ a nice job of making the company what it is now, should
89. (a) Replace ‘found’ with ‘fond’ now go away or leave it. ‘Let go’ refers to stop holding onto
90. (3) Replace ‘strand’ with ‘stand’ something, and in this context it means to stop holding
91. (d) onto this job in the company or the responsibilities. Rein
92. (e) as a noun could refer to anything which is a means of
93. (d) 94. (b) 95. (c) 96. (a) 97. (c) control. So, the given statement is correct: option (a) is
98. (d) 99. (d) 100. (b) 101. (d) 102. (c) the answer. Option (b) is not more appropriate than (a)
because ‘stepping down’ can also mean reducing the level/
103. (b) 104. (c)
scope, so then it would mean that Rahul Bajaj should not
105. (c) Sohan was pleased at the news yesterday. (the sentence leave the company completely, which is different to what

ww
is in past tense)
106. (a) She did not like the movie, nor did I.
is stated in the statement. Option (c) is wrong, because the
word ‘off ’ is different from ‘of ’. Option (d) is wrong, because

w.E
107. (a) Old habits die hard.
108. (b) One cannot be indifferent to one’s health, can one?
109. (a) The mother with her children was expected.
delegating responsibility means giving the responsibility to
someone else, but that would slightly change the statement,
as it does not say that Rahul Bajaj should leave and give
110. (c)
asy
111. (b) Contentious means causing or likely to cause an
his responsibility to someone; this would be like adding
information from our side, which is not allowed.
argument. Best alternative is quarrelsome.
112. (c)
En 3. (c) The given statement is trying to mean that as the
standard of education has improved or picked up, expensive
113. (c)
114. (b)
gin private school has grown and spread all over the country (all
over the country, and every corner of the country, have the
same meaning). ‘Blooming up’ is wrong because it means
115. (a) Treated us to which also means offered us
116. (c) Scrutinising means revising
eer
growing up and is used to express a positive connotation.
Here, it is clear that the author wants to state things in a
117. (b) Since she has been directing
118. (c) So long as will be the exact phrase ing
negative shade.
Now, ‘have started blooming or have started growing or
119. (b) Latest will be the correct word here
120. (c) Gentry is plural in number. .ne
have started mushrooming’ are all redundant sentences as
by just stating ‘have grown/spread/mushroomed’ it can still
121. (b)
122. (b) Here, even in a little quantity should be used. To use
plural form is not proper.
t
mean that the action started sometime in the past and is still
going on i.e. present perfect simple. So options (a) and (b)
can be ruled out. Option (a) is wrong too, because then the
123. (c) Conditional sentence is in past simple. Hence, would you tense would be simple past, which would mean that these
take ... should be used. private school grew or spread in the past, but this is not the
124. (d) Here, generality is evident. Hence, present simple should case in present. Option (d) is wrong, because ‘to blossom’
be used. is to develop and reach a promising state, but the sentence
has not defined how much these schools have grown and
125. (b) He will revise it on coming back.
what this stage of growth can be classified as. (an example
126. (b) The members of the student’s union did not sit for the
of the use of blossomed: their friendship blossomed into
examination in protest.
romance).
127. (d) She was kept from her assignment because of her
4. (a) The given statement is correct. Option (b) cannot
loquacious roommate.
be true, because saying something is ‘important’ and
128. (d) James epitomises everything that a teacher should
something is a ‘fact’ have two different meanings. A fact
be. Epitomises means to personify, typify the traits/
refers to something that can be argued upon, and there is
characteristics.
a certain proof to affirm its validity, whereas ‘important’
129. (a) The businessman is respectfully connected.
means something that is crucial or vital or necessary, and

Downloaded From : www.EasyEngineering.net


Downloaded From : www.EasyEngineering.net

128  l  Sentence Correction

that is completely correct in the context as the speaker is taking funds, or not borrowing money from banks, hence
stressing that the ‘two factors’ are vital. Options (c) and (d) the amount of funds with the banks increased. Option (b) is
are wrong, because understanding something is different right because swell means increase in magnitude or number
from considering it important; understanding something and that fits into the context. Option (a) is wrong because
means you ‘get it’, but ‘understanding the importance’ is a flourished means to grow quickly or rapidly, but there
different thing. Option (c) is also incorrect, because there is nothing to indicate in the statement to show that there
is nothing in the statement to suggest that the matter is was this ‘rapid pace’. Option (c) can be rejected, because
‘urgent’. enhanced can also mean increase in quality too, so if we
5. (b) Let us understand the statement first: the speaker is had to pick between swell or enhance, then swell is more
expressing that it is very important to ensure that the appropriate for this statement. Option (d) is wrong because
company does not go for diversification (increasing the flummoxed is a very different word, as explained already.
number of products/services that a company produces) 8. (c) 9. (d) 10. (d) 11. (b)
i.e. the company should not go in different directions 12. (c) 13. (b)
or set different goals. When we say ‘it must be noted’ we 14. (c) The modal ‘should’ need not be repeated since the
are suggesting that the people consider or understand conjunction ‘and’ is being used. The should in the sentence

ww
the importance, whereas ‘notice’ refers to observing. It is
clear from the context explained above that ‘noted’ is the
appropriate word. Hence, option (b) is correct and (a)
15.
will be used for both the parts.
(b) ‘in’ is the correct preposition to be used with the verb

w.E
is incorrect. Option (c) is wrong too, because ‘it must be
pointed out’ refers to showing or talking about something
so that others will notice it, hence it will be clearly out of
16.
circulated.
(c) ‘craziness’ is no word, the right noun is craze. If that
is maddening is chosen it will be followed by ‘that is

asy
the context. Option (d) is similar to (b), the only difference
being of ‘should’ instead of the presence of ‘must’. The
responsible’ which will not be appropriate ‘mad craze’ is the
correct use, and this craze is not for the noun ‘speed’ but the
verb ‘speeding’, i.e., driving vehicles at high speed.

En
difference between must and should is only of the degree.
When saying ‘you must’, you are saying that it is required
17. (c) Co-operate has an implicit meaning of together. It means
‘work together’, so together must be eliminated. Similarly
and necessary, whereas in saying ‘you should’, you are
merely suggesting something. Looking at the context, ‘must’
is the more appropriate word.
gin ‘dividing’ means ‘breaking up’ so ‘up’ from ‘dividing’ must
also be eliminated.
6. (a) The given statement means that the British Airspace is
‘focusing’ on spreading its network, but probably having
18.
eer
(c) The second part of the sentence is an action taken as a
result of an incident, so the first part of the sentence must
their flights in more European countries. Let’s first look at
the words: concentrating, pursuing, stressing, and focusing. ing
give the incident and use the word ‘since’. (b) is not the right
option because it will make the first part active voice and
second passive voice which must not happen. Similarlity of
Focusing and concentrating are synonymous to each
other and they refer to directing attention on someone or
19. .ne
voice should be maintained in the sentence.
(a) “arriving” is not in agreement with “sailed” in option (d)
something. ‘To pursue’ is ‘to chase, follow or seek to attain
or accomplish a goal over a long period of time’. ‘Stressing’
means ‘emphasizing something’. Hence, the appropriate
word for the sentence would be concentrating or focusing.
20.
“here” is incorrect.
t
(b) “reason” is not the correct word for this context. There
is no need to put an ‘s’ in verb in case of third person plural
We have to pick between the options (a) and (d) now: number.
forging can refer to creating something strong or enduring. 21. (a) The question tests the idiomatic expression. ‘Recognizing
We cannot say if the company is looking to form links for the promise’ is correct expression and ‘promise of X’ as well
the long term, or the ones that would hopefully last, or it is as ‘for Y’ is idiomatic expression. Both criteria are met and
just something they are trying out for sometimes. So, option thus the sentence is correct in the given form.
(a) is correct. (a) Correct. This option correctly uses the object ‘promise’
7. (b) We have to basically pick the most appropriate word and uses the idiomatic expression ‘for the advancement of ’.
among: flourished, swelled, enhanced, and flummoxed. (b) ‘promise’ to is with a different meaning from ‘promise
Let’s look at the meanings of these words: for’. The second sense is correct in the given situation.
Flourish refers ‘to grow or develop rapidly or vigorously’; (c) Recognizing the promise is correct idiom, while this
swell means ‘grow or expand’; enhanced can refer to option uses ‘the promising research’ giving a sense that they
‘something has been made better or increased’; flummoxed recognize the ‘research’ rather than the ‘promise’ of it.
means ‘bewildered or perplexed’. The given statement means (d) The error of illogical shift of object is not corrected.
that the appetite of banks was lost and corporate were not Moreover, the use of ‘in advancing’ is not idiomatic.

Downloaded From : www.EasyEngineering.net


Downloaded From : www.EasyEngineering.net

Sentence Correction  l 129

22. (d) Dangling modifier and illogical predication. 31. (b) the first to switch off.
The sentence starts with a prepositional phrase that should 32. (a) not just because.
logically modify the subject. In the given sentence the 33. (d) in support of.
prepositional phrase by ‘finding, dating, and interpreting...’ 34. (d) 35. (d) 36. (d) 37. (d)
illogically modifies ‘our curiosity’, resulting in dangling 38. (a)
modifier error. Further more, the use of ‘curiosity’ as subject 39. (d) is correct because whatever is important should be
makes it mean that our curiosity ‘shows how have become understood.
what we are’. This is the error of illogical predication. 40. (b) is correct as the word “swelled” is the better option to use
(a) The illogical predication and dangling modifier error than “flourished”, as swelled means growing.
with the prepositional phrase ‘by finding...’ make this option 41. (a) is the correct choice as gramatically it is the only one
incorrect. which is making sense and the bill should be passed,
(b) The prepositional phrase ‘by finding...’ cannot function “on” being wrongly used.
as subject of the verb ‘satisfy’. This would have been correct, 42. (d) (a) is verbose; (b) has structural and parallelism errors.
In (c) preposition ‘from’ is omitted. Only (d) is correct in
though awkward, in the absence of preposition ‘by’.
terms of parallelism and the right use of the subjunctive

ww
(c) The subject ‘our past’ is illogically modified the
prepositional phrase beginning the sentence creating
dangling modifier error. The problem of illogical predication

43.
form.
(d) Visiting a court for the primary purpose does not

w.E
persists as the structure means ‘our past satisfies our
curiosity’.
(d) Correct. The use of archaeology as subject removes both
warrant the use of an article. So ‘the court’ is incorrect.
When we imply a legal right, we use the term ‘title to’ and
not ‘title of.

asy
the dangling modifier error and illogical predication error.
The sentence is parallel with parallel verbs ‘satisfies’ and
44. (b) The term ‘because’ cannot be used with ‘reason’. So
option I can be ruled out. Since the first part of the sentence
already states ‘one of the reasons’, the use of the phrase ‘for
‘shows’.
23. (d) Wordiness and parallel structure. En the reason’ in the second part of the sentence is incorrect. So
option (c) can be ruled out. ‘So that’ does not make sense in
The sentence presents a redundancy problem by using the
words ‘likely’ and ‘potential’ together. The sentence uses gin
45.
the given context.
(c) The original version is confusing because the clause
a relative clause in stead of a phrase making the structure
wordy.
eer
which had become tarnished is awkwardly separated from
dream and the prepositional phrase through hard work is
(a) Redundancy of expression makes this sentence incorrect.
(b) The expression is wordy and unparallel because ‘stories
about X’ and ‘that there would be a potential crash’ are not
ing
awkwardly separated from the verb it modifies, earn. In
choices (b) and (d), the prepositional phrase is misplaced.

parallel.
(c) The use of relative clause after stories makes it not only
46.

.ne
(d) In the original underlined portion, those is incorrect. To
express respectively two singular points of view, the noun or
pronoun which refers to each must be singular; those is, of
wordy but also unparallel to the prepositional phrase after
‘and’.
(d) Correct. The sentence uses prepositional phrase 47. (a) (a) gives the correct usage.
t
course, plural. Only choice (d) provides a clearly singular
reference, one, for each viewpoint.

using concise structure and is parallel because it use two 48. (c) (c) shows the correct usage of tenses.
prepositional phrases with conjunction ‘and’. 49. (b) (b) corrects the misplaced modifier.
24. (d) Idiom make a beeline for something means : to go 50. (a) (a) gives the correct usage.
straight towards something / somebody as quickly as you 51. (d) Only (d) shows the correct sequence of tenses.
can. 52. (b) In the given sentence comparison has been made
25. (a) Idiom in the nick of time means : at the very last moment; between “large and experienced firms” and “large and
just in time before something bad happens. inexperienced firms” which is best brought out by option
26. (e) Idiom on cloud nine means : extremely happy. (b) is incorrect because ‘are’ is missing.
27. (c) passing the bucks 53. (d) Since the sentence is saying something about expectations
28. (b) Idiom rack your brain (s) means : to think very hard or to be met in future. Therefore (d) is the correct choice. Also
for a long time about something. ‘supporting the expectations’ should be followed by that.
54. (a) (b), (c) and (d) would be wrong because warning and
29. (a) runs out.
was warning will render the sentence inconsistence in terms
30. (c) rages on.
of tense.

Downloaded From : www.EasyEngineering.net


Downloaded From : www.EasyEngineering.net

130  l  Sentence Correction

55. (a) Would have had to attend 70. (a) The sentence requires an improvement. The underlined
56. (c) ascent which portion must be rectified to ‘you despair of the success of
57. (a) as much as they possibly could your undertaking’.
58. (b) Alternative should be ‘Loyalty’ 71. (d) The statement requires no improvement. The underlined
59. (c) at portion of the statement suits best with the meaning of the
60. (c) Here we compare the climate of Karnataka with the statement.
72. (c) The underlined portion of the statement requires an im-
climate of Tamil Nadu and not with the Tamil Nadu itself;
provement. It may be rectified as ‘here is something pretty
hence we use ‘than that of’.
that Vinita can wear to the party.
61. (a) If someone suffers from an unpleasant or difficult
73. (a) The underlined portion requires an improvement. It may
experience or situation, then we use ‘suffer from.’ Ex: Shiela
be rectified as ‘of owning’. So, the sentence will be as ‘I have
is suffering from ill health. Lately factories are suffering from
dreamt all my life of owning a beautiful maroon colored car’.
a desperate shortage of labours.
74. (a) The underlined portion in the sentence requires an im-
62. (a) The correct arrangement of sentence is - I gave Sana the
provement. It may be rectified as ‘he watched the sun go
keys.
down’. So, the sentence will be ‘sitting on the top of the hill
63.
ww
(a) When you are using if to talk about something that is
unlikely to happen or is impossible, use the past tense in the
if-clause, not present. Ex: If someone gave me (NOT gives
he watched the sun go down’.
75. (b) The underlined portion of the sentence has to be rectified.

64.
65.
w.E
or would give me) the money, I’d buy a car tomorrow.
(c) ‘He compensated me for the loss.’ is the correct answer.
(b) The proposition ‘to’ is complementary with accountable.
‘Soon to’ may be replaced with ‘about to’. So, the sentence
will be as ‘the office is about to close’.
76. (a) The underlined portion of the sentence needs to be

66.
asy
(a) Recently denotes not long ago and thus usually takes
perfect tense.
rectified. It may be replaced with ‘out of the way’. So, the
sentence will be as ‘He has achieved nothing out of the way
worth mentioning’.
67.
En
(c) Present tense shows what exits or happening now. It also
denotes a habit which in this case is the timing of having
77. (c) ‘On’ may be replaced with ‘upon’. So, the sentence will
be as ‘I prevailed upon him to vote for you’.

68.
lunch.
(a) Here ‘agriculturally’ is the adverb that adds to the meaning gin
78. (b) The underlined portion may be replaced with ‘the student
made studying his top priority’. So, the sentence will be as
of the adjective ‘strong’. Other examples are ‘slowly’ in
‘He ran slowly’, ‘very’ in ‘It’s very hot’, or ‘naturally’ in eer
‘Eager to pass his final exams the student made studying his
top priority’.

69.
‘Naturally, we want you to come.’
(b) Superior always takes preposition ‘to’. Ex: Your computer
ing
79. (c) The underlined portion in the sentence may be replaced
with ‘unless he is invited’. So, the sentence will be as ‘Mr.
Dev will not go to the wedding reception unless he is invited’.
is far superior to mine.

.ne
t

Downloaded From : www.EasyEngineering.net


Downloaded From : www.EasyEngineering.net

10
PTER
CHA

Choosing the Correct Sentence

T ww
hese questions can be related to the sentence correction or error finding questions in a way, but they are framed in a different
manner. In these questions, a sentence is written in four different ways, and we have to pick the most appropriate sentence
out of these four. It might happen that nothing may be underlined here (it is not necessary), so the student needs to read the

w.
directions carefully.

HOW TO SOLVE
Ea
(i) First of all, read all the four statements to know the context and subject of these statements, and the information it is trying to
convey.
syE
(ii) Even without knowing what the actual statement would be like, while reading the options, you can eliminate the ones which
certainly seem to have a grammatical error.

ngi
(iii) After reading all the statements and understanding what is being conveyed through the sentence, frame the grammatically
correct and concise sentence in your mind. Re-reading the options and finding the answer just by elimination could eat up your
time.
nee
(iv) These sentences are sometimes quite long, and are designed to confuse the students as to how they differ and what the right
sentence should be like. So, you need to focus hard on each of the sentence.

rin
(v) Sometimes, the choice between two options could hinge on one word. In this situation, you would have to rely on the fact that
you have understood the context well and also know the usage of the word.

TIPS g .ne
(i) Improper sentence construction is often the error in these questions, so make sure you know how to construct a sentence.
(ii) Find out the subject and verb of the sentence. This will help you to spot the correct sentence.
(iii) See if adverbs and adjectives are used appropriately. t
(iv) Knowing the verb tense might not be easy here. The problem is that you do not have any base sentence(s) that can help you
ascertain the proper tense form. You are on your own here.
(v) Redundancy is another common error in these sentences. Keep in mind though that a redundant sentence is not always the
wrong one. Again it is your own discretion that you should base your answer on. Most importantly, the sentence should make
sense grammatically as well as logically.

SKILLS REQUIRED
Refer Grammar Section.

COMMON MISTAKES
(i) Marking an option because of a gut feeling: Always use a grammatical rule to select or rule out a sentence.
(ii) Comparing the options: This is one of those types of questions which if done by elimination, could use up more than the
required time.
(iii) Logical sense: Sometimes students only concentrate on finding the sentence which is grammatically error-free, but forget that a
sentence has to be logically correct too.

Downloaded From : www.EasyEngineering.net


Downloaded From : www.EasyEngineering.net

132  l  Choosing the Correct Sentence

EXAMPLES
Directions for Examples 1–12:  In each of the questions below, four different ways of writing a sentence are indicated. Choose the best
way of writing the sentence.
Example 1.
A. The main problem with the notion of price discrimination is that it is not always a bad thing, but that it is the monopolist
who has the power to decide who is charged what price.
B. The main problem with the notion of price discrimination is not that it is always a bad thing, it is the monopolist who has
the power to decide who is charged what price.
C. The main problem with the notion of price discrimination is not that it is always a bad thing, but that it is the monopolist
who has the power to decide who is charged what price.
D. The main problem with the notion of price discrimination is not it is always a bad thing, but that it is the monopolist who
has the power to decide who is charged what price.
(a) A (b) B (c) C (d) D
Explanation  (c); Option (c) is right, as it is grammatically correct and makes logical sense. Option (a) is incorrect, because
it is logically incorrect: ‘the main problem is not that it is always a bad thing’. This opens up the window that sometimes, price

ww
discrimination is not a bad thing. Option (b) is wrong, because the second clause of the sentence, which comes after the comma has
ambiguity. It does not have a ‘that’; that is used to single out or point to something spoken by the speaker. The given statement does

w.E
not make clear what the main problem is. Option (d) is incorrect, because of a similar reason to the previous option. In the part of
the sentence ‘notion of price discrimination is not it is always a bad thing’ ‘that’ should come after ‘not’ to make the sentence more
clear.
Example 2.

asy
A. A symbiotic relationship develops among the contractors, bureaucracy and the politicians, and by a large number of devices
costs are artificially escalated and black money is generated by underhand deals.

En
B. A symbiotic relationship develops among contractors, bureaucracy and politicians, and costs are artificially escalated with a
large number of devices and black money is generated through underhand deals.

gin
C. A symbiotic relationship develops among contractors, bureaucracy and the politicians, and by a large number of devices
costs are artificially escalated and black money is generated on underhand deals.
D. A symbiotic relationship develops among the contractors, bureaucracy and politicians, and by large number of devices costs
are artificially escalated and black money is generated by underhand deals.
(a) A (b) B (c) C eer (d) D
Explanation 
ing
(b); The most important distinction here is the incorrect positioning of the phrase: ‘by large number of devices’.
It should not be placed after the comma after politicians. This is so, because otherwise the flow of the sentence is lost.
Example 3.
.ne
A. The distinctive feature of tariffs and export subsidies is that they create difference of prices at which goods are traded on the
world market and their price within a local market.

the world market and their prices in the local market. t


B. The distinctive feature of tariffs and export subsidies is that they create a difference of prices at which goods are traded with

C. The distinctive feature of tariffs and export subsidies is that they create a difference between prices at which goods are traded
on the world market and their prices within a local market.
D. The distinctive feature of tariffs and export subsidies is that they create a difference across prices at which goods are traded
with the world market and their prices within a local market.
(a) A (b) B (c) C (d) D
Explanation  (c); The use of ‘difference … between’ is the whole distinguishing point here.
Example 4.
A. Any action of government to reduce the systemic risk inherent in financial markets will also reduce the risks that private
operators perceive and thereby encourage excessive hedging.
B. Any action by government to reduce the systemic risk inherent in financial markets will also reduce the risks that private
operators perceive and thereby encourage excessive gambling.
C. Any action by government to reduce the systemic risk inherent due to financial markets will also reduce the risks that private
operators perceive and thereby encourages excessive hedging.
D. Any action of government to reduce the systemic risk inherent in financial markets will also reduce the risks that private
operators perceive and thereby encourages excessive gambling.
(a) A (b) B (c) C (d) D

Downloaded From : www.EasyEngineering.net


Downloaded From : www.EasyEngineering.net

Choosing the Correct Sentence  l 133

Explanation  (b); This one is easy, because people would gamble more when there is reduced risk and hence even if you don’t
know the meaning of hedging (any technique designed to reduce or eliminate financial risk; for example, taking two positions that
will offset each other if prices change) you can reduce the answer to either option (b) or (d). Once you read them, you can quickly
negate option (d) for a subject-verb disagreement.
Example 5.
A. Creativity in any field is regarded not only as valuable for itself but also as a service to the nation.
B. Creativity in any field is not regarded only as valuable on its own, but also as a service to the nation.
C. Creativity, in any field, is not only regarded as valuable, but also as a service to the nation.
D. Creativity in any field is regarded not only as valuable in itself but also as a service to the nation.
(a) A (b) B (c) C (d) D
Explanation  (c); Option (c) is concise and correct. Option (a) makes creativity important for itself, which is not the in-
tended meaning. Options (b) and (d) are awkward and incorrectly worded.
Example 6.
A. If precision of thought had facilitated precision of behaviour, and if reflection had preceded action, it would be ideal for
humans.
B. It would be ideal for humans if reflection preceded action and precision of thought facilitated precision of behaviour.

(a) A ww
C. It would be ideal for humans if precedence of reflection was followed by action and precision of thought, by precise behaviour.
D. It would have been ideal for humans, if precise action and behaviour preceded precise reflection.
(b) B (c) C (d) D
Explanation 
w.E (b); Option (a) is awkward and incorrectly worded. Option (d) is incorrect because the verb tense used here
makes it look like humans are no more or the situation being talked about was in the past. This isn’t true and hence this option can
be negated. Option (c) changes the meaning of the sentence and makes it incorrect.
Example 7.
asy
A. We are forced to fall back on fatalism as an explanation of irrational events.
B. We are forced to falling back on the fatalism as an explanation of irrational events.

En
C. We are forced to fall back on fatalism as explanations of irrational events.
D. We are forced to fall back to fatalism as an explanation of irrational events.
(a) A
Explanation 
(b) B
gin
(c) C (d) D
(a); Fatalism is a philosophical doctrine according to which all the events are predetermined, i.e. everything
that happens was bound to happen, and what will happen, will happen and humans are powerless to change that. The statement

eer
means that when there is an irrational event or unusual event, then the only explanation we have of it, is fatalism. The correct idiom
is ‘fall back on’ which negates option (d).

ing
Option (c) is wrong also because ‘explanation’ is a countable noun, hence it needs a determiner, and also, ‘fatalism’ is a singular
noun, not two or more, hence options (c) and (b) can be ruled out; the correct form is ‘an explanation’.
Example 8.

.ne
A. From the sixteenth century onwards, people started feeling disdainful and self-conscious about their body and its products
that led to a heightened focus on emotional and bodily regulations.
B. The heightened focus on controlling the body and emotions comes from disdain and self-consciousness about the body and
its products, found in the sixteenth century.
C. From the sixteenth century onwards, a growing disdain for and self-consciousness about the body and its products took
hold, leading to a heightened focus on emotional and bodily regulation.
t
D. The heightened focus on emotional and bodily regulations started from the sixteenth century onwards, when people felt
disdain and self-consciousness about the body and its products.
(a) A (b) B (c) C (d) D
Explanation  (c); Option (c) is the best because it captures the essence of the sentence correctly. Other options make the least
sense.
Example 9.
A. The running of large businesses consist of getting somebody to make something that somebody else sold to somebody else
for more than its cost.
B. The running of a large business consists of getting somebody to make something that somebody else will sell to somebody
else for more than it costs.
C. The running of a large business consists of getting somebody to sell something that somebody else made for more than it
cost.
D. The running of large businesses consist of getting somebody to make something else that somebody else will sell to somebody
else for more than it costs.
(a) A (b) B (c) C (d) D

Downloaded From : www.EasyEngineering.net


Downloaded From : www.EasyEngineering.net

134  l  Choosing the Correct Sentence

Explanation  (b); It will be noted that ‘running of something’ would always take ‘consists’ and not ‘consist’. Again, the sub-
ject-verb agreement case leads to the negation of a few options {(a) and (d)}. Option (c) is also incorrect because how can someone
make something for less than what it costs?
Example 10.
(a) He made a blunder mistake. (b) I have learnt this lesson word by word.
(c) She does not know swimming. (d) He got his daughter married.
Explanation  (d); Option (a) is incorrect because blunder itself refers to a mistake, so the word ‘mistake’ makes the given
sentence redundant. Option (b) is wrong, because the idiom ‘word by word’ means one word at a time, and this does not make
sense in the given sentence. Option (c) is not right because it should be ‘she does not know how to swim’.
Example 11.
(a) The average male investor expects to see all his investment stocks moving up, and they are often disappointed.
(b) The people in my country are smarter than in other countries.
(c) The number of failures of product in final quality inspection are increasing every year.
(d) Before restructuring a firm, the CEO must consider the employees.
Explanation  (a); Option (a) has two grammatical errors: pronoun and verb inconsistency with the word “they are”. The cor-

ww
rection would be “he is”. Another error in the given sentence is that there both gerund and infinitive are used, either one of it must
be used and thereby, show consistency; correction would be to replace moving up with to move up. Option (b) has a comparison

w.E
error. It is comparing people in a country with something not specific in other countries. The correction would be: “are smarter
than those in other countries.” Option (c) has a subject-verb disagreement. The correction would have ‘products’.
Example 12.

asy
(a) Each of the girls living in the orphanage had been ill-treated by her family before they were abandoned.
(b) Each of the girls living in the orphanage were ill-treated by their family before they were abandoned.
(c) Each of the girls living in the orphanage had been ill-treated by her family before she was abandoned.

Explanation  En
(d) Each of the girls living in the orphanage was ill-treated by her family before she had been abandoned.
(c); Option (a) is incorrect because of the pronoun they used in the sentence leading to the pronoun-anteced-

gin
ent error. ‘Each’ refers to a singular noun, so the pronoun should be ‘her’. It also has a wrong verb ‘were’. It should be ‘was’ because
of the singular noun. Option (b) is wrong, because of two errors. First is the pronoun-antecedent error with ‘they’ and ‘their’, and

eer
second is the error with the verb tense. The given sentence refers to two past events, in such a way that one continuous event (ill-
treated) occurred before another, so the correct tense is the past perfect progressive tense. Option (c) uses this tense correctly. Op-
tion (d) is wrong, because it uses the wrong verb tense.
Example 13.
ing
(a) It was thought that freedom and prosperity would spread gradually throughout the world through an orderly process, and it
was hoped that tyranny and injustice would continually diminish.
.ne
(b) It was gradually thought that throughout the world, freedom and prosperity would spread through an orderly process, and
it was hoped that tyranny and injustice would continually diminish.
t
(c) Through an orderly process, it was thought that freedom and prosperity would spread gradually throughout the world, and
it was hoped that tyranny and injustice would continually diminish.
(d) It was thought, through an orderly process that freedom and prosperity would spread gradually throughout the world and it
was hoped that tyranny and injustice would continually diminish.
Explanation  (a); Before picking the right option, let us understand the meaning of gradually and orderly: gradually means
in a gradual manner, or bit-by-bit, whereas orderly means methodically arranged, or organized. Option (a) has used these words
appropriately. Option (b) is wrong because ‘gradually thought’ makes no sense. Options (c) and (d) are wrong because they sound
ambiguous: it seems as though the thought came through a gradual process, which of course is not what is being conveyed.
Example 14.
(a) Each of the six boys in the class has finished their task. (b) One must finish his task in time.
(c) Either Ram or Shyam will give their book. (d) Each of the girls must carry their own bag.
Explanation  (b); Option (b) is correct because ‘in time’ means ‘without being tardy’. Option (a) is incorrect because of the
improper pairing of the noun “Each of the six boys” with the pronoun “their”. Here a singular noun is being paired with a plural
pronoun. Option (c) and option (d) also make the same grammatical mistake as does option (a).
Directions for Examples 15 & 16:  In each question, there are five sentences or parts of sentences that form a paragraph. Identify the
sentence(s) or part(s) of sentence(s) that is/are correct in terms of grammar and usage. Then, choose the most appropriate option.

Downloaded From : www.EasyEngineering.net


Downloaded From : www.EasyEngineering.net

Choosing the Correct Sentence  l 135

Example 15.
A. When I returned to home, I began to read B. everything I could get my hand on about Israel.
C. That same year Israel’s Jewish Agency sent D. a shaliach a sort of recruiter to Minneapolis.
E. I became one of his most active devotees.
(a) C & E (b) C only (c) E only
(d) B, C & E (e) C, D & E
Explanation  (c); Sentence A is incorrect, because the correct form is ‘returned home’. The preposition ‘to’ is not required
here.
Sentence B is wrong, because the idiom or phrase is ‘hands on’ not ‘hand on’, apart from the correct usage of the well known
phrase, it’s logical to think that you would use both your hands to read or hold onto something, especially while reading a book.
Sentence D is wrong because there should be a comma after ‘a shaliach’. An appositive phrase and the noun should be separated
with a comma. Same is the case with sentence C.
Example 16.
A. So once an economy is actually in recession,
B. the authorities can, in principle, move the economy.
C. out of slump - assuming hypothetically.

ww
D. that they know how to - by a temporary stimuli.
E. In the longer term, however, such policies have no affect on the overall behaviour of the economy.
(a) A, B & E (b) B, C & E (c) C & D

w.E
(d) E only
Explanation 
(e) B only
(e); Sentence A lacks a proper punctuation after the initial ‘So’.
Sentence C is wrong, because the hyphen is used incorrectly, there should be a semicolon instead. Sentence D is wrong because

about in the phrase.


asy
‘stimuli’ is the plural form of stimulus, so the correction would be ‘a temporary stimulus’ as only one, particular stimulus is talked

Sentence E is wrong because instead of ‘affect’, there should be ‘effect’ in the sentence. (See affect-effect)

En
gin
eer
ing
.ne
t

Downloaded From : www.EasyEngineering.net


Downloaded From : www.EasyEngineering.net

136  l  Choosing the Correct Sentence

Practice Exercise
LEVEL-I
DIRECTIONS (Qs. 1–15): In the following sentences, four (c) Because of his tennis elbow injury, Pete Sampras has not
options are given. You are required to identify the best way of been and possible never would be able to pick up the bat
writing the sentence in the context of the correct usage of standard again.
written English. While doing so, you have to ensure that the (d) Because of his tennis elbow injury, Pete Sampras has not
message being conveyed remains the same in all the cases. been able to and possibly never will be able to pick up
1. If he was to decide to go to college, one would ommend that the bat again.
he plan to go to × 42, Ahmedabad. 5. Had he realised how close he was to failing, he would not
(a) If he was to decide to go to college, one would 1 have gone to the party.
recommend that he plan to go to: × 42, Ahmedabad. (a) Had he realised how close he was to failing, he would
(b) If he were to decide to go to college, opej would not have gone to the party.
recommend that he plan to go to × 42, Ahmedabad. (b) If he would have realised how close he was to failing, he

ww
(c) Had he decided to go to college, one would recommend
that plan to go to × 42, Ahmedabad.
(d) In the event that he decides to go to college one would
would not have gone to the party.
(c) Had he had realised how close he was to failing, he
would not have gone to the party.

w.E
recommend that plan to go to × 42, Ahmedabad.
2. Except for you and I, everyone brought a present for the
little birthday girl. 6.
(d) When he realized how close he was to failing, he did not
go to the party.
The Indian cricket team’s winning it’s first game of the 2007

little birthday girl


asy
(a) Except for him and I, everyone brought a present for the

(b) With the exception of you and I, everyone brought a


World Cup excited the fans of the team.
(a) The Indian cricket team’s winning it’s first game of the
2007 World Cup excited the fans of the team.
present for the little birthday girl
En
(c) Except for you and I, everyone had brought a present for
(b) The Indian cricket team having won its first game of the
2007 World Cup, excited the fans of the team.
the little birthday girl
(d) Except for you and me, everyone brought present for the
little birthday girl gin (c) The Indian cricket team’s having won its first game of
the 2007 World Cup excited the fans of the team.
(d) The Indian cricket team’s winning its first game of the
3. When one reads the Hindi literature of the twentieth
century, you find a striking contrast between the writings of 7. eer
2007 World Cup excited the fans of the team.
Poor product quality angers Mr. Rajnish who wonders if it is
Munshi Premchand and later day writers of popular Hindi
fiction.
(a) When one reads the Hindi literature of the twentieth ing
part of a strategy by marketers.
(a) Poor product quality angers Mr Rajnish, who wonders if
it is part of a strategy by marketers.
century, you find a striking contrast between the
writings of Munshi Premchand and later day writers of .ne
(b) Poor product quality angers Mr. Rajnish, who wonders
if marketers are part of the strategy.
popular Hindi fiction.
(b) When you read the Hindi literature of the twentieth
century, one finds a striking contrast between the
writings of Munshi Premchand and later day writers of
8.
part of a strategy by marketers
t
(c) Poor product quality angers Mr. Rajnish, wonders if it is

(d) Poor product quality angers Mr. Rajnish, who wonders


if they are part of a strategy by marketers.
Having bowed our heads, the priest in the temple led us in
popular Hindi fiction.
(c) When one reads the Hindi literature of the twentieth prayer.
century, he finds a striking contrast between the writings (a) Having bowed our heads, the priest in the temple led us
of Munshi Premchand and later day writers of popular in prayer.
Hindi fiction. (b) After we bowed our heads, the priest in the temple led
(d) If one reads the Hindi literature of the twentieth century, us in prayer.
you find a striking contrast between the writings of (c) After we had bowed our heads, the priest in the temple
Munshi Premchand and later day writers of popular led us in prayer.
Hindi fiction. (d) After having bowed our head, the priest in the temple
4. Because of his tennis elbow injury, Pete Sampras has not and led us in prayer.
possibly never will be able to pick up the bat again. 9. Being a realist, the detective could not accept the statement
(a) Because of his tennis elbow injury, Pete Sampras has not of the accused that UFOs had caused the disturbance,
and possibly never will be able to pick up the bat again. (a) Being a realist, the detective could not accept the
(b) Because of his tennis elbow injury, Pete Sampras has not statement of the accused that UFOs had caused the
and possibly will never be able to pick up the bat again. disturbance.

Downloaded From : www.EasyEngineering.net


Downloaded From : www.EasyEngineering.net

Choosing the Correct Sentence  l 137

(b) Since he was a realist, the detective could not accept 15. Using it wisely, leisure promotes health, long life, efficiency,
the statement of the accused that UFOs had caused the and happiness.
disturbance. (a) Using it wisely, leisure promotes health, long life,
(c) Being that he was a realist, the detective could not accept efficiency, and happiness.
the statement of the accused that UFOs had caused the (b) If used wisely, leisure promotes health, long life,
disturbance. efficiency, and happiness.
(d) Realist that he was, the detective could not accept the (c) Having used it wisely, leisure promotes health, long life,
statement of the accused that UFOs had caused the efficiency, and happiness.
disturbance. (d) If it is used wisely, leisure promotes health, long life,
10. The shopkeeper hadn’t hardly any of those kind of goods. efficiency, and happiness.
(a) The shopkeeper hadn’t hardly any of those kind of DIRECTIONS (Qs. 16–27):  In the following questions, only one
goods. statement is grammatically correct. Identify the correct one.
(b) The shopkeeper hadn’t hardly any of those kinds of 16. (a) In the course of there journey, they were able to absolve
goods. a lot of local cultures
(c) The shopkeeper had hardly any of those kind of goods. (b) In the coarse of their journey, they were able to absorb a
(d) The shopkeeper had hardly any of those kinds of goods. lot of local culture
11. If we cooperate together by dividing up the booty, we shall

ww
be able to work together smoothly in the future.
(a) If we cooperate together by dividing up the booty, we
shall be able to work together smoothly in the future.
(c) In the course of their journey, they were able to observe
a lot of local culture’s
(d) In the course of their journey, they were able to observe

w.E
(b) If we cooperate by dividing up the booty, we shall be
able to work together smoothly in the future.
(c) If we cooperate by dividing up the booty together, we
a lot of the local culture
17. (a) Your advise does not seem to have had any effect
(b) Your advice do not seems to have had any effect
(c) Your advice do not seem to have had any effect

asy
shall be able to work together smoothly in the future.
(d) If we cooperate with each other by dividing the booty,
we shall be able to work together smoothly in the future.
(d) Your advise does not seem to have had any effects
18. (a) Ever since the sting operation, there has been much
12. The process by which the community influences the actions
of its members is known as social control En opposition from they who maintain that it had been an
unathorised act.
(b) ever since the sting operation, there has been much
(a) The process by which the community influences the
actions of its members is known social control.
(b) The process by which the community influence the
gin opposition from they who maintain that it was an
unauthorised act.
actions of its members is known as social control.
(c) The process by which the community goes about eer
(c) Ever since the sting operation, there has been much
opposition from they who maintain that it was an
unauthorised act.
influencing the actions of its members is known as
social control.
(d) The process by which the community influences the
ing
(d) Ever since the sting operation , there has been much
opposition from those maintaining that it was an

13.
actions of its members is known as social control.
Sherry, a little girl with little talent for cooking, enjoys
unauthorised act.

.ne
19. (a) ‘Are these gloves belonging to you?’ she asked.
(b) ‘Does this gloves belong to you?’ she asked.
preparing Fried Rice.
(a) Sherry, a little girl with little talent for cooking, enjoys
preparing Fried Rice.
(b) Sherry, is a little girl who has little talent for cooking but
(c) ‘Do these gloves belongs to you?’ she asked.
(d) ‘Do these gloves belong to you?’ she aksed. t
20. (a) I live in a house in a street in the countryside. The street
is called “Bear Street” and the house is old-more than
she enjoys preparing Fried Rice. 100 years old!
(c) Sherry, a little girl who has little cooking talent, enjoys (b) I live in the house in the street in the countryside. The
preparing Fried Rice. street is called “Bear Street” and the house is old-more
(d) Sherry, a girl with little talent for cooking, enjoys than 100 years old!
preparing Fried Rice. (c) I live in a house in the street in the countryside. The

14. Of all the persons I have ever met, Arjit is the most street is called “Bear Street” and the house is old - more
remarkable person. than 100 years old!
(a) Of all the persons I have ever met, Arjit is the most (d) I live in a house in a street in the countryside. The street
remarkable person. is called “Bear Street” and a house is old - more than 100
(b) Arjit is the most remarkable person of all the persons I years old!
have ever met. 21. (a) The teachers will be able to visit our schools and compare
(c) Of all the persons I have ever met, Arjit is the most our teaching methods to their own.
remarkable person. (b) The teacher will be able to pay a visit to our schools and
(d) Of all the persons I have ever met, Arjit is the most compare teaching methods for their own.
remarkable.

Downloaded From : www.EasyEngineering.net


Downloaded From : www.EasyEngineering.net

138  l  Choosing the Correct Sentence

(c) The teachers will be able to visit our schools and compare being promoted into risky and precarious leadership
our teaching methods with their own. positions where the chance of failure is high.
(d) The teachers will be able to visit our and compare their (d) Women at the top of the ladder are being promoted
teaching method with their own. for risky and precarious leadership positions where the
22. (a) Could you give me the amount that you filled out in the chance of failures is high, according to a new research
check which was sent? from the University of Exeter, England.
(b) Could you give me the amount what you filled out in the 27 . (a) She was one of the first authors to point at the importance
check you sent? of women’s subsistence activities particularly in rural
(c) Could you give me the amount for which you filled out areas in predominantly agricultural countries, and the
in the check you sent? underestimation of such activities in the conventional
(d) Could you give me the amount wherein you filled out in method of national income accounting.
the check you sent? (b) She was one of the first authors to point out the
23. (a) I have completed the work yesterday. importance of women’s subsistence activities
(b) I did completed the work yesterday. particularly in rural areas in predominantly agricultural
(c) I have had completed the work yesterday. countries, and such underestimation of activities in the
(d) I completed the work yesterday. conventional method of national income accounting.
24. (a) For the first time in our history, the new millionaires are (c) She was one of the first authors to point at the importance

ww
looked up with pride and even reverence for they are a
new meritocracy who are creating value by innovating
in the global knowledge economy.
of women’s subsistence activities, particularly in rural
areas in predominantly agricultural countries, and
such underestimation of activities in the conventional

w.E
(b) For the first time in our history, the new millionaires are
looked on with pride and even reverence for they are a
new meritocracy who are creating values by innovating
method of national income accounting.
(d) She was one of the first authors to point out the
importance of women’s subsistence activities in
in the global knowledge economy.
asy
(c) For the first time in our history, the new millionaires are
looked up to with pride and even reverence for they are
predominantly agricultural countries, and the
underestimation of such activities in the conventional
methods of national income accounting.

En
a new meritocracy who are creating value by innovating
in the global knowledge economy.
DIRECTIONS (Qs. 28–37): In the following questions, four
different ways of writing a sentence are indicated. Choose the best
(d) For the first time in our history, the new millionaires are
looked up with pride and even reverence for they are a
new meritocracy who are creating values by innovating
gin
way of writing the sentence.
28 . A.  The main problem with the notion of price

25.
in the global knowledge economy.
(a) Business is changing so rapidly that those who can eer
discrimination is that it is not always a bad thing, but
that it is the monopolist who has the power to decide
who is charged what price.
foresee the changing needs of the market will not
only become rich but also create millions of jobs and
transform our poor hierarchical society.
ing
B. The main problem with the notion of price discrimination
is not that it is always a bad thing, it is the monopolist
who has the power to decide who is charged what price.
(b) Business is changing so rapidly that not only those
who can foresee the changing needs of the market .ne
C. The main problem with the notion of price
discrimination is not that it is always a bad thing, but
will become rich but also create millions of jobs and
transform our poor, hierarchical society.
(c) Business is changing so rapidly that those who can
foresee not only the changing needs of the market
who is charged what price.
D. The main problem with the notion of price t
that it is the monopolist who has the power to decide

discrimination is not it is always a bad thing, but that


will become rich but also create million of jobs and it is the monopolist who has the power to decide who is
transform our poor, hierarchical society. charged what price.
(d) Not only is business changing rapidly that those who can (a) A (b) B
foresee the changing needs of the market will became (c) C (d) D
rich but also create millions of jobs and transform our 29 . A.  A symbiotic relationship develops among the
poor, hierarchical society. contractors, bureaucracy and the politicians, and by a
26. (a) According to a new research from the University of large number of devices costs are artificially escalated
Exeter, England, women being on top of the ladder are and black money is generated by underhand deals.
promoted into risky and precarious leadership positions B. A symbiotic relationship develops among contractors,
where the chances of failures is high. bureaucracy and politicians, and costs are artificially
(b) According to new research from the University of escalated with a large number of devices and black
Exeter, England, women are being promoted at the money is generated through underhand deals.
top of the ladder into risky and precarious leadership C. A symbiotic relationship develops among contractors,
positions where the chance of failure is high. bureaucracy and the politicians, and by a large number
(c) According to a new research from the University of of devices costs are artificially escalated and black
Exeter, England, women at the top of the ladder are money is generated on underhand deals.

Downloaded From : www.EasyEngineering.net


Downloaded From : www.EasyEngineering.net

Choosing the Correct Sentence  l 139

D. A symbiotic relationship develops among the about the body and its products, found in the sixteenth
contractors, bureaucracy and politicians, and by large century.
number of devices costs are artificially escalated and C. From the sixteenth century onwards, a growing disdain
black money is generated by underhand deals. for and self-consciousness about the body and its
(a) A (b) B products took hold, leading to a heightened focus on
(c) C (d) D emotional and bodily regulation.
30 . A. The distinctive feature of tariffs and export subsidies D. The heightened focus on emotional and bodily
is that they create difference of prices at which goods regulations started from the sixteenth century onwards,
are traded on the world market and their price within a when people felt disdain and self-consciousness about
local market. the body and its products.
B. The distinctive feature of tariffs and export subsidies (a) D (b) C
is that they create a difference of prices at which goods (c) B (d) A
are traded with the world market and their prices in the 34 . A. As I sat on the park bench enjoying the cool night air
local market. and gazing at the people who were taking a break from
C. The distinctive feature of tariffs and export subsidies the tensions of the world, two young lovers passed by
is that they create a difference between prices at which me.
goods are traded on the world market and their prices B. As I sat on the park bench and was enjoying the cool

ww
within a local market.
D. The distinctive feature of tariffs and export subsidies is
that they create a difference across prices at which goods
night air and gazing at the people who were took a break
from the tensions of the world, two young lovers passed
by me.

(a) A
w.E
are traded with the world market and their prices within
a local market.
(b) B
C. As I was sitting on the park bench and was enjoying
the cool night air and was gazing at the people who
were taking a break from the tensions of the world, two
(c) C
asy
(d) D
31 . A. Any action of government to reduce the systemic risk
inherent in financial markets will also reduce the risks
young lovers passed me by.
D. As I sat on the park bench enjoying the cool night air
and had been gazing at the people who were taking a

excessive hedging. En
that private operators perceive and thereby encourage break from the tensions of the world, two young lovers
passed me by.
B. Any action by government to reduce the systemic risk
inherent in financial markets will also reduce the risks
that private operators perceive and thereby encourage
gin (a) A
(c) C
(b) B
(d) D
35 . A. Where the problems arises is when we decide that we
excessive gambling.
C. Any action by government to reduce the systemic eer
already know what it takes to do a job.
B. Where the problems arise is when we have decided we
risk inherent due to financial markets will also reduce
the risks that private operators perceive and thereby
encourages excessive hedging.
ing
already know what it takes to do a job.
C. Where the problems arise is when we decide that we
already know what it takes to do a job.
D. Any action of government to reduce the systemic risk
inherent in financial markets will also reduce the risks .ne
D. Where the problems arise is when we decide that we
have already known what it takes to do a job.
that private operators perceive and thereby encourages
excessive gambling.
(a) A
(c) C
(b) B
(d) D
(a) A
(c) C
(b) B
(d) D
t
36 . A. Burger King is planning to expand their product baskets
by including low-calorie diet for health conscious
32 . A. Creativity in any field is regarded not only as valuable consumers, faced with a growing backlash against
for itself but also as a service to the nation. processed food products.
B. Creativity in any field is not regarded only as valuable B. Faced with a growing backlash against processed
on its own, but also as a service to the nation. food products in the West, Burger King is including
C. Creativity, in any field, is not only regarded as valuable, low-calorie diet for health conscious consumers and
but also as a service to the nation. planning to expand its product basket.
D. Creativity in any field is regarded not only as valuable in C. Including low-calorie diet for health conscious
itself but also as a service to the nation. consumers and faced with a growing backlash against
(a) D (b) C processed food products in the West, Burger King is
(c) A (d) B planning to expand his product basket.
33 . A. From the sixteenth century onwards, people started D. Faced with a growing backlash against processed food
feeling disdainful and self-conscious about their body products in the West, Burger King is planning to expand
and its products that led to a heightened focus on its product basket by including low-calorie diet for
emotional and bodily regulations. health conscious consumers.
B. The heightened focus on controlling the body and (a) A (b) B
emotions comes from disdain and self-consciousness (c) C (d) D

Downloaded From : www.EasyEngineering.net


Downloaded From : www.EasyEngineering.net

140  l  Choosing the Correct Sentence

37 . A. The Union Budget is aiming to provide access to masses 44 . I wanted to know who was in the interview panel but the
and encourage private participation to set up or expand interview coordinator told me that you can not know their
existing facilities. names beforehand.
B. The Union Budget is aiming at providing access to 45 . They asked me about eve-teasing and it’s consequences in our
masses and encouraging private participation for setting society. I said it promoted many a problems in our society.
or expanding existing facilities. 46 . Then they told me to take the name a corporate personality
C. The Union Budget aims to provide access to masses who is engaged in many charities and I said Narayan Murthy,
and encourage private participation to setting up or but they said, “Your answer is wide off the mark”.
expanding existing facilities. DIRECTIONS (Qs. 47–49): Each question consists of four
D. The Union Budget aims at provision of access to masses sentences. Some sentences are grammatically incorrect or
and encourage private participation to set up or expand inappropriate. Select the option that indicates these grammatically
existing facilities. incorrect and inappropriate sentence(s).
(a) A (b) B 47 . A. Harish told Raj to pled guilty.
(c) C (d) D B. Raj pleaded guilty of stealing money from the shop.
DIRECTIONS (Qs. 38–46):  Read the following sentences and C. The court found Raj guilty of all the crimes he was
identify errors. Check grammar, usage, semantics, mechanics, charged with.
redundancy, punctuation, spelling and style elements. D. He was sentenced for three years in jail.

ww
Mark (a) if the sentence has one error.
Mark (b) if the sentence has two errors.
Mark (c) if the sentence has three errors.

(a) A and B
(c) A, C and D
(b) B and D
(d) B, C and D
48 . A. A changing perception as to India has also helped.

w.E
Mark (d) if the sentence has four or more errors.
38 . We were preparing for SSC last year from the GT as GT is
one of the best institute for SSC Entrance preparation.
B. India is no longer a backward, third world country.
C. Helping boost the image are favourable opinions of
leading international financial institutions.

asy
39 . GT material goes under continuous upgradation every year.
40 . GT is very strict about its schedule. It will never postpone or
prepone any class.
D. India will be the third biggest economy by 2050, just
behind China and the US, in that order.
(a) A and D (b) A only
41 . GT has many good faculties who are very good in their
respective subjects and you can go to them anytime and En (c) B only (d) No error
49 . A. I remember seeing a full page advertisement with a
bright background colour.
discuss about your problems. Once the English faculty
discussed about an idiom and asked us to translate it word gin B. The only other item on the page was a very small line of
type in a lighter shade of the same colour.
to word.
42 . Whenever there is a new Mathematics question, usually I
am knowing the answer but I am simply forgetting it. eer
C. Since I can still recall the ad, it must made an impression.
D. Using colour to impact your design works best when it
involves the unexpected or extreme.
43. Once I met a brilliant student in GT and I asked, “What is
your good name?”, and he replied, Myself Udayan Sarkar. I
(a) C only
(c) D only ing (b) B only
(d) No error
thought when can I get so many marks as him.

LEVEL-II .ne
DIRECTIONS (Qs. 1–7):  In the following questions, a sentence
is written in four different ways. Choose the option which gives the
most effective and grammatically correct sentence. Pay attention to
continually diminish.
2. (a) He must again learn to invoke the energy of growing
t
world and it was hoped that tyranny and injustice would

grammar, word choice and sentence construction. things and to recognise, that one can be taking from the
1. (a)  It was thought that freedom and prosperity would earth and the atmosphere only so much as one puts back
spread gradually throughout the world through an into them, as did the ancient in India centuries ago.
orderly process, and it was hoped that tyranny and (b) As did the ancient in India centuries ago, he must
again learn to invoke the energy of growing things and
injustice would continually diminish.
to recognize that one can take from the earth and the
(b) It was gradually thought that throughout the world,
atmosphere, only so much as they put into them.
freedom and prosperity would spread through an
(c) He must again learn to invoke the energy of growing
orderly process, and it was hoped that tyranny and things and to recognize, as did the ancient in India
injustice would continually diminish. centuries ago, that one can take from the earth and the
(c) Through an orderly process, it was thought that freedom atmosphere, only so much as one puts back into them.
and prosperity would spread gradually throughout the (d) He must again learn, as did the ancient in India
world, and it was hoped that tyranny and injustice centuries ago, to invoke the energy of growing things
would continually diminish. and to recognize, that one can be taking from the earth
(d) It was thought, through an orderly process that freedom and the atmosphere, only so much as one puts back into
and prosperity would spread gradually throughout the them.

Downloaded From : www.EasyEngineering.net


Downloaded From : www.EasyEngineering.net

Choosing the Correct Sentence  l 141

3. (a) Seating arrangements influenced the kind of interaction (a) Unless you submit an application along with a copy of
that takes place your ration card you will not get a duplicate licence.
(b) seating arrangements’s influence the kind of interaction (b) You should require a duplicate license if you submit an
that takes place application along with a copy of your ration card.
(c) Seating arrangements influence the kind of interactions (c) If you submit your application along with your ration
that take place card you do not need duplicate license.
(d) Seating arrangements influence the kind of interaction (d) If you submit an application along with your ration card
that take place you will get only a license.
4. (a) Any organisation, large or small, trades with many 9. The Manager would like you to help him locate the default.
different people and companies (a) If you help him locate the default, the Manager would
(b) Any organization, large or small, trade with many like you.
different peoples and companies (b) The Manager desires that you should provide him the
(c) Any organization, large or small, trade with many necessary assistance to locate the default.
different people and company (c) The Manager feels that if you do not help him the fault
(d) Any organisation, large or small, trades with many will not be located.
different people and companies (d) The Manager expects that the default should be located

ww
5. The trend toward a decrease in the working hours is already
evident in the longer weekend given to employees in many
multinational organisations.
only with your help.
(e) None of these
10. Ritesh noticed the cat’s soft hair, sharp paws, and keen sense

w.E
(a) The trend toward a decrease in the working hours is all
ready evident in the longer weekend given to employees
in many multinational organisations.
of hearing.
(a) Ritesh noticed the cat’s soft hair, sharp paws, and keen
sense of hearing.

asy
(b) The trend toward a decrease in the working hours
is already evident in the longer weekend given to
employees in many multinational organisations.
(b) Ritesh noticed the cat’s soft hair, sharp paws, and that
his sense of hearing was keen.

(c) The trend toward decrease in the working hours


is already evident in the longer weekend given toEn (c) Ritesh noticed the cat’s soft hair, that he had sharp paws,
and a very keen sense of hearing.
(d) Ritesh noticed the cat’s soft hair, keen sense of hearing
employees in many multinational organisation.
(d) The trend toward a decrease in the working hours is gin and also that it had sharp paws.
11. The principal seldom ever wants to try and face the facts.
all in already evident in the longer weekend given to
employees in many multinational organisations.
6. They watched. They wondered. They were unable to find the eer
(a) The principal seldom ever wants to try and face the
facts.
reason.
(a) They watched and wondered till they were unable to
facts.
ing
(b) The principal seldom if ever wants to try and face the

(c) The principal seldom ever wants to try to face the facts.
find the reason.
(b) They watched and wondered but were unable to find the
reason. .ne
(d) The principal seldom wants to the to face the facts.
12. Anyone interested in flying planes can learn much if you
(c) They had watched and wondered but were unable to
find the reason.
(d) They watched and wondered despite being unable to
find the reason.
have access to a flight simulation machine.

have access to a flight simulation machine. t


(a) Anyone interested in flying planes can learn much if you

(b) Anyone interested in flying planes can learn much if he


7. He is sure to receive his pay. It is due to him. Why then does has access to a flight simulation machine.
he worry? (c) Anyone interested in flying planes can learn much if
(a) Why does he worry, till he is sure to receive his pay due access is available to a flight simulation machine.
to him? (d) Anyone interested in flying planes can learn much from
(b) Why should he worry as the pay due to him is sure to be access to a flight simulation machine.
received? 13. The reason I came late to office today is because my car
(c) Why does he worry as he should be sure to receive the broke down.
pay due to him? (a) The reason I came late to office today is because my car
(d) None of the above sentences is correct. broke down.
(b) Why I came late to office today is because my car broke
DIRECTIONS (Qs. 8–25):  For each statement there are four
different sentences given below it. Pick out the one that most down.
appropriately conveys the meaning of the statement. (c) The reason I was late to office today is because my car
broke down.
8. Should you need a duplicate licence you must submit an (d) The reason I came late to office today is that my car
application along with a copy of your ration card. broke down.

Downloaded From : www.EasyEngineering.net


Downloaded From : www.EasyEngineering.net

142  l  Choosing the Correct Sentence

14. The entire cast and crew of the film, enjoyed splashing in the 19 . The twelve-hour work day not only has been reduced to one
pool, bathing in the ocean, and, particularly, to sun bathe on of ten hours but also, in some lines of work, to below eight
the shore. hours.
(a) The entire cast and crew of the film, enjoyed splashing (a) The twelve-hour work day not only has been reduced
in the pool, bathing in the ocean, and, particularly, to to one of ten hours but also, in some lines of work, to
sun bathe on the shore. below eight hours.
(b) The entire cast and crew of the film, enjoyed splashing (b) Not only has the twelve-hour work day been reduced
in the pool, to have a bath in the ocean, and, particularly to one of ten hours but also, in some lines of work, to
to sun bathe on the shore. below eight hours.
(c) The entire cast and crew of the film, enjoyed swimming (c) The twelve-hour work day has not only been reduced
in the pool to bathe in the ocean, and, particularly sun to one of ten hours but also, in some lines of work, to
bathing on the shore. below eight hours.
(d) The entire cast and crew of the film, enjoyed swimming (d) The twelve-hour work day has been reduced not only
in the pool, bathing in the ocean, and, particularly, sun to one of ten hours but also, in some lines of work, to
bathing on the shore. below eight hours.
15. Crossing the street, a bus almost crushed us to death. 20 . If some Indians look at where they are going, it can be seen

ww
(a) Crossing the street, a bus almost crushed us to death.
(b) A bus almost crushed us, crossing the street.
(c) As we crossed the street, a bus almost crushed us.
that our goal is money.
(a) If some Indians look at where they are going, it can be
seen that our goal is money.

w.E
(d) A bus, crossing the street, almost crushed us.
16. The moral of the entire story is how money doesn’t make
you happy.
(b) If some Indians look back to where they are going, it can
be seen that our goal is money.
(c) If some Indians look at where they are going, it can be

you happy.
asy
(a) The moral of the entire story is how money doesn’t make

(b) The moral of the entire story is that money doesn’t make
seen that their goal is money.
(d) If some Indians look at where they are going, they can
see that their goal is money.
you happy.
En
(c) In this novel, its moral of the story is how money doesn’t
21 . Unless they reverse present policies immediately, the world
may suffer irreversible damage from the unregulated use of
make you happy.
(d) That money does not make you happy, is the entire
moral of the story. gin Bio weapons.
(a) Unless they reverse present policies immediately,
the world may suffer irreversible damage from the
17. When one travels by Air Sitara you often find that the prices
are high and that the journey experience is extremely poor. eer
unregulated use of Bio weapons.
(b) Unless present policies are reversed immediately,
(a) When one travels by Air Sitara you often find that
the prices are high and that the journey experience is
extremely poor. ing
the world may suffer irreversible damage from the
unregulated use of Bio weapons.
(c) Unless present policies are reversed, the world may
(b) When you travel by Air Sitara one often finds that
the prices are high and that the journey experience is .ne
suffer irreversible damage by the unregulated use of Bio
weapons.
extremely poor.
(c) As you travel by Air Sitara you often find that the prices
are high and that the journey experience is extremely
poor.
t
(d) Unless present policies are reversed, the world may
suffer irreversible damage through the unregulated use
of Bio weapons.
22 . If she were to win the Olympic medal, I for one would be
(d) If you travel by Air Sitara you often find that the prices surprised.
are high and that the journey experience is extremely (a) If she were to win the Olympic medal, I for one would
poor. be surprised.
18. Depending on skillful suggestion, argument is seldom used (b) If she were to go on to win the Olympic medal, I for one
in advertising. would be surprised.
(a) Depending on skillful suggestion, argument is seldom (c) If she becomes the winner of the Olympic medal, I for
used in advertising. one would be surprised.
(b) Argument is seldom used by advertisers, who depend (d) In the event that she would win the Olympic medal, I for
on skillful suggestion instead. one would be surprised.
(c) Skillfull suggestion is depended on by advertisers 23 . The soldiers were told to take a long arduous hike, pitch
instead of argumentation. their camps, have dinner, and that they should be in bed by
(d) Suggestion, which is skillful, is used in place of 10 p.m.
argumentation by advertisers. (a) The soldiers were told to take a long arduous hike, pitch
(e) Suggesting skillfully is what advertisers do apart from their camps, have dinner, and that they should be in bed
argumentation. by 10 p.m.

Downloaded From : www.EasyEngineering.net


Downloaded From : www.EasyEngineering.net

Choosing the Correct Sentence  l 143

(b) The soldiers were told to take an arduous hike, pitch (d) He must again learn, as did the ancient in India
their camps, have dinner, and that they should be in bed centuries ago, to invoke the energy of growing things
by 10 p.m. and to recognize, that one can be taking from the earth
(c) The soldiers were told to take a long arduous hike, pitch and the atmosphere, only so much as one puts back into
camp, have dinner, and be in bed by 10 p.m. them.
(d) The soldiers were told to take a long arduous hike, 28 . (a) He made a blunder mistake.
pitching their camps, have dinner, and be in bed by 10 (b) I have learnt this lesson word by word.
p.m. (c) She does not know swimming.
24 . Not only was he efficient but also welcoming in nature. (d) He got his daughter married.
(a) He not only was competent but also friendly in nature. 29 . (a) The average male investor expects to see all his
(b) Not only was he competent but also friendly in nature. investment stocks moving up, and they are often
(c) He not only was competent but friendly too in nature. disappointed.
(d) He was not only competent but also friendly in nature. (b) The people in my country are smaller than in other
25 . With the exception of Dipanjan and I, everyone in the class countries
finished the assignment before the teacher came. (c) The number of failures of product in final quality
(a) With the exception of Dipanjan and I, everyone in the
inspection are increasing every year.
class finished the assignment before the teacher came.

ww
(b) With the exception of Dipanjan and me, every-1 one
in the class finished the assignment before the teacher
30
(d) Before restructuring a firm, the CEO must consider the

. (a) 
employees.
Pele, whom many people consider is the greatest

w.E
came.
(c) With the exception of Dipanjan and me, eveiy-1 one in
the class had finished the assignment before the teacher
came.
footballer of all times, represented Brazil in four World
Cups.
(b) When the news about the decline in the quarterly sales

asy
(d) With the exception of Dipanjan and I, everyona in the
class had finished the assignment type the bell rang.
of the product broke out, it was difficult to say who the
company would hold responsible for this disaster.
(c) When the tension with the business partners increased,
DIRECTIONS (Qs. 26–34): In each question, a sentence is
En
written in four different ways. Choose the option which gives the Mr. Singh decided to visit them personally and talk to
whomever is willing to sort the discord
most effective and grammatically correct sentence. Pay attention to
grammar, word choice and sentence construction.
26 . (a)  It was thought that freedom and prosperity would gin (d) Who do you think was supposed to meet Mr. Brown
from the news bureau, the well-known author, during
spread gradually throughout the word through an
orderly process, and it was hoped that tyranny and eer
his week-long visit to Delhi?
31 . (a) Returning home from the dinner, we were annoyed to
find that the porch light is broken again.
injustice would continually diminish.
(b) It was gradually thought that throughout the world,
freedom and prosperity would spread through an ing
(b) If you keep on losing your composure on minor issues,
that isn’t going to get you no where
orderly process, and it was hoped that tyranny and
injustice would continually diminish.
.ne
(c) In the meeting, Mr. Mehta informed the executives that
they have to fulfill the target regardless of the stringency
(c) Through an orderly process, it was thought that freedom
and prosperity would spread gradually throughout
the world, and it was hoped that tyranny and injustice
would continually diminish.
of the deadline

t
(d) The call for the assembly was very disappointing, as we
haven’t hardly initiated our discussion on a new venture
outside the main conference bloc.
(d) It was thought, through an orderly process that freedom 32 . (a)  The main problem with the notion of price
and prosperity would spread gradually throughout the discrimination is that it is not always a bad thing, but
world and it was hoped that tyranny and injustice would
that it is the monopolist who has the power to decide
continually diminish.
who is charged what price
27 . (a) He must again learn to invoke the energy of growing
(b) The main problem with the notion of price discrimination
things and to recognize, that one can be taking from the
is not that it is always a bad thing, it is the monopolist
earth and the atmosphere only so much as one puts back
into them, as did the ancient in India centuries ago. who has the power to decide who is charged what price.
(b) As did the ancient in India centuries ago, he must (c) The main problem with the notion of price
again learn to invoke the energy of growing things and discrimination is not that it is always a bad thing, but
to recognize that one can take from the earth and the that it is the monopolist who has the power to decide
atmosphere, only so much as they put into them. who is charged what price.
(c) He must again learn to invoke the energy of growing (d) The main problem with the notion of price
things and to recognize, as did the ancient in India discrimination is not it is always a bad thing, but that
centuries ago, that one can take from the earth and the it is the monopolist who has the power to decide who is
atmosphere, only so much as one puts back into them. charged what price.

Downloaded From : www.EasyEngineering.net


Downloaded From : www.EasyEngineering.net

144  l  Choosing the Correct Sentence

33 . (a)  A symbiotic relationship develops among the (a) B (b) C


contractors, bureaucracy and the politicians, and by a (c) A and D (d) None of these
large number of devices costs are artificially escalated 37. A. Mary returned the dress to the store, which was torn at
and black money is generated by underhand deals one of the seams.
(b) A symbiotic relationship develops among contractors, B. We enjoy meeting people having different interests.
bureaucracy and politicians, and costs are artificially C. Obstinate and surly, the manager’s attitude alienated his
escalated with a large number of devices and black employees.
money is generated through underhand deals. D. As with other children in her neighborhood who were
(c) A symbiotic relationship develops among contractors,
home-schooled, Joan sometimes missed being in a
bureaucracy and the politicians, and by a large number
classroom with her peers.
of devices costs are artificially escalated and black
(a) B (b) C
money is generated on underhand deals.
(c) A and D (d) None of these
(d) A symbiotic relationship develops among the
contractors, bureaucracy and politicians, and by large 38 . A. In contrast to the trapeze artists who fumbled their
number of devices costs are artificially escalated and routine, the antics of the circus clowns kept the audience
black money is generated by underhand deals. entertained for hours.

ww
34 . (a) The distinctive feature of tariffs and export subsidies
is that they create difference of prices at which goods
are traded on the world market and their price within a
B. The clothes hanging on the racks inside the store looked
more appealing than in the store window.
C. There are about the equivalent number of gymnasium

w.E
local market
(b) The distinctive feature of tariffs and export subsidies
is that they create a difference of prices at which goods
members in the boxing class as in the aerobics class.
D. Brett decided to use his own money, but not his parents’
credit card, to pay for the stereo.

local market.
asy
are traded with the world market and their prices in the

(c) The distinctive feature of tariffs and export subsidies



(a) B
(c) A and D
(b) C
(d) None of these

En
is that they create a difference between prices at which
goods are traded on the world market and their prices
39 . A. Julia was able to climb the tree so fast as her brothers.
B. The blue dress looks more flattering on you than the red
one.
within a local market.
(d) The distinctive feature of tariffs and export subsidies is gin C. Three times more students attended the prom this year
than last year.
that they create a difference across prices at which goods
are traded with the world market and their prices within
a local market. eer
D. Joe went to bed early because his will to succeed in the
race the following morning was greater than playing

DIRECTIONS (Qs. 35–44): Identify the correct sentence or


sentences. Check grammar, usage, semantics, mechanics, redundancy,
(a) B
ing
pool with his friends.

(c) A and D
(b) C
(d) None of these
punctuation, spelling and style elements.
35 . A. Upon entering the restaurant, the maître d’hôtel handed .ne
40 . A. Sam was away on vacation longer than his friends.
B. Most of the audience did not enjoy the concert, likening
us a menu.
B. Working diligently and carefully, the faucet was fixed in
no time at all.
C. Employing ground breaking techniques in alternative
it to grinding up metal.
t
C. Owning a car is still Dan’s goal, like that of his parents
when they were his age.
D. Covering the floors with tiles costs twice as much as
medicine, the patient’s health improved in a few days.
linoleum.
D. David tried a handful of desserts from the table, which
ultimately gave him an upset stomach. (a) B (b) C
(a) B (b) C (c) A and D (d) None of these
(c) A and D (d) None of these 41 . A. Like a woman I once met on the bus, the hostess’ attire
36 . A. The quick-witted hostess pitched a tent over the garden was somewhat flamboyant.
party, a way to protect the guests from the imminent B. The tycoon contributed more to the candidate’s
rain. campaign than anyone else in the industry.
B. Similar in so many ways, the parents were still able to C. Unaccustomed to being spontaneous, Jill couldn’t
distinguish between the twins. decide whether she should be spending her bonus on a
C. Based on the recent decline in enrollment, the new computer.
admissions office decided to reevaluate its recruitment D. The new mother spent most of the day worrying over
strategies. her son’s safety.
D. Unaccustomed to the rigors of college life, James’s (a) B (b) C
grades dropped. (c) A and D (d) None of these

Downloaded From : www.EasyEngineering.net


Downloaded From : www.EasyEngineering.net

Choosing the Correct Sentence  l 145

42 . A. Most of us are not aware that eating some varieties of 46 . A. As the growing economy makes increasing demands on
mushrooms result in death. infrastructure inputs, these problems could worsen in
B. The interference with nature often brings disaster; thus the coming year.
tree-felling sometimes turns fertile land into a dust B. Therefore, addressing infrastructure gaps needs to doing
bowl. our topmost priority next year.
C. That eminent professor, who discovered the ancient C. The second risk lies in the global macroeconomic
cure that everyone is talking about, refused to give a imbalances, reflected in the twin deficits of the US and
press-interview. rising surpluses of Asia.
D. Merely speeches made from all sorts of public platform D. The longer these imbalances have persisted, the greater
cannot bring about an economic revolution. has become the risk of a disruptive correction.
(a) A (b) B (a) A only (b) B only
(c) C (d) D (c) D only (d) C only
43 . A. Led by religious fervor and misplaced enthusiasm to 47 . A. The only problem is that the wickets that are on offer for
expose the hypocrisy of the government, if it anyhow domestic cricket are hardly conducive for batsman.
could, the opposition was most outspoken. B. It is here that BCCI needs to show the same will, as they
B. He hesitated to accept the offer, as he didn’t think the have shown in making players play domestic cricket.

wwsalary would be enough for a man with a family of six.


C. I remember having read somewhere that the first
satellite was launched by Russians.
C. For years now, pitches have been prepared to suit the
home team’s strengths and that is fine so long as the
pitch is not a lottery.

w.E
D. The tallest among them expressed a keen desire to sit
under the shade of a tree.
(a) A (b) B
D. If one goes by the scores in some of the matches,
especially Delhi’s games, then it is quite obvious that the
pitches prepared are sub-standard and not conducive to

44
(c) C
. A. 
(d) D
asy
Her memory played her false and she could not
a fair contest between bat and ball.
(a) A only
(c) D only
(b) B only
(d) C only
remember who had invented electricity.
En
B. Except Sudha and possibly our mother, all others have
DIRECTIONS (Qs. 48 & 49) :  In each question, there are five
sentences or parts of sentences that form a paragraph. Identify the
agreed to come back soon.
C. He did not know how to do the work in time.
D. I am wondering why are you tinkering with the wires;
gin
sentence(s) or part(s) of sentence(s) that is/are correct in terms of
grammar and usage. Then, choose the most appropriate option.

you might get a shock.


(a) A (b) B eer
48. A. So once an economy is actually in recession,
B. the authorities can, in principle, move the economy.
C. out of slump - assuming hypothetically.
(c) C (d) D
DIRECTIONS (Qs. 45-47) :  In each of the following questions, ing
D. that they know how to - by a temporary stimuli.
E. In the longer term, however, such policies have no affect
one or more of the sentences is/are incorrect. Identify the incorrect
sentence(s).
45 . A. In the long history of the world, only a few generations
(a) A, B & E
(c) C & D
.ne
on the overall behaviour of the economy.
(b) B, C & E
(d) A & B
have been granted the role of defending freedom in its
hour of maximum danger.
B. I do not believe that any of us would exchange places
with any other people or any other generation.
49 . A. It is sometimes told that democratic.
B. government originated in the city-states. t
C. of ancient Greece. Democratic ideals have been handed
to us from that time.
C. The energy, the faith, the devotion which we bring to D. In truth, however, this is an unhelpful assertion.
this endeavor will light our country and all who serve it. E. The Greeks gave us the word, hence did not provide us
D. The glow from that fire can truly light the world. with a model.
(a) B only (b) A (a) A, B & D (b) B, C & D
(c) C only (d) No error (c) B & D (d) B only

Downloaded From : www.EasyEngineering.net


Downloaded From : www.EasyEngineering.net

146  l  Choosing the Correct Sentence

Hints & Solutions


LEVEL- I 22. (a) Option (b) and (d) puts a question in question and it seems
that amount of cheque is not known to sender as well.
1. (b) is the best way of expressing the idea. In option (c), use of the 'which' is not correct for amount of
2. (d) The correct answer corrects the incorrect use of I in the payment.
other sentences. Option (a) is grammatically correct.
3. (c) The improper use of the pronouns one and you is corrected
23. (d) The work was done previous day. So, simple past tense
in Choice (c).
should be used as used for something done in past. So option
4. (d) The omission of the past participle been is corrected in
Choice (d). (d) is correct. Other options are incorrect.
5. (a) There is no error in the original sentence. 24. (c) The phrase ‘look up’ means to become better (e.g., the
6. (d) The correct pronoun is its. economy in looking up) whereas ‘look up to’ means admire
7. No error. Poor product quality is singular, so the singular noun or respect and ‘look on’ means watch. Since the sentence says
‘it’ must also be correct. ‘pride and even reverence’, ‘look up to’ is the right phrase.

ww
8. (d) Having bowed our heads is a dangling modifier. Option (d)
is the best way of rephrasing it.
9. (a) There is no error in the original sentence.
Further ‘value’ means how much something is worth whereas
‘values’ means beliefs about what is right or wrong and what
is important in life. In the context of ‘innovating in the global

w.E
10. (d) This corrects the double negative (hadn’t hardly) and also
uses those with kinds correctly.
11. (d) Both together and up are unnecessary since their2meaning
is included in the words cooperate and divide.

knowledge economy’ the right word is ‘value’ not ‘values’.
25. (a) In this question the position of ‘not only’ has been
changed. The correlative conjunction ‘not only ... but also’ is

asy
12. (b) This question tests the agreement between subject and verb
and pronoun and antecedent are both involved. Community
(singular) needs a singular verb, influences. Also, the pronoun
placed immediately before the two things joined. Here the two
things are ‘becoming rich’ and ‘creating millions of jobs.’ Choice
(a) alone conveys this idea. Choice (b) leads us to believe not

is best. En
which refers to community should be singular (its). Choice (b) only those ... but also some others. Choice (c) implies different
aspects of the market and choice (d) implies something else is
13. (a) No error. Option (d) is also correct grammatically, but it
changes the message.
14. (d) The other options repeat the word ‘person’ unnecessarily. gin changing along with business.
26 . (c) In the second part of the sentence, since the verb ‘is’ is
15. (b) One way of correcting a dangling participle is to change
the participial phrase to a clause. Choices (b) and (d) substitute eer
the subject is ‘the chance of failure not the chances of failures’
(rules not a and d). The word ‘promoted’ is followed by ‘to’ or
‘into’ not ‘at’ (rules at b)
clauses for the phrase. However, choice (d) changes the meaning
of the sentence.
16. (d) Option (a) is clearly wrong (their, rather there);
27
ing
. (d) The phrase ‘point out’ means to mention something
in order to make others notice whereas ‘point at’ is to point
(b) Course; (c) Cultures;
17. (d) Advise is verb and advice is noun., so (a) and (d) are ruled
out. (c) is ruled out because of ‘do not seem’. .ne
with your finger. The former meaning is intended in the
context. Similarly, ‘such underestimation’ is wrong because no
underestimation has been mentioned.
18. (c) Replace ‘they’ with ‘those’.
19. (d) Option (a) is not right grammatically as use of 'belonging'
keeps sentence incomplete without a proper helping verb.
In option (b) 'does' with 'you' is not right.
option will cause failure. t
28 . (c) is correct because it rightly states that exercising any other

29 . (a) is the best option and can’t be changed with any other
options given there as it explains fully the problem of the
In option (c), 'belongs' for 'gloves' is not right. As gloves takes
plural verb. narrator.
In option (d), 'do' with 'you' and 'belong' for 'gloves' is 30 . (a) is correct answer. It most suitably answers the question of
grammatically right. the author.
20. (a) In option (b) article 'the' before 'house' and 'street' is not 31 . (b) is the answer as realistic details combined with romantic
right. As it does not make the house a specific object associated temperament can makeup a good novel.
with 'I'. 32 . (a) is the best option as none of the others discusses any
In option (c) again article 'the' is wrongly placed. involvement of welfare aid.
In option (d) 'a' used before 'house' introduces new house 33 . (a) is the correct answer. It is a polite and courteous way of
which is not related to person in sentence but second part of
saying that he should transfer some of the control.
sentence only tells the quality of the house mentioned earlier.
Option (a) is correct. 34 . (a) The phrase ‘pass somebody by’ means to happen without
affecting somebody (e.g.. Life passed me by.) In this context it
21. (c) In option (b), use of 'pay a visit' for teachers is not right.
should be ‘pass by me’ which means physically they went by my
In option (a), 'compare with' is better than 'compare to' for
side (rules out C and D). In II the tense is not correctly used.
methods. 'Compare to' is used for persons. The present continuous (was enjoying) cannot go with the past
Option (d) does an illogical comparison of same methods of (took a break).
teachings of visiting teachers.

Downloaded From : www.EasyEngineering.net


Downloaded From : www.EasyEngineering.net

Choosing the Correct Sentence  l 147

35 . (c) In a ‘takes into doing’ is awkward construction. In B ‘have 10. (a) No error. The phrases are all parallel: soft hair sharp paws,
decided’ and ‘already known’ D are incorrect. and keen sense of hearing
36 . (d) In A the subject and verb disagree – while ‘king is singular 11. (d) Option (d) is the best way of phrasing the message.
‘their ‘ is plural. In B the meaning is totally distorted. In C both 12. (b) This corrects the unnecessary switch in the pronouns,
‘semantics’ and parallelism’ (including faced) are wrong. anyone you.
37 . (b) The correct idiomatic expression is ‘aim at doing something’
13. (d) The reason is that is preferable to The reason is because.
or ‘aim for something’ ‘aim to’ is ‘informal’. The ‘aiming ........
encouraging is correctly used in (B). 14. (d) Parallel structure requires the use of the verbal noun as the
38 . (d) Last year we prepared for the SSC at (or with) GT, as GT object of the verb enjoyed: Enjoyed what? Splashing, bathing;
is one of the best institutes for SSC Entrance preparations (or and sun bathing, Enjoy should not be followed by an infinitive
preps). construction.
39 . (d) GT’s material undergoes continual upgrading, year after 15. (c) The other choices have misplaced modifiers.
year. (‘Upgradation’ is not a standard English word) 16. (b) The clause that money doesn’t make you happy is the
40 . (d) GT people are very strict about schedules. They would predicate nominative of the verb is.
never adjust any classes. (‘Prepone’ is not a standard English 17. (d) This was an unnecessary shift of pronoun. Do not shift from
word) you to one. Choice (d) changes the meaning unnecessarily.
41 . (d) GT has many good faculty members who are very good at
18. Option (b) is the best way of phrasing the answer. The original
their respective subjects and one can approach them anytime
and discuss one’s problems. Once the English teacher discussed sentence contains a dangling participle de-pending. The other

ww
an idiom and asked us to translate it word for word.
42 . (d) Whenever there is a new question in Mathematics, I usually
know the answer but forget it inadvertently. (Simply means “in
choices change the emphasis presented by the author.
19. (d) Since the words but also precede a phrase, the words not
only should precede the previous phrase.

w.E
a simply manner, not “just”)
43 . (d) Once I met a brilliant student at PT and asked him his
name (or asked what his name was). “Udayan Sarkar”, said
he. I wondered when I could get as many marks as he. (“Good
20 . Option (d) is the most appropriate answer. The pronoun they
has to match with their.
21 . (b) Choice (a) suffers from the use of the ambiguous pronoun
name” is not standard English usage)
asy
44 . (d) I wanted to know who were there on the interview panel
but the interview coordinator told me that I could not know
they. Choices (c) and (d) are correct grammatically but they
change the message by dropping the word immediately.
22 . (a) No error. Some of the other sentences are also grammatically

45
their names in advance.
. (d) They asked me about sexual harassment and its
En correct, but are rejected on the grounds that Choice (a) is more
concise than them.
23 . (c) This choice does not violate the parallel structure, neither
repercussions on our society. I said it gives rise (not gave rise)
to many problems in our society. (‘Eve-teasing’ is not a standard
English word) gin does it change the original message
24 . (d) This choice eliminates the error in parallel structure.
46. (d) Then they asked me to name a corporate personality who
was engaged in many acts of charity – to which I replied,
“Narayan Murthy”, but they said that my answer was wide of eer
25 . (c) This corrects the two errors in this sentence the error in case
(me for I) and the error in tense (had finished for finished).
26 . (a) Option (b) is incorrect because of the placement of the
the mark.
47 . (b) A and C are correct sentences. In B, ‘guilty of stealing’ to
be replaced by ‘guilty to stealing’. Similar mistake is in D, where ing
word 'gradually'. Options (c) and (d) are incorrect. The 'orderly
process' has to refer to the spread of freedom and prosperity
‘for three’, must be changed to ‘to three’.
48 . (none) The incorrect part is C in which plural verb is used
while the subject helping boost the image is singular. C part is
and (d).
.ne
and not to the 'thought process' as mentioned in options (c)

49
put in option. Hence (none) is the answer.
. (a) In C part the correct sentence is ‘it must make an
impression’. t
27 . (c) Options (a) and (d) have a parallelism error; Option (b) has
a pronoun error; Thus, option (c) gives the correct sentence.
28 . (d) Blunder – big mistake. I is verbose/ error of redundancy
word. She does not know how to swim.
LEVEL- II 29 . (d) I has subject – verb agreement error. it should be–he is
often dissappointed, II should be the people in my country are
1. (a) Option (b) is incorrect because of the placement of the smarter than those (the people) in other countries
word 'gradually'. Options (c) and (d) are incorrect. The 'orderly
The number of failures is of a product (d) has no error.
process' has to refer to the spread of freedom and prosperity
and not to the 'thought process' as mentioned in options (c) 30 . (c) Option (a) should read “Pele, whom many people consider
and (d). as....”. Option (b) should read “....it was difficult to say whom the
2. (c); Options (a) and (d) have a parallelism error; Option (b) has company would....”. Option (d) should read “..... Mr Brown, the
a pronoun error; Thus, option (c) gives the correct sentence. well-known author from the news bureau......”
3. (c) Option (a) doesn’t have agreement of tense. 31 . (c) Option a should read “..... porch light was broken again”.
Option (b) has apostrophe s. Option (d) needs takes instead of Option b should read “.... that isn’t going to get you anywhere”.
take. Option d should read “....we have hardly initiated.....”.
4. (a) (d) is also correct, but as (a) is earlier of two, so (a) will be 32 . (c) Gramatically c is the best choice as the sentence is in a
the answer. logical sequence.
5. (b) The given sentence is correct. So (b) is the correct option as
33 . (b) Comes across as a best option as a, c and d are grammatically
it repeats the original sentence.
not correct with A having the before politicians, c mixing up
6. (b) 7. (d) 8. (a) 9. (b)

Downloaded From : www.EasyEngineering.net


Downloaded From : www.EasyEngineering.net

148  l  Choosing the Correct Sentence

propositions and consumptions, and d having an error in by to the grinding of metal.


large number, which should be replaced with by a large number. C. Owning a car is still Dan’s goal, as it was of his parents
34 . (c) a, b and d are eliminated and answer is c because as the when they were his age.
difference is created between prices and not of or across as is D. Covering the floors with tiles costs twice as much as
given in the other option. covering them with linoleum.
35 . (d) 41 . (d)
A. Upon entering the restaurant, we were handed a menu by A. Like a woman I once met on the bus, the hostess was
the maître d’hôtel. dressed somewhat flamboyantly.
B. Working diligently and carefully, she (or they or someone B. The tycoon contributed more to the candidate’s campaign
etc.) fixed the faucet in no time at all. than did anyone else in the industry.
C. Employing ground-breaking techniques in alternative C. Unaccustomed to being spontaneous, Jill couldn’t decide
medicine, the doctors saw the patient’s health improve in a whether to spend her bonus on a new computer.
few days. D. The new mother spent most of the day worrying about her
D. David tried a handful of the table’s desserts, which son’s safety.
ultimately gave him an upset stomach. 42 . (c)
36 . (d) A. Use ‘results’ in place of ‘result’. It agrees with the subject

ww
A. The quick-witted hostess pitched a tent that protected the
guests at the garden party from the imminent rain.
B. Similar in so many ways, the twins could still be
‘eating’ (singular).
B. Delete ‘the’ before ‘interference’. The latter is an uncountable
noun and does not require any article here.

w.E
distinguished by their parents.
C. Based on the recent decline in enrollments, plans were
made by the admissions office to reevaluate its recruitment
C. Correct
D. Use ‘Mere’ (adjective) in place of ‘Merely’ (adverb);
‘speeches’ (noun) can be modified only by an adjective.
strategies.

asy
D. Unaccustomed to the rigors of college life, James allowed
(or couldn’t help etc.) his grades to drop.
43 . (b)
A. Use ‘somehow’ in place of ‘anyhow’.
B. Correct
37 . (d)
En
A. Mary returned the dress, which was torn at one of the
C. Use ‘by the Russians’ in place of ‘by Russians’.
D. Use ‘in’ in place of ‘under’.
seams, to the store.
B. We enjoy meeting people who have different interests.
C. The manager’s obstinate and surly attitude alienated his gin
44 . (c)
A. It should be, ‘discovered’ instead of ‘invented’. We discover
something that already exists and invent something that
employees.
D. Like other children in her neighborhood who were home- eer
did not exist before.
B. Use ‘excepting’ in place of ‘except’.
C. Correct

38 . (d)
schooled, Joan sometimes missed being in a classroom
with her peers.
ing
D. It should be ‘why you are’ instead of ‘why are you’. It
is a statement, not a question. Hence, no subject-verb
A. In contrast to the trapeze artists who fumbled their routine,
the circus clowns kept the audience entertained for hours
inversion.
45 . (d) All the form sentences are correct.
.ne
46 . (b) In B part with plural subject ‘infrastructure gaps’ should be
with their antics.
B. The clothes hanging on the racks inside the store looked
more appealing than those in the store window.
C. There are about as many gymnasium members in the
followed by plural verb ‘need’.
t
47 . (b) In B part the proper sentence is that BCCI needs to show
the same will, as it has shown in making players play domestic
cricket.
boxing class as there are in the aerobics class.
48 . (d) A and B are correct. C is incorrect because semicolon
D. Brett decided to use his own money rather than his parents’
should be used instead of hyphen after ‘out of slump’. Stimuli is
credit card to pay for the stereo.
plural so article ‘a’ in phrase ‘a temporary stimuli’ is incorrect
39 . (d) in statement D. The word ‘affect’ should be replaced by word
A. Julia was able to climb the tree as fast as her brothers did. ‘effect’ in statement E. Effect means a change which is a result of
B. The blue dress looks more flattering on you than the red an action whereas affect is to produce a change or difference in
one does. something. Since there is no option with A and B. Choose the
C. Three times more students attended the prom this year option B. Hence, correct answer is option (d).
than did last year. 49 . (c) The correct verb should be ‘said’ instead of ‘told’ in A so it
D. Joe went to bed early because his will to succeed in the race is incorrect. B is correct. In sentence C, the phrase ‘have been
the following morning was greater than his desire to play handed to us’ should be ‘have been handed over to’, hence it
pool with his friends. is grammatically incorrect. D is correct. The word ‘hence’
40 . (d) should be substituted by ‘but’ in sentence E. So, correct pair of
A. Sam was away on vacation longer than his friends were. sentences are BD and correct option is (c).
B. Most of the audience did not enjoy the concert likening it

Downloaded From : www.EasyEngineering.net


Downloaded From : www.EasyEngineering.net

11
PTER
CHA

Vocabulary — Word Bank

L ww
earning is a constant process, and the same applies to learning new words. There are no magic tricks to this, but the more
words you know, the better you will be able to express yourself. The key to increasing the number of words you know is to have
a habit of doing what is essential for this process and being committed to it. We will list some of these methods which you can
adopt for building a better vocabulary.

w.E
EFFECTIVE WAYS OF INCREASING VOCABULARY
(1) Always be on the lookout for new words:

asy
A lot of people just skip a new word when they see it, and then forget to look it up. The most important thing while learning new
words is to find and almost hunt for new words. Whenever you see one, make sure you understand what that means and how it
is used.
En
Whenever you encounter a new word, first try to think what it would mean by looking at the context of the text. Try to guess

gin
what would be the most logical meaning of that word. Then, grab a dictionary and see what is written there about that word. This
would indeed inhibit your flow of reading and take you longer to finish pages, but in the long run, it is beneficial because learning
new words will only help you in easy understanding and fast-reading long texts.
eer
Make it your routine, to note down all the new words you have learned in that particular day; because it is important to revise

ing
the words as there are chances that you may not remember all the new words you have learned.
(2) Read
(a) … as much as you can:
.ne
The best way to find the words you should know is to read whatever you can get your hands on. Reading also helps in

remember that word, then it would also help your confidence. You should make a habit of reading a new book/novel/ t
revising the newly learned words as you might come across one of those words in the text you have just read. If you correctly

magazine every week and a newspaper every day. If you can do more, then it can only be better for you. When you pick up a
new word, make sure you understand its meaning through some reputed dictionary and write that down somewhere.
Reading is quintessential for learning new words, so you have to read, even if you do not like it.
(b) … challenging books:
Many people just do not read literature books, or even the academic books written by foreign writers. But, you must consider
these as rich sources of learning new words. For example, a classic literature book is bound to present more new words as
compared to any other novel.
You should start with the classics because of two reasons. Firstly, these are trusted authors, so their writing is time tested
and will have the correct use of words and proper grammar. Secondly, the classic English literature is a source of profound
knowledge and hence, you will come across a wide variety of words.
(c) … poems:
Good poems are famous for their concise word usage. You are certain to learn a lot of adjectives, especially by reading poems.
A page of poem would probably give you more new words than a page of novel.

Downloaded From : www.EasyEngineering.net


Downloaded From : www.EasyEngineering.net

150  l  Vocabulary—Word Bank

(3) Maintain a diary:


Having a diary is very important if you do not want to forget the words you learn. You can list all the words you have just learned
in this diary. Write down in this diary, any new word you encounter and all the meanings it has. You can also write the meaning
of this word in your words. You do not need to copy what the dictionary has to say. Also write a sentence using that word; thereby,
showing how this word is used.
Make sure you revise this diary daily, or once in a week. It all depends on your learning pace. Once you have found new words,
learning them is all about constantly revising those words until they stick in your memory and you are able to use them while
you speak or write.
What we would suggest is to allot some of your time to vocabulary learning. You will use this time to see those words you learned
that day, and revise the ones that you have already learned. It is also beneficial to set targets like, how many words you plan to
learn by the end of the week/month and work towards achieving those goals.
(4) Using a dictionary:
Almost all of us know how to use a dictionary. It is a great source for vocabulary building and you can follow following steps
towards using it better:
(i) Use your own dictionary and keep it where you do your reading so that you are more likely to use it. Nowadays, a lot of

ww mobile dictionary apps are also available, so if possible, have one in your mobile, as it will enable you to quickly find the
meaning of a word.
(ii) In your dictionary, encircle the word which you have just learnt. This way, when you are just having a look at your dictionary,

w.E
or looking for a word in it, you will come across this circle and hence, will be able to revise these previously learned words.
It serves as a quick review.
(iii) Make sure that you read all that is given about that word in the dictionary: some words have multiple meanings and you

asy
should remember them all. Dictionaries provide synonyms of that word in order to make you understand the meanings of
the word you are looking for, and reading all those synonyms will help you understand this word better.
Reading the history of this word, or its origin can also help in remembering the meaning of the word.
(5) Knowing the roots:
En
More than half of the English words are derived from Greek and Latin, i.e. they have Greek and Latin roots. If you are good with

gin
identifying the roots of the words, then you may be able to understand the meaning of the word even without looking in the
dictionary; it is quite an effective technique.
We will look at the word sophomore as an example:

eer
It refers to a second-year undergraduate student, but what does it mean to be sophomoric? Identifying the root behind the word
will help us understand this. We know what philosophy means: “study of proper behaviour or search for wisdom” i.e. it relates

ing
a bit to studying. Now, both sophomore and philosophy have the same Greek root, so from that we can derive that there will be
some similarity or connection in their meanings. The other part of the ‘–mor’ shares the Greek root with moron. So sophomores
are people who think they know a lot, but they actually do not, i.e. a wise fool is known as sophomoric.
.ne
There are great resources available for learning words through their word roots. Look up these resources and start on your way
of vocabulary freedom today.

ROOT WORDS OR WORD ROOTS


A root word is the most basic form of a word that is able to
bell, belli
Meaning: war
t
Examples: Antebellum - Belonging  to a period before a
convey a particular description, though or meaning. It does not
have a prefix or a suffix (at the end of a word). Example ACT, war, especially the American Civil War, bellicose - warlike;
ALL, ALTER etc. belligerent - hostile, ready to fight; rebel - person who opposes
and fights, rebellion - refusal to accept some authority, code
Ambi, Amphi or convention.
Meaning: both, on both sides, around
Examples: ambidextrous - able to use both hands equally; bene
ambidexterity, ambiguous - having more than one meaning; Meaning: good, well
ambivalence - conflicting or opposite feelings toward Examples: benefactor - person who gives money to a cause;
a person or thing, ambivalent, ambient, amboceptor, beneficial - producing a good effect; benevolent - showing
amputation, ancipital. kindness or goodwill
ami, amic, imic calc
Meaning: Friend Meaning: stone
Examples: Amicable, Inimical, amiable - friendly, pleasant, Examples: calcite; calcium- the flame of acetylene gas
lovable; amity - friendly and peaceful relations; amorous - generated by reaction of calcium carbide with water;
showing romantic love. calcification- impregnation with calcareous matter

Downloaded From : www.EasyEngineering.net


Downloaded From : www.EasyEngineering.net

Vocabulary—Word Bank  l 151

carn fid
Meaning: flesh, meat Meaning: faith
Examples: carnivorous - flesh-eating; carnal - pertaining to Examples: confide - place trust in someone, fidelity -
the body or flesh; incarnate - given bodily form faithfulness; fiduciary - a trustee;
cata flect
Meaning: down, against completely, intensive, Meaning: bend
Examples: cataclysm - a flood or other disaster, catalog - Examples: deflect - to bend course because of hitting
a complete --ing; catastrophe - turning for the worst, a something; inflection - a bending in the voice’s tone or pitch;
substantial disaster flexible - easily bending.
cede, ceed, cess gastr
Meaning: go, yield Meaning: stomach
Examples: exceed - to go beyond the limits; recede - to go Examples: gastric - pertaining to the stomach; gastronomy
back; accessible - easily entered, approached, or obtained; - serving the stomach by providing good food; gastritis -
inflammation of the stomach.
chrono
Meaning: time gen

ww
Examples: chronic - lasting for a long time; chronological -
arranging events in time order, synchronize - happening at
the same time
Meaning: genesis, birth, production, formation, kind
Examples: genealogy - the study of the history of a family;
generation - all the people born at approximately the same
demos
w.E
Meaning: people
Examples: democracy - government of the people;
ger
time; genetic -relating to heredity encoded in the genes.

Meaning: old age

among people in a region


asy
demographic - the study of people; epidemic - spreading Examples: geriatrics - medicine pertaining to the elderly;
gerontocracy - the rule of the elders; gerontology - the
science of aging.
derma
Meaning: skin
En
Examples: dermatologist - a doctor for the skin; pachyderm
graph
Meaning: writing, recording, written
- a class of animals with very thick skin (elephant,
rhinoceros); dermatitis - inflammation of the skin gin
Examples: Graphology - the study of handwritings;
autograph - written with one’s own hand; seismograph - a
ego
Meaning: self helic
eer
machine noting strength and duration of earthquakes.

Meaning: spiral, circular


Examples: egoistic - self-centered; alter ego - a higher aspect

endo
of oneself; egomania - excessive preoccupation with oneself.
ing
Examples: helicopter - an aircraft with horizontal rotating
wing; helix - a spiral form; helicon - a circular tuba.

Meaning: within, inside


Examples: endotherm - a creature that can keep its inside
helio
Meaning: sun .ne
erg
temperature fairly constant; endocrine - relating to glands
that secrete directly into the blood or lymph; endogamy -
the custom to marry within one’s clan, tribe etc.
t
Examples: heliotropism - movement or growth in relating
to the sun; heliograph - apparatus used to send message with
the help of sunlight; helianthus - genus of plants including
sunflowers.

Meaning: work homo, homeo


Examples: ergonomics - study of the working environment; Meaning: like, alike, same
energy - the power to accomplish work; energetics - science Examples: homogeneous - of the same nature or kind;
that looks at energy and its transformation. homonym - sounding alike; homeopath - a therapy that is
based on treating “same with same”
aesth
hydro
Meaning: feeling, sensation, beauty
Examples: esthetician - someone who beautifies; aesthetic - Meaning: liquid, water
pertaining to a sense of beauty; kinesthesia - the sensation Examples: hydrate - to add water to; hydrophobia - intense
of bodily movement. fear of water; hydroponics - growing plants in liquid nutrient
solution; hydraulic - operated by force created by a liquid.
fac
icono
Meaning: make, do
Examples: artifact - an object made by a person; factory - a Meaning: image
place where things are made; malefact - a person who does Examples: icon - an (often religious) image, in modern usage
wrong. a simplified graphic of high symbolic content; iconology

Downloaded From : www.EasyEngineering.net


Downloaded From : www.EasyEngineering.net

152  l  Vocabulary—Word Bank

- science of symbols and icons; iconoclast - someone who osteo


destroys religious images and traditional beliefs. Meaning: bone
idio Examples: osteoarthritis - inflammation caused by
Meaning: peculiar, personal, distinct degeneration of the joints; osteopathy - therapy that uses
among others manipulation of the skeleton to restore health;
Examples: idiomatic - Peculiar to a particular language;
osteology - the study of bones.
idiosyncracy - a physical or mental characteristic typical or
a particular person; idiot - someone who is distinctly foolish oxi/oxy
or stupid. Meaning: sharp
ject oxymoron - combining two ideas that sharply contradict
each other; oxidize - corrode a surface.
Meaning: throw
Examples: eject - to throw someone/something out; para
interject - to throw a remark into a discussion; project - to Meaning: beside, beyond, abnormal, assistant
cast or throw something. Examples: parasite - an organism that lives on and off
another living being; parallel - alongside and always an equal
jud
distance apart; paragraph - a portion of a written document
Meaning: law that presents a distinct idea.

ww
Examples: judgment - a decision of a court of law; judicial
- having to do with judges or courts of law; judiciary - a
system of courts of law.
para
Meaning: protection from
Examples: parachute - protection from falling; parasol - an
kine
w.E
Meaning: motion, division, kinetics - study of the force of
motion;
umbrella used to protect from the sun;
pathos

asy
Examples: psychokinesis or telekinesis - the ability to move
objects with your mind; cinematography - motion picture
making.
Meaning: feeling, emotion
Examples: antipathy - a feeling of great dislike; apathy - a
lack of feeling or interest; empathy - ability to understand
lab
Meaning: work, En another’s feelings.
radic, radix
Examples: collaborate - to work with a person; elaborate - to
work out the details; laborious - requiring a lot of hard work. gin
Meaning: root
Examples: eradicate - pull out at the roots; radical -
fundamental, looking at things from a drastic point of view;
lact
Meaning: milk
radio eer
radish - an edible root of the mustard family.

Examples: lactate - to give milk, nurse; lactose - the sugar

logos
contained in milk; lactic acid.
ing
Meaning: radiation, ray
Examples: radioactive - emitting radiation; radiologist -

Meaning: word, doctrine, discourse


Examples: logic - correct reasoning; monologue - a long
rhod
Meaning: red
.ne
someone diagnosing or treating via radiation.

speech by one speaker; analogy - similarity, especially


between things otherwise dissimilar.
loqu, locu
Meaning: speak
rhodium - an element which produces a red solution;t
Examples: rhododendron - a flower with red/pink flowers;

rhodopsin - a purple pigment in the retina that is needed for


vision.
Examples: eloquent - speaking beautifully and forcefully;
rid
loquacious - very talkative; elocution - art of public speaking.
Meaning: laugh
mal Examples: deride - to make fun of someone; ridicule - to
Meaning: bad, ill, wrong make fun or mock; ridiculous - silly, causing laughter.
Examples: malcontent - wrong content; malaria - “bad air”,
scop
infectious disease thought to originate from the “bad air” of
Meaning: see, examine, observe
the swamps, but caused by the bite of an infected mosquito;
Examples: microscope - a device used to see tiny things;
malicious - showing strong ill will.
periscope - a seeing instrument on a submarine; telescope
meter - a device used to see over a distance.
Meaning: measure scrib, script
Examples: audiometer- an instrument that measures hearing
Meaning: write, written
acuteness; chronometer- an instrument that measures time; Examples: inscribe - to write letters or words on a surface;
metric - measured. scribe - a person who writes out documents; describe - to
represent with words or pictures.

Downloaded From : www.EasyEngineering.net


Downloaded From : www.EasyEngineering.net

Vocabulary—Word Bank  l 153

tax vid
Meaning: arrangement Meaning: see
Examples: syntax - the systematic arrangement of words; Examples: evident clearly seen
taxonomy - the science of classification; ataxia - loss of the vince, vic
ability to coordinate muscle action.
Meaning: conquer
techno Examples: convince - to win someone over; invincible - not
Meaning: technique, skill able to be conquered; victory - the conquest of an enemy.
Examples: technology - the practical application of
vis, vid
knowledge; technocracy - rule of technology; technologically
Meaning: see
- characterized by technology.
Examples: vision - the ability to see; envision - to picture in
tele the mind; evident - clearly visible.
Meaning: far, distant, complete
viv/ivit
Examples: telephone - a device to talk to a distant person;
telescope - a device to view distant objects; television - a Meaning: live, life
device to receive pictures from afar; telecommuting - Examples: revival - the act of bringing back to life; vital -
working remotely, bridging the distance via virtual devices. pertaining to live; vivacious - high-spirited and full of life.

terra
ww
Meaning: land, earth
Examples: extraterrestrial - existing outside the earth;
voc/i


Meaning: voice, call
Examples: advocate - to speak in favor of; equivocate - to use

theo w.E
terrain - ground or land; territory - an area of land. misleading language that could be interpreted two different
ways; vocalize - to produce with your voice.
vor, vour
Meaning: god

asy
Examples: monotheism - belief in one god; polytheism
- worshiping more than one god; theology - the study of


Meaning: eat
Examples: carnivorous - meat-eating; voracious - desiring

ultra
religion, god, etc.
En xeno
or eating food in great quantities; devour - to eat quickly.

Meaning: beyond, extreme, more than


Examples: ultrahigh - extremely high; ultramodern - more
modern than anything else; ultrasonic - sound waves beyond
gin


Meaning: foreign
Examples: xenoblast, xenon - A colourless odourless inert
gaseous element occurring in the earth’s atmosphere in

urb
human hearing.
eer
trace amounts, xenogamy, xenophobic - afraid of foreigners;
xenogenesis - the creation of offspring that is completely
Meaning: city
Examples: suburb - residential area on the edge of a city;
xyl
foreigners.
ing
different from either parent; xenophile - attracted to

vac
urban - relating to a city; urbanology - the study of city life.

Meaning: empty


Meaning: wood
.ne
Examples: xylem - The woody part of plants: the supporting
Examples: evacuate - to empty a dangerous place; vacant -

verb
empty, not occupied; vacation - a time without work.

zo
and water-conducting tissue, consisting primarily of
t
tracheids and vessels, xylocarp; xyloid- resembling wood;
xylophone-an organ percussion stop of similar tone quality

Meaning: word
Meaning: animal, animal life, living being
Examples: verbalize - to put into words; adverb - a word
Examples: zoology - study of animals; zooid - One of the
relating to a verb; proverb - a short saying that expresses a
well-known truth. distinct individuals forming a colonial animal such as a
bryozoan or hydrozoan; zooplankton - Animal constituent
vers, vert of plankton; mainly small crustaceans and fish larvae.
Meaning: turn
zyg
Examples: reverse - to turn around; introvert - being turned
Meaning: a pair, yoke
towards the inside; version - a variation of an original;
Examples: Heterozygous - having dissimilar alleles at
controversy - a conversation in which positions are turned
corresponding chromosomal loci, zygote - a cell formed by
against each other.
the union of two gametes and the organism developing from
vice that; zygomorphic - capable of division into symmetrical
Meaning: acting in place of, next in rank halves by only one longitudinal plane passing through the
Examples: vice-president - the person next in rank to the axis.
president

Downloaded From : www.EasyEngineering.net


Downloaded From : www.EasyEngineering.net

154  l  Vocabulary—Word Bank

PREFIXES
Prefix Meaning Example
a- not Asynchronous (not synchronous), apolitical
acro- high Acrobatic (The performance of stunts while in flight in an aircraft), acrophobia
ante- before Antemeridian (Before noon), antecede, anteroom
anti- Against, opposed to anti-war (against war), antibody, antisocial
On its own or one’s own,
auto- Automatic (Operating with minimal human intervention), autocrat, autograph
by its own action
bi- two Bicentric (Having two centers), biannual, binoculars
co- with, joint Coauthor (Joint author), coeducation, collision
contra- contrasting Contralateral (On or relating to the opposite side (of the body)), contrary

counter-
ww Opposite,
complementary
Counteractive (Opposing, neutralizing or mitigating an effect by contrary action) ,
counteroffensive
Extrajudicially (Beyond the usual course of legal proceedings; legally unwarranted),
extra-

hexa- w.E
Beyond, out of scope

six
extraordinary
Hexadecimal (Of or pertaining to a number system having 16 as its base), hexagon
im-
in-
not
not asy Impossible (Not capable of occurring), impatient
Insane (Afflicted with or characteristic of mental derangement), inappropriate
intra- within
En
Intragroup (Occurring within an institution or community), intrastate
ir-

macro-
not

Very large in scale gin


Irresponsible (Showing lack of care for consequences), irredeemable
macro instruction (A single computer instruction that results in a series of instructions in
machine language), macro economics

mid-
Used in combination to eer
Midafternoon (The middle part of the afternoon), midway, midterm

multi-
denote the middle
Multiple ing
Multibank (A bank holding company owning several banks), multimedia, multicoloured
non-
per-
Not
For each .ne
Nonacceptance (The act of refusing an offer), non-profitable, non-friction
Percent (A proportion in relation to a whole), permanent
pseudo-
re-
Fake, False
Again, anew
Pseudobulb (A solid bulblike enlargement of the stem of some orchids), pseudonym
Reaccept (Consider or hold as true), reappraise, reawake
t
un- Not unorganized (Not having or belonging to a structured whole), unbelievable
up- up; “increase” Upward (Spatially or metaphorically from a lower to a higher position)
ad- movement to advance (expert, better) adulterate
trans- across, beyond transnational, transatlantic
before in time place or
post- prelude, preadolescent, precondition
order
pro- favouring, in support of pro-african
semi half, partly semifinal, semicircle, semiconscious
eu- good, well euphoria, euphemism
fore- in front of forebear, forecast

Downloaded From : www.EasyEngineering.net


Downloaded From : www.EasyEngineering.net

Vocabulary—Word Bank  l 155

SUFFIXES
Suffix Meaning Example
-able able to be excitable, portable, preventable
-ac pertaining to cardiac, hemophiliac, maniac
acity (-ocity) quality of perspicacity, sagacity, velocity
-ade  act, action or process, product blockade, cavalcade, promenade,
-age action or process passage, pilgrimage, voyage
-aholic (-oholic) one with an obsession for workaholic, shopaholic, alcoholic
-al relating to bacterial, theatrical, natural
-algia pain neuralgia, nostalgia,
-an (-ian) relating to, belonging to Italian, urban, African
-ance state or quality of brilliance, defiance, annoyance
-ant a person who applicant, immigrant, servant

ww
-ar
-ard
of or relating to, being
a person who does an action
lunar, molecular, solar
coward, sluggard, wizard
-arian
w.E
-arium (orium)
a person who
a place for
disciplinarian, vegetarian, librarian
terrarium, aquarium, solarium
-ary
-ate asy
of or relating to
state or quality of (adj.)
literary, military, budgetary
affectionate, desolate, obstinat
-ation
-ative
En
action or process
tending to (adj.)
creation, narration, emancipation
creative, preservative, talkative
-cide
-cracy
act of killing
rule, government, power gin homicide, suicide, genocide
bureaucracy, aristocracy, theocracy
-crat
-cule
someone who has power
diminutive (making something small) eer
aristocrat, bureaucrat, technocrat
molecule, ridicule,
-cy
-cycle
state, condition or quality
circle, wheel ing
efficiency, privacy, belligerency
bicycle, recycle, tricycle
-dom
-dox
condition of, state, realm
belief, praise .ne
boredom, freedom, wisdom
orthodox, paradox
-ectomy
-ed
-ee
surgical removal of
past tense
receiver, performer
appendectomy, hysterectomy
called, hammered, laughed
nominee, employee, devotee
t
-eer associated with/engaged in engineer, volunteer
-emia blood condition anemia, hypoglycemia, leukemia
-en makes the word a verb awaken, fasten, strengthen
-ence state or condition, action absence, dependence, negligence
-ency condition or quality clemency, dependency, efficiency
inclined to performing/causing, or one
-ent competent, correspondent, absorbent
who performs/causes
-er more bigger, faster, happier
-er action or process flutter, ponder, stutter
-er a person who does an action announcer, barber, teache
-path one who engages in homeopath, naturopath, psychopath
pathy feeling, diseased sympathy, apathy, neuropathy

Downloaded From : www.EasyEngineering.net


Downloaded From : www.EasyEngineering.net

156  l  Vocabulary—Word Bank

FOREIGN WORD AND PHRASES Circa - about (‘circa 1930’)


Foreign words and phrases are generally not asked directly. Confoere - colleague
But the knowledge of foreign words and phrases will help Contretemps - an unexpected or untoward event; a hitch
you in reading comprehension and other types of common
questions. So, make yourself familiar with the common Corrigenda - a list of errors (in a book)
foreign words and phrases. Coup d’etat - violent change in government.
Ab initio - from the beginning. Coup de grace - a finishing stroke. (The coup de grace
Ab origin - from the origin. of the Russian Revolution was the total
annihilation of the Czar family)
Addenda - list of additions’. (addenda to a book)
Cul-de-sac - a blind alley (The failure of the Policy of
Advalorem - according to value.
non-alignment in 1962 saw our foreign
Ad hoc - a body elected or appointed lor a definite reach a cul-de-sac)
work. (ad hoc comm ittee).
Deback’ - complete tout (debacle of opposition in
Ad infinitum - to infinity. the election)
Alma mater - a school which one has attended. Defacto - actual or actually (de facto recognition to
A la carte

Alamode
ww - according to the bill of fare. (a la carte
dishes are available)
- according to the custom (fashion). (a la
Dejure
a state)
- from the law, by law.

Alter ego w.E mode silk)


- the other self, intimate friend, (Kissinger
is the alter ego of Nixon)
Denovo
Denouement
De profundis
- anew, again (trial of a case)
- the end of a plot (in play)
- out of the depths
Amende
honorable asy
- satisfactory apology, reparation. Dernier resort
Detente
- last resort
- easing of strained relations especially
Amour propre
Ancien regime
- self love
- the forrner order of things. En Dramatis
between states.
- characters of the drama or play.
A posteriori
A priori
- empirical
- from cause to effect, presumptive. (every gin
personae
Elite - the best part; the pick. (The elite of town)

Apropos
science cannot be taught a priori)
- in respect of
Enfant terrible

eer- a terrible child; one who makes


disconcerting remarks.
An couran
Au fait
- fully acquainted with matters.
- expert
Entrepreneur

En masse ing
- person in effective control of a business
organization.
- in a body. (They took leave en masse)
Au revoir - until we meet again (to say au revoir at
parting) En rapport
Entourage
- in harmony
.ne
- friends, group of people accompanying a
Avant propos
Beau ideal
Beaumonde
Beaux esprits
- preliminary matter, preface
- the ideal of perfection.
- the world of fashion.
- men of wit.
Errata
Esprit de corps
dignitary.
- list of errors t
- the animating spirit of a collective body,
as a regiment.
Bete noire - a special aversion (India has always been
bete noire for Pakistan) Etcetera - and the rest.
Bona fide - good faith (His bona fide in the matter Eureka - I have found it
cannot be doubted) Ex-officio - in virtue of his office.
Bizarre - odd, fantastic. Expose - a statement
Ban voyage - a good voyage or journey to you. Expost facto - acting retrospectively
Casus belli - that which causes or justifies war. Fait accompli - a thing already done.
Cause celebre - a celebrated or notorious case in law Faus pas - a false step; slip in behaviour.
Charge d’ affaires - diplomat inferior in tank to an Hoipolbi - the rabble.
Ambassador. Impasse - a deadlock. (Talks reached an impasse)
Chefd’ oeuvre - masterpiece (Mona Lisa is Vinci’s Chef-d’ Inextenso - at full length
oeuvre)
In memoriam - in memory

Downloaded From : www.EasyEngineering.net


Downloaded From : www.EasyEngineering.net

Vocabulary—Word Bank  l 157

Intoto - entirely. (The committee’s


WORD LIST
recommendations were accepted in tote)
Ipso facto - by that very fact. Given below is a list of words placed in alphabetical order. Each
Laissez faire - non interference word is followed by a few of its synonyms. Note these words
whenever you come across them. You should be familiar with
Mala Fide - with bad faith most of the words for which synonyms are given if you have
Mal-a-propos - ill timed. done all the exercises till this point thoroughly. So, this list
Modus operandi - manner of working. (of a gang, group etc.) will be giving you synonyms for the words which you know.
Mutatis - with the necessary changes (rules will Thus learning will be easier.
mutandis come into force mutatis mutandis) Abandon - Leave, desert, forsake
Noblesse oblige - rank imposes obligation. Abase - Degrade, disgrace, humiliate
Nota ‘bene - note well Abhor - Hate, loathe, detest
Par excellence - pre-eminently. Abridge - Shorten, abbreviate
Pari passu - side by side. Absolute - Unalterable, unrestricted,
Per se - by itself. unconditional
Piece de - a resistance piece, the main dish of a Absurd - Ridiculous, silly, foolish
resistance
ww
Poste restante
meal.
- to remain in the post office till called for.
(said of letters)
Abundant
Accessory
Adept
- Ample, plentiful
- Additional, auxiliary, subsidiary
- Proficient, skilled, expert
Post mortem
Prima facie w.E - (examination) made after death.
- at first view or consideration. (prima
facie a good case)
Adherent
Adhesive
Admire
- Follower, stickler
- Sticky, glue, gum
- Praise, adore, esteem

Pro forma asy


Pro bone publico - for the good of the public
- for the sake of the form.
Adore
Adversity
- Respect, idolise, worship, admire
- Misery, misfortune
Pro rata
Protégé
- according to rate or proportion.
- one under the protection of another.
(S.Vietnam is U.S.’s protege). En Affliction
Alien
- Distress, sorrow, sadness
- Foreign, stranger, unknown
Quid pro quo
Raisond’etre
- an equivalent, something in return.
- the reason for a thing’s existence. gin
Alive
Alleviate
- Lively, vivacious, living
- Relieve, lighten, ease

Resume - a summary or abstract (of a discussion


Etc.)
Alms
Amend
Amicable eer - Gratuity, donation, grant
- Improve, change, emend
- Suitable, friendly, lovable, amiable
Sanctum
sanctorum
Seiratim
- holy of holies. (temple, church etc.)

- in a series
Anxiety
Apathy ing - Eagerness, misgiving, worry
- Indifference, neutrality
Sine die
Status quo
- without a day being appointed.
- the existing condition. (status quo on
Appalling
Apposite .ne
- Terrific, terrifying, dreadful, horrible
- Apt, suitable, well chosen

Stet
Sub judice
border should be maintained)
- let it stand.
- before a court, not yet decided. (The case
is sub judice)
Appraise
Apprehend
Arbitrary
Assent
- Evaluate, estimate
- Seize, fear, arrest
- Despotic, wayward
- Agree, consent, acquiesce
t
Sub rosa - under the rose; confidentially Astonish - Astound, surprise, amaze, bewilder
Sui gereris - in a class by itself Audacious - Bold, courageous, daring
Summon bonum - the chief good. Aversion - Dislike, detestation, hostility, hatred
Terra incognita - an unknown country Base - Mean, low, ignoble
Tour deforce - a notable feat or strength of skill. Beg - Implore, ask, beseech, solicit
Ultra vires - beyond one’s authority Behaviour - Conduct, deportment, way, demeanour
Verbatim - word for word Brave - Courageous, intrepid, bold, daring,
Vice versa - conversely valiant
Videliect - namely Brisk - Active, fast, quick, busy, alert
Vis-à-vis - opposite; face to face Brittle - Frail, fragile
Viva voce - an examination conducted orally. Brutal - Animal, savage, beastly, cruel
Vox populi, vox - The voice of the people is The voice of Burglar - Thief, bandit, brigand, stealer
die God. Bystander - Spectator, onlooker, beholder
Zeitgeist - spirit of the age. Calculate - Estimate, count, reckon, compute

Downloaded From : www.EasyEngineering.net


Downloaded From : www.EasyEngineering.net

158  l  Vocabulary—Word Bank

Callous - Hard, indifferent, cold-blooded Decipher - Translate, interpret, solve, explain


Calm - Cool, confident, quiet, serene, tranquil Decorum - Decency, etiquette, propriety, gravity
Cancel - Annual, withdraw, revoke, delete Decree - Law, edict, ordinance, mandate,
Candid - Sincere, straightforward. frank judgement
Captive - Prisoner, confined, jailed, bonded Defamation - Calumny, disparagement, debasement
Cause - Make, originate, induce, generate, Defection - Abandonment, desertion
create Defer - Postpone, delay
Censor - Cut off, prohibit, ban Deference - Respect, reverence, honour
Censure - Blame, condemn, reprove, reprimand Deformity - Disfigurement, malformation, ugliness
Character - Letter, emblem, type, OR nature, Dejected - Depressed, distressed, downhearted,
disposition, quality downcast
Charity - Philanthropy, benevolence Delectable - Charming, delightful, pleasant
Chaste - Pure, immaculate, virgin, refined Delegate - Commission, depute, authorise
Chatter - Babble, ramble, talk, discourse Deliberate - Knowingly done, intentional, forcible
Cheat - Defraud, gull, outwit, dupe Delicacy - Softness, nicety, slenderness,
Cite
Clothes
Colossal
ww -
-
-
Quote, mention, name, adduce
Apparel, attire, dress, garb
Huge, gigantic, enormous, big
Delusion
Demeanour
-
-
refinement, purity
Illusion, fancy, error, false belief
Behaviour, conduct, bearing
Commence
Commensurate w.E-
-
Begin, start
Equivalent, suitable, applicable,
proportionate
Demise
Demolish
Demure
-
-
-
Death, decease
Break, destroy, annihilate
Modest, coy, humane
Conceal
Confess
-
- asy
Hide, cover, shelter, disguise
Admit, acknowledge, reveal, agree
Denomination
Denounce
-
-
Name, appellation, designation
Accuse, malign, criticise, defame,
Confuse or
confound
- Mix, perplex, astonish, Amaze,
bewilder
En Deny -
condemn
Contradict, refuse, disavow, withhold
Consequent
Conspiracy
Convict
-
-
-
Following, resultant, outcome
Plot, intrigue, treason
Felon, culprit, criminal, guilty
gin
Deride
Descant
-
-
Ridicule, mock, taunt
Discourse, expatiate, enlarge

Cowardly
Coy
-
-
Craven, dastardly, fearful, poltroon
Modest, shy, reserved
Desire
Desolate
Despise eer -
-
-
Wish, long for, crave, covet
Lonely, deserted, solitary, devastated
Condemn, dislike
Crafty - Artful, adroit, dexetrous, cunning,
deceitful
Despondency
Despotic ing
-
-
Despair, dejection, hopelessness
Arbitrary, tyrannical, illegal
Crazy
Credence
-
-
Mad, insane, silly
Belief, faith, trust, confidence
Destitute
Destruction
-
- .ne
Needy, poor, miserable, indigent
Ruin, demolition, ravage
Crisis

Criterion
Criticism
-

-
-
Turning point, emergency, decisive
moment
Test, touchstone, standard, yardstick
Analysis, review, stricture
Detain
Detest
Dethrone
Devastate
-
-
-
-
Depose, remove (from office)
Ruin, demolition, ravage
t
Lock in, arrest, hold, custody
Despise, abhor, dislike

Cruel - Brutal, unmerciful, beastly, savage Devoid - Lacking, empty, vacant


Cynical - Captious, incredulous, sarcastic, Devout - Religious, reverent
morose
Dexterity - Adroitness, cleverness, skill
Danger - Hazard, risk, peril
Diabolical - Fiendish, devilish, wicked
Dash - Run, rush, fly
Diatribe - Tirade, denunciation
Dastardly - Cowardly, invaliant, afraid, fearful
Dictatorial - Tyrannical, arbitrary, despotic
Dawn - Daybreak, appear, (sunrise), begin
Diffident - Hesitating, doubtful, distrusting
Deadly - Fatal, lethal, destructive
Digression - Excursion, deviation, misguidance
Dearth - Scarcity, lack, want
Diligence - Care, industry, effort
Debase - Degrade, defame, disparage,
Dire - Terrible, awful, horrible; miserable
Decay - Decompose, rot, decline in power,
Disapprove - Condemn, reject, disallow
wealth, waste, wither, fade
Disavow - Deny, refuse
Decease - Death, demise, end
Disciple - Follower, learner, student
Deceit - Fraud, cheating, forgery
Disclose - Reveal, tell, uncover, divulge

Downloaded From : www.EasyEngineering.net


Downloaded From : www.EasyEngineering.net

Vocabulary—Word Bank  l 159

Disconsolate - Sad, cheerless, miserable Embarrass - Vex, confuse, entangle


Discredit - Disbelieve, doubt, disgrace Embezzle - Steal, peculate, cheat
Disgust - Abhorrence, dislike, detestation Embody - Incorporate, include, comprise
Dismay - Disappointment, discouragement Emolument - Salary, wage, remuneration
Disorder - Disease, illness, OR untidiness, Emulate - Compete, rival, vie against, copy
uncleanliness Enchant - Charm, bewitch, hypnotise
Disown - Disclaim, deny, renounce Encompass - Surround, encircle
Disparage - Debase, decry, defame Encounter - Come across, combat, fight
Dispose - Adjust, arrange, incline Encroach - Trespass, intrude, invade
Dispute - Argument, controversy, altercation Endeavour - Attempt, effort, aspiration
Disregard - Neglect, overlook, disrespect Endorse - Back, approve, ratify
Dissolute - Corrupt, mean, lax, licentious Endurance - Patience, continuance, fortitude
Distaste - Abhorrence, dislike, detestation Enfranchise - Liberate, free, (also: give right to vote)
Distorted - Blurred, maligned, changed, disguised, Enlighten - Illuminate, edify, elaborate
deformed, misrepresented Enmity - Hostility, hatred, animosity
Distress
Divert
Divine
ww -
-
-
Affliction, depression, misery
Turn aside, deflect, deviate
Heavenly, metaphysical, godlike
Enormous
Enrage
-
-
Big, huge, colossal, gigantic
Infuriate, madden, incense, irritate

Divulge
Docile
Doctrine
w.E
-
-
-
Reveal, uncover, disclose
Amenable, tractable, submissive
Precept, principle, teaching
Ensue
Entangle
Enterprise
-
-
-
Succeed, follow, result
Ravel, involve, perplex
Undertaking, venture, endeavour

Dogmatic -
preachy asy
Categorical, authoritative, firm,
Enthusiasm
Entice
Entreat
-
-
-
Zeal, ardour, interest
Allure, tempt, seduce, attract
Beseech, implore, beg
Dolt
Domicile
-
- Dwelling, home, residence En
Blockhead, stupid, fool, idiot, dullard
Entwine
Enumerate
-
-
Encircle, surround, encompass
Count, number one by one
Dotage
Downright
-
-
Senility, imebcility
Simple, unquestionable, blunt, frank gin
Enunciate
Envoy
-
-
Declare, publish, propound, reveal
Legate, messenger, ambassador
Dread
Drench
Drowsy
-
-
-
Apprehend, fear
Soak, wet
Sleepy, comatose, lazy, lethargic
Epoch
Equivocal eer -
-
Era, time, age
Doubtful, Ambiguous, uncertain

Dubious
Ductile
-
-
Suspicious, doubtful, unreliable
Pliant, yielding, flexible
Eradicate
Erroneous
ing
-
-
Root out, extirpate, annihilate
Wrong, false

Dupe
Dwindle
-
-
Cheat, befool, steal
Shrink, diminish, decrease
Erudite
Esteem
Eulogy
-
-
- .ne
Learned, scholarly, lettered
Admire, appreciate, adore, respect
Laudation, praise, extolling, felicitation
Earnest
Eccentric
Economise
Ecstasy
-
-
-
-
Eager, ardent, intent,anxious, sincere
Irregular, anomalous, abnormal, odd
Save, retrench
Trance, enchantment, rapture
Evidence
Evince
Exact
-
-
-
Testimony, proof, witness
Show, manifest, demonstrate
Extort, oppress, loot
t
Exaggerate - Amplify, overstate
Efface - Blot out, obliterate, destroy
Excerpt - Extract, quotations
Effeminate - Womanly, weak, unmanly
Exile - Expulsion, banishment, expatriation
Efficacy - Energy, virtue, potence
Exonerate - Acquit, absolve, release
Egotistic - Self-centered, egoist, self-conceited
Exorbitant - Excessive, too much, very high
Egregious - Conspicuously bad, sinful, monstrous,
Extinguish - Quench, terminate, destroy, put out
shocking
Extravagant - Excessive, lavish, stylish
Elaborate - Explain, discuss, elucidate
Exuberant - Abundant, plentiful
Elevated - Elated, promoted, upgraded, risen
Exult - Triumph, rejoice, delight
Eliminate - Remove, replace, dismiss, discard
Fable - Story, legend, myth, fiction
Eloquence - Oratory, rhetoric, finery (of speech)
fluency of expression Fabricate - Construct, forge, invent
Emanate - Originate, proceed, spring, issue Fabulous - Fictitious, mythical, exaggerated
Emancipate - Free, deliver, liberate Facile - Fluent, ready, glib (of writing), pliable,
docile, tractable

Downloaded From : www.EasyEngineering.net


Downloaded From : www.EasyEngineering.net

160  l  Vocabulary—Word Bank

Faction - Clique, cabal, discord, section Fray - Battlefield, combat, brawl


Fallacy - Deception, illusion, mistake Frisk - Skip, dance, caper, frolic
Falter - Waver, hesitate, delay, flounder Frivolous - Vain, foolish, trivial
Famine - Hunger, starvation, scarcity of food Frugal - Economical, thrifty
Fanatical - Bigoted, enthusiastic Futile - Useless, hopeless, in vain
Fancy - Liking, conception, craving, whim Gaiety - Hilarity, jollity, festivity
Farcical - Droll, comic, extravagant Gainsay - Contradict, dispute, controvert, deny
Fascinate - Charm, bewitch, attract Gallantry - Courage, bravery, heroism
Fastidious - Particular, over-nice, squeamish Garbage - Filth, waste, useless, throwaway, trash
Fatal - Deadly, lethal, mortal Garner - Accumulate, collect, gather
Fatigue - Weakness, exhaustion, tiredness Garrulous - Prattling, chattering
Feeble - Weak, frail, dim Gawky - Awkward, clumsy
Felicitate - Congratulate, compliment Gay - Happy, merry, joyous
Felicity - Joy, happiness, good luck Generous - Noble, magnanimous, kind, liberal
Felon - Criminal, sinner, guilty, bandit Genteel - Well-bred, well-cultured, polite, refined
Ferment
Ferocity ww -
-
Excite, agitate
Fierceness, vehemence, fanaticism
Genuine
Ghastly
-
-
True, authentic, real
Horrible, horrifying, horrific
Fervent
Fervour
Festivity
w.E
-
-
-
Glowing, heated, impassioned
Warmth, glow, vehemence
Gaiety, merry-making
Ghost
Glimmer
Glimpse
-
-
-
Phantom, spirit, spectre
Shine, flash, gleam
Glance, (quick) look, (brief) view
Fetter
Feud
Fickle
-
-
-
asy
Shackle, bind, imprison
Dispute, broil, strife
Changeable, vacillating, varying
Glisten
Gloom
Glutinous
-
-
-
Shine, beam, glow
Depression, melancholy, loneliness
Sticky, viscous
Fiendish
Figurative
-
-
Devilish, diabolical, malignant
Typical, imaginative, emblematic,En Gluttonous
Grandeur
-
-
Greedy, gorging, voracious
Splendour, magnificence, glory

Filthy -
metaphorical
Dirty, foul, nasty gin
Grapple
Gratification
-
-
Grasp, clutch, seize
Satisfaction, enjoyment
Fissure
Flaccid
-
-
Crevice, rift, narrow opening
Soft, loose, weak
Gravity
Greed eer
-
-
Seriousness, importance, calmness
Avarice, ravenous, envious, covetous
Flatter
Fleece
Flexible
-
-
-
Adore, please, praise
Rob, despoil, cheat
Variable, pliable, changeable
Grievance
Grotesque
-
-
ingHardship, complaint, trouble
Horrifying, contorted, bizzare,
whimsical
Flimsy
Flounder
-
-
Transparent, thin, trivial, tenuous
Stumble, falter, wallow, struggle
Grudge
Gullible
-
- .ne
Grievance, begrudge, resent
Simple, easy, pliable, credulous
Fluctuate
Flutter
Folly
-
-
-
Undulate, waver, vacillate
Flip, quiver, ruffle, agitate
Absurdity, silliness, imprudence,
foolishness
Hallucination
Hamper
Hapless
-
-
-
Delusion, illusion, nightmare
Impede, block, fetter, bind
Unfortunate, unlucky
t
Harangue - a lengthy speech, oration
Fondle - Caress, touch, rub Hardship - Trouble, adversity, difficulty
Foray - Incursion, inroad, venture Haughty - Arrogant, overbearing, imperious
Forbearance - Abstaining, refraining Havoc - Devastation, destruction, ruin
Forbid - Prohibit, disallow, debar Hearty - Sincere, warm, ardent
Forebode - Betoken, indicate, augur Heave - Raise, lift
Forlorn - Disconsolate, cheerless, distressed, Hedge - Fence, hem
abandoned, lonely Heed - Advise, note, consider, mind
Formidable - Dreadful, difficult, hard to overcome Heinous - Atrocious, odious, wicked
Fragile - Weak, feeble, slender, delicate Hem - Border, edge, fringe, outskirt
Fragrant - Odorous, balmy, soothing Herculean - Colossal, laborious, excessive
Frailty - Weakness, delicacy, fragileness Heterogeneous - Dissimilar, unlike, different, diverse,
Frantic - Frenzied, mad, distracted varied
Fraudulent - Dishonest, cheating, deceitful Hideous - Terrific, horrible, filthy

Downloaded From : www.EasyEngineering.net


Downloaded From : www.EasyEngineering.net

Vocabulary—Word Bank  l 161

Hilarious - Exceedingly, funny, boisterously merry, Infer - Gather, conclude, deduce


amusing, joyous Influence - Authority, effect, power
Hindrance - Impediment, hampering, obstruction, Infringe - Break, violate, transgress, encroach
obstacle
Ingredient - Constituent, component, element
Histrionic - Theatrical, dramatic
Inherent - Inborn, innate, built-in
Hoax or Humbug - Dupe, cheat, befool
Inhibition - Restraint, check
Homage - Deference, salute, worshipping
Iniquitous - Unjust, wrong, unfair
Horizon - Verge, limit, skyline
Initiate - Start, begin, inchoate
Hostile - Adverse, opposing, inimical
Innocuous - Harmless, mild, innocent
Hover - remain in one place in the air, remain
Insanity - Madness, lunacy, mania
suspended, linger
Insidious - Deceitful, treacherous
Humane - Compassionate, caring, benevolent
Instantaneous - Immediate, Sudden, quick
Humiliate - Abase, insult, mock, defy
Instigate - Arouse, misguide, provoke
Humorous - Amusing, laughable, comical
Intact - Untouched, unscathed, whole,
Hypocrisy - Pretence, imposture, deceit
unbroken, undamaged, unimpaired,
Idolise

Illogical
ww
Illegal or Illicit
-
-
-
Adore, worship, admire
Unlawful, prohibited
Inconsequent, irrational
Integrity -
entire
Oneness, entirety, completeness,
honesty, wholeness, soundness
Imitate
Immaculate
Immature
w.E -
-
-
Copy, follow, mimic, emulate
Spotless, stainless, perfect
Crude, childish, unseasoned
Intellectual
Intentional
Intercourse
-
-
-
Intelligent, rational, learned
Deliberate, intended,
Intimacy, association, communication
Imminent
Immodest
-
- asy
Impending, near, due, threatening
Indecent, indelicate, unchaste
Intermission - Suspension, stoppage, pause, cessation,
interval
Immortal
Impassioned
-
-
Everlasting, undying, endless
Fervent, frenzied, fanatical
En Intervene
Intimate
-
-
Interpose, mediate
Close, tender, friendly or informal,
advertise, inform
Impeachment
Implicit
Impostor
-
-
-
Imputation, accusation
Implied, assumed, inferred
Cheat, conman, charlatan
gin
Intolerable
Intransigent
-
-
Unendurable, unbearable
Uncompromising, inflexible
Impracticable
Impressive
-
-
Impossible, (merely) theoretical
Striking, affecting, extraordinary
Intrepid
Intricate eer -
-
Brave, fearless
Complex, difficult, complicated
Impunity
Inane
-
-
Exemption (from punishment)
Empty, silly, idiotic
Intuition
Inveterate ing
-
-
Insight, premonition, instinct
Habitual, deep-rooted
Inarticulate
Incense
-
-
Voiceless, indistinct, inexpressive
Infuriate, enrage, anger
Involuntary
Irksome
Irresolute
-
-
- .ne
Compulsory, unwilled, reflex
Annoying, disagreeable, tedious
Wavering, confused, vacillating
Incessant
Incognito
Inculcate
Inculpate
-
-
-
-
Unceasing, continual
(Identity) Concealed, secretly, stealthily
Instil, enforce, pass on, generate
Blame, incriminate
Itinerant

Jaded
-

-
nomadic
Tired, weary
t
Travelling (on a circuit), wandering,

Jargon - cant, technical language, gibberish


Incumbent - Compulsory, obligatory, binding
Jocose - Jocular, humorous
Incursion - Inroad, foray, venture
Jocular - Inclined to joke.
Indefatigable - Tireless, assiduous
Jovial - Merry.
Indict - Accuse, charge
Judicious - Prudent.
Indiscriminate - Mixed, undistinguished, confused,
Juxtaposition - Closeness, nearness
wanton
Kiosk - Stand (open on one side), booth
Indolence - Apathy, inactivity, laziness,
sluggishness, lethargy Knead - Work up (into dough), press, squeeze
Industrious - Busy, active, tireless Lacerate - Tear (tissue toughly), mangle
Inevitable - Unavoidable, certain, sure Lachrymose - Given to shedding tears.
Inexorable - Relentless, indefatigable Lackadaisical - Listless.
Infallible - Unfailing, unerring, certain Laconic - (Briskly) Short, concise, pithy
Infectious - Catching, contaminating, corrupting, Languid - Listless, spiritless
transmittable

Downloaded From : www.EasyEngineering.net


Downloaded From : www.EasyEngineering.net

162  l  Vocabulary—Word Bank

Languish - Die (with hunger or desire), weaken, Ominous - Portentous.


droop Onerous - Burdensome or oppressive.
Lascivious - Lustful. Onus - A burden or responsibility.
Lassitude - Weariness, tiredness Palate - The roof of the mouth.
Latent - Hidden, concealed Palatial - Magnificent.
Laudable - Praiseworthy. Palliate - To cause to appear less guilty.
Legacy - A bequest. Palpable - Perceptible by feeling or touch.
Legitimate - Genuine, lawful Panoply - A full set of armor.
Levity - Frivolity. Paragon - A model of excellence.
Libel - Defamation. Pariah - A member of a degraded class; a social
Licentious - Wanton. outcast.
Lithe - Supple. Paroxysm - A sudden outburst (of any kind of
Loquacious - Talkative. activity).
Lustrous - Shining. Paucity - Fewness.
Malaise - A condition of uneasiness or ill-being. Pellucid - Translucent.
Malevolence
Malleable ww -
-
Ill will.
Pliant.
Penchant
Penurious
-
-
A bias in favor of something.
Excessively sparing in the use of
money.
Mawkish
Mellifluous
Mendacious
w.E -
-
-
Sickening or insipid.
Sweetly or smoothly flowing.
Untrue.
Penury
Peremptory
-
-
Indigence.
Precluding question or appeal.

Mendicant
Mesmerize
-
-
A beggar.
To hypnotize. asy Perfidy
Perfunctory
Peripatetic
-
-
-
Treachery.
Half-hearted.
Walking about.
Meticulous
Mettle
-
-
Over-cautious.
Courage.
En Perjury
Permeate
-
-
A solemn assertion of a falsity.
To pervade.
Mien

Moderation
-

-
The external appearance or manner of
a person.
Temperance.
gin
Pernicious
Persiflage
-
-
Tending to kill or hurt.
Banter.

Modicum
Mollify
-
-
A small or token amount.
To soothe.
Perspicacity
Perturbation
eer -
-
Acuteness or discernment.
Mental excitement or confusion.

Mordant
Moribund
-
-
Biting.
On the point of dying.
Petulant
Phlegmatic
Pique ing
-
-
-
Displaying impatience.
Not easily roused to feeling or action.
To excite a slight degree of anger in.
Morose
Multifarious
-
-
Gloomy.
Having great diversity or variety.
Plea -
.ne
An argument (to obtain some desired
action).
Mundane

Munificent
Myriad
-

-
-
Worldly, as opposed to spiritual or
celestial.
Extraordinarily generous.
A vast indefinite number.
Plenary
Plethora
Poignant
Polyglot
-
-
-
-
Entire.
Excess; superabundance. t
Severely painful or acute to the spirit.
Speaking several tongues.
Nadir - The lowest point. Ponderous - Unusually weighty or forcible.
Nefarious - Wicked in the extreme. Portent - Anything that indicates what is to
Negligent - Apt to omit what ought to be done. happen.
Neophyte - Having the character of a beginner. Pragmatic - Practical(values), empirical
Noxious - Hurtful. Precarious - Critical, dangerous
Nugatory - Having no power or force. Preclude - To prevent.
Obdurate - Impassive to feelings of humanity or Precocious - Advanced (in development),
pity. overforward, premature
Obfuscate - To darken; to obscure. Predilection - Preference, partiality, inclination
Oblique - Slanting; said of lines. Predominate - To be chief in importance, quantity, or
degree.
Obstreperous - Boisterous.
Preposterous - (Very) Absurd, ridiculous
Odious - Hateful.
Prerogative - (Special) Right, privilege
Odium - A feeling of extreme repugnance, or of
Presage - To foretell.
dislike and disgust.

Downloaded From : www.EasyEngineering.net


Downloaded From : www.EasyEngineering.net

Vocabulary—Word Bank  l 163

Prescience - Knowledge of events before they take Raucous - Harsh.


place. Recalcitrant - Stubborn, refractory
Preternatural - Extraordinary. Recluse - One who lives in retirement or
Prim - Stiffly proper. seclusion.
Pristine - Primitive. Recondite - Incomprehensible to one of ordinary
Probity - Virtue or integrity tested and understanding.
confirmed. Recuperate - To recover.
Proclivity - A natural inclination. Redoubtable - Formidable.
Procrastination - Delay. Redundant - Wordy, repetitious, superfluous,
Prodigious - Large, immense needless
Profligate - Immoral, wanton, reckless, dissolute, Refractory - Not amenable to control.
licentious Regale - To give unusual pleasure.
Profuse - Produced or displayed in Regicide - The killing of a king or sovereign.
overabundance. Reiterate - To say or do again and again.
Prolix - Verbose. Relapse - To suffer a return of a disease after
Promiscuous
Propinquity ww -
-
Indiscriminate, impure, casual
Nearness, proximity Relegate -
partial recovery.
Assign a lower position, banish,
Propitious
Prosaic
Protagonist w.E -
-
-
Kindly disposed.
Unimaginative.
Leading character), hero(ine)
Repast
Repine
-
-
demote
A meal; figuratively, any refreshment.
To indulge in fretfulness and
Providential
Prudence
Puerile
-
-
-
Fortunate, lucky
Caution.
Childish.
asy Reprisal
Reprobate
-
-
faultfinding.
(Injury in) Return, retaliation, revenge
One abandoned to depravity and sin.
Pugnacious
Puissant
-
-
Quarrelsome.
Powerful, influential, mighty En Repudiate
Resilience
-
-
Disavow, disclaim
The power of springing back to a
Punctilious - Exact (in formalities), ceremonious,
conscientious gin
Resonance -
former position
Able to reinforce sound by sympathetic
Pungency - The quality of affecting the sense of
smell. Respite
eer -
vibrations.
Interval of rest.
Pusillanimous
Putrefy
Pyre
-
-
-
Without spirit or bravery.
Decay, rot, decompose
A heap of combustibles arranged for
Restive
Revere
Reverent ing
-
-
-
Resisting control.
To regard with worshipful veneration.
Humble.

Qualm -
burning a dead body.
A fit of nausea.
Rotund
Ruffian
-
- .ne
Round from fullness or plumpness.
A lawless or recklessly brutal fellow.
Quandary
Quibble
-
-
Doubt, dilemma, plight
An utterly trivial distinction or
objection.
Ruminate

Sagacious
Salacious
-

-
-
swallowed and regurgitated.
Wise, shrewd, astute
t
To chew over again, as food previously

Obscene, foul, indecent, lecherous


Quiescence - Being quiet, still, or at rest; inactive
Quirk - Twist, quibble, deviation Salubrious - Healthful; promoting health.
Quixotic - (Foolishly) Chivalrous, unrealistic, Salutary - (Morally) Healthy, salubrious,
whimsical beneficial
Rabble - Throng (of the vulgar), crowd, Sanguine - Ardent, confident, optimistic
proletariat Sardonic - Ironical, scornful, derisive
Rabid - Furious, mad, fanatical Satiate - Gratify (fully), surfeit, saturate
Raconteur - A person skilled in telling stories. Satyr - A very lascivious person.
Raillery - Jesting (language), banter, ridicule Savor - To perceive by taste or smell.
Ramify - To divide or subdivide into branches or Schism - Disjunction, split
subdivisions. Scribble - Hasty, careless writing.
Rampant - Rife, widespread Sedition - Plotting (against government),
Ramshackle - Dilapidated, tumbledown, rickety incitement, insurgence
Rapacious - Seize by force, avaricious Sedulous - Persevering in effort or endeavor.

Downloaded From : www.EasyEngineering.net


Downloaded From : www.EasyEngineering.net

164  l  Vocabulary—Word Bank

Severance - Separation. Travesty - A grotesque imitation.


Sinecure - Any position (having emoluments with Trenchant - Cutting deeply and quickly.
few or no duties). Trepidation - Nervous uncertainty of feeling.
Sinuous - Curving in and out. Trite - Made commonplace by frequent
Sluggard - A person habitually lazy or idle. repetition.
Solace - Comfort in grief, trouble, or calamity. Truculence - Ferocity.
Solvent - Having sufficient funds to pay all debts. Truculent - Having the character or the spirit of a
Somniferous - Tending to produce sleep.
savage.
Somnolent - Sleepy.
Turbid - In a state of turmoil; muddled
Soporific - Causing sleep; also, something that
Turgid - Swollen.
causes sleep. Turpitude - Depravity.
Sordid - Filthy, morally degraded Ubiquitous - Being present everywhere.
Specious - Plausible. Umbrage - A sense of injury.
Spurious - Not genuine. Unctuous - Oily.
Squalid - Having a dirty, mean, poverty-stricken

Stanch ww -
appearance.
To stop the flowing of; to check.
Undulate
Untoward
Upbraid
-
-
-
To move like a wave or in waves.
Causing annoyance or hindrance.
To reproach as deserving blame.
Stingy
Stolid w.E-
-
Cheap, unwilling to spend money.
Expressing no power of feeling or
perceiving.
Vagary
Vainglory
-
-
A sudden desire or action
Excessive, pretentious, and

Submerge
Subterfuge
-
- Evasion. asy
To place or plunge under water.
Valorous
Vapid
-
-
demonstrative vanity.
Courageous.
Having lost sparkling quality and
Succinct
Sumptuous
-
-
Concise.
Rich and costly.
En Variegated -
flavor.
Having marks or patches of different
Supercilious

Superfluous
-

-
Exhibiting haughty and careless
contempt.
Being more than is needed.
gin
Vehement -
colors; also, varied.
Very eager or urgent.
Supernumerary
Supersede
-
-
Superfluous.
To displace.
Venal
Veneer eer -
-
Mercenary, corrupt.
Outside show or elegance.
Supine
Supplicate
-
-
Lying on the back.
To beg.
Venial
ing
- That may be pardoned or forgiven, a
forgivable sin.
Suppress - To prevent from being disclosed or
punished.
Veracious
Veracity
-
- .ne
Habitually disposed to speak the truth.
Truthfulness.
Surcharge
Surfeit
Susceptibility
-
-
-
An additional amount charged.
To feed to fullness or to satiety.
A specific capability of feeling or
emotion.
Verbiage
Verbose
Verdant
Veritable
-
-
-
-
Wordy.
Green with vegetation.
Real; true; genuine.
t
Use of many words without necessity.

Taciturn - Disinclined to conversation. Vestige - (A visible) trace, mark, or impression


Taut - Stretched tight. (of something absent, lost, or gone).
Temerity - Foolhardy disregard of danger; Virago - Loud talkative women, strong statured
recklessness.
women
Terse - Pithy.
Virtu - Rare, curious, or beautiful quality.
Timorous - Lacking courage.
Visage - The face, countenance, or look of a
Torpid - Dull; sluggish; inactive.
Torrid - Excessively hot. person.
Tortuous - Abounding in irregular bends or turns. Vitiate - To contaminate.
Tractable - Easily led or controlled. Vituperate - To overwhelm with wordy abuse.
Transgress - To break a law. Vivify - To endue with life.
Transitory - Existing for a short time only. Vociferous - Making a loud outcry.
Travail - Hard or agonizing labor. Volatile - Changeable.

Downloaded From : www.EasyEngineering.net


Downloaded From : www.EasyEngineering.net

Vocabulary—Word Bank  l 165

Voluble - Having great fluency in speaking. Ambivalent - The word means simultaneous
Whimsical - Capricious. attraction towards and repulsion
Winsome - Attractive. from an object, person or action. The
attitude of educated Indians to love-
ONE WORD - A SMALL COLLECTION
marriages is ambivalent
Abdicate - Renounce a throne, high office or Anachronism - That which appears to be in the wrong
dignity period
Abolish - Do away with Anarchy - Lawlessness and disorder caused by
Accelerate - To move faster absence of control
Accomplice - One associated with another especially Anecdote - A short interesting or amusing story
in wrong-doing Anthology - A collection of poems or writings
Acoustics - Science of the production, Aphorism - (or maxim) A wise saying in a few
transmission, reception and effect of words
sound Aphrodisiac - A medicine causing sexual excitement
Acrobat
Adolescence ww -
-
One who performs gymnastic feats
The period of life from puberty to
maturity
Apiary
Apprentice
-
-
A place where bees are kept
A person who works under someone

Actuary

Adulterate
w.E -

-
One who calculates insurance and
annuity premium etc
Make impure by the addition of
Arboreal
Armistice
-
-
to learn that person’s skill
Those who live in trees
(or cease-fire or truce) An agreement

Aggression -
inferior substance
asy
Unprovoked attack of one country by
Ascetic -
to stop fighting
One who avoids physical pleasures and
comforts

Amnesty -
another
General pardon En Astrology - The art of understanding the influence
of heavenly bodies
Abattoir - A building where animals are killed for
meal (or slaughterhouse) gin
Astronomy
Aviary
-
-
Scientific study of heavenly bodies
A place for keeping birds
Ad hoc
Aeronautics
-
-
Made for a particular purpose
The science of the operation and flight
Backlog
eer - It means an accumulation of arrears.
Example: I must clear my heavy

Aesthete -
of aircraft
A person with a highly developed Backwater ing
-
backlog of arrears.
A Dart of a river out of the main

Agnostic -
sense of beauty
One who believes that nothing can be Barbarian - .ne
stream, where the water does not move
An uncivilised person

Agoraphobia
Alibi
-
-
known about God
Fear of open spaces
It is Latin for elsewhere. It is actually
Barbecue

Barometer
-

-
cooked over an open fire t
A metal flame on which meat etc. is

An instrument for measuring the air


pressure
a plea of having been elsewhere at the
time of commission of an act. But it Beverage - A liquid for drinking
is now used in the sense of an excuse. Bibliography - A list of writings on a subject
Example: He offered no alibi for his Biennial - Happening once every two years
absence from duty.
Bigamy - System of two marriages
Alimony - Compensatory allowance given to wife
after divorce Biodata - (or resume or curriculum vitae)
Allergic - Allergy means, a heightened sensitivity A short written account of one’s
to a substance as food, medicine etc. education and past employment
Allergic means having an aversion to. Black Hole - An area in outer space into which
everything near it, including light, is
He is allergic to hard work.
pulled-
Altruist - One who is habitually kind to others
Bleach - Make white or pale by mean~ of
Alumnus - A former student of a school or college chemicals or sunlight

Downloaded From : www.EasyEngineering.net


Downloaded From : www.EasyEngineering.net

166  l  Vocabulary—Word Bank

Blue Blood - The quality of being a noble person by Clock-and- - Stories that deal with adventure and
birth Dagger - exciting mystery
Blueprint - The word originated in the engineering Clot - A half-solid mass or lump formed from
industry where it means the final stage a liquid, especially blood
of paper design. So it may mean the Clubfoot - A badly-shaped foot twisted out of
final plan or layout. Example: The position from birth
blueprint of the Five-Year Plan is ready. Coagulate - Change from a liquid into a solid by
Bonsai - The art of growing a plant in a pot that chemical action
is prevented from reaching its natural Cold war - Severe political struggle between
size countries, without actual fighting
Bon Vivant - One who likes good wine and food and Colloquial - Suitable for ordinary, informal, or
cheerful companions familiar conversation
Bookworm - (or nerd) One who is too fond of Colonnade - A row of pillars supporting a roof or
reading and study arches
Botany - The scientific study of plants Coma - A state of long unnatural deep
Bottleneck
ww - It is a narrow passage, a place,
stage or condition that checks Combustible -
unconsciousness
(or Inflammable) That can catch fire
and burn easily

w.E progress. Example: We must rem


o”e all bottlenecks in the swift
implementation of policies.
Comrade - A close companion who shares difficult
work
Boulevard

Bourgeois
-

-
side
asy
A broad street having trees on each

Belonging to the middle class


Congenital
Connotation
-
-
Existing at or from one’s birth
The feeling or ideas that are suggested
by a word
Bric-a-brac - Small objects kept for decoration
En Consortium - A combination of several companies,
banks, etc. for a common purpose
Bullion
Bust
-
-
Bars of gold or silver
A piece of sculpture showing a person’s gin
Consul - A person appointed by a govt. to
protect and help its citizens and its

Cabal -
head, shoulders, and upper chest
A small group of people who make
secret plans for political action
Contemporary eer -
interests in trade in a foreign city
A person living at the same time as

Calligraphy
Canine
-
-
The art of beautiful writing by hand
Of a dog
Contretemps
ing
-
another
An unlucky and unexpected event,
socially uncomfortable
Cannibal - One who eats human flesh
Corinthian -
.ne
Typical of the most richly decorated
style of ancient Greek building
Cardiac
Catch-22
-
-
Connected with the heart
A situation from which one is
prevented from escaping by something
that is part of the situation itself
Corrigendum

Counterfeit
-

-
t
Something (to be) made correct in a
printed book
Made exactly like something real in
order to deceive
Celestial - Of the sky or heaven Countervailing - Acting with equal force but opposite
Cerebral - Connected with the brain effect
Couture - The business of making and selling
Chalet - A wooden house with a steeply sloping
fashionable women’s clothes
roof
Cradle - A small bed for a baby
Charlatan - One who deceives others by falsely
claiming to have a skill Creativity - The ability to produce new and original
Calmistry - The art of telling one’s character or ideas
future by examining one’s hands Criminology - The scientific study of crime and
Celibacy - One who does not indulge in carnal criminals
pleasure Crossroads - A point at which an important decision
Clean sweep - A complete victory must be taken
Cruise - A sea voyage for pleasure

Downloaded From : www.EasyEngineering.net


Downloaded From : www.EasyEngineering.net

Vocabulary—Word Bank  l 167

Cuisine - A style of cooking Empirical - Based on practical experience of the


Daredevil - One who is prepared to take dangerous world we see and feel
risks Enigmatic - That which is mysterious and very hard
D-day - A day on which an important to understand
operation is to begin Entomology - The scientific study of insects
Debacle - A sudden complete failure Epic - A long narrative poem
Decanter - A container for holding alcoholic Epicurean - Lover of physical/material
drinks, especially wine Ergonomics - The study of the conditions in which
Defeatism - The practice of thinking in a way people work most effectively with
that shows an expectation of being machines
unsuccessful Estuary - The wide lower part or mouth of a
Deficit - The amount by which something is less river
than what is needed Evaporate - To change into steam and disappear
Déja vu - The feeling of remembering something
Evolution - Gradual development from simpler

Depression
ww -
that in fact one is experiencing for the
first time
A long period of seriously
Excise -
forms
Tax on goods produced and used

Designate
w.E
-
reduced business activity and high
unemployment
Chosen for an office but not yet
Expletive -
inside a country
An often meaningless word used for
swearing

Disarmament - asy
officially placed in it
Reduction of weapons by a
Expressionism - A style of painting which expresses
feelings rather than describing objects
and experiences
Dissection -
government
Cutting up the body of a plant or
En Extrovert - One who likes to spend time with
others
Dividend -
animal for studying
The money which is divided among
shareholders gin
Facet - Any of the many flat sides of a cut
jewel
Dome
Dormitory
-
-
A rounded roof on a building
A large room containing a number of
Faeces
eer - The solid waste material passed from
the bowels

Down-and-out -
beds
One who is suffering from lack of
Fallacy
Farce
ing
-
-
A false idea or belief
A light humorous play full of silly
things happening
money, work, etc, and is unable to
change the situation Farrier -
.ne
One who makes and fits shoes for
horses
Dragnet

Dregs
-

-
A system of connected actions and
methods for catching criminals
Sediment in a liquid that sinks to the,
bottom and is thrown away
Febrile
Felony
-
-
Of or caused by fever
t
A serious crime such as murder or
armed robbery
Drudgery - Hard uninteresting work Fiance - (feminine fiancée) The person one is
going to marry
Dutch - Of the Netherlands (Holland)
Filament - A thin thread
Eagle-eyed - Looking with very keen attention and
noticing small details Flogging - Severe beating with a whip or stick
Eaves - The edges of a roof which come out Flora - All the plants of a particular place,
beyond the walls country, or period
Eddy - A circular movement of water, wind, Fluvial - Of, found in, or produced by rivers
dust, etc. Foible - A small rather strange and stupid
Elastic - Able to spring back into shape after personal habit
being stretched Foolscap - A large size of paper, especially writing
Electrocute - To kill by passing electricity through paper
the body Foray - A sudden rush into enemy country
Embargo - An official order forbidding trade with
another country

Downloaded From : www.EasyEngineering.net


Downloaded From : www.EasyEngineering.net

168  l  Vocabulary—Word Bank

Foreman - A skilled and experienced worker in Hangar - A big building where aircraft are kept
charge of other workers between flights
Fortnight - Two weeks Harpoon - A spear with a long rope, used for
Fourth Estate - Newspapers and their writers, hunting large sea animals
considered with regard to their Haven - A place of calm and safety
political influence Headgear - A covering for the head
Freckle - A small flat brown spot on the skin Headstrong - Determined to do what one wants in
Freight - Goods carried by ship, train. plane, etc. spite of all advice
Frill - A decorative edge to a piece of material Heat-stroke - Fever and weakness caused by too
much heat
Frontispiece - A picture or photograph at the
Heirloom - A valuable object passed on for
beginning of a book
generations
Fumigate - To clear of disease, bacteria etc. by
Herbivore - A plant-eating animal
means of chemical smoke
Furrow - A long narrow track cut by a plough Hide - An animal’s skin, when removed to be

Galaxy
Gastronomy
ww -
-
A large group of stars
The art and science of cooking and
Hinterland -
used for leather
The inner part of a country

Gelatine w.E-
eating good food
A clear substance used for making
jellies
Histrionics

Holocaust
-

-
Behaviour resembling a theatrical
performance
Great destruction and the loss of many

Geocentric
Gigolo
-
- asy
Having the Earth as the central point
A man who is paid to be a woman’s
Holster -
lives
A leather holder for a pistol

Glacier -
lover
En
A mass of ice moving very slowly down
Hooligan

Hothead
-

-
A noisy rough person who causes
trouble
One who does things too quickly,

Glut -
a mountain valley
A larger supply than is necessary gin
Hub -
without thinking
The central part of a wheel
Goatee - A little pointed beard on the bottom of
the chin
Hump
Ideology
eer-
-
A lump on the back of a camel
A set of ideas on which a political or
Go-Getter - One who is forceful, determined, and
likely to succeed in getting what one
wants Idolatry ing
-
economic system is based
The worship of idols
Good Samaritan - One who helps others in trouble,
without thinking of oneself
Illegible
Immortal
-
-
.ne
Difficult or impossible to read
That which will never die
Gorge
Graffiti
Grange
-
-
-
A deep narrow valley with steep sides
Drawings or writing on a wall
A large country house with Farm
Implacable

Improvident
-

-
less angry
t
Impossible to satisfy, change, or make

One who does not save for the future


buildings Incarnate - In physical form rather than in the
Green Belt - A stretch of land, around a town or form of a spirit or idea
city, where building is not allowed, so Incorporeal - Without a body
that fields, woods, etc. remain
Grunt - Short deep rough sound of a pig Inedible - Not suitable for eating

Gubernatorial - Of a governor Inflate - To fill until swelled with air or gas

Guinea pig - A person who is subject of some kind Ingest - To take food into the stomach
of test Innate - That which one is born with
Halitosis - A condition in which one has bad Inseminate - To put male seed into a female
breath
Intelligentsia - Those who are highly educated and
Handbook - ‘A short book giving all the most
often concern themselves with ideas
important information about a subject
and new developments

Downloaded From : www.EasyEngineering.net


Downloaded From : www.EasyEngineering.net

Vocabulary—Word Bank  l 169

Intestate - Not having made a will Machete - A knife with a broad heavy blade
Invective - A forceful attacking speech used for Magnum Opus - A great work of art
blaming someone Malady - That which is wrong with a system
Invoice - A list of goods supplied, stating
Malaise - A feeling of pain without any particular
quantity and price
pain or appearance of disease
Irreproachable - So good that no blame at all could be
Malcontent - One who is dissatisfied with the
given
existing state of affairs.
Journal - A serious magazine produced by a
Male Chauvinist - A man who believes that men are
specialist society
better than women
Junta - A council or assembly that deliberates
Malign - To speak evil of, especially to do so
in secret upon the affairs of
falsely and severely.
government.
Mane - The long hair on the back of a horse’s
Juxtapose - To place side by side or close together
neck
Kennel - A small hut for a dog Manual - A book giving information about how
Kimono
Knuckle ww -
-
A long loose garment made of silk
The joint between the finger and the Market Forces -
to do something
The free operation of business and
trade without govt. controls
Lackey
w.E-
hand
One who behaves like a servant by
always obeying
Mascot

Massacre
-

-
Chosen as a symbol and thought to
bring good luck
The unnecessary and indiscriminate
Lead Time

Lecher
-

-
asy
The time taken in planning and
producing a new product
One who continually looks for sexual Materialism -
killing of human beings.
Too great interest on in money &

Leonine -
pleasure
Of or like a lion En material etc, rather than spiritual
matters

Levee - An embankment beside a river or


stream or an arm of the sea, to prevent gin
Mechanics - The science of the action of forces on
objects

Levy -
overflow.
An official demand and collection,
Megalomania

Mercantile eer -

-
The belief that one is more important
than one really is
Of trade and business

Libertarian -
especially of a tax
One who believes that people should
have freedom of expression
Meteorology
ing
- The scientific study of weather
conditions
Lien - A legal claim or hold on property, as
security for a debt or charge.
Midriff -
.ne
The part of the human body between
the chest and the waist
Limerick
Linchpin
-
-
A humorous short poem with five lines
An important member which keeps
the whole group together
Militia

Miniature
-

-
belonging to a regular army
A very small painting
t
Those trained as soldiers but not

Literati - People with great knowledge of Mirage - The appearing of objects which are not
literature really there
Livery - Uniform of a special type for servants Misnomer - A name wrongly or mistakenly applied.

Locale - A place where something particular Moccasin - A simple shoe made of leather
happens Modus Operandi - A method of doing something typical
Logger V - One whose job is to cut down trees of someone
Loom - A machine on which thread is woven Mogul - A person of very great power. wealth,
into cloth and importance
Lore - Old beliefs, not written down, about a Monarchy - Rule by a king or queen
particular subject Monomaniac - One who keeps thinking of one
Lowbrow - One who has no interest in literature, particular idea only
art, etc. Moralistic - Having unchanging narrow ideas
Lullaby - A pleasant song used for causing about right and wrong
children to sleep

Downloaded From : www.EasyEngineering.net


Downloaded From : www.EasyEngineering.net

170  l  Vocabulary—Word Bank

Morbid - Caused by or denoting a diseased or Research machines


unsound condition of body or mind. Opprobrium - The state of being scornfully
Motto - A few words taken as the guiding reproached or accused of evil.
principle Orderly - A soldier who attends an officer
Multinational - A company having operations in many
Ornithology - The scientific study of birds
different countries
Mundane means - Worldly/earthly Ostentation - A display dictated by vanity and
intended to invite applause or flattery.
Mycology - The study of fungi (plural of fungus)
Ostracism - Exclusion from intercourse or favor, as
Namesake - A person with the same name as yours in society or politics.
is your Outcast - One who is forced from one’s home or
Nautical - Of sailors, ships, or sailing without friends
Necromancy - The practice which claims to learn Overhaul - Thorough examination and repair if
about the future by talking with the necessary
dead Pacemaker - A small machine that regularises
Nemesis
Newfangled ww -
-
Just and unavoidable punishment
New (idea. machine etc) but neither Palaeography -
heartbeats
The study of ancient writing

Nihilism
w.E
-
necessary nor better
The belief that nothing has meaning or
value
Panacea

Panache
-

-
A remedy or medicine proposed for or
professing to cure all diseases.
An admirably stylish manner of doing
Nodding
Acquaintance - asy
A very slight familiarity Panegyric -
things effortlessly
A formal and elaborate eulogy, written
or spoken, of a person or of an act.
Nosegay -
or worn on a dress
En
A small bunch of flowers, to be carried
Pariah
Parricide
-
-
One who is not accepted by society
Act of murdering one’s father
Notary -

No-win Situation -
A public official who makes written
statements official
That which will end badly whichever
gin
Parting Shot - A last remark made at the moment of
leaving

Nursery -
choice one makes
A place where small children are taken
Passive Smoking
eer - The breathing in of smoke from the
cigarettes that others are smoking

Oar -
care of
A long pole used for rowing a boat
Patent
ing
- The right to make or sell a new
invention
Obstetrics - The branch of medicine concerned
with childbirth
Paunch
Peanuts
-
- .ne
A man’s fat stomach
Too small a sum of money
Obtrude

Obtrusive
-

-
To be pushed or to push oneself into
undue prominence.
Tending to be pushed or to push
oneself into undue prominence.
Peeping Tom

Penance
-

-
they are undressing t
One who secretly looks at others when

Making oneself willingly suffer for


one’s wrongs
Obviate - To clear away or provide for, as an Perdition - Everlasting punishment after death
objection or difficulty.
Perjury - A lie told on purpose
Odoriferous - Having a smell, especially a pleasant
one Persona on Grata - One who is not acceptable or welcome
Off-White - White with some grey or yellow Petrology - The scientific study of rocks
Oligarchy - A collective government formed by a Phonetics - The study and science of speech sounds
few persons
Phylum - A main division of animals or plants
One-Upmanship - The art of getting an advantage over
others without actually cheating Pigment - The natural colouring matter of plants
Ontology - The branch of philosophy concerned and animals
with the nature of existence Pillion - A seat for a second person on a
Operational - The study of how best to build and use motorcycle
Pithead - The entrance to a coalmine

Downloaded From : www.EasyEngineering.net


Downloaded From : www.EasyEngineering.net

Vocabulary—Word Bank  l 171

Placate - To bring from a state of angry or Postscript (or - A note added at the end of a letter
hostile feeling to one of patience or P.S.)
friendliness. Pulmonary - Of or having an effect on the lungs
Plaintiff - One who brings a charge against Punctilious - Strictly observant of the rules or forms
someone in a court prescribed by law or custom.
Platitude - A written or spoken statement that is Punter - One who makes a bet on horserace
flat, dull, or commonplace. results
Platonic - A friendly, not sexual, relationship Pus - A thick yellowish liquid produced in
between a man and a woman an infected wound
Plebeian - Of the lower social classes Putsch - A sudden secretly planned attempt to
Poetaster - A writer of bad poems remove a govt. by force
Poker Face - A face that shows nothing of what one Quartet - Four singers or musicians performing
is thinking or feeling together
Porcine - Of or like a pig Quixotic - Trying to do the impossible, often so
as to help others, while getting oneself
Pork
Portend ww
-
-
Meat from pigs
To indicate as being about to happen, Raconteur -
into danger
One who is good at telling stories in an

Post-Haste
Pot-Boiler
-
-
w.Eespecially by previous signs.
In a great hurry
A book of low quality produced
Raillery
Ranger
-
-
interesting way
Friendly joking al someone’s weakness
The keeper of a forest

Powder Keg - asy


quickly to get money
Something dangerous that might
Real Estate - Property in the form of land and
houses

Précis -
explode

En
A shortened form of a piece of writing
Real Politick - Politics based on practical facts rather
than on moral aims
Prescient

Prevaricate
-

-
Able to imagine or guess what will
probably happen
To use ambiguous or evasive language
gin
Rebuff - A peremptory or unexpected rejection
of advances or approaches.

for the purpose of deceiving or


diverting attention.
Recant

eer- To withdraw formally one’s belief (in


something previously believed or
maintained).
Prey - An animal that is hunted and eaten by
another
Recumbent
Red-Handed ing
-
-
Lying down on the back or side
In the act of doing something wrong
Prima Donna - The main woman singer in an opera
company Redundant -
.ne
means exceeding what is natural, usual
or necessary. The idea of too much is
Prodigal

Profile
Projection
-

-
-
One wasteful or extravagant, especially
in the use of money or property.
A side view of someone’s head
Something that sticks out from a
Reflation -
t
inseparable from it. It is not a synonym
of inappropriate.
A govt. policy of increasing the amount
of money
surface
Relic - Something old that reminds us of the
Propellant - An explosive for firing a bullet or a past
rocket
Renaissance - A renewal of interest in some
Protagonist - means first actor in a play. It means one particular kind of art, literature, etc.
who takes the leading part in a drama,
Renal - Of the kidneys
novel or any other sphere. Clearly the
word is not an antonym of antagonist Requisition - An official demand or request
which means one who contends with Rescind - To make void, as an act, by the
another. enacting authority or a superior
Proscribe - To reject, as a teaching or a practice, authority.
with condemnation or denunciation. Resonance - Sound produced in one object by
Prosody - The rules by which the patterns of sound waves from another
sounds are arranged in poetry Retribution - A severe deserved punishment

Downloaded From : www.EasyEngineering.net


Downloaded From : www.EasyEngineering.net

172  l  Vocabulary—Word Bank

Revisionism - The questioning of the main beliefs of Shaman - A priest believed to have magical
an already existing political system powers and cure people
Rhyme - To end with the same sound, including Shibboleth - A once-important custom which no
a vowel longer has much meaning
Right-hand Man - One’s most useful and valuable helper Shoot - A new growth from a plant
Ringleader - One who leads others to do wrong or Short-change - To give back less than what actually
make trouble should be given back
Riviera - A warm stretch of coast, popular with Siamese twins - Those joined together from birth at
holiday makers some part of their bodies
Rodent - A small herbivore with strong sharp Side Effect - An unwanted effect happening in
addition to the intended one
long front teeth
Signatory - Any of those who sign an agreement
Rolling Stone - One who.travels around a lot and has
no fixed address or responsibilities Sill - The flat piece at the base of a window
Rosary - A string of beads used for counting Singsong - A repeated rising and falling of the
prayers
Roving eye
ww - Sexual interests that pass quickly from
one person to another
Skyscraper
Sleeping Partner
-
-
voice in speaking
A very tall modern city building
A partner in a business who takes no
Rubber Stamp

Ruling
w.E -

-
One who acts only to make official the
decisions already made by another
An official decision of a court
Slip-up
Small Fry
-
-
active part
A slight unintentional mistake.
A young or unimportant person
Rung -
steps of a ladder asy
Any of the cross-bars that form the
Smokestack - The tall chimney of a factory or a ship
Saboteur
Salve
-
-
One who practices sabotage
(or Ointment) An oily substance forEn Snippet

Socialite
-

-
A short piece from something spoken
or written
A person well known for going to
Sapient -
putting on a cut, wound, etc.
Wise and full of deep knowledge
gin
Sociology -
fashionable parties
The scientific study of societies and
Scaffold - A structure built up from poles and
boards for workmen to stand on Solidarity eer -
human behaviour in groups
Loyal agreement of interests, aims, or
Scalp
Sceptical
-
-
The skin on the top of the human head
Unwilling to believe a claim or
promise.
Somnambulism
ing
-
principles among a group
The habit of walking about while asleep

Scraps - Pieces of food not eaten at a meal, and


thrown away
SOS

Souvenir
-

- .ne
An urgent message from someone in
trouble
An object kept as a reminder of
Scuba

Seam
-

-
An instrument used for breathing
while swimming underwater
A line of stitches ‘joining two pieces of
Spatial
Spectacle
-
-
something
Connected with space
A grand public show or scene
t
cloth, leather, etc.
Spindle - A machine part round which
Sedentary - Done while sitting down
something turns
Seer - One who can see into the future Splinter - A small sharp-pointed piece of wood
Seismic - Of or caused by earthquakes broken off something
Sportsmanship - A spirit of honest fair play
Seller’s Market - Where there are not many goods for
sale Sprig - A small end of a stem or branch with
Sensationalism - The intentional producing of leaves
excitement or shock Stallion - A fully-grown male horse kept for
breeding
Septic - Infected with disease bacteria
Standard-bearer - An important leader in a moral
Sexagenarian - One who is between 60 and 69 years argument or movement
old Statesman - A political leader who is respected as
Sexism - The belief that one sex is not as good as being wise, honourable, and fair-
the other minded

Downloaded From : www.EasyEngineering.net


Downloaded From : www.EasyEngineering.net

Vocabulary—Word Bank  l 173

Stellar - Of the stars Territorial waters - The sea near a country’s coast over
Sticking Point - Something that prevents an agreement which it has legal control
Testamentary - Of or done according to a will
Stock Broker - One whose job is buying and selling
shares and debentures for others Thatch - Roof covering of straw, reeds, etc.
Stoic - One who is indifferent to joys/sorrows Thermal - Of heat
Stooge - One who habitually does what another Thorax - The part between the neck and the
person wants abdomen
Stratagem - A trick to deceive an enemy Thromhosis - Having a clot in a blood tube or the
Strategist - A person skilled in planning, especially heart
of military movements. Topiary - The art of cutting trees into decorative
Stride - A long step in walking. shapes
Touchstone - Something used as a test or standard
Strobe Light - A light which goes on and off very
quickly Tract - A short piece dealing with a religious
or moral subject
Subcutaneous
Sub Judice
ww -
-
Beneath the skin
A legal case being considered in court
Traitor
Transient
-
-
One who is disloyal to one’s country
One who or that which is only of
Subsidy

Superannuated w.E
-

-
Money paid by the govt. to make prices
lower
Too old for work
Transmogrify
Transpire
-
-
temporary existence.
To change completely as if by magic
It does not mean to happen or occur
Surety

Surreal
-

-
asy
One who takes responsibility for the
behaviour of someone
Having a strange dreamlike unreal
but to become known. Example: It
transpired at the meeting that he was

Swarm -
quality
En
A large group of insects moving in a
Treatise -
going to be our next President.
A serious book or article that examines

Sweet Tooth -
mass
A liking for sweet and sugary things gin
Tribunal -
a particular subject.
A court of people officially appointed
to deal with special matters.
Sword of - Something bad that may happen at any
time
Troglodyte
Trousseau
eer-
-
One who lives in a cave.
The personal possessions that a woman
Damocles
Tactile - Of the sense of touch Tunnel Vision ing
-
brings with her when she marries.
A condition in which one can see only
Take-home Pay - Wages left after all taxes, pension
payments, etc. have been paid Turf -
straight ahead
.ne
A surface made up of earth and a thick
Tannery

Tarot
-

-
A Place where animal skin is made into
leather
A set of 22 cards used for telling the
future
Tutelage

Tyrant
-

-
covering of grass

under instruction.
t
The act of training or the state of being

A ruler with complete power, who


Tautology - is redundancy, which consists of rules cruelly and unjustly
needless repetition of meaning in Tyro - One slightly skilled in or acquainted
other words. Example: audible to the with any trade or profession.
ear return back, One after another in Underling - A person of low rank in relation to
succession. another
Taxonomy - The system of putting plants and Undermanned - Not having enough workers
animals into various classes Unguent - A thick oily substance used on the skin
Technocrat - A highly skilled specialist in charge of
Unisex - Of one type used by both male and
organisation
female
Teller - One who is employed to receive and
Upholstery - A comfortable covering and filling for
pay out money in a bank
a seat
Tenure - The act, right, or period of holding
Valise - A small bag used while travelling
land or a job

Downloaded From : www.EasyEngineering.net


Downloaded From : www.EasyEngineering.net

174  l  Vocabulary—Word Bank

Vase - A decorative container used to put Wheeler-dealer - One who is skilled at making profitable
flowers in or successful deals
Vendor - A seller of small articles that can be Whirlpool - A place with circular currents of water,
carried about which can pull objects down into it
Vertebrate - A living creature which has a backbone Wholesale - The business of selling goods to
Vicissitude - A change, especially a complete shopkeepers
change, of condition or circumstances, Wit - The ability to say clever and amusing
as of fortune. things
Vinous - Of wine Wizard - One who has magic powers

Voluntary - Done willingly, without being forced Word Blindness - (or dyslexia) Difficulty in seeing the
difference between letter shape
Wade - To walk through water
Workaholic - One who likes to work too hard
Walkout - Leaving a meeting as an expression of
Working - Enough practical knowledge to do
disapproval
knowledge something
Wardrobe - A large cupboard in which one hangs

Wasteland ww -
up clothes
Empty, unproductive, usually ugly land
Wreckage
Wretch
-
-
The broken parts of a destroyed thing
An unfortunate or unhappy person

Waterloo

Weakling
w.E
-

-
A severe defeat after a time of unusual
success
One who lacks physical strength or
Xenophobia

Yeoman service
-

-
Fear of strange or foreign people,
customs, etc.
Great and loyal service, help, or

Wean - asy
strength of character
To transfer (the young) from Yuppie -
support
A young person in a professional job
with a high-income

Westher-beaten -
dependence on mother’s milk to
another form of nourishment.
Marked or damaged by the force ofEn Zeitgeist - The intellectual and moral tendencies
that characterize any age or epoch.
wind. sun, rain. etc.
gin
Zoology - The scientific study of animals

eer
ing
.ne
t

Downloaded From : www.EasyEngineering.net


Downloaded From : www.EasyEngineering.net

12
PTER
CHA

Word Power

SYNONYMS

ww
Nothing different from the previous chapter, except that this time around you have to look for the synonym instead of the antonym
of the given word.
How to Solve
w.E
Again, the format is not that different. You are given a sentence or a paragraph about a word which is highlighted. If you have not seen
the word before or don’t know the meaning of it, just read the sentence and try to figure it out for yourself. After you have done this,

asy
from the options, choose the word that is an exact or close synonym of this highlighted word.
Examples

En
Direction for Examples1 to 5: For each of the words below, a contextual usage is provided. Pick the word from the alternatives
given, that is closest in meaning in the given context.
Example 1.

(a) Harsh criticism (b) Acute distrust


gin
Opprobrium: The police officer appears oblivious to the opprobrium generated by his blatantly partisan conduct.
(c) Bitter enmity (d) Stark oppressiveness
Explanation 
eer
(a); Opprobrium refers to the state of extreme dishonour or a state of disgrace from public abuse. This word
can be considered as an opposite to “getting attention from doing something good” i.e. bad behaviour results in opprobrium. Op-

ing
probrium is synonymous with censure, which means harsh criticism. The given statement means that the police offer seemed to
lack awareness of the opprobrium he had created by conducting himself in a biased manner. Therefore, option (a) is right.
Example 2.
Portend: It appears to many that the US “war on terrorism” portends trouble in the Gulf.
(a) Introduces (b) Evokes (c) Spells (d) Bodes.ne
Explanation 
t
(d); Portend means “be a sign of something to come, esp. something important or bad”, which is closest in
meaning to ‘Bodes’. Option (a) and option (b) are not foretelling and hence can be discarded. Option (c) is a close one, but can be
discarded because it carries a ‘certainty’ with it, which is not the case with ‘bodes’.
Example 3.
Prevaricate: When a videotape of her meeting was played back to her and she was asked to explain her presence there, she started
prevaricating.
(a) Speaking evasively (b) Speaking violently (c) Lying furiously (d) Throwing a tantrum
Explanation  (a); Prevaricate means “be deliberately ambiguous or unclear in order to mislead or withhold information”.
Thus, option (a) seems the best choice.
Example 4.
Restive: The crowd became restive when the minister failed to appear even by 10 p.m.
(a) Violent (b) Angry (c) Restless (d) Distressed
Explanation  (c); Restive means ‘being in a tense state’. It is an adjective and has a connotation that the noun/pronoun that it
would be describing would be restless or not at ease. Restive implies anxiousness and hence option (c) is the best choice.
Example 5.
Ostensible: Manohar’s ostensible job was to guard the building at night.
(a) Apparent (b) Blatant (c) Ostentatious (d) Insidious

Downloaded From : www.EasyEngineering.net


Downloaded From : www.EasyEngineering.net

176  l  Word Power

Explanation (a); Ostensible means ‘appearing as such but not necessarily so’. The closest option has to be option (a).
Directions for Questions 6 to 9:  Each of the following questions has a paragraph with one italicized word that does not make sense.
Choose the most appropriate replacement for that word from the options given below the paragraph.
Example 6.
Intelligent design derives from an early 19th-century explanation of the natural world given by an English clergyman, William
Paley. Paley was the popularizer of the famous watchmaker analogy. Proponents of intelligent design are crupping Paley’s
argument with a new gloss from molecular biology.
(a) destroying (b) testing (c) resurrecting (d) questioning
Explanation  (c); You have to understand the paragraph to be able to answer this one correctly. The proponents of intelligent
design theory (which comes from the works of Paley) will not be ‘destroying’ or ‘testing’ or ‘questioning’ Paley’s argument with a
new gloss (an outward or token appearance or form).
Example 7.
Women squat, heads covered, beside huge piles of limp fodder and blunk oil lamps, and just about all the cows in the three towns
converge upon this spot. Sinners, supplicants and yes, even scallywags hand over a few coins for a crack at redemption and a
handful of grass.
(a) shining (b) bright (c) sputtering (d) effulgent

ww
Explanation  (c); Going with the tone of the paragraph, we can see that only some adjective that has a negative connotation
can fit in the blank. This is true only of option (c), sputtering (the noise of something spattering or sputtering).
Example 8.

Explanation 
w.E
It is klang to a sensitive traveller who walks through this great town, when he sees the streets, the roads, and cabin doors crowded
with beggars, mostly women, followed by three, four, or six children, all in rags and importuning every passenger for alms.
(a) amusing (b) irritating (c) disgusting (d) distressing
(c); The degree of anguish caused by the different scenes as described by the narrator, is very high for a sensi-

asy
tive traveller. Thus, the best option is option (c), disgusting.
Example 9.

(a) witty (b) rude En


Or there is the most fingummy diplomatic note on record: when Philip of Macedon wrote to the Spartans that, if he came within
their borders, he would leave not one stone of their city, they wrote back the one word - “If ’.
(c) simple (d) terse
Explanation 
gin
(d); The incident clearly speaks volumes about the laconicism (brief expressions/speech) of the Spartans. The
correct answer is option (d).
ANTONYMS
Simple. Get out there and find the antonym for the highlighted word. eer
How to Solve
ing
The question format is designed in such a manner that a word is highlighted and is used in a sentence to give you a contextual aid,

.ne
just in case you do not know the word. This means that if you have not seen the word before or don’t know the meaning of it, just
read the sentence and try to figure it out for yourself. After you have done this, from the options, choose the word that is an exact or
close antonym of this highlighted word.
Examples
t
Direction for Examples 10 to 14:  For each of the words below, a contextual usage is provided. Pick the word from the alternatives given
that is farthest in meaning in the given context.
Example 10.
Specious: A specious argument is not simply a false one but one that has the ring of truth.
(a) Deceitful (b) Fallacious (c) Credible (d) Deceptive
Explanation  (c); Specious means ‘plausible but false’, a specious argument for example, would be one which seems to be
good, true, logical or reasonable, but is not so. It is evident that such a statement, or something that is specious, cannot be even con-
sidered to be close to credible. Option (c) is right. A specious argument would be (a) deceitful, i.e. untrustworthy and fraudulent. It
would also be (b) fallacious, because that argument is false, even thought it might not seem so. It would be (c) deceptive too, as you
would feel or think that it is right/good and you will be deceived, because actually it is wrong/bad.
Example 11.
Obviate : The new mass transit system may obviate the need for the use of personal cars.
(a) Prevent (b) Forestall (c) Preclude (d) Bolster
Explanation  (d); Obviate means to prevent something from taking place, or to eliminate the requirement of something. The
given statement indicates that obviate refers to eliminate a need or requirement. It says that the mass transit system may eliminate
the need for personal vehicles. Option (d) is right, because bolster means increasing the amount, or supporting or strengthening,

Downloaded From : www.EasyEngineering.net


Downloaded From : www.EasyEngineering.net

Word Power  l 177

which is of course just the opposite of what obviate means. Option (a) is wrong, because it is expected that the mass transit system
may prevent the use of personal vehicles, so it is a similar word. Option (b) is wrong because forestall means to keep or prevent
something from happening and in the context, it would mean to prevent the need for using personal modes of transportation. Op-
tion (c) is wrong, because preclude can be considered a synonym of forestall or obviate.
Example 12.
Disuse: Some words fall into disuse as technology makes objects obsolete.
(a) Prevalent (b) Discarded (c) Obliterated (d) Unfashionable
Explanation  (a); Disuse is used to describe the state or condition of something that has not been used and is neglected,
which is exactly the context mentioned in the question. Option (a) is correct, because prevalent means ‘very common’, so that is
just the opposite of what disuse means. Option (b) is wrong, because discarded means ‘thrown away’ or neglected or not given
importance. Option (c) is wrong too, because obliterated is a similar word: to obliterate, means to remove or completely get rid of
something, which is also what disuse could mean. Option (d) is wrong too, because unfashionable means not in current use, i.e. not
according to the current fashion.
Example 13.
Parsimonious: The evidence was constructed from very parsimonious scraps of information.
(a) Frugal (b) Penurious (c) Thrifty (d) Altruistic
Explanation  (d); Even though ‘Altruistic’ (showing unselfish concern for the welfare of others) is not an exact antonym of

ww
‘parsimonious’ (excessively unwilling to spend), still, they seem close to being opposites where money is concerned. ‘Frugal’, ‘penu-
rious’ and ‘thrifty’ are similar to or are synonyms of the given word.
Example 14.

w.E
Facetious: When I suggested that war is a method of controlling population, my father remarked that I was being facetious.
(a) Jovian
Explanation 
(b) Jovial (c) Jocular (d) Joking
(a); Being facetious means being entertaining, or provoking laugher. In the given context, when the narrator

asy
suggests that wars are a way of controlling population, his father considers it a joke and terms it as a laughable thought. Option (a)
is right, because Jovian means something related or in accordance with a Roman deity, ‘Jupitar’, (the planet Jupiter). So, it is clearly
out of context here. Option (b) is wrong, because jovial can be used to describe someone who shows good humour, which is a simi-

En
lar word to the given one. Option (c) is wrong, because a jocular person is someone who makes a lot of jokes and is often happy.
Option (d) is wrong too because joking as explained already, is a similar word to the given one.

HOMONYMS
gin
Each of two or more words having the same spelling or pronunciation but different meanings and origins (e.g., pole and pole)

eer
1. Pole: A long, slender, rounded piece of wood or metal, typically used with one end placed in the ground as asupport for
something:a tent pole

northern and southern ends of the axis of rotation.


• Homo means Same
ing
2. Pole: Either of the two locations ( North Pole or South Pole) on the surface of the earth (or of a celestialobject) which are the

• Nym means Name


• Homonyms means same name .ne
• Pseudonym means Fake Name
We can define homonyms in other words that the words that sound the same or spelled the same but have different meanings.
Example: Bear/Bear or stalk/stalk, Left/left
Bear- A bear (animal) can bear (tolerate) very cold temperatures.
t
Left- The driver turned left (opposite to right) and left (departed from) the main road.
Fair- I went to the country Fair.
Fair- It is not fair that I can’t go to the party.
Fire: The flames of the fire glowed brightly.
Fire: The boss will fire the bad employee.
Pen: Pen the animals in the corral.
Pen: Write your name with a blue pen.
Stalk: The stem of plant.
Stalk: To follow, to track, to pursue.
That creepy stranger stalks the woman who leaves up theflower stalks in the park.
More Examples:
• I never wear a watch when I watch TV.
• The actors in the playwillplaychess.
• Scale the fish,then weigh it on thes cale.
• Shut your mouth so a fly doesn’tflyinto it!

Downloaded From : www.EasyEngineering.net


Downloaded From : www.EasyEngineering.net

178  l  Word Power

• Itireat the thought of changing atire.


• Don’t park the car near the park bench.
• Raise your right hand if you have the right answer.
Homonyms are often the cause of a spelling or word usage error.
Example:
• We accept (receive) a gift and we except (exclude) someone from a gift list.
• We can go to (a preposition) the store, tell someone we want to go too (also) and we can take two (a number) friends.
• Accept (to receive) and Except (excluding)
• Acts (things done) and Ax (chopping tool)
• Ad (advertisement) and Add (short for addition)
• Affect (to influence) and Effect (result)
• Aid (to assist) and Aide (an assistant)
• Air (stuff we breath) and Heir (one who will inherit)
• Aisle (walkway) and Isle (island)



ww
Allusion (an indirect reference) and Illusion (a misconception)
Ant (insect) and Aunt (parent’s sister)
Bald (hairless) and Bawled (cried aloud)

• w.E
Band (a group) and Banned (forbidden)
Capital (city) and Capitol (wealth and resources)



asy
Climactic (great intensity) and Climatic (weather conditions)
Die (to become dead) and Dye (colouring agent)
Elicit (to bring out) andIllicit (unlawful)

• En
Emigrate from (leave one country) and Immigrate to (enter another country)
Fair (even-handed) and Fare (payment)


Fairy (imaginary magic person) and Ferry (river-crossing boat)
Gilt (gold-plated) and Guilt (did wrong) gin


Gorilla (large ape) and Guerrilla (military soldier)
Knead (working bread dough) and Need (must have) eer


Mail (postal delivery) and Male (masculine person)
Principle (a basic truth) and Principal (head of a school/sum of money) ing


Scene (visual location) and Seen (past tense of saw)
Than (a comparison) and then (shows time) .ne



There (a place) and Their (belongs to them) and They’re (they are)
To (a preposition) and Too (an adverb) and Two (a number)
Your (possessive pronoun) and You’re (you are)
Homonyms have to be both a homograph and a homophone, or can it be just one or the other
t
HOMOPHONES
Some Homonyms may spell differently but sound alike, like one (the number) and won (having been victorious) are called homo-
phones.
Examples:
Air: I love fresh air.
Heir: The prince is the queen’s heir.
Pair: I washed my pair of Socks
Pear: I ate a pear for lunch.
Waste: Don’t waste the remaining paper.
Waist: Tie a belt around your waist.
Rain: We need rain to end the drought.
Reign: The queen has had a long reign.

Downloaded From : www.EasyEngineering.net


Downloaded From : www.EasyEngineering.net

Word Power  l 179

Night: It was a starry night.


Knight: The knight never used his sword.
To: I ride the bus to school.
Too: She does too!
Two: It is two o’clock.
Hair: My hair is a mess.
Hare: Have you read the story about the tortoise and the hare?
Meat: Vegetarians do not eat meat.
Meet: Where shall we meet for lunch?

HOMOGRAPHS
These words have the same spelling. They may or may not have the same sound and meaning.
Examples:
1. The words “lead” and “lead”
Lead us through the dark woods.

ww
The package was as heavy as lead.
2. The word “been”
I have been [bin] tired lately. (U.S. accent)

w.E
I have been [bean] tired lately. (Canadian accent)
More Examples:
1. Please close the door.
We sat close to each other.
2. I want to live in Paris one day. asy
He likes to fish with live bait.
3. I have such a fit (tantrum) En
When these words don’t fit (match)!
4. The wind blew the leaves away. gin
Wind up the toy and watch it go!
5. The white dove is a beautiful bird.
She dove into the swimming pool.
eer
6. The desert is sandy, hot, and dry.
Don’t desert a friend in need. ing
7. Like when all through the spring (season).
All the deer jump and spring (bounce) .ne
To remember:

Homophones
Same Sound
YES
Same spelling
MAYBE
Same meaning
MAYBE
t
air, heir, err No ® air, heir, err No ® air, heir, err
Yes ® a tire, to tire Yes ® gases, gasses
Homonyms YES YES NO
a scale, to scale a scale, to scale a scale, to scale
Homonyms MAYBE YES MAYBE
Yes à a tire, to tire a tire, to tire No à a tire, to tire
No à to lead, lead (metal) to lead, lead (metal) Yes à been (“bin v/s “bean”)
Some more examples of homonyms, homophones and homographs:
• Your, you’re • Here, Hear
• They’re, Their, There • Knew, New
• Too, To, Two • No, Know
• For, Four • Accept, Except
• By, Bye, Buy • Affect, Effect
• Who’s, Whose • All Ready, Already

Downloaded From : www.EasyEngineering.net


Downloaded From : www.EasyEngineering.net

180  l  Word Power

• Pair, Pare, Pear • Ewe/You


• Past, Passed • Flea/Flee
• Vary, Very • Flew/Flu/Flue
• Waist, Waste • Flower/Flour
• Ware, Wear, Where • For/Four/Fore
• Ad, Add • Foul/Fowl
• Dew, Do, Due • Frees/Freeze
• Right, Rite, Wright, Write • Not/Knot/Naught
• Praise, Prays, Preys • Gym/Jim
• Ware, Wear, Where • Grays/Graze
• Ewe, Yew, You • Ring/Wring
• Air, Err, Heir • Rose/Rows
• Aisle, I’ll, Isle • Tacks/Tax
• Hair, Hare • Tail/Tale
• Dough/Doze • Wax/Whacks
PARONYMS

ww
A paronym is just a word that’s derived from the same root as another word. Many errors in speech and writing are due to mix-ups
involving paronyms.
Definition: The words from same roots with similar pronunciations but different spellings and meanings are called Paronyms. Like:

beauteous w.E
sage/sagacious, just/justice, example/exemplary, proscribe/ prescribe, industrial/industrious, except/accept, affect/effect, beautiful/

Paronym isn’t widely used to mean “Cognate”, a word that is similar to a word in another language.
Example:
asy
a. Accept: To take or receive that which is offered
Except: excluding
En
Sentence: His mom must accept that her son likes all vegetables except for Cabbage.
b. Collision: noun – crash, clash, conflict
Collusion: noun – a secret agreement that is oftentimes illegal
gin
Sentence: The collision resulted from the collusion over traffic signs.
c. Alternatively: Use one, then the other, then the first again.
Alternately: Use one instead of the other. eer
Easily Confusing Paronyms
1. Allusion- an indirect reference, a hint ing
Illusion- a distortion of the senses, anything that seems to be something that it is not
Sentence: Your allusion to the illusion in the movie is most transparent.
.ne
2. Choose- To pick, to elect, to decide
Chose- simple past tense form of “to choose”
Choice- an option, a decision
Sentence: I chose to choose the ethical choice.
t
3. Complement (noun) - something that completes
Complement (verb) - to complete
Compliment (noun) - an expression of praise, congratulations, or encouragement
Compliment (verb) - to express a favorable opinion
Sentence: The compliment from my boss complemented the celebratory party.
4. Than- Used in comparisons
Then- at that time, next
Sentence- Then selects a cover larger than the couch.
5. Diffuse- to spread over
Defuse- to make less danger
Sentence: The SWAT team defused the bomb while the police diffused the crowd.
6. Decent- fair, good enough, okay
Descent- a way down, a drop to lower position
Dissent- to disagree, to differ from
Sentence: the experienced hiker dissented to taking the decent down the mountain

Downloaded From : www.EasyEngineering.net


Downloaded From : www.EasyEngineering.net

Word Power  l 181

7. Desert- a barren area of land with little to no precipitation


Dessert- a sweet confection served at the end of a meal
Sentence- We shall gorge on desserts once we leave this godforsaken desert

ODD ONE OUT


There are a lot of different varieties of vocabulary questions. A common type (incidentally, not very common on the CAT) is
‘finding the ip. Your job is to find the relationship, identify the word that doesn’t fit and mark the respective option as the answer.
Tips
If you are not able to form a connection with the meaning of words, working backwards from the options can be a good idea too
in this question type. You could negate words based on a certain characteristic that you are sure should not be there in the solution.
How to Solve
These questions are pretty basic, such that their format remains the same, and mostly there are never any tweaks or changes in
how these questions are asked. The best way to solve these questions is to just read all the given words, and try to form connections.
These connections could be of any type and any subject, the point is just to find something which connects the words. If you end
up connecting three words, and such a connection leaves out the fourth word, then you might have a right answer.

ww
There isn’t much theory to this, and this isn’t a topic that is asked often on difficult papers, hence it’s better to go to practice right away.
Examples
Example 15.

Explanation  w.E
(a) Sun (b) Moon
(d) Sun, moon and earth are all celestial bodies.
(c) Earth (d) Tides

Example 16.
(a) A
Explanation 
(b) E
(d) A, E, and U are vowels. asy (c) U (d) H

Example 17.
(a) Bike (b) Car En
(c) Boat (d) Truck
Explanation 
Example 18.
(c) Bike, Car and Truck are all land vehicles.

(a) Pray (b) Sing (c) Listen


gin (d) Talk
Explanation 
ties. eer
(c) When praying, singing or talking, we use our vocal capabilities. While listening, we use our hearing capabili-

Example 19.
(a) Understanding (b) Realizing (c) Seeing ing (d) Antipathy
Explanation 
.ne
(d) ‘Understanding’, ‘Realizing’ and ‘Seeing’ are all about gauging the real meaning of something, or about how
one can sympathize with others. Option (d) is contrary to these thoughts.

MAXIMUM FIT
As the name suggests, you have to select that option which fits or is appropriate for the most number of sentences/blanks. These
questions are also rarely asked, but you should still practice a few of these.
Let us see what these questions are all about:
t
Four sentences, generally from different contexts, are given and four options representing a word each are provided. Now, we
have to pick that option which is appropriate for, if not all, then the most number of given sentences.
How to solve
(1) Read all the given sentences while making sure you understand their context, or what they are trying to convey.
(2) Read all the given options, and try fitting the words in the different sentences. Note down how many blanks can the word (in
a particular option) fill.
(3) Choose that option which fits in the most sentences.
Tips
(1) The given options may have two or more words whose meanings are a bit similar. These can be tricky, so be careful.
(2) Make sure that you understand the meaning of the given words, because without that, it is a bit of a risk to attempt this type
of question.
(3) If the question looks complex, see if you can differentiate the given words on the basis of the different parts of speech they
may belong to. For example, they may be nouns, verbs, adjectives, etc. This division could help you find the right answer, but
do not forget that some words can act as both, verb and noun.

Downloaded From : www.EasyEngineering.net


Downloaded From : www.EasyEngineering.net

182  l  Word Power

Common mistakes
(A) Picking the option which looks appropriate for the sentence is not the right way to solve this type of question. You have to
make sure that the meaning or the definition of the word fits in appropriately with the given sentence.
(B) As we know, a word may have multiple meanings, and some students may disregard this while they eliminate an option. It is
a mistake which the question-creator will have in his/her mind while framing this question, so do not fall in this trap.
Examples
Example 20.
A. Professional studies has become the ___________ of the rich.
B. Every citizen has the ___________ to speak, travel and live as he pleases.
C. He has a definite ___________ over all his rivals.
D. Sheron no longer has the ___________ of the company’s bungalow and car.
(a) advantage (b) privilege
(c) right (d) concession
Explanation  (a); Advantage means ‘the quality of having a superior or more favourable position’. It can be considered as a
head start in a race. This word cannot be used in A, because ‘advantage of the rich’ does not make sense. It can be used in B. Using

ww
this in C makes sense, because it would mean that that person is in a more benefiting or favourable position compared to his rivals.
Its usage makes sense in D too, because then it would mean that Sheron no longer has the benefit of having the bungalow and the
car given by the company.

and D. w.E
Privilege means “a special advantage” and it is special in the sense that it is not enjoyed by all. It can be used in two sentences: A

Right can be used in only sentence B.

Example 21.
A. People want ___________ .
asy
Concession (a contract granting the right to operate a subsidiary business) cannot be used in any of the sentences.

En
B. A bad ___________ case had come in - a person with a smashed arm.
C. And then, without warning, ___________ struck.

(a) tragedy (b) accident gin


D. The dogs were the first to recognize the signs of oncoming ___________ .

(c) calamity
Explanation 
(d) order
(c); The correct words are written with the sentences:
People want (tragedy/order). eer
A bad (accident) case had come in - a person with a smashed arm.
And then, without warning, (calamity) struck. ing
The dogs were the first to recognize the signs of oncoming (calamity).
Calamity can be used in two sentences which is the most of any of these options.
Example 22. .ne
The men there have fought ___________ and emotional withdrawal, and were more capable of helping Jim.
A.
B. But ___________ does occasionally inflict all the adults.
C. A person who is deeply hurt feels very ___________
D. It is hard to survive this feeling of ___________
t
(a) dejected (b) lonely
(c) trouble (d) depression
Explanation  (d); The correct words are written with the sentences:
The men there have fought (depression) and emotional withdrawal, and were more capable of helping Jim.
But (depression/trouble) does occasionally inflict all the adults.
A person who is deeply hurt feels very (dejected/lonely).
It is hard to survive this feeling of (depression).
Example 23.
A. I have the power of ___________.
B. Down with a very high fever, he suffers from frequent fits of ___________.
C. They are now bitter enemies - all because of a small ___________.
D. Her ___________ is the most creative thing she has ever possessed.
(a) illusion (b) imagination
(c) hallucination (d) misunderstanding

Downloaded From : www.EasyEngineering.net


Downloaded From : www.EasyEngineering.net

Word Power  l 183

Explanation (b); The correct words are written with the sentences:


I have the power of (illusion/imagination).
Down with a very high fever, he suffers from frequent fits of (hallucination).
They are now bitter enemies - all because of a small (misunderstanding).
Her (imagination) is the most creative thing she has ever possessed.
Example 24.
A. Communism states that every individual must live for the ___________ .
B. The ___________ of the affairs of the nation is deplorable.
C. ___________ have been laid down by the United States : states The Statesman.
D. No ___________ has succeeded in gaining complete autonomy from the Federal government.
(a) state (b) nation
(c) government (d) condition
Explanation  (a); The correct words are written with the sentences:
Communism states that every individual must live for the (state/nation).
The (state/condition) of the affairs of the nation is deplorable.

ww
() have been laid down by the United States : states The Statesman. (Had option (d) been ‘conditions’, it could have fit in here, but
since this is not so, none of the options fit in this sentence.)
No (state) has succeeded in gaining complete autonomy from the Federal government.

w.E
asy
En
gin
eer
ing
.ne
t

Downloaded From : www.EasyEngineering.net


Downloaded From : www.EasyEngineering.net

184  l  Word Power

Practice Exercise
LEVEL-I
DIRECTIONS (Qs. 1 - 45):   In the following questions, each word 17. DENOUEMENT
in capital letters is followed by four words or phrases. Choose the one (a) Outcome (b) Eschew
which is similar in meaning to the word given in the, capital letters. (c) Action (d) Character
1. ACCOUTREMENTS 18. MANACLE
(a) Relatives (b) Companions (a) Fetter (b) Ornament
(c) Blemishes (d) Belongings (c) Spectacles (d) Order
2. APOLITICAL 19. FEIGN
(a) Antipolitical (b) Terrorist (a) Hesitate (b) Pretend
(c) Not interested in politics (d) Subversive (c) Deserve (d) Attend
3. AZURE 20. ASSAY

ww
(a) Blue
(c) Brown
4. BONHOMIE
(b) Green
(d) Round and big
(a) Attack
(c) Case
21. REVULSION
(b) Accept
(d) Evaluate

w.E
(a) Friendliness
(c) Very
5. CANDOUR
(b) Wrath
(d) Greed
(a) Apathy
(c) Disgust
22. GRIT
(a) Bold
(b) Violence
(d) Avenge

(b) Courage
(a) Enthusiasm
(c) Freedom
6. FLUCTUATION asy
(b) Openness
(d) Intimacy (c) Grease
23. SALACIOUS
(d) Level

(a) Increase
(c) Stability
(b) Change
(d) Decrease
En (a) Obscene
(c) Wholesome
24. CLOUT
(b) Wise
(d) Confident
7. BASHFUL
(a) Haughty
(c) Shy
(b) Discreet
(d) Upset gin (a) Fear
(c) Joke
(b) Claw
(d) Power
8. TRIVIAL
(a) Significant (b) Momentous
25. FOMENT
eer
(a) Instigate
(c) Frustrate
(b) Shield
(d) Waver
(c) Unimportant
9. RIVALRY
(a) Competition
(d) Critical

(b) Cooperation ing


26. REPERCUSSION
(a) Reaction (b) Acceptance
(c) Compromise
10. COLLAPSE
(a) Rise
(d) Campaign

(b) Failure
(c) Resistance
27. PROPITIOUS
(a) Favourable .ne
(d) Magnificence

(b) Similar
(c) Rebirth
11. OMINOUS
(a) Threatening
(c) Ubiquitous
(d) Debt

(b) Powerful
(d) Burdensome
(c) Humble
28. TALISMAN
(a) Fiction
(c) Aptitude
(d) Versatile

(b) Charm
(d) Strength
t
12. PROCRASTINATE 29. PENURY
(a) Multiply (b) Irregular (a) Destitution (b) Digestive
(c) Postpone (d) Predict (c) Pension (d) Vigour
13. HIATUS 30. AFFRONT
(a) Atrocious (b) Gap (a) Exile (b) Contour
(c) Dominance (d) Obscure (c) Eruption (d) Indignity
14. FOMENT 31. AMICABLE
(a) Vex (b) Waste (a) Nebulous (b) Abominable
(c) Renounce (d) Instigate (c) Harmonious (d) Delicate
15. VAGARY 32. DREGS
(a) Caprice (b) Emptiness (a) Cream (b) Power
(c) Enthusiasm (d) Truthfulness (c) Debris (d) Accoutrements
16. MACABRE 33. LOQUACIOUS
(a) Innocent (b) Tarried (a) Obstinate (b) Secondary
(c) Gruesome (d) Pleasing (c) Talkative (d) Competitive

Downloaded From : www.EasyEngineering.net


Downloaded From : www.EasyEngineering.net

Word Power  l 185

34. WAIVE 52. SUBLIME


(a) Restrict (b) Relax (a) Base (b) Concise
(c) Permit (d) Admit (c) Partial (d) Insist
35. ELAN 53. MANSION
(a) Flair (b) Spiritual (a) Castle (b) Hovel
(c) Inspiration (d) Boredom (c) House (d) Hotel
36. GAUCHE 54. PARTITION
(a) Vain (b) Rich (a) Maintain (b) Enjoin
(c) Polished (d) Tactless (c) Unify (d) Break
37. BROWBEAT 55. PRIMITIVE
(a) Ambitious (b) Challenging (a) Polite (b) Naive
(c) Intimidate (d) Tarnish (c) Weak (d) Sophisticated
38. IMPOSTURE 56. TERMAGANT
(a) Claim (b) Status (a) Charming (b) Turbulent
(c) Destruction (d) Deception (c) Brawling (d) Shrewish
39. PROBOSCIS 57. TRANSIENT
(a) Search (b) Probe

ww
(c) Snout
40. PARLEY
(d) Prove
(a) Transitory
(c) Permanent
58. REGRESSION
(b) Fleeting
(d) Momentary

w.E
(a) Discuss
(c) Sweeten
41. FURLOUGH
(a) Soldiers holiday
(b) Deliver
(d) Race

(b) Wild growth


(a) Reverse
(c) Regenerate
59. VENGEFUL
(a) Forgiving
(b) Relapse
(d) Retreat

(b) Revenge
(c) Wooden plough
42. PUNCTILIOUS asy
(d) Till (c) Vindictive
60. TIRADE
(d) Vicious

(a) Prude
(c) Meticulous
43. ENCOMIUM
(b) Wasteful
(d) Timid
En (a) Censure
(c) Discredit
(b) Declamation
(d) Eulogy

(a) Verve
(c) Doggerel
(b) Eulogy
(d) Force gin
61. SALUTATORY
(a) Sad
(c) Derivative
(b) Valedictory
(b) romising
44. INVIDIOUS
(a) Irritable
(c) Sinful
(b) Harsh
(d) Unpopular eer
62. TRACTABLE
(a) Irreligious (b) Incapable

45. LACHRYMOSE
(a) Terse (b) Mournful
63. CYNICAL
ing
(c) Unmanageable

(a) Trusting
(d) Unreal

(b) Gallant
(c) Indecent (d) Lecherous
DIRECTIONS (Qs. 46 - 90):   A word in capital letters is followed
by four words. Choose the word that is most nearly opposite in
(c) Brazen
64. GRISLY
.ne
(d) Tiresome

meaning to the word given in capital letters.


46. ELAN
(a) Brashness (b) Dignity
(a) Fragrant
(c) Pleasant
65. CADAVEROUS
(a) Skilful
(b) Haggard
(d) Shapely

(b) Obese
t
(c) Composure (d) Nervousness (c) Lucid (d) Inquisitive
47. IRASCIBLE 66. SOLICITIOUS
(a) Kind (b) Friendly (a) Concerned (b) Indifferent
(c) Roguish (d) Clever (c) Eager (d) Noisy
48. NEBULOUS 67. RAMPANT
(a) Concrete (b) Unkind (a) Furious (b) Stylish
(c) Undramatic (d) Uninteresting (c) Restrained (d) Healthy
49. PENCHANT 68. PROLIFIC
(a) Disinclination (b) Lone (a) Barren (b) Backward
(c) Directness (d) Lack of skill (c) Reckless (d) Profound
50. PERFUNCTORY 69. SEQUESTER
(a) Quick (b) Slow (a) Slump (b) Isolate
(c) Careful (d) Loud (c) Unify (d) Simple
51. TRANSIENT 70. TRITE
(a) Urgent (b) Youthful (a) Hackneyed (b) Correct
(c) Original (d) Eternal (c) Original (d) Certain

Downloaded From : www.EasyEngineering.net


Downloaded From : www.EasyEngineering.net

186  l  Word Power

71. COMPASSIONATE 90. EQUIVOCAL


(a) Indecisive (b) Unsympathetic (a) Mistaken (b) Quaint
(c) Unlawful (d) Untrustworthy (c) Clear (d) Universal
72. RESTIVE
DIRECTIONS (Qs. 91-105):   For the following questions, choose
(a) Buoyant (b) Placid
the option which does not belong to the group (ODD-ONE-OUT).
(c) Resistant (d) Insolent
73. GRADUAL 91. (a) dull (b) gloomy
(a) Energetic (b) Dynamic (c) omnipresent (d) boring
(c) Rapid (d) Enthusiastic 92. (a) archenemy (b) archrival
74. RUDIMENTARY (c) archetype (d) duplicity
(a) Developed (b) Polite 93. (a) dishonour (b) laud
(c) Pale (d) Weak (c) brand (d) stigmatise
75. SALVAGE 94. (a) disclaim (b) intimate
(a) Burn (b) Remove (c) hint (d) suggest
(c) Confuse (d) Lose 95. (a) allegory (b) parody
76. SEDENTARY (c) joke (d) lampoon
(a) Vivid (b) Afraid

ww
(c) Indolent
77. SCRIMP
(d) Active
96. (a) comedy
(c) tragedy
97. (a) flourish
(b) protagonist
(d) car
(b) burgeon

w.E
(a) Lavish
(c) Meticulous
78. DELECTABLE
(a) Agonising
(b) Parsimonious
(d) Polite

(b) Appetising
(c) nursery
98. (a) nemesis
(c) sham
(d) sprout
(b) pretense
(d) deception
(c) Distasteful
79. GRAVE asy
(d) Laborious 99. (a) bonus
(c) benevolent
1 00. (a) entice
(b) beneficial
(d) bond
(b) lure
(a) Noble
(c) Solemn
80. CONFIDANT
(b) Inconsequential
(d) Senile
En (c) whim
101. (a) grand
(d) beguile
(b) majestic
(a) Turncoat
(c) Confederate
(b) Arrogant
(d) Firm gin
(c) august
102. (a) mercurial
(d) raunchy
(b) mutable
81. AMALGAMATE
(a) Merge
(c) Impoverish
(b) Consecrate
(d) Split eer
(c) staunch
103. (a) bucolic
(c) bilious
(d) fickle
(b) choleric
(d) cantankerous
82. BOISTEROUS
(a) Serene (b) Tumultuous
104. (a) pelf
(c) lucre ing (b) moolah
(d) tantrums
(c) Brazen
83. ASSET
(a) Credibility
(d) Opaque

(b) Liability
1 05. (a) munch
(c) crush
.ne
(b) masticate
(d) revive
(c) Poverty
84. SMOTHER
(a) Repress
(c) Nurture
(d) Prosperity

(b) Cheer
(d) Irritate
t
DIRECTIONS (Qs. 106 -110): In each of the following questions
four words are given of which two words are most nearly the same
or opposite in meaning. Find the two words which are most nearly
the same or opposite in meaning.
85. REPRIMAND 106. (A) Expanded (B) Proclaimed
(a) Reward (b) Appreciate (C) Shrunk (D) Facilitated
(c) Encourage (d) Praise (a) A - D (b) B - D
86. TANGIBLE (c) C - D (d) B - C
(a) Gentle (b) Elusive (e) A - C
(c) Refined (d) Palpable 107. (A) Indelible (B) Erasable
87. ZOOM (C) Insignificant (D) Temporary
(a) Soothe (b) Plummet (a) A - C (b) C - B
(c) Subjugate (d) Refute (c) A - B (d) B - D
88. PREDILECTION (e) C - D
(a) Antipathy (b) Ignorance 108. (A) Intangible (B) Restless
(c) Dissonance (d) Disharmony (C) Vast (D) Meagre
89. DOCILE (a) C - A (b) C - D
(a) Unmanageable (b) Dutiful (c) C - B (d) B - A
(c) Submissive (d) Painful (e) B - D

Downloaded From : www.EasyEngineering.net


Downloaded From : www.EasyEngineering.net

Word Power  l 187

109. (A) Cutting (B) Establishing (a) A - D (b) A - B


(C) Transferring (D) Pruning (c) A - C (d) B - D
(a) A - B (b) C - D (e) C - D
(c) B - C (d) A - C 119. (A) Handy (B) Sparse
(e) A - D (C) Redundant (D) Exhausted
110. (A) Fixed (B) Stiff (a) A - C (b) B - C
(C) Indelible (D) Soapy (c) B - D (d) C - D
(a) A - B (b) A - D (e) A - B
(c) A - C (d) B - C 120. (A) Timid (B) Conceited
(e) C - D (C) Humane (D) Modest
DIRECTIONS (Qs. 111-115): In each of these questions, two of (a) A - C (b) B - D
the words are related in some way, i.e. they are similar or opposites. (c) B - C (d) A - D
Pick out the option which represents that pair. (e) C - D
111. (A) moderate (B) easy DIRECTIONS (Qs. 121-125) : In each of the following questions
(C) significant (D) strenuous four words are given, of which two words are most nearly the same
(a) B-D (b) A-B or opposite in meaning. Find the two words which are most nearly

ww
(c) A-C
(e) C-D
(d) B-C the same or opposite in meaning and indicate the number of the
correct letter combination.

w.E
112. (A) focus
(C) vital
(a) A-B
(B) trivial
(D) site
(b) B-D
121. (A) instigate
(C) construe
(a) A-C
(B) enquire
(D) interpret
(b) A-B
(c) A-C
(e) B-C
113. (A) defer
(d) C-D

asy
(B) dispute
(c) C-D
(e) A-D
122. (A) superficial
(d) B-D

(B) superfluous
(C) prefer
(a) B-C
(D) challenge
(b) A-C En (C) enlightened
(a) A-C
(D) surplus
(b) A-B
(c) B-A
(e) C-D
114.(A) Withstand
(d) B-D

(B) Climate
gin
(c) B-C
(e) A-D
123. (A) appalling
(d) B-D

(B) sinister
(C) Hot
(a) A - B
(D) Surrender
(b) B - C (a) A-B eer
(C) perturbed (D) threatening
(b) B-D
(c) A - D
(e) C - D
(d) B - D (c) A-C
(e) D-C ing (d) A-D

115. (A) Perky


(C) Honest
(a) A - B
(B) Lively
(D) Kind
(b) B - C
124. (A) imprison
(C) excruciate
(a) B-D .ne
(B) torture
(D) extract
(b) B-C
(c) C - D
(e) A - C
(d) B - D

DIRECTIONS (Qs. 116 -120): In each of the following questions


four words are given of which two words are most nearly the same
(c) A-B
(e) A-C
125. (A) pertinent
(C) irrelevant
(d) C-D

(B) impolite
(D) insecure
t
or opposite in meaning. Find the two words which are most nearly (a) A-C (b) B-D
the same or opposite in meaning. (c) C-D (d) A-D
116. (A) Fallacy (B) Adage (e) B-C
(C) Dictum (D) Endorse DIRECTIONS (Qs. 126-145 ): Select single word or phrase
(a) B - D (b) C - D which means most nearly the same as the given idiomatic phrase.
(c) B - C (d) A - D 126. An office with pay but little responsibility
(e) A - B (a) sinecure (b) presidency
117. (A) Elevate (B) Frugal (c) factotum (d) plutocracy
(C) Exult (D) Lament 127. A group of small ships
(a) C - D (b) A - B (a) archipelago (b) fleet
(c) B - C (d) B - D (c) flotilla (d) tugs
(e) A - D 128. A person in the habit of pointing out others faults
118. (A) Surreptitious (B) Taciturn (a) interloper (b) officious
(C) Exaggerate (D) Covert (c) gossamer (d) gadfly

Downloaded From : www.EasyEngineering.net


Downloaded From : www.EasyEngineering.net

188  l  Word Power

129. The study of handwriting as a guide to character DIRECTIONS (Qs. 146 -148) : Out of the four alternatives, choose
(a) forensic (b) graffiti the one which best expresses the meaning of the given word and mark
(c) hieroglyphics (d) graphology it in the Answer Sheet. [SSC CHSL 2012]
130. A long boring speech 146. Apprise :
(a) horology (b) harangue (a) Praise (b) Inform
(c) valediction (d) filibuster (c) Conceal (d) Assess
131. Accidental solving of a crime 147. Periodic :
(a) investigation (b) detection (a) Infrequent (b) Continuous
(c) sixth-sense (d) serendipity (c) Occasional (d) Regular
132. A commonplace and unoriginal statement 148. Gruesome :
(a) Sullen (b) Hideous
(a) witticism (b) preamble
(c) Exhausting (d) Insulting
(c) prevarication (d) platitude
133. Mutual loyalty among group members DIRECTIONS (Qs. 149 -151) : Choose the word opposite in
(a) etiquette (b) homogeneity meaning to the given word and mark it in the Answer sheet.
(c) espirit de corps (d) rendezvous [SSC CHSL 2012]

ww
134. Poetry that is silly
(a) observe (b) doggerel
149. Knack :


(a) Talent
(c) Dexterity
(b) Dullness
(d) Balance

w.E
(c) dirge
135. Inscription on a grave
(a) epigram
(d) limerick

(b) oration
150. Pernicious :
(a) Prolonged
(c) Ruthless
(b) Ruinous
(d) Beneficial
(c) obituary
136. A place where bees are kept
(a) aviary
asy
(d) epitaph

(b) artifice
151. Opulence :
(a) Luxury (b) Transparency
(c) diadem
137. Something short-lived
(d) apiary
En (c) Weath (d) Poverty

(a) ephemeral
(c) interim
(b) Epicurean
(d) illusory gin
DIRECTIONS (Qs. 152 -155) : Out of the four alternatives choose
the one which can be substituted for the given sentences.
[SSC CHSL 2012]
138. Interested only in money
(a) marmoreal (b) mordant eer
152. Someone who scientifically studies the birds:
(a) earthologist (b) orthopeadic
(c) mercenary
139. Scientific study of birds
(d) munificent
ing
(c) orthodondist (d) ornithologist
153. Something which is imagined to be real but actually does
(a) aviation
(c) microbiology
140. A very light and thin material
(b) ornithology
(d) supersonics
not exist.
(a) figment
.ne
(b) insight

(a) gossamer
(c) coterie
(b) gazelle
(d) poultice
141. Mercy-killing of patients who are incurably ill
(c) mirage
154. Someone having many skills:
(a) versatile
(c) cyclostyle
(d) shadow

(b) projectile
(d) anglophile
t
(a) circumspection (b) benignity 155. To officially take private property away to seize.
(c) euthanasia (d) apoplexy (a) offer (b) confiscate
142. General act of forgiveness on a national occasion (c) annex (d) hijack
(a) benediction (b) emancipation DIRECTIONS (Qs. 156 & 157): In the following questions out of
(c) investiture (d) amnesty the four alternatives, choose the one which best expresses the meaning
143. Passing off someone else’s writing as one’s own of the given word and mark it in the Answer Sheet.
(a) copying (b) patenting
(c) plagiarism (d) adaptation [SSC CHSL 2013]
144. A lake of sea water 156. Wily
(a) archipelago (b) lagoon (a) Angry (b) Wise
(c) gorge (d) inundation (c) Stupid (d) Cunning
145. The science of making watches 157. Temerity
(a) graphology (b) morphology (a) Paucity (b) Verity
(c) aneroid (d) horology (c) Audacity (d) Simplicity

Downloaded From : www.EasyEngineering.net


Downloaded From : www.EasyEngineering.net

Word Power  l 189

DIRECTIONS (Qs. 158 & 159): In the following questions choose 171. Veteran
the word opposite in meaning to the given word and mark it in the (a) Activist (b) Enthusiast
Answer Sheet. [SSC CHSL 2013] (c) Novice (d) Master
158. Insipid 172. Superfluous
(a) Tasty (b) Colourful (a) Essential (b) Excess
(c) Colourless (d) Dull (c) Unwanted (d) Necessary
159. Relinquish DIRECTIONS (Qs. 173-177): In the following questions out of the
(a) Relish (b) Continue four alternatives, choose the one which best expresses the meaning
(c) Vanish (d) Quench of the given word and mark it in the Answer Sheet.
DIRECTIONS (Qs. 160-162): In the following questions out of [SSC CHSL 2014]
the four alternatives choose the one which can be substituted for the 173. Persevere
(a) Fickle (b) Persist
given words /sentence. [SSC CHSL 2013]
(c) Constant (d) Polite
160. A former student of a school, college or university 174. Petition
(a) Alumnus (b) Genius (a) Rotation (b) Administration
(c) Scholar (d) Learner (c) Appeal (d) Vocation

ww
161. A building in which aircraft are housed
(a) Hangar
(c) Dockyard
(b) Granary
(d) Garage
175. Proposition
(a) Intimation (b) Protestation

w.E
162. A short story based on your personal experience
(a) Parable
(c) Anecdote
(b) Legend
(d) Fable
(c) Proposal
176. Vivacious
(a) Imaginary
(c) Perceptible
(d) Invitation

(b) Lively
(d) Languid

asy
DIRECTIONS (Qs. 163-167): In the following questions out of
the four alternatives choose the one which can be substituted for the
177. Sporadic
(a) Timely (b) Scattered
given words sentence [SSC CHSL 2014]
163. A poem of fourteen lines. En (c) Frequent (d) Irrelevant
DIRECTIONS (Qs. 178-182): In the following questions out of the
(a) Ballad
(c) Sonnet
(b) Psalm
(d) Carol
gin
four alternatives, choose the one which best expresses the meaning
of the given word. [SSC Sub Insp. 2012]
164. Incapable of error.


(a) Erroneous
(c) Unbeatable
(b) Incorrigible
(d) Infallible eer
178. Indiscriminate
(a) undifferentiated (b) instant

165. One who believes everything he or she hears.


(a) Credulous (b) Credible
179. Literal
ing
(c) sensible

(a) verbatim
(d) discreet

(b) formal
(c) Creditable (d) Credential
166. An allowance made to a wife by her husband, when they are (c) idealistic
180. Intricate .ne
(d) outdated
legally separated.
(a) Alimony
(c) Matrimony
167. Wild imagination.
(b) Parsimony
(d) Honorarium
(a) puzzling
(c) complicated
181. Expeditiously
(b) illusive
(d) assertive t
(a) Whim (b) Fantasy (a) rudely (b) gently
(c) Fancy (d) Memory (c) meekly (d) quickly
DIRECTIONS (Qs. 168-172): In the following questions choose 182. Idiosyncrasies
the word opposite in meaning to the given word and mark it in the (a) demands (b) needs
Answer Sheet. [SSC CHSL 2014] (c) ideologies (d) eccentricities
168. Equilibrium DIRECTIONS (Qs. 183-187): In the following questions out of
(a) Work out (b) Disturb the four alternatives, choose the one which can be substituted for
(c) Imbalance (d) Univenness the given words/sentence. [SSC Sub Insp. 2014]
169. Immortal 183. An act or notion to look back in the past
(a) Eternal (b) Permanent
(a) Retrospective (b) Postnatal
(c) Deathly (d) Temporary
(c) Retrogressive (d) Primitive
170. Focus
184. Medicine to counteract the effect of a poison
(a) Disappear (b) Disperse
(a) Emetic (b) Antidote
(c) Link (d) Layer
(c) Anti-venom (d) Antiseptic

Downloaded From : www.EasyEngineering.net


Downloaded From : www.EasyEngineering.net

190  l  Word Power

185. A collection of poems 200. Amorphous


(a) Pathology (b) Anthology (a) Amoral (b) Definite
(c) Oncology (d) Pedology (c) Perfect (d) Irregular
186. One who studies mankind 201. Unitary (a) Single
(a) Anthropologist (b) Physicist (a) Single (b) Triple
(c) Pathologist (d) Philanthropist (c) Multiple (d) Double
187. An opinion contrary to popular belief 202. Adulteration
(a) Paradox (b) Orthodoxy (a) Purification (b) Normalization
(c) Hearsay (d) Heresy (c) Rejuvenation (d) Consternation
DIRECTIONS (Qs. 188-192 ): In the following questions choose DIRECTIONS (Qs. 203-207 ): In the following questions out of the
the word opposite in meaning to the given word. four alternatives, choose the one which best expresses the meaning
of the given word and mark it in the Answer sheet.
[SSC Sub Insp. 2012] [SSC Sub Insp. 2014]
188. Resourcefulness 203. Scorn
(a) scarcity (b) stupidity (a) concise (b) despise
(c) incompetence (d) bankruptcy (c) bias (d) fierce

ww
189. Evolve
(a) withdraw
(c) suspend
(b) withhold
(d) stop
204. Catastrophe
(a) tragedy
(c) violence
(b) anger
(d) hatred

w.E
190. Antiquated
(a) renewed
(c) modern
(b) unique
(d) renovated
205. Abjure
(a) renounce
(c) pronounce
(b) announce
(d) denounce
191. Pathetic
(a) comic
(c) dramatic
asy
(b) ridiculous
(d) trivial
206. Assess
(a) overload
(c) permit
(b) measure
(d) enter
192. Fastidious
(a) ugly (b) shabby En 207. Elastic
(a) free (b) liberal
(c) shallow (d) discourteous
DIRECTIONS (Qs. 193-197) : In the following questions, out gin (c) flexible (d) broad
DIRECTIONS (QS. 208-212): In the following questions out of
the four alternatives, choose the one which can be substituted for the
of the four alternatives, choose the one which best expresses the
meaning of the given word.
193. Abnormal
[SSC Sub Insp. 2013]
eer
given words/sentences and indicate it by blackening the appropriate
oval [ ] in the Answer Sheet.


(a) Unnatural
(c) Unique
(b) Aggressive
(d) Informal ing
(a) Worker
[SSC Sub Insp. 2014]
208. One who offers his service of his own free will
(b) Slave
194. Venal
(a) Corrupt
(c) Legible
(b) Comprehensible
(d) Forgivable
(c) Volunteer
209. One who is always doubting .ne
(d) Servant

195. Conjurer
(a) Magician
(c) Performer
196. Invoice
(b) Jester
(d) Trickster
(a) Sceptic
(c) Rationalist
210. A collection of slaves
(a) Coffle
(b) Deist
(d) Positivist

(b) Crew
t
(c) Company (d) Cortege
(a) Word (b) Sound 211. A professional soldier hired to serve in a foreign army
(c) Statement (d) Language (a) Mercenary (b) Liquidator
197. Ameliorate (c) Venal (d) Hireling
(a) Improve (b) Degrade 212. Not likely to be easily pleased
(c) Motivate (d) Agree (a) Fastidious (b) Infallible
DIRECTIONS (Qs. 198-202) : In the following questions, choose (c) Fatalist (d) Communist
the word opposite in meaning to the given word. DIRECTIONS (Qs. 213-217): In the following questions choose
[SSC Sub Insp. 2013] the word opposite in meaning to the given word and mark it in
198. Debacle the Answer sheet. [SSC Sub Insp. 2014]
(a) Success (b) Response 213. Safe
(c) Acceptance (d) Agreement (a) rash (b) insecure
199. Abusive (c) beneficial (d) harsh
(a) Laudatory (b) Profuse 214. Redundant
(c) Effusive (d) Noble (a) repentant (b) surplus
(c) singular (d) required

Downloaded From : www.EasyEngineering.net


Downloaded From : www.EasyEngineering.net

Word Power  l 191

215. Fair 227. The study of plant life


(a) untrue (b) unjust (a) Geology (b) Zoology
(c) coarse (d) harsh (c) Botany (d) Geography
216. Boisterous 228. Exclusive possession or control of anything
(a) serenity (b) calm (a) Mono-mania (b) Monotheism
(c) cheerful (d) courageous (c) Monopoly (d) Monoism
217. Substantial DIRECTIONS (Qs. 229-231): In the following questions choose
(a) flimsy (b) hefty the word opposite in meaning to the given word and mark it in the
(c) actual (d) excess Answer Sheet. [SSC Multi tasking 2013]
DIRECTIONS (Qs. 218-220): In the following questions out of the 229. Predilection
four alternatives, choose the one which best expresses the meaning (a) Predicament (b) Afterthought
of the given word and mark it in the Answer Sheet. (c) Aversion (d) Postponement
[SSC Multi tasking 2013] 230. Pompous
218. Barbaric (a) Uppish (b) Humble

ww
(a) Thorny
(c) Premeditated
(b) Uncivilized
(d) Barber’s
(c) Meek
231. Serene
(d) Grandiose

219. Hurdle
w.E
(a) Suspicion
(c) Opposition
(b) Throw
(d) Obstacle
(a) Calm
(c) Ruffled
(b) Angry
(d) Bitter
DIRECTIONS (Qs. 232-238): In the following question out of the
220. Deter
(a) To hinder asy
(b) To neglect
four alternatives, choose the one which can be substituted for the
given words/sentences and indicate it by blackening the appropriate
(c) To disapprove (d) To differ
DIRECTIONS (Qs. 221 - 223) : Choose the word opposite in En oval [•] in the Answer Sheet.
[SSC Multi tasking 2013]
meaning to the given word and mark it in the Answer Sheet.
[SSC Multi tasking 2013] gin
232. Belief in many gods
(a) pantheism (b) monotheism
221. Indolent
(a) Solvent (b) Diligent

eer
(c) polytheism
233. A cluster of flowers on a branch
(d) atheism

(c) Malovolent
222. Coherent
(d) Brilliant
(c) wreath ing
(a) bouquet (b) inflorescence
(d) incandescence
(a) Distorted
(c) Inept
(b) Disorganized
(d) Carefree actions .ne
234. A person who believes that only selfishness motivates human

223. Brutal
(a) Adamant
(c) Fearless
(b) Humane
(d) Criminal
(a) agnostic
(c) sceptic
235. A highly skilled musician
(b) cynic
(d) misogynist t
DIRECTIONS (Qs. 224 - 228) : In the following questions, out of (a) artiste (b) virtuoso
the four alternatives choose the one which can be substituted for the (c) performer (d) diva
given words / sentence. [SSC Multi tasking 2013] 236. A method of boiling briefly to cook food slightly
224. One who is unable to pay one’s debt (a) steam (b) bake
(a) Borrower (b) Bankrupt (c) saute (d) parboil
(c) Bank-roll (d) Extravagant 237. The group, especially in the arts, regarded as being the most
225. Instrument that magnifies objects experimental
(a) Microscope (b) Periscope (a) avant-garde (b) iconoclast
(c) Stethoscope (d) Telescope (c) revolutionary (d) nerd
226. Animals which live in water 238. One who helps people by giving them money or other aid
(a) Barren (b) Wild (a) benefactor (b) beneficiary
(c) Domestic (d) Aquatic (c) tycoon (d) patriot

Downloaded From : www.EasyEngineering.net


Downloaded From : www.EasyEngineering.net

192  l  Word Power

LEVEL-II
DIRECTIONS (Qs. 1-10):   Each question below consists of a 15. VERACIOUS
word printed in capitals, followed by four small lettered words or (a) False (b) Varied
phrases. Choose two small lettered words or phrases that are most (c) Image (d) Truthful
nearly the same in meaning to the word in capitals. 16. PERTURB
1. SHAM (a) Stipulate (b) Turn around
(a) feign (b) spread (c) Disturb greatly (d) Compatible
(c) mellow (d) hypocrisy 17. ACQUIESCENCE
2. WILE (a) Quiet Submission (b) Great Diligence
(a) during (b) guile (c) Intense Confrontation (d) Subtle Variation
(c) entice (d) guilt 18. WHEEDLE
3. FIAT (a) Emaciated (b) Flattery
(a) decree (b) edict (c) Purge (d) Scant

4. SECT
ww
(c) auto

(a) crowd
(d) design

(b) belief
19. IMMISERATION
(a) Immigration
(c) Impoverishment
(b) Immersion
(d) Discrimination

5. RANK w.E
(c) faction

(a) bottom
(d) party

(b) complete
20. BEATITUDE
(a) Accursed
(c) Retreated
(b) Cleansed
(d) Blessed
(c) lever
6. AVER asy
(d) flagrant 21. BEDIZEN
(a) Nomadic Arab (b) Gaudily Dressed
(a) avoid
(c) avow
(b) attest
(d) deny
En (c) Honest Citizen
22. CACHINNATE
(d) Tumultuous Event

7. CANT
(a) jargon
(c) argot
(b) contradiction
(d) talk
gin (a) Sing Aloud
(c) Laugh Aloud
(b) Speak Aloud
(d) Cry Aloud

8. DOLE
(a) senator (b) put
23. CANOODLE

eer
(a) Cuddle
(c) Brazen
(b) Canonise
(d) Articulate
(c) distribute
9. MIRE
(d) parcel (out) 24. TUMESCENT
(a) Shrink ing (b) Annoy
(a) bog
(c) briar
(b) mush
(d) entangle
(c) Engorge
25. TWADDLE .ne
(d) Enlighten

10. TILT
(a) charge
(c) fall
(b) incline
(d) admire
DIRECTONS (Qs. 11-40)   Choose the word which best expresses
(a) Smart
(c) Obscene
26. ENNUI
(b) Indecisive
(d) Waffle t
(a) Lassitude (b) Confront
the closest meaning of the given word. (c) Enthuse (d) Entrap
11. FRUGALITY 27. LOW-KEY
(a) Foolishness (b) Extremity (a) official (b) secret
(c) Enthusiasm (d) Economy (c) subdued (d) complicated
12. HARBINGER 28. STIPULATION
(a) Massenger (b) Steward (a) imitation (b) signal
(c) Forerunner (d) Pilot (c) excitement (d) requirement
13. EXODUS 29. ANTITHESIS
(a) Genius (b) Uninteresting person (a) fixed dislike (b) musical response
(c) Worm (d) Arthropod (c) lack of feeling (d) direct opposite
14. EQUANIMITY 30. ENTRENCHED
(a) Resentment (b) Dubiousness (a) filled up (b) fortified
(c) Duplicity (d) Excitement (c) followed by (d) kept down

Downloaded From : www.EasyEngineering.net


Downloaded From : www.EasyEngineering.net

Word Power  l 193

31. AMENABLE 48. CANTANKEROUS


(a) religious (b) masculine (a) Irascible (b) Co-operative
(c) proud (d) agreeable (c) Adamant (d) Captivate
32. AFFLUENT 49. EMBLAZON
(a) neutral (b) sentimental (a) Extol (b) Cinder
(c) wealthy (d) handsome (c) Embalm (d) Subtle
33. COUNTERPART 50. INVEIGH
(a) hindrance (b) peace offering (a) Harangue (b) Celebrate
(c) password (d) complimentary (c) Endorse (d) Neglect
34. DISPARAGE 51. LEAVEN
(a) separate (b) compare (a) Static (b) Transform
(c) refuse (d) belittle (c) Coherent (d) Diffuse
35. INTREPID 52. OPPROBRIUM
(a) middle (b) tolerant (a) Ignominy (b) Opportunity
(c) fearless (d) rude (c) Obituary (d) Honour
36. GRANDIOSE 53. PARSIMONIOUS

ww
(a) imposing
(c) boring
37. PARLEY
(b) unpretentious
(d) lanky
(a) Extravagant
(c) Partial
(b) Penurious
(d) Passionate

w.E
(a) fraud
(c) conclave
38. PASSE
(b) paraphrase
(d) spectacle
54. INSIDIOUS
(a) Deceitful
(c) Insincere
(b) Apparent
(d) Tepid

(a) rude
(c) modern (d) chicasy
(b) old-fashioned
55. RAPACIOUS
(a) Avaricious
(c) Decorated
(b) Satiated
(d) Subconscious
39. UTOPIA
(a) holiday home (b) music
En 56. SOPORIFIC
(a) Somnolent (b) Unromantic
(c) vacant
40. SIMPLETON
(a) dunce
(d) perfect state

(b) tattler gin (c) Alert


57. UBIQUITOUS
(a) Universal
(d) Slumber

(b) Rare
(c) genius (d) quack
DIRECTIONS for (Qs. 41- 80) :  A word in capital letters eer
(c) Ensnare
58. PUISSANCE
(d) Conscientious

is followed by four words. Choose the word that is most nearly


opposite in meaning to the word given in capital letters.
(a) ignorance
(c) impotenceing (b) approbation
(d) repudiation
41. ZOOM
(a) Soothe
(c) Subjugate
(b) Plummet
(d) Refute
59. RECONDITE
(a) miniature
(c) arable .ne
(b) philosopher
(d) obvious
42. PREDILECTION
(a) Antipathy
(c) Dissonance
(b) Ignorance
(d) Disharmony
60. RESTITUTION
(a) inflation
(c) deprivation
61. SCAD
(b) cataclysm
(d) constitution
t
43. DOCILE
(a) Unmanageable (b) Dutiful (a) Allocation (b) Restraint
(c) Submissive (d) Painful (c) Dearth (d) Provision
44. EQUIVOCAL 62. LIMBER
(a) Mistaken (b) Quaint (a) Orderly (b) Groomy
(c) Clear (d) Universal (c) Sturdy (d) Stiff
45. PROSAIC 63. OBLIQUITY
(a) Pensive (b) Imaginative (a) Straightforwardness (b) Conformity
(c) Rhetorical (d) Pacified (c) Praise (d) Self-righteousness
46. LEVITY 64. SPUNK
(a) Praise (b) Blame (a) Success (b) Loss of prestige
(c) Solemnity (d) Frivolity (c) Lack of intelligence (d) Timidity
47. OBLOQUY 65. STILTED
(a) Praise (b) Cruel (a) Tenative (b) Informal
(c) Slander (d) Dialogue (c) Verbose (d) Senseless

Downloaded From : www.EasyEngineering.net


Downloaded From : www.EasyEngineering.net

194  l  Word Power

66. PREVARICATE DIRECTIONS for (Q. 91-98)   Find the odd one out from the
(a) State truthfully (b) Postulate group of words.
(c) Emphasise (d) Consider thoughtfully 91. (a) Bludecon (b) Drazon
67. BILK (c) Blackjack (d) Order
(a) Reduce in size (b) Make famous 92. (a) temporal (b) ephemeral
(c) Renovate (d) Pay in full (c) transient (d) eternal
68. CAVIL 93. (a) abominable (b) tempestuous
(a) Discern (b) Disclose (c) abhorrent (d) detestable
(c) Introduce (d) Commend 94. (a) absolve (b) exonerate
69. AMUSING (c) exculpate (d) disburden
(a) silent (b) later 95. (a) captivate (b) enchant
(c) boring (d) nice (c) fascinate (d) dazzle
70. AMBIGUITY 96. (a) reprimand (b) dissipate
(a) lucidity (b) basal (c) chastise (d) castigate
(c) lovable (d) necessity 97. (a) reprehend (b) censure
71. AMASS (c) sprawl (d) chide
(a) demote (b) remote 98. (a) defraud (b) cheat

ww
(c) scatter
72. ECSTASY
(a) anguish
(d) better

(b) appeal
(c) swindle (d) allure
DIRECTIONS (Qs. 99-118): Select single word or phrase which

w.E
(c) amenable
73. ANTIDOTE
(a) medicine
(d) lucidity

(b) poison
means most nearly the same as the given idiomatic phrase.
99. The right to vote
(a) ballot (b) electorate
(c) anodyne
74. HAUGHTINESS
(a) unskilled
asy
(d) amity

(b) affability
(c) franchise (d) graffiti
100. A person who believes he is always ill
(a) maverick (b) metaphysics
(c) adduce
75. APPLAUD
(d) abject
En (c) neophyte (d) hypochondriac
101. One who assails traditional beliefs
(a) placate
(c) order
76. DISARRAY
(b) denounce
(d) conserve
gin (a) critic
(c) anthropologist
(b) atheist
(d) iconoclast
102. A person who chooses to live in a foreign country
(a) disorder
(c) bore
(b) order
(d) store eer
(a) traveller
(c) immigrant
(b) explorer
(d) refugee
103. An official inquiry into the cause of an unnatural death
77. BADGER
(a) bad
(c) pacisty
(b) sober
(d) assure
(a) inquest
ing
(c) post-mortem
(b) investigation
(d) instigation
78. BENIGN
(a) benevolent (b) malevolent
104. A massive moving entity
(a) miasma
(c) jamboree .ne
(b) juggernaut
(d) typhoon
(c) blessing
79. BLASPHEMOUS
(a) irreligious
(c) reverent
(d) curse

(b) inferior
(d) blarney
105. A happening that cannot be altered
(a) invincible
(c) fait-accompli
(b) fate
(d) congruence
106. A person with good artistic taste
t
80. PARADOXICAL
(a) rational (b) crazy (a) gourmet (b) gourmand
(c) daft (d) zany (c) connoisseur (d) judge
107. A flirting woman
DIRECTIONS for (Qs. 81-90):   Match each of the words on the (a) virago (b) shrew
left with one of those on the right that is most nearly opposite in (c) martinet (d) coquette
meaning. 108. Forcefully enlisting private property for military use
81. whelp (a) alien (a) occupation (b) commandeering
82. hirsute (b) judicious (c) trespass (d) encroachment
83. denizen (c) parent 109. A block of individual apartments
84. immutable (d) to antagonise (a) flats (b) congruence
85. conciliate (e) to purify
(c) architecture (d) condominium
86. asinine (f) to welcome
110. A contraption used during French Revolution for beheading
87. duplicity (g) variable
88. adulterate (h) honesty people
89. exacerbate (i) hairless (a) guinea (b) gabardine
90. spurn (j) to mollify, appease (c) guillotine (d) icicle

Downloaded From : www.EasyEngineering.net


Downloaded From : www.EasyEngineering.net

Word Power  l 195

111. An imaginary but a perfect society 125. A. Lugubrious B. Cheerful


(a) idealism (b) cornucopia C. Welter D. Gaffe
(c) utopia (d) plethora (a) B-D (b) A-B
112. Genocide based on race or religion (c) C-D (d) B-D
(a) apartheid (b) segregation 126. A. Paltry B. Oriental
(c) animism (d) pogrom C. Occidental D. Obnoxious
113. Theft of someone else’s writings (a) A-D (b) B-D
(a) facsimile (b) plagiarism (c) B-C (d) C-D
(c) forgery (d) parody 127. A. Exorbitant B. Moderate
114. A gentle breeze C. Ephemeral D. Mammoth
(a) south wind (b) squall (a) A-B (b) C-D
(c) draft (d) zephyr (c) B-C (d) A-D
115. Next month 128. A. Peccable B. Lassitude
(a) ultimo (b) instant C. Jeopardy D. Perfect
(c) praximo (d) pronto (a) B-C (b) A-D

ww
116. Fire works display for amusement
(a) glitter (b) crackers
(c) C-D
129. A. Gaucherie
C. Eulogise
(d) B-D
B. Dexterity
D. Dearth

w.E
(c) pyrotechnics
117. Interesting narrator of stories
(a) novelist
(d) sparklers

(b) broadcaster
(a) B-C
(c) A-B
130. A. Fallacious
(b) C-D
(d) B-D
B. Pusillanimity
(c) raconteur
118. Exact and methodical thinking
(a) non sequitur
asy
(d) orator

(b) ratiocination
C. Fortitude
(a) B-C
D. Fallible
(b) A-B

(c) rationalism (d) psychiatry


DIRECTIONS (Qs. 119-148) : In each of these questions fourEn (c) C-D
131. A. Incarcerate
(d) A-D
B. confine

words are given marked a, b, c and d. Two of these words are most
nearly the same are opposite in meaning. Identify those two words: gin C. Impeccable
(a) A-B
(c) B-D
D. Felicitate
(b) C-D
(d) B-C
119. A. Umpteen
C. Countless
B. Waspish
D. Testimony eer
132. A. Dis interested
C. Emulate
B. comprehensible
D. Impartial
(a) A-C
(c) C-D
(b) B-C
(d) A-B
(a) C-D
(c) A-D ing (b) B-C
(d) C-A
120. A. Buffoon
C. Candid
(a) A-D
B. Scholar
D. Gullible
(b) B-C
133. A. Inamorata
C. Imbecile
.ne
B. Beloved
D. Novice

(c) C-D
121. A. Pandemonium
C. Sporadic
(d) A-B
B. Realm
D. Tranquility
(a) A-C
(c) C-D
134. A. Adumbrate
C. Zany
(b) A-B
(d) B-C
B. Overshadow
D. Vulpine
t
(a) A-B (b) B-C (a) A-B (c) B-C
(c) C-D (d) A-D (c) C-D (d) D-A
122. A. Fidgety B. Elation 135. A. Cantankerous B. Querulous
C. Nervous D. Exuberant C. Amicable D. Occult
(a) B-C (b) A-B (a) B-D (b) A-A
(c) B-D (d) A-C (c) A-C (d) B-C
123. A. Oblivious B. Kudos 136. A. Jouisance B. Joyousness
C. Conscious D. Adulation C. Betrothed D. Usurp
(a) A-B (b) B-C (a) A-B (b) B-C
(c) B-D (d) C-D (c) C-D (d) B-D
124. A. Iconoclast B. Critic 137. a hunky-dory B. Choleric
C. Luscious D. Naive C. Shoddy D. Enervate
(a) A-C (b) A-B (a) B-C (b) C-D
(c) B-C (d) B-D (c) A-C (d) D-A

Downloaded From : www.EasyEngineering.net


Downloaded From : www.EasyEngineering.net

196  l  Word Power

138. A. Certitude B. Hoipolli DIRECTIONS (Qs. 151 & 152) :   In each of these questions, four
C. Certainty D. Hostile words are given marked as A, B, C and D two of these words are
(a) C-B (b) A-B most nearly the same or opposite in meaning. Identify those two
(c) A-C (d) B-D words:
139. A. Hood wink B. Sincere 1 51. (A) Enthralling (B) Respecting
C. Gauche D. Fiasco (C) Projecting (D) Alluring
(a) A-B (b) B-C (a) A - B (b) C - D
(c) B-D (d) A-C (c) B - C (d) A - D
140. A. Flummox B. Enlighten 1 52. (A) Swoop (B) perturb
C. Fastidious D. Esoteric (C) Plump (D) Boil
(a) A-B (b) B-C (a) A - D (b) A - C
(c) D-C (d) A-D (c) B - C (d) B - D
141. A. Exorcise B. Bedevil DIRECTIONS (Qs. 153 & 160) :   In each of the following
C. Diatribe D. Hyperbole questions, a word is given for which a usage is provided. Out of the
(a) A-C (b) B-D given alternatives, choose the one which best expresses the meaning
(c) D-A

ww
142. A. Draconian
C. Bliss
(d) A-B
B. Cruel
D. Noble
of the given word.
153. AUTOCHTHONOUS : The Dravidians were the autochtho-
nous population of India before the arrival of Aryans.
(a) A-C
(c) A-B
w.E
143. A. Deify
(b) B-D
(d) C-D
B. Blasphemy
(a) original
(c) indigenous
(b) majority
(d) important
154. EGREGIOUS : A generation ago, parents would not tolerate
C. Deluge
(a) B-D
(c) C-D
(b) B-C
(d) A-B
asy
D. Hubris such egregious behaviour, but nowadays they just smile and
say that it’s just a phase, all adolescents go through.
(a) indolent (b) impertinent
144. A. Clandestine
C. Graceful
B. Cosmopolitan
D. Furtive En (c) unwanted (d) outrageous
155. GNOME : I like the style of this book- every chapter starts
(a) A-B
(c) C-D
(b) B-C
(d) A-D gin with a gnome
(a) dwarf
(c) aphorism
(b) question
(d) logo
145. A. Bellicose
C. Peaceful
(a) A-B
B. Affront
D. Adroit
(b) B-C
eer
156. CULPABILITY : The culpability of one, who unknowingly
sins, has different connotations in these Eastern regions.

(c) B-D
146. A. Frugal
(d) A-C
B. Spendthrift ing
(a) accountability
(c) Praiseworthiness
(b) Laudatory
(d) usefulness
157. PERSPICUOUS : A perspicuous argument is far more

C. Anomaly
(a) A-B
D. Anathema
(b) B-C solution
.ne
effective than mere rhetoric when it comes to achieving a

(c) C-D
147. A. Asinine
C. Tranquil
(a) A-B
(d) A-D
B. Besolted
D. Knowledgeable
(b) B-C
(a) synthetic
(c) ineffective
(b) Lucid
(d) obscure
t
158. ATROPHY : The brain like an unused limb, will atrophy if
not exercised enough.
(a) thrive (b) deteriorate
(c) C-D (d) A-D (c) succeed (d) degenerate
148. A. Crass B. Nexus 159. VAPIDITY : The vapidity of the hostess’s conversation
C. Elegant D. Credulous ensured that the evening was a failure.
(a) B-D (b) A-D (a) Provocativeness (b) dullness
(c) A-C (d) C-B (c) agitation (d) interesting
160. PEREMPTORY : Beauty that has lost all modesty becomes
DIRECTIONS (Q. 149 & 150) :   Choose a single word which is
peremptory and vain.
opposed to the meaning contained in the sentence. (a) aggressive (b) illusive
149. He was magnanimous and his benevolence made him give (c) democratic (d) permanent
to charity. DIRECTIONS (Qs. 161-165) : Below is given a single word with
(a) mean (b) cruel options to its meaning in different contexts. You have to select all
(c) snobbish (d) tyrannical those options which are synonyms of the word when the context
1 50. Biannual is is changed. Select the correct alterative from (a), (b), (c), (d) and
(a) once in two years (b) every year (e) which represents all those synonyms.
(c) twice a year (d) after every two years [IBPS PO 2011]

Downloaded From : www.EasyEngineering.net


Downloaded From : www.EasyEngineering.net

Word Power  l 197

161. MASK 169. (A) Purposefully (b) Inaccurately


(1) Cover (2) Hide (C) Inadvertently (d) Unchangeably
(3) Conceal (4) Disguise (a) A–C (b) A–B
(c) B–C (d) B–D
(a) Only (1) (b) Both (2) and (4)
(e) A–D
(c) Only (2), (3) and (4) (d) Only (1), (2) and (3) 170. (A) germane (B) generate
(e) All (1), (2), (3) and (4) (C) reliable (D) irrelevant
162. REGULAR (a) B–D (b) B–C
(1) Present (2) Common (c) A–B (d) C–D
(3) Indiscriminate (4) Uniform (e) A–D
(a) Only (4) (b) Both (2) and (4)
(c) Both (1) and (3) (d) Only (2), (3) and (4) DIRECTIONS (Qs. 171-175) : In each of the following questions
(e) All (1), (2), (3) and (4) four words are given, of which two are most nearly the same or
163. LABOUR opposite in meaning. Find the two words which are most nearly
(1) Expedite (2) To move faster the same or opposite in meaning and mark the number of the
(3) Controlled (4) Toil correct letter combination as your answer.

ww
(a) Only (4)
(c) Both (2), (3) and (4)
(e) All (1), (2), (3) and (4)
(b)
(d)
Both (1) and (3)
Only (1), (3) and (4)
171. (A) Discomfit (B) Baffle
[SBI PO 2011]

164. MEAN
w.E
(1) Imply (2) Understand
(C) Epicure
(a) A-B
(c) A-D
(D) Enumerate
(b) A-C
(d) B-C
(3) Average

(a) Only (3) asy


(4) Characterized by
malice
(b) Both (1) and (4)
(e) B-D
172. (A) Testimony (B) Aura
(c) Only (1), (3) and (4)
(e) All (1), (2), (3) and (4) En
(d) Only (1), (2) and (4) (C) Augment
(a) A-B
(D) Decrease
(b) B-C
165. ALONE
(1) Exclusively (2) Morose gin (c) C-D
(e) B-D
(d) A-D

(3) Solitary
(a) Only (1)
(4)
(b)
Human being
Both (1) and (3)
eer
173. (A) Unkempt
(C) Slackening
(B) Unremitting
(D) Distasteful
(c) Both (2) and (3)
(e) All (1), (2), (3) and (4)
(d) Only (1), (3) and (4)
(a) A-B
(c) C-D
ing (b) B-C
(d) A-D
DIRECTIONS (Qs.166-170): In each of the following questions
four words are given of which two words are most nearly the same
(e) B-D
174. (A) Gregarious .ne
(B) Quixotic
or opposite in meaning. Find the two words which are most nearly
the same or opposite in meaning and indicate the number of the
correct letter combination, by darkening the appropriate oval in
your answer sheet. [SBI PO 2013]
(C) Sociable
(a) A-B
(c) C-D
(D) Discernible
(b) B-C
(d) A-C
t
166. (A) Consent (B) Nascent (e) B-D
(C) Emerging (D) Insecure
(a) A–C (b) B-D 175. (A) Apathetic (B) Wrath
(c) B–D (d) A–D (C) Whirl (D) Twirl
(e) A–B (a) A-B (b) A-C
167. (A) Elated (B) Eccentric
(c) A-D (d) B-C
(C) Explicit (D) Abnormal
(a) A–B (b) B-D (e) C-D
(c) A–C (d) A–D DIRECTIONS (Qs. 176 - 180) : In the following questions out
(e) D–C of the four alternatives, choose the one which best expresses the
168. (A) Abundance (B) Incomparable meaning of the given word and mark it in the Answer-Sheet.
(C) Projection (D) Plethora [SSC CGL, 2012]
(a) A–C (b) A-B
(c) C–D (d) B–D 176. Vociferous
(e) A–D (a) violent (b) loud
(c) secret (d) true

Downloaded From : www.EasyEngineering.net


Downloaded From : www.EasyEngineering.net

198  l  Word Power

177. Fictional DIRECTIONS (Qs. 191-193) : Choose the word opposite in


(a) genuine (b) authentic meaning to the given word and mark it in the Answer Sheet.
(c) fanciful (d) real [SSC CGL 2013]
178. Trivial
191. Fabricate
(a) crucial (b) significant
(c) vital (d) ordinary (a) Unearth (b) Construct
179. Impudent (c) Demolish (d) Renovate
(a) Vigilant (b) Astute 192. Gregarious
(c) Insolent (d) Arrogant (a) Sociable (b) Societal
(c) Unsociable (d) Solitary
180. Pompous
193. Pragmatic
(a) Pretentious (b) Supportive
(a) Indefinite (b) Vague
(c) Demanding (d) Flashy
(c) Optimistic (d) Idealistic
DIRECTIONS (Qs. 181- 185) : In the following questions choose DIRECTIONS (Qs. 194 - 200) : In the following questions, out
the word opposite in meaning to the given word and mark it in of the four alternatives, choose the one which can be substituted
the Answer-Sheet. for the given words/sentence.

ww
181. Cultivated
[SSC CGL 2012]
194. That which has a double meaning
[SSC CGL 2013]

w.E
(a) Crude
(c) Suave
182. Impertinent
(b) Genteel
(d) Refined
(a) doubtless
(c) controversial
195. Incapable of making errors
(b) uncertain
(d) ambiguous

(a) Insolent
(c) Cheeky asy
(b) Impudent
(d) Courteous
(a) infallible
(c) impervious
(b) incorrigible
(d) inexplicable
183. Divulge
(a) Disseminate (b) Dissemble
En 196. Governed by a sense of duty
(a) conscious (b) sensible
(c) Publicize
184. Appreciation
(d) Transmit
gin (c) intelligent (d) conscientious
197. The depository where state records and documents are
preserved
(a) Aspersion
(c) Commendation
(b) Admiration
(d) Compliment
eer
(a) museum
(c) emporium
(b) library
(d) archive
185. Supple
(a) Pliant
(c) Rigid
(b) Pliable
(d) Flexible
(a) unuseding
198. That which is no longer fashionable or in use
(b) ancient

DIRECTIONS (Qs. 186 - 190) : In the following questions out


of the four alternatives, choose the one which can be substituted
(c) obsolete
199. Murder of a king
(a) homicide .ne
(d) old

(b) fratricide
for the given words / sentence.
[SSC CGL 2012]
186. A round or cylindrical container used for storing things such
(c) regicide
200. A place where birds are kept
(a) Aviary
(d) parricide

(b) House
t
as food, chemicals or rolls of film (c) Aquarium (d) Apiary
(a) tankard (b) canister DIRECTIONS (Qs. 201 - 203) : In the following questions, out
(c) vessel (d) casket of the four alternatives, choose the one which best expresses the
187. A place of permanent residence meaning of the given word.
(a) abode (b) dormitory [SSC CGL 2013]
(c) domicile (d) apartment
188. That cannot be altered or withdrawn 201. Condone
(a) irrevocable (b) irretrievable (a) Forgive (b) Support
(c) irrefutable (d) irresistible (c) Forget (d) Defend
189. Money paid to employees on retirement 202. Analogy
(a) gratuity (b) gift (a) Difference (b) Comparison
(c) pension (d) arrears (c) Addition (d) Deletion
190. A place where clothes are kept 203. Allure
(a) closet (b) drawer (a) Extol (b) Excite
(c) wardrobe (d) cupboard (c) Entice (d) Elicit

Downloaded From : www.EasyEngineering.net


Downloaded From : www.EasyEngineering.net

Word Power  l 199

DIRECTIONS (Qs. 204-206): In Question Nos. 161 to 163, 208. (A) critical (B) equitable
out of the four alternatives, choose the one which best expresses (C) impartial (D) unearth
the meaning of the given word and mark it in the Answer Sheet. (1) A - B (2) B - C
[SSC CGL, 2014] (3) A - D (4) B - D
204. Persist (5) C - D
(a) Resist (b) Leave 209. (A) shining (B) raise
(c) Quit (d) Insist (C) flourish (D) thrive
205. Eventually
(1) A - B (2) B - C
(a) previously (b) briefly
(3) C - D (4) B - D
(c) finally (d) successfully
206. Impeccable (5) A - C
(a) remarkable (b) unbelievable 210. (A) Affable (B) rude
(c) flawless (d) displeasing (C) pacify (D) cajole
DIRECTIONS (Qs.207-211) : In each of the following questions (1) A - B (2) B - C
four words are given of which two words are most nearly the (3) C - D (4) A - D
same or opposite in meaning. Find the number of correct letter (5) B - D

ww
combination.
207. (A) dominate
(C) determined
(B) radical
[SBI PO 2014]

(D) monopolise
211. (A) energetic
(C) diffuse
(1) A - B
(B) partial
(D) dispassionate
(2) B - C

w.E
(1) A - B
(3) A - D
(5) C - D
(2) B - C
(4) B - D
(3) C-D
(5) B - D
(4) A - D

asy
En
gin
eer
ing
.ne
t

Downloaded From : www.EasyEngineering.net


Downloaded From : www.EasyEngineering.net

200  l  Word Power

Hints & Solutions


LEVEL-I 22. (b) Grit is a sense of determination to face a difficult condition
i. e., courage.
1. (b) Accoutrements means additional requirements for a dress, 23. (a) Salacious and Obscene both mean vulgar and offending in
blemishes are spots so can’t be the synonym, belongings refer to sexual matters. Wholesome is pleasant or enjoyable.
objects one owns and this too cannot be the answer. Of relatives 24. (d) Clout means influence or dominance or power in the
and companions the latter will be the correct choice because respective field.
relatives implies intrinsic relations while companions are only 25. (a) Foment means to provoke something evil or to do wrong.
additional people with you. Shield is to cover or protect someone or something. Waver
2. (c) ‘A’ when added as a prefix does not imply anti but an means to get gradually weak in intensity or effort or voice.
indifference or apathy e.g. ‘amoral’ and ‘immoral’ immoral 26. (a) Repercussion means the consequent action or result or
means not moral while amoral is only someone who does not reaction of an action.
care about morals. 27. (a) Propitious means favorable. Versatile is someone who has
3. (a) Azure means blue. many talents or skills
4.

5.
ww
(a) Bonhomie means cheerful, friendly, while wrath means
extreme anger.
(b) Candour means frankness, Enthusiasm is excitement and
28. (b) Talisman is an object supposed to bring good luck, that
would be a charm, fiction is a figment of imagination, untrue.
29. (a) Penury and destitution both mean extreme poverty, vigour

6. w.E
Intimate is closeness or nearness.
(b) Fluctuation refers to a change, it may be an increase or
conversely a decrease
is active physical or mental strength, digestive is related to
digestion.
30. (d) Affront means an insult, indignity is humiliation which
7.

asy
(c) Bashful means a shy person, haughty is very proud and
snobbish, discreet is someone who is cautions and prudent, not
showy.
also means insult, exile is being sent away from one’s country
usually is a punishment, contours are the outlines of an object
or geographical area. Eruption is a sudden break out.
8. (c) Trivial means insignificant or unimportant, momentous
means for/of the moment, critical is someone who looks for
En 31. (c) Amicable means friendly and harmonious means an
agreement and working together with perfect cooperation,

9.
faults.
(a) Rivalry means competition, campaign is an organised
course of action. gin friendliness is always harmonious, nebulous means not in a
definite shape, abominable is hateful, delicate is soft.
32. (c) Dregs means left over, last and useless part of something,
10.

11.
(b) Collapse means to fall down or lose strength, of the given
options the nearest to its meaning is failure.
(a) Ominous is a sign of something bad in future, a forboding, eer
debris also means the remaining ruins of a structure,
accoutrements means additional items needed with a dress.
33. (c) Loquacious is to talk a lot, i.e.., talkative, obstinate mean
threat is a warning for future, ubiquitous is something that is
found everywhere, burdensome is thing that appears like a
stubborn,
ing
34. (b) Waive is used as the discounts or relaxations given in a loan

12.
burden to the owner.
(c) Procrastinate is to delay and so is postpone, predict is to
guess something about future. .ne
or a debt, e.g., ‘I will waive off the interest for last two months’
thus the synonym will be release, permit is to allow, admit is to
accept, restrict is to stop or control.
13.

14.
(b) Hiatus is a break or a gap in a process, atrocious is bad
and crude, while dominance is extreme and forceful influence.
Obscure is anything not clear.
(d) Foment is to stir up, and instigate means to initiate or incite
t
35. (a) Elan means vivacious, i.e., full of colour, life and enthusiasm,
flair means natural ability, means inner enthusiasm to do a
thing, spiritual is ‘of the spirit’, inspiration is motivation.
36. (d) Gauche is lacking ease or grace of manner, i.e. someone dull
and incipient, Vain means false pride, polished is sophisticated
which is very similar to stir up or provoke, renounce is to give
up and vex is annoyed. and refined so opposite to Gauche, tactless is someone who is
dumb or dull.
15. (a) Vagary is a capricious idea, enthusiasm is excitement,
37. (c) Browbeat is to scare someone by powerful words, timidate
caprice means whim or sudden fancy.
is also to overpower or influence greatly, incite fear. Tarnish is
16. (c) Macabre means gruesome or horrible, Tarried is delayed. to lose lusture.
17. (a) Denouement is the final part of a drama or a story, i.e. the 38. (d) Imposture means deception or cheating.
outcome, eschew is to avoid doing something bad. 39. (c) Proboscis means long flexible snout, probe is to question.
18. (a) Manacle is a chain or something used to bind, fetters are 40. (a) Parley is to discuss, sweeten is to make something sweet.
also chains, ornaments are piece of jewelry. 41. (a) Furlough means leave or absence.
19. (b) Feign is to pretend or act something that one is not, hesitate 42. (c) Punctilious means very careful about details, meticulous
is reluctant. also means the same, prude is a person who has an exxagerated
20. (d) Assay is to evaluate something for its quality. sense of propriety and gets shocked easily. Timid is a person
21. (c) Revulsion is opposite of attraction, it means a repellant who is shy, fearful.
disgusting feeling, avenge is to take revenge, apathy means 43. (b) Encomium is formal praise and eulogy is a piece of spoken
indifference or cold attitude toward a thing or person. or written praise, verve is enthusiasm or vigour, doggerel is bad
verse.

Downloaded From : www.EasyEngineering.net


Downloaded From : www.EasyEngineering.net

Word Power  l 201

44. (a) Invidious means improper, this may cause irritation or 67. (c) Rampant is widespread, something that can’t be controlled
anger. or kept in limits, furious is angry and restrained is controlled,
45. (b) Lachrymose is tearful, mournful will also be tearful, terse is kept in limits.
brief, lecherous is someone extremely interested in sex. 68. (a) Prolific means fecund or very productive, Barren means
46. (d) Elan means vigour or energy and enthusiasm, opposite unproductive, reckless is wildly impulsive, profound is
of enthusiasm would be nervousness, composure means something with much depth (of meaning).
calmness, dignity is calm and serious manner while brashness 69. (c) Sequester is to isolate or confiscate and unify means to
is opposite to composure. bring together. Slump is a sudden fall down in an economy.
47. (c) Irascible is hot tempered, or someone who easily gets Isolate will be a synonym to sequester.
angry, roguish is playful, it is more clearly an antonym to hot 70. (c) Trite and hackneyed mean something that has been
tempered than friendly or even kind. Both words Irascible and overused and thus has lost its impact, original on the other
roguish concern the temperament of a person, while others are
hand is the get unused new in character, being the first form of
about his nature.
something still has its impact.
48. (a) Nebulous is indistinct something that does not have a clear
outline, Concrete is solid and of definite shape. 71. (b) Compassionate is someone who cares or is concerned
49. (a) Penchant is a natural inclination towards something, lone about another person, unsympathetic is one who does not
means only, alone, single. care or understand another’s condition or position. Indecisive
50. (c) Perfunctory is a task performed only for the sake of it, done means someone who cannot take decisions easily, unlawful is
carelessly without much attention. something against law, and Untrustworthy is someone who

ww
51. (d) Transient is something that is momentous or not fixed,
which passes away with time, eternal is what lasts forever.
52. (a) Sublime is magnificent and great, base is anything low,
cannot be trusted.
72. (b) Restive is something very difficult to control and placid is
calm and peaceful so can be controlled easily. Buoyant is very

w.E
ordinary, concise means short or compact, partial is not
complete, and Insist is to make a request.
53. (b) Mansion is a very big house, while hovel is a small, dingy
house, castle is a large fortified residence.
cheerful and insolent is rude.
73. (c) Gradual refers to a slow and steady process, Rapid means
quick and swift, dynamic means changing.
74. (a) Rudimentary means still in the initial stage, developed will

means command or an order.


asy
54. (c) Partition is to divide, unify is to bring together, enjoin

55. (d) Primitive means of the very old time, and thus not very
be opposite to this.
75. (a) Salvage is to save something from destruction, burn will be
to destroy.
mannered, Naive means inexperienced.
56. (a) Termagant means quarrelsome or someone who always En
developed or refined, sophisticated is well-refined and well 76. (d) Sedentary means something inactive or sleeping. Indolent
means rude.
quarrels while charming is the one who pleases. Turbulent
means agitated. Brawling is to fight and create noise, shrewish
also means quarrelsome. gin
77. (a) Scrimp means to live on very little money and Lavish is to
live extravagantly, i.e., spending a lot of money, parsimonious
are means for making money.
57. (c) Transient is what doesn’t last forever, i.e. what is not
permanent. Transitory, fleeting, momentary all means the same
as transient. eer
78. (c) Delectable is delightful or pleasant, distasteful is something
unpleasant, agonising is something that gives pain and anxiety,
appetising means to rouse or increase appetite, laborious means
58. (c) Regression means to go back or return; reverse, relapse
and retreat all mean going back. Regenerate on the other hand
means to create a new life, go ahead. Thus is the antonym.
ing
something that needs or shows much effort.
79. (b) Grave means seriously important, and inconsequential is
something of no importance at all, solemn means serious and
59. (a) Vengeful is full of vengeance, i.e., anger and revenge;
forgiving will be opposite to taking revenge; Vindictive is to grow old and weaker.
.ne
calm and can be synonymously used with Grave, senile is to

80. (a) Confident is the person you tell your secrets and who keeps

attack and Vicious is something evil.
60. (d) Tirade is a formal and strong criticism, Censure,
declamation and discredit are synonymous to Tirade, analogy
means excessive praise.
61. (a) Salutatory is the welcome speech and valedictory is a
t
that secret, turncoat is a person who changes allegience from
one to another and does not remain loyal, arrogant means rude,
firm means strict and strong, confederate means joined by a
treaty or agreement.
farewell speech, derivative means something that has been 81. (d) Amalgamate is to blend two things together, split is to
derived from another. break up, merge is synonymous to amalgamate, impoverish is
62. (c) Tractable is something that can be managed easily, i.e., to make poor inquality and consecrate is to devote something
manageable. for religious purpose.
63. (a) Cynical is a person who believes the motives and intentions 82. (a) Boisterous means with lot of energy and noise, serene is
of other people are always bad or evil, i.e., he always suspects, calm, tumultous means turbulent or agitated, Brazen means
doubts mistrusts, as opposite to a trusting person. Gallant is shameless and opaque is an object through which one can see.
very brave, brazen means shameless and Tiresome is long and 83. (b) Asset means a useful property of a person, liability means
tiring work. a debt or an obligation, credibility means believable, prosperity
64. (c) Grisly means unpleasant, fragrant is something that smells means progress.
pleasant, haggard is very thin and lean. 84. (c) Smother means to suffocate or stifle, i.e, to prevent the
65. (b) Cadaverous is very slim or thin, obese is very fat, lucid growth of something, nurture is to help the growth, repress also
means clearly expressed, inquisitive is to be very curious, and means to prevent the growth and thus is a synonym to smother.
skillful is someone who shows great skill. 85. (b) Reprimand means to scold, while appreciate means to
66. (b) Solicitous is someone who is anxious about a person’s praise, praise is not the right antonym because praise is a noun
welfare, concerned will be a synonym to solicitous. Indifferent while reprimand and appreciate are both verbs. Reward is to
is one who does not care, eager is full of desire. give prize.

Downloaded From : www.EasyEngineering.net


Downloaded From : www.EasyEngineering.net

202  l  Word Power

86. (b) Tangible means available physically, concretely, elusive 103. (a) ‘Bucolic’ (of or concerning shepherds, rural) does not

is something that eludes, i.e, is difficult to catch, touch or belong to the group of ‘choleric’ (irascible, angry) (b), ‘bilious’
understand, palpable is a synonym to tangible. (bad tempered, affected by a disorder of the bile) (c) and
87. (b) Zoom is to rise fast and sharply. Plummet is to downslide ‘cantankerous’ (bad-tempered, quarrelsome) (d), they are
very fast, soothe is to make someone or something calm. synonyms.
Refute is to deny or contradict, subjugate means to bring under 104. (d) ‘Tantrums’ (outburst of bad temper or petulance) does
control. not belong to the group of ‘pelf ’ (money, wealth) (a), ‘moolah’
88. (a) Predilection means a special liking for someone. Antipathy (money) (b) and ‘lucre’ (financial profit or gain) (c), they are
means hate for someone or a strong dislike. Dissonance is synonyms.
difference of opinion. 105. (d) ‘Revive’ (bring back to life, consciousness or strength) does
89. (a) Docile means submissive and therefore can be controlled not belong to the group of ‘munch’ (eat steady with marked
very easily, unmanageable is someone or something that cannot action of the jaws) (a), ‘masticate’ (grind or chew with one’s
be controlled or managed easily. teeth) (b) and ‘crush’ (c), they are synonyms.
90. (c) Equivocal means doubtful or ambiguous, clear will be 106. (e) 107. (c) 108. (b) 109. (e) 110. (a) 111. (a)
opposite to that, Quaint means strangely delightful.
112. (e) 113. (d) 114. (c) 115. (a) 116. (c) 117. (a)
91. (c) ‘Omnipresent’ does not belong to the group of ‘dull’ (a),
‘gloomy’ (depressed, sullen, dismal, depressing, dark) (b) and 118. (a) 119. (b) 120. (b) 121. (c) 122. (d) 123. (d)
‘boring’ (d); they are synonyms. Except (c), all other options are 124. (b) 125. (a) 126. (a) 127. (c) 128. (d) 129. (d)

ww
wrong.
92. (d) ‘Duplicity’ (double-dealing, deceitfulness) does not belong
to the group of ‘archenemy’ (a), ‘archrival’ (b) and ‘archetype’
130. (b) 131.
136. (d) 137.
142. (d) 143.
(d)
(a)
(c)
132.
138.
144.
(d)
(c)
(b)
133.
139.
145.
(c)
(b)
(d)
134.
140.
(b)
(a)
146. (b) 147. (d)
135. (d)
141. (c)

w.E
(an original model; a prototype) (c); they share the prefix ‘arch’
(chief, superior, preeminent of its kind).
93. (b) ‘Laud’ (praise or extol, especially in hymns) does not belong
148. (b) 149. (b) 150. (d) 151. (d) 152. (d) 153. (a)
154. (a) 155. (b)
156. (d) ‘Wily’ means ‘cunning’ which also means skillful or clever.

asy
to the group of ‘dishonour’ (a), ‘brand’ (stigmatise, mark with
disgrace) (c) and ‘stigmatise’ (d), they are synonyms.
94. (a) ‘Disclaim’ does not belong to the group of ‘intimate’ (state or
make known, imply, hint) (b), ‘hint’ (c) and ‘suggest’ (d), they
157. (c) ‘Audacity’ best expresses the meaning of ‘temerity’ which
also means ‘arrogance’


are synonyms.
95. (a) ‘Allegory’ does not belong to the group of ‘parody’ En 158. (a) The opposite of ‘insipid’ is ‘tasty’. The word ‘insipid’ means
dull, boring or colorless.
159. (b) The opposite of ‘relinquish’ is ‘continue’. Relinquish means
(humorous, exaggerated imitation of an author, literary work,
style etc.) (b), ‘joke’ (c) and ‘lampoon’ (a satirical attack on a
person) (d), they are types of comedy. gin to ‘give up’.
160. (a) 161. (a) 162. (c)
96. (d) ‘Car’ does not belong to the group of ‘comedy’ (a),
‘protagonist’ (the chief person in a drama, story etc.) (b),
‘tragedy’ (c), they are theatrical / literary terms. eer
163. (c) A sonnet is a short rhyming poem with 14 lines. The original
sonnet form was invented in the 13/14th century by Dante and
an Italian philosopher named Francisco Petrarch. The form
97. (c) ‘Nursery’ (a room or place equipped for young children;
a place where plants, trees etc. are reared for sale) does not
belong to the group of ‘flourish’ (grow vigorously, thrive) (a),
ing
remained largely unknown until it was found and developed
by writers such as Shakespeare. Sonnets use iambic meter in
‘burgeon’ (begin to grow rapidly, flourish) (b) and ‘sprout’ (put
forth, produce, begin to grow, put forth shoots) (d), they are
each line and use line-ending rhymes.

.ne
164. (d) “Fallible” means capable of making mistakes - or, easier
to remember - capable of failing.Infallible means exactly the
growth-related terms.
98. (a) ‘Nemesis’ (retributive justice, a downfall caused by this)
does not belong to the group of ‘pretense’ (pretending; make
believe; a pretext or excuse) (b), ‘sham’ (feign, pretend) (c) and
‘deception’ (d), they are synonyms.
opposite - incapable of failing.

too readily, especially without proper or adequate evidence.


t
165. (a) A credulous person is one who is willing to believe or trust

166. (a) Alimony is a legal obligation on a person to provide financial


99. (d) ‘Bond’ (noun – a thing that ties another down or together; support to his or her spouse before or after marital separation
verb - adhere, hold together) does not belong to the group of or divorce.
‘bonus’ (a), ‘beneficial’ (advantageous, having benefits) (b) and 167. (b) Fantasy is the faculty or activity of imagining impossible or
‘benevolent’ (wishing to be good; actively friendly and helpful), improbable things.
they share the root “bon”/”bene” meaning something good. 168. (c) Equilibrium means balance. Its opposite is imbalance.
100. (c) ‘Whim’ (a sudden fancy, a caprice) does not belong to the 169. (d) immortal means death-defying or endless. Its opposite is
group of ‘entice’ (persuade by the offer of pleasure or reward) temporary.
(a), ‘lure’ (entice) (b) and ‘beguile’ (charm; amuse) (d), they are 170. (a) Focus means the state or quality of having or producing
synonyms. clear visual definition. Its opposite is disappear.
101. (d) ‘Raunchy’ (coarse, earthy, boisterous, sexually provocative) 171. (c) veteran means experienced. Whereas, Novice means a
does not belong to the group of ‘grand’ (a) ‘majestic’ (b) and person just learning something.
‘august’ (venerable, impressive) (c), they are synonyms. 172. (d) Superfluous means unnecessary. Its opposite is necessary.
102. (c) ‘Staunch’ (trustworthy, loyal, strong) does not belong to 173. (b) Persevere means continue in a course of action even in the
the group of ‘mercurial’ (of a person sprightly, ready-witted, face of difficulty or with little or no indication of success. Persist
volatile) (a), ‘mutable’ (liable to change, fickle) (b) and ‘fickle’ means continue in an opinion or course of action in spite of
(d), they are synonyms. difficulty or opposition.

Downloaded From : www.EasyEngineering.net


Downloaded From : www.EasyEngineering.net

Word Power  l 203

174. (c), Petition means a formal written request, typically one signed The word Laudatory (Adjective) means : expressing praise
by many people, appealing to authority in respect of a particular or admiration.
cause.
175. (c), Proposition means a plan or scheme proposed. 200. (b) The word Amorphous (Adjective) means : shapeless ;
176. (b), Vivacious means attractively lively and animated. irregular, having no definite shape.
177. (b), sporadic means occurring at irregular intervals or only in 201. (c) The word Unitary (Adjective) means : single : forming
a few places; scattered or isolated.
178. (b) The meaning of Indiscriminate (adj.) : random, chaotic one unit.
“Instant” word best expresses the meaning of given word. Multiple = many in number
179. (a) The meaning of Literal (adj.) word for word, exact, real. 202. (a) The word Adulteration (Noun) means : making impure
“Verbatim” word best expresses the meaning of given
word. by mixing; contamination.
180. (c) The meaning of Intricate (adj.) : complicated, elaborate. The word Purification (Noun) means : making something
“Complicated” word best expresses the meaning of given pure by removing substances that are dirty. harmful.
word. 203. (b) Scorn means contempt toward something.
181. (d) The meaning of Expeditiously (adverb) : actively, intently. 204. (a) Catastrophe means an event causing great and usually
“Quickly” word best expresses the meaning of given word. sudden damage or suffering.

ww
182. (d) The meaning of idiosyncrasies (noun) : oddity, way of
doing something “Eccentricities” word best expresses the
meaning of given word.
183. (a) 184. (b) 185. (b) 186. (a) 187. (d)
205. (a) Abjure means solemnly renounce.
206. (b) Assess means to get measure of.
207. (c) Elastic means able to encompass much variety and change;

w.E
188. (c) The meaning of Resourcefulness (adj.) : ability to cope
with difficult situation.
The meaning of Incompetence (noun) : Lack of ability.
flexible and adaptable.
208. (a) A volunteer is a person who freely offers to take part in
an enterprise and undertake a task.

asy
∴ “Incompetence” is opposite to given word.
189. (d) The meaning of Evolve (verb) : adj. develop, progress. The
meaning of stop (noun) : end, halt.
209. (a) Scepticism means a person inclined to question or doubt
accepted opinions.
210. (a) Coffle means a line of slaves / animals fastened or driven
∴ “Stop” is opposite to given word.
En
190. (c) The meaning of Antiquated (adj.) : obsolete, ancient. The
along together.
211. (a) A mercenary is a hired professional soldier who fights for
any state or nation without regard to political principles.
meaning of Modern (adj.) new, up to date.
∴ “Modern” is opposite to given word.
191. (a) The meaning of Pathetic (adj.) : Sad, affecting. The meaning gin
212. (a) Fastidious means difficult to please.
213. (b) Safe means free from harm. Its opposite is insecure.


of comic (noun) : funny.
∴ “Comic” is opposite to given word.
192. (b) The meaning of Fastidious (adj.) very careful. The meaning eer
214. (c) Redundant means excessive. Its opposite is singular.
215. (b) Fair means impartial. Its opposite is unjust.
216. (b) boisterous means noisy and mischievous. Its opposite is
of shabby (adj.) : broken-down.
∴ “Shabby” is opposite to given word.
calm.
ing
217. (a) Flimsy means insubstantial and easily damaged.
193. (a) The word Abnormal (Adjective) means: unusual, irregular
; unnatural ; different from what is usual.
218. (b) means uncivilised
219. (d) means obstacle
.ne
Look at the sentence :
They thought his behaviour was abnormal.
194. (a) The word Venal (Adjective) means : corrupt; prepared to
220. (a) means to hinder
221. (b) malevolent means malicious, spiteful, wicked
222. (b) disorganised means muddled, jumbled, confused
t
do dishonest or immoral thing in return for money. 223. (b) humane means gentle, benevolent, civilised
Look at the sentence : 224. (b) bankrupt means insolvent or penniless
Venal leaders should be denied vote.
225. (a) microscope
195. (c) The word Conjurer (Noun) means: a person who performs
226. (d) aquatic
magic tricks ; magician.
227. (c) botany
196. (c) The word Invoice (Noun) means: list of goods that have
228. (c) monopoly can also mean domination
been sold ; bill ; statement.
229. (c) The opposite of ‘predilection’ is ‘aversion’.
197. (a) The word Ameliorate (Verb) means: to make something 230. (b) The opposite of ‘pompous’ is ‘humble’.
better; improve. 231. (c) The opposite of ‘serene’ is ‘ruffled’.
198. (c) The word Debacle (Noun) means : a situation that is a 232. (c) Belief in many Gods-Polytheism.
233. (b) Cluster of flowers on a branch-Inflorescence.
complete failure and causes embarrassment.
234. (b) A person who believes that only selfishness motivates
199. (a) The word Abusive (Adjective) means: expressing praise human action-Cynic.
or admiration. 235. (b) A highly skilled musician-Virtuoso.

Downloaded From : www.EasyEngineering.net


Downloaded From : www.EasyEngineering.net

204  l  Word Power

236. (d) A method of boiling briefly to cook food slightly Parboil. 13. (a) Exodus means outflow and influx is opposite with a
237. (a) The group, especially in arts, regarded as being the most meaning inflow. Home coming is opposite for the word
experimental Avant-garde. expatriate— which means going out of one’s native land.
238. (a) One who helps people by giving them money or other aid Restoration and return is also not suitable.
Benefactor. 14. (d) Equanimity refers to maintaining calm and balance. Its
opposite is excitement where calm is lost. Resentment is anger.
LEVEL-II 15. (d) “Veracious” means ‘truthful or honest’.
16. (c) “Perturb” means ‘to disturb or disquiet greatly in mind’.
1. (a), (d) ‘Sham’ : (as verb) feign, pretend; (as noun) imposture, 17. (a) “Acquiescence” means ‘passive assent or agreement without
pretence; ‘feign’ (1) and ‘hypocrisy’ (d) are closest synonyms, objection’. This is closest in meaning to “quiet submission”,
‘spread’ (b) and ‘mellow’ (soft and rich, softened or matured) which means “giving in without a fight”.
(c) are not. Except (a) and (d) other options are wrong. 18. (b) ‘Wheedle’ means ‘to try to influence (a person) by smooth
2. (b), (c) ‘Wile’ : (as noun) a stratagem, a trick or cunning flattering or beguiling words or acts. ‘Flattery’ is closest in
procedure; (as verb) lure or entice; ‘guile’ (treachery, deceit, meaning to ‘wheedle’.
Emaciated: made or became extremely thin, especially as a
cunning) (b) and ‘entice’ (persuade by the offer of pleasure or
result of starvation.
reward) (c) are closest synonyms; ‘during’ (a) and ‘guilt’ (d) are
Purge: to rid of whatever is impure or undesirable; cleanse;
not. Except (b) and (c) other options are wrong. purify
3. (a), (b) ‘Fiat’ : an authorisation, a decree or order; ‘decree’ (a)

ww
and ‘edict’ : (an order proclaimed by authority) (b) are closest
synonyms; ‘auto’ (c) and ‘design’ (d) are not. Except (a) and (b)
other options are wrong.
Scant: limited; meagre
Thus options (a), (c) and (d) are eliminated.
19. (c) ‘Immiseration means misery or impoverished’.
Immigration: settling in a country or region to which one is not

w.E
4. (c), (d) ‘Sect’ : a party or faction in a religious body, a religious
denomination, a body of people subscribing to religious
doctrines different from others in the same religion; ‘faction’ (a
native
Immersion: placing under water or other liquid
Impoverishment: poverty

asy
small organised, dissenting group) (c) and ‘party’ (d) are closest
synonyms. Except (c) and (d) other options are wrong.
5. (b), (d) ‘Rank’ : (here) offensive, loathsome, indecent, flagrant,
Discrimination: partiality; discernment
Therefore, the word closest in meaning to immiseration is
impoverishment.

En
gross, complete etc.; ‘complete’ (b) and ‘flagrant’ (d) are closest
synonyms, ‘bottom’ (a) and ‘lever’ (c) are not. Except (b) and
20. (d) ‘Beatitude’ means ‘supreme blessedness exalted happiness’.
Accursed: under a curse; doomed; ill-fated
Cleansed: made clean
(d) other options are wrong.
6. (b), (c) ‘Aver’ : assert, affirm, attest (certifying the validity of,
be evidence of) (b) and ‘avow’ (admit, confess) (c) are closest gin Retreated: withdrew, as into safety or privacy; retirement;
refuge; seclusion
Blessed: sacred; holy
synonyms; ‘avoid’ (a) and ‘deny’ (d) are not. Except (b) and (c)
other options are wrong.
eer
Therefore, the word closest in meaning to beatitude is blessed
21. (b) ‘Bedizen’ means to dress or adorn in a showy, gaudy or
tasteless manner
7. (a), (c) ‘Cant’ : insincere, pious, moral talk, language peculiar to
a class, profession, sect etc.; ‘jargon’ (words used by a particular
profession or group) (a) and ‘argot’ (the jargon of a group or ing
Therefore, ‘gaudily dressed is the closest in meaning
22. (c) Cachinnate means to laugh loudly or immoderately
Therefore, laugh aloud is the closest in meaning.
class, formerly esp. of criminals) (c) are closest synonyms;
‘contradiction’ (b) and ‘talk’ (d) are not. Except (a) and (c) other
options are wrong. .ne
23. (a) Conoodole means caress fondle or pet amorously
Cuddle: to hold close in an affectionate manner; hug tenderly
8. (c), (d) ‘Dole’ : a charitable (esp. sparing, niggardly gift of food,
clothes or money), ‘distribute’ (c) and ‘parcel’ (out) (d) are
closest synonyms; ‘senator’ (a) and ‘put’ (b) are not. Except (c)
and (d) other options are wrong.
Brazen: shameless or impudent
Articulate: uttered clearly in distinct syllables t
Canonise: to consider or treat as sacrosanct or holy; to glorify

Therefore, the word closest in meaning to canoodle is cuddle.


24. (c) ‘Tumescent’ means ‘becoming swollen swelling; slightly
9. (a) (d) ‘Mire’ : a stretch of boggy or swampy ground, mud, dirt; tumid’.
‘bog’ (a) and ‘entangle’ (cause to get caught in a snare or among Shrink: to contract or lessen in size
obstacles) (d) are closest synonyms; ‘mush’ (soft pulp; feeble Annoy: to bother (a person) in a way that displeases, troubles,
sentimentality) (b) and ‘briar’ (any prickly bush esp. of a wild or slightly irritates; to harass
rose) (c) are not. Except (a) and (d) other options are wrong. Engorge: to swallow greedily; glut or gorge
10. (a), (b) ‘Tilt’ : assume or cause to assume a sloping position, Enlighten: to instruct; impart knowledge to
strike or charge at; ‘charge’ (a) and ‘incline’ (b) are the Therefore, the word closest in meaning to ‘tumescent’ is
‘engorge’.
synonyms; ‘fall’ (c) and ‘admire’ (d) are not. Except (a) and (b),
25. (d) ‘Twaddle’ means ‘trivial, feeble, silly, or tedious talk or
other options are wrong.
writing’.
11. (d) Frugal is someone who thinks twice before spending The words smart, indecisive and obscene are not related to
money. Economical is direct synonym for it. There is no fight twaddle.
with other wayward easy options. ‘Waffle’ means ‘to talk foolishly or without purpose idle away
12. (c) Harbinger - One that indicates or foreshadows what is time talking’.
to come; a forerunner. Although pilot projects/samples also 26. (a) ‘Ennui’ is a noun meaning ‘a feeling of utter weariness and
somewhat forerunners but that is figurative meaning and discontent resulting from satiety or lack of interest boredom;
second best option. listlessness’.

Downloaded From : www.EasyEngineering.net


Downloaded From : www.EasyEngineering.net

Word Power  l 205

Lassitude: weariness of body or mind from strain, oppressive Transform: to change


climate, etc.; lack of energy; listlessness. Coherent: logically connected; consistent
The words ‘confront’, ‘enthuse’ and ‘entrap’ are verbs and are Diffuse: to pour out and spread, as a fluid
not related to the noun ennui Therefore, the word that is most opposite in meaning to leaven
Therefore, the word closest in meaning to ‘ennui’ is ‘lassitude’. is ‘static’.
27. (b) 28. (a) 29. (d) 30. (b) 31. (d) 52. (d) ‘Opprobrium’ means ‘disgrace arising from exceedingly
32. (c) 33. (d) 34. (d) 35. (c) 36. (a) shameful conduct; ignominy’.
37. (c) 38. (b) 39. (d) 40. (a) Ignominy: disgrace; dishonour
41. (b) Zoom is to rise fast and sharply. Plummet is to downslide Opportunity: a situation or condition favourable for attainment
very fast, soothe is to make someone or something calm.
of a goal
Refute is to deny or contradict, subjugate means to bring under
Obituary: a notice of the death of a person
control.
42. (a) Predilection means a special liking for someone. Antipathy Honour: high public esteem; fame; glory
means hate for someone or a strong dislike. Dissonance is Therefore, the word that is most opposite in meaning to
difference of opinion. ‘opprobrium’ is ‘honour’.
43. (a) Docile means submissive and therefore can be controlled 53. (a) Parsimonious means frugal or stingy miserly
very easily, unmanageable is someone or something that cannot Extravagant: spending much more than is necessary or wise;
be controlled or managed easily. wasteful

ww
44. (c) Equivocal means doubtful or ambiguous, clear will be
opposite to that, Quaint means strangely delightful.
45. (c) Prosaic means ordinary, whereas Rhetorical is impressive,
Penurious: extremely stingy
Partial: being a part; biased or prejudiced
Passionate: having intense emotion or strong feeling

w.E
ornate and is meant to influence. Pensive means sad and
thoughtful.
46. (c) Levity means lack of seriousness whereas solemnity means
seriousness. Frivolity is something that lack a serious intention
54.
Therefore, the word that is most opposite in meaning to
‘parsimonious’ is ‘extravagant’.
(b) ‘Insidious’ means ‘intended to entrap or beguile; stealthily
treacherous or deceitful’.
and can be taken synonymous with levity.
asy
47. (a) Obloquy is severe criticism publicly, an abuse. Slander
means false charges on someone.
Deceitful: misleading
Apparent: readily seen; obvious
Insincere: not sincere; not honest in the expression of actual
48. (b) ‘Cantankerous’ means ‘disagreeable to deal with;
contentious peevish cranky
En feeling
Tepid: moderately warm; characterized by a lack of force or
Irascible: easily angered; irritable; grouchy
Co-operative: working or acting together willingly for a
common purpose or benefit gin enthusiasm
Therefore, the word that is most opposite in meaning to
‘insidious’ is ‘apparent’.
Adamant: utterly unyielding in attitude or opinion; inflexible;
uncompromising
Captivate: to attract and hold the attention or interest of;
55.
eer
(b) Rapacious means given to seizing for plunder or the
satisfaction of greed
fascinate.
Therefore the word that is most opposite in meaning to
‘cantankerous’ is ‘co-operative’. ing
Avaricious: greedy
Satiated: satisfied, as one’s appetite or desire, to the point of
boredom
49. (d) ‘Emblazon’ means ‘to decorate with brilliant colours to
proclaim’.
Extol: to praise highly
.ne
Decorated: furnished or adorned with something ornamental
Subconscious: the part of the mind below the level of conscious

Cinder: a partially or mostly burned piece of coal, wood, etc


Embalm: to treat (a dead body) so as to preserve it, as with
chemicals, drugs, or balsams
Subtle: faint, tenuous, or rarefied, as a fluid or an odour; fine or
56.
perception

‘rapacious’ is ‘satiated’. t
Therefore, the word that is most opposite in meaning to

(c) ‘Soporific’ means ‘causing sleep; sleepiness; drowsiness’.


Somnolent: tending to cause sleep
delicate in meaning or intent; difficult to perceive or understand Unromantic: pragmatic; matter-of-fact
Therefore, the word that is most opposite in meaning to
Alert: fully aware and attentive
emblazon is subtle
Slumber: to sleep, especially lightly; doze; drowse
50. (c) ‘Inveigh’ means ‘to protest strongly or attack vehemently
Therefore, the word that is most opposite in meaning to
with words’.
Harangue: a scolding or a long or intense verbal attack; rant soporific is alert
Celebrate: to observe (a day) or commemorate (an event) with 57. (b) ‘Ubiquitous’ means ‘existing or being everywhere esp at
ceremonies or festivities the same time; omnipresent’.
Endorse: to approve, support, or sustain Universal: pertaining to, or characteristic of all or the whole;
Neglect: to pay no attention or too little attention to applicable everywhere or in all cases; general
Therefore, the word that is most opposite in meaning to Rare: coming or occurring far apart in time; unusual;
‘inveigh’ is ‘endorse’. uncommon; exceptional
51. (a) ‘Leaven’ refers ‘to a substance as yeast or baking powder Ensnare: to capture in, or involve as in, a snare; entrap
that causes fermentation and expansion of dough or batter or Conscientious: scrupulous; upright
an element that produces an altering or transforming influence’. Therefore, the word that is most opposite in meaning to
Static: showing little or no change ‘ubiquitous’ is ‘rare’

Downloaded From : www.EasyEngineering.net


Downloaded From : www.EasyEngineering.net

206  l  Word Power

58. (c) Puissance is “having great power” while impotence means 93. (b) tempestuous - resembling or pertaining to a violent storm.
“powerless”. Other words are related to ‘hateful’.
59. (d) Recondite means ‘not understood by many people’. so, 94. (d) disburden - to get rid of a burden; unload. Other words are
obvious is the correct antonym. related to ‘acquital from blame, crime etc’.
60. (c) Restitution is the act of giving back something that was lost 95. (d) dazzle - to overpower with strong light. Other words are
or stolen to its owner. Deprivation means ‘the fact of not having related to charm.
something that one needs. 96. (b) dissipate - to scatter; to dispel. Other words are related to
61. (c) Scad means in large numbers/amounts or in abundance. ‘criticism’.
Opposite would be dearth or shortage. 97. (c) sprawl - to stretch; to speed. Other words are related to
‘blame’; reprove.
62. (d) Limber refers to doing physical exercises in order to stretch
98. (d) allure - ability to fascinate; charm. Other words are
and prepare your muscles before taking part in a sporting
synonymous.
activity. Stiff refers to firm and difficult to bend or move. 99. (c) 100. (d) 101. (d) 102. (c) 103. (a) 104. (b)
63. (a) Obliquity refers to deceptiveness, opposite to straight 105. (c) 106.(c) 107. (d) 108. (b) 109. (a) 110. (c)
forwardness. 111. (c) 112.(d) 113. (b) 114. (d) 115. (c) 116. (c)
64. (d) Spunk refers to courage or determination, while Timidity 117. (c) 118.(b)
refers to not being brave. 119. (a)A. Umpteen means in large number, countless.
65. (b) Stilted refers to very formal written or verbal B. Waspish means bad tempered.

ww
communication.
66. (a) Prevaricate refers to avoiding /giving direct answer to a
question in order to hide the truth.
D. Testimony means evidence, proof
Umpteen is synonym to option (c).
120.(d) A. Buffoon means a fool.
B. Scholar means a person with good knowledge.

w.E
67. (d) Bilk means to cheat somebody especially by taking money
from them or to evade payment to somebody.
68. (d) Cavil refers to making unnecessary complaints about
something or to criticise for petty reasons while commend
C. Candid means frank, unbiased
D. Gullible means a person who is easily duped or cheated.
A-B are antonym in nature.
121. (d)A. Pandemonium means chaos


69.
74.
(c) 70. (a) 71. (c)
(b) 75. (b) 76. (b) 77. (c)
asy
refers to praising somebody, especially publicly.
72. (a) 73.
78.
(b)
(b)
B. Realm-domain, sphere
C. Sporadic is scattered, occurring casually.
D. Tranquility is peace
79. (c) 80. (a)
81. (c) ‘Whelp’ : a young dog, a puppy, an ill-mannered child or En Thus, A and D are antonym to each other.
122. (d)A. Fidgety means nervous, restless, uneasy.
youth; ‘parent’ is the right antonym.
82. (i) ‘Hirsute’ : hairy, shaggy, untrimmed; ‘hairless’ is the right
antonym. gin B. Elation means to feel proud, cheer
C. Nervous is feeling uneasy.
D. Exuberant means luxuriant, lavish.
83. (a) ‘Denizen’ : an inhabitant or occupant; ‘alien’ (unfriendly,
unfamiliar) is the right antonym.
eer
A and C are synonym to each other.
123.(d) A. Oblivious is forgetful.
B. Kudos means fame renown.
84. (g) ‘Immutable’ : unchangeable; ‘variable’ (changeable) is the
right antonym.
85. (d) ‘Conciliate’ : make calm and amenable, pacify; ‘to antagonise’ ing
C. Conscious means being aware of
D. Adulation- to praise excessively
Thus C is antonym to A.
(evoke hostility or enmity) is the right antonym.
86. (b) ‘Asinine’ : stupid, of or concerning asses; ‘judicious’ (sensible,
prudent) is the right antonym. .ne
124.(b) A. Iconoelast means critic, dissident rebel.
B. Critic means who says and gives opinion about others.
C. Juscious means succulent, juicy
87. (h) ‘Duplicity’ : double-dealing, deceitfulness; ‘honesty’ is the
right antonym.
88. (e) ‘Adulterate’ : debase by adding other or inferior substances;
‘to purify’ is the right antonym.
D. Naive means artless frank.
Thus A and B are synonymous.
125.(b) A. Lugubrious means dismal
B. Cheerful means happy, gay
t
89. (j) ‘Exacerbate’ : make pain, anger etc. worse; ‘to mollify’ or ‘to C. Welter is a confused mixture of things or people.
D. Gaffe means a blunder.
appease’ (to pacify) is the right antonym.
Thus A and B are synonymous.
90. (f) ‘Spurn’ : to reject with disdain, treat with contempt; ‘to 126.(c) A. Paltry means worthless, trifling.
welcome’ is the right antonym. B. Oriental is related to east
91. (b) While bludgeon, black jack and order are both nouns and C. Occidental is to west.
verbs, dragon is only a noun and can’t be used as a verb. D. Obnoxious means unpleasant, offensive
Bludgeon —A short heavy club, usually of wood, that is thicker Thus B and C are antonym.
or loaded at one end tr.v. bludgeoned, bludgeoning, bludgeon– 127.(a) A. Exorbitant means excessively high price.
To hit with or as if with a heavy club. B. Moderate means not in extreme, not excessive
“Blackjack”, a small easily concealable club weapon. C. Ephemeral means short-lived.
Dragon—A dragon is a legendary creature, typically with D. Mammoth large extinct species of elephant.
serpentine or reptilian traits. Thus option (A) shows the antonym variant.
Order is also used for as noun and verb. 128.(b) A. Peccable means imperfect
B. Lassitude mens weariness.
92. (d) “Temporal’, ‘ephemeral’ and ‘transient’ mean ‘lasting a very
C. Jeopardy means great danger or risk.
short time; short-lived; transitory’. This is the exact opposite of D. Perfect is completely good.
‘eternal’. Thus A and D are antonyms.

Downloaded From : www.EasyEngineering.net


Downloaded From : www.EasyEngineering.net

Word Power  l 207

129.(c) A. Gaucherie means clumsiness D. Hyperbole means exaggeration


B. Dexterity means skillful; adroitness Exorcise is to purge the man from evil and bedevil is to
C. Eulogise means praise, extol. trouble thus A is antonym of B.
D. Dearth means scarcity. 142.(c) A. Draconian means severe.
Thus A and B are antonym to each other. B. Cruel mens harsh
130.(a) A. Fallacious means mistaken belief, faulty. C. Bliss means happiness
B. Pusillanimity is cowardly. D. Noble is excellent character, magnanimous.
C. Fortitude-courage in pain or adversity. Thus A is synonym to B.
D. Fallible-one who commits fault. 143.(d) A. Deify to worship as god, make a god of
Thus B is antonym to C. B. Blasphemy is to show disrespect.
131.(a) A. Incarcerate means to imprison, confine. C. Deluge means heavy flood
C. Impeccable means flawless. D. Hubris means pride.
D. Felicitate is to congratulate Thus A is antonym to B.
Thus A is synonym of B. 144.(d) A. Clandestine means secretive
132.(c) A. Disinterested means impartial, not influenced B. Cosmopolitan means including masses
B. Comprehensible means including all. C. Graceful means charm
C. Emulate is to copy. D. Furtive means secretive
D. Impartial means supporting one person or group than Thus A is synonym to D.
another. 145.(d) A. Bellicose means quarrelsome, warlike

ww
Thus both are synonym to each other.
133.(b) A. Inamorata means beloved
B. Beloved means much loved
B. Affront means insult openly.
C. Peaceful means calm, quiet
D. Adroit means skilful.

w.E
C. Imbecile is stupid or person of weak intellect.
D. Novice means beginner.
Thus A and B are synonyms.
134.(a) A. Adumbrate means indicate faintly,
Thus A is antonym to C.
146.(a) A. Frugal means economical
B. Spendthrift means extravagant person
C. Anomaly means abnormality

C. Zany means a clown, buffoon


D. Vulpine means crafty; cunning
asy
B. Overshadow means to throw a shadow over. D. Anathema means a curse
Thus A is antonym to B.
147.(d) A. Asinine means stupid, foolish.
Thus A and B are synonym.
135.(c) A. Cantankerous is bad tempered, quarrelsome
En B. Besotted means made silly by love
C. Tranquil means calm, peace.
B. Querulous means complaining
C. Amicable means friendly
D. Occult means mysterious. gin D. Knowledgable means having lot of knowledge.
Thus A is antonym to D.
148.(c) A. Crass means unrefined
Thus A is antonym to C
136.(a) A. Jouisance means pleasure, joy.
B. Joyousness means merry, happiness. eer
B. Nexus means a connection
C. Elegant means refined, graceful
D. Credulous means gullible.
C. Betrothed means promise to marry, bind with
D. Usurp means seize or assume right.
Thus A and B are synonymous.
149.
ing
Thus A is antonym to C.
(a) “Magnanimous” means ‘generous’ and “benevolence”
means ‘charitableness’.
137.(c) A. Hunky-dory means excellent.
B. Choleric means easily angered.
C. Shoddy means inferior, pretentious.
the given sentence.
.ne
Meanness, selfishness are opposed to the meaning contained in

150. (d) Biannual after every two years opposed the meaning

D. Enervate means to deprive of strength or vigour.
Thus A and C are antonym.
138.(c) A. Certitude means certainty.
B. Hoipolli means common people.
contained in the sentence.
t
151. (d) Enthralling (To hold spellbound; captivate) and alluring
(very attractive or tempting; enticing; seductive) are synonyms.
152. (b) Swoop (to come down upon something in a sudden, swift
C. Certainty means sure. attack) and perturb (to disturb or disquiet greatly in mind;
D. Hostile means unfriendly, enemy agitate) are synonyms.
Thus A is synonymous to C. 153. (c)
154. (d) 155. (a) 156. (b)
139.(a) A. Hood wink is to deceive, cheat 157. (d) 158. (b) 159. (a) 160. (d)
B. Sincere means really believing what you say, not 161. (e) Dictionary Meaning of MASK -
pretending 1. A covering worn on the face to conceal one’s identity,
C. Gauche means tactless. 2. A protective covering for the face or head.
D. Fiasco means failure
Thus A is antonym of B. 162. (d) 163. (a)
140.(a) A. Flummox means bewilder, confuse 164. (c) Dictionary Meaning of MEAN -
B. Enlighten means to give knowledge, clarify 1. (a) To be used to convey; denote.
C. Fastidious means not easy to please (b) To act as a symbol of; signify or represent
D. Esoteric means intelligible to those with special 2. To be mean – cheap, selfish or negativity attached to
knowledge. it.
Thus A is antonym to B. 165. (b) Dictionary Meaning of ALONE -
141.(d) A. Exercise means to drive out the evil, 1. Being apart from others; solitary.
B. Bedevil is confuse, trouble 2. Being without anyone or anything else; only.
C. Diatribe is bitter criticism in speech.

Downloaded From : www.EasyEngineering.net


Downloaded From : www.EasyEngineering.net

208  l  Word Power

3. Considered separately from all others of the same 190. (c) Wardrobe
class. 191. (a) Fabricate means make up something artificial or untrue
4. Being without equal; unique while demolish means destroy completely which is just
Synonyms: alone, lonely, lonesome, solitary opposite.
166. (c) emerging/ nascent mean almost the same (just coming up) 192. (c) Gregarious denotes tending to form a group with others
of the same species and unsociable is the opposite.
167. (b) eccentric / abnormal mean almost the same (strange
193. (a) Pragmatic means concerned with practical matters while
behaviour) indefinite denotes vague or not clearly defined or stated.
168. (e) abundance and plethora mean the same ( large quantity 194. (d) Ambiguous means having more than one possible
of something) meaning.
169. (a) purposefully / inadvertently are opposites , former means 195. (a) If someone or something is infallible, that means they are
deliberately and latter by mistake incapable of failure or error.
196. (d) Conscientious is the one who is guided by or in accordance
170. (e) germane / irrelevant are opposites – germane means
with conscience or sense of duty and right and wrong.
relevant 197. (d)
171. (a) same 198. (c) Obsolete means no longer in use
172. (c) opposite 199. (c) regicide means the act of killing a king.
173. (b) opposite 200. (a) 0Aviary is a building where birds are kept.

ww
174. (d) same
175. (e) same
176. (b) The meaning of word Vociferous (Adjective) is :
201. (a) condone and forgive are similar in meaning.
202. (b) Analogy denotes comparison.
203. (c) Allure and entice both denote provoking someone to do
something through (often false or exaggerated) promises

w.E
outspoken, blunt.
Its synonym should be : Loud.
177. (c) The meaning of word Fictional (Adjective) is :
or persuasion.
204. (d) ‘Persist’ means ‘to insist’.
205. (c) ‘Eventually’ means ‘finally’.
Its synonym should be : fanciful
asy
Imaginary, unreal, fabricated, mythical

178. (d) The meaning of word Trivial (Adjective) is : not important.


206. (c) ‘Impeccable’ means ‘flawless’.
207. (c) Dominate (Verb) = to control or have a lot of influence
over somebody/something; to be the most important
Its synonym should be : ordinary.
179. (c) The meaning of word Impudent (Adjective ) is :
En feature of something.
Monopolise (Verb) = to have or take control of the larger
Insolence.
It’s synonyms should be : Insolent.
180. (a) The meaning of word pompous (Adjective) is : self gin part of something.
Hence, synonymous relationship.
208. (b) Equitable (Adjective) = fair and reasonable; treating
Important.
It’s synonym should be : Pretentious.
181. (a) The meaning of word cultivated (Adjective) is : Eductated. eer
everyone in an equal way.
Impartial (Adjective) = not supporting one person or
group more than other; unbiased.
It’s antonym should be : Crude.
182. (d) The meaning of word Impertinent (Adjective) is : Ill ing
Hence, synonymous relationship.
209. (c) Flourish (Verb) = to develop quickly and be successful;


mannered, disrespectful.
It’s antonym should be : courteous.
183. (b) The meaning of word Divulge (verb) is : Reveal, make
to grow well; thrive.
Look at the sentence:
.ne
Few businesses are flourishing in the present economic
known.
Its antonym should be : Dissemble.
184. (a) The meaning of word Appreciation (Noun) is :
Thankfullness.
climate.
t
210. (a) Affable (Adjective) = pleasant, friendly and easy to talk
to; genial.
Rude (Adjective) = discourteous; impolite; showing lack
It’s antonym should be : Aspersian. of respect for other people.
185. (c) The meaning of word Supple (Adjective) is : Flexible. Hence, antonymous relationship.
It’s Antonym should be : Rigid. 211. (e) Partial (Adjective) = showing or feeling too much support
186. (b) canister for one person, idea etc. in a way that is unfair; biased.
187. (c) Domicile Dispassionate (Adjective) = not influenced by emotion;
188. (a) Irrevocable impartial.
189. (a) Gratuity Hence, antonymous relationship.

Downloaded From : www.EasyEngineering.net


Downloaded From : www.EasyEngineering.net

13
PTER
CHA

Relationship Based Problems

I ww
n this section you are given two words, and you have to understand the relationship that exists between those two words. For
example, they could be synonymous to each other, or differ in terms of degree or intensity of something. After knowing this
relationship, you have to choose which of the given options matches this relationship the most. (Each option will have two words

w.E
which may or may not be related).

TERMINOLOGY

asy
(1) Relationship: Relationship refers to the state of connectedness between entities. Talking in terms of words, a relationship would
define how two words are related or connected to each other. We need to precisely understand the meanings of both the words
to be able to derive or infer the appropriate relations between them.

En
(2) Antonyms: Anti is a Greek root which means opposite. A word that has the opposite meaning of another word is its antonym, i.e.,
antonym of a particular word has an opposite meaning. For example, the antonym of 'stop' is 'go'. The words stop and go are said

words.
gin
to be antonymous to each other. So, the relationship between such two words is antonymous. Antonyms are also called opposite

A lot of antonyms are pretty obvious and evident, like good and bad. Some words can be easily changed into their antonyms by

eer
adding the prefix un, non, in, for e.g., the antonym of like is unlike; contagious and non-contagious; expensive and inexpensive
and so on.

ing
(3) Synonyms: A word that has the same meaning to some other word is its synonym, i.e., synonym of a word is that word which
has the same meaning. For example, a synonym of funny is laughable. A word with the exact same meaning of another word is
tough to find, because English language offers us different shades/degrees of an expression, but not always the exact synonyms.

.ne
For example: angry and furious can be considered synonymous but furious is a higher degree of anger. This means that when we
use ‘angry’ we mean someone has lost his/her temper, whereas when we use ‘furious’, we mean that the person is extremely angry.
(4) Homophones: A homophone is a word which sounds similar to another word, but has a different meaning/spelling i.e., these

t
words sound the same, but have different meanings. For example: right and write sound similar but right means correct; whereas,
write refers to communicating or expressing by writing. The words right and write are said to be homophonous to each other,
hence the relationship between such words is termed as homophonous. Homophones can give rise to ambiguity when we are
listening, but in such cases we have to use our common sense or logic to choose the apt word.
(5) Rhyming Words: Rhyming words can be differentiated from homophonic words in terms that the rhyming words sound similar
with respect to the last syllable. Rhyming words are often used to beautify a piece of literary work, especially poetry. Examples of
rhyming words: bat and hat, found and sound, etc. Talking in terms of the questions asked in exams, very few are normally based
on this relationship.
(6) Intensity: Intensity refers to the amount, degree, level or magnitude. Two words are said to differ in intensity if they refer to the
same thing, but on different levels. For example: happy and ecstatic: both the words refer to the state of being happy, but ecstatic
means a state of great delight or very happy.

COMMON MISTAKES
(1) Selecting the first relationship that looks similar: It is possible that more than one option would be depicting a similar
relationship. We have to make sure that we select the one which is the closest to the one mentioned in the question.
(2) Synonymous or not: It can be quite tricky to understand whether the given words are synonymous, or they just talk about the
same thing but at different levels or intensity. The only way to be clear about this possible problem is to understand the word
given in the question carefully, and see which option would best replicate that relationship.

Downloaded From : www.EasyEngineering.net


Downloaded From : www.EasyEngineering.net

210  l  Relationship Based Problems

(3) Watch out for homophones: Questions which have homophonic words might not have any other evident relationship between
them. So after seeing two words which do not seem to have any relationship, you may get confused as to how to select the right
option. In such cases, always check out if those words are homophonous to each other.
(4) Opposite relationship: Relationship between two words A & B might not be similar to the relationship between B & A. For
example – Relationship between brick and wall is of ‘component to whole’, but this relationship is not similar to that between
book and page, as the relationship here is of ‘whole to component’. You can get trapped this way into making a mistake.
(5) Subject of comparison: If the given question has two words which refer to the same subject or thing at different levels, then the
most appropriate option would be the one which refers to that same subject. (In case, there are more than one option that depict
this relationship of varying levels or degree)
TIPS
(1) Wrong relationships: When creating a relationship between two entities, ensure that you identify them as nouns, adjectives
or verbs, adverbs etc. This helps because sometimes in the flow of the test, you establish a relationship, pick out the seemingly
correct answer, only to realise that there are actually two very similar correct options. How do you distinguish between these?
The ‘Parts of Speech’ angle works here. Make sure that the two words in the option belong to the same part of speech like their
counterparts in the question.
(2) It is always advisable to re-read and analyse the words in the question at least twice.

ww
(3) Make sure you do not end up spending too much time in these questions, which you may, if the understanding of the relationship
looks complex.
HOW TO SOLVE

w.E
(1) The questions will give you two words, and you would be asked to pick the option which replicates this relationship. So, first of
all, understand these two words individually.
(2) Analyse how these two given words can be logically related to each other.

asy
(3) It is advisable to read all the options, even if you think you have spotted the same relationship before doing so.
(4) Select the most appropriate option from the given ones, and you are done.
EXAMPLES
Directions (Examples 1-5):  Select the best alternative. En
Example 1.
India: Delhi gin
(a) Australia: England (b) Bombay: Delhi
eer
(c) USA: Washington D.C. (d) Europe: Australia.
Explanation (c): Delhi is India’s capital. Similarly, Washington is USA’s capital.
Example 2.
I : eye
(a) bye : by (b) sound : round (c) you : me
ing (d) car: truck

.ne
Explanation (a): ‘I’ and ‘eye’ are homonyms (words that are pronounced or spelled the same way but have different meanings). The
only other option like this is option (a).
Example 3.
Car : tyre
(a) mechanic: engine (b) building: brick (c) engine: steering wheel (d) glass: pen
t
Explanation (b): Tyres are a part of the whole car. Similarly, bricks are a part of a building.
Example 4.
4. Simple : challenging
(a) study: examination (b) sane: crazy (c) house: office (d) son: father
Explanation (b): ‘Simple’ and ‘Challenging’ are antonyms. Similarly, ‘sane’ and ‘crazy’ are antonyms.
Example 5.
Absent : present
(a) pre: post (b) late: early (c) monday: sunday (d) swimming: walking
Explanation (b): ‘Absent’ and ‘Present’ are antonyms. Similarly, ‘late’ and ‘early’ are synonyms. Option (a) is close, but since they are
not complete words but prefixes, we will choose option (b).

Downloaded From : www.EasyEngineering.net


Downloaded From : www.EasyEngineering.net

Relationship Based Problems  l 211

Practice Exercise
LEVEL-I
DIRECTIONS (Qs. 1-42):  In these questions, two words will 15. BEWILDERMENT : CONFUSION : :
be given to you, and these two words will have some kind of a (a) Bursa : sack (b) Bewitched : alliteration
relationship which relates them in a particular way. After finding (c) Fantod : nervousness (d) Coracle : lodestar
this relationship, you have to pick the right option which has words 16. PREAMBLE : STATUTE : :
related in a similar way: (a) Prologue : novel (b) Movement : sympathy
1. LYING : PERJURY : : (c) Sketch : drawing (d) Index : book
(a) Statement : Testimony (b) Seeing : Observing 17. SAUNTER : STROLL : :
(c) Taking : Stealing (d) Eating : Dining (a) Amble : path (b) Gyrate : twist
2. PREHISTORIC : MEDIEVAL : : (c) Perambulate : walk (d) Baby carriage : walk
(a) Akbar : British (b) Present : Future

ww
(c) Shakespeare : Tennyson (d) Colossus : Elephant
3. LOUD : STENTORIAN : :
18. AUTHOR : MANUSCRIPT : :
(a) Optician : spectacles
(c) Architect : blueprint
(b) Engineer : bridge
(d) Doctor : stethoscope

w.E
(a) Mild : Noisy

4. INDEFATIGABLE : INVETERATE : :
(a) Tireless : Tired
(b) Painful : Prickly
(c) Adjective : Descriptive (d) Bright : Resplendent

(b) Tired : Habitual


19. LIBERTINE : IMMORALITY : :
(a) Victim : depravation
(c) Altruist : selflessness
(b) Miser : selfishness
(d) Policeman : law
(c) Tireless : Habitual
5. MISANTHROPE : HUMANITY : : asy
(d) Impoverished : Habitual 20. SIMMER : BOIL : :
(a) Glide : drift (b) Drizzle : downpour
(a) Chauvinist : Patriot
(c) Agnostic : God
En
(b) Misogynist : Women
(d) Witch : Magic
(c) Gambol : play
21. SEETHE : ANGER : :
(d) Stagnate : flow

6. BACTERIA : ILLNESS : :
(a) Medicine: Germs
(c) Knife: Laceration
(b) Calcium: Bones
(d) Fire: Explosion gin (a) Chortle : distress
(b) Snarl : confusion
(c) Fidget : uneasiness
7. POETRY : PROSE : :
(a) Stanza : Chapter (b) Art : Fiction eer
(d) Waddle : embarrassment
22. KEY : IGNITION : :
(c) Clause: Sentence
8. SIP: GULP : :
(a) Soup : Water
(d) Novel: Variation

(b) Tent: Hut ing


(a) Helmet : motorcycle
(c) Switch : light
(b) Boot : saddle
(d) Pad : helicopter
(c) Touch: Push
9. ROOM : HOUSE : :
(d) Cup: Glass
23. TALLY : VOTES : :
(a) Census : population
(c) Taxation : revenue .ne
(b) Government : laws
(d) Team : athletes
(a) Chair : Room
(c) Wheel: Car
10. HANDCUFFS : PRISONER : :
(a) Shoes : Feet
(b) Cabin: Ship
(d) Cockpit: Plane

(b) Leash: Dog


24. CACOPHONOUS : HARMONIOUS : :
(a) Beauty: peace t
(b) Bellicose: pacific
(c) Tempestuous: stormy (d) Considerate: sympathetic
(c) Tail: Kite (d) Ring: Finger 25. MITIGATE : PUNISHMENT : :
11. LORRY : TRUCK : : (a) Place: placement (b) Change: order
(a) Torch : Light (b) Motor : Car (c) Monotonous: change (d) Alleviate: pain
(c) Coal : Coke (d) Petrol : Gasoline 26. PRAISE : EXTOL : :
12. ACTION : REACTION : : (a) Stuff: cram (b) Hurry : run
(a) Introvert : Extrovert (b) Assail : Defend (c) Insure : rely (d) Dislike : loathe
(c) Diseased : Treatment (d) Death : Rebirth 27. INSIPID : PIQUANT : :
13. WRITING : PLAGIARISM : : (a) Relish: spice (b) Tasty: bland
(a) Confidence : Deception (c) Sweet: dessert (d) Flat: spicy
(b) Money : Misappropriation 28. SURGEON : SCALPEL
(c) Gold : Theft (a) Musician : Instrument (b) Carpenter : Cabinet
(d) Germ : Disease (c) Sculptor : Chisel (d) Baker : Oven
14. CONDENSATION : REFRIGERATE : : 29. TRIANGLE : HEXAGON
(a) Evaporation : heat (b) Consumption : cook (a) Cone : Sphere (b) Rectangle : Octagon
(c) Oration : listen (d) Exhaustion : buy (c) Pentagon : Heptagon (d) Angle : Quadrilateral

Downloaded From : www.EasyEngineering.net


Downloaded From : www.EasyEngineering.net

212  l  Relationship Based Problems

30. STARE : GLANCE DIRECTIONS (Qs. 43-50): In the following questions, two
(a) Gulp : Sip (b) Confide : Tell words will be given to you, and these two words will have some
(c) Hunt : Stalk (d) Step : Walk kind of relationship which relates them in a particular way. After
31. CLOTH : TEXTURE
finding this relationship, you have to pick the option which has
(a) Body : Weight (b) Silk : Cloth
(c) Wood : Grains (d) Ornaments : Gold words related in a dissimilar way:
32. UNHAPPY : MISERABLE : : 43. DULCET : RAUCOUS
(a) Vocal : Quiet (b) Mournful : Oblivious (a) Sweet : Song (b) Crazy : Sane
(c) Passionate : Appreciative (d) Content : Euphoric
(c) Palliative : Exacerbating (d) Theory : Practical
33. SNAKE : REPTILE : :
(a) Snake : Cobra (b) Bird : Mammal 44. OPENING : INAUGURATION : :
(c) Wasp : Insect (d) Iion : Amphibian (a) Memory : Commemoration
34. ILLITERATE : EDUCATION : : (b) Display : Exhibition
(a) Naive : experience (b) Well-mannered : rude (c) Admission : Initiation
(c) Miserly : money (d) Pristine : elegance
(d) Solemn : Vestment
35. OPAQUE : LIGHT : :
(a) Absorbent : liquid (d) Desert : sand 45. OVATION : APPLAUSE : :
(a) Grief : Loss (b) Rout : Defeat

ww
(c) Autoclave : germ-free (d)
36. MOSAIC : TILE : :
(a) Fabric : tapestry
(c) Document : author
(b)
(d)
Hermetic : air

Portrait : paint
Coverlet : cloth
(c) Triumph : Failure
46. MANAGER : OFFICE : :
(d) Pathway : Ruin

w.E
37. FEATHER : BIRD : :
(a) Snake : scale
(c) Human : epidermis
(b)
(d)
Bark : tree
Hump : camel
(a) Doctor : Patient
(c) Bank : Account
47. BRAVE : VALOUR : :
(b) Curator : Museum
(d) Fruit : Seed

38. WATER : TIP : :


(a) Professor : tenure
(c) Bank teller : interest
(b)
(d) asy
Pharmacist : prescription
Sales person : commission
(a) Look : See
(c) Man : Management
(b) Cynic : Doubt
(d) Thief : Victim
39. PUPIL : EYE : :
(a) Elbow : Wrist (b) Teeth : Mouth
En 48. TREASURE : CHEST : :
(a) Money : Account (b) Coins : Mint
(c) Arm : Leg
40. ROCK : MARBLE : :
(a) Ebony : Ivory
(d)

(b)
Fist : Hand

Woods : Mahogany gin (c) Finance : Exchequer


49. WOOL : WARMTH : :
(a) Radio : Broadcast
(d) Cash : Vault

(c) Scissors : Glue


41. SAPLING : TREE : :
(a) Puppy : Dog
(d)

(b)
Wings : Bird

Canine : Feline
eer
(b) Person : Success
(c) Marketing : Advertising
(c) Cat : Lion
42. HINT : SUGGESTION : :
(d) Poodle : Terrier
ing
(d) Spring : Elasticity
50. UMPIRE : GAME : :
(a) Shade : Spectrum
(c) Nuance : Distinction
(b)
(d)
Trace : Existence
Remnant : Preservation
(a) Chef : Banquet
(c) Moderator : Debate .ne
(b) Legislator : Election
(d) Prodigy : Wonder

LEVEL-II
t
DIRECTIONS (Qs. 1-44):  For each of the following questions 5. STYGIAN: DARK : :
select the answer pair that expresses a relationship most similar to (a) Abysmal : Low (b) Cogent: Contentious
that expressed in the capitalised pair. (c) Fortuitous : Accidental (d) Cataclysmic : Doomed
1. EASE : ALLEVIATE : : 6. CONTIGUOUS : ABUT : :
(a) Hint : Allocate (b) Revolt : Repudiate (a) Possible: Occur (b) Synthetic: Create
(c) Collapse : Rise (d) Question : Interrogate (c) Simultaneous: Coincide (d) Constant: Stabilise
2. SECRET : CLANDESTINE : : 7. FRAGILE : CRACK : :
(a) Overt : Furtive (b) Covert : Stealthy (a) Irreducible : Reduce (b) Cemetery : Death
(c) Open : Close (d) News : Rumour (c) Hydro: Water (d) Pliable: Bend
3. LIMPID : MURKY : : 8. REHEARSAL : PERFORMANCE : :
(a) Dazed : Cloudy (b) Obscure : Vague (a) Satisfaction : Appetite (b) Applause: Audience
(c) Bright : Gloomy (d) Nebulous : Dim (c) Engagement: Marriage (d) Entrapment: Game’
4. DRAMA : AUDIENCE : : 9. SORROW : DEATH : :
(a) Brawl : vagabonds (b) Obscure : vague (a) Laugh : Cry (b) Happiness: Birth
(c) Art : critics (d) Movie : actors (c) Fear: Hate (d) Confusion: Anger

Downloaded From : www.EasyEngineering.net


Downloaded From : www.EasyEngineering.net

Relationship Based Problems  l 213

10. ISLAND : OCEAN : : 28. DEFORESTATION : TREES ::


(a) Hill : Stream (b) Forest: Valley (a) Inoculation : Vaccine
(c) Tree: Field (d) Oasis: Desert (b) Defamation : Enemies
11. LOYALTY : TRAITOR : : (c) Amnesty : Deserters
(a) Truthfulness : Liar (b) Hope : Optimist (d) Sterilization : Microorganism
(c) Diligence : Worker (d) Understanding : Sage 29. KERNEL : SHELL : :
12. PREHISTORIC : MEDIEVAL (a) Caterpillar : Pupa (b) Larva : Cacoon
(a) Akbar : British (b) Present : Future (c) Lassitude : Syncope (d) Passenger : Car
(c) Shakespeare : Tennyson (d) Colossus : Elephant 30. CICERONE: SIGHTSEER
13. DOUBT : FAITH (a) Understudy : Actor (b) Audio : Video
(a) Atheist : Religious (b) Sceptic : Pious (c) Baedeker : Tourist (d) Mentor : Guidance
(c) Iconoclast : Idol (d) Apostate : State 31. QUISLING: TREASON
14. PREMISE : CONCLUSION (a) Accomplice : Guilt (b) Murderer : Homicide
(a) Assumption : Inference (b) Hypothesis : Theory (c) Renegada : Loyalty (d) Perjurer : Arson
(c) Knowledge : Ideas (d) Brand : Marketing 32. POLLYANNA: OPTIMISM
15. LOVE : AFFECTION (a) Diehard : Resistance

ww
(a) Happiness : Joy
(c) Enemy : Hatred
16. BOTANY : PLANTS ::
(b) Amity : Harmony
(d) Sorrow : Misery
(b) Environmentalist : Pollution
(c) Malingerer : Illness
(d) Reactionary : Change

w.E
(a) Geology : Earth
(c) Teacher : Class
17. EAT : GORGE ::
(b) Stars : Astronomy
(d) Disease : Etiology
33. LARGE : GARGANTUAN:
(a) Emaciated: Thin
(c) Obese: Stout
(b) Wise: Machiavellian
(d) Small: Tiny
(a) Drink : Guzzle
(c) Expel : Admit
18. LONELINESS : PRIVACY ::
asy
(b) Saunter : Walk
(d) Advanced : Primitive
34. INTERLUDE : REST
(a) Vacation : Work
(c) Sabbatical : Freedom
(b) Retirement : Retreat
(d) Intermission : Respite
(a) Passion : Apathy
(c) Destiny : Fateful En
(b) Composure : Equanimity
(d) Illusory : Unpredictable
35. RACISM : APARTHEID
(a) Sexism : Chauvinism (b) Parochialism : Linguism
19. FAKE : GENUINENESS ::
(a) Flawless : Perfection
(c) Deceitful : Intent
(b) Nebulous : Definition
(d) Amoral : Wicked
gin (c) Nationalism : Identity (d) Communalism : Religion
36. SECULAR : SPIRITUAL

20. PHYSICIAN : SURGEON ::


(a) Nurse : Doctor eer
(a) Amoral : Moral
(c) Noisome : Hostile
37. SINISTER : HARM
(b) Wet : Dank
(d) Praise : Embarrassment

(b) Optometrist : Ophthalmologist


(c) Urologist : Gynecologist
(d) Optician : Obstetrician
ing
(a) Malignant : Humor
(c) Ominous : News
(b) Horrifying : Trepidation
(d) Benevolent : Bountiful

21. BULL : BUYS ::


(a) Wolf : Defrauds (b) Lion : Brokers
38. LIMPID : MURKY
(a) Dazed : Clouded
(c) Bright : Gloomy .ne
(b) Obscure : Vague
(d) Nebulous : Dim
(c) Tiger : Speculates
22. FICKLE : STABLE : :
(a) Laughter : Barn
(d) Bear : Sells

(b) Equine : Horse


(c) Impulsive : Considered (d) Fundamental : Elemental
39. PARADIGM : PATTERN
(a) Skeleton : Flesh
(c) Plant : Genus
t
(b) Method : System
(d) Dinosaur : Tyrannosaurus
40. MALAPROPISM : WORDS
23. SOUND : DECIBEL ::
(a) Anachronism : Time (b) Ellipsis : Sentence
(a) Earthquake : Tremors (b) Noise : Intensity
(c) Jinjanthropism : Apes (d) Catechism : Religion
(c) Light : Radiance (d) Length : Metres
41. ANTERIOR : POSTERIOR
24. MAMMAL : COW ::
(a) In : Out (b) Top : Bottom
(a) Woods : Tree (b) School : Fish
(c) Head : Tail (d) Front : Rear
(c) Reptile : Snake (d) Tusk : Elephant
42. IMPECUNIOUS: MENDICANT: :
25. EXTRACT : TOOTH ::
(a) Cut : Nail (b) Uproot : Stump (a) Prodigal: Philanthropist (b) Petulant: Complainer
(c) Pull out : Pin (d) Cut : Wood (c) Quizzical : Critic (d) Compulsive : Liar
26. HUG : AFFECTION :: 43. ERROR: INFALLIBLE : :
(a) Dour : Humous (b) Nod : Assnet (a) Emotion : Invulnerable (b) Defect : Intolerable
(c) Virus : Influenza (d) Feet : Walk (c) Flaw: Impeccable (d) Cure : Irreversible
27. ADMONISHMENT : CASTIGATION :: 44. TEARS: LACHRYMOSE : :
(a) Anxiety : Fear (b) Perjury : Corruption (a) Words : Verbose (b) Speeches: Morose
(c) Provocation : Instigation (d) Peccadillo : Sin (c) Jests : Ironic (d) Requests: Effusive

Downloaded From : www.EasyEngineering.net


Downloaded From : www.EasyEngineering.net

214  l  Relationship Based Problems

DIRECTIONS (Qs. 45-59):  Of the given four pairs three pairs 54. CHATTER : TALK : :
express the relationship similar to that expressed in the capitalised (a) Scurry : Move (b) Sleep : Drowse
pair. Select that pair which is not related in this way. (c) Limp : Walk (d) Chant : Sing
45. EDITOR : NEWSPAPER : : 55. TEPID : HOT : :
(a) Lecturer : University (b) Teacher : School (a) Winter : Summer (b) Pat : Slap
(c) Nurse : Hospital (d) Architect : Design (c) Storm : Rain (d) Topple : Tumble
46. INCUBATOR : INFANT : : 56. TRAITOR : COUNTRY : :
(a) Cooler : Wine (b) Archives : Document (a) Potentate : Kingdom (b) Bureaucrat : Government
(c) Hive : Bee (d) Greenhouse : Plant (c) Apostate : Religion (d) Tailor : Law
47. INVESTMENT : SOUND : : 57. BARGE : VESSEL : :
(a) Technique : Inept (b) Physique : Sturdy (a) Shovel : Instrument (b) Book : Anthology
(c) Fabric : Silk (d) Intellect : Noble
(c) Rim : Edge (d) Training : Preparation
48. PLANT : PARASITE : :
58. SAIL : SHIP : :
(a) Transient : Permanent (b) Wild : Prey
(a) Propeller : Dog (b) Radar : Satellite
(c) Flora : Fauna (d) Sailor : Pirate
(c) Hydrogen : Balloon (d) Accelerator : Car
49. TORCH : LIBERTY : :
59. Trailer : Picture : :

ww
(a) Laws : Court
(c) Balance : Equity
50. WHISPER : SPEAK : :
(b) Scales : Justice
(d) Weights : Measure
(a) Truck : Cargo
(c) Edition : Novel
(b) Theatre : Play
(d) Commercial : Product
60. If ‘Asinine’ is for ‘Donkey’, then

w.E
(a) Listen : Bear
(c) Brush : Touch
51. PHILATELIST : STAMPS : :
(b) Request : Ask
(d) Heat : Chill
(a) ‘Vulpine’ is for ‘Fox’
(c) ‘Avian’ is for ‘Cow’
61. If Stallion is for ‘Mare’ then
(b) ‘Vulpine’ is for ‘Vulture’
(d) ‘Avian’ is for ‘Dove’

(a) Carpenter : Saw


(c) Numismatist : Coins
52. MINER : QUARRY : :
asy
(b) Runner : Sneakers
(d) Astrologer : Predictions
(a) ‘Ewe’ is for ‘Ram’ (b) ‘Ram’ is for ‘Ewe’
(c) ‘Goose’ is for ‘Gander’ (d) ‘Sow’ is for ‘Boar’

(a) Carpenter : Saw


(c) Poet : Sonnet
(b) Painter : Brush
(d) Sculptor : Atelier En 62. Actor is to Thespian as Harlequin is to :
(a) Conjuror (b) Hypnotist

53. GLOSSARY : WORDS : :


(a) Lexicon : Words (b) Catalogue : Dates gin (c) Comedian (d) Buskar
63. Venom is to Toxin as Venial is to :
(a) Venal (b) Maleficence
(c) Atlas : Maps (d) Thesaurus : Rhyme

eer (c) Dislike (d) Gall

Hints & Solutions ing


LEVEL-I .ne
by option (b). Options (a) and (c) do relate in the way that

1. (a) Perjury means the offence of wilfully telling an


untruth or making a misrepresentation under oath. So the
relationship between lying and perjury is one of degree or
t
Akbar existed before the British took control of India, and
Shakespeare came before Tennyson did, but these are not
the same as that expressed in the question. Both items have
to be of the same type, i.e. time period.
extent. The correct relationship is in option (a), as testimony 3. (d) Stentorian refers to very loud, hence the relationship in
means a formal, written or spoken statement, especially one
the given words is one of ‘degree’. This relationship of degree
given in a court of law. Thus this also represents a ‘degree’
can be seen in option (d) as resplendent can refer to being
relationship. In all the other options, this relationship of
very bright or shiny. Option (a) is wrong, because ‘mild’
degree doesn’t exist. In (b), seeing and observing are distinct
is a general term, and isn’t especially talking about mild
activities, in option (c), stealing is a form of taking, it doesn’t
voice or sound; whereas, in the question, degree is asked of
tell the extent to which something is taken and similarly
the same thing, i.e. sound. Option (c) is wrong, because it
in option (d), dining means having dinner, which involves
eating. just doesn’t refer to the same relationship: painful refers to
2. (b) Prehistoric refers to ‘something related to or denoting something that gives physical pain or distress, and prickly
the period before written records’ and Medieval refers to refers to something that is very irritable, which can be used
something related to the Middle Ages. The relation is of to define someone’s nature; The degree relationship in the
‘time’ and precedence, i.e. the medieval period followed the same sense, is not evident. Option (b) doesn’t display the
prehistoric period. This relation of order of time is depicted same relationship.

Downloaded From : www.EasyEngineering.net


Downloaded From : www.EasyEngineering.net

Relationship Based Problems  l 215

4. (c) 'Indefatigable' means incapable of being fatigued. correct relationship is drizzle : downpour, as drizzle precedes
Tireless is the correct synonym. Inveterate means to be downpour as simmer is a situation of cooking just below the
confirmed in a habit. Habitual is the correct synonym. boiling point.
5. (b) 'Misanthrope' is a person who hates or distrusts 21. (c) Seethe is to be extremely angry. Similarly fidget is to
humankind, one who is against humanity. A misogynist is a keep moving your body because you are nervous, bored,
person who is against women. excited etc., which is similar in meaning to uneasiness.
22. (c) A key is used to start the ignition. Similarly a switch is
6. (d) Relation - Cause - effect;
used to start the light.
Bacteria causes illness, as fire causes explosion, laceration
23. (a) Tally is to calculate the number of something, say votes
means tearing of flesh and it may be caused by a knife, but in this case. Similarly census is the process of counting
that is only an accidental effect and not a permanent cause. something, say population.
7. (a) Relation - different kinds of writing forms; stanza and
24. (b) Cacophonous (meaning being loud) is opposite in
chapter are also different kinds of styles of composition meaning to harmonious (meaning friendly or peaceful).
different from each other, clause is a part of a sentence and Similarly bellicose (meaning warlike) is opposite in meaning
some art can be fiction. to pacific (meaning peaceful).

ww
8. (c) Relation - degree of intensity; just as gulp is higher
in degree of intensity to sip, push is higher in degree of
intensity to touch.
25. (d) Mitigate means to lessen something, say punishment in
this case. Similarly alleviate means to lessen something, say
pain in this case.

w.E
9. (d) Relation - Part - Whole; room is an essential unit of a
housing entity, cockpit is also an essential unit of a plane,
26. (b) Extol is to praise somebody very much. So it is a higher
degree of praise. Similarly loathe is a higher degree of dislike.
Note that hurry doesn’t necessarily means to run.

an exclusive unit of car.


asy
cabin is not an exclusive unit of ship similarly wheel is not

10. (b) Relation - binding thing - binded subject;



27. (d) Insipid (meaning tasteless) is opposite to piquant
(meaning spicy taste). Similarly flat is related to spicy.

En
Like a prisoner is chained with handcuffs, dog is chained by
a leash.
28. (c) Here worker-tool relationship has been shown. Surgeon
uses scalpel to perform operation. Similarly, Sculptor uses
chisel to carve out statue.
11. (d)
12. (b) Reaction is in response to an action just as a defence gin
29. (b) A triangle has three sides while a hexagon has six sides.
Similarly, a rectangle has four sides and an octagon has eight
is in response to an assail. All other pairs have a different
relationship. 30.
sides.
eer
(a) First word is of higher intensity than the second word.
13. (b) Plagiarise is to copy writings and present them as one’s
own. Similarly, misappropriation of money is to apply
31.

ing
(c) Texture of cloth gives an idea of cloth whether it is rough
or somooth. Similarly, grain is the pattern of lines seen or
felt in wood.
wrongfully or dishonestly, as funds entrusted to one’s care.
14. (a) When we refrigerate, there is condensation or cooling,
32.
.ne
(d) This problem is based on degree of intensity. Someone
who is miserable is extremely unhappy while someone who
and similarly, on heating, there is generally evaporation.
15. (c) Bewilderment is a state of confusion and fantod is a
state of extreme nervousness or restlessness.
16. (a) Preamble is opening/introduction and statute a written
33.

34.
is euphoric is extremely.
t
(c) The relationship here is of member and class. A wasp is
a kind of insect, as a snake is a kind of reptile.
(a) This is a degree of intensity analogy. Someone who is
law. Similarly, prologue is an introduction to a play or a iliterate lacks an education, while someone who is naive
poem, etc. lacks experience.
17. (b) Saunter means to walk in a slow relaxed way. Hence it is
35. (d) Something that is opaque blocks light completely.
a synonym of stroll. Similarly gyrate, which means to move Likewise, something that is hermetic is completely sealed
around (a part of body) in circles is synonyms with twist. against the escape or entry of air.
18. (c) An author writes a manuscript. Similarly an architect 36. (d) A coverlet is made up of many pieces of cloth, just as
designs a blueprint. mosaic is made up of many pieces of tile.
19. (c) Libertine is a person (man) who leads an immoral life. 37. (b) This is a part to whole problem. The bird is covered by
Similarly an altruist is a person who leads a selfless life. its feathers completely, just as a tree is covered completely by
20. (b) Simmer is to cook something just below the boiling its bark. Neither (a) or (c) can be correct since the order is
point. Glide and drift means quick or steady movement. reverse.
Drizzle is light rain whereas downpour is a sudden heavy 38. (d)
rain. Stagnate is to stop something, say the flow of water. 39. (b) Just as the pupil is a part of the eye. so also the teeth are
Gambol is to jump or run about in a lively way. So the part of the mouth.

Downloaded From : www.EasyEngineering.net


Downloaded From : www.EasyEngineering.net

216  l  Relationship Based Problems

40. (b) 2. (b) The given words in question are synonyms, as both
41. (a) A sapling is a baby tree, just as a puppy is a baby dog. can refer to ‘conducted with or marked by hidden aims
42. (c) or methods’ and the words given in option (b) are also
43. (a) Dulcet refers to ‘something which is very pleasing synonyms as both can refer to secret or something that’s
or pleasant’; whereas, raucous means ‘something that’s not done openly. Option (a) is wrong, because it doesn’t
very loud, harsh and unpleasing’. The given words are depict the same relationship: overt can refer to observable or
antonymous. something that’s not secret or hidden whereas, furtive refers
Option (a) is right because sweet or pleasing cannot be to something that is done quietly or cautiously. Option (c)
antonymous to song. A song can, in fact, be very sweet or is wrong: open is the opposite of close; these are not similar
pleasing. words. Option (d) is wrong, because news and rumour are
again, antonyms.
Option (b) is similar to the relationship in the given question
3. (c) The relationship between the given words is of
as words in it are antonyms to each other: crazy means
antonyms: ‘limpid’ refers to clear and bright, whereas murky
lacking sanity or showing madness whereas sanity refers to
means dark or gloomy. So, it’s easy to see that option (c)
having a proper and logical state of mind.
is correct. Option (a) is wrong, because dazed refers to a

ww
Option (c) is wrong, because palliative is something which
smoothens or reduces pain or sorrow, but isn’t expected to
cure or remove the trouble. For example: a heating pack
state of confusion or a state of mental numbness, whereas
cloudy refers to something that is not clear. Option (d) is
wrong, because nebulous means ‘lacking definition or form’

w.E
is a palliative when it is implied to temporality reduce the
pain due to strained muscles. Exacerbating refers to making
things worse or making a situation more painful or difficult.
whereas dim means ‘not bright or lacking light’.
4. (c) Drama is basically an art or dramatic work that is
performed by the actors for an audience. So, the given

too. asy
Option (d) is wrong because the words here are antonyms words are interconnected in the way that drama is done
for the audience or in front of them, but they are not a
44. (d) Relation - Synonyms;
Vestment means the dress worn by a priest while solemn
En constituent of drama. Option (c) captures this relationship
well, as drama is nothing but performing art and critics are

45.
means serious, all other pairs are synonyms.
(b) Relation - similar meaning;
gin the ones who watch an art or its performance and give their
views on it. Option (a) is incorrect, because brawl means

46.
Rout means a terrible defeat, grief means sorrow, triumph
means success.
(b) Relation - Person - Workplace; eer
a noisy or loud fight; whereas, vagabonds are those people
who move around a lot and have no permanent homes. It
fails to match the relationship mentioned in the question:

47.
Curator is the person who takes care of the museum.
(b) Relation - person - quality; ing
brawl is not something which is done for the vagabonds or
done in front of them. Option (b) is wrong, because obscure
Valour means bravery and cynic is a person who doubts
constantly the motives and intensions of people around .ne
and vague are synonyms which are used for something that
is unclear or not easily visible or not clearly understandable.

48.
him.
(d) Relation - thing - place where it is kept;
Vault is a kind of locker where cash is placed. Account is not
t
The term ‘vagabonds’ is vague in terms that they are not sure
of where they are going or what is next for them. Option (d)
is incorrect, because actors are actually the constituents of a
movie; they are a part of it and that is not the relationship
really a place in which money is put.
mentioned in the given words in the question.
49. (d) Relation - thing - its characteristic quality; as wool gives
5. (c) The meanings of the words are as follows:
warmth, the spring has elasticity, it is the characteristic
Stygian: dark or gloomy; infernal; hellish
defining quality of spring.
Fortuitous: happening or produced by chance; accidental
50. (c) Relation - arbiter - situation;
‘Stygian’ means ‘dark’; therefore, their relationship is
As an umpire is in a match, the moderator is in a debate, he synonymous.
acts as a judge of whether the rules were followed or not. ‘Abysmal’ means ‘immeasurably deep’, which is different
LEVEL-II from ‘low’.
The only other synonymous pair is ‘fortuitous’ and
1. (d) This is an easy one. Ease and alleviate are synonyms and ‘accidental’.
hence we just have to find a synonym pair in the options. 6. (c) ‘Contiguous’ means ‘in contact or in close proximity
This pair is located in option (d). Question and Interrogate without actually touching’.
are two synonyms. All other options display no such ‘Abut’ means ‘adjacent; touch or join at the edge or border’.
relationship. Thus the words are synonymous.

Downloaded From : www.EasyEngineering.net


Downloaded From : www.EasyEngineering.net

Relationship Based Problems  l 217

‘Possible’ means ‘that may or can be’ and ‘occur’ means ‘to 19. (b) Something that is fake is lacking in genuineness.
take place or to happen’. Similarly, something that is nebulous is hazy or vague,
‘Simultaneous’ means ‘existing or occurring at the same time’ lacking in definition.
and ‘coincide’ means ‘to happen at the same time or during 20. (b) While the physician can diagnose and treat a patient with
the same period’. Therefore, these words are synonyms. drugs, the surgeon treats a patient through an operation.
7. (d) Relation - Property - effect; i.e. ‘something which is The same relationship works in the case of an optometrist
fragile cracks,’ similarly something that is pliable bends. and an ophthalmologist.
Cemetery is burial ground, irreducible is what cannot be 21. (d) The relationship here is one of action. A bull is a person
reduced. who buys on the stock market, while a bear is one who sells
8. (c) Relation - Preceding event - succeeding event; as on the stock market.
Rehearsal precedes performance, engagement precedes
22. (c) The answer is (c). The relationship here is one of
marriage, appetite means desire, it may or may not be
opposites. Although the word pair fundamental : elemental
succeeded by satisfaction and these cannot be counted as
is also opposite, it does not also show the idea of something
events, whereas the given words are events not feelings,
applause means appreciation. that changes fand something that is fixed.

ww
9. (b) Relation effect : Cause;
Just as sorrow is caused by death happiness is caused by
birth.
23. (d) Sound is measured in decribels just as length is measured
in meters.
24. (c)

w.E
10. (d) Relation - surrounded object - surroundings; an island
is a piece of land surrounded by water on all sides as an oasis
is surrounded by desert on all sides.
25. (b) Just as you extract a tooth, you uproot a stump.
26. (b)
27. (d)

asy
11. (a) Relation - emotion - person with opposite emotion; for
all other three options, the feelings in both the words means
28. (d)
29. Kernel means the core of a seed and hence it could refer to

carefully, sage is a wise, holy man.


En
the same, optimist means hopeful, diligence means working the kernel being inside the shell.
Option (a) — caterpillar and pupa are two stages of an
12. (b) The relation between the two words is that of chronology,
prehistory came before medieval times just as present comes
before future. Even though Akbar came before British and
gin insect. One is not inside another hence it does not have the
same relationship.
Shakespeare before Tennyson these are people while the
given pair of words are divisions of time. eer
Option (c) lassitude means laziness and syncope refers to
the loss of one or more letters in the interior of a word.
13. (b) Relation between the two words is of antonyms. Sceptic
is a person with doubt and pious is the person with faith. ing
Option (d) is not about the natural relationship also all
passengers are not related to car.
Atheist is a person who does not believe in god, hence there
is no doubt. Iconoclast is a person who attacks cherished
.ne
Option (b) carries the same relationship almost.
30. (c) Cicerone is an old term for a guide, one who conducts
belief.
14. (a) A conclusion is drawn from a premise, similarly an
inference can be drawn from an assumption. Hypothesis
is also an assumption but a theory a usually drawn from
t
visitors and sightseers to museums, galleries, etc., and
explains matters of archaeological, antiquarian, historic or
artistic interest. Baedeker is any guide book, pamphlet, or
the like, containing information useful to tourist.
proofs.
31. (b) Quisling means ‘a person who helps an enemy that
15. (d) The relation in the given pair of words is that of degree.
has taken control of his or her country’. Treason means
Affection is higher degree of love. Just as misery is a higher
‘the crime of doing something could cause danger to your
degree for sorrow. Joy and happiness are simply synonyms.
country, such as helping its enemies during a war.’ Hence
16. (a) Botany is the study of plants, just as geology is the study
of the earth. In option b and d, the same relationship holds both the given words are synonyms. Similarly homicide and
but in the reverse order. Etiology is the study of disease and murderer show the relation of synonyms.
astronomy is the study of stars. 32. (a) Pollyanna means ‘a person who is always cheerful and
17. (a) This is a degree of intensity analogy. To guzzle is to expect only good things to happen.’ Optimism means ‘a
drink a lot, rapidly. Likewise, to gorge is to eat intensely. person who always expect good things to happen or things
18. (b) Just as loneliness and privacy are synonyms, describing to be successful.’ Hence both given words are synonyms.
the state of being alone or separate from others, composure Similarly diehard and resistance are synonyms.
and equanimity are synonyms describing a calm or peaceful 33. (d) Gargantuan means ‘extremely large’ similarly tiny means
state of mind. ‘very small’. Hence, small : tiny is right option.

Downloaded From : www.EasyEngineering.net


Downloaded From : www.EasyEngineering.net

218  l  Relationship Based Problems

34. (b) Interlude is the period of rest. Similarly, Retirement is 48. (d) Relation - Prey - Predator;
the period of retreat (a period of time when one stops his Plant is attacked by a parasite just as sailors are attacked by
usual activities and goes to a quiet place for prayer and pirates. Transient is an adjective not a noun as the first word
thought). of given pair, it means something that does not last forever.
35. (a) Chauvinism is a kind of sexism, in the same way as 49. (b) Relation - symbol - quality symbolised; as torch suggests
Apartheid is a kind of Racism. liberty, scale is a symbol of justice.
36. (a) Amoral has no morals, in the same way as a secular 50. (c) Relation - milder action - high intensity action; just
person is not spiritual. as whisper is lower in intensity to speak, brush is lower in
37. (b) Something that brings harm is termed as sinister. intensity to touch. Brush means to touch lightly in passing.
Similarly, something that brings worry or fear about 51. (c) Relation - collector - collected things;
Philatelist is the person who collects stamps and numismatist
something that may happen i.e. trepidation is has to be
is one who collects coins. Astrologer is the person who
horrifying.
studies the movements of stars and planets.
38. (c) Relation between the words is of antonym. Limpid
52. (d) Relation - Worker - place of work;
means clear and murky means dark, unclear. Dazed means
Sonnet is a poem with fourteen lines.
stunned or bewildered, clouded means unclear or confused.

ww
Obscure means indirect, indistinct and vague means not
clear, nebulous means indistinct. Gloomy and bright are
53. (c) Relation - collection - collected objects;
Lexicon is a list of words not a collection. Glossary is a
specific, smaller collection, lexicon is a large, universal

w.E
also antonyms.
39. (b) Paradigm is an example or a model for something. So
a pattern can be a paradigm. Method and system have a
collection. An atlas is a collection of some maps not all
possible maps in the world.
54. (a) Relation - Intense action - less intense action; chatter is

asy
similar relation to each other just as there can be method of
a particular process, and a system of doing it. None of the
to talk hurriedly and scurry is to move hurriedly.
55. (b) Relation - Intensity;


other pairs have a similar relation.
40. (a) Malapropism means comical confusion of words,
En Tepid means milky, hot, pat is to slap gently. So slap is more
intense action.
similarly anachronism is related to time, ellipsis is a kind of
punctuation. Catechism is a series of questions.
41. (d) Posterior means the rear and anterior means the front.

gin
56. (c) Relation - betrayer - betrayed person/organisation;
apostate is a person who abandons his religious belief.
57. (b) Barge is a flat-bottom boat which is used for carrying
42. (b) The state/feature given by first words leads to the other.
A mendicant lives by asking people for money and food eer
or transporting heavy loads; whereas, a vessel can mean a
craft or a vehicle designed for water transportation. So, the
because he is impecunious (having little or no money).
Similarly, a bad-tempered and unreasonable (petulant) ing
relation in the word is ‘type of ’ i.e. barge is a type of vessel.
Shovel is an instrument, rim is edge, and training is
preparation. Thus, the only option that does not match the
person, especially because he cannot do or have what he
wants, would habitually be a complainer. .ne
given relationship is option (b), because an anthology may
be a type of a book, but not the other way around.
43. (c) The words are opposites. Impeccable means flawless.
44. (a) Lachrymose means tearful. Similarly Verbose refers to
using or containing more words than are needed.
hydrogen, only a dog doesn’t have a propeller.
59. (d)
t
58. (a) A ship has a sail, a satellite has a radar and a balloon has

60. (a) ‘Asinine’ means ‘foolish; of or like an ass or donkey’.


45. (d) Relation - worker - place of working; a designer does
not work in a design but works on a design. Similarly, ‘vulpine’ means ‘of or resembling a fox’. ‘Avian
46. (d) Relation - Place of nurture - nurtured object; incubator means of or pertaining to birds in general and not
is where the infants are kept to save their lives. Though wine specifically doves.
61. (b) A ‘stallion’ is a male horse while a mare is a female
is kept in coolers to cool it, it is not the place where it is
horse. Therefore, the relationship is Male /Female :
nurtured or saved from any danger, hive is the house of the
In options 1, 3 and 4 this relationship has been inverted.
bee but not a nurturing place, where it is kept specifically In option 2, ram, a male sheep, precedes ewe, a female sheep.
to be protected and nurtured. Greenhouse is where young Thus, the Male: Female relationship is analogous to the
plants are kept to help them grow and prevent their death. given words.
47. (b) Relation - Noun - its quality; 62. (c) Actor and thespian are synonyms. Likewise, harlequin
Inept means unsuitable or unskillful, which will not be and comedian mean ‘anusung, comic characters.
a positive quality for technology as in the given relation. 63. (b) Venom and Toxin are synonyms, meaning a poisonous
Sturdy is a positive quality of physique. Silk is a noun and liquid. Maleficence is synonym of venial, meaning ‘not very
not an adjective. Noble is not an adjective used with intellect. serious and therefore able to be forgiven’.

Downloaded From : www.EasyEngineering.net


Downloaded From : www.EasyEngineering.net

14
PTER
CHA

Fill in the Blanks (FIB)

w
W ww
e have been solving fill in the blank questions since our school days. This is why, at first sight, they look pretty simple. But
beware, they can often get tricky.
FIBs are asked in various exams to test the vocabulary and comprehension skills. SSC, Banking, CDS etc., are just some

quite straightforward to you.


.Ea
of the exams that these are a part of. If your vocabulary is good, and you know how to judiciously employ words, this section will be

But fill in the blanks are sometimes asked in a more complex manner: they test not only your vocabulary, but also your grammar.

syE
We will look at such questions in more depth later on, along with some different types and approaches.

TYPES OF FIBs
(A) Vocabulary Based
ngi
These questions will ask you to fill the blank with appropriate words. One or more incomplete sentences will be given to you and

are frequently asked in many exams.


nee
your job is to complete those sentences from the given options. These are the most common types of fill in the blank questions and

Sometimes there can be more than one blank, and in those cases, you have to pick an option, which will have the words that satisfy
all the blanks.
How To Solve
rin
(i) Read the entire statement(s), i.e. the complete and the incomplete ones.
(ii) Try to understand the context or the subject of the statement.
(iii) Analyse what word would logically fit in the statement. g .ne
(iv) If you are unable to predict what should be there, have a look at all the options given to you, and if it still looks a bit complicated,
then fit in all the options to the statement and see which option seems most appropriate.
t
(v) Select the option which is most apt for the given statement. Make sure that option abides the context of the statement. Ensure
that the selected option has words that fit all the blanks (in case of multiple blanks).
(B) Grammar Based
The grammar based fill in the blank questions have grammatical rules at their base and the student is tested for the application of
these. The grammar based FIB questions differ from the vocabulary based fill in the blank questions in the essence that the latter are
asked more frequently on examinations than the former.
Example
If you stumble ___ a new idea, you better write it.
(a) at (b) upon (c) across (d) on
Explanation (b): The correct answer is option (b) which implies that you ‘stumble upon’ a new idea. It means to discover some-
thing.
Most often in grammar FIBs, either you would know the answer, or you wouldn’t. There wouldn’t be a gray line in the middle of
the two. This is because either you would know that rule or usage, or you wouldn’t. It is just too hard to make a good guess in grammar.

COMMON MISTAKES
(a) Students do not read the entire statement, i.e., they just read till the blank, and start hunting for the right option.
(b) Understanding the context of the statement/passage is crucial. If you are unable to gauge the context, it might be a good thing to
skip the question.

Downloaded From : www.EasyEngineering.net


Downloaded From : www.EasyEngineering.net

220  l  Fill in the Blanks (FIB)

(c) There are always different shades/degrees of a particular word, and all the degrees might look right, but since we are asked to
pick the most appropriate option, you need to grasp the given passage/statement for that and choose the option that fits most
with the author’s tone; e.g., if the author is ecstatic about something, then he might use words like ‘jubilant’ (instead of ‘happy’),
‘incredible’ (instead of ‘surprising’) and so on.
(d) A slightly more complex version of the fill in the blanks is when you are asked to fill two blanks. Now, if you have only understood
one part of the statement, or you are just sure about one of the blanks, then do not base your answer only on that. In an attempt to
somehow solve the question, students find or understand one of the blanks and go hunting for that one word-type in the options
to select the right one. Options will most definitely be framed in a way to fail this kind of approach. There would be more than
one options satisfying one of the blanks, but exactly one option satisfying both the blanks.
(e) Beware of synonyms: If you think you have found out the right word for a particular blank, and have decided to eliminate other
options, then stay on the lookout for synonyms of that word present in the other options. There might be a better combination
of words that you happened to overlook.
(f) You would probably know this, but we have to state it anyway: when the question asks to fill more than one blank, the words in
an option are always arranged sequentially, i.e., the second word in an option is never meant for the first blank. So, never try to
fit in words like that.

TIPS
ww
(1) You can verify the option you have picked by filling it in the blanks, and reading the entire statement/passage again.
(2) If you have to fill more than one blank, then you may eliminate an option based on one incorrect word.

w.E
(3) Remember! Re-reading is always an option. The test is more about understanding than speed. And even if you are taking a speed
based test, the trade-off is minimal.

FROM BEGINNER TO MASTERY


Example 1. asy
En
Astronomy, it has been said, is the oldest and the noblest of the sciences. (_______) it is one of the few sciences for which most
present-day educators seem to find little time.
(a) As (b) Yet

gin
(c) But (d) Thereby
Explanation (b): There is a mood swing between the two sentences, which is best highlighted by a ‘yet’ or a ‘but’. It is better to use a
‘yet’ because it links the two sentences in a better fashion.
Example 2.
eer
Often we (_______) ourselves of the pleasure of making friends with the stars and shut our eyes to the glories of the heavens

ing
above because we do not realise how simple a matter it is to become acquainted with the various groups of stars as they cross our
meridian, one by one, day after day and month after month in the same orderly (_______).
(a) abandon, line
(c) fulfil, episode
(b) deprive, sequence
(d) strip, passage
.ne
Explanation (b): The sentence is talking about the beauty of constellations and how common man has ignored this. Thus, the first

blank talks about the movement of stars in a sequence, which makes the correct word: ‘sequence’.
Example 3.
t
blank should have a word that signifies ‘not getting, not obtaining, staying away’ which is highlighted in ‘deprive’. The second

Let us (_______) then that the time we choose for our observation of the heavens is the last of the month while our charts are
given for the first of the month.
(a) suppose (b) wonder
(c) proclaim (d) believe
Explanation (a): The tone in the sentence is that of ‘opining, or speculating or conjecturing’. The correct word here is then ‘suppose’.
Example 4.
Broadleaf woods are characterised by complex fibre conditions, absence (_______) resins, and greater weights.
(a) because of (b) of
(c) causing (d) by
Explanation (b): With ‘absence’, we use ‘of ’.
Example 5.
Oak trees are (_______) by oblong, thin-shelled kernels, protruding from hard scaly cups and called acorns.
(a) seen (b) found
(c) characterized (d) differentiated

Downloaded From : www.EasyEngineering.net


Downloaded From : www.EasyEngineering.net

Fill in the Blanks (FIB)  l 221

Explanation (c): The part of the sentence that follows the blank identifies characteristics of ‘oak trees’. Thus, the correct word is
‘characterised’.
Example 6.
Since her face was free of (_______) there was no way to (_______) if she appreciated what had happened.
(a) make-up, realise (b) expression, ascertain
(c) emotion, diagnose (d) scars, understand
Explanation (b): The second word can help us ascertain the correct option. Only ‘ascertain’ fits correctly there. All other options, viz.
‘realize’, ‘diagnose’ and ‘understand’ are not apt.
Example 7.
In this context, the (_______) of the British labour movement is particularly (_______).
(a) affair, weird (b) activity, moving
(c) experience, significant (d) atmosphere, gloomy
Explanation (d): This one can be solved by finding out the correct fit for the first blank itself. ‘Atmosphere’ seems an apt fit, whereas
‘affair’ and ‘experience’ leave something to be desired. Option (b) is incorrect because of a weak first word.
Example 8.

ww
The (_________) regions of Spain all have unique cultures, but the (_________) views within each region make the issue of an
acceptable common language of instruction an even more contentious one.
(a) different, competing (b) divergent, distinct

w.E
(c) distinct, disparate (d) different, discrete
Explanation (d): The second word choices are not tough and one can see that the speaker wants to talk about the different views that
are not converging or much less, not even overlapping. The apt word for this would be ‘discrete’.
Example 9.
asy
Early (_________) of maladjustment to college culture is (_________) by the tendency to develop friendship networks outside
college which mask signals of maladjustment.
(a) prevention, helped
(b) identification, complicated En
(c) detection, facilitated
(d) treatment, compounded gin
eer
Explanation (b): Clearly, the sentence is trying to say that some people who have problems adjusting to college culture are difficult to
identify, because they tend to make friends outside the college, thereby hiding the other problems that are associated with living

Example 10.
ing
without friends. This way, it is difficult to isolate them, because the symptoms are masked.

The British retailer, M&S, today formally (_________) defeat in its attempt to (_________) King’s, its US subsidiary, since no
potential purchasers were ready to cough up the necessary cash.
(a) ratified, auction (b) announced, dispose .ne
(c) conceded, offload (d) admitted, acquire
Explanation (c): A ‘subsidiary’ is used to imply a company that is owned by some other company. Option (c) is correct because
conceding means admitting, and offloading means taking the load off, which would mean giving away or selling the subsidiary
and thereby, removing all the controls that M&S has over this subsidiary. Option (a) is wrong because ratified is something
t
which is officially sanctioned or approved and saying that the company ‘formally officially approved defeat’ is redundant too.
An auction is a public selling of something to the highest bidder. This can be the right word for this blank, but ratified is
inappropriate for the first blank. Option (b) is wrong, because ‘dispose’ means ‘to get rid of ’. Option (c) is wrong, because acquire
means capture or gain, but the company is selling or giving away its subsidiary.
Example 11.
This simplified (_________) to the decision-making process is a must read for anyone (_________) important real estate,
personal, or professional decisions.
(a) primer, maximizing (b) tract, enacting
(c) introduction, under (d) guide, facing
Explanation (b): The statement refers to something which simplified the process of making a decision, and this is important for
someone who is dealing with real estate, personal or professional decisions.
Option (b) is correct, because tract means a brief treatise on a subject of interest; the word enacting would be right, because it
means making something happen, which in this case would mean making decisions.
Option (a) is incorrect, because primer just means an introductory book. It cannot reveal an entire simplified approach to make
a decision. Also, ‘maximizing decisions’ does not make sense.

Downloaded From : www.EasyEngineering.net


Downloaded From : www.EasyEngineering.net

222  l  Fill in the Blanks (FIB)

Option (c) is wrong because just an introduction cannot explain the entire decision making progress. Also, ‘under’ is wrong for
the second blank, because ‘under decisions’ is inappropriate.
Option (d) is wrong, because ‘facing’ refers to encountering or dealing, and one does not face decisions.
Example 12.
Physicians may soon have (_________) to help paralysed people move their limbs by bypassing the (_________) nerves that
once controlled their muscles.
(a) instruments, detrimental
(b) ways, damaged
(c) reason, involuntary
(d) impediments, complex
Explanation (b): Option (b) is correct because the sentence means that physicians have found methods to help treat paralyzed
people. The right word for second blank is ‘damaged’, because paralyzed people are those who have lost the ability to move a
body part and hence the respective nerves of that part are in a damaged state. Option (a) is wrong because ‘detrimental’ refers to
something which has caused an injury. This cannot refer to muscles.
Option (c) is wrong, because using ‘reason’ in the sentence would make it illogical, because they are physicians and of course
they already have a reason to treat their patients, and anyway the sentence goes on to mention a way of treating them, not a reason.

ww
‘Involuntary’ is used to describe something which is done without consciousness, control or will.
Option (d) is incorrect, because an ‘impediment’ is used to describe something which slows or blocks progress; and the word
‘complex’ is inappropriate, because complex would mean that nerves are complicated in structure.
Example 13.
w.E
The Internet is a medium where users have nearly (_________) choices and (_________) constraints about where to go and what
to do.
(a) unbalanced, nonexistent
(b) embarrassing, no asy
(c) unlimited, minimal
(d) choking, shocking
En
constraints. Evidently, option (c) is the right answer.
Example 14.
gin
Explanation (c): We know that the Internet is a very useful medium in many aspects. It gives us a lot of choices with very few

eer
The best punctuation is that of which the reader is least conscious; for when punctuation, or lack of it, (_________) itself, it is
usually because it (_________).
(a) obtrudes, offends
(c) conceals, recedes
(b) enjoins, fails
(d) effaces, counts ing
.ne
Explanation (a): The speaker states that the best punctuation is one that the reader is not conscious about, or the one that goes
unnoticed. The next clause refers to something which is contrary to what has been mentioned. Option (a) is correct, because
obtrude means ‘to force, or impose on someone’, and offends refers to ‘causing resentment, or anything which is against the \

it offends.”
Option (b) is wrong, because ‘to enjoin’ is ‘to give an order’.
t
rules or laws’. So, this clause would then mean that: “when the punctuation imposes itself on the reader, it is generally because

Option (c) is wrong, because conceal means ‘to disguise, hold back or hide’. ‘Recede’ means ‘retreating or pulling back’. These
words would not help make sense of the sentence.
Option (d) is wrong, because ‘effaces’ means ‘removing or erasing by rubbing.’
Example 15.
The Athenians on the whole were peaceful and prosperous; they had (_________) to sit at home and think about the universe
and dispute with Socrates, or to travel abroad and (_________) the world.
(a) leisure, explore (b) time, ignore
(b) ability, suffer (d) temerity, understand
Explanation (a): Athenians refers to the ‘residents or citizens of the Greek city, Athens’. The speaker says that they were peaceful
(calm and tranquil) and prosperous (flourishing financially, or in materialistic terms). For the first blank, we have to pick what
attribute they had which enabled them to sit at home and think about the universe, dispute with Socrates and travel abroad.
‘Time’ (option (b)) and leisure (option (a)) would both be appropriate. The Athenians could do all these because they had
enough free time for it, but leisure is the more appropriate word here. ‘Ability’ refers to the quality to be able to do something, but
it can be argued that almost everyone has the ability to do such work. ‘Temerity’ is wrong, because it means audacity, the speaker
is just talking about spending time thinking, arguing or travelling. He is not talking about going to a war.

Downloaded From : www.EasyEngineering.net


Downloaded From : www.EasyEngineering.net

Fill in the Blanks (FIB)  l 223

For the second blank now: when they would be travelling abroad, they would be ‘exploring’ it. ‘Ignore’ seems logically incorrect
and the same can be said about the word ‘suffer’. ‘Understand’ can also be correct, but then ‘temerity’ as explained earlier is the wrong
word. Hence, only option (a) is right.
Example 16.
Their achievement in the field of literature is described as (_________), sometimes it is even called (_________).
(a) magnificent, irresponsible
(b) insignificant, influential
(c) significant, paltry
(d) unimportant, trivial
Explanation (d): The given statement is informing how the achievement of a group of people to literature is described. The second
clause refers to something which would be of a higher degree or intensity to what has been mentioned, so for that to be true,
both words would be similar, differing only in degree, with the second one being of a higher degree than the other. Only option
(d) has such words.
Example 17.
From the time she had put her hair up, every man she had met had grovelled before her and she had acquired a mental attitude
toward the other sex which was a blend of (_________) and (_________).

ww
(a) admiration, tolerance
(b) indifference, contempt
(c) impertinence, temperance

w.E
(d) arrogance, fidelity
Explanation (b): The speaker is describing a woman in front of whom every man grovelled (showed submission). We can say that
she captivated men and they admired her. She would definitely have the opinion that men easily fall for women and that they
are superficial.

asy
Option (b) is correct, because indifference means not showing interest. Seeing that she does not have to do anything to impress
men, the woman might have developed an indifferent attitude towards them. The other word is contempt (feeling of disrespect and

En
dislike) which could arise from thinking that men are superficial.
Option (a) is wrong, because admiration refers to liking something, and tolerance refers to accepting something or someone

gin
even if you do not like or respect it. This word could fit in, but ‘admiration’ is not the right word for the first blank.
Option (c) is wrong, because ‘impertinence’ means insolence or cheekiness and temperance means restraint and moderation.
Option (d) is wrong, because arrogance refers to an overbearing pride, which could be right, but the other word is ‘fidelity’ which
is the quality of being faithful and does not fit in the blank.
Example 18. eer
ing
Every human being, after the first few days of his life, is a product of two factors: on the one hand, there is his (_________)
endowment; and on the other hand, there is the effect of environment, including (_________).
(a) constitutional; weather
(b) congenital; education
(c) personal; climate .ne
(d) economic; learning
(e) genetic; pedagogy t
Explanation  (b): The given statement is about what affects human beings in the primary years of their lives. ‘Endowment’ refers to
‘natural abilities or qualities’, so the word congenital and genetic can fit in. ‘Congenital’ refers to what is present at the time of
birth, but not necessarily inherited, and genetic refers to something that is related to genes or produced by a gene. ‘constitutional’
refers to something that is related to or is of a constitution; ‘personal’ means something that one owns and ‘economic’ is what is
related to the economy.
For the second blank, the phrase ‘on the other hand’ means ‘something that will be mentioned ahead will be contrasting to what
has been mentioned already’. Education is the right word, because it is what affects an individual. Weather does not make any sense,
and similarly climate can be ruled out. ‘Learning’ can be ruled out too as learning is done by the self, but education is what is taught
by others, which has a major effect in shaping an individual. Pedagogy means the principles or methods of instruction, so this may
also be a factor, but education is a broader and more appropriate word. Hence, option (b) is correct.

Downloaded From : www.EasyEngineering.net


Downloaded From : www.EasyEngineering.net

224  l  Fill in the Blanks (FIB)

Practice Exercise
LEVEL-I
DIRECTIONS (Qs. 1 -100) :  Each of the following sentences 13. The prisoner showed no _________ for his crimes.
is followed by four words or group of words. Fill in the blanks (a) hatred (b) obstinacy
with the appropriate word or group of words. (c) remorse (d) anger
1. One dark night a Darvesh ___________ passing by a dry 14. It is inconceivable that in many schools children are
well. subjected to physical _________ in the name of discipline.
(a) wasn’t (b) happened to be (a) violation (b) exercise
(c) discovered in (d) found to (c) violence (d) security
2. Nordisk have recently ___________ a product called 15. We have not yet fully realised the _________ consequences
Glucometer. of the war.

ww
(a) started
(c) launched
(b) commissioned
(d) begun
3. I had already published a novel and it was an unexpected
(a) happy
(c) grim
(b) pleasing
(d) exciting
16. The Romans were _________ science.

w.E
success. I thought my ___________ .
(a) days were up
(c) lady luck was happy
(b) chances were good
(d) fortune was made
(a) bad in
(c) bad for
(b) bad to
(d) bad at
17. To ___time, please go___ foot and not by bus.

to ___________ the flames.


(a) cover
asy
4. The neighbour grabbed the boy, and rolled him on the road

(b) kill
(a) spend, with
(c) utilise, on
(b) kill, towards
(d) gain, on
(c) burn out (d) fizz out
5. Sam asked me to keep this secret ___________ . En 18. After a recent mild paralytic attack his movements are_____
restricted; otherwise he is still very active.
(a) secret
(c) amongst us
(b) in myself
(d) between us gin (a) entirely
(c) nowhere
(b) slightly
(d) frequently
19. Despite his _____ he had to suffer.
6. Sometimes the greatest inventions ___________ an idea of
startling simplicity.
(a) stumbles upon (b) hinge upon eer
(a) punishment
(c) negligence
(b) fault
(d) innocence
(c) starves without (d) lacks
7. Real friends, genuinely wanting the best for the organisation, ing
20. Ravi was always_______ of the good fortune of others.
(a) miserable (b) irritable
___________ different garbs.
(a) come in
(c) dressed in
(b) clad in
(d) clothed in

(c) greedy
.ne
(d) envious
21. Cellular phone service has______ in a new phase of

8. There was a major accident. The plane crashed. The pilot


_________ did not see the tower.
(a) likely (b) probably
communication.
(a) called
(c) resulted
(b) ushered
(d) started t
22. The present____has not prevented the railway minister
(c) scarcely (d) hurriedly from announcing an increasing number of superfast trains.
9. The car we were travelling in _________ a mile from home. (a) crisis (b) syndrome
(a) broke off (b) broke down
(c) generation (d) stability
(c) broke into (d) broke up
23. She was a devoted wife and looked____ her husband very
10. What are you _________ in the kitchen cupboard?
well.
(a) looking in (b) looking on
(a) after (b) at
(c) looking to (d) looking for
11. I did not see the point of _________ waiting for them, so I (c) for (d) upon
went home. 24. It is a penal_____to bribe a public servant.
(a) hanging around (b) hanging on (a) charge (b) offence
(c) hang together (d) hanging up (c) code (d) tight
12. He lost confidence and _________ of the deal at the last 25. The teacher_____the boy’s comic which he was reading
minute. during the Mathematics period.
(a) backed out (b) backed on (a) possessed (b) procured
(c) backed down (d) backed onto (c) abandoned (d) seized

Downloaded From : www.EasyEngineering.net


Downloaded From : www.EasyEngineering.net

Fill in the Blanks (FIB)  l 225

26. This is a_____ translation of the speech. 41. The Chairman will come here at 5 p.m. to_____ a lecture.
(a) literary (b) literal (a) attain (b) speak
(c) verbal (d) verbatim (c) talk (d) deliver
27. She put____ a brief appearance at the end of the party. 42. Gokhale’s patriotic speeches____ people to dedicate their
(a) on (b) in lives to the nation.
(c) across (d) up (a) forced (b) inspired
28. Once he has signed the agreement, he won’t be able to (c) instigated (d) prompted
back______. 43. Undoubtedly, English is the most___spoken language in the
(a) up (b) in world today.
(a) at (d) out (a) elaborately (b) greatly
29. The master dispensed_______the services of his servant.
(c) widely (d) broadly
(a) up (b) with
44. He has not attained the age of 18. He was, therefore, no____
(c) from (d) through
to vote in this election.
30. That charming girl was the ____ of all eyes.
(a) power (b) claim
(a) target (b) cynosure
(c) right (d) authority
(c) doggerel (d) ambition

ww
31. In spite of her other_____, Kasthuri still managed to find
time for her hobbies.
(a) occupations (b) preoccupations
45. The Defence Minister said today that the Government
was determined to____the accord and tulfil the legitimate
aspirations of the people.

w.E
(c) predilections (d) business
32. The journey may be made by sea or___by road.
(a) alternately (b) alteringly
(a) implement
(c) practise
(b) invest
(d) perform
46. The unprecedented rise in the price of gold in India, contrary
(c) conversely
asy
(d) alternatively
33. AIDS is not a disease that can be___through the air or by
to the___elsewhere in the world, is baffling the experts.
(a) outlook (b) trend
insects.
(a) circulated (b) transferred
En
(c) cost (d) pattern
47. A controversial issue was_____ by a member of the
(c) transmitted (d) conducted
34. The official____ the Chief Minister of the situation in the
town. gin Opposition in the Assembly, but there was no time to
discuss it.
(a) risen (b) raised
(a) apprised
(c) appraised
(b) informed
(d) asked
(c) arose
eer (d) aroused
48. Infant mortality rate in China has____ from 200 per
35. His residence is on the______ of Hyderabad.
(a) rim
(c) periphery
(b) core
(d) outskirts
(a) retarded ing
thousand to 14 per thousand.
(b) declined

36. ____ at the major ports has led planners to develop satellite
ports near them.
(c) contracted

.ne
(d) minimised
49. A five-year-old boy was_____ from his school on Monday
last by his servant for a ransom of Rs 8, 000.
(a) density
(c) transportation
(b) containment
(d) congestion
37. Success comes to those who are vigilant not to permit______
from the chosen path.
(a) driven
(c) escorted
(b) arrested
(d) kidnapped t
50. The car driver was arrested for rash driving and his licence
was_____ by the police.
(a) distraction (b) deviation
(a) impounded (b) prescribed
(c) alienation (d) diversion
38. The members were_____ of the date of the meeting well in (c) suspended (d) penalised
advance. 51. When people around you are losing their heads, it is very
(a) communicated (b) conveyed difficult to remain serene. It needs a lot of____.
(c) ignorant (d) inform (a) patience (b) strength
39. Their ____ to scale the mountain peak was an absolute (c) courage (d) goodness
failure. 52. He has already made up his mind on this issue. Now it
(a) attempt (b) desire is____ to argue with him.
(c) anxiety (d) proposal (a) sympathetic (b) vague
40. _____ between labour and management is inevitable in any (c) futile (d) contradictory
industrial society. 53. It is not fair to cast___on honest and innocent persons.
(a) co-ordination (b) competition (a) aspirations (b) aspersions
(c) friction (d) association (c) inspiration (d) adulation

Downloaded From : www.EasyEngineering.net


Downloaded From : www.EasyEngineering.net

226  l  Fill in the Blanks (FIB)

54. The audience____ at the end of drama. 68. Though fond of many acquaintances, I desire_____ only
(a) applauded (b) appraised with a few.
(c) exploded (d) appreciated (a) introduction (b) cordiality
55. The last____ were performed before the body was cremated. (c) intimacy (d) encounter
(a) rites (b) writes 69. The Director pointed out in favour of the manager that the
(c) rights (d) withers profitability of the plant had _____ since he had taken over.
56. Although I was _________ of his plans, I encouraged (a) arisen (b) increased
him, because there was no one else who was willing to (c) developed (d) declined
help. 70. He became the Governor of a Province_____.
(a) sceptical (b) remorseful (a) by and large
(c) fearful (d) excited (b) in the course of time
57. You have no business to _________ pain on a weak and (c) at times
poor person. (d) little by little
(a) inflict (b) put 71. His answer was such______ I expected him to give.
(c) direct (d) force (a) that (b) which

ww
58. Her uncle died in a car accident. He was quite rich. She
suddenly _________ all her uncle’s money.
(a) succeeded (b) caught
(c) as (d) like which
72. Opinions concerning him had remained nearly ____, his
daily habits had presented scarcely any visible change.

w.E
(c) gave (d) inherited
59. Your present statement does not _________ what you said
last week.
(a) static
(c) limpid
(b) biased
(d) stationary
73. The speaker did not properly use the time as he went
(a) accord to
(c) accord with asy
(b) accord in
(d) accord for
on_______ on one point alone.
(a) devoting (b) deliberating

to Scotland.
(a) notion (b) expression En
60. I had a vague _________ that the lady originally belonged (c) diluting (d) distributing
74. After a short holiday Rajni came back totally_______.
(a) rejuvenated (b) reborn
(c) imagination (d) theory
61. Happiness consists in being _________what we have? gin (c) refurbished (d) revamped
75. I had not expected to meet him; it was quite an ______
(a) contented to
(c) contented for
(b) contented with
(d) contented in
meeting.
eer
(a) organised (b) intentional
62. His rude behaviour is a _________ his organisation.
(a) disgrace for
(c) disgrace upon
(b) disgrace on
(d) disgrace to
ing
(c) undesirable (d) accidental
76. The window of our room ____ he rear.
(a) overlooks (b) opens
63. No child is _________ understanding. One has to wait and
provide proper guidance.
(c) opposes
.ne
(d) adjoins
77. The final electoral rolls have been intensively revised
(a) dull to
(c) dull of
(b) dull in
(d) dull for
64. I am fully _________ the problems facing the industry.
(a) alive with (b) alive to
through house-to-house_____.
(a) documentation
(c) enumeration
t
(b) categorisation
(d) investigation
78. Mountaineering institutes___to the young climbers the
(c) alive for (d) alive on technical knowledge which has been accumulated over the
65. Among the factors_____ to the low productivity in years.
agriculture, the first place is occupied by irregular water (a) impart (b) indicate
supply. (c) apply (d) help
(a) producing (b) showing 79. In his address to the teachers, the Vice-Chancellor____
(c) resulting (d) contributing certain measures being taken for improving the quality of
66. The two brothers look so_____ that it is difficult to tell one college education.
from the other. (a) declined (b) directed
(a) identical (b) same (c) advised (d) highlighted
(c) similar (d) resembling 80. The President today______ the committee with the
67. His most striking _____ is the enthusiasm which he brings induction of the five new general secretaries in place of
to everything he does. those dropped.
(a) factor (b) attitude (a) reconstituted (b) reviewed
(c) characteristic (d) character (c) formed (d) enlarged

Downloaded From : www.EasyEngineering.net


Downloaded From : www.EasyEngineering.net

Fill in the Blanks (FIB)  l 227

81. Traffic problems in Bombay are as serious as in any other 94. The influence of the environment on man is revealed by
city in India; and they are complicated by digging of roads an_____ study.
by the corporation on this or that ____. (a) anthropological (b) ecological
(a) aspect (b) pretext (c) epigraphic (d) numismatic
(c) intention (d) instance 95. She had a terrible night caused by a(n)_____ during her
82. Nowadays there exists a spirit of___ among the various sleep.
departments of the University. This has led to a number of (a) incubus (b) debility
interdisciplinary research publications due to interaction of (c) obsession (d) delusion
various research groups. 96. Alexander Solzhepitsyn’s works will be_______ by every
(a) co-operation (b) education lover of liberal thought and they will bring home to him
(c) casteism (d) favouritism how restrictive freedom is in the Russian system.
83. The stock market is very_____ at the moment. (a) rejected (b) skimmed
(a) sensible (b) sensitive (c) compiled (d) perused
(c) intensive (d) remunerative
97. A great literary or artistic work is known as______.
84. Man is still a____in the labour market.
(a) par excellence (b) bete noire

ww
(a) guilt
(c) endemic
(b) possibility
(d) commodity
85. Survival of mankind itself is in danger due to__________of
(c) peccadillo (d) magnum opus
98. ____ at the major ports has led planners to develop satellite

w.E
atomic weapons.
(a) perpetuation
(c) provocation
(b) regularisation
(d) proliferation
ports near them.
(a) density
(c) transportation
(b) containment
(d) congestion

asy
86. His remarks were filled with _____ , which sounded lofty
but presented nothing new to the audience.
99. The carriage foundered in a snowdrift and it took two hours
to_____ it.
(a) exert (b) pillage
(a) aphorisms
(c) bombast
(b) platitudes
(d) adages
En (c) exacerbate (d) extricate
100. Since the British were masters of the seas, no ____ power
87. When indecision grips a nation, free men feel the need
for an _____ruler and are prepared to throw democracy
overboard. gin could venture into Indian waters under British rule.
(a) territorial (b) continental
(a) optimistic
(c) eccentric
(b) autocratic
(d) energetic eer
(c) maritime (d) geo-political
DIRECTIONS (Qs. 101-105) : Each sentence below has two
88. India has the_______ of high saving and low growth rates.
(a) irony (b) similarity ing
blanks, each blank indicating that something has been omitted.
Choose the set of words for each blank which best fits the meaning
of the sentence as a whole. [SBI Clerk 2012]
(c) difference (d) paradox
89. Few countries can____India in variety, colour and richness
of dance-forms. .ne
101. The students _________ not reach on time _________ of
the transport strike.

(a) rival
(c) parallel
(b) depict
(d) fight
90. The entire village condoled ____ the jawan’s widow in her
(a) can, as
(c) may, account
(e) should, for
(b) could, because
(d) will, despite
t
102. They were _________ to vacate that house as _________
bereavement. as possible.
(a) in (b) for (a) asked, soon (b) ordered, easy
(c) to (d) with (c) shown, early (d) told, later
91. The battalion operating from the mountain was able to (e) found, fast
tie____ three enemy divisions. 103. The teacher _________ the concept by _________ practical
(a) up (b) down examples.
(c) on (d) with (a) showed, telling (b) gave, speaking
92. His attitude to his boss was so_____ that it caused a good (c) liked, citing (d) found, looking
deal of repulsion. (e) explained - quoting
(a) refulgent (b) arrogant 104. The manager told us _________ Ramesh was very anxious
(c) sycophantic (d) hybrid _________ the meeting.
93. His fears were explicitly betrayed by his______ voice. (a) about, in (b) that, before
(a) phonetic (b) tremulous (c) like, during (d) the, for
(c) tenuous (d) malodorous (e) said, after

Downloaded From : www.EasyEngineering.net


Downloaded From : www.EasyEngineering.net

228  l  Fill in the Blanks (FIB)

105. She did not tell _________ that she _________ attended 114. It is difficult to speak a language fluently unless ______
the party. regularly.
(a) someone, have (b) no one, has (a) it is in practice (b) it is by practising
(c) him, not (d) her, can (c) it will be practised (d) it is practised
(e) anyone, had (e) it had been practised.
115. By the time he was eighteen years old, Peter _______ to
DIRECTIONS (Qs. 106-110): Each sentence below has a blank, make a living and support his family.
each blank indicates that something has been omitted. Choose (a) begins (b) began
the word that best fits the meaning of the sentence as a whole. (c) started beginning (d) would begin
[SBI Clerk 2012] (e) has just begun
106. Rajeev was upset because he _____________ forgotten his
best friend’s birthday? DIRECTIONS (Qs. 116-120) : Sentences are given with blanks
(a) have (b) shall to be filled in with an appropriate word (s). Four alternatives are
(c) will (d) might suggested for each question. Choose the correct alternative out
(e) had of the four. [SSC Sub Insp. 2012]
107. Raj was _____________ introvert and would prefer to sit in

ww
the library rather than go out with friends.
(a) the
(c) like
(b) a
(d) for
116. He will dispense ______ your services.
(a) of
(c) on
(b) off
(d) with
(e) an
w.E
108. Shanku was born_____ a silver spoon in his mouth and was
very proud of his wealth.
117. I have given her ______ .
(a) a work (b) some works
(a) with
(c) on
(e) wishing
asy
(b) along
(d) within
(c) a piece of work (d) many works
118. He is a popular teacher. He seems to be ______ for that
profession.

En
109. Laxmi lost an important file and rather than confessing
her___ she blamed Sandra for losing it.
(a) cut down
(c) cut in
(b) cut off
(d) cut out
(a) respect
(c) attitude
(b) image
(d) default gin
119. He was ______ pertaining his innocence.
(a) stridently (b) harshly
(e) mistake
110. Jacob was a rich old man who lived_____ alone in a huge
house because his children did not care about him. eer
(c) confidently (d) quickly
120. If you were found guilty of exceeding the speed limit, you
(a) only
(c) more
(b) all
(d) too ing
______ to pay a fine.
(a) would have (b) would had
(e) little

DIRECTIONS (111-115): Pick out the most effective word/


(c) will have

.ne
(d) have

DIRECTIONS (Qs. 121 - 125) : Sentences are given with blanks


phrases from the given alternatives marked (a), (b), (c), (d) and
(e) given below each sentence to fill in the blank to make the
sentence meaningful and grammatically correct.
[IBPS Clerk 2012]
of the four:
121. Let us quickly __________.
t
to be filled in with an appropriate word(s). Four alternatives are
suggested for each question. Choose the correct alternative out
[SSC CHSL, 2012]

111. By the middle of the 19th Century, the urban population of (a) muddle (b) huddle
England _________ the rural population. (c) hurdle (d) puddle
(a) have exceed (b) exceeded 122. Rajesh’s car wasn’t __________ Ramesh’s, so we were too
(c) was to exceeds (d) exceeds exhausted by the time we reached home.
(e) will exceed
(a) such comfortable (b) as comfortable as
112. The house that the actress lives in is beautiful, but the
(c) comfortable enough (d) so comfortable that
surroundings are ________ unpleasant.
123. I don’t suppose that Pramod will be elected __________
(a) quite (b) more and more
how hard he struggles as he is not completely supported by
(c) a little quite (d) very little
the committee.
(e) quite few
(a) although (b) seeing as
113. My friend and I decided to watch a play, however ______
(c) no matter (d) however
enjoyed it.
124. Regular exercise is conducive__________ heath.
(a) hardly of us (b) some few of us
(a) in (b) to
(c) we scarcely only (d) neither of us
(c) for (d) of
(e) no one of us

Downloaded From : www.EasyEngineering.net


Downloaded From : www.EasyEngineering.net

Fill in the Blanks (FIB)  l 229

125. Can you please __________ my web site just before I 138. Take this medicine regularly and you will get rid __________
publish it ? this disease.
(a) find out (b) go through (a) at (b) from
(c) set out (d) look up (c) of (d) over
139. Statistics __________ always my worst subject.
DIRECTIONS (126-130) : In the following questions, sentences
(a) are (b) were
are given with blanks to be filled with an appropriate word(s).
Four alternatives are suggested for each question. Choose the (c) is (d) have
correct alternative out of the four as your answer. 140. When she retired, she handed __________ the charge to the
Vice-President.
[SSC Sub Insp. 2013]
(a) over (b) out
126. There are not solitary, free-living creatures ; every form of (c) across (d) off
life is ______ other forms. DIRECTIONS (141-145): In these questions, two sentences (I)
(a) dependent on (b) parallel to
and (II) are given. Each sentence has a blank in it. Against each
(c) overshadowed by (d) segregated from
127. I’ll take ______now as I have another’s appointment some five options are suggested. Out of these, only one fits at both the
where else. places in the context of each sentence. Mark that option as your

ww
(a) departure
(c) permission
(b) my leave
(d) leave from work
128. A garden knife is ______used for right pruning.
answer. [IBPS Clerk 2013]
141. I. The report ended on a .......... note.
II. They must take .......... steps to deal with the problem.


w.E
(a) generally
(c) systematically
(b) compulsorily
(d) daily
129. Serious threat to our ecology and environment can be ______
(a) criticising
(c) positive
(e) cursory
(b) huge
(d) fancy

with organic cultivation.


(a) hastened
(c) aggravated asy
(b) impeded
(d) combated
142. I. In today’s world parents usually .......... that their
children do not talk to them as they are more involved
in electronic gadgets and social media.

with and love of India.


En
130. ‘My India’ by Corbett deals ______ the author’s familiarity II. He would .......... about every other child on the
playground and his parents would fight with other
(a) in
(c) at
(b) of
(d) with
gin parents so as to protect him.
(a) feel (b) defend
DIRECTIONS (Qs. 131 - 140) : In the following questions,
sentences are given with blanks to be filled with an appropriate
word(s). Four alternatives are suggested for each question.
(e) observe
eer
(c) fight (d) complain

143. I. He was good with Mathematics so he could not fathorn


Choose the correct alternative out of the four.
[SSC Multi tasking 2013] ing
why other people cribbed about such an .......... subject.
II. In a world so riddled with greed and corruption where
131. The new government took __________ last year.
(a) out (b) after .......... to trust others.
(a) good .ne
every man seems to be out to get another, it is not very

(b) exciting
(c) over

(a) incentive
(c) interactive
(d) upon
132. Mohan’s career has taken some __________ twists and turns.
(b) interesting
(d) intuitive
(c) wise
(e) interesting
(d) easy
t
144. I. The area under our eyes in connected to our kidneys,
so any .......... change in the dark circles indicates
133. The bus __________ fifty passengers fell __________ the
river. dehydration or accumulating toxins.
(a) with; into (b) for; upon II. Many Indian adults are struggling with weight, thus
(c) over; on (d) of; at indulging in weight loss programmes that require
134. It is raining __________ . Do not go out. .......... changes in their current lifestyle.
(a) heavily (b) fast (a) drastic (b) frequent
(c) soundly (d) strongly (c) ambiguous (d) severe
135. She tries to adjust __________ her relations. (e) aggressive
(a) for (b) at 145. I. Usually fund-raising events and charity auctions raise
(c) so (d) with a large amount of money as people from all sections
136. She was remarkably __________ in singing and dancing. of the society get an opportunity to .......... for a cause.
(a) accomplished (b) conducive II. Today, a person needs to .......... very hard for surviving
(c) fluctuating (d) cooperative in the corporate race.
137. Sheila gained an advantage __________ me. (a) work (b) contribute
(a) upon (b) from (c) effort (d) donate
(c) on (d) over (e) dedication

Downloaded From : www.EasyEngineering.net


Downloaded From : www.EasyEngineering.net

230  l  Fill in the Blanks (FIB)

DIRECTIONS (146-150) : In following questions, sentences 152. A child is the future of a family _________ nation.
are given with blanks to be filled in with an appropriate word (a) just as (b) as a
(s). Four alternatives are suggested for each question. Choose (c) like a (d) as well as of a
the correct alternatives. [SSC Sub Insp. 2014] 153. If strict security measures were taken, the tragedy might
have been ________
146. China is a big country, in area it is bigger than any other (a) restrained (b) averted
country __________ Russia. (c) removed (d) controlled
(a) accept (b) except 154. The deceased left _______ children.
(c) expect (d) access (a) behind (b) for
147. The masks worn by the men helped them _______their (c) with (d) by
identity. 155. The statue _________ a global symbol of freedom
(a) conceal (b) congeal (a) stands against (b) stands to
(c) masquerade (d) cheat (c) stands for (d) stands as
148. On the ______ occasion of Laxmi Puja the Mathurs bought DIRECTIONS (Qs. 156-160) : In some parts of the sentences
a new car. given with blanks to be filled in with an appropriate word(s). Four
(a) officious (b) auspicious
alternatives are suggested for each question. Choose the correct

ww
(c) fortuitous (d) prosperous
149. Precautions are to be taken with any one who seems ______.
(a) contagious (b) infectious
alternative out of the four.
156. Is not learning superior ____ wealth ?
[SSC CHSL, 2014]

w.E
(c) diseased (d) defiled
150. The treasure was hidden ______ a big shore.
(a) on (b) underneath
(a) than
(c) by
(b) from
(d) to
157. A group of agitators ____ the mob to break down the Vice-
(c) toward
asy
(d) off

DIRECTIONS (Qs. 151-155) : In the following questions,


Chancellor’s door.
(a) wished
(c) threatened
(b) excited
(d) incited

word(s). Four alternatives are suggested for each question.En


sentences are given with blanks to be filled in with an appropriate 158. Turn the lights ______ before you go to bed.
(a) on (b) off
Choose the correct alternative out of the four and indicate it by
blackening the appropriate oval in the Answer Sheet. gin (c) out (d) down
159. There is no ______ evidence to support your assertion.
[SSC CHSL, 2013]
151. The human mind seems to have built-in ________against
original thought. eer
(a) facile
(c) facetious
(b) fictitious
(d) factual

(a) prejudices
(c) interests
(b) ideas
(d) safeguards
(a) at
(c) on ing
160. Throw a stone ______ the fierce dog.
(b) upon
(d) above

.ne
t

Downloaded From : www.EasyEngineering.net


Downloaded From : www.EasyEngineering.net

Fill in the Blanks (FIB)  l 231

LEVEL- II
DIRECTIONS (Qs. 1-80): Select the most appropriate set of 8. Scientists, working to save the earth, have ___________ dry
words from the given choices to fill in the blanks. water that soaks carbon three times better than water, and
1. The organization takes its cue from the person on the top. I hence helps ___________ global warming.
always told our business leaders their personal ___________ (a) aided, cut (b) created, combat
determined their organization’s ___________. (c) built, stop (d) produced, increase
(a) serendipity, faux pas 9. The first round of the contest had the students ___________
(b) predilection, despair themselves and ___________ about their hobbies.
(c) intensity, success (a) introducing, talking
(d) oddity, conformity (b) sensitizing, sketching
(c) showcasing, planning
2. The Himalayas ran from east to west and cut off the cold
(d) acclimatizing, mentioning
winds from the north. This allowed agriculture to prosper
10. The varsity’s poll process for ___________ of new candidates
and _____________ wealth, but it also ____________

ww
barbarian invaders from the north.
(a) attracted, dissipated (b) created, attracted
has ___________ poor response with only ten thousand
applications being received till date.
(a) entrusting, seen (b) registration, evoked

w.E
(c) created, restricted (d) attracted, evicted
3. Our _______________ diversity may also be of some value.
Because we have always learned to live with pluralism, it is
(c) entrance, made (d) admission, made
11. It is time to ________ ongoing programmes and ______
new horizons.

the diversity of global economy.


(a) stupefying, negotiate asy
possible that we may be better prepared to _____________ (a) value, choose
(c) reject, consider
(b) speculate, experiment
(d) assess, seek
12. This approach would ________ the enormous illiteracy
(b) plural, alleviate
(c) variegated, annihilate En problem to be ________ in a holistic manner.
(a) enable, tackled (b) focus, viewed
(d) dreary, exasperate
4. My inward petition was instantly _______________. First, a gin (c) envision, dealt (d) combine, judged
13. The _______ of criminalisation of politics needs to be
delightful cold wave descended over my back and under my
feet, ________________ all discomfort.
eer
_________ far more seriously.
(a) lacuna, dealt (b) issue, addressed
(a) acknowledged, banishing
(b) repudiated, infuriating
(c) acceded, exacerbating ing
(c) system, broken (d) continuation, suppressed
14. He ___________ a wrong act because it was ___________
for him to do so due to circumstantial forces.
(d) decimated, assuaging
5. Nature is ______ and unchangeable , and it is ______ as to
(a) compelled, necessary
(b) refused, dangerous .ne
whether its hidden reasons and ______ are ______ to man
or not.
The option that best fill the blanks in the above sentence
would be:
(c) did, avoidable
(d) committed, inevitable t
15. Many teachers ___________ the lack of ___________ for
leaving the job.
(a) relentless, indifferent, actions, understandable (a) cited, reason (b) explained, force
(b) persistent, heartless, actions, comprehensible (c) claimed, understanding (d) argued, culprit
(c) inexorable, apathetic, activities, explicable 16. Skeptics would not ___________ that the earth actually
(d) ineseapable, unconcerned, activities, intelligible moves, let alone that it ___________ around the sun.
6. Nabeesa was not_____ by the criticism and paid no_____ (a) permit, orbits (b) accept, revolves
even when her best friend talked against her. (c) experience, circles (d) assume, went
17. Unpredictable __________ of the child could not lead the
(a) threatened, warning (b) troubled, mind
consultants to any ___________.
(c) deterred, heed (d) bothered, attention
(a) performance, setting (b) belief, judgment
7. He has a____tongue; his pinching sarcasm has______
(c) operation, purpose (d) behaviour, conclusion
everyone who has come into contact with him.
18. A public, servant who is guilty will not ___________
(a) wanton, immunised
punishment and no ___________ person will be punished.
(b) vitriolic, alienated
(a) be, sincere (b) flee, guilty
(c) pungent, animated
(c) defend, common (d) avoid, uninformed
(d) recalcitaant, humanised

Downloaded From : www.EasyEngineering.net


Downloaded From : www.EasyEngineering.net

232  l  Fill in the Blanks (FIB)

19. Few professions can ___________ the sheer variety and 30. Football evokes a ___________ response in India compared
constant ___________ of being a doctor. to cricket, that almost ___________ the nation.
(a) like, struggle (b) share, enthusiast (a) tepid, boiling
(c) match, challenge (d) draw, work-load (b) lukewarm, electrifies
20. The organisation ___________ to popularise Indian classical (c) turbid, fascinating
music among the youth which has lost ___________ with (d) apocryphal, genuinely fascinates
its cultural roots. 31. Though one eye is kept firmly on the___________, the
(a) endeavours, touch (b) wishes, interest company now also promotes ___________ contemporary
(c) efforts, experience (d) exerts, intrigue art.
21. One of the major critiques of the examination system is that (a) present, experimental
it ___________ to a spirit of ___________ competition (b) future, popular
among the students. (c) present, popular
(a) results, defective (b) accompanies, adequate (d) market, popular
(c) develops, intense (d) takes, severe 32. The law prohibits a person from felling a sandalwood tree,
22. The ___________ of the chronic balance of payments deficit even if it grows on one’s own land, without prior permission
which has ___________ the Finance Ministry under three from the government. As poor people cannot deal with

ww
Prime Ministers is very real.
(a) temptation, reviled
the government, this legal provision leads to a rip-roaring
business for ___________, who care neither for the

w.E
(b) understanding, menaced
(c) impact, underestimated
(d) dilemma, plagued
___________ , nor for the trees.
(a) middlemen, rich
(c) touts, rich
(b) the government, poor
(d) touts, poor

asy
23. Our Constitution was based on the belief that the free
___________ of ideas, people and cultures is essential to the
___________ of a democratic society.
33. It will take some time for many South Koreans to
___________ the conflicting images of North Korea, let
alone to___________ what to make of their northern
(a) selection, concurrence
(b) interchange, preservation En cousins.
(a) reconcile, decide (b) understand, clarify
(c) reversal, upholding
(d) dissemination, congruence
24. As this country has become more ___________ industrial
gin (c) make out, decide (d) reconcile, understand
34. The manners and ___________ of the nouveau riche is a
recurrent ___________ in the literature.
and internationalised, it has like all Western democracies in
the ___________ of the executive. eer
(a) style, motif
(c) wealth, theme
(b) morals, story
(d) morals, theme
(a) urbanised, role (b) objective, wealth
(c) synthesised, efficiency (d) civilized, convenience ing
35. These issues are extremely ________ and any knee jerk
reaction will ultimately result in a loss of ________ for all
25. More is ___________ of conditions of the tribals in
Maharashtra than ___________ conditions of those in the
other parts of the country.
shareholders
(a) Unassociated, curare
.ne
(b) Ambiguous, plutocracy
(c) Nuanced, opportunity (d) Contexed, serendipity
(a) certain, the
(c) aware, of
(b) known, of
(d) aware, of
26. Although ___________ is not a very desirable feeling we
need a certain amount of it to ___________ well.
36. Growth under this government has been ________ high
and remarkably ________ even during the worst global
economic crisis.
(a) Impededly, flippant
t
(b) Relatively, intractable
(a) anxiety, exist (b) grief, enjoy (c) Obstructedly, rigid (d) Sustainedly, resilient
(c) pain, bestow (d) impatience, preach 37. There are different and ________ versions about what
27. Although he is ___________ person, he occasionally loses happened in the city, but one thing is certain. It is a dastardly
his ___________ act that must be condemned ________
(a) quiet, power (b) cheerful, grief (a) Dissimilar, concertedly
(c) balanced, temper (d) thoughtful, Anxiety (b) Contrary, obviously
28. In a ___________ tone, the leader made a powerful (c) Conflicting, unequivocally
___________ to the mob. (d) Unique, without conflict
(a) realistic, zeal (b) lower, conviction 38. They ________ their seats away from the curved wall panels
(c) loud, argument (d) soft, appeal to give themselves more space as the flight attendant brought
29. The tunnel was so ___________ and congested, that we drinks from the gallery, which was ________ with family’s
became ___________. favorite snacks and beverages.
(a) long, enthusiastic (b) deep, Cautious (a) Swiveled, stocked (b) Hinged, lacquered
(c) dark, frightened (d) crowded, isolated (c) Pended, embellished (d) Retracted, thronged

Downloaded From : www.EasyEngineering.net


Downloaded From : www.EasyEngineering.net

Fill in the Blanks (FIB)  l 233

39. Cairn cannot ________ bring into picture some ________ 48. The genocides in Bosnia and Rwanda, apart from being
outsider which has little experience and necessary consents mis-described in the most sinister and _________ manner
to deal in the oil field. as 'ethnic cleansing', were also blamed, in further hand-
(a) Peremptorily, ascribed washing rhetoric, on something dark and interior to
(b) Complaisantly, endorsed __________ and perpetrators alike.
(c) Democratically, aberrant (a) innovative, communicator
(d) Arbitrarily, unrelated (b) enchanting, leaders
40. Economic growth is on auto-pilot, unlikely to be derailed by (c) disingenuous, victims
any lapse into ________ and controls or to be ________ by (d) exigent, exploiters
serious policy reforms. 49. As navigators, calendar makers, and other _________ of
(a) Growth, blowed (b) Boom, berated the night sky accumulated evidence to the contrary, ancient
(c) Recession, reduced (d) Dirigisme, boosted astronomers were forced to __________ that certain bodies
41. A growing number of these expert professionals might move in circles about points, which in turn moved in
___________ having to train foreigners as the students end circles about the earth.
up ___________ the teachers who have to then unhappily (a) scrutinizers, believe (b) observers, agree
content with no jobs at all or new jobs with drastically (c) scrutinizers, suggest (d) observers, concede

ww
reduced pay packets
(a) are, supplanting
(c) resist, challenging
(b) welcome, assisting
(d) resent, replacing
50. Every human being, after the first few days of his life, is
a product of two factors: on the one hand, there is his
__________ endowment; and on the other hand, there is

the ___
w.E
42. Women should be paid the same as men-when they do the
same job, for surely, what is sauce for the______ is sauce for
the effect of environment, including _________
(a) constitutional, weather (b) congenital, education
(c) personal, climate (d) economic, learning
(a) goose, gander
(c) fox, vixen asy
(b) cock, hen
(d) buck, doe
51. It____ not look like a great deal today, but back then it
was a coup: no man before ____ to import tea directly into
43. He has a____tongue; his pinching sarcasm has______
everyone who has come into contact with him.
(a) wanton, immunised En Ireland.
The option that will best fill the blanks in the above sentences

(b) vitriolic, alienated


(c) pungent, animated gin would be:
(a) may, has dared
(c) might, have dared
(b) may, had dared
(d) might, have ever dared
(d) recalcitaant, humanised
44. Though one eye is kept firmly on the___________, the
eer
52. The head ____ was annoyed to see a ____ in the soup.
The option that would best fill the blanks in the above
company now also promotes ___________ contemporary
art.
(a) present, experimental (b) future, popular ing
sentence would be:
(a) chief, house fly
(c) chief, house-fly
(b) chef, housefly
(d) chef, house fly
(c) present, popular (d) market, popular
45. The law prohibits a person from felling a sandalwood tree, .ne
53. There is much difficulty _____ getting _____ this place and
it is not possible to reach _____ without the grace of the
even if it grows on one’s own land, without prior permission
from the government. As poor people cannot deal with
the government this legal provision leads to a rip-roaring
business for ___________, who care neither for the
lord.

would be:
(a) in, to, it (b) to, to, it
t
The option that best fill the blanks in the above sentence

___________ , nor for the trees (c) to, in, it (d) in, in, in
(a) middlemen, rich (b) the government, poor 54. When you want to digitalise a city __________ with
(c) touts, rich (d) touts, poor millions, you don’t bet __________ the odds.
46. It will take some time for many South koreans to (a) proceeding, into (b) teeming, against
___________ the conflicting images of North Korea, let (c) undergoing, adhere (d) dangling, for
alone to___________ what to make of their northern 55. The numbers __________ by the legitimate online music
cousins service providers indicate that a growing number of users
(a) reconcile, decide (b) understand, clarify are __________ to buy music.
(c) make out, decide (d) reconcile, understand (a) morphed, ignoring (b) labelled, thriving
47. In these bleak and depressing times of ___________ prices, (c) figured, fanatic (d) touted, willing
non-performing governments and ___________ crime 56. In India is __________ on protecting its resources,
rates, Sourav Ganguly has given us, Indians , a lot to cheer international business appears equally __________ to
about safeguard its profit.
(a) escalating, increasing (b) spiralling, booming (a) dreaded, fragile (b) stubborn, weak
(c) spiralling, soaring (d) ascending, debilitating (c) bent, determined (d) approaching, settled

Downloaded From : www.EasyEngineering.net


Downloaded From : www.EasyEngineering.net

234  l  Fill in the Blanks (FIB)

57. Brands __________ decision-simplicity strategies make (c) few will know, or care about
full use of available information to __________ where (d) when a lot of water will have passed under the bridge,
consumers are on the path of decisions making and direct who will care
them to the best market offers. 67. When we call others dogmatic, what we really object to is
(a) diluting, divulge (b) tempting, maintain ___________ .
(c) imputing, overdrive (d) pursuing, assess (a) their giving the dog a bad name
58. Lack of financing options, __________ with HR and (b) their holding dogmas that are different from our own
technological __________ , make small and medium
(c) the extremism that goes along with it
enterprises sector the most vulnerable component of our
(d) the subversion of whatever they actually believe in
economy.
concomitantly
(a) except, loophole (b) coupled, challenges
(c) armed, benefits (d) registered, strategies 68. Although it has been more than 50 years since Satyajit Ray
59. The water transport project on the west coast is ____ to made Pather Panchali, ___________ refuse to go away from
get a shot in the arm with a new plan in which the Road the mind.
Development Corporation will build the infrastructure and (a) the haunting images (b) its haunting images

ww
___ a private party to operate the service.
(a) scheduled, let
(c) set, sanctions
(b) verge, permit
(d) bound, task
69.
(c) its haunted images (d) the haunt of its images
___________, the more they remain the same.
(a) People all over the world change

w.E
60. As the weekend finally rolled around. the city folk were only
___ happy to settle down and laugh their cares _____.
(a) just, afar (b) too, away
(b) The more people change
(c) The more they are different

(c) extremely, off


asy
(d) very, up
61. The flood of brilliant ideas has not only ___ us, but has also
70.
(d) The less people change
The stock markets ___________. The state they are in right
now speaks volumes about this fact.
(a) overwhelmed, extend (b) enjoyed, stretch
En
encouraged us to ___ the last date for submission of entries.
(a) is the barometer of public confidence
(b) are the best indicators of public sentiment

(c) dismayed, decide (d) scared, scrap
62. ___ about prolonged power cuts in urban areas, the
authorities have decided to ___ over to more reliable and gin (c) are used to trade in expensive shares
(d) are not used to taking stock of all markets
eco-friendly systems to run its pumps.
(a) Worried, shift (b) Frantic, move

71.
penetrate.eer
This is about ___________ a sociological analysis can

(c) Troubled, jump (d) Concerned, switch


63. The high cutoff marks this year have ___ college admission- ing
(a) as far as
(b) the outer limit that
seekers to either ___ for lesser known colleges or change
their subject preferences.
(a) cajoled, ask (b) pressured, sit
(c) just how far into the subject
(d) just the relative distance that
.ne
(c) forced, settle (d) strained, compromise
64. _________ of illiteracy from a nation that is set to become
the most populated in the world is by no ________easy.
72.

(a) set out to


(c) thought to
(b) went to
(d) thought of
t
I am always the first to admit that I have not accomplished
everything that I ___________ achieve five years ago.

(a) Countering, task (b) Driving, measure


(c) Curbing, way (d) Eradication, means 73. This is not the first time that the management has done
65. I am an entertainer, ___________ , I have to keep smiling some ___________ .
because in my heart laughter and sorrow have an affinity. (a) tough talk (b) tough talking
(a) even if I have tears in me (c) firm talk (d) firm talking
(b) even though I am depressed inside 74. The present Constitution will see ___________ amendments
(c) while entertaining people but its basic structure will survive.
(d) in the entertainment business (a) much more (b) many more
66. Political power is just as permanent as today’s newspaper. (c) too many more (d) quite a few more
Ten years down the line, ___________, who the most
75. Education is central because electronic networks and
powerful man in any state was today. software-driven technologies are beginning to ___________
(a) who cares the economic barriers between nations.
(b) nobody will remember what was written in today’s (a) break down (b) break
newspaper or (c) crumble (d) dismantle

Downloaded From : www.EasyEngineering.net


Downloaded From : www.EasyEngineering.net

Fill in the Blanks (FIB)  l 235

76. Science is a sort of news agency comparable ___________ 85. Lack of financing options, __________ with HR and
to other news agencies. technological __________ , make small and medium
enterprises sector the most vulnerable component of our
(a) principally (b) in principle
economy.
(c) in principal (d) in spirit and form (a) except, loophole (b) coupled, challenges
77. Most political leaders acquire their position by causing (c) armed, benefits (d) registered, strategies
a large number of people to believe that these leaders are
DIRECTIONS (Qs. 86-90) : The following questions consist of
___________ by altruistic desires.
a single sentence with one blank only. You are given six words as
(a) actuated (b) convinced
answer choices and from the six choices you have to pick up two
(c) categorized (d) led
correct answers, either of which will make the sentence meaningfully
78. Every one will admit that swindling one’s fellow beings is a
complete. [IBPS PO 2011]
necessary practice; upon it, is based really sound commercial
success ___________. 86. The ability of a woman to do well does not ______ on
(a) sell what you cannot buy back whether it is a man’s world or not, because everyone has
his/her own opportunities.
(b) buy what you will sell to another at a higher price
(1) trust (2) depend
(c) buy cheap and sell dear (3) reckon (4) live

ww
(d) sell what you can, do not buy from a competitor
79. The petitioner had ______ an immediate stay form the court
(5) rest
(a) (4) and (5)
(6) believe
(b) (2) and (3)

w.E
on allotment of the Hats ________ of investigation into the
alleged irregularities.
(a) asked file process (b) sought, completion 87.
(c) (1) and (6)
(e) (3) and (4)
(d) (2) and (5)

Drugs worth ` 3 lakhs were ......... from the apartment by


the police.
(c) propagated, finish

asy
(d) demanded, course
80. Modern music doesn't king ______ to longevity, sometimes
it is hard even to remember what the biggest hit from a
(1) manufactured
(3) seized
(2) ruptured
(4) confiscated
_____ of years ago was.
(a) itself, couple (b) much few En (5) bought
(a) (1) and (4)
(c) (3) and (5)
(6) compared
(b) (2) and (3)
(d) (5) and (6)
(c) money some (d) mother, spatter
DIRECTIONS (Qs. 81-85): Each question below has two blanks, gin
88.
(e) (3) and (4)
An organization .......... to the mission of road safety has
each blank indicating that something has been omitted. Choose the
set of words for each blank that best fits the meaning of the sentence eer
prepared an action plan for reducing accidents and related
injuries and fatalities.
(1) specified (2) inaugurated
as a whole. [SBI PO 2011]
81. When you want to digitalise a city __________ with millions, ing
(3) committed
(5) succumbed
(4) kicked off
(6) dedicated
you don’t bet __________ the odds.
(a) proceeding, into (b) teeming, against
(a) (3) and (6)
(c) (3) and (5)
(e) (1) and (3) .ne
(b) (1) and (5)
(d) (4) and (6)
(c) undergoing, adhere
(e) falling, above
(d) dangling, for

82. The numbers __________ by the legitimate online music


service providers indicate that a growing number of users
89.
t
A man reportedly .......... two passports with the same
photograph, but under different names was arrested by the
commissioner’s Task Force.
are __________ to buy music. (1) possessing (2) examining
(a) morphed, ignoring (b) labelled, thriving (3) surrendering (4) mastering
(c) figured, fanatic (d) painted, interested (5) holding (6) fixating
(e) touted, willing
(a) (2) and (3) (b) (3) and (6)
83. In India is __________ on protecting its resources,
international business appears equally __________ to (c) (1) and (5) (d) (1) and (4)
safeguard its profit. (e) (4) and (5)
(a) dreaded, fragile (b) stubborn, weak 90. The Hollywood star and the Bollywood heroine are being
(c) bent, determined (d) approaching, settled ......... as the next big onscreen couple.
84. Brands __________ decision-simplicity strategies make (1) labeled (2) explained
full use of available information to __________ where (3) worshiped (4) touted
consumers are on the path of decisions making and direct (5) exclaimed (6) shouted
them to the best market offers.
(a) (2) and (4) (b) (1) and (3)
(a) diluting, divulge (b) tempting, maintain
(c) imputing, overdrive (d) pursuing, assess (c) (2) and (6) (d) (1) and (4)
(e) employing, trust (e) (3) and (4)

Downloaded From : www.EasyEngineering.net


Downloaded From : www.EasyEngineering.net

236  l  Fill in the Blanks (FIB)

DIRECTIONS (Qs. 91-95) : The following questions consist of a 96. ________ pollution control measures are expensive, many
single sentence with one blank only. You are given six words denoted industries hesitate to adopt them.
by A, B, C, D, E & F as answer choices and from the six choices (a) Although (b) However
(c) Because (d) Despite
you have to pick two correct answers, either of which will make the
97. It is not ______________ for a man to be confined to the
sentence meaningfully complete. [IBPS PO 2012] pursuit of wealth.
91. ___________ before the clock struck 8 on Saturday night, (a) healthy (b) easy
India Gate was swamped with people wearing black tee- (c) possible (d) common
shirts and holding candles. 98. ______________ his being innocent of the crime, the judge
(A) Minutes (B) Time sentenced him to one year imprisonment.
(C) Later (D) Quickly (a) Inspite of (b) In case of
(E) Since (F) Seconds (c) On account of (d) In the event of
(a) (B) and (E) (b) (A) and (C) 99. It is a story of two men and a batch of ______________
(c) (A) and(F) (d) (B) and (D) armoured cars.
(e) (C) and (E) (a) deceased (b) diseased
92. The States should take steps _________ to the process of (c) decrepit (d) defeated
teachers” appointments as the Centre has already sanctioned 100. Although there is ______________ gunfire, there is no stiff

ww
six lakh posts.
(A) fasten
(C) hasten
(B) move
(D) speed
resistance to the revolutionary army.
(a) bitter (b) meagre

w.E
(E) early
(a) (D) and (F)
(c) (C) and (F)
(F) quicken
(b) (A) and (C)
(d) (D) and (E)
(c) continuous (d) sporadic
DIRECTIONS (Qs. 101 - 105) : In the following questions,
sentences are given with blanks to be filled with an appropriate

93.
(e) (B) and (D)
asy
A senior citizen’s son_________threatened her every day
and physically harmed her, forcing her to transfer her
word(s). Four alternatives are suggested for each question. Choose
the correct alternative out of the four and indicate it.
[SSC CGL, 2013]
properly to him.
(A) superficially (B) mistakenly En 101. Mr. Murugan has been in this college ________ 2010.
(a) for (b) since
(C) allegedly
(E) doubtfully
(D) miserably
(F) purportedly gin (c) after (d) before
102. We attended a ________ discourse.
(a) (C) and (F)
(c) (C) and (E)
(e) (A) and (C)
(b) (A) and (E)
(d) (D) and (F)
eer
(a) spiritual
(c) spirituality
(b) spirituous
(d) spiritually
103. The valley is known for its ________ growth of vegetation.
94. Medical teachers said that the management had continued
to remain_________to their cause leading to the stretching
of their strike.
ing
(a) luxurious
(c) luxuriant
(b) luxury
(d) luxuriously
(A) unmoved
(C) unconcerned
(B) lethargic
(D) apathetic religion. They are universal. .ne
104. Satyajitray’s films ________ all barriers of caste, creed and

(E) indifferent
(a) (B) and (C)
(c) (A) and (E)
(e) (D) and (E)
(F) bored
(b) (C) and (F)
(d) (A) and (D)
(a) transcend
(c) trancend
(b) transcends
(d) transend
105. I could hardly recognize him ________ I saw him.
(a) after (b) but
t
95. The parents had approached the high court to_________the (c) and (d) when
government order after their children, who passed UKG,
were denied admission by a school. DIRECTIONS (Qs. 106-110) : Each question below has two
(A) void (B) quash blanks, each blank indicating that something has been omitted.
(C) annual (D) stay Choose the set of words for each blank that best fits the meaning of
(E) lift (F) post the sentence as a whole. [SBI PO 2013]
(a) (A) and (D) (b) (B) and (C)
(c) (C) and (E) (d) (E) and (F) 106. In an effort to provide _________ for higher education to
(e) (C) and (D) all, most of the universities have been providing education
without adequate infrastructure, thus churning out
DIRECTIONS (Qs. 96 - 100) : In the following questions, sentences _________ graduates every year.
are given with blanks to be filled in with an appropriate word(s). (a) chances, fresh
Four alternatives are suggested for each question. Choose the (b) platform, capable
correct alternative out of the four and indicate it by blackening the (c) opportunities, unemployable
appropriate rectangle in the Answer-Sheet. (d) prospects, eligible
[SSC CGL 2012] (e) policy, incompetent

Downloaded From : www.EasyEngineering.net


Downloaded From : www.EasyEngineering.net

Fill in the Blanks (FIB)  l 237

107. The move to allow dumping of mercury _________ An (a) forecasted-shrank (b) forecast-shrank
outcry from residents of the area who _________ that (c) forecast-shrink (d) predicted-expand
high levels of mercury will affect their health and destroy (e) predictions-wan
ecologically sensitive forest area. 112. It is the role of the state to ____ crime and protect people
(a) resulted, insist (b) provoked, fear
and property. If the state is unable to prevent a crime it falls
(c) incited, determined (d) activated, accept
(e) angered, believe upon the state to __ the victim.
108. Even as the _________ elsewhere in the world are struggling (a) prevent-support (b) preventing-encourage
to come out of recession, Indian consumers are splurging on (c) prevent-supporting (d) forbid-discourage
consumer goods and to _________ this growth, companies (e) forbid-discouraging
are investing heavily in various sectors. 113. A person who is clean and tidy in how he dresses up
(a) economies, meet (b) countries, inhibit commands better_________from those around him than
(c) governments, measure (d) nations, inflict those “who have a–and unkempt appearance_________
(e) companies, counter (a) respectful - slovenly (b) respect - slovenly
109. Drawing attention to the pitfalls of _________ solely on (c) respected - untidy (d) respect - tidy
Uranium as a fuel for nuclear reactors, Indian scientists
(e) respect - careful
warned that Uranium will not last for long and thus research

ww
on Thorium as its must be revived _________.
(a) using, substitute
(b) believing, replacement
114. Today we have achieved a milestone by completing 60 years
of independence. It’s now the time for everyone or every
Indian to undergo_________of the achievements we already

w.E
(c) depending, reserve
(d) reckoning, option
(e) relying, alternative
made and also those that are to be still_________
(a) self-introspection-achiver
(b) self-examination-achieve

asy
110. ________ has been taken against some wholesale drug
dealers for dealing in surgical items without a valid license
(c) introspection-achieved
(d) search-found
and maintaining a stock of _________ drugs.
(a) Note, overwhelming (b) Step, impressive
(c) Execution, outdated (d) Action, expired En (e) cross-inspection - made
115. Education is an essential means of_________women with
(e) Lawsuit, invalid
DIRECTIONS (Qs. 111-115) : Pick out the most effective pair of gin the knowledge, skills and self-confidence necessary to
fully_________ in the development process.
(a) empower- include
words from the given pair of words make the sentences meaningfully
complete. [SBI PO, 2014]
111. Weather officials have __ below-normal rains this year.
eer
(b) empowering - participate
(c) empowered - participating
If the predictions come true, farm output could __ as most
of India’s farmlands depend on rainwater for irrigation. ing
(d) empowerment - participate
(e) strengthening - participate

.ne
t

Downloaded From : www.EasyEngineering.net


Downloaded From : www.EasyEngineering.net

238  l  Fill in the Blanks (FIB)

Hints & Solutions


LEVEL-I 14. (c) If it would have been ‘security’ then it would have been
‘provided’ in the statement not ‘subjected’. Exercise does not fit.
1. (b) Violation (to breech) is also out of place. Use of ‘physical’ in
2. (c); Option (c) is correct, because a product is launched. Launch statement indicates ‘violence’. It fits best in the blank.
refers to introducing something to the public for the first time. 15. (c) War is a negative word and it is synonym with mass
Start means to just begin or set in motion, it is synonymous destruction, death hence certainly has grim consequences.
with ‘begun’; whereas, commissioned means ‘to give an official Happy, Pleasing, exciting all have positive sense; so can not be
approval’. linked with war.
3. (d); The speaker is saying that his novel was a success, but that 16. (d) Suppose you do not know exact meaning of all the options.
is not what he/she expected. Option (d) is correct, because Let us try solving it with critical reasoning approach. ‘Bad for
making a fortune means achieving a lot of success, prosperity science’ does not look sense full. ‘Bad to’ does not suit. ‘Bad in
or making lots of money. So, it refers to achieving success, science’ looks convincing but it is slang not the right answers.

ww
which is what the context is. Option (a) is wrong, because ‘days
were up’ is an idiom which refers to ‘ending’ or ‘coming to an
end’, this ending could be of anything, like success, happiness
etc. But the author, in fact, got success, so this option is logically
People usually use - I am bad in mathematics but not in science.
This is not right as per Standard English. With subjects ‘at’ is
used as connector.
17. (d) A clever way to tackle this question would be to see the

w.E
incorrect. Option (b) is wrong because chances are not talked
about after the result and similarly, option (c) can be ruled out
too.
second as a prepositional usage. You go on foot. So, our choices
narrow down to (c) and (d). Now, would you be utilising time
if you go on foot? Perhaps not. So, (c) is eliminated. If you

asy
4. (b); The statement probably talks about a situation in which a
boy catches fire and the neighbour saves him. The neighbour
did that by grabbing and then rolling him on the road to
do something in order to gain time you do it in order to give
yourself enough time. For what? To think of an excuse or a way
out of a difficult situation.

En
extinguish the fire. We have to pick the right word that describes
this action. Option (b) is correct, because to ‘kill the flames’
18. (b) Note that the paralytic attack was mild, which gives us the
clue that its effect would be slight. This is further substantiated
would mean extinguishing the fire and this is exactly what is
needed here. Option (a) is wrong. Option (c) is wrong, because
the flames do not burn out, the fuel does. Option (d) is wrong, gin by the second part of the sentence:” …otherwise he is still very
active” means there is some (slight) trouble but not much.
19. (d) As a matter of rule, who should suffer? The guilty?. But the
because fizz refers to become bubbly or frothy, so it is clearly
not the right word here.
5. (d); Option (d) is correct, as the secret was kept between eer
word despite points to the contrary. Hence, innocence.
20. (d) When someone else has good fortune (luck), the general
the speaker and Sam. Option (a) is wrong, because then the
statement becomes redundant. Option (b) is incorrect. Option ing
feeling is: “Why is he the favoured one? Why not I?” This is a
feeling of envy.
21. (b) Ushered (show or guide somewhere) seems to be the most
(c) is wrong, because amongst is used in the case when more
than two people or entities are involved.
6. (b); The speaker is talking about how sometimes the greatest .ne
appropriate in the context of the sentence.
22. (a) Only something negative can prevent a good measure. We
inventions are formed on or depend on simple ideas or basic
thoughts. Option (b) is the correct option because it means ‘to
depend on’, which fits in perfectly with the statement as then
it would mean that some of the greatest inventions depend on
t
have two choices representing negativeness: (a) and (b). But
(b) is not applicable: A syndrome is a medical condition that is
characterised by a particular group of signs and symptoms.
23. (a) Let us look at the look phrases. If you look after someone,
simplistic ideas. you do what is necessary to keep him healthy, safe, or in good
7. (a); The given statement refers that real friends i.e. the ones who condition. If you look after something, you are responsible for
really feel for the organization, are witnessed in different forms it. It is your duty to see that it functions all right. If you look
or appearances. Although, ‘garb’ refers to clothes, and thereby ahead you think about what is going to happen in the future;
options (b), (c), and (d) do look probable, but ‘come in’ refers to you make plans for the future. On the contrary, if you look back
‘approaching someone or appear in front of someone’ which is you think about things that happened in the past. If A looks
what the author means. down on B, A considers B to be inferior or unimportant. If you
8. (b) The word ‘probably’ means likely (very/most). look forward to something that is going to happen, you want
9. (b) The word ‘broke down’ means collapse.
it to happen because you think you will enjoy it. If you look
10. (d) The word ‘looking for’ means in search of.
into a problem you try to find out about it; you examine the
11. (a) The word ‘hanging around’ means linger or wait around.
facts related to the problem. If you look on while something
12. (a) The word ‘backed out’ means go back on or withdraw from.
happens, you watch it happening without taking part yourself.
13. (c) Remorse is an emotion/action/feeling after the crime
If you look up a fact or a piece of information, you find it out by
is done. Hatred is for the other people who saw/hear/ gone
through crime but certainly not for the prisoner. Crime is done looking in a reference book, list etc. If you look up to someone,
in anger and a cause. Obstinacy is stubbornness. Only emotion especially someone, older than you, you respect and admire
that suits the context is Remorse. him.

Downloaded From : www.EasyEngineering.net


Downloaded From : www.EasyEngineering.net

Fill in the Blanks (FIB)  l 239

24. (b) The word penal means “punishable by law”. Hence, offence co-ordination. If there is friction between people, there is
is the correct choice. An offence is a crime that breaks a disagreement and argument between them. That is exactly
particular law and requires a particular punishment. A sin, on what happens “between labour and management.”
the other hand, is a moral wrong. If you sin, you break the laws 41. (d) Deliver is used in the following senses. If you deliver
of God. something somewhere, you take it there. For example: The
25. (d) The “comic’: (comics book) changes hands - it goes from postman delivers letters at our homes. When you deliver what
the boy to the teacher. Only (b), (c) and (d) give this sense of you have promised to do or make, you do it or make it. For
change of hands. Of these, seized is the most appropriate. If example: The manager promised to get ads but failed to deliver.
someone in authority (here “the teacher”) seizes your property If you deliver A into B’s care, you give B responsibility for
(here “the boy’s comic”), they take it from you, often by force. A. For example: 11cr mother delivered her to me before she
Of course, no boy would willingly part with his “comic” (her mother) died. If you give a lecture or speech in public,
26. (b) A literal translation is one in which you translate each ‘word you deliver it. [The given question has this usage.] When you
deliver a baby, you help the woman who is giving birth to the
of the original work. Since each language has its own idiom, a
baby. If A delivers a blow to B, A hits B. If someone delivers you
literal translation leads to awkward usage. A literal translation from something, he rescues or saves you from it.
is therefore not considered good. A good translation gives 42. (b)
the meaning of each expression, sentence or paragraph, using 43. (c) If a language is spoken by many people or throughout a
words that sound natural. large area, we say that it is widely spoken.
27. (b)

ww 28. (d) 29. (b)


30. (b) If you are the cynosure of all eyes, you are a centre of
attention or interest.
44.

45.
(c) If you have a right to do something (here “vote”), you are
morally or legally entitled to do it. “The age of 18” is the legal
barrier here.
(a) If you implement something (here “an accord” - that is,

w.E
31. (b) Your preoccupations are the activities that take up your
attention (or time), leaving little room for other things.
32. (d) What is the difference between alternately and alternatively?
Alternatively implies that you have a choice (alternative) —
46.
agreement), you ensure that it is brought into practice.
(b) The trend shows which way the graph is moving: is it rising
or falling? The word “rise” gives us the clue.

asy
either by sea or by road. In the given question conjunction “or”
gives us the clue. When we say alternately it means first by sea
then by road, the third time by sea again, the fourth time by
47. (b) If you raise a subject (here, “a controversial issue”), an
objection, or a question, you mention it or bring it to someone
attention (here, to the attention of the members of the

this case, the conjunction should be “and”. En


road again, and so on. Since both the routes are being used in
48.
“Assembly”).
(b) Here, the infant mortality rate (IMR) moves from a higher
level (200) to a lower level (14). Hence, declined.
33. (c) If A transmits a disease to B, A has the disease and causes B
to have it. Here, “disease” gives us the clue.
34. (a) Both (a) and (b) are correct but apprised is a formal usage. gin
49. (d) The word “ransom” gives us the clue. A ransom is the
money (` 8, 000) that has to be paid to the kidnapper (here,
“servant”) so that he will set free the person kidnapped (here,
Such formal usages are preferred when the persons involved
include “the Chief Minister” himself - a stately personage.
35. (d) The outskirts of a city (here “Hyderabad”) or town are the
50. eer
the “five-year-old boy”).
(a) If something (here, the driver’s licence) is impounded by

part that are farthest away from its centre.


36. (d) If there is congestion in a place, the place is extremely ing
the police, customs officers or other officials, they officially take
possession of it because a law or rule has been broken (here,
“rash driving”). Impounded thus means seized in a specific
crowded and blocked with traffic or people. The problem of
congestion is thus essentially a problem of space. It could be
got rid of through addition of space (by developing “satellite
51.
52.
context.
(a)
.ne
(c) He has already decided (“made up his mind”). In order that
ports”). Blockade is slightly different. It is the action that is
taken to prevent goods or people from entering or leaving it.
The problem here is political - blockades are the creation of
strikers or political groups. And the problem could persist even
53.

54.
he “made up his mind”. Now, it is futile.
t
the “argument” might succeed, it should have been done before

(b) If you cast aspersions on someone, you suggest that he is


not very good in some way.
(a) When people (here ‘audience’) applaud they clap their
at the newly-developed satellite ports.
hands in order to show approval, for example when they have
37. (b) (a), (b) and (d) are close choices. Distraction happens
enjoyed a drama or concert.
to your thoughts. When we are talking of ‘path’, it is either 55. (a) A rite is a traditional ceremony carried out by a particular
deviation or diversion. While diversion is moving away from group or within a particular society. The last rites are performed
the main course (road), deviation is moving away from the when a person dies.
normal (here “chosen”) path. 56. (a) The word ‘sceptical’ means suspicious or doubtful.
38. (d) (e) is rejected because the date became known to the 57. (a) The word ‘inflict’ means burden someone with or impose.
members; the members were not known. Similar is the case 58. (d) The word ‘inherit’ means become heir to or take over.
with (1) and (2), (3) is rejected because it does not make sense. 59. (c) For statements, phrase ‘accord with’ is used. See the use of
You can know (be informed) of something in advance. But you phrase in one of the judgments of a court -
can’t be ignorant (not aware) “in advance ‘However, the evidence suggests that the administration of
39. (a) (b), (c) and (d) are the pre-action phase. One fails or complaints does not always accord with established procedures.’
succeeds only when one does a work. That is, when one tries. Use of ‘Accord to’ is not right in this sentence.
Hence, attempt. (e) makes no sense at all. 60. (a) Vague notion is a right choice. Imagination is in itself
40. (c) Students of physics must be well aware of friction. Friction vague (unclear). While expressions cannot be vague. Theory is
is the force that makes it difficult for things to move freely out of place for the context. Thoughts can be vague or unclear
when they are touching each other. It thus shows a lack of or unstable.

Downloaded From : www.EasyEngineering.net


Downloaded From : www.EasyEngineering.net

240  l  Fill in the Blanks (FIB)

61. (b) ‘With’ is used for ‘contentment’. All other options are not it involves both adding (“induction”) and “dropping”, it is a
valid and do not have any sense. “reconstitution” — forming it again.
62. (d) ‘Disgrace to’ is perfect as per Standard English Usage. ‘for’ 81. (b) A pretext is a reason which you pretend has caused you to
can be used with disgrace as - do something. That is, you tell people that X is the reason, but
‘His behaviour is a disgrace for all the love showered by society in fact it is not.
to his acts.’ In this type of question it is very important to read 82. (a) The words “interdisciplinary” and “interaction” point to the
and reread the sentence to get the true sense of the situation working together (co-operation) of the various departments.
described in the statement. 83. (b) If something is sensitive to a physical force, it is easily
63. (b) ‘dull of ’ and ‘dull for’ can be out rightly rejected as they do affected by it. A sensitive “stock market” implies that there
not fit well with the word ‘understanding’. ‘Dull in’ is correct.
are chances of big changes in the stock market index due to
When we understand it enters --- in ---- our mind so with
government changes or certain governmental decisions; etc.
understand ‘in’ is used.
64. (b) The verb ‘alive ‘exclusively takes ‘to’ with it. Alive to means 84. (d) A commodity is something that is sold for money. “Man
-’ interested in’ /having a lively interest is still a commodity” implies that he is treated as a thing; the
65. (d) If something contributes to an event or situation, it is one human touch is missing.
of the causes of it. Here factors has been used in the sense of 85. (d) If you are even moderately interested in news, you must
causes. If you contribute money or resources to something, you have heard of the NPT. It stands for Nuclear non-proliferation
give them to help achieve a particular purpose. Treaty — a treaty which aims at halting the proliferation

ww
66. (a) If two persons are similar, you will point out to certain
likenesses. Yet you can easily tell one from the other. In other
words, you can differentiate between the two. But the given 86.
(increasing in number very quickly) of nuclear (atomic)
weapons.
(b) An aphorism is a short witty sentence (not lofty) which

w.E
sentence says differentiation is difficult. Which means the
resemblance is to a very high degree. In other words, identical.
67. (c) We are talking of his most striking (remarkable) quality.
The characteristics of a person, thing are the qualities or
expresses a general truth or observation. Bombast is the use
of long, important sounding words with little meaning in an
attempt to impress others. (Note that the definition is silent on
presented nothing new”.) An adage is something which people

asy
features that belong to them and make them recognisable.
68. (c) The correct choice should have a meaning similar to
acquaintances but have a greater intensity. An acquaintance is a
often say and which expresses a general truth about some
aspect of life. (It need not be lofty.)

person whom one knows, especially through work or business,


but who is not a close (intimate) friend.
En 87. (b) Autocratic is the adjective from autocracy (autos = self +
kratos = power). An autocratic ruler concentrates all power
in himself. He makes decisions without asking anyone else’s
69. (b) (d) is rejected because the word should suggest that the fact
goes “in favour of ” the manager. If the profitability declines it
would go against the manager. A good managet adds to the gin
88.
advice.
(d) Common sense tells us that high saving rates should lead
profitability. In other words, the profitability increases.
70. (b) If something changes or becomes true in the course of
time, it has so become over a long period of time. 89. eer
to (greater investment and therefore) “high” growth rates. But
here it is “low”. This is a paradox
(a) If A rivals B, both A and B are of the same standard or
71. (c) ‘I expected him to give’ denotes the kind (category) of the
answer; it is not the result of ‘his answer’. So use as, not that.
72. (d)
ing
quality. In other words, A matches B. “Few countries can rival
India” means there is hardly any country that can match India.

73. (b) If you deliberate on something, you consider it carefully,


often in formal meetings with other people.
90.
India is almost unique.

.ne
(d) You condole with a person on/over his or her great
misfortune.
74. (a) What does a holiday do to you? It makes you fresh (as if
you were young) again. (2) is rejected because you do not get a
new life. (3), (4) and (5) apply to things, not to persons.
75. (d) There were no expectations of the meeting; it was only “by
chance”. In other words, the meeting was accidental.
91.

92.
93.
(c) Easy
t
(b) If you tie down someone, you restrict his freedom in some
way.

(b) “Fear” makes your voice tremble. In other words, it


76. (a) If a building or window overlooks a place (here “the rear” becomes tremulous.
- that is, the backyard), you can see the place clearly from the 94. (b) Ecology is the pattern of relations of plants, animals and
building or the window. people to each other and their surroundings (environment).
77. (c) Enumeration is the naming of things on a list (here Anthropology is the scientific study of the human (Greek
“electoral rolls”) one by one. anthropos = man) race, including its different types and. its
78. (a) beliefs, social habits and organisation, etc. Epigraphy is the
79. (d) If you highlight a point or problem, you throw greater light study of inscriptions. An inscription is writing carved into
on it than on others. That is, you emphasise it or make others something made of Stone or metal, for example, a gravestone,
think about it. monument, or medal. Numismatics is the study of coins or
80. (a) When you review something, it is merely a study, not an
medals. ‘Ecumenical activities, ideas, and movements try to
action (implementation). But “induction” is an action. So we
reject (2). Again, “the committee” already existed, otherwise unite different Christian Churches.
how could five persons have been “dropped”. And you don’t 95. (a) An incubus is a male devil supposed to have sex with a
form that which already exists. So (3) is rejected. (4) is rejected sleeping woman. Broadly, incubus means a very worrying
because “enlarging” would only involve “induction”, no problem. In literary usages, as in the given sentence, it means a
“dropping”. Similar is the case with supplemented (5). Since bad dream and is synonymous with incubus.

Downloaded From : www.EasyEngineering.net


Downloaded From : www.EasyEngineering.net

Fill in the Blanks (FIB)  l 241

96. (d) Peruse means to read. A perusal (reading) of “Solzhenitsyn’s 148. (b) On the auspicious occasion of Laxmi puja, the Mathurs
works” will “bring home to him” (make him understand) bought a new car.
the truth about “Freedom” in Russia — that freedom is only 149. (b) Precautions are to be taken with anyone who seems
nominal; it is, in fact, “restrictive”. Note that Russia here is a infectious. [ infectious means likely to transmit or spread in
part of the USSR, not the post-1991 Russia. (c) gives the sense a rapid manner. Contagious and diseased is not used in this
of reading hurriedly”. context because they refers to already having infection].
97. (d) Magnum means ‘big’ (or great). (Hence a magnifying 150. (d) The treasure was hidden off the shore. When something is
glass makes things appear big.) Opus means ‘work’. (When hidden “off the shore,” it just means that it’s hidden somewhere
you operate something, a computer for example, you make it near it.
work.) Combining the two, we get magnum opus great work. 151. (a) 152. (d) 153. (b) 154. (a) 155. (d)
But it is used only in the sense of “a great literary or artistic 156. (d) Is not learning superior to wealth?
work”. 157. (d) A group of agitators incited the mob to break down the
98. (d) If there is congestion in a place, the place is extremely Vice- Chancellor’s door. (Incited means encourage or stir up
crowded and blocked with traffic or people. The problem of (violent or unlawful behaviour).
congestion is thus essentially a problem of space. It could be 158. (b) Turn the lights off before you go to bed.
got rid of through addition of space (by developing “satellite 159. (d) There is no factual evidence to support your assertion.
ports”). Blockade is slightly different. It is the action that is 160. (a) Throw a stone at the fierce dog. [To throw a stone to

ww
taken to prevent goods or people from entering or leaving it.
The problem here is political - blockades are the creation of
strikers or political groups. And the problem could persist
someone is so that they catch it, though if they do not you might
unintentionally hit them with the stone instead! But,
To throw a Stone at someone is intentionally to hit them with

w.E
even at the newly-developed satellite ports.
99. (d) When a ship founders, it fills with water and sinks. Similarly
the carriage foundered (that is, sank) in the snowdrift (deep
pile of snow formed by the wind). The trapped (sunk) carriage
the stone.

LEVEL-II

to be extricated. asy
then had to be freed from the snowdrift. In other words, it had

100. (c) Maritime is used to describe things relating to the sea and
1. (c) (a) Serendipity (chance), faux pas (mistake) : inappropriate
pair. (b) predilection (liking), despair (loss of hope) :

to ships.
101. (b) En inappropriate pair. (c) intensity (strength of feeling or effort),
success : appropriate. (d) oddity (strangeness), conformity
(following guidelines) : inappropriate.
102. (a)
103. (e) Obviously the concept has to be explained and practical gin
2. (b) (a) Dissipated (weakened) : inappropriate. (b) created,
attracted : appropriate (especially since the conjunction is
examples can only be quoted.
104. (b) eer
‘but’ indicating a disadvantage following an advantage. (c)
inappropriate since ‘restricted’ or ‘limited’ would need ‘and’ in
the sentence, not ‘but’, since both features would be positive.
105. (e) She would not have told any one that she lad attended the
party.
106. (e) 107. (e) 108. (a) 109. (e) 110. (b) ing
(d) evicted (thrown out) : inappropriate.
3. (a) (a) Stupefying (amazing), negotiate (cope with) :
appropriate. (b) plural (many), alleviate (reduce something
111. (b)
116. (d)
112. (a)
117. (c) 118.
113. (d)
(d) 119.
114. (d)
(a)
115. (b)
120. (a) .ne
negative) : inappropriate. (c) annihilate (eliminate, wipe out)
: inappropriate. (d) dreary (boring), exasperate (annoy) :
121. (b)
126. (a)
122. (b)
127. (b)
123. (c)
128. (c)
124. (b)
129. (c)
131. (c) over, here it means came into force or effect
132. (b) interesting, exciting or fortunate
125. (b)
130. (d)
inappropriate.
4. (a) (a) Acknowledged (accepted), banishing (removing) :
t
appropriate. (b) infuriating (angering) : inappropriate. (c)
exacerbating (making worse) : inappropriate. (d) decimated
(annihilated) : inappropriate.
133. (a) with; into, here it means consisting of and plunged 5. (d) c is ruled out as apathetic and indifferent are followed by
134. (a) heavily or massively the preposition ‘to’ using heartless for nature is erroneous
135. (d) with \ (d) is correct, also intelligible is comprehensible.
6. (c) (c) and (d) are close choices. Both heed and attention are
136. (a) accomplished, proficient or skilful
acceptable for the second blank. Now, on to the first blank. You
137. (d) over are bothered with or about something, but you are deterred by
138. (c) of, here it means cured of it.
7. (b) Go for the second blank first. What is sarcasm? It is saying
139. (c) is, as it is a subject so it is a singular word
the opposite of what you mean. One gets sarcastic in order to
140. (a) over or give the responsibility to other person mock or insult others. Take an example of sarcasm: You meet
141. (c) 142. (d) 143. (d) 144. (a) 145. (a) an ugly girl. She asks you to accompany her. Repelled by her
146. (b) China is a big country. In area it is bigger than any other ugliness, you tell her sarcastically, “I don’t go around with
country except Russia. [except means other than, accept means beautiful (obviously, you mean ugly!) girls.” Such a statement
consent, expect means to anticipate and access means entrance]. pinches the girl. A girl whom you have treated with such
147. (a) the masks worn by the men helped them conceal their “pinching sarcasm” will never be friendly or sympathetic
identity. Conceal means hide. to you in future. You have thus alienated her. Those who are

Downloaded From : www.EasyEngineering.net


Downloaded From : www.EasyEngineering.net

242  l  Fill in the Blanks (FIB)

thus “pinchingly sarcastic” have a vitriolic tongue. That is, Since the issue is nuanced so it has to be studied carefully and
their words are full of bitterness and hate, and so cause a lot of given proper time to be analysed otherwise it would result in a
distress and pain. loss of opportunity. All other choices are irrevelent.
8. (b) 9. (a) 10. (b) 11. (d) 36. (d) Since the given sentence speaks about positive aspects of
12. (a) 13. (b) 14. (d) 15. (c) growth therefore.
16. (b) 17. (d) 18. (b) 19. (c) \ Sustainedly (uniformly) is best suited, relatively is irrelevant
20. (b) 21. (d) 22. (d) 23. (b) because there is no comparison made in the sentence. the first
24. (a) 25. (b) 26. (b) 27. (c) word of all other choices are absurd.
28. (d) 29. (c) 37. (c) The sentence to speaks about a bad event that has happened
30. (b) We know that in India, cricket is much more popular than in the city and that it should be strictly condemned without
football, so how football affects or provokes a reaction amongst any doubt therefore unequivocally
Indians should be nothing or very less when compared with 38. (a) Food items cannot be lacquerred, embellished decorated
cricket’s popularity. Option (b) is right, because ‘lukewarm’
or thronged they can only be stocked (arranged in an orderly
means mild or ‘the state of showing less enthusiasm or interest’,
and electrifies is what ‘excites intensely’. These words fit manner) therefore choice (a) is correct
perfectly in the given context. 39. (d) According of the sentence. An experienced and known
31. (b) ‘One eye is kept’ means that some (not full) attention or person would be right to deal in the oil field. Thus
concern is shown. So, logically, option (b) is correct, because 40. (c) Here unlikely to be derailed means unlikely to be slowed

ww
the company has kept an eye on the future. Though they are
concerned about the future, yet the company does promote
what is popular and liked at the present time. We can easily
make out why the other options are wrong.
41.
down or unlikely to become off track.
\ Option (c) is most suited.
(a) Expert professionals are having to train, and students
supplant (replace) teachers.
32.
w.E
(d) The given statement says that it is illegal to fell or cut
down a sandalwood tree without taking permission from
the government; then this statement goes onto say that ‘as’
42. (a) What is sauce for the goose is sauce for the gander means
what is acceptable for one person in a particular situation
should be acceptable for another person in a similar situation.

asy
poor people cannot really comprehend these laws or know
about them well, some people take advantage of this (this
can be logically inferred from the statement) Option (d) is
43. (b) Go for the second blank first. What is sarcasm? It is saying
the opposite of what you mean. One gets sarcastic in order to
mock or insult others. Take an example of sarcasm: You meet

En
correct because ‘touts’ are those people who persuade others,
generally in order to do some business; this fits perfectly into
the sentence as the ‘touts’ persuade the innocent poor people,
an ugly girl. She asks you to accompany her. Repelled by her
ugliness, you tell her sarcastically, “I don’t go around with
and in this context, these touts do not care about trees or the
poor (because if these poor people are caught, they will be in
trouble). It is now easily evident why rest of the options are gin beautiful (obviously, you mean ugly!) girls.” Such a statement
pinches the girl. A girl whom you have treated with such
“pinching sarcasm” will never be friendly or sympathetic

33.
incorrect.
(a) The key word in the given statement is ‘let alone’. It is
synonymous with ‘never mind’, which is used to indicate that eer
to you in future. You have thus alienated her. Those who are
thus “pinchingly sarcastic” have a vitriolic tongue. That is,
their words are full of bitterness and hate, and so cause a lot of
something is far less likely than the one already mentioned. For
example, “He cannot even run a kilometre, let alone participate
in a marathon.” So, the thing which is mentioned before this

44. ing
distress and pain.
(b) Since the company in the present promotes popular
‘let alone’ phrase has to be of a lesser degree or extent.
Option (a) is correct, because reconcile means ‘solving a
matter, or restoring friendly relations’ (North and South Korea
45.
the future.
.ne
contemporary (of the present time) art it also keeps an eye on

(d) The touts persuade the people to sell illegally thereby


separated after World War II), so it will take them time to
conclude what really went wrong with North Korea.
Option (b) is wrong, because ‘understanding’ just means ‘to
comprehend’ or ‘get it’ and clarify means ‘to make things clear’
46.
t
making a profit, not caring about either the poor or the trees.
(a) North and South Korea were divided after IInd World War
thus it will take them sometime to reconcile (restore relations)
when South Koreans cannot even decide what to do of the
but does not refer to coming to a conclusion. Option (c) is North Koreans.
wrong because ‘make out’ can mean ‘detect with senses’. Option 47. (c) Prices are usually called as spiralling up as well as the
(d) is wrong because, understand is different from deciding. increasingly soaring crime rates. Soaring means increasing
34. (d) Nouveau riche refers to people who have recently gained rapidly.
wealth. The speaker is describing how these people are a 48. (c) The situation due to the genocides is sinister, which means
constant theme for literature. Option (d) is right, because threatening. So the other word which will fill the first blank has
‘morals’ refers to the principles of what is wrong and right to complement sinister. Thus the options which may fit in can
behaviour. This fits well in the sentence, because then the be disingenuous (which means lacking in frankness, candor, or
speaker would mean: the manner (way or method in which sincerity) or tragic. The other three words are positive words. In
something is done) and morals (principles of right and wrong the last part of the sentence, 'to ……………. and perpetrators
behaviour) of the newly rich. Also, ‘theme’ is a good fit for the alike' the contrast between the blank word and perpetrators
second blank. (to commit: to perpetrate a crime) is quite clear. So the correct
35. (c) Nuance means a subtle difference in colour, meaning, tone, option is victims, which is antonym to perpetrators.
etc; a shade or graduation 49. (d) Students would be a very casual option for the first blank.
knee jerk reaction– impulsive reaction which might not be So we have to choose among scrutinizers and observers.
always appropriate because here decesion/reactions are taken However, the second part of the sentence describes how the
without much thinking . ancient astronomers were forced to change their opinion in the

Downloaded From : www.EasyEngineering.net


Downloaded From : www.EasyEngineering.net

Fill in the Blanks (FIB)  l 243

face of contradictory evidence. When someone is compelled to Option (a) is wrong, because ‘the’ instead of ‘its’ makes the
accept defeat, 'concede' is the best word that can be used. sentence ambiguous by not relating to the preceding clause.
50. (b) There is a clear-cut contrast in the sentence which is clear Option (c) is wrong, because the images were not ‘haunted’.
from the presence of the phrases 'on the one hand' and 'on the ‘Haunted’ is used to describe something that is troubled, not
other hand'. The word in the first blank has to be connected what is troubling.
with something that is present at the time of birth - thus Option (d) is wrong, because ‘haunt’ as a noun only means a
'congenital'. However, the "effect of the environment" qualifies frequently visited place.
'education'. 69. (b) The second clause identifies that it is a part of a comparative
51. (b) Sentence is given in two parts first one is given in the correlative, so option (a) can be ruled out as it does not make
present tense and second part is given in the past tense. So logical sense anyway. Option (b) is correct, because it hints
options going against this are wrong. These options are (a), (c) towards an idiom that the more you change, the more you
and (d). In first blank ‘may’ fits well and in the second blank remain the same.
‘had dared’ is grammatically right. 70. (b) First of all, the stock markets’ is a plural noun and hence
52. (b) Sentence is about seeing a fly in a soup. This is major will not take a singular reference. This rules out option (a).
concern to a chef. Hence, options (a) and (c) are rejected. The From options (b), (c) and (d), we need to choose something
word housefly is not hyphenated and a space in between house that will change the state of the stock markets and make them
and fly changes the meanings of the sentence. striking. Only option (b) does it correctly.
53. (a) There is difficulty “IN” doing something reaching a place is

54.
ww
getting to a place.
“It” pronoun used for place.
(b)
71. (c) The given statement is showing the extent or degree to
which a sociological analysis can discover or penetrate. Option
(c) is correct.
55.
56.
57.
(c)
(d)
w.E
(d) others do not fit
Option (a) is incorrect, because it does not say anything about
what is the analysis penetrating into.
Option (b) is incorrect, because ‘outer limit’ is basically a slang,
which is used to show the maximum extent.
58.
59.
60.
(b)
(d) bound - task
(a) just - afar asy 72.
Option (d) is wrong, because it is awkward.
(a) The given sentence states that the speaker did not succeed in
all the aspects or fields that he had planned to, five years ago.
61.
62.
(a) overwhelmed - extend
(d) Concerned - switch
En Option (a) is correct, because ‘set out to’ refers to what was
planned or specifically decided, so it fits perfectly into the
63.
64.
65.
(c) forced - settle
(d)
(b) The speaker in the given statement, is trying to emphasize gin sentence.
Option (b) is incorrect. Options (c) and (d) are incorrect,
because ‘thought’ means something that was merely considered
his work and what he is supposed to do, irrespective of his
emotional state. Options (c) and (d) hence, can be ruled out.
The second clause refers to smiling and being happy, so the eer
or contemplated, it does not mean that it was planned or it was
the final decision. Also, ‘thought to’ is a wrong usage of the

66.
contrasting thought would be of sadness and (b) sounds more
appropriate. Hence, option (b) is the right one.
(a)
73.
ing
preposition: ‘to’.
(a) Option (a) is correct, because ‘tough talk’ is considered a
style of management in which the employers or management
67. (b) Dogmatic is used to describe someone who has an arrogant
attitude based on unproved theories. For example, if you
74.
manner.
.ne
talk to the employees in a tough, hard and arguably, insensitive

(a) The given statement refers that there will be more


dogmatically try to prove that the world will end in 2014, you
will just be considered funny. The statement is about what
really makes us say that someone else is dogmatic. Option (b)
is correct, because dogmas are a set of beliefs that one has and
t
amendments, but that its basic structure will still remain
the same. Option (b) is correct, because ‘many more’ means
‘an increase in the number’. Option (a) is incorrect, because
‘much’ is used in cases of uncountable nouns. Option (c) is
is pretty firm about them, so when we meet people who go incorrect, because ‘too many more’ refers to a huge number of
against our ideas, and hold fast to their own, we term them amendments, and we have already explained that it’s not what
as dogmatic. For example, a non-religious person could say the statement refers to. Option (d) is incorrect, because ‘quite’
that a religious person is dogmatic. Option (a) is clearly wrong. refers to the ‘degree’ or ‘intensity’, so it is saying that there will
Option (c) is wrong, because extremism refers to any political be very few more amendments, which is not what the author
theory which favours immoderate uncompromising policies, intends.
so it is clearly out of the context. Option (d) is wrong, because 75. (a) ‘Education is central’ is highlighting the importance and
it doesn’t refer to the point in question. It simply repeats the role, electronic networks and software-driven technologies
meaning of being dogmatic. have. The sentence has a positive connotation.
68. (b) The given statement is probably mentioning a 50-year old Option (a) is correct, because ‘break down’ refers to ‘falling
movie that was very moving, and the speaker is saying that (s) apart or collapsing, or failure to function’ and the barriers are
he hasn’t forgotten those horrifying images yet. Option (b) physical or metaphorical walls which hinder free movement,
is correct, because ‘its’ makes the sentence more clear, as the hence, this is the appropriate word.
pronoun ‘its’ refers to the movie and those images are described Option (b) is wrong, because ‘break’ refers to ‘destroying the
as ‘haunting’ which means that the images in the movie were integrity of ’ or ‘get ruined’, but as we are talking about ‘barrier’,
disturbing and disquieting. ‘break down’ is more appropriate.

Downloaded From : www.EasyEngineering.net


Downloaded From : www.EasyEngineering.net

244  l  Fill in the Blanks (FIB)

Option (c) is wrong, because crumble means ‘break into pieces’. 79. (b) 80. (a) 81. (b) 82. (e)
Option (d) is wrong, because dismantle can be considered 83. (c) 84. (d) 85. (b)
synonymous with crumble. 86. (d) Option (d) is the answer and there are two reasons for it.
76. (b) The statement refers that Science and news agencies share
First one is that from the context of situation independence
similarities. Both of them share a common principle in terms
of women from the man’s world is indicated so some word to
of making the people more aware, so they are comparable ‘in
show dependency should be used. Depend and Rest both show
principle’. Option (b) is correct. Option (a) is incorrect, because
dependence so these are the answers. Second one is that as blank
principally means ‘mainly’ or chiefly’. Option (c) is incorrect,
space is followed by ON a word that connects grammatically so
because principal means the most important element or
it should be used. Both ‘depend on’ and ‘rest on’ are the correct
someone who is the head of a school. Option (d) is incorrect,
usage, so it confirms the selection on basis of the first reason.
because ‘in spirit and form’ means a very close relationship
Other options are not logical.
which seems too farfetched.
77. (a) Some politicians nowadays make fake promises in trying to 87. (e) Confiscated – means to officially take something away from
make people to vote for them, so they try tactics of deceiving somebody, especially as a punishment.
voters to vote for them. They make others believe that they have Seized definitely fits in the blank therefore option E is correct.
‘altruistic’ (showing that you are unselfish and your actions are

ww
totally concentrated on helping others) motives.
Option (a) is correct, because it means ‘motivated. The
88. (a) Organization and mission are two words which can be
connected with committed or dedicated both the verbs -
organizations committed/ dedicated to …..Objectives. Other

w.E
politicians make the voters believe that they are motivated
by altruistic desires. Option (b) is wrong because ‘convinced’
means that you are just in agreement with something. Option
words like kicked off/inaugurated/ succumbed do not match
for objectives of organization.

asy
(c) is wrong, because ‘categorized’ means ‘to place in categories’.
Option (d) means that they are ‘guided’ by altruistic desires,
but the word ‘motivate’ is more appropriate, as it shows a
89. (c) Holding and Possessing both gives the same meaning and
sense to the sentence. Surrendering is inappropriate. Mastering
can not be linked with passports. Fixating is also incorrect for
higher degree of effect.
78. (c) Swindling means depriving someone of possession or En the blank space as it gives no logical sense to the sentence.
Examining can grammatically fit into the blank but not in the
money through deception or some act of deceiving. As the
statement points out, it is considered a required or necessary gincontext of passage and would give a different direction to what
is being said in the sentence.
thing to make money, or have commercial success. We have
to pick the right option by saying what this tactic is based on.
We have to pick what refers to ‘taking someone’s money by eer
90. (d) ‘Next big thing’ is always a projection or a decision based
on current situation. It is neither worshipped nor shouted nor
deception’.
Option (c) is right, because ‘dear’ is also used to describe of projection. ing
explained. Rather next big thing is touted or labeled on basis

something that is expensive. Buying cheap and selling at a high


cost is an act of swindling.
91. (a)
94. (b)
92.
95.
(b)
(c)
.ne
93. (c)
96. (c) Because
Option (a) is incorrect, because if there is an item which you
cannot buy back from the one you sold it to, then you might get
in trouble. It is also a tactic which companies apply, but this is
not what the companies base their commercial success on, or it
97. (a) Healthy 98. (a) Inspite of
100. (d) Sporadic
t
99. (c) Decrepit

101. (b) Since is usually followed by a time expression (Last year;


this morning, 4 o'clock etc) or by a clause in the simple past
doesn’t describe a policy which companies adopt. tense. Use the present perfect or the past perfect in the other
Option (b) is wrong, because of a similar reason to option (a), clause. Use for when you the length of time that something has
but option (c) is more appropriate, as it says buy ‘cheap’ and been or had been happening. Ex; we have known each other for
sell at a very high rate, but option (b) does not quite point to ten years (Not since ten years).
making that huge margin of profit. 102. (c) 103. (a) 104. (a) 105. (d) 106. (c)
Option (d) is wrong, because it means selling whatever you can, 107. (b) 108. (a) 109. (e) 110. (d) 111. (c)
irrespective of the cost at which you bought it. 112. (a) 113. (b) 114. (c) 115. (b)

Downloaded From : www.EasyEngineering.net


Downloaded From : www.EasyEngineering.net

15
PTER
CHA

Cloze Test

CLOZE TEST Explanation : Given the above passage, students' answers may
then vary depending on their vocabulary skills and their personal
Cloze tests are common on all bank exams. They usually require opinions. However, the placement of the blank at the end of

ww
you to choose the correct choice out of four possibilities.
A cloze test (also cloze deletion test) is an exercise, test, or
the sentence restricts the possible words that may complete the
sentence; following an adverb and finishing the sentence, the
word is most likely an adjective. Romantic, chivalrous or gallant

w.E
assessment consisting of a portion of text with certain words
removed (cloze text), where the participant is asked to replace
the missing words. Cloze tests require the ability to understand
may, for example, occupy the blank, as well as foolish or cheesy.

HOW TO TACKLE A CLOZE TEST

asy
context and vocabulary in order to identify the correct words or
type of words that belong in the deleted passages of a text.
Example : A language teacher may give the following passage
• Read the text through trying to understanding the general
meaning.
to students:
En • Look at each missing word gap and try to imagine what the
correct word should be.
Today, I went to the ________ and bought some milk and eggs. I
knew it was going to rain, but I forgot to take my ________, and
ended up getting wet on the way ________.

gin
Decide which part of speech (adjective, noun, gerund, etc.)
needs to be used to fill each gap.
Explanation: Students would then be required to fill in the
blanks with words that would best complete the passage. Context

eer
Read the text again, trying to fill a gap as you come to it by
imagining what the correct answer should be.
in language and content terms is essential in most, if not all, cloze
tests. The first blank is preceded by "the"; therefore, a noun, an
adjective or an adverb must follow. However, a conjunction fol-

ing
Read the text another time, this time choose the correct
answer from the five answers given.
lows the blank; the sentence would not be grammatically correct
if anything other than a noun were in the blank. The words "milk

.ne
If you are unsure of any given answer, try reading the
sentence with each of the possibilities.
and eggs" are important for deciding which noun to put in the
blank; "market" is a possible answer; depending on the student,
however, the first blank could either be store, supermarket, shop
or market while umbrella or raincoat fit the second.


Try to eliminate the obvious false choices.
t
Always think about the overall meaning of the text (i.e.,
whether the text is negative, positive, etc.) to make sure that
your answer choice fits the context.
Example : I saw a man lay his jacket on a puddle for a woman
crossing the street. I thought that was very ______. • Trust your intuition. If you feel a word is right instinctively,
it probably is correct.

Downloaded From : www.EasyEngineering.net


Downloaded From : www.EasyEngineering.net

246  l  Cloze Test

Practice Exercise
LEVEL-I
DIRECTIONS (Qs. 1-210): In each of the following passages, PASSAGE -2 (16 - 25)
there are blanks each of which has been numbered. These numbers
Women have (16) made (17) in the corporate workplace but
are printed below the passage and against each, some words are
certainly not as much as they had (18) We have new laws, rules
suggested, one of which fits the blank appropriately. Find out the
and (19) relating to women in the workplace, but what we have not
appropriate words.
changed much is the male (20) Women have fallen short in their
PASSAGE -1 ( 1 - 15) goals because we (21) the potency of the male need to (22) their
power. We can abide (23) by the laws and rules we create in order
The most attractive and unique feature of crossword puzzle is
that it is a game one can play alone. There are several kinds of to (24) women an equal opportunity in the corporate workplace
crossword puzzles. One kind is the prize competition in which the and still not (25) the problems that afflicted and efentually capsized

ww
(1) who finds the correct answer gets a big prize (2) the answers
are very hard to find , since several (3) appear equally appropriate
: bad, mad or sad, for (4) in the clue sentence, “people are seldom
the women’s raft.
16. (a) perhaps (b) seldom
(c) optimistically (d) undoubtedly

w.E
popular” Such (5) attract people who are fond of gambling, because
by (6) a small entry fee, they can win big prizes. The (7) type of
crossword puzzle is one in which (8) is only one possible answer
to every clue. But (9) answer is elusive and calls for some detective
17. (a) progress
(c) attempts
18. (a) predisposed
(c) expected
(b) efforts
(d) decisions
(b) prescribed
(d) informed

asy
work (10) our part. The clue gives only hints about the word . A
(11) like’ mate changes to flesh for food I (12) elude you till you
19. (a) activities
(c) policies
(b) cases
(d) problems

En
realise that by changing the (13) of I mate’ you getr ‘ meat ‘ your
comprehension and your (14) knowledge are put to the test. The
effort to (15) such crosswords is an intellectual exercise.
20. (a) hatred (b) patterns
(c) achievements (d) behaviour

1. (a) persons
(c) actor
(b) person
(d) actors gin
21. (a ) retaliate
(c) risk
22. (a) maintain
(b) respect
(d) minimisse
(b) know
2.

3.
(a) although
(c) but
(a) words
(b) though
(d) as
(b) letters eer
(c) absolve
23. (a) partially
(d) evolve
(b) occasionally

4.
(c) alphabets
(a) illustration
(c) example
(d) letter
(b) assumption,
(d) examples
(c) them
24. (a) share
(c) assureing (d) excessively
(b) deny
(d) donate
5. (a) puzzles
(c) races
(b) competitions
(d) competition
25. (a) overcome
(c) judge (d) explore
.ne
(b) mentioned

6.

7.
(a) buying
(c) giving
(a) second
(c) two
(b) paying
(d) gives
(b) another
(d) first
PASSAGE -3 (26 - 35)
t
Architecture is a unique (26) of art and science that has (27) out
of man’s primary need for shelter. It is concerned with the design
8. (a) their (b) this and (28) of buildings in their sociological, technological and
(c) there (d) that environmental context. This field is not only (29) but also provides
9. (a) this (b) an the (30) of designing and building pleasing and (31) refined
(c) any (d) another struclures to serve various needs. (32) the fairly large number of
10. (a) in (b) on practising architects . the countrywide (33) in the initial earnings
(c) upon (d) over in the field are relatively (34) what you make thereafter will depend
11. (a) word (b) clue
enterely on your (35).
(c) puzzle (d) sports
26. (a) prooess (b) procedure
12. (a) did (b) shall
(c) will (d) had (c) blend (d) subject
13. (a) words (b) spelling 27. (a) arisen (b) earned
(c) lettering (d) place (c) contrived (d) fizzled
14. (a) overall (b) common sense 28. (a) appearance (b) repairing
(c) general (d) no need (c) decoration (d) construction
15. (a) solve (b) think of 29. (a) rewarding (b) vast
(c) dissolve (d) think (c) enocouraging (d) strenuous

Downloaded From : www.EasyEngineering.net


Downloaded From : www.EasyEngineering.net

Cloze Test  l 247

30. (a) readily (b) amenities PASSAGE -5 (51 - 60)


(c) infrastructure (d) facility Faced with an (51) number and variety of products on the market,
31. (a) readily (b) Principally managers are finding it more difficult to (52) demand and plan
(c) aesthetically (d) ideologically production and orders (53) As a result (54) forecasts are increasing
32. (a) Despite (b) Regarding and , along with them , the costs of those errors Many managers
(c) Assuming (d) Having today, (55) speed is the (56), have turned to one or another popular
33. (a) deterioration (b) spurt production scheduling system. But these tools tackle only part
(c) lethargy (d) slack of the problem (57) really needed is a way to (58) forecasts and
34. (a) exorbitant (b) negligible simultaneously redesign planning processes to (59) the impact
(c) unpredictable (d) modest of (60) forecasts.
35. (a) need (b) experience 51. (a) equal (b) exact
(c) expectation (d) appearance (c) eccentric (d) unprecedented
52. (a) register (b) accept
PASSAGE 4 (36 - 50) (c) ignore (d) predict
53. (a) spontaneously (b) positively
It was with pleasant surprise that Swaminathan stumbled into his
(c) accordingly (d) quickly
own set, which he had thought was not at school .except Rajam
54. (a) exorbitant (b) inadequate
and Mani all were there. Under the huge tama-rind tree they were

ww
playing some game Swaminathan joined them with a low, ecstatic
cry. The (36) disappointed him. They turned their (37) to him with
a faint (38) and returned to their game. What (39) Swaminathatn
(c) frequent (d) inaccurate
55. (a) not withstanding (b) believing
(c) visualizing (d) neglecting

w.E
most was that even the (40) Somu was grim. something seemed
to be (41) somewhere. Swaminathan assumed an easy (42) and
shouted, “ Boys, what about (43) place for me in the (44) ? “
56. (a) lacuna
(c) source
57. (a) what’s
(b) outcome
(d) answer
(b) that’s

asy
Nobody answered this Swaminathan paused and (45) that he was
waiting for a (46) in the game. “It is a (47) “There are (48) people
(c) one
58. (a) improve
(c) vitiate
(d) managers
(b) negate
(d) obtain

this, “ you said tail, didn’t you? “ asked Sankar .


36. (a) response (b) answer En
who can be very (49) as tails, “ said Samuel . The (50) laughed at 59. (a) popular
(c) minimize
(b) counter
(d) substantiate

(c) reply
37. (a) heads
(d) attention
(b) faces gin
60. (a) popular
(c) unpredicted
(b) erroneous
(d) absolute
PASSAGE -6 (61 - 70)
(c) profiles
38. (a) expression
(c) smile
(d) hands
(b) assessment
(d) around eer
Now-a-days, under the (61) system of education, however good
it may be, when a young man comes out of the university, there
39. (a) surprised
(c) astonished
(b) bewildered
(d) seen ing
seems to be this (62) in him that the higher the standard of living
rises, the less should a man work. Thus, mathematically, higher
40. (a) sympathetic
(c) genial
41. (a) improper
(b) hearty
(d) friendly
(b) wrong .ne
the standard of living, according to this misconceived notion, the
less the (63) ultimately, what? should be the highest standard of
living then? (64) work ! This leads to an unhealthy (65) among
(c) unreasonable (d) right
42. (a) accent
(c) tone
43. (a) a little
(b) modulation
(d) nothing
(b) a small
t
the workers . A typist who types over twenty letters a day asks
his (66) how many letters he had typed that day. The latter (67) “
fifteen”. The former thinks , “ Tomorrow I should type only fifteen
or even (68). This tendency is quite (69) and may ultimately lead
(c) some (d) few to (70) even one’s family life may be affected adversely due to
44. (a) match (b) game such tendency.
(c) sport (d) athleties 61. (a) extinct (b) proposed
45. (a) announced (b) reported (c) developed (d) modern
(c) proclaimed (d) shouted 62. (a) apprehension (b) realisation
46. (a) situation (b) position (c) anxiety (d) misconception
(c) place (d) going 63. (a) work (b) time
47. (a) fellow - feeling (b) disaster (c) salary (d) energy
(c) pity (d) condition 64. (a) Ample (b) No
48. (a) Suddenly (b) curtly (c) Minimum (d) Less
65. (a) ambition (b) jealousy
(c) briefly (d) accidently
(c) delay (d) dispute
49. (a) efficient (b) capable
66. (a) colleague (b) client
(c) skilful (d) learned
(c) boss (d) subordinate
50. (a) balance (b) remainder
67. (a) remembers (b) types
(c) rest (d) other
(c) suggests (d) replies

Downloaded From : www.EasyEngineering.net


Downloaded From : www.EasyEngineering.net

248  l  Cloze Test

68. (a) more (b) all 85. (a) rebellion (b) subversion
(c) less (d) some (c) mania (d) revolution
69. (a) unnatural (b) unfortunates 86. (a) always (b) never
(c) healthy (d) discouraging (c) greatly (d) hardly
70. (a) retardation (b) denial 87. (a) complaint (b) surmise
(c) evil (d) complexity (c) accusation (d) idea
88. (a) myth (b) story
PASSAGE - 7 (71 - 80) (c) fact (d) reality
In view of the (71) demand for personnel with commerce 89. (a) ruin (b) downfall
background, in the post-liberalisation period, cousses in (c) harm (d) defeat
commerce have (72) the attention of students and parents. There 90. (a) destroy (b) suffocate
is growing (73) for these courses not only in schools but also in (c) damage (d) injure
colleges. But the (74) of commerce education in schools leaves
PASSAGE - 9 (91 - 100)
(75) to be desired. Its popularity, importance and quality, to a
All writers are vain, selfish and lazy, and at the very (91) of
large extent, depends much on the teaching methodology being their motives there lies a mystery. Writing a book is a horrible,
(76) in schools. Of coourse , the (77) review and (78) of syllabi exhausting struggle, like a long (92) of some painful illness. One
also (79). But this aspect is (80) taken care fo suitably, by the

ww
concerned organisations
71. (a) diminishing
(c) increaseing
(b) exaggerated
(d) tremendous
would never undertake such a thing if one were not (93) by
some demon whom one can neither (94) nor understand. For
all one knows that demon. is simply the same (95) that makes a

w.E
72. (a) attracted
(c) distorted
73. (a) commotion
(b) ameliorated
(d) encouraged
(b) contempt
baby (96) for attention. And yet it is also true that one can write
nothing readable unless one constantly struggles to (97) one’s
own personality. Good prose is like a window pane. I cannot say
with certainty which of my motives are the strongest, but I know
(c) demand
74. (a) awareness
(d) dissatisfaction
(b) quality asy which of them deserve to be followed. And (98) through my
work, I see that it is (99) where I lacked a political purpose that I
(c) intricacy
75. (a) every thing
(c) nothing
(d) necessity
(b) much
(d) scope En wrote lifeless books and was (100) into purple passages, sentences
without meaning, decorative adjectives and humbug generally.
91. (a) mention (b) bottom
76. (a) abandoned (b) assimilated
(c) contemplated (d) practised gin (c) idea
92. (a) bout
(d) consideration
(b) feeling
77. (a) expert
(c) partial
78. (a) formulation
(b) general
(d) periodical
(b) discussions eer
(c) source
93. (a) driven but
(c) driven on
(d) condition
(b) driven for
(d) driven at
(c) perusal
79. (a) desires
(d) reduction
(b) encompasses
94. (a) help
(c) like
95. (a) feeling ing (b) resist
(d) conquer
(b) sense
(c) flourishes
80. (a) being
(c) never
(d) matter
(b) duly
(d) seldom
(c) desire
96. (a) squall
(d) instinct
(b) play .ne
PASSAGE - 8 (81 - 90)
Many parents greet their children’s teenage yerars with needless
dread. While teens (81) assault use with heavy-metal music, (82)
(c) desire
97. (a) overcome
(c) efface
98. (a) looking in
(d) look
(b) change
(d) deface
(b) looking for
t
(c) looking at (d) looking back
outlandish clothes and spend all (83) time with friends, such
99. (a) interestingly (b) seldom
behaviour (84) adds upto full-scale revolt. Teenage (85), according (c) constantly (d) invariably
to psychologist. Laurence Steinberg, has been (86) exaggerated. 100. (a) inspired (b) betrayed
Sociologist Sanford Dornbusch agrees. “The (87) that teenagers (c) moved (d) introduced
inevitably rebel is a (88) that has the potential for great family (89),”
says Dornbusch. He believes the notion can (90) communication PASSAGE - 10 (101 - 110)
during this critical time for parents to influence youngsters. Broadly speaking letters may be said to (101) into two classes:
81. (a) can (b) must the formal and (102) Formal letters (103) of official or business
(c) may (d) should matters and are (104) to an employer, officials of a department
82. (a) show (b) dress or institutions. Letters to the (105) of a newspaper also belong to
(c) put (d) flaunt this class. In fact all (106) using formal pattern is of this category.
83. (a) her (b) his The (107) has to be precisely stated. It must be (108) in style and
(c) their (d) our quite (109) No (110) element has any place in it.
84. (a) sporadically (b) always 101. (a) describe (b) fall
(c) infrequently (d) scarcely (c) escape (d) rise

Downloaded From : www.EasyEngineering.net


Downloaded From : www.EasyEngineering.net

Cloze Test  l 249

102. (a) affectionate (b) abusive PASSAGE - 12 (121 - 130)


(c) personal (d) flattering I want to avoid violence. Non-violence (121) the first article of my
103. (a) consist (b) apprise faith. It is also the (122) article of my creed. But I had to (123) my
(c) contain (d) comprise choice. I had either to submit (124) a system which I considered
104. (a) addressed (b) prayed (125) done irreparable harm to my country, (126) incur the risk
of the mad fury (127) my people bursting forth, when they (128)
(c) respected (d) typed
the truth from my lips. I (129) that my people have sometimes
105. (a) printer (b) publisher
(130) mad. I am deeply sorry for it.
(c) salesman (d) editor 121. (a) was (b) be
106. (a) writings (b) columns (c) is (d) being
(c) correspondence (d) letters 122. (a) last (b) common
107. (a) language (b) content (c) simple (d) unique
(c) criticism (d) objection 123. (a) make (b) select
108. (a) lucid (b) florid (c) prepare (d) do
(c) high (d) descriptive 124. (a) over (b) in
109. (a) creative (b) objective (c) against (d) to
125. (a) is (b) was
(c)
110. (a)
(c) ww
critical
unifying
malicious
(d)
(b)
(d)
subjective
divisive
personal
(c) had
126. (a) nor
(c) or
(d) has
(b) but
(d) and

w.E PASSAGE - 11 (111 - 120)


Dhritrashtra, the elder brother of the Pandu, was blind by birth.
When he grew young, he was married to Gandhari. She loved him
127. (a) with
(c) in
128. (a) listen
(b) by
(d) of
(b) get

asy
so (111) that she (112) up her own eyes with a cloth. A hundred
sons were bom to Dhritrashtra and Gandhari. They were called
(c) understood
129. (a) know
(c) knew
(d) understand
(b) knows
(d) known
Kauravas. Duryodhana was the (113) son of Dhritrashtra, who

En
was quite obstinate by nature. When Kunti (114) to Hastinapur
130. (a) going
(c) goes
(b) gone
(d) went
with her five sons, Duryodhana did not (115) it. He wanted that
the Pandavas should live in the forest with their mother, Kunti. In
fact he wanted to rule (116) Hastinapur without any interference gin PASSAGE - 13 (131 - 140)
Why is it that the CRP, BSF and the Army give a better account
in his affairs, although outwardly he made a (117) of happiness.
His mother Gandhari consoled Kunti but her son Duryodhana eer
of themselves than the state police? It is the structure of the state
police that needs to be looked (131), In a district, or city, where
there is a separate police force, a majority of the policemen are
did not like this at all. Pandavas were so good and (118) that every
one who saw them (119) them. The people of Hastinapur began
to give greater (120) to the Pandavas than to the Kauravas.
ing
(132) in police stations. The police station staff is often over-
extended and no (133) is available at that level. It is the state force

111. (a) closely


(c) keenly
(b) deeply
(d) excessively .ne
that has to be called (134) But it takes time to (135) forces. If the
law and order situation (136) simultaneously at many places, there
is manpower (137) Thus once rioting breaks (138), it takes time
112. (a) closed
(c) bound
113. (a) eldest
(b) folded
(d) blinded
(b) greatest
t
to bring it under control so that there is a time (139) between the
first incident and the appearance of the adequate force to (140)
the situation. The public hue and cry about the delayed arrival of
the police force is not unjustified.
(c) dearest (d) loveliest
131. (a) through (b) on
114. (a) refreated (b) retracted
(c) into (d) at
(c) receded (d) retumed 132. (a) detained (b) deployed
115. (a) concede (b) approve (c) deputed (d) employed
(c) understand (d) like 133. (a) reserve (b) force
116. (a) on (b) over (c) company (d) squad
(c) upon (d) from 134. (a) upon (b) up
117. (a) show (b) pretention (c) at (d) on
(c) face (d) pretext 135. (a) move (b) arrange
118. (a) feeble (b) attractive (c) mobilise (d) prepare
(c) innocent (d) noble 136. (a) worsens (b) proliferates
119. (a) glorified (b) praised (c) degenerates (d) aggravates
(c) respected (d) applauded 137. (a) default (b) shortage
120. (a) importance (b) praise (c) shortcoming (d) failure
(c) consideration (d) admiration

Downloaded From : www.EasyEngineering.net


Downloaded From : www.EasyEngineering.net

250  l  Cloze Test

138. (a) away (b) in 148. (a) sure (b) put


(c) off (d) out (c) shown (d) seen
139. (a) lag (b) interval (e) made
(c) delay (d) difference 149. (a) no (b) rich
140. (a) control (b) restrain (c) short (d) poor
(c) step (d) prevent (e) plenty
PASSAGE - 14 (141 - 150) 150. (a) but (b) not
[SBI Clerk 2012] (c) and (d) so
Emperor Akbar was fond of (141) tricky questions to Birbal. (e) only
One day he asked Birbal what he would (142) if he were given PASSAGE - 15 (151 - 160)
a choice between justice and a gold coin. “The gold coin,” said
[IBPS Clerk 2012]
Birbal. Akbar was (143) aback. He had known Birbal for many Once upon a time there lived a wise man by the (151) of Mamad.
years and he knew that Birbal was a just person. Then how could He never lied. All the people in the land, knew about him. The
he choose the gold coin. “You would prefer a gold coin to justice king heard about Mamad and (152) his subjects to bring him to
?” He asked, incredulously. “Yes,” said Birbal. The other courtiers the palace. He looked at the wise man and asked: “Mamad, is it

ww
were amazed by Birbal’s (144) of idiocy. For years they had been
trying to discredit Birbal in the emperor’s eyes but without success
and now the man had gone and (145) it himself ! They could not
true, that you have never lied ?” “It is true, your Majesty.” “And
you will never lie in your life” questioned the king. “I am sure of
that,” replied Mamad.

w.E
believe their good fortune. “I would have been dismayed if even
the lowliest of my servants had said this,” continued the emperor.
“But coming from you it’s shocking - and sad. I did not (146) you
(153) days passed and the king called Mamad once again.
There was a big crowd, the king was (154) to go hunting. The king
held his horse by the mane, his left foot was already on the stirrup.

be so shallow ?” asy
were so debased ! I never expected this from you. How could you He. (155) to Mamad. “Go to my summer palace and tell the queen
I will be with her for lunch. Tell her to (156) a big feast. You will
have lunch with me then.” Mamad bowed down and went to the

En
One (147) for what one does not have, Your Majesty !” said
Birbal, quietly. “You have (148) to it that in our country justice queen. Then the king laughed and said. “We won’t go hunting and
now Mamad will lie to the queen. Tomorrow we will laugh on his
is available to everybody. So as justice is already available to me
and as I’m always (149) of money I said I would choose the gold
coin.” The emperor laughed. He thought to himself, ‘I should have gin
behalf. “But the wise Mamad went to the palace and said, “maybe
you should prepare a big feast for lunch tomorrow, and maybe
known that Birbal would come up with a witty reply as always.’
He was so pleased with Birbal’s reply that he gave him (150) one
won’t.”
eer
you shouldn’t. Maybe the king will come by noon, and maybe he

“(157) me will he come, or will he not ?” – asked the queen.


but a thousand gold coins.
141. (a) showing
(c) naming
(b) asking
(d) finding
ing
“I do not know whether he put his right foot on the stirrup, or he
put his left foot on the ground after I left”. Everybody (158) for
the king. He came the next day and said to the queen. “The wise
(e) telling
142. (a) look (b) said .ne
Mamad, who never lies, (159) to you yesterday.” But the queen told
him Mamad’s exact words. And the king (160), that the wise man
(c) think
(e) find
143. (a) pushed
(d) choose

(b) fallen
151. (a) name
(c) call
(e) label
(b) sound
(d) identity
t
never lies, and says only that, which he see’s with his own eyes.

(c) pulled (d) sent 152. (a) demanded (b) send


(e) taken (c) ordered (d) request
144. (a) idea (b) display (e) sanctioned
(c) reply (d) place 153. (a) Several (b) Most
(c) Lots (d) Glorious
(e) showing
(e) Long
145. (a) speak (b) thought 154. (a) just (b) tired
(c) done (d) create (c) schedule (d) planned
(e) told (e) about
146. (a) felt (b) said 155. (a) wished (b) order
(c) know (d) accept (c) said (d) featured
(e) saw (e) send
156. (a) leave (b) prepare
147. (a) asks (b) chooses
(c) figure (d) show
(c) look (d) find (e) prove
(e) wish

Downloaded From : www.EasyEngineering.net


Downloaded From : www.EasyEngineering.net

Cloze Test  l 251

157. (a) Say (b) Rescue 166. (a) foster (b) seek
(c) Reveal (d) Tell (c) ask (d) built
(e) Understand (e) collect
158. (a) lied (b) left 167. (a) broke (b) fall
(c) heard (d) ran (c) degrade (d) destroyed
(e) waited (e) eliminate
159. (a) wrong (b) spoke 168. (a) even (b) very
(c) lied (d) did (c) elaborate (d) much
(e) deceive (e) enormous
160. (a) felt (b) understands 169. (a) cautious (b) careful
(c) realised (d) convinced (c) worried (d) afraid
(e) rejoiced (e) asking
170. (a) simplest (b) easier
PASSAGE - 16 (161 - 170)
(c) real (d) contrast
[IBPS Clerk 2013]
(e) wrong

ww
Trust is the basic tenet for all relationships, so building an
environment of trust is one of the (161) important things one
can do to (162) a positive work environment. It is a philosophy PASSAGE - 17 (171 - 180)

w.E
that must be demostrated in everything you and your staff does.
Trust is about doing what you say you are going to do and being
who you say you (163). It is about showing your staff in everything
[SSC CHSL 2012]
Parents tend to spoil their own children either by overindulgence

asy
you do that you are reliable, responsible and accountable and that
they can (164) on you for consistency. Also, letting them know
or by deprivation. Childhood should be the time for (171)
from primary selfishness to sharing, for learning to (172) with
deprivation and disappointment and learning to (173) failure,
you (165) the same from them. When your words and behaviour
En
are congruent you (166) trust. It will take sometime for your staff
since breaking a toy and forgetting a homework assignment are
(174) serious than breaking a marriage or forgetting to prepare
members to learn that you are a person of your word. If they see
that you are consistent you will build trust, but if they see that gin
for career advancement. But (175) deprivation, as is common in
(176) families, leaves many children (177) the stage of personal
your words don’t match your behaviour their trust in you will
be (167). The unfortunate thing about trust is that it takes a long
time to build, but is very fragile and breaks easily. Once broken,
eer
gratification. They lack resources for developing a sense of (178)
towards others and a wish to care for them. And children (179) be

it takes an (168) longer time to regain and it may never be fully


rebuilt. Therefore, it is of primary importance that you are (169) ing
poor to be underprivileged. The 180 are also deprived, cheated
out of learning how to face life.
171. (a) changing (b) turning
of all your words and behaviour and ensure that they are worthy
of your employees’ trust. Even while dealing with uncomfortable
(c) shifting
172. (a) cope up .ne
(d) removing
(b) adjust
situations, if you are honest and upfront it will make thing (170)
for everyone.
161. (a) most
(c) extreme
(b) single
(d) high


(c) fight
173. (a) overcome
(c) remove
(d) cope
(b) eliminate
(d) forget
t
174. (a) far better (b) far less
(e) crucial (c) far worse (d) far more
162. (a) believe (b) accept 175. (a) much (b) exhaustive
(c) create (d) lift (c) excessive (d) every
(e) add 176. (a) undernourished (b) uncoordinated
163. (a) is (b) will (c) uncooperative (d) underprivileged
(c) are (d) would 177. (a) stuck in (b) stuck on
(e) could (c) stuck about (d) stick in
164. (a) trust (b) rely 178. (a) gratitude (b) responsibility
(c) believe (d) expect (c) concern (d) friendship
(e) try 179. (a) would not (b) should not
165. (a) are (b) belong (c) need not (d) had not
(c) demands (d) expect 180. (a) over confident (d) over cautious
(e) harbour (c) over enthusiastic (d) over indulged

Downloaded From : www.EasyEngineering.net


Downloaded From : www.EasyEngineering.net

252  l  Cloze Test

PASSAGE - 18 (181 - 185) 194. (a) visit (b) visited


[SSC CHSL 2012] (c) visiting (d) visits
Auctions are public (181) of goods, conducted by an (182) 195. (a) for (b) on
auctioneer. He encourages buyers to (183) higher prices and (c) of (d) with
finally names the (184) bidder as the buyer of the goods. This 196. (a) is (b) are
is called ‘knocking down’ the goods, for when the bidding ends (c) were (d) was
the auctioneer (185) a small hammer on a table in front of him. 197. (a) are (b) was
181. (a) sale (b) marketing (c) is (d) were
(c) promotion (d) viewing 198. (a) will (b) would
182. (a) authoritative (b) allowed (c) could (d) can
(c) authentic (d) approved 199. (a) much (b) very
183. (a) bid (b) buy (c) too (d) more
(c) get (d) bargain 200. (a) either (b) because
184. (a) smartest (b) highest (c) or (d) and
(c) biggest (d) strongest

ww
185. (a) bangs
(c) smashes
(b) thrashes
(d) hits
PASSAGE - 20 (201- 210)
[SSC Sub Insp. 2012]
Growing urbanization and literacy, especially female

w.E PASSAGE - 19 (186 - 190)


[SSC CHSL 2013]
One fine morning a (186) man knocked at the doors of the home
education, will no doubt change the plight of women in
India. But the (201) will be slow and painful. Are we (202)
to wait until this happens while more and more girls meet

asy
for the aged run by nuns. He told the nun in charge that as he was
(187) to Delhi, he wanted to leave his servant-maid to the (188)
their (203) end? Not if we lay calim to an India that is
(204) and aspires to attain superpower status. There is only one
of the nuns. He assured the nun of sending some money every
month (189) she was an orphan. The nun (190) her saying that
En quick (205), which is not to wait for (206) to change, but to compel
the police force to (207) their attitude. The attitude of the police
she had got an excellen master.
186. (a) gentle
(c) nice
(b) bad
(d) good gin
force will become (208) if there is a big stick poised to hit them.
They will be more (209) to women victims and keener to catch
187. (a) moved
(c) changed
(b) shifted
(d) transferred eer
the (210) if they knew that they themselves will be punished or
sacked. In an insensible society, fear is the only key.
188. (a) care
(c) custody
189. (a) because
(b) home
(d) protection
(b) and
201. (a) method
(c) process ing (b) manner
(d) section

(c) though
190. (a) loved
(d) if
(b) praised
202. (a) keen
(c) made .ne
(b) prepared
(d) asked
(c) consoled (d) condoled
PASSAGE - 20 (191- 200)
[SSC CHSL, 2014]
203. (a) tragic
(c) systematic
204. (a) growing
(b) lethal
(d) methodical
(b) shining
t
(c) competing (d) plodding
Delhi (191) the capital of India. People from all parts of the
205. (a) decision (b) thought
country and the world (192) to Delhi. There (193) many historical
buildings. People (194) the Rajghat, Shantivan and Vijayghat. We (c) idea (d) solution
visited Delhi last year (195) our cousins. There (196) many other 206. (a) attitudes (b) nature
historical cities. Agra (197) one of them. We (198) visit Agra (c) behaviour (d) opinions
and Jaipur next time. The Red Fort of Delhi and the Hawa Mahal 207. (a) alter (b) reverse
of Jaipur were (199) famous for their Mughal (200) Rajasthani (c) change (d) cleanse
architecture respectively.
208. (a) democratic (b) lenient
191. (a) was (b) are
(c) liberal (d) easy
(c) is (d) were
192. (a) came (b) comes 209. (a) affectionate (b) considerate
(c) come (d) coming (c) attentive (d) concerned
193. (a) has (b) were 210. (a) culprits (b) rowdies
(c) is (d) are (c) hooligans (d) None of these

Downloaded From : www.EasyEngineering.net


Downloaded From : www.EasyEngineering.net

Cloze Test  l 253

LEVEL-II
DIRECTIONS (1-151): In each of the following passages, there are 15. (a) at least (b) minimum
blanks each of which has been numbered. These numbers are printed (c) for (d) approximately
below the passage and against each, some words are suggested, one of 16. (a) condition (b) climate
which fits the blank appropriately. Find out the appropriate words. (c) atmosphere (d) weather
17. (a) decision (b) work
PASSAGE - 1 (1 - 10)
(c) deed (d) action
Some people believe that marriages are made in heaven. One
18. (a) demoralization (b) doom
cannot say (1) this is true or not. However, in America now many
(c) disobedience (d) disaster
(2) who get married seek to evolve a fool proof (3) to ensure that
19. (a) draw (b) take
the marriage survives. However, the idea of married persons (4)
(c) lead (d) grow
the burden of domestic chores, instead of all the dirty work being
20. (a) labour (b) communal
dumped on the woman has (5) been propagated by the feminist
(c) religious (d) community
(6) and it has gradually, if grudgingly, been (7) This arrangement
may work on a temporary basis but taking (8) of a particular
PASSAGE - 3 (21 - 30)

ww
domestic work on a permanent basis will pose problems of
its own. For instance, taking out the rubbish may (9) make a
refreshing change from washing baby’s nappies. However, a
The fact is that democracy is the only (21) system that can
preserve the Indian entity, constitute its whole beyond the sum
of the various fragments that go into its making and (22) its

w.E
contract is a contract and must be (10) You may not like it but
this is what life is coming out to be in the most modern of the
countries in the world.
invaluable genius of stumbling along (23) a humane order. If
democracy in India is done (24) with, India itself will (25) in
the morass of the various particularities that (26) it. Universal
1.

2.
(a) whether
(c) however
(a) pairs
(b) still
(d) that
(b) men asy adult (27) and a periodic (28) of the popular mandate (29) by the
government is (30) for democracy and hence India’s integrity.
21. (a) viable (b) fundamental

3.
(c) couples
(a) method
(d) women
(b) project En (c) possible
22. (a) suggest
(d) acceptable
(b) promote

4.
(c) plan
(a) executing
(c) undertaking
(d) system
(b) dividing
(d) sharing
gin (c) encourage
23. (a) with
(c) to
(d) inspire
(b) towards
(d) for
5. (a) long
(c) always
(b) sometimes
(d) occasionally
24. (a) off
(c) awayeer (b) up
(d) out
6.

7.
(a) group
(c) section
(a) affirmed
(b) lobby
(d) faction
(b) approved
25. (a) vanish

26. (a) form ing


(c) surrender
(b) sink
(d) finish
(b) define

8.
(c) accepted
(a) control
(d) favoured
(b) care
(c) outline
27. (a) sensibility (b) education .ne
(d) constitute

9.
(c) charge
(a) occasionally

10. (a) respected


(d) duty
(b) often
(c) intermittently (d) rarely
(b) executed
(c) participation (d) franchise
28. (a) survey
(c) action
29. (a) obtained
(b) appearance
(d) renewal
(b) gained
t
(c) maintained (d) obeyed (c) secured (d) reserved
30. (a) dispensable (b) indispensable
PASSAGE - 2 (11 - 20)
(c) obvious (d) desirable
We will be crushed if civil (11) is started. It will be a (12) to the
clock. The technique of (13) against it has been (14) We may
PASSAGE - 4 (31 - 40)
have to wait (15) a year or more, for a suitable (16) to be created.
The feeling of insecurity among the people has increased even
Any hasty (17) may result in complete (18) Labour troubles will
though there is (31) use of the para-military and the military.
(19) to violence and there will be (20) discord.
The excesses (32) by the two forces, particularly by the first, do
11. (a) riot (b) disturbance
not (33) the type of anger it should because they are seen (34)
(c) war (d) disobedience
those who are trying to disintegrate the country. Even a (35) for
12. (a) damage (b) reversal
more say in the administration is shouted down. Human rights
(c) setback (d) rewinding
organisations and Civil liberty activists are considered a (36), if
13. (a) defence (b) protection
not unpatriotic. Even the (37) cases of cruelty are not pursued by
(c) fighting (d) guarding
the government (38) the inquiry should (39) the forces and (40)
14. (a) learnt (b) practised
the instrument on which it has come to depend.
(c) rehearsed (d) perfected

Downloaded From : www.EasyEngineering.net


Downloaded From : www.EasyEngineering.net

254  l  Cloze Test

31. (a) free (b) reduced universities of the country has been steadily (57). Consequently,
(c) little (d) liberal thousands of graduates and postgradutes come out of these
32. (a) cleared (b) committed institutions and stand in (58) waiting to get some (59) jobs (60)
(c) condoned (d) displayed in the country.
33. (a) resist (b) evoke
51. (a) to (b) with
(c) convey (d) suppress
34. (a) obeying (b) ignoring (c) from (d) by
(c) supporting (d) fighting 52. (a) factors (b) features
35. (a) demand (b) desire (c) reasons (d) methods
(c) prayer (d) plea 53. (a) process (b) system
36. (a) nuisance (b) help (c) procedures (d) policy
(c) threat (d) boon
54. (a) fall (b) rise
37. (a) minor (b) blatant
(c) new (d) concealed (c) down (d) decrease
38. (a) because (b) for 55. (a) evaluation (b) assessment
(c) till (d) lest (c) result (d) competence
39. (a) disperse (b) derail 56. (a) flood (b) growth

40. ww
(c) demoralise
(a) blunt
(c) change
(d) degrade
(b) raise
(d) lose
(c) increase
57. (a) falling
(d) slope
(b) diminishing

w.E PASSAGE - 5 (41 - 50)


Children are loved by all human beings. But (41) this world of
human (42) there ib no (43) nuisance than a boy (44) the age of
(c) rising
58. (a) lines
(c) rows
(d) growing
(b) long
(d) queues

asy
fourteen. He is neither ornamental (45) useful. It is impossible
to (46) affection on him as on a (47) boy and he is always getting
59. (a) managerial
(c) prestigious
(b) nice
(d) available

En
(48) the way. If he talks with a childish lisp he’is called a baby,
and if he answers in a grown up way he is called impertinent.
Then he is (49) the unattractive, growing age. He grows out (50)
60. (a) vacate
(c) created
(b) posted
(d) available

his clothes, with indecent haste. His voice begins to break and
loses its childish charm. gin PASSAGE - 7 (61 - 70)
Most of us are (61) of open conflict and avoid it if we can. And
there is a (62) to expressing and working through conflict. If
41. (a) of
(c) for
42. (a) affairs
(b) on
(d) in
(b) life eer
the working through involves harsh words and name-calling
people feel deeply hurt and relationships can be (63). Sometimes
(c) beings
43. (a) worst
(d) world
(b) bad ing
permanently. Some group members may be afraid that if they
really (64) their anger, they may go out of control and become
(c) worse
44. (a) at
(c) on
(d) better
(b) of
(d) in .ne
Violent, or they may do this. These fears can be very (65) and
based on experience. So why take the risk ? Why not avoid

45. (a) and


(c) nor
46. (a) show
(c) repose
(b) or
(d) so
(b) shower
(d) showering
t
conflict at all costs ? Conflict is rather like disease (66) is best,
that means attending to areas where (67) may occur before they
become an issue. If you have not (68) a conflict happening, your
next choice is to treat it early, 93. or hope that it goes away. If
47. (a) tiny (b) big it goes away over time fine. If it (69). then you will still have to
(c) small (d) little handle (treat) it and it is likely to be more (70).
48. (a) through (b) in 61. (a) scared (b) careful
(c) on (d) off (c) reckless (d) aware
49. (a) with (b) on 62. (a) challenge (b) measure
(c) of (d) at
(c) principle (d) risk
50. (a) of (b) from
(c) beyond (d) through 63. (a) established (b) maligned
(c) damaged (d) rebuilt
PASSAGE - 6 (51 - 60) 64. (a) sublimate (b) express
Delinking of jobs (51) degrees is one of the salient (52) of (c) minimize (d) regulate
our education (53). There has been a steep (54) in (55) in the 65. (a) baseless (b) imaginary
academic field in recent years. There is a (56) of degree holders (c) exaggerative (d) real
in the country. As a result, university degrees have lost their 66. (a) cure (b) diagnosis
value and charm while the number of students in colleges and (c) prescription (d) prevention

Downloaded From : www.EasyEngineering.net


Downloaded From : www.EasyEngineering.net

Cloze Test  l 255

67. (a) harmony (b) discomfiture PASSAGE - 10 (77 - 82)


(c) disagreement (d) consensus It was entirely a case of (77) romanticism. In West Bengal,
68. (a) expressed (b) ignored there were no big landowners and the bookish concept that
(c) induced (d) seen the killing of an (78) “jotedar” would inspire thousands of
69. (a) doesn’t (b) won’t oppressed peasants to swell the ranks of revolutionaries (79).
The police infiltrated their ranks, anti socials sneaked into the
(c) don’t (d) not
party which finally split into a dozen (80). Naxalites became
70. (a) credible (b) serious
targets for the police, other parties and their own brethren. A
(c) fraudulent (d) urgent
terrifying bloodbath took place and the trauma has not yet been
PASSAGE - 8 (71 - 73) forgotten. An (81) is taking place in Bihar today which cannot
have a happier ending. The basic reason why Naxalites failed was
Von Nuemann and Morgenstern assume a decision framework in
that in W. B. land reforms have been implemented. In Bihar, no
which all options are thoroughly considered, each option being
government so far has made any (82) attempt to introduce land
independent of the others, with a numerical value derived for the
reforms and eliminate benami holdings.
utility of each possible outcome (these outcomes reflecting, in 77. (a) misguided (b) excellent
turn, all possible combinations of choices). The decision is then

ww
made to maximize the expected utility.
(71), such a model reflects major simplifications of the way
(c) momentous
78. (a) famished
(c) inspiring
(d) perfect
(b) exploiting
(d) helping

w.E
decisions are made in the real world. Humans are not able to
process information as quickly and effectively as the model
assumes; they tend not to think (72) as easily as the model
79. (a) succeeded
(c) ventured
80. (a) commitments
(b) triumphed
(d) flopped
(b) factions

asy
calls for; they often deal with a particular option without really
assessing its (73), and when they do assess alternatives, they may
(c) objects
81. (a) tautology
(c) encore
(d) dimensions
(b) policy
(d) regulation
be extremely nebulous about their criteria of evaluation.
71. (a) Regrettably (b) Firstly
En 82. (a) foolish
(c) incompetent
(b) serious
(d) lightening
(c) Obviously
72. (a) Quantitatively
(d) Apparently
(b) Systematically
gin PASSAGE - 11 (83 - 86)
Let's (83) fruitfully then. We walk thinking of a hundred useless
(c) Scientifically
73. (a) Substitutes
(c) Utility
(d) Analytically
(b) Disadvantages
(d) Alternatives
eer
things, worrying over so many things, (84) at so many things
– posters, ads, vehicles, people. Let us walk with a purpose. By

PASSAGE - 9 (74 - 76) ing


doing this, not only will our physical health improve; but our
mind will become calm and (85), confusion will go, efficiency
will increase, we will experience peace and (86) spiritually. Those
In a large company, (74) people is about as common as using
a gun or a switch-blade to (75) an argument. As a result, most
.ne
who are otherwise extremely busy to think of God can walk
effectively. But you'll say : "We are busy; we can't walk leisurely".
managers have little or no experience of firing people, and they
find it emotionally traumatic; as a result, they often delay the act
interminably, much as an unhappy spouse will prolong a bad
marriage. And when the firing is done, it’s often done clumsily,
83. (a) talk
(c) live
84. (a) spitting
(b) walk
(d) love
(b) throwing
t
(c) looking (d) cursing
with far worse side effects than are necessary. 85. (a) cool (b) clammy
Do the world-class software organizations have a different (c) mature (d) querulous
way of firing people? No, but they do the deed swiftly, humanely, 86. (a) enjoy (b) rise
and professionally. (c) proliferate (d) progress
The key point here is to view the fired employee as a “failed
PASSAGE - 12 (87 - 101)
product” and to ask how the process (76) such a phenomenon in
the first place. [IBPS PO 2011]
As the country embarks on planning (87) the 12th Plan (2012-17)
74. (a) dismissing (b) punishing
period, a key question mark (88) hangs over the process is on the
(c) firing (d) admonishing energy requirements.
75. (a) resolve (b) thwart Growth is energy hungry, and the aspirations of growing at 9-10%
(c) defeat (d) close will (89) huge demands on the energy resources of the country.
76. (a) derived (b) engineered In this energy Jigsaw, renewable energy will (90) like never before
(c) produced (d) allowed in the 12th Plan and the (91). By the rule of the thumb, India will
(92) about 100 gigawatts 100,000 megawatts of capacity addition

Downloaded From : www.EasyEngineering.net


Downloaded From : www.EasyEngineering.net

256  l  Cloze Test

in the next five years. Encouraging trends on energy efficiency 99. (a) Against (b) For
and sustained (93) by some parts of the government the Bureau (c) With (d) Is
of Energy Efficiency in particular needs to be complimented for (e) Ever
this-have led to substantially lesser energy intensity of economic 100. (a) Capacity (b) Through
growth. However, even the tendered demand numbers are (c) Project (d) Versus
(94) to be below 80Gw. As against this need the coal supply (e) Against
from domestic sources is unlikely to support more than 25 Gw 101. (a) Generating (b) Lightening
equivalent capacity. Imported coal can add some more, but at a (c) Making (d) Touching
much (95) cost. Gas-based electricity generation is unlikely to (e) Saving
contribute anything substantial in view of the unprecedented gas PASSAGE - 13 (102 - 111)
supply challenges. Nuclear will be (96) in the foreseeable future. [IBPS PO 2012]
Between imported coal, gas, large hydro and nuclear, no more
than 15-20Gw equivalent can be (97) to be added in the five-year Greenhouse gases are only (102) of the story when it comes to
time block. (98) (99) this, capacity addition in the renewable global warming. Changes to one part of the climate system can
energy based power generation has touched about 3Gw a year. (103) additional changes to the way the planet absorbs or reflects
In the coming five years, the overall capacity addition in the energy. These secondary changes are (104) climate feedback’s,
electricity grid (100) renewable energy is likely to range between and they could more than double the amount of warming caused

ww
20Gw and 25Gw. Additionally, over and above the grid-based
capacity, off-grid electricity applications are reaching remote
places and (101) lives where grid-based electricity supply has
by carbon dioxide alone. The primary feedback are (105)to snow
and ice, water vapour, clouds, and the carbon cycle.
The most well (106) feedback comes from melting snow and ice

87. (a) Onw.E


miserably failed.

(c) For
(b) Against
(d) Onwards
in the Northern Hemisphere. Warming temperatures are already
(107) a growing percentage of Arctic sea ice, exposing dark
ocean water during the (108) sunlight of summer. Snow cover
(e) At
88. (a) Where asy
(b) That
on land is also (109) in many areas. In the (110) of snow and ice,
these areas go from having bright, sunlight-reflecting surfaces
that cool the planet to having dark, sunlight-absorbing surfaces
(c) Inside
(e) Who
89. (a) Replace
(d) Always

(b) Forward En that (111) more energy into the Earth system and cause more
warming.
(c) Subject
(e) Demand
(d) Place
gin
102. (a) whole
(c) material
(e) most
(b) part
(d) issue
90. (a) Light
(c) Publish
(e) Find
(b) Pass
(d) Feature
103. (a) raise
(c) refer eer (b) brings
(d) stop
91. (a) Earlier
(c) Publicity
(b) Likewise
(d) Next
(e) cause
104. (a) sensed
(c) nothing ing (b) called
(d) but
(e) After
92. (a) Consumed
(c) Require
(b) Waste
(d) Highlight
(e) term
105. (a) due
.ne
(b) results
(e) Generate
93. (a) Developmental
(c) Efforts
(e) Practices
(b) Structures
(d) Projections
(c) reason
(e) because
106. (a) done
(c) ruled
(d) those

(b) known
(d) bestowed
t
(e) said
94. (a) Likely (b) Sure 107. (a) mastering (b) sending
(c) Unsure (d) Unexpected (c) melting (d) calming
(e) Unlikely (e) increasing
95. (a) Expected (b) Nominal 108. (a) make-shift (b) ceasing
(c) Excelled (d) Higher (c) troubled (d) perpetual
(e) Lower (e) absent
96. (a) Marginal (b) Failure 109. (a) dwindling (b) manufactured
(c) Success (d) Dangerous (c) descending (d) generating
(e) Maximum (e) supplied
97. (a) Sure (b) Certain 110. (a) progress (b) reduced
(c) Linked (d) Remarked (c) existence (d) midst
(e) Expected (e) absence
98. (a) As (b) When 111. (a) repel (b) waft
(c) But (d) However (c) monitor (d) bring
(e) If (e) access

Downloaded From : www.EasyEngineering.net


Downloaded From : www.EasyEngineering.net

Cloze Test  l 257

PASSAGE - 14 (112 - 121) last five decades (128) developmental activities have taken
place. Slope failure and (129) of soil due to such (130) are (131)
[IBPS PO 2013] environmental hazards.
Traditional bank architecture is based on bank branches. These 122. (a) special (b) tremendous
branches ensure the physical (112) of a customer’s savings. A (c) extreme (d) developmental
customer may go there to deposit and withdraw money, (113) (e) supportive
loans and (114) in other financial transactions. In the past two 123. (a) information (b) feasibility
decades banking architecture has changed the Automated Teller
(c) debate (d) participation
Machine (ATM) has been a big (115) and credit and debit cards
have created new financial spaces. (116) the bank branch has (e) attitude
remained the bedrock of the banking system after all a person 124. (a) protecting (b) covering
needs a bank account in a branch before he can operate a debit (c) towards (d) surrounding
or ATM card. This may be about to change as technocrats now (e) of
(117) cell phones as the new architecture of virtual banks. This 125. (a) concerned (b) biased
has the potential to make branches (118). Cell phone banking (c) casual (d) desperate
looks especially relevant for India, since it can penetrate the (e) confused
countryside cheaply and (119). The world over cell phones

ww
are spreaing at a (120) rate and in India alone new cell phone
connection are growing at the rate of six million a month a rate
of customer (121) that no bank can dream of.
126. (a) grey
(c) dry
(e) uneven
(b) deteriorating
(d) rustic

w.E
112. (a) Knowledge
(c) presence
(e) guarantee
(b) security
(d) confidentiality
127. (a) continued
(c) multiple
(e) insignificant
(b) drastic
(d) unjustified

113. (a) negotiate


(c) credit
(e) sanction
asy
(b) advance
(d) disburse
128. (a) unique
(c) umpteen
(b) disastrous
(d) extensive
114. (a) pursue
(c) operate
(b) interact
(d) enable En (e) amazing
129. (a) pollution (b) cultivation
(e) engage
115. (a) drawback
(c) consequence
(b) hurdle
(d) luxury
gin (c) deployment
(e) erosion
(d) demand

(e) innovation
116. (a) Despite (b) Although eer
130. (a) urgency
(c) activities
(b) misconception
(d) action
(c) Even
(e) Until
117. (a) View
(d) Yet

(b) realize
(e) misdeed
131. (a) major
ing
(c) effective
(b) demaging
(d) dastinctive
(c) Display
(e) assess
(d) engineer (e) concerned
.ne
118. (a) essential
(c) extant
(e) expired
119. (a) moderately
(b) obsolete
(d) retreat

(b) occasionally
PASSAGE - 16 (132 - 141)

t
[SBI PO 2012]
There is a considerable amount of research about the factors
that make a company innovate. So is it possible to create an
(c) compulsorily (d) indiscriminately
environment (132) to innovation? This is a particularly pertinent
(e) effectively
(133) for India today. Massive problems in health, education etc.
120. (a) phenomenal (b) gradual
(c) proportionate (d) competitive (134) be solved using a conventional approach but (135) creative
(e) projected and innovative solutions that can ensure radical change and
121. (a) discount (b) base (136). There are several factors in India’s (137). Few countries
(c) expansion (d) satisfaction have the rich diversity that India or its large, young population
(e) relationship (138). While these (139) innovation policy interventions certain
additional steps also required. These include (140) investment in
PASSAGE - 15 (122 - 131) research and development (141) the government and the private
[SBI PO 2011] sector, easy transfer of technology form the academic world
There has been a (122) change in people’s (123) towards and etc. To fulfill its promise of being prosperous and to be at the
awareness (124) the environment. People are more (125) about forefront, India must be innovative.
the (126) condition of their environment. Land in all societies 132. (a) stimuli (b) conducive
is subjected to (127) usage, viz. crop production, forest cover, (c) incentive (d) facilitated
grassland, urbanisation and industrialisation etc. During the (e) impetus

Downloaded From : www.EasyEngineering.net


Downloaded From : www.EasyEngineering.net

258  l  Cloze Test

133. (a) objective (b) controversy them from accessing the most basic human rights and needs.
(c) doubt (d) question The outlook is bleak.
(e) inference Women make up 70% of the world’s working hours and (148)
134. (a) cannot (b) possibly only 10% of the world’s income and half of what men earn. This
(c) should (d) never leads to greater poverty, slower economic growth and a (149)
(e) must standard of living. In developing countries, millions of women
135. (a) necessary (b) apply also die each year as a result of gender-based (150). This deep-
(c) need (d) consider rooted gender discrimination creates a bleak outlook for women
(e) requires in developing countries. For millions of girls living in poverty,
136. (a) quantity (b) advantages it is often those closest to them who work against the child’s
(c) increase (d) chaos interests and their immediate environment is often (151) and
(e) growth sometimes, down-right harmful. Parents arrange marriages
137. (a) challenges (b) praises when you are a child. Neighbours say, if you are a girl, you must
(c) favour (d) leverage limit your activities to your home. Friends say, it is OK not to go
(e) esteem to school.
138. (a) blessed (b) enjoys 142. (a) made (b) make
(c) endows (d) prevails (c) look (d) has made

ww
(e) occurs
139. (a) aid (b) jeopardise
(e) complete
143. (a) equality (b) affection

(e) cater
w.E
(c) promotes

140. (a) acute


(c) restricting
(d) endure

(b) utilising
(d) inspiring
(c) inequality
(e) justice
144. (a) vulnerable
(c) vulnerability
(d) support

(b) weakness
(d) specific
(e) increased
141. (a) both asy
(b) besides
(e) weakest
145. (a) tackle (b) tackling
(c) combining
(e) also
(d) participating

En (c) fight
(e) fought
146. (a) increase
(d) tackled

(b) support
PASSAGE - 17 (142 - 151)
[SBI PO 2014] gin (c) weak
(e) influencing
(d) reduce

Women (142) up half the world’s population and yet represent a


staggering 70% of the world’s poor. We live in a world in which
women living in poverty face gross (143) and injustice from eer
147. (a) obstacles
(c) pleasantries
(e) encouragements
(b) make
(d) pebbles

birth to death. From poor education to poor nutrition to (144)


and low pay employment, the sequence of discrimination that a
148. (a) earns
(c) spend ing (b) carried
(d) earn
woman may suffer during her entire life is unacceptable but all
too common.
Millennium Development Goal 3 is to promote gender equality
(e) spends
149. (a) slow
(c) low .ne
(b) slower
(d) lowest
and empower women. This MDG is critical for (145) poverty
and improving prospects for women. But how can women break
gender based stereotypes to minimise discrimination and (146)
gender based violence when they are trapped in societies with
(e) lower
150. (a) violence
(c) virulent
(e) violation
(b) violent
(d) corruption
t
socio-cultural practices which routinely discriminate them from 151. (a) functional (b) natural
having equal opportunities in education, health and livelihood? (c) artificial (d) dysfunctional
These women are invisible and the (147) in their way prevent (e) disfunctional

Downloaded From : www.EasyEngineering.net


Downloaded From : www.EasyEngineering.net

Cloze Test  l 259

Hints & Solutions


LEVEL- I For Questions 71-73:
Let’s first get some background knowledge into what has been
1. (b) 2. (c) 3. (a) 4. (c) 5. (b) 6. (b) described in the passage. Von Nuemann and Morgenstern
7. (a) 8. (c) 9. (b) 10. (b) 11. (c) 12. (b) were amongst the most famous mathematicians and
13. (b) 14. (c) 15. (a) 16. (d) 17. (a) 18. (c) economists; they together formed the mathematical field
19. (c) 20. (d) 21. (b ) 22. (a) 23. (d) 24. (c) of ‘game theory’ and how it could be applied to economics.
25. (a) 26. (c) 27. (a) 28. (d) 29. (b) 30. (c) The given paragraph talks about the theory they developed:
31. (c) 32. (a) 33. (b) 34. (d) 35. (b) 36. (a) they formulated a theory which one can use to make a
37. (a) 38. (c) 39. (a) 40. (c) 41. (b) 42. (c) choice among the different available options. The paragraph
43. (a) 44. (d) 45. (c) 46. (c) 47. (c) 48. (c) mentions a term, ‘expected utility’ which is basically a way to
49. (b) 50. (c) 51. (d) 52. (d) 53. (c) 54. (d) balance risk versus reward with the help of a mathematical
55. (b) 56. (d) 57. (a) 58. (a) 59. (c) 60. (b) function. Utility can be described as a measure of one’s
61. (d) 62. (d) 63. (a) 64. (b) 65. (a) 66. (c) preferences amongst the options or choices available.
67. (d) 68. (c) 69. (d) 70. (a) 71. (c) 72. (a) We just have to grasp the context of passage in order to
73. (c) 74. (b) 75. (b) 76. (d) 77. (d) 78. (c) pick the right options. So, they are basically talking about a
79. (d)
ww
80. (d) 81. (c) 82. (d) 83. (c) 84. (b)
85. (a) 86. (c) 87. (d) 88. (d) 89. (a) 90. (b)
91. (b) 92. (a) 93. (d) 94. (b) 95. (c) 96. (a)
seemingly complex and thorough mathematical model which
is aimed at helping someone make a better choice.
71. (d) Option (d) is correct, as apparently is an adverb which

w.E
97. (c) 98. (d) 99. (d) 100. (c) 101. (b) 102. (c)
103. (a) 104. (a) 105. (d) 106. (c) 107. (a) 108. (a)
109. (b) 110. (d) 111. (b) 112. (c) 113. (a) 114. (d)
115. (d) 116. (b) 117. (a) 118. (d) 119. (b) 120. (a)
means ‘as far as one can see or knows’. This adverb is trying
to indicate that the rest of the sentence followed by this word
is an information which one can see or know after analyzing
this decision framework. So, this word makes proper sense.

asy
121. (c) 122. (a) 123. (a) 124. (d) 125. (c) 126. (c)
127. (d) 128. (b) 129. (a) 130. (b) 131. (b) 132. (d)
133. (e) 134. (b) 135. (c) 136. (c) 137. (a) 138. (d)
Option (a) is incorrect, because regrettably means
unfortunately or unluckily, so this is used when telling
something which is not good or not desired, but as we
139. (c) 140. (b) 141. (c) 142. (d) 143. (b) 144. (a)
145. (c) 146. (a) 147. (b) 148. (d) 149. (a) 150. (a)
151. (a) 152. (c) 153. (a) 154. (e) 155. (c) 156. (b) En can see, the phrase which follows it is only adding more
information about what has been talked about previously in
157. (d) 158. (e) 159. (c) 160. (c) 161. (a) 162. (c)
163. (c) 164. (b) 165. (d) 166. (d) 167. (d) 168. (a)
169. (b) 170. (b) 171. (c) 172. (d) 173. (a) 174. (b) gin the passage. This is being done dispassionately. It does not tell
anything that is unfortunate or undesired.
Option (b) is wrong, because obviously is used for something
175. (c) 176. (d) 177. (a) 178. (b) 179. (c) 180. (d)
181. (a) 182. (d) 183. (a) 184. (b) 185. (a) 186. (a)
187. (d) 188. (c) 189. (a) 190. (c) eer
that is very ‘visibly clear and unmistakably true’. But for
someone who is just reading about this new theory or system
for the first time or someone who does not know much
191. (c) Delhi is the capital of India. (The sentence is in present
tense).
192. (b) People from all parts of the country comes to Delhi. ing
about it, what this sentence goes onto say cannot be deemed
‘obvious’. It will not be appropriate to use that word.
72. (d) Option (d) is correct, because analytically means ‘by virtue
193. (d) There are many historical buildings.
194. (a) People visit the Rajghat, Shantivan and Vijaygath.
.ne
of analysis’. It is used when we mention or tell something
that has been derived or deduced after some analysis has
been done; hence, what we mention then represents some
195. (d) We visited Delhi last year with our cousins.
196. (b) There are many other historical cities.
197. (c) Agra is one of them.
198. (a) We will visit Agra and Jaipur next time.
199. (b) The Red Fort of Delhi and Hawa Mahal of Jaipur were
t
logical reasoning. The statement which has this word missing
states that model is designed, because humans do not think
‘quickly and ‘effectively’, and the clause followed by the blank
mentions that human often do not assess options, which
means that they do not think analytically.
very famous. Option (a) is wrong, because quantitatively means in ‘a
200. (d) for their Mughal and Rajasthani architecture quantitative matter’ which is inappropriate here. Option (b)
respectively. is wrong, because systematically means ‘in a systematic or
201. (c) 202. (b) 203. (a) 204. (b) 205. (d) planned manner’. The context in the sentence is of assessing
206. (a) 207. (c) 208. (c) 209. (b) 210. (a) and analyzing, not planning. Option (c) is wrong, because
scientifically means ‘in a scientific manner’ or, thinking in the
LEVEL- II way a scientist would. Although, scientific thinking involves
analysis, yet this is a different form of thinking as it involves
1. (a) 2. (c) 3. (d) 4. (d) 5. (a) 6. (b) 7. ( c ) taking the principle of science into consideration, but the
8. (c) 9. (a) 10. (a) 11. (d) 12. (c) 13. (c) 14. ( a ) given context is not pointing in that direction.
15. (a) 16. (c) 17. (d) 18. (d) 19. (c) 20. (b) 21. ( a ) 73. (a) Option (a) is right, because substitutes or alternatives
22. (b) 23. (b) 24. (c) 25. (b) 26. (d) 27. (d) 28. ( a ) mean ‘options or the different choices available to choose
29. (b) 30. (b) 31. (d) 32. (b) 33. (b) 34. (d) 35. ( a ) from’. It is a fairly easy question because the words which
36. (a) 37. (b) 38. (d) 39. (c) 40. (a) 41. (d) 42. ( c ) follow are giving us a big hint. It states ‘… without really
43. (c) 44. (b) 45. (c) 46. (b) 47. (d) 48. (b) 49. ( d ) assessing its [3], and when they do assess alternatives’, so what
50. (a) 51. (c) 52. (b) 53. (b) 54. (a) 55. (d) 56. (a) humans are not doing, is assessing the alternatives. Hence,
57. (c) 58. (d) 59. (c) 60. (d) 61. (a) 62. (d) 63. (c) ‘substitutes’ is the right option. ‘Alternatives’ is also correct,
64. (b) 65. (d) 66. (d) 67. (b) 68. (b) 69. (a) 70. (b) but since it is used in the next line, we can avoid it.

Downloaded From : www.EasyEngineering.net


Downloaded From : www.EasyEngineering.net

260  l  Cloze Test

For Questions 74-76: correspond to the blank from any consideration, hence are
The passage talks about firing and questions the way in which ruled out.
it is done in large companies. The author thinks that firing is 85. (a) ‘Cool’ as option (a) goes well with ‘calm’ contextually.
done ‘clumsily’, in a way it should not be done. The word ‘clammy’ in option (b) means ‘unpleasantly damp
74. (c) The given blank is ‘In a large company, [81] people is …’ and sticky’ hence does not fit in while in option (c) the word
so we have to fill the word which refers to something done ‘mature’ again does not fit in. The word ‘querulous’ in option
to people. Option (c) is right because, as already stated, (d) meaning ‘complaining, peevish’ also does not suit the
the passage talks about the case of firing people in large requirement, hence can be ignored. Thus the options (b), (c)
companies. and (d) are dropped due to their being inappropriate.
75. (a) We have to fill the word in ‘firing people is about as 86. (d) The sentence reveals that when the confusion will go,
common as using a gun or a switch-blade to [82] an efficiency will increase and we will experience peace and,
argument’. So, in this argument, we can clearly see that it is ‘progress’ spiritually. The word ‘progress’ fits in admirably as a
referring to finishing off an argument or ending it, because if sequential build-up. The other options (a), (b) and (c) do not
you use a gun to deal with the argument, then it means you fit and hence are ignored.
are just ending or settling the case ‘permanently’. 87. (c) We plan for future as sentence is about planning for
76. (d) The statement in which we have to fill the blank future.
suggests to first consider that employee who is to be fired 88. (d)
as a ‘fail product’ and think how and why that happened. 89. (d) A word like ‘place’ or ‘put’ can be used. Other words
The key point is to see what makes that employee to would give different direction to the passage.

ww
perform badly. The causes should be looked at, to know
how the process enabled such a situation to occur. The
word ‘allowed’ is the most appropriate. Option (d) is
90. (d) Feature is best among the options for something to
appear in a plan.
91. (d) Next suits best.

w.E
correct.
77. (a) Having read the entire passage and then concentrating
on the first sentence, ‘misguided’ under option (a) comes out
the most suited one. Words like ‘excellent’, ‘momentous’ and
92. (c) Need of power for India is being mentioned so ‘require’ is
proper word for denoting it.
93. (c) An effort for fulfilling the energy need of India is being
discussed in this part of passage. Projections /development or

hence ruled out.


asy
'perfect' belonging to options (b), (c) and (d) are misfits and

78. (b) The second sentence in the passage mentions about the
other options are not discussed here.
94. (e) It is an unexpected line so either option (d) or (e) is to be
used. But the structure and place of blank space suits more to

En
‘jotedar’. The adjective which well defines him as ‘exploiting’ is
available as option (b). The other words under options (a), (c)
and (d) do not respond to the context and hence are ruled out
option (e).
95. (d) Contrasting word with the supply and cost should be
used. Supply is there but at higher cost. Supply is low but cost
as inappropriate.
79. (d) The passage brings out that the killing of the ‘jotedar’ was
not successful in boosting the revolutionaries, hence the word
gin is low. So first case is relevant here as per the context of the
passage.
96. (a) Limited nuclear power/energy is referred here up to a
‘flopped’ under option (d) becomes the correct answer. Here
the words ‘succeeded’ in option (a), ‘triumphed’ in option (b)
and ‘ventured’ in option (c) do not find context to the build
eer
particular time.
97. (e) Figures are not certain and only can be expected.
98. (a) A combination of words with question no. 33 has to be
up and hence can be ignored.
80. (b) The clue to the answer lies in ‘split into a dozen’. Following
this clue, option (b) carrying the word ‘factions’ fits in
found out.
ing
99. (a) Expectation line was crossed so for blank in 32. And 33.
As, against is to be used.
superbly. The other options (a), (c) and (d) having words like
‘commitments’, ‘objects’ and ‘dimensions’ are inappropriate for
100. (b)
.ne
‘Through’ gives a right usage for the context.
Other options are not in line with context and also not
usage over here, hence ruled out.
81. (c) The word ‘encore’ under option (c) means ‘the repetition
of an item or event’. This simplifies the process of ascertaining
option (c) as the correct answer as it fills up the blank
correctly. ‘Tautology’ under option (a) means saying the same
grammatically correct.

t
101. (d) With lives ‘touching’ is proper word to use. Making,
generating are incorrect with ‘lives’. Saving is illogical and
out of context. Lightening lives is a too strong word for the
context.
thing twice in different words, does not at all correspond as 102. (b) 103. (e) 104. (b) 105. (a) 106. (b)
a correct usage for the blank, hence incorrect. ‘Policy’ and 107. (c) 108. (d) 109. (a) 110. (e) 111. (d)
‘Regulation’ under options (b) and (d) are again inappropriate 112. (b) 113. (d) 114. (c) 115. (e) 116. (d)
fillers and hence can be sidelined. 117. (a) 118. (b) 119. (e) 120. (a) 121. (c)
82. (b) The last sentence in the passage reflects about the 122. (b) 123. (e) 124. (e) 125. (a) 126. (b)
government efforts of reforms. The word in option (b) 127. (c) 128. (d) 129. (e) 130. (c) 131. (a)
is ‘serious’ and which fits in perfectly in the blanks as it 132. (b) conducive ; the other words do not fit in correctly
ascertains the degree of application. Option (a) with ‘foolish’, 133. (d) question
option (c) with ‘incompetent’ and option (d) with ‘lightening’ 134. (a) cannot fits correctly
makes one conclude that these words are incorrect for usage, 135. (c) need
hence sidelined. 136. (e) growth
83. (b) This is the correct option only when the sentence is 137. (c) favour
viewed keeping in link the succeeding sentence which says 138. (b) enjoys
‘we walk thinking …’, while options (a), (c) and (d) do not fit, 139. (a) aid is used correctly meaning facilitate
hence are ruled out. 140. (e) increased
84. (c) The sentence deals with ‘posters, ads, vehicles, people’ and 141. (a) both
these are objects to be looked at and hence ‘looking’ under 142. (b) 143. (c) 144. (a) 145. (b) 146. (d)
option (c) is the correct usage. Options (a), (b) and (d) do not 147. (a) 148. (d) 149. (e) 150. (a) 151 (d)

Downloaded From : www.EasyEngineering.net


Downloaded From : www.EasyEngineering.net

16
PTER
CHA

Parajumbles

T ww
hese questions are asked in a lot of competitive exams, so it is important to master them. Another reason for spending time
practising these questions is that your skills or you abilities to tackle them can be greatly improved if you solve a bunch
of these questions. So, they are surely the ones which will get you great rewards for the time you spend practising them.
Personally, we feel that these are one of the high scoring question types.

w.E
An apparent problem with these questions is that there are no set rules, whereas for example, in sentence correction, we know that
your grammar skills will be put to the test. But, if you are an avid reader, this section should be quite easy for you as it just tests your

asy
reading and comprehension skills. There are still ways to tackle and crack these questions even if you have not read a lot of books.
But, first of all, we need to understand how these questions are framed for the students.
You may infer from the name that your task is to find a proper and logical arrangement from a paragraph which is jumbled, i.e.

En
the sentences of a paragraph are given in a random order and your task is to order these sentences in such a way that you end up with
a logical paragraph. Basically, a paragraph is jumbled and you have to arrange it in its original order, i.e. the order in which it was, or

gin
should be. So, it is like you have to tie the sentences in such a way that the paragraph when formed, becomes logically correct. It is
not necessary that all the sentences given in the question belong to the same paragraph. A sentence from some other paragraph may
exist to confuse the examinee.
eer
TERMINOLOGY
(1) Jumbled ing
The word ‘jumbled’ is used to describe something which is in disorder or not in proper order.
.ne
A paragraph as we know, consists of a set of sentences, and in the context of these questions, this set is jumbled and mixed, which
gives us a jumbled paragraph that may or may not make sense in the given order.
(2) Logical
t
The word ‘logical’ comes from the Greek root logos which means reason, idea or word. ‘Logical’ is used to describe something which
comes from clear reasoning. Therefore, calling something ‘logical’, means that it is based on sound and clear ideas. In these questions,
we have to create a paragraph which is logical, i.e. we have to make sure that the links we create between the given sentences are apt
and lead to a meaningful paragraph.
(3) Coherent
‘Coherent’ is used to describe something which is clear or easily understandable, and it is written or presented in a consistent manner.
While finding the right order of the given sentences, we should make sure that we arrange the information in a consistent manner,
i.e. it is not ideal to jump from one subject to another, and then come back to the old subject again (of course, that does not mean a
concluding sentence is wrong, because it refers to something mentioned in the beginning.) For example – we mention reasons why
air pollution is bad, and then the next sentence mentions where air pollution is most witnessed and then our next sentence again
mentions some other reason of why air pollution is bad.
The idea is that the coherent part of the bargain stays with us. Simply put, there has to be a smooth flow of ideas in the paragraph.

Downloaded From : www.EasyEngineering.net


Downloaded From : www.EasyEngineering.net

262  l Parajumbles

(4) Puzzle
A puzzle basically refers to a game which tests your ingenuity. A puzzle is a baffling problem, the solution for which is not easy to
find. We often say that the given set of sentences in these questions is puzzling, because there could be several answers to the question
and all may seem right.

TYPES OF PARA JUMBLE QUESTIONS


This type of question is presented only in two forms, and there are no major differences in how to solve, or tackle these types. Two
types in which these questions can be framed are:
(i) A set of four or five sentences will be numbered as 1, 2, 3, 4, 5 or labelled as a, b, c, d, e. This, of course might not be the correct
order. This is why it is known as the paragraph jumble question. After these sentences, four options will be given to you and each
would represent an order of sentences. Only one out of these four options would give the most appropriate and logical order to
form a meaningful and coherent paragraph.
(ii) A set of six (or more) sentences is given, in such a way that the first and the last statement are fixed, i.e. they actually represent
the first and last statement of a coherent and meaningful paragraph. The rest of the four statements, however, are jumbled just
like the previous type. So, the only difference from the previous type is that we know the initial and final statement of paragraph.

ww
The four sentences in-between can be dealt in the same way the ones of previous type.

HOW TO SOLVE

w.E
It would be wrong to say that there is a particular way for solving the jumbled paragraph questions, because we are dealing with
paragraphs arranged in a haphazard fashion. You can rely on tips and tricks to reach the answer to these questions, but the best thing

(i) Read all the sentences quickly.


asy
would be to identify links between sentences. Let’s discuss these in detail:

(ii) See if you have understood what the paragraph is about, and what it means. This is important because if you are able to understand

En
what the sentences are about, then picking the answer is not a risky job.
(iii) If you could not get what the passage is about, then at least try creating Mandatory Pairs (sentences that will definitely go
together).
gin
(iv) If you don’t have an answer from step (ii), use step (iii) to eliminate options.

TIPS
eer
A good thing about these questions is that there are quite a few tips and tricks to help you find the answers. These have been developed

ing
after solving a lot of questions, so you don’t have to. Just read these and take your examinations.
Just kidding. Of course, you have to read and solve a lot of questions before you form your own styles. But these will give you an
upper hand in quickly forming your strategies.
.ne
Some of the best ways to deal with these questions are detailed below. (A word of caution though, these are all just rules of thumb
and not definitive guidelines. Most often, they will work, but sometimes there will be exceptions. To be prepared for these situations,
you need practice.)
(i) Find the opening sentence, as that could quickly rule out some of the options. Finding the opening sentence will aid you in
t
starting to form the desired paragraph, but if your aim is to just reduce the options, then you can also try to pick the closing or
the final statement from the given ones. With these two, you could rule out a significant number of options.
(ii) If the given options are closely related, then try to find out the mandatory pairs in the sentences. A mandatory pair could be
understood this ways: Suppose after reading the different sentences, you conclude that say statement ‘B’ would definitely come
after statement ‘D’, so DB would become a mandatory pair. A couple of mandatory pairs can help you eliminate almost all the
incorrect options.
( iii) Look out for transition words – Transition words are those which make the transition from one idea to another smooth, i.e.
they aid in logically connecting the sentences. A transition word can give you a hint, in terms if that particular sentence comes
before, or after another sentence. Some of the transition words are given below:
also, again, as well as, besides, furthermore, in addition, likewise, moreover, similarly, consequently, hence, otherwise, subsequently,
therefore, thus, as a rule, generally, for instance, for example, for one thing, above all, aside from, barring, besides, in other words,
in short, instead, likewise, on one hand, on the other hand, rather, similarly, yet, but, however, still, nevertheless, first of all, to begin
with, at the same time, for now, for the time being, in time, later on, meanwhile, next, then, soon, in the meantime, later, while,
earlier, simultaneously, afterward, in conclusion, with this in mind, after all, all in all, to sum-up.

Downloaded From : www.EasyEngineering.net


Downloaded From : www.EasyEngineering.net

Parajumbles  l 263

(v) Watch out for personal pronouns – he, she, you, they, it etc. If a sentence contains a personal pronoun, then it means that it is
referring to some place, person or thing which has already been mentioned. hence, the sentence that introduces this particular
noun would come before the sentence that uses the personal pronoun. Sometimes there can be instances where the personal
pronoun precedes the noun in a paragraph. But this rarely happens.
( vi) Demonstrative pronouns are words like this, that, etc. If you see them in a sentence, it would mean that they are referring to
something which has been already mentioned. Hence, this also can help us in forming mandatory pairs.
( vii) Acronyms: If an acronym has been used in one sentence, there is a high chance that its full form would be used in some or
the other sentence. If this is the case, you could say that the sentence mentioning the full form would come before the one that
contains the acronym.
( viii) Chronological sequence: See if the sentences refer to any time period, or some specific years, because a properly organized
paragraph will mention these different times in a sequential manner.
(ix) Sometimes a sentence will just be explaining a concept, while another one would be giving an example. The relationship between
these would help you create another mandatory pair.
Most often, the example would follow the concept. The simple rule of the SPECIFIC following the GENERAL in common

ww
speech.
Sometimes, there could be a reversal though, where some author would go on to give an example first and then move on up from

w.E
that to cite the concept behind it.
(x) There is one easy and fun way to get better at jumbled paragraphs. Get paragraphs from some magazines, articles and jumble
them after copying them onto Word. Then play the game of challenging your mates to get the right order, and hence the right
paragraph.
asy
(xi) These questions are not considered very tough to solve, but the way you can make the difference is by being quick and effective.

COMMON MISTAKES
En
gin
(i) Emphasizing options: These questions can really be done quickly if you work cleverly with the given options. But, options can
only aid you; using them as your base to solve questions can lead to a failure. For example: If three of your given options have

eer
the opening sentence as A and the fourth option has D as the opening sentence, then do not rule that D cannot be the opening
sentence. It might just be a trick to confuse you.

ing
(ii) Following a strict approach: you need to be really flexible in the way you approach questions. Some students always look out for
the opening sentence at first, and find it sometimes could be really tricky and time consuming.
(iii) The final sentence would not always be a concluding sentence or a sentence which summarizes the entire paragraph, so if you do
not find a concluding sentence in the given set of question, it may just not exist.
.ne
(iv) Questions in which the opening and closing sentences are provided are relatively easier. But students sometimes miss linking the
second last sentence with the last (given) sentence.
t
(v) Thinking that the opening sentence would not have a pronoun: it depends on the style of writer that sometimes they write the
pronoun once and its antecedent is mentioned in the next sentence, so you cannot rule out an opening sentence just on that.

SKILLS REQUIRED
(i) Arguably, the most important skill for these types of questions is to be a good reader; but, you just cannot become a good reader
in a day, it takes time. So, if you have plenty of time in your hand, reading whatever you get your hands on! This will not only
help in para jumble or para completion question, but your hold on the entire verbal section will be strengthened. While you are
into reading, make sure you read varied topics, so that when some unusual or uncommon topic comes up in the exam, you do
not get fazed by it.
If you are looking to read specifically to improve or get better at such types of question, then just read the second or some later
paragraph of an article, or story and try to predict or think what the previous paragraph would have been saying, or referring to.
This practice will improve your cognitive skills.
(ii) Understanding the topic – If you are not able to understand the topic of the passage or the statements, then it becomes very
difficult to find links between the given statements. So, make sure you are able to discern what is being talked about in the
sentences.

Downloaded From : www.EasyEngineering.net


Downloaded From : www.EasyEngineering.net

264  l Parajumbles

(iii) Understanding the flow of information and the kind of approach the author has toward the subject of the paragraph – Knowing
this will help you greatly in picking the right option. You need to be able to answer questions like: What is the purpose of the
paragraph? Is the author criticizing, support or just describing something?

BEGINNER TO MASTERY:
Example 1.
Which sentence would come first, A or B?
A. Two neighbouring parishes used to compete, and the object was to drive the ball from some central spot to one, or other,
village.
B. Hurling was a fast and furious game, very similar to our game of hockey, and played with sticks and a ball.
Explanation  The correct order is BA. It is simple, since in A, we are talking about parishes that competed, but what did they
compete in? This must be referenced earlier, hence BA.
Example 2.
In which order will these parts of a sentence come?

ww
A. which I had so often climbed before, and
B. I climbed the three dilapidated flights of stairs,
C. knocked at a small door at the end of the corridor

Example 3. w.E
Explanation BAC would create an appropriate sentence.

Order the following jumbled paragraph:


asy
A. Suddenly he heard sounds -- strange, low, melodious sounds, as if the shell were remembering and repeating to itself the
murmurs of its ocean home.

En
B. A child and a man were one day walking on the seashore when the child found a little shell and held it to his ear.

D. The child's face filled with wonder as he listened.


Explanation  BADC
gin
C. ere in the little shell, apparently, was a voice from another world, and he listened with delight to its mystery and music.

Example 4.
Arrange the following to form an appropriate grammatical sentence. eer
A. extending from the Bay of Fonseca to the River Paz.
B. Salvador has a coast-line of about 160 miles ing
C. and the meridians of 87° and 90° W longitude
D. between the parallels of 13° and 14° 10' N latitude, .ne
E. lying on the Pacific Ocean,
Explanation 
Example 5.
EDCBA t
In what order should these parts of a sentence come to form a cohesive and grammatically correct sentence?
A. how old I do not know
B. possibly more
C. but I cannot tell because I have never aged as other men
D. I am a very old man;
E. nor do I remember any childhood
F. possibly I am a hundred,
Explanation DAFBCE
Directions for Examples 6 to 11:  In each of the following questions, the answer choices suggest alternative arrangements of four or more
sentences (denoted by A, B, C, D, E, F). Choose the alternative which suggests a coherent paragraph.
Example 6.
A. No other document gives us so intimate a sense of the tone and temper of the first generation English poets.
B. Part of the interest of the journal is of course historical.

Downloaded From : www.EasyEngineering.net


Downloaded From : www.EasyEngineering.net

Parajumbles  l 265

C. And the clues to Wordsworth’s creative processes which the journal affords are of decisive significance.
D. Not even in their own letters do Wordsworth and Coleridge stand so present before us than they do through the references
in the journal.
(a) BADC (b) BDAC (c) CBAD (d) DABC
Explanation  (a); It is evident that the paragraph is about a journal which has a huge historical significance, so picking
our opening sentence is quite easy, it would be sentence B. The rest of the sentences just describe how this journal has historical
significance, so B is an appropriate opening sentence. Now, there are couple of reasons to why sentence A would be our next one: it
defines or says why this journal has historical significance, and this sentence mentions the first generation English poets (this journal
gives a very intimate sense of the tone and temper of English poets), and then the rest of paragraph would mention a couple of those
very famous English poets. Now, D would come before sentence C because of the word And which links the two of them like glue.
It hints that this is the concluding sentence of the paragraph, or the final reason as to why this journal has such a huge historical
significance.
Example 7.
A. There is no complete knowledge about anything.
B. Our thinking is the outcome of knowledge, and knowledge is always limited.

ww
C. Knowledge always goes hand in hand with ignorance.
D. Therefore, our thinking which is born out of knowledge, is limited under all circumstances.
(a) BCAD (b) BCDA (c) DABC (d) CBDA

w.E
Explanation  (d); The given paragraph is about knowledge, and the argument or the theme of this paragraph is that knowledge
and ignorance are inter-related. Ignorance means lack of knowledge, so this argument that knowledge and ignorance go hand-in-

asy
hand means that there is never complete knowledge about something (which is what A says, hence it is an appropriate concluding
sentence for the paragraph) Our opening sentence would be C as it introduces this argument we have just mentioned. Now, we have
to pick between B and D, which is an easy choice as sentence D starts with therefore which means hence or thus, so this must be the
conclusion or result of something.
Example 8. En
gin
A. That Hollywood is a man’s world is certainly true, but it is not the whole truth.
B. Even Renaissance film woman, Jodie Foster, who hosts this compendium of movie history, confesses surprise at this.

(a) ADBC (b) ABDC (c) DCAB eer


C. She says that she had no idea that women were so active in the industry even in those days.
D. During the silent era, for example, female screenwriters outnumbered males 10 to 1.
(d) ABCD
Explanation 
ing
(a); After reading the given sentences, we can say that the paragraph is about an argument that the idea or
thought of Hollywood being a man’s world, is not completely true. So, obviously the paragraph would start with stating this argument

.ne
that this notion that Hollywood is a man’s world is not true, which is exactly what sentence A says. Now, logically, the second sentence
would be coming up with a point or fact to prove this. Sentences B and C refer to a woman called Jodie Foster who was surprised at

t
knowing ‘this’. Now what does ‘this’ refer to? It would be some fact which is surprising and which supports the idea that Hollywood
is not completely a man’s world. Hence, sentence D would be our second sentence, as it mentions a fact to prove this. We just have
to decide between B and C now, and it is a easy pick. As sentence C starts with a pronoun she, if this was the third sentence, then it
would become ambiguous as its antecedent would not be clear. But if sentence B comes before it, then it is pretty clear. Hence, C will
come after B.
Example 9.
A. Something magical is happening to our planet. B. Some are calling it a paradigm shift.
C. It’s getting smaller. D. Others call it business transformation.
(a) ABDC (b) ACDB (c) ABCD (d) ACBD
Explanation  (d); The given paragraph is about the argument that our planet is changing, therefore sentence A gives us the
introductory sentence by stating that something magical is happening, which means that some unexpected, mysterious or grand
change is underway. Sentence C then naturally follows after A, as it explains what this change is. Sentences B and D then just explain
what people think about this change. Sentence B will come before D, because D starts with others which indicates that something on
the topic has been mentioned before.
Example 10.
A. Chemists mostly don’t stock it: only a few government hospitals do but in limited quantities.
B. Delhi’s building boom is creating a bizarre problem: snakes are increasingly biting people as they emerge from their disturbed
underground homes.

Downloaded From : www.EasyEngineering.net


Downloaded From : www.EasyEngineering.net

266  l Parajumbles

C. There isn’t enough anti snake serum, largely because there is no centralised agency that distributes the product.
D. If things don’t improve, more people could face paralysis, and even death.
(a) BCAD (b) DBCA (c) ABCD (d) CABD
Explanation  (a); The given sentence talks of a problem which the people of Delhi are facing with snakes biting them. Sentence
B provides us the topic of the paragraph as the remaining three refer or talk about it. Sentences A and C are talking about the medicine
which helps in curing the snake bite, whereas D comments on what would happen if things do not improve. After sentence A, it
would be logical to describe what the problem is or how severe it is and then wrap it off with sentence D to say what will happen if
things (the problems mentioned in the passage) do not improve. One obvious problem is that snakes are biting people, and the next
one, as mentioned in the passage is that there is not enough anti-snake serum. If sentence A comes before C, then it would sound a
bit ambiguous as the antecedent of pronoun it would not be clear; therefore C would come before A and then the paragraph will be
concluded with D.
Example 11.
A. Trivial pursuits, marketed by the Congress, is a game imported from Italy.
B. The idea is to create an imaginary saviour in times of crisis so that the party doesn’t fall flat - on its collective face.
C. Closest contenders are Mani Shankar Aiyar, who still hears His Master’s Voice, and V. George, who is frustrated by the fact
that his political future remains Sonia and yet so far.

ww
D. The current champion is Arjun Singh for whom all roads lead to Rome, or in his case, 10 Janpath.
(a) ABDC
Explanation 
(b) ABCD (c) DCBA (d) CDBA
(a); The given paragraph is about ‘Trivial pursuits’. A gives us the opening sentence, as it states the topic of the

w.E
paragraph, B then naturally follows A, as it explains what this ‘trivial pursuit’ is about. D then explains who is the current champion,
or who is playing this game the best and C then would after D as it mentions who is the closest contender to become the champion
(it obeys the logical order to first mention the champion and then the closest contender).
Example 12.
asy
1. Liz Taylor isn’t just unlucky in love

En
A. She, and husband Larry Fortensky, will have to pay the tab-$432,600 in court costs.
B. The duo claimed that a 1993 story about a property dispute damaged their reputations.
C. Taylor has lost a defamation suit against the National Enquirer.
D. She is unlucky in law too.
6. Alas, all levels of the California court system disagreed. gin
(a) CDAB
Explanation 
(b) DCAB (c) DABC
eer (d) CDBA
(b); So, the complete reason as to why Taylor was unlucky: She lost a defamation suit, which also resulted in a

ing
loss of $432,600 in court cases. The only remaining place for sentence B is fourth, but it can be verified as it perfectly links with the
closing sentence. Thus, there are two simple mandatory pairs: 1D & B6. Statement 6 mentions that in the end, courts at all levels dis-
agreed, now disagreed to what? This would be answered by sentence B, which would come just before this final sentence. Thus, with
these two mandatory pairs, our answer clearly becomes option (b), DCAB.
Example 13. .ne
1. For many scientists, oceans are the cradle of life.
A. But all over the world, chemical products and nuclear waste continue to be dumped into them.
B. Coral reefs, which are known as the most beautiful places of the submarine world, are fast disappearing.
C. The result is that many species of fish die because of this pollution.
t
D. Of course Man is the main cause of these problems.
6. Man has long since ruined the places he goes to -- continents and oceans alike.
(a) ACBD (b) BACD (c) ABDC (d) BCAD
Explanation  (a); The given paragraph is about how ocean life is destroyed and not taken care of. The opening sentence states
that oceans are considered as the cradle of life (where life began or originated) by many scientists. So, oceans have a lot of scientific
and natural importance. Now, sentences A, B, C show how oceans are destroyed and the state they are in now. But, sentences D and
6 form a mandatory pair: sentence D states that man is the reason for all these (the ones mentioned in the paragraph about how
oceans are being depleted) problems, and the closing sentence continues that idea or thought by stating that it is what men have been
doing: ruining the places they have been to (the given paragraph, is of course about how man has ruined oceans). So, now we have
to find the order between A, B, and C. Sentence A would be second as it starts with the conjunction but which is used to introduce
a contrary sentence. So, 1 and A together would mean another mandatory pair: oceans are of great importance and in spite of that,
chemical products and nuclear wastes are dumped into them. So even though oceans are important, still man does not consider them
so. Now, logically, sentence C will come after A because the result of dumping wastes into ocean would of course severely affect the
living things in ocean, i.e. fishes. Sentence B then mention another example of how man has ruined ocean life.

Downloaded From : www.EasyEngineering.net


Downloaded From : www.EasyEngineering.net

Parajumbles  l 267

Example 14.
1. So how big is the potential market?
A. But they end up spending thousands more each year on hardware upgrades and software overhauls.
B. Analysts say the new machines will appeal primarily to corporate users.
C. An individual buyer can pick up a desktop computer for less than $2000 in America
D. For them, the NC’s best drawing card is its promise of much lower maintenance costs.
6. NC’s, which automatically load the latest version of whatever software they need, could put an end to all that.
(a) BCAD (b) DABC (c) BDCA (d) DCAB
Explanation  (c); The given paragraph is about new computers called NC’s and the potential market these machines have.
The opening sentence starts with a question referring to how huge is the potential market for these machines, or how successful this
product can be in the market. Sentence B would come next to it, as it explains what according to the analysts, is the potential market
for new machines: the market for new machines will be restricted to corporate users. Sentence D then continues the analysis done by
the analysts as it starts with the pronoun ‘them’, so according to analysts, NC’s are in the market, already knowing that it would attract
corporate users, but the weapon or the tool which they could be planning to widen the market and make the personal home users buy
new machines is by their promise that there will be much less maintenance costs. Now, the rest of paragraph would explain how this

ww
promise can work in favour of the NC’s. Although an individual buyer can buy a desktop computer for less than $2000, but he/she
ends up paying a lot more on hardware and software upgrades. The promise of an NC can put an end to such spending on software
upgrades, and sway the individual buyers in buying their new machines.
Example 15.
w.E
1. Until the MBA arrived on the scene, the IIT graduate was king.
A. A degree from one of the five IIT’s was a passport to a well-paying job, great prospects abroad and, for some, a decent dowry
to boot.
asy
B. From the day, he or she cracked the joint entrance exam, the IIT student commanded the awe of neighbours and close
relatives.

En
C. IIT students had, meanwhile, also developed their own special culture, complete with lingo and attitude, which they passed
down.

gin
D. True, the success stories of IIT graduates are legion and they now constitute the cream of the Indian diaspora.
6. But not many alumni would agree that the IIT undergraduate mindset merits a serious psychological study, let alone an
interactive one.
(a) BACD
Explanation 
(b) ABCD (c) DCBA
eer (d) ABCD
(a); The paragraph is about IIT graduates and how highly they were regarded. The opening sentence says that

ing
before MBA arrived, an IIT graduate was considered a king. Now, the next sentence would be the one which talks about IIT graduates
and reasons or shows how they were treated as kings. Therefore, B would be our follow-up to 1, as it says that the one who cracked IIT

.ne
was given a lot of respect and admiration from the neighbours and the society (sentence A cannot be the second sentence because it
merely describes the benefits of cracking the IIT and does not reason why they would be like the kings). So, the next statement would
follow the sequential order and mention how beneficial it was to become an IIT graduate, which is what sentence A says. Even with
these mandatory pairs, the only option that remains for us is option (a).
Example 16.
1. Some of the maharajas, like the one at Kapurthala, had exquisite taste
t
A. In 1902, the Maharaja of Kapurthala gave his civil engineer, photographs of the Versailles palace and asked him to replicate
it, right down to the gargoyles.
B. Yeshwantrao Holkar of Indore brought in Bauhaus aesthetics and even works of modern artists like Brancusi and Duchamp.
C. Kitsch is the most polite way to describe them.
D. But many of them, as the available-light photographs show, had execrable taste.
6. Like Ali Baba’s caves, some of the palaces were like warehouses with the downright ugly next to the sublimely aesthetic.
(a) BACD (b) BDCA (c) ABCD (d) ABDC
Explanation  (d); The given paragraph is about the kings or Maharajas of India and the artistic taste they had. Our second
sentence is easy to pick and the given opening sentence gives us a big hint: it says that a lot of Maharajas, like the Maharaja of
Kapurthala had exquisite (lavishly elegant and refined) tastes, so quite naturally, the next sentence would be about the king of
Kapurthala to continue what was mentioned in the previous sentence. So, sentence A is second as it describes how the taste of this
king was exquisite: he gave his civil engineer the pictures of Versailles (a beautiful city near Paris) and told him to replicate or copy
that city exactly. This leaves us with only options (c) and (d). Now, we have to keep this sequential order of mentioning Maharajas who
had this strange artistic sense, hence our next sentence can only be B (sentence D mentions the king who had a bad taste, not a nice or
elegant one, and we will show how sentence C would come after D, thereby forming a mandatory pair). So, sentence B describes how

Downloaded From : www.EasyEngineering.net


Downloaded From : www.EasyEngineering.net

268  l Parajumbles

the Maharaja of Holkar had a great taste. He brought the Bauhaus aesthetics and work of great artists like Brancusi and Duchamp.
The third sentence would change the topic in a way by saying that some of these kings had an execrable (of poor quality) taste, which
is why sentence D would come here, and this can be verified as sentence C is just a continuation of sentence D, so they are correctly
placed at three and four, respectively. Now, let’s see why C would come after D: Kitsch means excessively garish or sentimental art;
usually considered in bad taste, and sentence C says that kitsch is the most polite way to describe them. This pronoun, them, refers
to those kings who had an execrable taste, who are mentioned or described in the previous sentence. Hence, it follows a sequential
order.
Directions for Examples 17 to 30:  In each of the following questions, the answer choices suggest alternative arrangements of four or
more sentences. Choose the alternative which suggests a coherent paragraph.
Example 17.
A. However, the severed head could not grow back, if fire could be applied at once to the amputated part.
B. To get rid of this monstrosity was truly a Herculean task, for as soon as one head was cut off, two new ones replaced it.
C. Hercules accomplished the labour through the aid of an assistant who cauterized the necks as fast as Hercules cut off the heads!
D. One of the Twelve labours of Hercules was the killing of Hydra, a water monster with nine heads.

ww
(a) DCBA
Explanation 
(b) ABCD (c) DBAC (d) BDCA
(c); ‘Labours of Hercules’ is a series of 12 travels about the penance carried out by one of the greatest Greek

sentences).
w.E
heroes, Hercules. The second one is about the slaying of a nine-headed Hydra (this information, of course, is not given in the

So, this paragraph is describing this incident. We will first look for the introductory sentence. Sentence D is an appropriate

asy
beginning, as it introduces by saying that one of the 12 labours was about the killing of Hydra, whereas, the rest of the sentences only
describe how this was accomplished or how difficult this task was. To get the right order for the sentences, we must logically think
how this task would have been accomplished. So, after reading these sentences, we can say that: Killing this Hydra was a very difficult

En
task, because as soon as one of the heads was removed, two new ones would develop to replace it. But the head which was detached
by cutting would not grow back if some fire was applied to the wound. So, this is how Hercules was able to kill this Hydra. And he

gin
had an ally, who helped him to apply this fire to the part which was cut, or an assistant who cauterized (burned) the neck as soon as
the Hercules removed the heads. So, from this story, we can say that the order is as given by option (c).
Example 18.
eer
A. Still, Sophie might need open-heart surgery later in life and now be more prone to respiratory infections.

ing
B. But with the news that his infant daughter Sophie has a hole in her heart, he appears quite vulnerable.
C. While the condition sounds bad, it is not life threatening, and frequently corrects itself.
D. Sylvester Stallone has made millions and built a thriving career out of looking invincible.
(a) DCAB (b) DBAC (c) DBCA (d) DCBA
.ne
t
Explanation  (c); From the given sentences, we can say that the paragraph is about Sylvester Stallone and his daughter. We have
to first look for the opening sentence, and we can easily see that only D is an introductory sentence as it introduces and describes
Sylvester Stallone (also, it is present in all the options.) Sentence A stars with ‘still’, which is an adverb which means ‘with reference
to an action or condition’, but if this was to be the first sentence, then it would lead to ambiguity as the action or condition it refers
to would be unclear. Sentence B starts with a conjunction ‘but’ which is used to introduce a contradictory clause or phrase. Hence
this cannot be the opening sentence. Sentence C as an opening sentence, would also lead to ambiguity as it starts with a pronoun
whose antecedent would not be clear. We now have to pick our second sentence. Sentence B is the obvious choice, because it does
mention something contradictory to what has been mentioned in the opening sentence that Sylvester is invincible. But the news that
his daughter has a hole in the heart has made him prone or vulnerable, so he is no longer this invincible person. Sentence B also uses
a personal pronoun ‘his’ which links it with the opening sentence which describes Sylvester Stallone, as it is the logical antecedent of
‘his’ in the second sentence. So it verifies our choice. We just have to choose between C and A, and both describe or inform about the
state or condition of Sophie, Sylvester’s daughter. But the way both these sentences start, with some logical sense, it is pretty clear that
C comes before A. Logically speaking, the information given is that even though her condition is bad, still it is not something which
would threaten her life, but in spite of this, she does need an operation later on.
Example 19.
A. The Saheli Program, run by the US Cross-Cultural Solutions, is offering a three week tour of India that involves more than
frenzied sightseeing.

Downloaded From : www.EasyEngineering.net


Downloaded From : www.EasyEngineering.net

Parajumbles  l 269

B. Participants interested in women’s issues will learn about arranged marriages, dowries and infanticide.
C. Holiday packages include all sorts of topics, but female infanticide must be a first for tourism.
D. Interspersed with these talks and meetings are visits to cities like New Delhi and Agra, home to the Taj Mahal.
(a) ACBD (b) CDBA (c) ADBC (d) CABD
Explanation  (a); These statements talk about a Saheli program, so our opening statement would be the one which best
introduces this program. Sentence A is the most appropriate opening sentence, as the rest just describe this program (C cannot be
the opening sentence, because this paragraph is not about some holiday package.) Before picking our next sentences, we can clearly
see why sentence D would be the final one. It states ‘along with these talks and meetings… ‘, which refers to everything that has been
mentioned in the paragraph before. Hence, this hints towards it being a concluding sentence. Now, we have to pick between B and
C as our second sentence of the paragraph: sentence C speaks about what this program is by stating what is included in the holiday
packages of this program, whereas sentence B tells about what the participants interested will be able to do. So, our statement B would
be the second sentence as it follows the sequential order of continuing to tell what the program is, and talks just about the program,
not participants like the sentence which came before it, i.e. sentence A. But even if this order of B and C is not clear to you, you can
easily pick the answer as option (a), as only this option has the opening and closing sentences as A and D, respectively.
Example 20.

ww
A. Its business decisions are made on the timely and accurate flow of information.
B. It has 1700 employees in 13 branches and representative offices across the Asia-Pacific region.
C. For employees to maintain a competitive edge in a fast-moving field, they must have quick access to J P Morgan’s proprietary

(a) DBAC
w.E
trade related data.
D. J P Morgan is one of the largest banking institutions in the U S and a premier international trading firm.
(b) DCBA (c) CDAB (d) DCAB

asy
Explanation  (a); It is evident that the paragraph is about a company known as J.P Morgan, so our opening would be the one that
best introduces what J. P. Morgan is. Statement D is an appropriate beginning for this paragraph, whereas the rest of the sentences are

En
clearly not. Statements A and B as opening sentences would lead to ambiguity with the use of the pronouns in it. Sentence C merely
describes the company, it does not introduce it. So, D is our opening statement and now we have to pick what comes after it. It’s clear
that the next sentence would continue its introduction. Statements A and B can be deemed as statements which are describing this

gin
company, but sentence C says how useful or important this company is for the employees: the logical order of the paragraph would be
to complete introducing this company and tell what it is, and then say how important and useful this company is. Hence, C would be

eer
our closing statement. Now, our first sentence would link us or give us a hint about which sentence comes next: the opening sentences
say that J.P. Morgan is one of the largest banking institutions in the USA. So, if there is another sentence which informs us where it

ing
is located or in which regions it can be found, then that statement would logically come just after this. Sentence B does just that by
mentioning the number of offices and branches it has in the Asia Pacific region. (the logical order would never be like telling where a
particular company is found or situated, then tell something else about it and then again inform the other regions where it is situated).
Example 21.
.ne
A. After several routine elections, there comes a ‘critical’ election which redefines the basic pattern of political loyalties, redraws
political geography and opens up the frozen political space.
B. In psephological jargon, they call it realignment.
C. Rather, since 1989, there have been a series of semi-critical elections.
D. On a strict definition, none of the recent Indian elections qualifies as a critical election.
t
(a) ABCD (b) ABDC (c) DBAC (d) DCBA
Explanation  (b); The given paragraph is about a critical election. A will be our opening sentence, because it defines what is
meant by a critical election, so it is an apt introductory sentence for the paragraph. B will naturally come after A, as it continues de-
fining it by stating its other name. (B could have come before A, but none of the options have B as the opening sentence). Sentences
C and D then just tell about its occurrence, and here the keyword is rather in sentence C, which indicates that it will come after D: C
states that none of the recent elections can be deemed a critical election, rather (on the contrary or instead), there have been a string of
semi-critical elections (so the idea conveyed is that: there have not been crucial elections, but there have been semi-critical elections).
Example 22.
A. Good advertising can make people buy your products even if they suck.
B. A dollar spent on brainwashing is more cost-effective than a dollar spent on product improvement.
C. That’s important because it takes pressure off you to make good products.
D. Obviously, there’s a minimum quality that every product has to achieve: it should be able to withstand the shipping process
without becoming unrecognizable.
(a) BACD (b) ACBD (c) ADCB (d) BCDA

Downloaded From : www.EasyEngineering.net


Downloaded From : www.EasyEngineering.net

270  l Parajumbles

Explanation  The given paragraph is about good advertising and how beneficial it can be. Sentence A gives us the topic of the
paragraph by stating that good advertising can make people buy your product even if it not a good one, and the remaining sentences
refer to that (B will not be the opening sentence as it is not introducing the topic, it is just comparing the benefits of spending on good
advertising and on making the product better). Now, we have to see which sentence would come next: B sways from the topic a bit, or
would lead to a bit inconsistency if it comes after A and the same can be said about D, but C links perfectly with A by revealing why
it’s important: the keyword that’s indicates this link. B would come after C because it is following on this theme of good advertising:
C said that good advertising takes the pressure off from making good products, but there is still a need to make products of decent
quality, so that they can at least last the transportation.). Sentence D then just serves as a concluding sentence.
Example 23.
A. Almost a century ago, when the father of the modern automobile industry, Henry Ford, sold the first Model A car, he decided
that only the best would do for his customers.
B. Today, it is committed to delivering the finest quality with over six million vehicles a year in over 200 countries across the
world.
C. And for over ninety years, this philosophy has endured in the Ford Motor Company.
D. A vehicle is ready for the customer only if it passes the Ford ‘Zero Defect Programme’.
(a) ABCD (b) ACDB (c) ACBD (d) CDAB

ww
Explanation  (c); The given paragraph is talking about the Ford Motor Company. Sentence A would be our opening sen-
tence, as it states what the ethos or the philosophy on which this company is based by stating the thinking with which Henry Ford
created this company. It is then linked with sentence C, as it states this philosophy refers to what is mentioned in sentence A, and with

w.E
the words ‘and for over 90 years’ links with A as A talked about the philosophy when this company was formed, and this philosophy
has been the same since then (which is indicated by ‘and for over 90 years’) Sentence B then would come afterwards as the author
brought us to the topic of how it functions in the present with sentence C. That is what B does by stating how large the production of

asy
this company is in the present and to how many countries they sell their cars to and with that what principles (finest cars). D then just
concludes by stating how precise they are in the process of manufacturing their cars, so it naturally follows after B.
Example 24.

En
A. There was nothing quite like a heavy downpour of rain to make life worthwhile.
B. We reached the field, soaked to the skin, and surrounded it.
C. The wet, as far as he was concerned, was ideal.
gin
D. There, sure enough, stood Claudius, looking like a debauched Roman emperor under a shower. (1998)
(a) DCBA
Explanation 
(b) ABDC (c) BDAC

eer (d) BACD


(b); Sentence B serves as the introductory sentence by stating that a group of people reach the field and they
were soaked to the skin (a proverb which basically means to be extremely wet) and surrounded by water all around. And when they

ing
reached there, they saw Claudius who quite liked being under the rain, as he considered it ideal. Then in sentence A, the author gives
a concluding sentence, which is sarcastic to show how he/she did not like being under heavy rains.
Example 25.
A. Such a national policy will surely divide and never unite the people.
.ne
B. In fact, it suits the purpose of the politicians; they can drag the people into submission by appealing to them in the name of
religion.
C. In order to inculcate the unquestioning belief, they condemn the other states which do not follow their religion. t
D. The emergence of the theocratic states where all types of crimes are committed in the name of religion, has revived the
religion of the Middle Ages.
(a) ABCD (b) DBCA (c) DBAC (d) CDAB
Explanation  (b); The given paragraph is about religion being used as a reason for committing crimes and exacting punish-
ment and how such a thing will only harm the nation. Sentence D works as the opening sentence as it gives us the topic of the para-
graph by stating that the theocratic states where crime are done on the name of religion is similar to how religion was considered or
used in the Middle ages (of course, this is a negative sentence as in the middle ages, people would go to any extent, even defy human-
ity just to serve the purpose of religion, as they thought). Sentence B then says what this thing really does; with the words in fact it
suits the purpose of politicians, the author is trying to say how politicians actually use religion to help their purpose (with the words
theocratic states he did refer to politicians who support or comply with religion as a reason to do the unlawful things). Sentence C
then continues saying how politicians use this to their advantage (C would come after B because as evident, B initiates telling how
politicians make use of religion to help their selfish purpose). Sentence A then concludes the paragraph by saying that all this would
only harm the nation, as it could lead to divisions within the country in terms of religions.
Example 26.
A. The Director walked into the room and took a look around the class.
B. Mitch wanted to scream - the illogicality of the entire scene struck him dumb.
C. The managers stared at him with the look of fear that no democratic country should tolerate in its people.

Downloaded From : www.EasyEngineering.net


Downloaded From : www.EasyEngineering.net

Parajumbles  l 271

D. Mitch walked out of the room - it was his irrevocable protest against an insensible and insensitive situation.
(a) ACBD (b) BDAC (c) BCAD (d) ABCD
Explanation  (a); The given situation describes a particular scene and describes how Mitch reacted to this situation. Sentence
A introduces us to this scene by stating that the Director came inside the room and he looked around. Sentence C then continues
the scene (logically, the people inside the room were managers at whom the director was looking). C stated that managers got scared
and were in fear in the presence of the Director (who is of course the boss of the managers). Sentence B and D then just describe how
Mitch reacted to this scene. Sentence B says what he thought of this scene and then D naturally follows after B as it states what he did
in reaction to this scene.
Example 27.
A. The establishment of the Third Reich influenced events in American history by starting a chain of events which culminated
in war between Germany and the United States.
B. The Neutrality Acts of 1935 and 1936 prohibited trade with any belligerents or loans to them.
C. While speaking out against Hitler’s atrocities, the American people generally favoured isolationist policies and neutrality.
D. The complete destruction of democracy, the persecution of Jews, the war on religion, the cruelty and barbarism of the allies,
caused great indignation in this country and brought on fear of another World War.
(a) ABCD (b) CBDA (c) CDBA (d) ADCB

ww
Explanation  (d); The given paragraph is talking about the times during World War II and its relation with or effects on
America. Sentence A introduces us to the paragraph by stating that the formation of Third Reich resulted in a string of events which
led to the war between Germany and United States. (Third Reich is another name for Nazi Germany, or the Germany when it was

w.E
ruled by Adolf Hitler) Sentence D, then naturally follows after A by stating how this Nazi Germany was. Sentence C then explains
what Americans thought of the policies they adopted before and during those wars and then sentence C clarifies by mentioning those
policies. This leaves us with only option (d).
Example 28.
asy
A. If caught in the act, they were punished, not for the crime, but for allowing themselves to be caught another lash of the
whip.

En
B. The bellicose Spartans sacrificed all the finer things in life for military expertise.
C. Those fortunate enough to survive babyhood were taken away from their mothers at the age of seven to undergo rigorous
military training.
gin
D. This consisted mainly of beatings and deprivations of all kinds like going around barefoot in winter, and worse starvation so
that they would be forced to steal food to survive.
eer
E. Male children were examined at birth by the city council and those deemed too weak to become soldiers were left to die or
exposure.
(a) BECDA
Explanation 
(b) ECADB (c) BCDAE
ing (d) ECDAB
(a); The given paragraph is about Spartans, who were the residents of the ancient Greek city of Sparta and well

.ne
known for being great fighters and warriors. So, our opening sentence would be the one which introduces Spartans the best. As we
can see, only sentence B introduces Spartans. So this will be our first sentence. Now, this first sentence would link us to the next one:
the first sentence states that Spartans were bellicose, which means that they were bottleful or combative and it also says that they

they sacrificed. Sentence A talks about some punishment they were given. Sentence D as the next one would be ambiguous, because t
sacrificed finer things in life for the sake of military expertise, so logically, the next sentence would explain or reason this to show how

the antecedent of this would be unclear. Sentence C cannot be the next one either as it says that those fortunate to survive boyhood,
but then what was the tough thing or thing to survive from? Hence it would not make logical sense as the second sentence, but would
make perfect sense after sentence E and this is why E is our second one. sentence.
Example 29.
A. Perhaps the best known is the Bay Area Writing Project, founded by James Gray in 1974.
B. The decline in writing skills can be stopped.
C. Today’s back-to-basics movement has already forced some schools to place renewed emphasis on the three Rs.
D. Although the inability of some teachers to teach writing successfully remains a big stumbling block, a number of programmes
have been developed to attack this problem.
(a) BCDA (b) ADCB (c) ACBD (d) CABD
Explanation  (a); The given paragraph is basically around the argument that the decline in writing skills can be put to an end,
and that is why B is our opening sentence. Sentence C then presents a way in which this decline can be stopped by mentioning that
there is renewed emphasis on the three Rs in schools (three Rs refers to the basic set of education, or the fundamentals of education:
reading, writing and arithmetic). Now these are of course, taught by teachers in school to students, so sentence D naturally follows
sentence C, as it states that the inability of teachers sure is an obstacle, but there are programs designed to tackle that as well, and then
sentence A names one of those programs.

Downloaded From : www.EasyEngineering.net


Downloaded From : www.EasyEngineering.net

272  l Parajumbles

Example 30.
A. By reasoning, we mean the mental process of drawing an inference from two or more statements or going from the inference
to the statements which yield that inference.
B. So logical reasoning covers those types of questions which imply drawing an inference from the problems.
C. Logic means, if we take its original meaning, the science of valid reasoning
D. Clearly, for understanding arguments and for drawing the inference correctly it is necessary that we should understand the
statements first.
(a) ACBD (b) CABD (c) ABCD (d) DBCA
Explanation  (b); The given paragraph is basically explaining the section of questions which are based on logical reasoning
and what these are based on. The keyword to decide the order is ‘so’ and we will explain why. Sentence C explains what logic means,
then sentence A explains what reasoning means and then B would come after these two, as it explains what logical reasoning means
(author first describes logic , then reasoning and then comes sentences C as it starts with the word so which means hence: so we if
we combine these sentences, then the logical order is presented like, logic means this, reasoning means this, hence logical reasoning
means this). Then in D, author just concludes the key to solving these questions by stating that you really need to be able to under-
stand the arguments first.
Example 31.

ww
1. It is often said that good actors can get out of play more than the author has put into it.
A. A good actor, bringing to a part his own talent, often gives it a value that the layman on reading the play had not seen in it,
but at the utmost he can do no more than reach the ideal that the author has seen in his mind’s eye.

w.E
B. In all my plays I have been fortunate enough to have some of the parts acted as I wanted; but in none have I had all the parts
so acted.
C. That is not true.

performance he visualized.
asy
D. He has to be an actor of address to do this; for the most part the author has to be satisfied with an approximation of the

6. is so obviously inevitable, for the actor who is suited to a certain role may very well be engaged and you have to put up with

(b) DACB En
the second or third best, because there is no help for it.
(a) BACD (c) CADB (d) DCBA
Explanation 
gin
(c); The opening statement starts by saying that the actors who are good or talented can get more out of the
script, i.e. make the play better than it actually was, as created by the author. But reading all the remaining statements, we can see that

eer
the paragraph says that this notion or thought is not right. So, logically, the second statement of the paragraph would be saying that
this thought is not true and then go on to explain and say why it is so. Hence, the first would be (C). The statement (A) then explains
the previous statement by saying that the most an actor can do is match the ideas or imaginations of the author, so we can say that C

ing
and A form a mandatory pair in a way, so the latter explains the former. Now, we have two statements remaining. Statement (D) men-
tions a pronoun ‘this’ which must be referring to a thought or idea mentioned before, which logically is in statement B. This statement

.ne
mentions a new thought that the speaker was fortunate that he had some of the parts acted or written in the way he would like or
want. It can be inferred that this is something which the author deems important for an actor to perform well and do ‘justice’ to the
play written by the author. Statement D also mentions something which links it with the final statement i.e. (6). It says that it has to

t
be an actor of address to do this’ which means that the actor has to be able to look into it or put some efforts towards it, i.e. whether
a particular play or act suits him or not. This statement also mentions that whatever the actor does, the author has to be first satisfied
with it, and the author has to feel that the actor justified the play. Our choice can easily be verified by the given concluding statement
that an actor may not be suited to the approximations or the expectations of the author, and in such situations, that actor would have
no option for the actor but to perform at inferior levels, just because the circumstances and the author demands so.
Example 32.
1. The wind had savage allies.
A. If it had not been for my closely fitted helmet, the explosions might have shattered my eardrums.
B. The first clap of thunder came as a deafening explosion that literally shook my teeth.
C. I didn’t hear the thunder, I actually felt it - an almost unbearable physical experience.
D. I saw lighting all around me in every shape imaginable.
6. When very close, it was raining so torrentially that I thought I would drown in mid-air.
(a) BCAD (b) CADB (c) CBDA (d) ACDB
Explanation  (a); the opening statement states that the wind had savage allies. ‘Savage’ is an adjective which means wild and
menacing, and ‘allies’ means associates or the ones who accompany. So, the first given statement means that the winds came with
dangerous allies, and it can inferred from the given statements that it refers to thunder/ lightning and rains. Hence, our paragraph
will talk about these allies, but as we can see, rains have already been mentioned and introduced in the concluding statement, so that
leaves us with thunder. We can see that the statements A, B, C discuss how powerful the thunder was, or the impact it had on the
speaker; whereas statement D mentions lightning, noting about the physical impact or effect it created. So, our next statement would

Downloaded From : www.EasyEngineering.net


Downloaded From : www.EasyEngineering.net

Parajumbles  l 273

be either one of A, B, C, or statement D, but none of the options have D as the third statement, so we will rule that out. So, the order
will be that first of all, the thunder will be described, then the presence of lightning and then rains in the concluding statement (the
information always follow a sequence, it would not be deemed right to talk about how powerful thunder was, then lightning and
then mention again how strong the thunder was). Therefore, our next to last statement would be D. It also correlates to the final and
concluding statement as lightning and rains are usually mentioned together. Now, we have to just decide the order of statements A, B,
C and we will be done with this question. First of all, we will look at which statement among A, B and C best introduces thunder. It is
evident that statement B best introduced the thunder with ‘first clap of thunder’. Now, we have just to have pick between A and C, but
if you have a look at the options, we already arrived at the answer by picking the first statement out of the four. It can be argued that
A can come before C, and the reverse could be deemed correct too. But, if we had to pick then it would be that C will come ahead of
A with the conditional statement ‘if ’ just signifying how loud the thunder was.
Example 33.
1. A thorough knowledge of the path or course to be followed is essential for achieving success.
A. Seniors must show the path clearly by laying down the precise expectations of the management in terms of job description,
key result areas and personal targets.
B. They should also ‘light the path’ by personal example.
C. Advice tendered or help offered must be objectively evaluated for its effectiveness in achieving the desired goal.

ww
D. A display of arrogance and a false sense of ‘self-worth’, in order to belittle those who come to help prove dysfunctional.
6. The individuality of each employee must be respected.
(a) CDAB (b) CADB (c) BADC (d) ABCD

w.E
Explanation  (d); If you read all the statements in the given order, you would see that there is nothing wrong and it is not a
jumbled paragraph at all. Anyway, we will explain why it is so. The first statement says that it is very important to have in-depth
information or knowledge of the path or the way to be followed if one wants to have success. So, this is basically mentioning how

asy
to be successful. We have to now pick the second statement of this paragraph, and logically it would present another idea or suggest
something in order to be successful. It’s clear that statement B will not be the first statement, as it starts with the pronoun ‘they’
and if the first statement started with this pronoun, then it will not be clear in terms of what this pronoun refers to. All of the given

En
statements are in a way giving advice to someone, but to whom? Who is our subject? The only one which can be logically inferred
from the given set of options is ‘seniors’ and how they should be, is mentioned in the given statement. So, statement A would be the

gin
first statement. It says that the seniors must clearly explain what is expected from the management, and what are the key result areas
and personal target. Sentence B then should be the next one for a couple of reasons: it starts with a pronoun they and to help remove

eer
ambiguity, this statement should come just after seniors have been mentioned. Other reasons could be that the sentence before it
mentions that seniors must lay down what should be done, and then should also ‘light the path’ i.e. show the management how it
should be done, and in a way, lead by example. So, they should tell what is to be done, and then also show practically how it should be

ing
done. Now, there is a link between C and D which ensures the order in which they come. C mentions that any help offered to them
must be first evaluated well in terms of how effective and good it is to achieve the desired result. Sentence D then mentions what
would happen if that help offered is not taken seriously or given its due respect.
Example 34.
.ne
1. Michael Jackson, clearly no admirer of long engagements, got married abruptly for the second time in three years.
A. The latest wedding took place in a secret midnight ceremony in Sydney, Australia.
B. It is also the second marriage for the new missus, about whom little is known.
C. The wedding was attended by the groom’s entourage and staff, according to Jackson’s publicist.
t
D. The bride, 37 year old Debbie Rowe, who is carrying Jackson’s baby, wore white.
6. All that is known is that she is a nurse for Jackson’s dermatologist.
(a) CBAD (b) BDCA (c) DCBA (d) ACDB
Explanation  (d); The paragraph is about Michael Jackson’s recent marriage. The opening statement mentions that Jackson
got married for the second time in three years, so the next sentence, abiding by the sequential order, would inform about this new
wedding. Sentence A states that this latest wedding took place in a secret midnight ceremony in Sydney, now if there is a sentence
which describes or gives more information about this wedding, then it would come next. Hence, C is our third sentence as it
informs who attended this wedding. Now, the remaining sentences are about Michael Jackson’s wife, so we will look for a sentence
which best introduces her. It is evident that sentence D, with the appositive phrase, introduces or describes his wife. Now, the only
remaining spot for B is fourth, but it can be verified as it makes a perfect link with the closing sentence: sentence B says that not
much known is about her, and sentence 6 continues this thought by saying that all that is known is that she is a nurse of Jackson’s
dermatologist.
Example 35.
1. Whenever technology has flowered, it has put man’s language-developing skills into overdrive
A. Technical and technoid terms are spilling into the mainstream almost as fast as junk-mail is slapped into e-mail boxes.

Downloaded From : www.EasyEngineering.net


Downloaded From : www.EasyEngineering.net

274  l Parajumbles

B. The era of computers is no less.


C. From the wheel with its axle to the spinning wheel with its bobbins, to the compact disc and its jewel-box, inventions have
trailed new words in their wake.
D. “Cyberslang is huge, but it’s parochial, and we don’t know what will filter into the larger culture” said Tom Dalzell, who wrote
the slang dictionary Flappers 2 Rappers.
6. Some slangs already have a pedigree.
(a) BCAD (b) CBAD (c) ABCD (d) DBCA
Explanation  (b); The given paragraph is about how the advancement in technologies has affected or influenced the language
developing skills of humans. The opening sentence states that with the growth of technology, the language-developing skills have
been put to excessive use or have been over-worked. So, the next sentence would continue on this idea and reason it. Sentence C
nicely continues this idea by giving examples of how technology has advanced and led to a string of new words (for example, ad-
vancement from wheel to axle), so with these advancements, a lot of new words have come up which have put the language devel-
oping skills in an overdrive. We can see that the remaining sentences talk about advances in technology and their effects in terms of
computers. So, our next sentence would introduce this thought that the computer era has also contributed to the list of new words.
Therefore, B is our next sentence. D and 6 form a mandatory pair, as our closing sentences refer to slangs, and this is what sentence
D is about. So, that just leaves A with the third position, but it makes sense anyway because after discussing that computer era has

ww
also had an effect, sentence A mentions how technical terms are becoming popular and coming into the mainstream.
Directions:  In each of the following example, the answer choices suggest alternative arrangements of four or more sentences. Choose the
alternative which suggests a coherent paragraph.
Example 36.
w.E
A. But instead you are faced with another huge crag and the weary trail continues
B. No, the path winds on and another mountain bars your way.

asy
C. When for days you have been going through a mountain pass, a moment comes when you are sure that after winding around
the great mass of rock in front of you, you will come upon the plain.
D. Surely after this you will see the plain.
(a) CDBA (b) BADC
En (c) CADB (d) BCAD
Explanation 
gin
(c); After reading all the four statements we can see that only C can be deemed fit enough to be the introduc-
tory statement: statement A starts with the conjunction but, which is used to introduce a contrasting thought. Since that thought
has already been mentioned previously, this cannot be our introductory statement. Statement B starts with ‘No’, so again, it refers to

eer
something that has already been mentioned; whereas statement D mentions a pronoun this, and if it is the opening statement then
it would become ambiguous, as its antecedent would not be clear.

ing
Now, this paragraph basically talks about travelling through a mountain pass (a route through a mountain) and when you are
on this kind of a path, after a certain time, you hope that a plain or a simple path will come, but that does not happen as another
mountain comes up; so, basically we have to build our paragraph around this. We have already chosen C as our first statement, now

.ne
statement B starts with the conjunction but which as stated above is used to introduce a contradictory thought, and logically, A
would come after C as it mentions that we come across with another huge crag (a steep cliff or rock) and the weary tail (referring to
the curvy nature of the mountain pass) continues. So in combination, statements C and A mean that after a moment you hope that

t
a plain path with come and you will get rid of this winding path, but that does not happen as another crag comes up and the twisted
path continues. Following the same theme, after crossing this crag in the previous statement, you would be hoping that now definitely
this mountain pass has come to an end and you will see the plains, and this is exactly what statement D refers to. And the remaining
statement is B which continues the theme that you are not yet able see the plain as another mountain comes in your way.
Example 37.
A. Where there is division, there must be conflict not only division between man and woman, but also division as racial,
religious and linguistic.
B. We said the present condition of racial divisions, linguistic divisions has brought on so many wars.
C. Also we went into the question of why does this conflict between man and man exist.
D. May we continue with what we were talking about last evening?
(a) ABCD (b) DABC (c) BCAD (d) BDAC
Explanation  (b); It is an interesting question because the given set of four sentences do not represent a paragraph out of an
article or some essay, rather just a conversation between two or more people. The first sentence would be D, as in this, the speaker
asks for permission to discuss which was talked about last evening; and the fact that the given statements have words like we said,
we went to question in the given sentences indicate that the speaker is reminding his listener(s) what was said or discussed last eve-
ning. So, the opening sentence would be D in which the speaker asks for permission to talk about what was discussed last evening,
and the speaker goes on to remind those things or topics which were discussed. So, our second sentence would reveal or show the
topic of discussion, which means that sentence A comes next. Now, we have to pick between sentence B and C. Here, the key word
is ‘also’ as it starts sentence C, this indicates that with this sentence, the speaker finished mentioning what was discussed. (If this

Downloaded From : www.EasyEngineering.net


Downloaded From : www.EasyEngineering.net

Parajumbles  l 275

sentence which began with also came before another topic was mentioned, then the paragraph would not have a proper sequential
order)
Example 38.
A. But the last decade has witnessed greater voting and political participation by various privileged sections.
B. If one goes by the earlier record of mid-term elections, it is likely that the turnout in 1998 will drop by anything between four
and six percentage points over the already low polling of 58 per cent in 1996.
C. If this trend offsets the mid-term poll fatigue, the fall may not be so steep.
D. Notwithstanding a good deal of speculation on this issue, it is still not clear who benefits from a lower turnout.
(a) BACD (b) ABCD (c) DBAC (d) CBDA
Explanation  (a); The given paragraph is talking about the expected turnout in 1998. B will be our opening sentence as it
gives us the topic of discussion (we will form further links to verify why this is our opening sentence). B states that according to the
record of the midterm elections, there will be a four to six percent drop in the number of people who will vote in 1998, as compared
to the number of people who voted in 1996, so this sentence indicates that there is a firm reason as to why lesser amount of people
will vote in 1998.
Now, sentence A starts with the word but which means that it is introducing a contrary clause. It mentions a reason why there
might be more voters in 1998 as compared to 1996. It says something contrary to what is mentioned in B hence, the ‘but’ links these

ww
sentences. So, B will be followed by A. C will come next because it is linked with A: C states that if this trend (of voting and political
participation by various privileged sections) continues then it would offset (make up) mid-term poll fatigue (this logically refers to
the drop seen in the number of voter for mid-term elections, as indicated by the record of mid terms election. In 1998, (elections are

w.E
mid-term) the fall (in the number of people who would vote in 1998) would not be so steep (lesser than expected four to six percent).
D then just acts a concluding sentence by saying that it is not sure who will benefit from this expected fall in the number of voters
for 1998.
Example 39.

asy
A. I sat there frowning at the chequered table cloth, chewing the bitter cud of insight.
B. That wintry afternoon in Manhattan, waiting in the little French restaurant, I was feeling frustrated and depressed.

En
C. Even the prospect of seeing a dear friend failed to cheer me as it usually did.
D. Because of certain miscalculations on my part, a project of considerable importance in my life had fallen through.
(a) ADBC
Explanation 
(b) BCDA

gin
(c) BDCA (d) ABCD
(b); The author in the given passage is talking about some night when he was waiting for his friend and was
a in bad mood because an important project that he was part of, had failed. There are a couple of evident mandatory pairs in the

eer
given set of questions. Sentence A states that he was frowning at the chequered table cloth and chewing the bitter cud of insight,
which means that the author was thinking over and over again something which he did not happen as he would have wanted to,

ing
and he is pretty upset about it. Sentence D starts with because and it gives the reasoning of something which has been mentioned
before, and that logically refers to sentence A as D gives the reason for him being sad. Even B and C form a mandatory link: B states
that he was waiting in the restaurant feeling depressed and C states that even the prospect of seeing a dear friend failed to make him

.ne
feel better (he was in a restaurant, waiting for his dear friend). So, B and C together form a logical and meaningful pair.
Example 40.
A. To be culturally literate is to possess the basic information needed to thrive in the modern world.
B. Nor is it confined to one social class; quite the contrary.
C. It is by no means confined to “culture” narrowly understood as an acquaintance with the arts.
D. Cultural literacy constitutes the only sure avenue of opportunity for disadvantaged children, the only reliable way of
t
combating the social determinism that now condemns them.
E. The breadth of that information is great, extending over the major domains of human activity from sports to science.
(a) AECBD (b) DECBA (c) ACBDE (d) DBCAE
Explanation  (a); The paragraph is about cultural literacy and its importance. So, our opening sentence would the one which
best introduces cultural literacy. Only sentence A introduces it by stating that cultural literacy means having basic information re-
quired to be successful in the modern world. (Sentence D just describes how useful it is). So, A is our opening sentence and this sen-
tence links us to the sentence which would come after this: it states that cultural literacy is to posses all the basic information required
to be successful, so E as the next sentence would fit perfectly as it describes how this information varies and has a huge spectrum. E
describes that this information has several sources of knowledge. Hence, being ‘culturally literate’ is a broad term. You could argue
that sentence C also describes information but if it were our second sentence, then the pronoun it would make this sentence ambigu-
ous as the antecedent of it would not be clear. Does it refer to information or just cultural literacy? But, after E, the sentence C, makes
perfect sense and does not have ambiguity as by following the sequential order, it then describes more about information and the
kind of spectrum it has. Sentences C and B in fact, make another mandatory pair, as nor at the start of B indicates that it initiates a
negative sentence similar to the one that has just been mentioned, so it makes a perfect link with sentence C. Now, the only remaining
sentence is D which shows why cultural literacy is important, which follows the sequential order as after describing cultural literacy
completely, its importance was described.

Downloaded From : www.EasyEngineering.net


Downloaded From : www.EasyEngineering.net

276  l Parajumbles

Example 41.
A. He was carrying his jacket and walked with his head thrown back.
B. As Anette neared the lamp she saw a figure walking slowly.
C. For a while Michael walked on and she followed twenty paces behind.
D. With a mixture of terror and triumph of recognition she slackened her pace.
(a) ABCD (b) BADC (c) BCDA (d) ACBD
Explanation  (b); The given paragraph just describes a situation in which Anette saw a man walking ahead of her. We will
combine the information given by these four sentences and arrange this information in the logical order which will give the right
option. Anette was walking somewhere and as she came close to the lamp, she saw someone who was walking slowly with a jacket in
his hand and his head thrown back. After seeing that she slowed her pace, and she felt scared but also felt good about the fact that she
saw him before she got close enough. So, that guy kept on walking for a bit and Anette was 20 steps behind him; thereby, making sure
he does not know that she is there.
Example 42.
A. His left hand concealed a blackjack, his right hand groped for the torch in his pocket.
B. The meeting was scheduled for nine o’ clock, and his watch showed the time to be a quarter to nine.
C. The man lurked in the corner, away from the glare of the light.

ww
D. His heart thumped in his chest, sweat beads formed themselves on his forehead, his mouth was dry.
(a) CABD
Explanation 
(b) BDAC (c) BADC (d) ABCD
(a); The given sentence probably describes a situation where a serious or dangerous meeting was to take place,

w.E
certainly one which the man described in the passage was worried about. Sentence C introduces us to this situation or scene by stat-
ing that there was a man lurking in the dark. The word lurking is the key here, as it links us to the second sentence. ‘Lurking’ means
‘waiting in hiding to attack’. So, C basically states that this man was waiting in a dark place, trying to hide himself from someone. Sen-

asy
tence A continues from C by describing him more: he was hiding a blackjack (a tool which is used to attack someone), this sentence
further solidify the intentions of this man. A also says that he was searching for a torch in his pocket (it makes sense, as he was in a
dark place). Sentence C then describes that he had a watch which showed that meeting did not take place at the scheduled time and
then D just describes how this man was feeling.
En
You can argue that D can come before C, but selecting our first two sentences leads to a unique sentence so it is not like picking

gin
the relative order between C and D is a stumbling block. But sentence D also makes some sense after D: the meeting did not take place
at the time it was scheduled to take place, so the man might have thought that something is not right, or things are not taking place
as they should and this could have been the reason for his fear which is described in sentence D.
Example 43.
eer
A. He was bone- weary and soul- weary, and found himself muttering, “Either I can’t manage this place, or “it’s unmanageable.”

him in routine work that had no significance.


ing
B. To his horror, he realised that he had become the victim of an amorphous, unwitting, unconscious conspiracy to immerse

C. It was one of those nights in the office when the office clock was moving towards four in the morning and Bennis was still
not through with the incredible mass of paper stacked before him.
.ne
D. He reached for his calendar and ran his eyes down each hour, half- hour, and quarter-hour to see where his time had gone
that day, the day before, the month before.
(a) ABCD
Explanation 
(b) CADB (c) BDCA (d) DCBA
t
(b); The given paragraph is about Bennis who was constantly too involved and busy in work. Sentence C presents
us the perfect introductory sentence by stating that it was on one of those nights (when it was four in the morning and Bennis still
had lots of work to be done) when he realized a certain thing. Now, this situation of working till the morning and still having not
completed the work, led him to have a look at the calendar to realize and see where all his time went (he was working a lot and almost
of his time was spent on working in office). This now links us to sentence A which means that the situation made him say things like
either he cannot manage this place, or it is unmanageable (he of course referred to the massive amounts of work he was always given.
Bone-weary and soul-weary just mean that he was extremely tired due to or because of his office work. Sentence B then works as the
concluding sentence by stating that all this brought him an awful realization, which is mentioned there.
Example 44.
A. Since then, intelligence tests have been mostly used to separate dull children in school from average or bright children, so
that special education can be provided to the dull.
B. In other words, intelligence tests give us a norm for each age.
C. Intelligence is expressed as intelligence quotient and tests are developed to indicate what an average child of a certain age can
do -- what a 5 year old can answer, but a 4-year-old cannot, for instance.
D. Binet developed the first set of such tests in the early 1900s to find out which children in school needed special attention.
E. Intelligence can be measured by tests.
(a) CDABE (b) DECAB (c) EDACB (d) CBADE

Downloaded From : www.EasyEngineering.net


Downloaded From : www.EasyEngineering.net

Parajumbles  l 277

Explanation  (c); The given paragraph is about testing intelligence and the use of it. Here the key is the correct sequential order
of information flow. As we know that quite often, picking the opening sentence is the most important aspect of solving these ques-
tions, because you can then form links to proceed to the subsequent sentences. So, we will look at each of the sentences to see if they
fit as the opening sentence. Sentence A starts with the phrase since then and Sentence B starts with in other words. These phrases in-
dicate that something has been mentioned already, so of course none of them can form the opening sentence. Now, sentence C shows
how the data collected by testing one’s intelligence is used, or the use of testing intelligence. Sentence D says who developed these tests
and why, whereas sentence E merely introduces the topic by saying that intelligence can be tested. Hence logically, this would be our
opening sentence. Now, let us look at which sentence would come next. Even if finding the opening sentence looks complex, there
are a couple of mandatory pairs in the given sentences and finding them will be enough to pick the right answer (as it would lead to a
unique option). Sentence D mentions that Binet developed such tests (tests of intelligence) in 1990s to find out which student should
be given special care or attention. Now, sentence A says that since then these tests have been used to separate bright children from
the dull ones. They keyword is ‘since’ because it is used to refer to a time period, hence the logical link exists between these sentences:
since 1990s, these tests are used to differentiate bright and dull students. Now, we look at the other mandatory pairs: Sentence B starts
with the phrase in other words which means that ‘something which already has been mentioned, can be expressed in other words
as’ and logically, that could only be Sentence C, as both of these sentences mean the same thing in different words. Now, we have the
right order after finding mandatory pairs and the opening sentence. It could either be EDACB or ECBDA, but the former would be

ww
appropriate, as in it, the second sentence continues the introduction of these tests by mentioning when they were developed and who
developed them.
Example 45.

w.E
A. But in the industrial era destroying the enemy’s productive capacity means bombing the factories which are located in the
cities.
B. So in the agrarian era, if you need to destroy the enemy’s productive capacity, what you want to do is burn his fields, or if
you’re really vicious, salt them.

asy
C. Now in the information era, destroying the enemy’s productive capacity means destroying the information infrastructure.
D. How do you do battle with your enemy?

En
E. The idea is to destroy the enemy’s productive capacity, and depending upon the economic foundation, that productive
capacity is different in each case.

(a) FDEBAC
Explanation 
(b) FCABED
gin
F. With regard to defense, the purpose of the military is to defend the nation and be prepared to do battle with its enemy.
(c) DEBACF (d) DFEBAC
(a); The given passage is basically about the purpose of military with regard to defending from and attacking an

eer
enemy. After reading the given sentences, we can say that sentence E tells us the purpose of military for defense, whereas, sentences
A, B and C tell ways of battling with the enemy and these are the answers put forward in the question in sentence D (hence A, B and

ing
C are bound to come after D), so we can say that sentences A, B, C and D form a part of the paragraph, whereas F forms another part.
So, the paragraph can either start with D or F (F forms a part of the sentence in itself as it is talking about something very different
from the rest of the sentences, so it will either come in the beginning or in the end to form a paragraph which has a sequential order

.ne
of information). As we have already said that F and D, & A, B, and C are two parts of a paragraph, so the probable beginnings are:
FD or DF. Now, we just have to decide the correct order between A, B and C to reach a unique option. Sentence A tells how to battle
in the industrial era, B informs about battling in the agrarian era; whereas C talks says the way of battling an enemy in the present

t
era. Hence, we just have to arrange these three sentences in terms of their respective time periods, which is called time sequencing.
Industrial era came after the agrarian or agricultural era, and both of these belong to the past i.e. are not of the present, therefore the
logical order is BAC.

Downloaded From : www.EasyEngineering.net


Downloaded From : www.EasyEngineering.net

278  l Parajumbles

Practice Exercise
LEVEL-I
Directions (Qs. 1-50) :  The sentences given in each question, S : In recent years, however, even mighty IBM found that
when properly sequenced, form a coherent paragraph. Each market domination was not enough if you fall behind
sentence is labelled with a letter. Choose the most logical order on concept.
of the sentences from amongst the given choices so as to form a (a) P S R Q (b) R P Q S
coherent paragraph. (c) R S Q P (d) S R P Q
1. P : The computer took 45 hours of non-stop computation. 5. P : Of course, they have not made the blunders that some
Q : The computer was not allowed to learn from experience. others have made, but neither did they grow.
R : This is not really surprising, because with eleven items Q : The chief executive had done an excellent job in welding
of clothing the number of possible combinations is a group of motley and successful companies
given by multiplying 11,10,9,8 and so on, which gives into one profitable company.

ww
over 39 million combinations.
S : Someone once used the IBM computer to work out the
number of ways of getting dressed with eleven items of
R : It is not surprising that company had been sitting on a
cash hoard of about $24 billion year after year without
any attempt to use it for growth or development.

w.E
clothing. .
(a) Q S P R
(c) S Q R P
(b) S R P Q
(d) S P R Q
S : The chief executive of the General Electric Company in
England once told me that he was very happy when
there were no problems in any of his many divisions.

asy
2. P: In his first inaugural address he concluded with an
eloquent plea: “Ask not what your country can do for
you-ask what you can do for your country”. 6.
(a) Q P S R
(c) R Q S P
(b) S P Q R
(d) S Q R P
P : In the past, the customised tailoring units were localised
Q : John F. Kennedy, Democratic victor in the election
En
of 1960, was at 43, the youngest man ever to win the
to the township or city and catered exclusively to
domestic demand.
presidency.
R : On television, in a series of debates- with opponent
Richard Nixon, he appeared able, articulate and
gin Q : Traditionally, Indians preferred custom-made clothing
and the concept of ready-to-wear is a relatively recent
one.
energetic.
S : In the campaign, he spoke of moving aggressively into
R:
eer
Consumer awareness of styling issues and the
convenience afforded by ready-to-wear helped RMG
the new decade, for “the New Frontier is here whether
we seek it or not”.
(a) S P Q R (b) Q R S P
ing
industry make small inroads into the domestic market
in the 1980s.
S : The customised tailoring outfits have always been a
(c) R P Q S (d) Q P R S
3. P : One day, a 17-year old shepherd boy came to visit his (a) Q R S P .ne
major source of clothing for domestic market.
(b) Q S P R
brothers and asked, Why don’t you stand up and fight
the giant?”
Q : We all know the story of David and Goliath, in which
there was a giant who was bullying and harassing the
7.
(c) R S Q P . (d) S Q P R

t
P : Such a system will help to identify and groom executives
for positions of strategists.
Q : Evaluation of performance is more often than not
children in the village. done for the purpose of reward or punishment for past
R: But David said, No, he is not too big to hit; he is too big performance.
to miss.” R : They must become an integral part of the executive
S : The brothers were terrified and they replied, Don’t you system’ .
see he is too big to hit ?” S : Even where the evaluation system is for one’s promotion
(a) Q P S R (b) P R S Q to assume higher responsibilities, it rarely includes
(c) Q P R S (d) Q S R P terms that are a key for playing the role of strategist
4. P: Businesses often feel that since they have a lot of market effectively, e.g., the skills of playing the role
muscle and in such a dominant position, maintenance of change agent and creative problem solving.
will be enough. (a) S Q P R (b) S R Q P
Q : IBM has also suffered from lower priced clones when (c) R S Q P (d) Q S R P
the mystique of computers was off and there was no 8. P : Participation involves more than the formal sharing of
longer a need for reassurance from the solidity of Big decisions.
Blue. Q: Through anticipation individuals or organisations
R : IBM fell behind on the concept of “connectivity” and consider trends and make plans, shielding institutions
has suffered as a result. from trauma of learning by shock.

Downloaded From : www.EasyEngineering.net


Downloaded From : www.EasyEngineering.net

Parajumbles  l 279

R : Innovative learning involves both anticipation and (a) ACBD (b) CABD
participation. (c) ACDB (d) DBAC
S : It is an attitude characterised by the cooperation, 13. A. Because, if the manager’s subordinates are inefficient and
dialogue and empathy. ineffective and are not helped to increase their efficiency
(a) Q R S P (b) P Q R S and effectiveness, the task may not be achieved.
(c) R Q P S (d) S P Q R B. This must be just as true as the responsibility for
9. P : Almost a century ago, when the father of the modem achieving his prescribed tasks. .
automobile industry, Henry Ford, sold the first Model C. If it is achieved it is at too great a cost, or at the risk of
T car, he decided that only the best would do for his other effects, many of which are less obvious.
customers. D. It is often and that one of the prime responsibilities of a
Q : Today, it is committed to delivering the’ finest quality manager is the training and development of his staff.
with over six million vehicles a year in over 200 countries (a) ADBC (b) CABD
across the world. (c) BDAC (d) DBAC
R : And for over 90 years, this philosophy has endured in 14. A. Modern research, however, has proved that there were
the Ford Motor company. invaders even before the Aryans poured into this land.
B. It was thought that they came to a country which was

ww
S : Thus a vehicle is ready for the customers only, if it passes
the Ford ‘Zero Defect Programme’.
(a) P Q R S (b) P R Q S
Uncivilized and barbarian.
C. They had evolved a civilization higher than that of the


10. P :  w.E
(c) R S P Q (d) P R S Q
Finish specialists recommended a chewing gum
containing xylitol-a natural sweetener present in birch,
Aryan hordes who came in their wake.
D. Till recently the Aryans were regarded as the earliest
invaders of the land.

young children. asy


maple, corn and straw-to be used several times a day by

15.
(a) ABCD
(c) BDAC
A. 
(b) BCAD
(d) DBAC
Organizations today are becoming increasingly
Q : Chewing gum is a new solution that “may work
for parents whose children suffer from chronic ear
infections. En populated by youthful, highly skilled, highly educated
workers.

R : An experiment was conducted involving three hundred


and six children between two and six years. gin B. Two of the most prevalent and provocative organizational
dynamics of our time are the themes of
participation and change.
S : After Finish studies showed that xylitol is effective in
preventing cavities, a team of researchers decided to
C.
eer
These demands for participation are creating
pressures for internal organizational change which are
investigate its effects on a very similar type of bacteria
which causes ear infections.
(a) Q R S P (b) P Q R S
ing
matched only by external environmental pressures for
organizational change.

11.
(c) R Q P S (d) Q P S R
P : Such a national policy will surely divide and never unite .ne
D. These young, skilled and educated workers bring with
them demands for a voice in the determination of their
own organizational destiny-a chance to participate in
the pople.
Q : Infact, it suites the purpose of the politicians, they can
drag the people into submission by appealing to them
in the name of religion. 16.
(a) ABDC
(c) ABCD
(b) BADC
(d) BCDA
t
those decisions which affect their organizational lives.

A. We tend to see the similarity within a category as being


R : In order to inculcate, the unquestioning belief, they more important, and the similarity between different
condemn the other states which do not follow their categories as being less important, that either actually is.
religion. B. Given a small amount of information about a person,
S : The emergence of the theocratic states, where all types we are ready to classify them as a member of a particular
of crimes are committed in the name of religion of the group, and then to infer all kinds of additional facts
Middle Ages is dangerous. about them, as if all members of the group were the
(a) P Q R S (b) S Q R P same in most respects.
(c) S Q P R (d) R SP Q C. Our tendency to classify and label everything can lead
12. A. You would be very surprised indeed to find it hot. us into the error of seeing the world as made up of only
B. Cold, of course. those categories for which we have names.
C. And yet that was what I found when I visited North D. Expecting too much of the descriptive power of
Island, the northern part of New Zealand. languages is itself a serious cause of distorted thinking.
D. When you go bathing in a river or a pond, do you expect (a) DCBA (b) DBAC
the water to be hot or cold? (c) BDAC (d) BCDA

Downloaded From : www.EasyEngineering.net


Downloaded From : www.EasyEngineering.net

280  l Parajumbles

17. A. Some of us, as a result, gain an overall impression of B. The government on its part should show both sincerity
people as either all good or all bad, making further and flexibility in crafting a solution that will assure the
assumptions on this basis. Naga people a life of peace, dignity, and self-respect
B. An example of the assumption some patients make that recognizing the uniqueness of Naga history.
doctors with a good “bedside manner” are also more C. The government should also do everything possible
technically competent as others who do not relate as to remove the suspicion that it has been encouraging
well to their patients. certain Naga groups.
C. This is what the psychologists refer to as halo effect. D. No solution can be found through bloodshed.
D. We have a strong tendency of associate positive E. Growing internecine violence among them is a worry.
attributes with other positive attributes and negative (a) ACBDE (b) ABDEC
ones with each other. (c) DCBEA (d) ABCED
(a) DCBA (b) DABC 22. A. The latest decline draws attention to a fairly well known
(c) DBCA (d) DACB fact that stock exchanges in India take their cues as much
18. A.  In the past, the customized tailoring units were from developments overseas as from domestic ones.
localized to the township or city and catered exclusively B. While such reaction is an inevitable consequence of
to domestic demand. globalisation, it is the magnitude of the reaction that has
B. Traditionally, Indian preferred custom made clothing come as a surprise.

ww
and the concept of ready to wear is a relatively recent
one.
C. The stock indices the world over tumbled on the back
of some pessimistic news from the United States credit

w.E
C. Consumer awareness of styling issues and the
convenience afforded by ready to wear helped the RMG
industry makes small inroads into the domestic market
in the 1980s.
market, especially its home loan segment.
D. Fears over the failure of risky sub-prime mortgages seem
to have spilled over to other closely watched sectors of
the economy such as sales of new homes and consumer

asy
D. The customized tailoring outfits have always been a
major source of clothing for domestic market.
durables.
E. Last week the Dow recorded its highest fall since
(a) BDAC
(c) CDBA
(b) BCDA
(d) DBAC
En
19. A. Participation involves more than the formal sharing of
February, dragging the European bourses down.
(a) ADEBC
(c) BEDCA
(b) ABCDE
(d) AEDCB
decisions.
B. Through anticipation, individuals or organizations gin
23. A. Moreover, it was felt that leaders elected by villagers
themselves would find it easier to implement central
consider trends and make plans, shielding institutions
from trauma of learning by shock.
C. Innovative learning involves both anticipation and eer
government policies regarding taxes and family
planning.
B. Since 1998, elections to village councils, which
participation.
D. It is an attitude characterized by the co-operation, ing
comprise between three and seven members,have been
institutionalized and are now carried out every three
dialogue and empathy.
(a) ABCD
(c) CBAD
(b) BCDA
(d) DABC
years.

.ne
C. The council’s main responsibilities lie in deciding the
allocation and use of communal land, the running of
20. P : In a number of cases, the drivers have refused to carry
passengers according to the meter-reading despite it
being in working condition.
Q : For instance, according to a complaint, the driver of an
planning directives. t
village enterprises, and the implementation of family

D. Councils can also decide local matters like village


subscription to newspapers, the renovation of a school
auto rickshaw not only misbehaved but also ran away building, or the installation of cable television.
with Rs. 500 from Dhaula Kuan on October 13. E. Following the collapse of the village commune system
R: Refusal to carry commuters to their respective after the economic liberalization initiated in 1978, certain
destinations is another common complaint which has leaders within the Communist Party began pushing for
been lodged with the call centres. village self-governance as a means to counter political
S : What have been the most shocking are the complaints apathy and violent rebellion by creating mechanisms of
about misbehaviour by the rickshaw drivers with the participation and conflict resolution.
passengers. (a) DCBEA (b) CABDE
T : Similarly, another driver ran away with the luggage of (c) EABCD (d) BACDE
the passenger on the Karol Bagh-Paschim Vihar route. 24. A. The U.S. market will continue to be the dominant one
(a) S P R Q T (b) S R Q P T in the foreseeable future. The rupee could become even
(c) P R S Q T (d) P Q R S T stronger.
21. A.  The Naga groups, assuming they are serious about B. A greater recourse to hedging as well as striving for
negotiations, should make a realistic reassessment of multi-currency revenue streams automatically suggests
how far the Indian State can go to meet their demands. itself.

Downloaded From : www.EasyEngineering.net


Downloaded From : www.EasyEngineering.net

Parajumbles  l 281

C. Already one company, TCS, by resorting to these of some of India’s key military organizations and covert
methods extensively has turned in an above average services, are defended in a manner that would be
performance during the first quarter. considered unconscionable negligent in many parts of
D. Most IT companies have been grappling with more the world.
mundane problems such as a high level of attrition E. However, successful security depends not on crisis–
amidst rising wage costs and inability to secure the right time creativity but on the disciplined and effective
type and number of American visas. implementation of mundane, everyday protocols.
E. The BPO industry and many medium-sized software (a) ABDEC (b) AEBCD
exporters are reportedly operating on thin margins. (c) ADECB (d) ABEDC
(a) BCADE (b) ABCDE 28. A. On the Republican side, the Iowa results have left the
(c) DCBAE (d) EDABC picture somewhat murkier.
25. A. It will take extraordinary political commitment and B. Mike Huckabee beat the putative front–runner, Mitt
liberal public funding during the 11th Plan for affordable Romney, by a margin of 34.4 per cent to 25.4 per cent,
housing to become a credible goal. but is not expected to carry the momentum forward
B. The National Urban Housing and Habitat Policy of the into New Hampshire.
United Progressive Alliance Government seeks to make C. Mr. Huckabee’s victory is attributable largely to the

ww
access to housing, long acknowledged as a fundamental
right, a reality for all.
C. The task is staggering even if we go by conservative
strong support he got from evangelical Christians who
are estimated to make up as much as 60 per cent of the
caucus – goers.

w.E
estimates.
D. The housing shortage to be met during the Plan is 26.53
million units, which include the backlog from the 10th
D. Social and religious conservatives, will constitute only
small proportion of voters, in which national security
and fiscal concerns are expected to be the main issues.
Plan.
asy
E. If the existing stock of poor quality dwellings and the
growing urbanization–driven demand are taken into
E. However, while Mr. Romney might be free of a Huckabee
challenge in New Hampshire, he might be hard pressed

(a) ADBEC (b) ACDEB En


account, the real deficit will be even higher.[Feb. 2008]
to fend off John McCain and Rudolph Guiliani, both of
whom largely stayed out of the Iowa campaign.
(a) ABCDE (b) AEBCD
(c) ABEDC (d) ABCDE
26. A. The upsurge of public activism against the setting up of
Special Economic Zones, which eventually forced the
gin (c) ACDEB (d) ABCED
29. A. Using biofuels made from corn, sugarcane and soya
State Government to announce the scrapping of all 15
such projects, is an impressive case in point. eer
could have a greater environmental impact than burning
fossil fuels, according to experts.
B. Although the fuels themselves emit fewer greenhouse
B. Early last year, a similar agitation coerced the government
into calling for a revision of the Goa Regional Plan 2011,
a controversial document that opened up large swathes
ing
gases, they all have higher costs in terms of biodiversity
loss and destruction of farmland.
of land, including green belts and coastal stretches, for
construction. .ne
C. The EU has proposed that 10% of all fuel used in
transport should come from biofuels by 2020 and the
C. The broad–based agitation against SEZs has
demonstrated the power of popular protest in the State.
D. Those opposed to the projects had questioned the
propriety of the government acquiring large tracts of
of dollars a year.
D. But the new fuels have attracted controversy. t
emerging global market is expected to be worth billions

E. The problems of climate change and the rising cost of


land and then selling them to promoters at low prices. oil have led to a race to develop environment–friendly
E. A coastal State with an area of 3,700 square kilometers biofuels, such as palm oil or ethanol derived from corn
and a population of about 1.4 million, Goa has always and sugar cane.
been extremely sensitive to the impact of unrestrained (a) DECBA (b) ABCED
economic development. (c) ABECD (d) CDEBA
(a) CDEAB (b) BCDEA 30. A. The precision with which the crucial operation was
(c) EABCD (d) DABCE executed has unequivocally demonstrated ISRO’s
27. A.  India’s security apparatus responds well when beset capability to take up the more complex deep space
by crisis. However, successful security depends not on mission as distinct from numerous near-earth missions
crisis. in the past.
B. Potential targets must be secured as if terror strikes were B. The achievement has put India in the exclusive club
imminent. of space-faring nations that have ventured beyond the
C. Here the Indian system’s record is appalling. sphere of the earth’s gravitational influence.
D. Despite years of painful experience, sensitive government C. That ISRO brought this off in its first attempt is all the
installations in New Delhi, including the headquarters more commendable.

Downloaded From : www.EasyEngineering.net


Downloaded From : www.EasyEngineering.net

282  l Parajumbles

D. The successful critical manoeuver on November 8 E. Anil Kumble’s retirement from test cricket brings to an
that put Chandrayaan-I in an orbit around the moon end a sporting career of high distinction.
marked the completion of the most important phase of (a) BCDEA (b) DCABE
the Indian lunar mission. (c) EABCD (d) ABCDE
E. The rest of the mission involves only standard orbit
34. A. ‘It’s a tricky business’, says Allan H. Meltzer, an
manoeuvers, the likes of which the Indian Space economist at Carnegie mellon University and a former
Research Organization (ISRO) is quite used to, and the economic adviser to President Reagan.
performance of the on-board scientific instruments B. Some policy- makers are focused on staving off the
during the mission life of two years. opposite problem- deflation, or falling prices, as demand
(a) BCADE (b) DEABC weakens to the point that goods pile up without buyers,
(c) CDABE (d) EDCBA sending prices down and reducing the incentive for
31. A. To the joy of tens of millions of cricket fans, it established businesses to invest.
itself as the only team in this era to consistently C. That could shrink demand further and perhaps even
challenge—and not infrequently master—Australia deliver the sort of downward spiral that pinned Japan in
both at home and away. the weeds of stagnant growth during the 1990s.
B. In fact, since the turn of the millennium India has D. “There’s no math model that tells us when to do it or

ww
transformed itself into an all-round cricketing power
capable of winning in varying conditions anywhere.
C. Over the last decade, Australia and India have set up
how.”
E. But that, as most economists see it, is a worry for another
day.

w.E
Test cricket’s most absorbing rivalry.
D. Every champion needs a counterpoint; legacies, defined
referentially, draw from the dynamic between world- 35.
(a) ABCED
(c) BCDEA
(b) ACDEB
(d) ADEBC
A. Early in August, as his jeep wound its way through the
beater and contender.
asy
E. India, thanks to its natural style of calm aggressive
cricket, its distinctive ability to play at the level of the
piles of burning tyres that angry protestors had used to
barricade the road from Srinagar airport into the city,
former Chief Minister Farooq Abdullah, turned to a
opposition, and the confluence of some of the finest
En
cricketers in the contemporary game, fulfilled this role
journalist sitting next to him with a smile on his face.
B. “So,” he said, “are you here to write another article about
of contender.
(a) EDCAB
(c) CDEBA
(b) DCABE
(d) CDEAB
gin how I don’t know how to run a government?”
C. Having emerged as the single largest party in the J&K
Assembly, with 28 seats in a house of 87, the National
32. A. A nation has gone against its historical record.
B. Risen above its worst prejudices in one, emotional eer
Conference has the undeniable right to form and lead
the government.
incandescent moment.
C. Well, at least partly, and for a while Americans have
voted in larger numbers than they have in decades,
ing
D. To do so, however, it will need the support of the
Congress, which has picked up 17 seats.
E. More likely than not, Dr. Abdullah’s leadership - or
perhaps ever.
D. Millions of younger voters have been fired by the .ne
that of Jammu and Kashmir National Conference
President, his son Omar Abdullah- will soon be put to
youthful senator they have chosen to send to the White
House.
E. The African-American President-elect did far better
with white male voters than fellow-Democrat John 36.
the test.
(a) ABECD
(c) ADEBC
(b) ABDEC
(d) ABCED
t
A. This is the time of the year when people go out and shop
Kerry did four years ago. for their winter wear.
(a) ABCDE (b) CDABE B. But it seems economic recession has hit the fashion
(c) DECBA (d) BCEDA industry as well.
33. A.  Few men have commanded greater respect from C. We haven’t seen exclusive fashion shows from big
comrade and opponent alike; nobody has won more designers this winter.
tests for India. D. Designer Sandeep Khosla agrees, “Every industry has
B. For all the weight of his achievements—greater than any been hit and fashion is no different.
other Indian bowler, and among the finest in the world- E. Its effect could be seen on both couture and readymade
Kumble tended to be under-rated. segments.
C. This was partly because he had to share the stage with (a) ADBCE (b) ABCDE
Shane Warne and Muttiah Muralitharan, whose big spin (c) AEDBC (d) BCDEA
wizardry and world-beating success struck a chord with 37. A. And, in turn, corporate houses seek employees who will
the lay fan. benefit their company and help the company grow.
D. Kumble’s rigour and nuance demanded a more B. It is an exchange of value.
discerning taste. C. Both sides are seeking to benefit.

Downloaded From : www.EasyEngineering.net


Downloaded From : www.EasyEngineering.net

Parajumbles  l 283

D. Since we are on the eager prospective employee side, we C. The failed mission of Copenhagen is the immediate
need to please and satisfy our potential employers. cause of the resumption of this debate.
E. One seeks employment for personal gain, profit and D. While the main triggers of the debate are economic,
success. social and cultural factors also have a major role to play.
(a) ABCED (b) BCDEA E. As with so many other issues, clearly, here too ‘one man’s
(c) EABCD (d) DEABC meat is another man’s poison.’
38. A. After all, a story told on the large screen inevitably
(a) DECAB (b) CEDAB
differs from that told on the small screen.
B. This critical difference has an impact on viewership in (c) BCDAE (d) BACED
terms of age, income and occupation. 42. A. Many so-called indicators for stocks and indexes take
C. In this, the age of multimedia, we have to train ourselves on complex hues, such as taking on moving averages of
to understand that as a rule, the medium is the message. moving averages and so on.
D. It also has an impact on the expectations brought by the B. A moving-average-based indicator will always be a
public to bear on large and small screen performances, little late, and you should naturally be suspicious of any
and on the performers. ‘formula’ that can predict the next move, based purely
E. Never has the myth of ‘one size fits all’, been shown up so on moving averages of price.
effectively, therefore, as in the field of Media Studies. C. The moving average is simply a “smoothing” function

39. ww
(a) ABDCE
(c) CABDE
(b) CABED
(d) CDABE
A. Indeed, the reading-public of today seems to be more
– it gets rid of periodic volatility to tell you the recent
trend.
D. At best, they can tell you a trend, and if the hypothesis is

w.E
tolerant of this crossover than their predecessors might
have been.
B. Both writers and readers seem to enjoy cross-crossing
that the trend will sustain, and that bears out historically
in enough instances, you might have a hope with it.
E. But smoothing has its disadvantages; it reacts slowly to

asy
the line between documentation and fiction.
C. Beginning with Midnight’s Children, there has been
a steady breakdown of the disciplinary wall between
sudden changes, so it will only tell you that the trend has
changed after the trend has changed, sometimes too late
literature and history.

En
D. Editorial cartoons, once barely recognised as a source
of humour for the masses, are now studied as important
to actually take action.
(a) ABCDE (b) DBCAE

sources of historical documentation and literary value.


E. This has led to a revision in the view of what constitutes gin (c) ACEBD (d) BECAD
43. A. From Sweden to the UK to Greece to even the US in the
early part of the century, housing prices have fallen.
historical and literary debate, and of what constitutes
the sources of this debate.
(a) DECAB (b) CBAED eer
B. But wasn’t that just correlation?
C. Housing bubbles have been known to go bust in the

40.
(c) CABED (d) ABCED
A. Sub-Saharan Africa is often cited as a territory in which
starvation could be significantly reduced, were GM
ing
past, and in different countries.
D. “The real estate market has never gone down in any
meaningful way” – this statement was often quoted by
foods brought into worldwide circulation.
B. Farmers cite the steady impoverishment of the soil, and .ne
real estate agents and brokers in the US, and it might
have even been statistically valid, with over 50 years of
the deterioration in the quality of seeds, as excellent
reasons for protesting GM foods.
C. As with many cutting-edge discoveries, however, its
long-term consequences can be difficult to handle.
data supporting it.
t
E. While the argument is moot today (US House Prices are
still falling, after more than three years of a downward
trend) it remains alive in pockets of the world.
D. Genetically modified, or GM foods, are marketed
(a) CDABE (b) CDBAE
enthusiastically by some section of the developed world
(c) DBCAE (d) BEDAC
that claim they can cure the ills of the developing world.
E. A lack of transparency concerning ethical testing is 44. A. For pure vegetarians India is a heaven.
another reason given by the developing world for B. India can boast for its innumerable varieties of tasty and
receiving GM foods with caution rather than with nutritious vegetarian dishes.
celebration. C. These are also prepared using different methods of
(a) DACBE (b) BEDCA cooking like baking, boiling, frying etc.
(c) CADEB (b) DEBAC D. Vegetables are an integral part of our food and we
41. A. For example, cars in the developing world are often seen consume them in a number of ways.
as status symbols to be acquired, while in the developed E. Indians like their vegetable curries real hot ‘n’ spicy and
world they are seen as liabilities to be discarded. so add a number of spices to make them really exotic.
B. The size of the carbon footprint of nations in the (a) BCADE (b) AECDB
developing world has again come in for serious (c) CEBDA (d) DABEC
international discussion.

Downloaded From : www.EasyEngineering.net


Downloaded From : www.EasyEngineering.net

284  l Parajumbles

45. A. Over the last few decades green tea has undergone many E. The Buddha silently held up and twirled a flower
scientific and medical studies to determine the extent of and twinkled his eyes; several of his disciples tried to
its long health benefits. interpret what this meant, though none of them was
B. In China there is a proverb - “Better to be deprived of correct.
food for three days, than tea of one” -and they were (a) EBDAC (b) DBAEC
using the tea as a cure for headache, depression and (c) BCDEA (d) CADBE
many other ailments. 49. A. The band has gone through several drummers over the
C. But it’s true that Chinese people were well aware about years, though Travis has held the position since 1989
green tea from ancient time. and is the longest-serving.
D. We came to know about this green tea very late. B. ‘Judas Priest’ are an English heavy metal band from
E. If I had said that tea is a healthy drink some years Birmingham, England, formed in 1969.
before the introduction of green tea, I might have been C. Their popularity and status as one of the definitive heavy
ridiculed .. metal bands has earned them the nickname “Metal
(a) EDCBA (b) CDBAE
Gods” from their song of the same name.
(c) EBDAC (d) DECAB
D. They have been cited as an influence on many heavy
46. A.  Environment Education Unit of Centre for Science

ww
and Environment has always been working towards
providing easy-to-understand reading material.
B. Their new publication on this subject is an attempt to
metal musicians and bands.
E. The core line-up consists of lead vocalist Rob Halford,
guitarist Glenn Tipton, bassist lan Hill, and drummer

w.E
lend teachers a helping hand.
C. It unfolds in two sections: Climate Change: how to make
sense of it all, and Natural Resources: how to share and
Scott Travis.
(a) ACBDE
(c) CDEAB
(b) DABCE
(d) BEADC
care.
asy
D. Here, the key issues selected adhere strictly to
50. A. Since then, intelligence tests have been mostly used to
separate dull children in school from average or bright
children, so that special education can be provided to
curriculum guidelines.
E. However, they are introduced to students not as a
paragraph to memorise, but as an activity to do. En the dull
B. In other words, intelligence tests give us a norm for each
(a) ACEBD
(c) ABCDE
(b) DBCAE
(d) BECAD gin age.
C. Intelligences expressed as intelligence quotient and tests
47. A. A famous Japanese rock garden is at Ryoan-ji in North-
West Kyoto, Japan.
eer
are developed to indicate what an average child of a
certain age can do--what a 5 year old can answer, but a
4-year-old cannot, for instance.
B. The rocks of various sizes are arranged on small white
pebbles in five groups, each comprising five, two, three,
two, and three rocks. ing
D. Binet developed the first set of such tests in the early
1900s to find out which children in school needed
C. The garden is 30 meters long from East to West and 10
meters from North to South.
special attention.
.ne
E. Intelligence can be measured by tests.
D. The garden contains 15 rocks arranged on the surface of
white pebbles in such a manner that visitors can see only
14 of them at once, from whichever angle the garden is
viewed.
(a) CDABE
(c) EDACB
(b) DECAB
(d) CBADE
t
DIRECTIONS (Qs. 51 - 55) :  Arrange the sentences, A, B, C and
D to form a logical sequence between sentences 1 and 6.
E. There are no trees, just 15 irregularly shaped rocks of
varying sizes, some surrounded by moss, arranged in a 51. 1. The list of horrors goes on.
bed of white gravel/sand that is raked every day. A. And one in every five is malnourished.
(a) ACEBD (b) CAEDB B. This is because local clinics, ill-equipped to deal with
(c) DEABC (d) BADEC even small things, either don’t work or simply don’t
48. A.  When they gathered together, the Buddha was exist.
completely silent and some speculated that perhaps the C. Nobody has been able to figure out a way to reduce
Buddha was tired or ill. the speed that is at the root of India’s over-population
B. It is said that Gautama Buddha gathered his disciples problems: a baby born every second.
one day for a Dharma talk. D. There is such an acute shortage of treatment centers that
C. One of the Buddha’s disciples, Mahakasyapa , silently premier hospitals are choked with patients who show up
gazed at the flower and broke into a broad smile. to treat their coughs and colds.
D. The origins of Zen Buddhism are ascribed to the Flower 6. Kalyan Banerjee, a consultant at the hospital is worried.
Sermon, the earliest source for which comes from the (a) DACB (b) CDAB
14th century. (c) DBAC (d) CADB

Downloaded From : www.EasyEngineering.net


Downloaded From : www.EasyEngineering.net

Parajumbles  l 285

52. 1. It was something I knew that it would give me a chance 6. Disposable incomes are rising in the metros and big
to be my own boss. cities and time is at a premium.
A. I could have more except that supervision becomes (a) CADB (b) ADBC
difficult. (c) ABDC (d) CABD
B. Today I have 800 on my staff, office and members.
DIRECTIONS (Qs. 56-60): Rearrange the following eight
C. The moment I announced I was starting a business,
people started contacting me. sentences/groups of sentence (A), (B), (C), (D), (E), (F),
D. I was sick of working for others. (G) and (H) in the proper sequence to form a meaningful
6. Our clients are well-known industrialists and paragraph; then answer the questions given below them.
organizations, many of whom were in touch with me [SBI Clerk 2012]
when I was in the services. (A) Both Ram and Sham realised their mistakes and were
(a) ABDC (b) DBAC ashamed about what they had said.
(c) DCBA (d) BADC (B) Vivek happened to overhear their conversation and was
53. 1.  Fire ripped through another pipeline in southern very angry with both of them for criticising the tree.
Nigeria, killing at lease 40 people. (C) Two friends, Ram and Sham, were seeking respite from
A. The explosion was the third in two weeks. the searing heat of the midday sun when they saw a huge
B. Police were deployed to stop villagers from stealing fuel

ww
from other pipelines.
C. Local politicians have blamed this vandalism on cartels.
leafy tree.
(D) “It’s a plain tree,” said his friend. “Don’t waste your time
looking for fruits. It produces neither edible fruits nor

w.E
D. Once the cartels have siphoned off fuel, impoverished
locals move in to collect what they can for sale to passing
motorists.
6. But pipelines often explode, and the practice has left
good wood. It’s one of the most useless trees around.”
(E) “How can you say such a thing when you’re enjoying the
shade of this beautiful tree at this very moment?” snapped

(a) ABCD asy


about 2000 people dead in the past two years.
(b) ACBD
Vivek, unable to control his anger.
(F) They took shelter under the huge leafy tree and soon felt
cool and refreshed.
(c) CABD (d) CBDA

En
54. 1. Reservation should not exceed 50% for the civil services
for want of balance and efficiency.
(G) Busy belittling the tree, both Ram and Sham did not notice
that another person, Vivek, was lying on the other side of
A. If reservation is 50% it is adequate for aspirants from
reserved category and even unreserved category to get gin the tree taking shelter.
(H) “What sort of tree is this? Does it produce edible fruits?”
an equal opportunity.
B. The number of aspirants to the civil services in India is
very large and they come from various socioeconomic
eer
asked Ram.
56. Which of the following should be the SECOND sentence
after rearrangement?
backgrounds.
C. These aspirants come from both reserved and
(a) A
(c) E ing (b) B
(d) F
unreserved category.
D. But if reservation were to exceed 50% mark, a lot of
(e) G
.ne
57. Which of the following should be the EIGHTH (LAST)
deserving candidates from unreserved category would
be deprived of a chance.
6. Thus to achieve optimum efficiency, it is essential to
maintain a maximum of 50% reservation.”
sentence after rearrangement?
(a) A
(c) D
(e) F
(b) B
(d) E t
(a) ADBC (b) BCAD 58. Which of the following should be the FIFTH sentence
(c) CABD (d) BADC after rearrangement?
55. 1. This is a company that prides itself on its carefully (a) D (b) E
matured extensive distribution blocks and mentor (c) F (d) G
(e) H
network.
59. Which of the following should be the FIRST sentence
A. The company also plans a foray into the service sector
after rearrangement?
by setting up a chain of launderettes across the country.
(a) A (b) B
B. Yet today, pre-cooked chapatis and ready-made mixes
(c) C (d) D
are a big market.
(e) E
C. And that’s not all.
60. Which of the following should be the FOURTH sentence
D. Today the idea may appear a trifle ambitious but
after rearrangement?
remember that it was not so long ago that the same
(a) D (b) E
things were said about the market for ready-to-eat foods
(c) F (d) G
and branded cereals. (e) H

Downloaded From : www.EasyEngineering.net


Downloaded From : www.EasyEngineering.net

286  l Parajumbles

DIRECTIONS (61-65): Rearrange the given five sentences 6. Sometimes we don’t use words but make gestures (like
(A, B, C, D) and (E) in a proper sequence so as to form a traffic signs) or simple movements of the hand in order to
meaningful paragraph and then answer the given questions. communicate.
(a) PRSQ (b) QPRS
[IBPS Clerk 2013]
(c) PQRS (d) PSQR
A. With so many products and opportunities available in the 67. 1. Fires in the Steppes or bushes scared humans earlier.
market, it is very easy to get this planning wrong. P. Gradually, they learnt to appreciate the power of fire.
B. Planning, therefore, is imperative and should begin as early Q. It gave them light and warmth and kept away wild
as possible.
animals.
C. What amount will we need and when will we need it?
R. About 700,000 years ago, humans started fire
D. Most of us would put our children’s education above any
accidentally by lightning.
other priority in life including our own retirement.
E. So, let’s try to find the best solution by asking two important S. They could harden the tips of wooden spears and cook
question. meat in it.
61. Which of the following should be the SECOND sentence 6. Soon they learnt to produce fire by striking flintstone and
after rearrangement? pyrite with each other or by rubbing lumbers.
(a) D (b) B (a) QSPR (b) PQSR

(e) A ww
(c) C (d) E

62. Which of the following should be the FIFTH sentence after


(c) PQRS
68. 1. When the Impressionists
(d) QRSP

(c) C
w.E
rearrangement?
(a) A (b) B
(d) E
P. they made them look like
Q. everyday and often putting
R people you would see
S. painted pictures of people
(e) D
asy
63. Which of the following should be the FIRST sentence after
6. more emphasis on the scene.
(a) RQPS (b) SRQP
rearrangement?
(a) A
(c) E
(b) B
(d) C En (c) PRQS
69. 1. Sherlock Holmes is the
(d) SPRQ

(e) D
64. Which of the following should be the THIRD sentence after gin P. who is in a state of grace
Q. is raised to the status
R. because in him scientific curiosity
rearrangement?
(a) D
(c) C
(b) B
(d) E eer
S. exceptional individual
6. of a heroic passion.
(e) A
65. Which of the following should be the FOURTH sentence
(a) PRQS
(c) SPRQ ing (b) SRQP
(d) RPSQ
after rearrangement?
(a) A
(c) E
(b) B
(d) D
70. 1.The goals of our present system
P. schooling is to prepare
.ne
Q. students for the examination system
(e) C
DIRECTIONS (Qs. 66-71): The 1st and the last sentences/parts
of the passage / sentence are numbered 1 and 6. The rest of the
R. which will take them to the
S. of primary and secondary
6. best technical institutions in the country
(a) SPQR (b) QPRS
t
passage / sentence is split into four parts and named P, Q, R and
S. These four parts are not given in their proper order. Read the (c) PRQS (d) PSQR
passage/sentence and find out which of the four combinations is 71. 1. Egotism is the; most common fault of mankind.
correct. Then find the correct answer and indicate it by blackening P. However, with time it becomes an exaggerated form
the appropriate oval [ ] in the Answer Sheet. of self display.
[SSC CHSL 2013] Q. It is the product of a perfectly natural desire to display
66. 1. There are a lot of ways to communicate : speaking, oneself.
singing, clapping, hooting. R. This is necessary as it impairs the personality and
P. Even animals communicate with one another. frustrates all efforts at self improvement.
Q. Only humans can express their thoughts and feelings S. Beyond any shadow of doubt, it is a defect that ought
in words because of our superior brain. to be constantly hunted down, and scotched.
R. Both humans and animals also communicate through 6. One should always be on guard not to give into egotism.
body language. (a) RSPQ (b) PSRQ
S. But their ways differ from the humans. (c) PQRS (d) QPSR

Downloaded From : www.EasyEngineering.net


Downloaded From : www.EasyEngineering.net

Parajumbles  l 287

LEVEL-II
Directions (Qs 1–41): The sentences given in each question, C. Spirituality eliminates the disparity between appearance
when arranged in the proper sequence, form a coherent paragraph. and reality. Harmony, love and compassion are not
Each sentence is labelled with a letter/number. Choose the most the products of religion. They are spiritual products,
logical order of sentences from among the given choices to construct spiritual property.
a coherent paragraph. D. Religion strengthens the ego, because religion needs the
1. A. For if knowledge became too great for communication, individual to remain in the ego so that its strength and
it would degenerate into scholasticism, and the weak power can be maintained. We like those who enforce
acceptance of authority; mankind would slip into a new our ego and we dislike those who weaken it.
age of faith, worshipping at a respectful distance its new (a) CDBA (b) BCDA
priests. (c) CADB (d) dacb
B. The civilisation, which had hoped to raise itself up on 4. A. Gone are the days when parents were happy to see their
education disseminated far and wide, would be left two-year old frolic about in the company of sundry
precariously based upon a technical erudition that had aunts or cousins.
become the monopoly of an esoteric class monastically B. Parents are relieved to know that there are professional

ww
isolated from the world by the birth rate of terminology.
C. To find for new truths, old terms that all literate people
might understand.
people who can help them in the process. Leading
schools have a high regard for children who attend these

w.E
D. The function of the professional teacher was to mediate
between the specialist and the nation; to learn the
specialist’s language, as the specialist had learned the
preschools.
C. Welcoming these kids with open arms are a new breed
of women entrepreneurs who have cashed in on this
high parental anxiety to build a thriving business, that
knowledge and need.
(a) ABCD
asy
nature’s, in order to break down the barriers between

(b) BCDA
of preschools.
D. Now toddlers, barely out of their diapers are being
(c) CADB (d) DCAB
En
2. A. For too many people it is like a blind man in a dark room
pushed by overanxious parents to get cracking on
reading, writing, and arithmetic at the earliest.
looking for a black cat that is not there. The magazines
and mental health associations say psychiatric treatment
is a good thing, but what it is or what it accomplishes has
gin
5.
(a) ABCD
(c) CADB
(b) ADCB
(d) DACB
A. in this very short introduction to
not been made clear.
B. In recent years there have been many reports of a same eer
B. America’s still-influential legislative response to the

growing impatience with psychiatry, with its seeming


forevermore, its high costs, its debatable results, and its
vague, esoteric terms.
ing
C. I offer some basic ideas for a first understanding of this
profound crisis and
D. the Great Depression and the New Deal
C. Impatience has been expressed with increasing concern
not only by parents and the general public but by
(a) C, B, D, A
(c) A, D, C, B .ne
(b) A, B, C, D
(d) C, D, A, B
psychiatrists as well.
D. Although hundreds of thousand of words about
psychiatry are consumed by the public yearly, there has
been little convincing data to help a person in need of
6.
t
A. Once charged, the Prius can be driven for about 15
miles using electricity only.
B. The Toyota Prius, which for years dominated the hybrid
market in the UK, has some new rivals – and it’s no
treatment overcome the cartoon image of psychiatrists surprise to learn they are also made by Toyota.
and their mystical couches. C. After that it becomes a standard Prius hybrid in that the
(a) ABCD (b) BCDA 1.8-litre petrol engine drives the car and recharges the
(c) BADC (d) DACB battery.
3. A. If we are still on the ego level and no transformation has D. The company has just launched the plug-in Prius, which
been made, so there is a lack of development and we is a serious rival to the Vauxhall Ampera and one that is
could therefore never reach the level where one can love at least packed with proven technology.
the entire world. The family-love consciousness is of a (a) A, C, B, D (b) B, D, A, C
lower state than that of the global-love state. (c) B, A, C, D (d) D, B, A, C
B. The person who talks adversely about religion indirectly 7. A. The situations in which violence occurs and the nature
tries to weaken our ego; it hurts our religious feelings. of the violence tends to be clearly defined at least in
The strength of all religions lies in unaltered egos. The theory, as in the proverbial Irishman’s question: Is this a
love of a terrorist is a good example of religion-induced private fight or can anyone join in?
love and the love of Buddha is a good example of B. So the actual risk to outsiders, though no doubt higher
spiritually generated love. than our societies, is calculable.

Downloaded From : www.EasyEngineering.net


Downloaded From : www.EasyEngineering.net

288  l Parajumbles

C. Probably the only uncontrolled applications of force (a) ACBD (b) BCDA
are those of social superiors to social inferiors and even (c) BDAC (d) BACD
here there are probably some rules. 12. A. Marine life would suffer because a lid of warm water
D. However binding the obligation to kill, members of would prevent the circulation that normally brings
feuding families engaged in mutual massacre will be nutrients to the surface of the sea and the river waters.
genuinely appalled if by some mischance a bystander or B. But plant productivity would rise by 50 percent.
outsider is killed. C. A number of effects are expected from global warming.
(a) DABC (b) ACDB D. Also, the warmer climate could melt the floating ice
(c) CBAD (d) DBAC of the Arctic Ocean thereby resulting in a 20 ft. rise in
8. A. The likelihood of an accident is determined by how oceans and surrounding areas.
carefully the motorist drives and how carefully the (a) CDBA (b) BCDA
pedestrian crosses the street. (c) CABD (d) BADC
B. An accident involving a motorist and a pedestrian is 13. A. Hence, the educated and patriotic Indians must rise
such a case. to the occasion and blast the medieval thinking that
C. Each must decide how much care to exercise without promotes religious fanaticism.
knowing how careful the other is. B. The country has already enough religions to meddle in
D. The simplest strategic problem arises when two politics.

ww
individuals interact with each other and each must
decide what to do without knowing what the other is
doing.
C. Religion must be a guiding force to make the nation
strong.
D. However, it must not be allowed to be exploited for

w.E
(a) ABCD
(c) DBCA
(b) ADCB
(d) DBAC
9. A. In rejecting the functionalism in positivist organization
theory, either wholly or partially, there is often a move
garnering votes.
(a) BCAD
(c) BDAC
(b) CDBA
(d) ADBC

asy
towards a political model of organization theory.
B. Thus the analysis would shift to the power resources
14. A. Suggest or request rather than give orders.
B. A poor executive gives orders; he commands and
demands obedience.

En
possessed by different groups in the organization and
the way they use these resources in actual power plays
to shape the organizational structure.
C. Be a leader and not a boss.
D. A good one delegates wisely and counsels sanely.
C. At the extreme, in one set of writings, the growth
of administrators in the organization is held to be
completely unrelated to the work to be done and to be
gin (a) ABCD
(c) BDAC
(b) DCAB
(d) CBDA
15. A. The sickest of such men is least aware of them.
caused totally by the political pursuit of self-interest.
D. The political model holds that individual interests are eer
B. We find that what we thought was a cold or a sprain was
in fact gout.
C. However the diseases of the soul grow more obscure
pursued in organizational life through the exercise of
power and influence.
(a) ADBC (b) CBAD ing
even as they grow stronger.
D. The diseases of the body become more pronounced as
(c) DBCA (d) ABDC
10. A. Group decision making, however, does not necessarily
they grow.
(a) DCBA
(c) CBAD .ne
(b) BADC
(d) DBCA
fully guard against arbitrariness and anarchy, for
individual capriciousness can get substituted by
collusion of group members.
B. Nature itself is an intricate system of checks and balances,
meant to preserve the delicate balance between various
t
16. A. I feared however that some people might not understand
why I chose the name I did.
B. I have decided to ask him to write out my studio's name
in large characters that I will display on a pillar.
environmental factors that affect our ecology. C. His writing is quite unusual and imposing.
C. In institutions also, there is a need to have in place a D. My friend Is’ai Hsiang excels at large-style calligraphy.
system of checks and balances which inhibits the (a) DCBA (b) ABCD
concentration of power in only some individuals. (c) DBAC (d) DABC
D. When human interventions alter this delicate balance, 17. A. A carriage passes with a nasty, creaking noise.
the outcomes have been seen to be disastrous. B. If I am travelling in someone’s carriage, I dislike not only
(a) CDAB (b) BCAD the noise but also the owner.
(c) CABD (d) BDCA C. Annoying to think that the passengers may not be aware
11. A.  The operation of last week was as dramatic as the of it.
conceptualization of the gigantic statue of 350 tons. D. Such a person definitely is hateful.
B. The giant monolithic statue of Buddha was salvaged
(a) DCBA (b) ABCD
from the Hussainsagar lake.
(c) ACBD (d) BADC
C. The project was the brainchild and magnificent
obsession of late N.T. Rama Rao. 18. A. His influence over Nero was, at least initially, salutary.
D. It went down in the lake in 1990, when it was being B. He killed his mother and his brother and sent Seneca to
transported to the Rock of Gibraltar, its intended seat. exile.

Downloaded From : www.EasyEngineering.net


Downloaded From : www.EasyEngineering.net

Parajumbles  l 289

C. Seneca was a true follower of stoic philosophy. (a) AECBD (b) DECBA
D. But, eventually, Nero’s cruel propensities revealed (c) ACBED (d) DBCAE
themselves. 23. A. Both parties use capital and labour in the struggle to
(a) CADB (b) DCBA secure property rights.
(c) ABCD (d) BDCA B. The thief spends time and money in his attempt to steal
19. A. The ovules are arranged in rows. (he buys wire cutters) and the legitimate property owner
B. Each filament grows from a germ on an ovule. expends resources to prevent the theft (he buys locks)
C. Each will produce a seed. C. A social cost of theft is that both the thief and the
D. This seed is called a kernel. potential victim use resources to gain or maintain
(a) ACBD (b) BACD control over property.
(c) ABDC (d) DABC D. These costs may escalate as a type of technological arms
20. A. Since then, intelligence tests have been mostly used to race unfolds.
separate dull children in school from average or bright E. A bank may purchase more and more complicated and
children, so that special education can be provided to sophisticated safes, forcing safecrackers to invest further
the dull. in safecracking equipment.
B. In other words, intelligence tests give us a norm for each (a) ABCDE (b) CABDE

ww
age.
C. Intelligence is expressed as intelligence quotient and
(c) ACBED (d) CBEDA
24. A. Each year we still have to choose some mix of output that
is consistent with our existing production possibilities.

w.E
tests are developed to indicate what an average child of
a certain age can do–what a 5-year-old can answer, but a
4-year-old cannot, for instance.
B. Choosing what to produce – a mix of output – is one of
our most important economic decisions.
C. However promising the prospects for growth may be,

asy
D. Binet developed the first set of such tests in the early
1900s to find out which children in school needed
special attention.
we still have to live within our current production
constraints.
D. The fact that those limits may expand in future years
E. Intelligence can be measured by tests.
(a) CDABE (b) DECAB En does not make our current choices any easier.
E. There is still a limit to how much we can produce in any
(c) EDACB (d) CBADE
21. A. This very insatiability of the photographing eye changes
the terms of confinement in the cave, our world.
gin year.
(a) ABCDE (b) CEDAB

B. Humankind lingers unregenerately in Plato's cave, still


revelling its age-old habit, in mere images of truth.

(c) DCBEA
25. A. 
eer (d) ADCBE
The market mechanism gives undue weight to the
desires of the rich.
C. But being educated by photographs is not like being
educated by older images drawn by hand; for one thing, ing
B. Central planning creates the opportunity to direct
resources to the society’s most pressing needs, without
there are a great many more images around, claiming
our attention.
D. The inventory started in 1839 and since then just about .ne
the distractions of conspicuous consumption.
C. As a result, a market economy may produce lots of
frivolous goods while neglecting greater social needs.
everything has been photographed, or so it seems.
E. In teaching us a new visual code, photographs alter and
enlarge our notions of what is worth looking at and
D. The motivation for command economies is the
t
conviction that central planning is more likely to
produce the “right” mix of output than a decentralized
market mechanism.
what we have a right to observe.
(a) EABCD (b) BDEAC E. Although the goals of central planning may be worthy,
(c) BCDAE (d) ECDAB their implementation is fraught with difficulty.
(a) ABCDE (b) EDCBA
22. A. To be culturally literate is to possess the basic information
(c) DACBE (d) CBDAE
needed to thrive in the modern world.
26. A.  Nevertheless, we do think that certain aesthetic,
B. Nor is it confined to one social class; quite the contrary.
evaluative conceptions do relate to specific experiences
C. It is by no means confined to "culture" narrowly
in a non-trivial way, especially that of aesthetic
understood as an acquaintance with the arts. excellence.
D. Cultural literacy constitutes the only sure avenue of B. This is so because, typically, we think that the experience
opportunity for disadvantaged children, the only of beauty is such that we cannot leave it to others to be
reliable way of combating the social determinism that had.
now condemns them. C. It is rather intriguing that we will often try to persuade
E. The breadth of that information is great, extending over people of what we find beautiful, even though we do not
the major domains of human activity from sports to believe that they may subsequently base their judgment
science. of taste on our testimony.

Downloaded From : www.EasyEngineering.net


Downloaded From : www.EasyEngineering.net

290  l Parajumbles

D. Moreover, we are often aware of the contingency of our of the border more than our men in flannel enmeshed in
own judgments' foundation in our own experience. a contest that is telecast live to just about every corner on
E. Now the discussion within analytical aesthetics the subcontinent and to the entire Indian and Pakistani
concerning the question of what kinds of truth- values diasporas the world over.
adhere to aesthetic judgments of various kinds has D. A battle on the pitch is always more welcome than
evident bearing on the problem of aesthetic experience's a pitched battle when it comes to India and Pakistan.
relevance for evaluation. Little wonder then that the subcontinent is agog with
(a) CEBAD (b) CEDAB the current cricketing encounter that entails.
(c) EADCB (d) CBDAE E. It is hoped that these moves will formalise trade
27. A.  Enter the virtual assistants who are entrepreneurial between the two neighbours, which is at present largely
partners - highly skilled in their profession and able to
floundering along clandestine channels routed through
have an impact on the productivity of those they work
third countries.
with.
(a) ABCDE (b) BCDAE
B. But most of the small businesses started today will reach
(c) DCABE (d) DACBE
an impasse very quickly - they will be spending so much
time on administrative tasks that they can no longer 30. A. Anyone who feels threatened – be they businessmen,
concentrate on growing their business. politicians and even gang lords – seem to find them

ww
C. On the contrary, some believe that increasing numbers
of small businesses will afford administrative support
useful.
B. Propelled by the rise in the number of automobiles and
the increase in the use of glass in buildings, the market

w.E
experts with entrepreneurial spirit opportunities that
have never before been possible.
D. Traditionally, the need for assistance has left the small
business owner with several bleak options- hire an
of the films is growing. The films are increasingly
being used in banks, consulates and VIP vehicles. The
preference for polyester films remain high because of

asy
expensive “temp” for a band-aid-style solution, take
on a great deal of expense and responsibility with a
the low costs.
C. Films are readily available and when the job on hand is
“permanent” employee, or-perhaps worst of all-turn
away the work.
E. Corporate downsizing and the move towards small, En urgent, fixing the film on plain glass is the solution for
business can be up and running in a short period. In the
case of decorative etchings also, films provide instant
home-based businesses could appear to be bad news
for the over 3 million people whose expertise lies in the gin solutions. They are easy to maintain.
D. They can be made to reject heat and resist impact.
administrative support arena.
(a) DCABE
(c) DBACE
(b) ECBDA
(d) EDCBA eer
The performance of films used as security or safety to
prevent splintering and avert sharpnel injuries depends

28. A. In art, essentialism is the idea that certain concepts may
be expressed organically in certain media. ing
on thickness.
E. Polyester films intrinsically block ultraviolet (UV)
solar radiations and so find use in shops, homes and
B. Each medium has its own particular strengths and
weaknesses, contingent on its mode of communication.
C. This idea may be further refined and it may be said .ne
automobiles to prevent fading of upholstery and
furnishings. Applied on glass, they reject glare and
that the haiku is a poor vehicle for describing a lover’s
affection as opposed to the more organically correct
sonnet.
D. Essentialism is attractive to artists because it not only
ultraviolet rays.
(a) ABCDE
(c) CADBE
(b) EDABC
(d) DACBE t
31. A. The strength of the Indian handicrafts lies in its low
delineates the role of art and media but also prescribes a capital investments, abundant skilled manpower,
method for evaluating art. negligible import content and high export potential.
E. A chase scene may be appropriate for motion pictures, B. The other strength of the industry is the rich “hidden
but poorly realised in poetry because the essential treasures’ that are to be found across India.
components of the poetic medium are ill suited to C. India’s handicraft has been renowned from time
convey the information of a chase scene. immemorial for their intricate workmanship and
(a) DCEBA (b) BDACE exclusive designs. The handicrafts sector has emerged as
(c) DABEC (d) ABECD one of the most important foreign exchange earners for
29. A.  While this path breaking event has attracted India on a sustained basis.
international media and public attention, apart from D. Hand-printed textiles, shawls, zari goods, imitation
the inevitable punters and bookies, it rouses hopes of jewellery, embroidered and crocheted goods are
deepening people-to-people contact as well. exported to the US, UK, Germany, Canada, Japan and
B. It is this aspect that can prove to be a firm bedrock for Saudi Arabia. Special promotions in the Latin American
building up relations. countries, South Africa and new destinations in EU
C. There is little else that energises the people on both sides countries are being targeted.

Downloaded From : www.EasyEngineering.net


Downloaded From : www.EasyEngineering.net

Parajumbles  l 291

E. The progress in terms of product range, number of 35. A. In a 2003 driving poll conducted by The National Safe
companies and value of exports has been tremendous. Driving Test, 91% of all drivers admitted to engaging in
Indian handicrafts are exported mainly to the American risky driving behaviour over a six-month span.
and European continents and the Far East. B. So what’s wrong with him?
(a) CEDAB (b) BCDAE C. That’s what’s wrong with him.
(c) CADBE (d) DACBE D. He’s human, and he thinks he’s a great driver.
32. A. How coffee really came to India is still a mystery, but E. But don’t saddle up that high horse; you’re probably just
some say it first came in 1600. According to a legend it as bad yourself.
was Baba Budan, a pilgrim, who brought seven coffee (a) BDCEA (b) BCDAE
seeds from Yemen. These Mokka seeds were planted in (c) AECBD (d) ACBED
the hills of Chandragiri in Karnataka’s Chikamangular 36. A. Stop criticising other people.
district. B. Eliminate negative thought patterns and speech habits.
B. According to ‘The Economist’ magazine, coffee fuelled C. The word “should” can sap your energy if you use it to
chastise or blame yourself, so get rid of it.
the information exchanges of the 17th and the 18th
D. Negative thoughts cloud your judgment.
centuries.
E. Banishing words like “can’t,” “won’t” and “shouldn’t”
C. Though it was initially received with suspicion, with
from your vocabulary will have an immediate impact

ww
many calling coffee Satan’s drink, it did not take much
time for the brew to become a favourite drink all over
the world.
on people’s impression of you.
(a) BCAED
(c) ABCDE
(b) CDEBA
(d) BECDA

w.E
D. The popularity of the brew grew immensely.
E. The credit for commercialising the use of coffee goes to
the British. The British were responsible for the spread
37. A. Some say that power is the ability to exert influence
beyond those one actually controls.
B. All of these definitions can be grouped together under

in the span of a few centuries.


(a) ABCDE
asy
of the coffee bean from Arabia to Asia Minor to Europe

(b) BCAED
one broader one: Power is the ability to exert influence
— whether on an individual or an organisation – to
obtain a desired outcome.
(c) BADCE (d) DACBE
En
33. A. Okay, sainthood is probably harder, but not by a lot.
C. Power means different things to different people.
D. Others maintain that power boils down to the ability to
B. Excess bulk takes a lot of energy to maintain — up to
50 calories a day per extra pound of muscle mass you gin get what one wants.
E. Considering that we need income to obtain the things
add — and thousands of generations of evolution have
taught your body that that’s a waste of perfectly good
nuts, berries, and animal flesh. eer
that we need or want, power could also be loosely
defined as the ability to provide employment in order to
derive an income.
C. Your body is an extremely efficient machine.
D. Building muscle mass is one of the toughest tasks you
(a) ADBEC
(c) CDEAB ing
38. A. They are right.
(b) CADEB
(d) CBAED
can attempt.
E. It retains the amount of muscle mass that is required to
comfortably perform your daily routine — no more and hand in hand. .ne
B. It goes without saying that friends and money don’t go

no less.
(a) DECBA
(c) ABCDE
(b) DACEB
(d) EBADC
t
C. When you are talking about money or a favour with
some kind of financial value, it never ends well.
D. They say a favour will kill you faster than a bullet.
E. I don’t care how chummy you are, how close you are, or
34. A. He became good friends with Bill Gates and this led to a
how many hookers you’ve shared with a friend. When
historical business partnership a few years later. it comes to money, friendships are always put aside and
B. He basically headed every division within the company, ego comes into play.
eventually becoming president in 1998. (a) BDEAC (b) ACBDE
C. After working for Procter & Gamble for two years, (c) BECDA (d) BADEC
Ballmer attended Stanford Graduate School of Business 39. A. For he married for the fourth time when he was over
for one year. forty.
D. This second cousin of famous comedian Gilda Radner B. His loyalty to the State was well known.
embarked on his fateful path when he attended Harvard C. But he was incorruptible and had earned a name for
University in the 70s. strict impartiality in his family as well as outside.
E. He then joined Microsoft in 1980 as a business manager D. My father was a lover of his clan, truthful, brave and
and started to remould the corporate structure and generous, but short-tempered.
drive the company to profitability. E. To a certain extent he might have been given to carnal
(a) ECDAB (b) DBACE pleasures.
(c) DACEB (d) DCBEA (a) BDCAE (b) DBCAE
(c) DEACB (d) BCDEA

Downloaded From : www.EasyEngineering.net


Downloaded From : www.EasyEngineering.net

292  l Parajumbles

40. A. You can get the ultimate fitted suit and invest in a Brioni, D. You can see why a book might be a good way to tackle
for example, but if you’d prefer to save that money for a the issue of plastic surgery with kids, since how do you
new car, there are more affordable alternatives. do it on your own?
B. Suits are the basic component of the corporate uniform. E. Reconstructive surgery, you might call it, after the
C. But conservative doesn’t have to mean boring. natural disaster - sorry, precious and awe-inspiring
D. Donning a stylish suit conjures up a mature and miracle - that is having a child. And the beauty is, if
conservative look. you have a tummy tuck and breast implants in one hit,
E. A pinstripe or subtle pattern can spice things up, and you get a bonanza two-for-one offer on the therapy
will stand out in a sea of solid black, charcoal and navy. your child might need in the future. Please don’t try to
(a) EDACB (b) EDCBA redeem this offer, I am just being supercilious.
(c) BCDEA (d) BDCEA (a) BCED (b) DCBE
41. A. In 1789, the latter joined with the needy masses to defeat (c) BEDC (d) EBDC
king, barons, and clergy. 44. A. Given the atrocities perpetrated with impunity by state
B. The French Revolution, mainly a political revolt, broke forces, it is a moral imperative that the negotiations —
the power of the feudal order. scheduled to resume at the United Nations this month
C. They marched to triumph over lordly sloth, entail, — on a comprehensive treaty to regulate the sale of
privilege, superstition. France set creeds of the conventional arms should succeed.

ww
Enlightenment into its constitutions.
D. No longer could a noble class hinder the growth of
B. Its recent report calls on countries to codify the so-
called “golden rule” — not to allow the transfer of arms

w.E
capital and the bourgeoisie (middle classes).
E. The middle ranks then took the lead in a new social
order.
(a) BDACE (b) BDAEC
to states where there is a threat of grave abuses of human
rights and humanitarian law.
C. Every year, over 300,000 people are killed by
conventional weapons and millions injured, forcibly
(c) DBAEC
asy
(d) DACEB
DIRECTIONS (Qs 42-44): In each question, there are five
displaced, and bereaved because of armed violence,
according to Amnesty International.

En
sentences. The sentence labelled A is in its correct place. The four
that follow are labelled B, C, D and E, and need to be arranged
D. The consensus that has emerged, since the 2006
Resolution for a global pact, on underwriting provisions
related to protection of human rights and fundamental
in the logical order to form a coherent paragraph. From the given
options, choose the most appropriate option.
42. A. According to materialists, the universe was eternal, and gin freedoms in the proposed treaty is also a tacit
recognition of the brutalities committed systematically
therefore, there was no purpose or special creation in it.
B. Claiming that everything came into being as the eer
against innocent civilians in the conflict zones.
E. However, opposition by the United States, Russia, China
and India, besides others, to link the trade in arms to the
result of unconscious atoms assembling at random,
they believed that no matter how much complexity,
balance and magnificent regularity was exhibited by the ing
observance of human rights and humanitarian law by
recipient countries threatens to block progress.
external world, these were still the result of purposeless
coincidences.
(a) DECB
(c) EDBC
.ne
(b) DEBC
(d) ECBD
DIRECTION (Qs. 45–65):  Arrange the sentences A, B, C and D
C. Materialists imagined that all the balances, equilibrium,
harmony and order in the universe were solely the
results of chance.
D. Since materialism rejected the concepts of purpose and
to form a logical sequence between sentences 1 and 6.
45. 1. There are gangsters, and there are 1970s gangsters.t
A. As late as 1967, in Jean-Pierre Melville’s Le Samouraï,
creation to the universe, it also denied the existence of a Alain Delon floated through in a raincoat that looked
Creator. like it had been in storage since before Vichy.
E. Materialist minds had held such preconceptions ever B. We’ve given our hearts to mobsters in Tony Manero-
since the days of Ancient Greece. style flared suits, with shirt collars outside the lapels, big
(a) CBED (b) EBDC hair, proudly unwaxed chest hair, and those dick-duster
(c) DBEC (d) DCEB ‘taches’.
43. A. It’s called My Beautiful Mommy, and it’s a children’s book C. The French, whose homegrown gangster tradition is
published in the US next week. That all sounds regular second only to Hollywood’s, favoured the trenchcoat
and above board, you think, until I tell you why mommy look long after it was obsolete for Humphrey Bogart.
is so beautiful - it’s because she has had extensive plastic D. We were enamoured of gangsters with trenchcoats, felt
surgery. hats, spats and gats.
B. What do I know of the ways in which one might mess up 6. Mesrine: Killer Instinct - the headlong, two-part account
one’s child? Nothing at all. of France’s “Ennemi Publique Numero 1” of the 1970s -
C. But some might say the book, by Florida plastic surgeon proves both that the French can do flares with flair.
Dr Michael Salzhauer, and written for four to seven year (a) CDBA (b) ADCB
olds, is a step too far. (c) BDAC (d) DBCA

Downloaded From : www.EasyEngineering.net


Downloaded From : www.EasyEngineering.net

Parajumbles  l 293

46. 1.  Those who are neither average nor part of the fighting but by displaying superior military prowess?
cultural elite develop their own realm, the Lacerated D. How else could the organization throw its weight around
Consciousness of the bohemian. but by dispatching troops to a troubled region?
A. Where the state is irrational, the bohemian is quick to be 6. The MSC was further mandated to assist the Security
rational Council in arms regulation.
B. The bohemian poet uses the tools of culture against (a) ACDB (b) ADCB
culture. (c) CDBA (d) DCBA
C. Where the state is rational, the bohemian toys with the 50. 1. An attempt upon a crowned head or a president is
irrational. sensational enough in a way, but not so much as it used
D. These social critics, these talented drop-outs, possess a to be.
wit which sometimes achieves art. A. Horrible enough at first sight no doubt, and yet not so
6. Joining domestic morality and working class morality, effective as a person of ordinary mind might think.
the bohemian reminds the State of its evident failures. B. It has entered into the general conception of the
(a) ABCD (b) BACD existence of all chiefs of state.
(c) BCAD (d) DBCA C. No matter how revolutionary and anarchist in inception,
47. 1. I agree completely that we must cut the deficit and the there would be fools enough to give such an outrage the
national debt; frankly, I do not know anyone who thinks character of a religious manifestation.

ww
otherwise.
A. The desire to stop spending on security and start spending
D. Now let us take an outrage upon – say – a church.
6. And that would detract from the especial alarming

w.E
on domestic programs is natural, understandable and
unfortunately ill advised.
B. America has tried to reap a “peace dividend” several
times in the past.
significance we wish to give to the act.
(a) DABC
(c) BADC
51. 1. 
(b) BDAC
(d) ADCB
A nation will territorially encompass a number of

asy
C. That said, the idea that you can “get” the funds needed to
remedy the problem from the defence budget does not
different localities.
A. While the spatially smaller village, city, and region
make any sense.
D. It is more so when we haven’t yet achieved peace.
En
6. The threat landscape is actually worse today than it was
continue to exist, they are understood by their
inhabitants to be parts of the nation.
B. Because the nation exhibits only a relative cultural
on Sept. 11, 2001.
(a) ADCB (b) CADB gin uniformity, it is often difficult to distinguish it from
other territorial societies.
(c) CBAD (d) BADC
48. 1. But neither the ‘nowhere man’ nor the cosmic exile is a
real possibility. eer
C. However, during periods of intense patriotic enthusiasm,
such as during a war, the attachments of the inhabitants
of the local village, city, or region to the nation may
A. For the nowhere man, the common core of human
consciousness which is his only resource, is too meagre ing
become dominant; but such a situation can only be
episodic.
for it to generate a vision for him.
B. To think otherwise is to be self-deceived.
C. Deprived of these concepts and other comparable .ne
D. Thus, the common culture of the nation is only relative;
it is rarely complete such that the inhabitants of the
village, city, and region within the nation cease to
concepts, the nowhere man fails to form any vision at all
and, therefore, is incapable of making any judgments.
D. The candidates for culture-free concepts mentioned in
the definition of culture are in fact saturated in culture
t
recognize themselves as inhabitants of such localities.
6. It is tempting to avoid this difficulty by formulating
categories that are differentiated by degrees of cultural
uniformity, thereby distinguishing one form of
and are, therefore, linked to a point of view, whatever territorial relation from another.
the nature of this link may eventually turn out to be. (a) ADCB (b) ACDB
6. About the cosmic exile, I quote Gellner again: ‘It is not (c) ADBC (d) BADC
possible for us to carry out a total conceptual strip-tease 52. 1. When we read, another person thinks for us . We merely
and face bare data in total nudity.’ repeat his thoughts.
(a) ADCB (b) BDCA A. Just as man is the tool-making animal, the animal that
(c) BCDA (d) BADC cooks, the political animal, so too he is to us the reading
49. 1. In order to prevent wars and stop the ones that did animal, the creature by means of reading can bring
erupt, the UN needed a military capacity. himself to some degree of happiness and wisdom.
A. It set up the Military Staff Committee (MSC) as a B. In spite of the sneers of so many writers, we hold that
subsidiary body of the Security Council and charged it man rarely so innocent and fruitfully employed as he
with the planning of UN military operations. is when reading. That is why it relieves us to take up a
B. The UN Charter addressed these questions. book after being occupied with our own thoughts.
C. How else could the UN force warring parties—unwilling C. A book is a machine to think with. In learning to write
to yield to diplomatic or economic pressure—to cease the pupil does with his pen what the teacher has outlined
in pencil.

Downloaded From : www.EasyEngineering.net


Downloaded From : www.EasyEngineering.net

294  l Parajumbles

D. In the same way in reading the greater part of the work, B. The Pravda of the American government!
if thought is already done for us. To us the books are a C. During the entire anti-Osama campaign in was obvious
means of escape. that along with the twin towers of New York, the twin
6. So it comes about that if anyone spends the whole day pillars of the American democracy vis-à-vis the freedom
in reading, he gradually loses the capacity for thinking, of speech and free market both came crashing down.
just as the man who always rides at last forgets how to D. In fact, the capitalist-owned media blatantly printed
walk. press releases of the Pentagon as news items dictatorially
(a) CDAB (b) BCDA censored any opinion, which had a protesting tone.
(c) CADB (d) DACB 6. So much for freedom of speech and the illusion of
53. 1. One of the earliest and perhaps the most disturbing
democracy.
thing a human child has to learn is that the course of
(a) CDAB (b) BCAD
human life follows the clock–it can be predicted in
(c) BADC (d) CADB
advance
A. We take the course of human life for granted because we 56. 1. If all this sounds like the makings of a paperback thriller,
are familiar with it, just as we take for granted that we that’s hardly a coincidence.
have a characteristic size. A. But to Africa watchers the tale of treachery is closer to
B. Man is born helpless, like a kitten . This prolonged Danielle Steel, revolving around an ugly family feud

ww
dependence on parents has important psychological
consequences .
C. We plan our lives in terms of this fixed pattern, just as
raging within Obiang’s place.
B. Like Thatcher, Archer has denied involvement.
C. At stake is who will succeed the President after 25 years

w.E
we plan our houses; so that the few people; who are
more than 20% taller or shorter than usual, find them
difficult to live in.
of iron rule.
D. One of the alleged coup financers was thriller author
and former British politician Jeffrey Archer.

asy
D. When he is twenty, he will grow up, when he is eighty he
will be old, and before he is hundred he will die. Human
beings do not normally become able to breed until at
6. “These guys thought it was ripe for a coup, because there
was nonstop squabbling in the family”, says Vasset.
(a) DBAC (b) ACDB
least thirteen
En
6. I suppose the main reason this fixed programme affects
(c) CBDA (d) ABCD
57. 1. How do you find out what the solution to an equation
us emotionally is that it means that we have a fixed life
span. What psychologists call ‘complexes’ are formed
during this period. gin looks like when you are not solving it?
A. The mathematicians discovered that it is possible to
visualise the solution, even if you cannot write down a
(a) ABCD
(c) CADB
(b) BCDA
(d) DABC
54. 1. Seldom does an ambitious dream enjoy smooth sailing
eer
formula.
B. Although Poincare made little progress on chaos
all the way to completion.
A. In spite of free supplies and volunteer labour, money ing
beyond the realisation that it can occur, he introduced a
powerful new method for studying dynamic systems.
C. The idea gained serious currency in the astronomical
was an over-pressing concern.
B. The thousands of well-wishers who waited on the
dock in San Francisco to wave goodbye to HOPE’s first .ne
investigations of Poincare and it led him to one of the
first discoveries of what we now call chaos.
medical team were denied the spectacle.
C. Dr. Walsh was approached by an executive of the
company that was going to operate the Hope on its
maiden voyage.
D. Obviously you have to approach the whole problem
from some other direction. t
6. It was based upon a new kind of geometry, topology: the
qualitative geometry of the continuous.
D. In its fourteen years of bringing health care and (a) ABCD (b) DCAB
education to people around the world, the SS Hope (c) DACB (d) DCBA
endured many stormy seas, and a few had nothing to do 58. 1. Why are horses the same?
with the weather. A. It may be old and lame and in time it will die.
6. The man informed Walsh that the HOPE organisation B. A particular horse ‘flows’ naturally.
had no line of credit and an advance of $500,000 were C. But there is something which is common in all horses.
necessary. D. You probably don’t think they are at all.
(a) DABC (b) BDAC
6. But the ‘form’ of the horse is eternal and immutable.
(c) CDBA (d) DABC
(a) DCAB (b) DCBA
55. 1. The man who doesn’t want a war (Saddam) is termed as
(c) DBCA (d) BCDA
a threat to the world, while the man who is displaying
his cowboy brashness and raising war cries at the top of 59. 1. Sports broadcasters often have only a shaky grip on
his voice is projected as the one who wants peace! grammar and on the connection between words and
A. While the NASDAQ crashed, the government meaning.
demonstrated its respect for freedom of speech by A. This sort of thing is by no means confined to the sports
openly manipulating the print and electronic media. world.

Downloaded From : www.EasyEngineering.net


Downloaded From : www.EasyEngineering.net

Parajumbles  l 295

B. They have become confused in some journalistic 63. 1. As with the social system as a whole, the game has a life
minds with intended victims, but intended victims of its own.
are sometimes rendered as would-be victims, who A. The result is a group of people who, as the leagues become
apparently go out in the hope of being robbed. of a higher and higher class, are progressively insensitive
C. For example, we have all heard about alleged victims. to the possibility that things could be otherwise.
D. During one football game, the announcer told viewers B. Everyone grows up inside it, accepts it and fulfills its
that because of the way some of the boxes in the dictates as obediently as Helots.
Superdome were placed, he could not visually see them. C. Thus, in football, anyone who might question the
6. An ironic thing is happening now. wisdom or enjoyment of putting on heavy equipment
(a) BACD (b) CDBA on a hot day, would be regarded as not really a devoted
(c) DACB (d) CDAB athlete and probably “chicken”.
60. 1. Today, India is in this sad condition because in the past D. Far from questioning the principles of the activity, most
the children seem to have been neglected. men simply concentrate on executing these principles
A. There seem to be no intellectuals in our country and a more aggressively than anybody else.
country without intellectuals has no future. 6. The choice is made straightforward.
B. In our country today, however, wherever I look, I only (a) BDAC (b) CBDA

ww
see ‘animal’ human beings, extremely selfish, never
caring for other people or having a vision for the
country, love for the nation or love for the people.
(c) ABDC (d) BDCA
64. 1. Nevertheless, what is innovation?
A. Is innovation an ongoing process?

w.E
C. In the past I feel they must have been neglected because
of the texture of the people I see around me today.
D. All the progressive countries in the world are such
B. How do Indian companies achieve a grasp of it?
C. Should companies strive for breakthrough developments,
or should they focus on continuous improvement?
D. Many believe that the latter is the key to sustained

asy
because their progress is guided by, decided by, thought
of and planned by the intellectuals of the generation.
6. These higher ideals have all been dried up.
growth.
6. Nestle, for instance, is an unabashed practitioner of
(a) CADB
(c) CBDA
(b) CDAB
(d) CABD En continuous improvements in small measures and is
quite cynical about revolutionary breakthroughs.

61. 1. Among those savages, Herman learned that primitive


people, left to their own way of life, may be more happy gin (a) CABD
(c) ACBD
(b) BCDA
(d) BACD
65. 1. Status of men and women is equal in the eyes of law.
and good-humoured than those who have been afflicted,
so to speak, with civilisation.
A. “One peculiarity that fixed my admiration” Herman eer
A. Sexual harassment at work is one of the worst attacks on
a woman’s right of equal opportunities especially in the
developing countries and in countries where customary
wrote, “was the perpetual hilarity reigning through the
whole vale”. ing
laws hold powerful sway.
B. But millions of women work with low wages, low status,
B. This experience shaped Melville’s views of human nature
and wisdom of life in a deeply revolutionary way.
C. Blue devils, hypochondria and doleful dumps went and
age.
.ne
with no proper prospect of pension or care in their old

C. The principal reason why our race has become much


hid themselves among the nooks and crannies of the
rocks.
D. There seemed to be no cares, griefs, troubles or vexations.
6. It kindled a revolt against the decorous piety of New
images of Shakti. t
degenerated is that we have no respect for these living

D. In some countries like UK, too, women in education,


management, law, medicine, etc., are all too often paid
England folkways; it lifted him out of the mainstream of notably less salary than men, for work of similar nature.
Victorian culture. 6. Many say where women are respected, God is delighted
(a) ADCB (b) ACBD and where they are not, all the efforts and work go in
(c) ABCD (d) CBAD vain.
62. 1. You must recognise your rights and stand up for them. (a) BDCA (b) BCDA
A. You lose freedom of movement. (c) BACD (d) BCAD
B. If you do not, other people define your role for you, and 66. 1. A few years ago, hostility towards Japanese-Americans
you stop being yourself. was so strong that I thought they were going to reopen
C. You become the “encapsulated person” and pay the price the detention camps here in Kolkata.
for this with neurotic behaviour. A. Today Asians are a success story.
D. Your life space decreases. B. I cannot help making a comparison to the anti-Jewish
6. Many people have this difficulty because they don’t have sentiment in Nazi Germany when Jewish people were
a clear cut idea of what their rights are. successful in business.
(a) BADC (b) BACD C. But do people applaud President Clinton for improving
(c) BDAC (d) BCDA foreign trade with Asia?

Downloaded From : www.EasyEngineering.net


Downloaded From : www.EasyEngineering.net

296  l Parajumbles

D. Now, talk about the ‘Arkansas-Asia Connection’ is (3) In most of these books, the authors classify a measurement
broadening that hatred to include all Asian-Americans. strategy as either norm-referenced (NRM) or criterion-
6. No, blinded by jealousy, they complain that it is the referenced (CRM).
Asian-Americans who are reaping the wealth. (4) Another author points out how the type of interpretation that
(a) DBAC (b) ABDC an NRM offers is the relative performance of the students
(c) DABC (d) ACBD compared with that of all the others resulting in, ideally, a
bell curve distribution.
DIRECTIONS (Qs. 67-68): In each question, there are five (5) Numerous books on constructing and using language tests
sentences. The sentence labelled A is in its correct place. The four have been written by various authors.
that follow are labelled B, C, D and E, and need to be arranged (6) CRMs, on the other hand, are more specific, achievement or
in the logical order to form a coherent paragraph. From the given diagnostic tests intended to be used for motivating students
options, choose the most appropriate option. by measuring to what percent they have achieved mastery
67. A. The government was claiming that the city’s air was of the thought or learned material.
cleaner for the Olympics than it had been in a decade. (7) One of the authors clearly delineates the differences of
B. But stench from a waste-disposal plant was smothering these two types by focusing on the categories of “test
their homes. characteristics” and “logistical dimensions.”
C. After a lull, news of protests around China about all

ww
sorts of issues is again trickling out.
D. Freed from Olympic constraints, they felt it was time to
protest.
69. Which of the following should be the FIRST sentence after
rearrangement?
(a) 7 (b) 2

w.E
E. They were not alone.
(a) BEDC
(c) BDEC
(b) CDEB
(d) DEBC
(c) 3
(e) 5
(d) 4

70. Which of the following should be the SEVENTH (LAST)


68. A. 
asy
The paintings, sculptures, and balloons of Takashi
Murakami are colourful and attractive, and accessible in
their reference to lovable cartoon characters.
sentence after rearrangement?
(a) 1
(c) 3
(b) 2
(d) 4
B. Not stopping with the production of artworks,
Murakami shocked the world with his entrepreneurialEn (e) 5
71. Which of the following should be the FIFTH sentence after
collaboration with Louis Vuitton, when he challenged
the divide between art and commerce. gin rearrangement?
(a) 1 (b) 2
C. As a curator, Murakami challenges our notions of
history and culture.
D. Murakami uses his deep understanding of Western art
(c) 3
(e) 5
eer (d) 6

72. Which of the following should be the SECOND sentence


to integrate his work into its structure; working from the
inside to portray “Japanese-ness” as a tool to bring about (a) 1 ing
after rearrangement?
(b) 2
revolution in the world of art.
E. As an artist, Murakami questions the lines drawn
between East and West, past and present, high art and
(c) 3
(e) 6
.ne
(d) 4

73. Which of the following should be the THIRD sentence after


popular culture.
(a) BDEC
(c) CEBD
(b) CBDE
(d) DEBC
rearrangement?
(a) 1
(c) 7
(b) 2
(d) 4
t
DIRECTIONS (Qs. 69-73) : Rearrange the following seven (e) 5
sentences (1), (2), (3), (4), (5), (6) and (7) in the proper sequence DIRECTIONS (Qs. 74-78) : Rearrange the following six
to from a meaningful paragraph; then answer the questions given sentences (A), (B), (C), (D), (E) and (F) in the proper sequence
below them. to form a meaningful paragraph; then answer the questions given
[IBPS PO 2011] below them [IBPS PO 2012]
(1) To elaborate briefly on these characteristics and dimensions (A) If China is the world’s factory, India has become the world’s
that the author is talking about – NRMs are general tests outsourcing centre - keeping in line with this image.
intended to be used to classify students by percentile for (B) But India’s future depends crucially on its ability to compete
measuring either aptitude or proficiency for admissions into fully in the Creative Economy - not just in tech and software,
or placement within a program. but across design and entrepreneurship; arts, culture and
(2) Contrastingly, the CRM, such as a locally produced entertainment; and the knowledge-based professions of
achievement test, measures absolute performance that is medicine, finance and law.
compared only with the learning objective, hence a perfect (C) While its creative assets outstrip those of other emerging
score is theoretically obtainable by all students who have competitors, India must address several challenges to
a mastery of the pre-specified material, or conversely, all increase its international competitiveness as the world is
students may fail the test. in the midst of a sweeping transformation.

Downloaded From : www.EasyEngineering.net


Downloaded From : www.EasyEngineering.net

Parajumbles  l 297

(D) This transformation is evident in the fact that the world is 80. Which of the following should be the SECOND sentence
moving from an industrial economy to a Creative Economy after rearrangement?
that generates wealth by harnessing intellectual labour, (a) A (b) B
intangible goods and human creative capabilities. (c) C (d) D
(E) Its software industry is the world’s second-largest, its tech (e) F
outsourcing accounts for more than hall of the $ 300 billion 81. Which of the following should be the THIRD sentence after
global industry, according to a technology expert. rearrangement?
(F) If the meeting of world leaders at Davos is any indication, (a) A (b) B
India is rapidly becoming an economic’rock star’. (c) C (d) D
74. Which of the following should be the SIXTH (LAST) (e) E
sentence after the rearrangement ? 82. Which of the following should be FIFTH sentence after
(a) A (b) B rearrangement?
(c) C (d) D (a) A (b) B
(e) E (c) C (d) D
75. Which of the following should be the THIRD sentence after (e) E
the rearrangement ?
83. Which of the following should be the LAST (SIXTH)
(a) A (b) B
(c) C
(e) Eww (d) D

76. Which of the following should be the FIFTH sentence after


sentence after rearrangement?
(a) A
(c) C
(b) B
(d) D

(c) C
w.E
the rearrangement ?
(a) A (b) B
(d) F
(e) E
DIRECTIONS (Qs.84-88): Rearrange the following six
sentences (A), (B), (C), (D) and (F) in the proper sequence to
form a meaningful paragraph, then answer the questions given
(e) E
asy
77. Which of the following should be the FIRST sentence after
the rearrangement ?
below them. [SBI PO 2013]
(A) The group desired to enhance the learning experience in
(a) F
(c) C
(b) B
(d) A
En schools with an interactive digital medium that could be
used within and outside the class-room
(e) E
78. Which of the following should be the SECOND sentence
after the rearrangement ? gin
(B) Then the teacher can act on the downloaded data rather than
collect it from each and every student and thereby save his
time and effort.
(a) A
(c) C
(e) F
(b) B
(d) D
eer
(C) Eductor, decided the group of engineers, all alumni of the
Indain Institute of technology, when the founded Edutor
Technologies in August 2009.
DIRECTIONS (Qs. 79-83) : Rearrange the following sentences
(A), (B), (C), (D) (E) and (F) into a meaningful paragraph and ing
(D) They can even take tests and submit them digitally using the
same tablets and the teachers in turn can download the tests
using the company’s cloud services.
then answer the questions given below it. [IBPS PO 2013]
(A) Moreover salaries in public sector enterprises are not as .ne
(E) With this desire they created a solution that digitzes school
textbooks and other learning materials so that students no
competitive as those offered by private or foreign corporates,
connection
(B) This trend should be a wake up call for stakeholders to
examine why employees are seeking better opportunities
screen tablets. t
longer need to carry as many books to school and back as
before, but can access their study material on their touch-

(F) A mechanic works on motors and an accountant has his


with private companies in India and abroad. computer. Likewise, if a student has to work on a machine
or device, what should it be called?
(C) Public Sector Enterprises (PSEs) have been experiencing
84. Which of the following sentences should be the FIRST after
severe challenges in attracting motivating and retaining their
rearrangement?
key staff. (a) F (b) D
(D) Having identified these as the reasons employees leave (c) A (d) C
PSEs it is important empower stakeholders to find ways to (e) E
remedy the situation. 85. Which of the following sentences should be the THIRD
(E) One reason is that young employees lured away to private after rearrangement?
firms are more willing to undertake professional risks. (a) A (b) B
(F) Employees in specialist roles especially have become (c) D (d) E
increasingly difficult to retain. (e) F
79. Which of the following should be the FIRST sentence after 86. Which of the following sentences should be the SIXTH
rearrangement? (LAST) after rearrangement?
(a) A (b) B (a) A (b) F
(c) C (d) D (c) E (d) B
(e) E (e) D

Downloaded From : www.EasyEngineering.net


Downloaded From : www.EasyEngineering.net

298  l Parajumbles

87. Which of the following sentences should be the FOURTH (E) Allergic respiratory diseases such as hay fever and
after rearrangement? bronchial asthma have indeed become more common
(a) A (b) F in the last decades in all industrialized countries and
(c) E (d) B the reasons for this increase are still debated.
(e) D (F) Several studies have shown the adverse effects of
88. Which of the following sentences should be the FIFTH after ambient air pollution on respiratory health.
rearrangement? 89. Which of the following should be the LAST sentence after
(a) A (b) D rearrangement ?
(c) C (d) E (a) A (b) B
(e) F (c) C (d) D
(e) E
DIRECTIONS (Qs.89-93): Rearrange the following six 90. Which of the following should be the FIRST sentence after
sentences (A), (B), (C), (D), (E) and (F) in the proper sequence rearrangement ?
to form a meaningful paragraph; then answer the questions given (a) A (b) B
below them. [SBI PO 2014] (c) C (d) D
(e) E
(A) As a consequence, even if it is plausible that ambient

ww
air pollution plays a role for the onset and increasing
frequency of respiratory allergy, it is not easy to prove
this conclusively.
91. Which of the following should be the THIRD sentence after
rearrangement?
(a) A (b) B

w.E
(B) Another factor clouding the issue is that laboratory
evaluations do not reflect what happens during natural
exposure when atmospheric pollution mixtures are
92.
(c) C
(e) E
(d) D

Which of the following should be the FOURTH sentence

inhaled.
asy
(C) Interpretation of studies are confounded by the effect
after rearrangement ?
(a) A
(c) C
(b) B
(d) D

to outdoors and indoors allergens.


En
of cigarette smoke. exposure to indoor pollutants and
93.
(e) E
Which of the following should be the SECOND sentence
(D) However, despite evidence of a correlation between
the increasing frequency of respiratory allergy and the
increasing trend in air pollution, the link and interaction gin after rearrangement?
(a) A
(c) C
(b) B
(d) D
is still speculative. (e) E
eer
ing
.ne
t

Downloaded From : www.EasyEngineering.net


Downloaded From : www.EasyEngineering.net

Parajumbles  l 299

Hints & Solutions


LEVEL-I 13. (d) ‘D’ is stating a fact. ‘B’ is supporting the argument in ‘D’ ‘A’
highlighting what may happen if situation in ‘D’ is not achieved.
1. (d) S is the opening sentence, since it introduces the narrative. ‘C’ is indicating towards that eventuality.
P will follow S, as it tells the chronology of the event. That is , 14. (d) ‘D’ is stating a perception. ‘B’ is adding information to this
what happened next. This will be followed by R, since R begins perception. ‘A’ is proof against this perception. ‘C’ is a corollary
with the reflective pronoun, that indicates the action that of A.
happened in P. Thus, the right sequence is SPRQ.
15. (b) ‘B’ is a statement on a trend. ‘A’ is cause of that change. ‘D’
2. (b) Q is the opening sentence, as it introduces the subject John is the result of that cause. ‘C’ is the ultimate result.
F Kennedy, so we have to choose between (b) and (d). R will
16. (a) D shall be followed by C as C continues to talk about errors
follow Q and S will follow R . All R, S and P talk about the
in thinking. This will be followed by B, which talks about our
speeches made by the noun subject, so we will move from the
classification skills which has been emphasized in C.
most general or broadest detail to the more specific so, from ‘a

ww
series of debits’ to ‘the campaign’ to ‘first inaugural address’, so
the correct sequence is (b).
17. (b) The paragraph has to start with D. This is followed by A, as
‘as a result’ in A refers to the quality described in D. B follows A
as it gives an example of an assumption mentioned in A.

w.E
3. (a) Q is the opening line, as it introduces the narrative. This
will be followed by P, which furthers the narrative. S is the reply
to question asked in P and R has the rebuttal to the answer
18. (d) The paragraph starts with D. A has to follow B as ‘In the
past’ mentioned in A shows the contrast from ‘recent one’
given in S, So the sequences is QPSR.

asy
4. (a) P is the opening line because Q,R,S are illustrations or
examples of the statement made in P.
mentioned in B.
19. (c) The paragraph starts with C, which intoduces anticipation
and participation. This is followed by B, which talks about

En
5. (d) S is the opening sentence as it begins the narrative (most
of the ‘once told me’ lines are opening line), followed by Q,
anticipation. B is followed by A which talks about participation.
D follows A as ‘it’ in D refers to participation in A.
because it extends the statement with the same noun subject.
6. (b) Q is the opening sentence, it defines the status quo, followed gin
20. (c) P is the opening sentence followed by R which gives another
complaint, followed by S which gives yet another complaint Q
by S, because S illustrates about customised tailoring outfit, a
subject mentioned in Q ‘custom madeclothing’. Thus will be eer
and T are examples of S.
21. (d) A has to be followed by B, as A talks about the Nagas
followed by P, since P explains further customised tailoring
industry.
7. (d) Q is the opening sentence as it introduces the subject of
ing
position and B about the govennment. Further the ‘on its part’
indicate that B has to follow A. Further C follows B as it says, ‘’
The government should also’’, which means that something has
evaluation followed by S, which is linked with Q because, it
gives conditions attached with the subject of Q.
follows E.
.ne
already been discussed about the government’s role. Finally D

8. (c) R is the opening sentence as it has the subject. There


may be a confusion between P and R, but the subject of P-
participation, is an object in R, Thus R will be the just sentence,
followed Q and then P, as Q and P explain the objects of R.
further talks about the effects of the decline.
t
22. (b) ‘Such reaction’ in B refers to A, so B follows A. C and D

23. (c) The catch is CD and EA. D follows C as ‘councils can also
decide’ in D augment to what has been said in C. A follows E as
A adds to what has been said in E.
9. (d) P is the opening sentence, followed by R because the ‘this’
in R refers to the idea stated in the opening sentence and works 24. (d) E is the opening sentence and is followed by D, where E
as a link between them. This will be followed by Q, because the discusses the problems by BPO and software companies and
pronoun subject ‘it’ refers to Ford Motor company. D tells about the more mundane problems. Further ABC is
the right sequence, ‘hedging’ in B is derived from the currency
10. (d) Q is the opening sentence, it introduces the subject. This
changes discussed in A. ‘These methods’ in C refers to B.
will be followed by P which has a link with Q-Chewing gum,
then will be S which has a link with P-Finish. 25. (d) Clearly C must be followed by D, which must be further
followed by the E as E reitrates the housing shortage and says
11. (b) S is the opening sentence which introduces the subject
that the real deficit will be even higher. D and E provide the
followed by Q, the pronoun subject of Q replaces the noun
statistical proof of the staggering task mentioned in C. So this
subject of S. This will be followed by R because the ‘they’ in R
leads us to two options (b) and (d). Among them (b) seems to
refers to the ‘politicians’ in S.
be more appropriate as B again emphasises on but is being said
12. (d) ‘B’ is answering question asked in ‘D’-‘A’ is hinting towards is A and also that B cannot be the concluding statement of the
an alternate situation and ‘C’ is giving alternate example. paragraph.

Downloaded From : www.EasyEngineering.net


Downloaded From : www.EasyEngineering.net

300  l Parajumbles

26. (c) The paragraph is clearly taking about Goa state and hence in E is reffering to the situation in J&K assembly as mentioned
E has to be opening sentence is the paragraph. This is followed in C.
by A where the phrase, ‘is an impressive case in point’, which is

36. (b) Clearly A is the opening sentence of the paragraph
an example of what is being said in E. A is followed by E as ‘a
followed by B. C, D and E can come only after B as they reflect
similar agitation’ mentioned in B refers to the public activism
the effect of the economic recession. Hence the best sequences
mentioned in A. B is further followed by D and C.
is ABCDE.
27. (b) Clearly A must be followed by E as E is continuing what
37. (c) Clearly E has to be followed by A as they talk about what
was being said in A. E is followed by B which gives an example
employees and employers seek from each other. The use of
of everyday protocol mentioned in E. B is followed by C as
‘And, in turn’ in A shows that it comes after E. A is followed by
‘Here the Indian system’s record’ which is mentioned in C refers
B which is the summary of E and A. C continues the summary
to the kind of system which is being talked about in E and B.
started in B. D is the concluding sentence.
Finally D conclude the paragraph which shows the appealing
record of the Indian system. 38. (c) Only option (c) CABDE forms a logical sequence.
28. (d) A is the opening sentence of the paragraph as is clear from 39. (a) Only option (a) DECAB forms a logical sequence.
the options. This is followed by B as B states the results in the 40. (a) Only option (a) DACBE forms a logical sequence.
Iowa state. This is followed by C which states the reasons for
41. (c) Only option (c) BCDAE forms a logical sequence.

ww
Mr. Huckabee’s victory. C is followed by E which is followed
by D, as ‘will’ and ‘are expected to be’ mentioned in D refer to
situation in New Hampshire which is being mentioned in E.
42. (c) 43. (a) 44. (b) 45. (a)
46. (c) Note that in the selection of the sequence of sentences a

w.E
29. (c) In this paragraph B must be followed by E as E refers to the
climate change and the rising costs mentioned in B. Among the
given options only (c) follows this order.
logical connectivity is maintained. This can by further checked
by logically arranging the points in your own words.
The entire passage summarizes the fact that the Environment

asy
30. (b) Clearly D has to be followed by E as the rest of the mission
mentioned in E means that the first part of the mission must
have been mentioned in the preceding sentence. E is followed
Education Unit of Centre for Science and Environment strives
to make easy course were (A) helpful not only to students
but also to teachers (B) There are two section, one on climate

En
by A as it talks about the precision in which the whole mission
mentioned in D and E is executed. Further C must follow B as
change and the other on natural resources. (C) They are based
on the curriculum. (D) But they are not meant to be just learnt
it continues to talk about the achievement which is mentioned
in B.
31. (c) The catch in this paragraph is that D must be followed
gin by heart. The purpose is to introduce them in the form of an
activity (E).

by E as the contender introduced in D is further elaborated


by India fulfilling the role of contender. So we have only two eer
47. (a) The passage is about the famous rock garden Ryoan-ji in
Japan (A). The dimensions of the garden are then given (30
m long from East to West and 10 m from North to South).
options i.e., (c) and (d). E must be followed by B as B talks
about India becoming a cricketing power capable of winning
in varying conditions anywhere, which is further elaborated in ing
(C) There is a description of the garden. It has no trees only
15 irregularly shapped rocks in a specific arrangement (E).
Further details of the rocks arrangement are give rocks (B) The
A by consistently challenging Australia both at home and away
(mentioned in A).
.ne
interesting part is that only 14 can be viewed at a time. (D) The
description gets more detailed as we move along. It ends with a
32. (a) B must precede C as B explains the feeling responsible for
the Americans voting in large numbers (mentioned in C). A
has to come before B as the historical record mentioned in A is
the large number of voting by Americans which is also clear by
telling conclusion.
48. (b) Zen Buddhism started with the flower Sermon in the
t
14th century. (D) Gautama was giving a sermon. (B) it was a
silent Sermon (A) he held up a flower, no one spoke (E) only
the use of ‘perhaps ever’ mentioned in C. Hence ABCDE is the Mahakasyapa smiled happily. It is a story which has to be
correct sequence. connected in the proper order.
33. (c) Clearly B has to be followed by C as C explains the reason 49. (d) ‘Judas’ priest was an english heavy metal band (B) the
for Kumble being under-rated as mentioned in B. E has to description of the band member is given next (E) a persistent
be followed by A as A continues to talk about Anil Kumble’s problem is noted in case of the drummer (A) the heavy
career, the discussion of which started in E. Hence the correct influence of the band is mentioned next (D) last but not the
sequence is EABCD. least in a telling conclusion they have been spoken of as ‘metal
34. (d) Clearly A has to be followed by D as D completes what gods’ (C).
Allan H. Meltzer was mentioning in A. Also B has to be followed 50. (c) E is the statement which begins the passage, D talks about
by C as ‘that could shrink demand further’ is the result of what the first set of test for intelligence, this is followed by A on the
is happening in B. use of such intelligent tests, C tells us about the intelligence
35. (a) A is the opening sentence as is clear from the given options. quotient and tests which are developed to indicate what an
A is followed by B as B talks about what Farooq Abdullah said average child of a certain age can do and B ends the passage.
to the journalist. C is followed by D as D adds up to the situation 51. (d) Now logically A will follow C and B will follow D, this
in C. Also E has to come before C and D as the ‘test’ mentioned sequence is given only in option (d).

Downloaded From : www.EasyEngineering.net


Downloaded From : www.EasyEngineering.net

Parajumbles  l 301

52. (b) There is a link between 1 and D, ‘be my own boss’ and 4. (b) AD is the key. Passage must begin with A as it states that
‘sick of working for others’ so we have to choose from (b) & (c) ‘welcoming these kids with open arms are a new breed of
followed by B and then C. women ….. to build a thriving business, that of preschools’.
53. (a) There is link between 1 and A – ‘the fire’, ‘the explosion’. D must follow A as it mentions about ‘toddlers, barely out of
So we have to choose between (a) and (b). Thus we have to their diapers ….. at the earliest’. B, C and D cannot open the
choose whether B or C will follow A, but logically C will follow passage as they are the supportive link to the initial start. C
B, as ‘this’ of C is the predicate of B. So ABCD is the correct must follow D as it states that ‘welcoming these kids with open
sequence. arms ….. that of preschools’. B must follow C and also close
54. (b) B will follow 1 because B gives the explanatory background the paragraph as it states about ‘parents being relieved now …..
to the idea stated in 1. This will be followed by C. So the right who attend these preschools.’ The right sequence is ADCB.
sequence is given in (b). 5. (c); D logically follows A with ‘the Great Depression and
55. (b) There is a link 1 and A– ‘This company’, ‘The company’. A the New Deal’ being the topics of introduction. C marks the
will be followed by D because ‘this idea of D refers to the idea beginning of the next clause which is logically followed by ii.
stated in A– ‘foray into service sector by setting up a chain of Option (c) is thus the correct choice. Option (a) can be ruled
laundereltes’. out easily since the sentence cannot end with A. Option (b)
is incorrect since iii and iv cannot come together (too many

61 (b)
66.
ww
56 (d) 57 (a) 58 (d) 59 (c) 60 (a)
62 (d) 63 (e) 64 (e) 65 (c)
(d) 67. (b) 68. (d) 69. (c) 70. (a)
‘and’s). Option (d) looks correct but it is not the right sequence
because of ‘same’. The sentence is ambiguous since ‘same’ can
refer to any of the three things mentioned before i.e. the Great
71. (d)

LEVEL-II
w.E Depression, New Deal and America’s still influential response
6. (b); B is the first sentence as it introduces the main idea of the

asy
1. (d) DC is the key. Passage must begin with D as it initiates with
passage. B and iv make a mandatory pair since ‘the company’
in D refers to Toyota. A follows D as it elaborates some of the
features of the Prius. A and C make another mandatory pair as

to break down the barriers between knowledge and need’. C


En
‘The function of the professional teacher was to ….. in order

must follow D as it states that ‘to find for new truths ….. people
‘after that’ mentioned in C refers to what happens once the car
has run 15 miles. Thus, the correct sequence is B, D, A, C and

might understand’. A, B and C cannot begin the passage as


they provide supportive text. A should follow C as it mentions ginoption (b) is the correct answer
7. (b) Passage must begin with A as it introduces the ‘situation in
which violence occurs,’ ‘violence tends to be clearly defined’ etc.
about ‘knowledge for communication, ….. worshipping at a
respectful distance its new priests’. B would follow A and would
also close the passage as it states that ‘the civilisation, which had eer
B, C or D cannot begin the passage with their ‘So,’ ‘Probably’
and ‘However’ respectively, they only advance the argument. C
hoped to raise itself ….. monastically isolated from the world
by the birth rate of terminology’. The right sequence is DCAB.
ing
should follow A talking of ‘the only uncontrolled applications
of force,’ it tells that the only exception may be in ‘social
superiors’ killing ‘social inferiors.’ D should follow C telling that
2. (c) BA is the key. The passage must begin with B as it opens
up the topic mentioning ‘In recent years, these have been
many reports ….., its debatable results, and its vague, esoteric .ne
‘only when an outsider is killed even by mistake the futility and
horror of violence is realized.’ B should follow D and conclude
terms’. A should follow B as it is logically the supportive text
immediately required as it states that for too many people it is
like a blind man in a dark room ….. what it accomplishes has
not been made clear. A, C and D cannot begin the passage as
t
the passage appropriately with its ‘the actual risk to outsiders.’
Options (a) and (d): Passage wrongly begins with D in both the
options whereas D should follow C with its ‘genuinely appalled’
etc. Option (c): Passage wrongly begins with C whereas C
they are supportive in nature. D should follow A as it states that should follow A with its ‘uncontrolled applications of force.’
‘although hundreds of thousands of words about psychiatry Except (b) other options are wrong.
….. cartoon image of psychiatrists and their mystical couches.’
C should follow D and as well as close the paragraph stating 8. (d). DB and AC are the keys. Passage must begin with D which
‘impatience has been expressed ….. the general public but by introduces the ‘strategic problem’ of interaction between ‘two
psychiatrists as well.’ BADC is the correct sequence. individuals,’ A, B, C advance the argument. B should obviously
follow D with its illustration of ‘An accident’ picking up from
3. (a) CD is the key. Passage must begin with C as it talks
D’s argument, A should follow B with its ‘likelihood of an
about ‘spirituality eliminating disparity between ….. spiritual
accident’ etc. C should follow A and conclude the passage with
products, spiritual property’. D must follow C as it mentions
about ‘religion strengthens ….. we dislike those who weaken it’. its ‘Each must decide’ etc. Options (a) and (b): Passage wrongly
B must follow D as it states that ‘ the person who talks adversely begins with A in both the options whereas A should follow B
….. spiritually generated love’. A should follow B and also close with ‘how carefully the motorist drives.’ Option (c): Passage
the passage as it mentions about ‘if we are still on the ego level correctly begins with D, B correctly follows D but C wrongly
….. than that of the global – love state’. CDBA would be the follows D whereas C has to follow A and conclude the passage
right sequence with ‘Each must decide.’ Except (d) other options are wrong.

Downloaded From : www.EasyEngineering.net


Downloaded From : www.EasyEngineering.net

302  l Parajumbles

9. (a) AD is the key. Passage must begin with A as it introduces passage as D further tells the effect, i.e., ‘the warmer climate’
‘functionalism in positivist organization theory’ and ‘political could melt the floating ice’ etc. Option (a): passage correctly
model of organization theory.’ Passage cannot begin with B, begins the passage with C but D wrongly follows it whereas D
C or D with their ‘Thus,’ ‘At the extreme’ and ‘The political should follow B and conclude the passage, after B has told an
model’ respectively. D should obviously follow A as D talks ‘effect’ of ‘global warming’ D further tells the effect, i.e., ‘the
about ‘political model’ advancing the argument introduced warmer climate could melt the floating ice’ and so on. Options
by A. B should follow D telling about the major role of ‘actual (b), (d): passage wrongly begins with B in both the options
power,’ advancing the argument from D’s ‘exercise of power and instead of C whereas B should follow A which in turn should
influence.’ C should follow B and conclude the passage with its follow C, after A has talked of one effect, i.e., ‘a lid of warm
‘At the extreme’ etc. Option (b): Passage wrongly begins with water,’ B tells about another effect, i.e., ‘plant productivity
C whereas C should follow B and conclude the passage with would rise’ and so on. Except (c) other options are wrong.
its ‘political pursuit of self interest’ etc. Option (c): Passage
13. (b). CD is the key. Passage must begin with C as it introduces
wrongly begins with D whereas D should follow A, precede B
‘religion’ as ‘a guiding force,’ A, B and D cannot begin the
with its ‘exercise of power and influence.’ Option (d): Passage
passage with their ‘Hence,’ ‘The country has already’ and
correctly begins with A but B wrongly follows it whereas B
‘However’ respectively. D should obviously follow C which says
should follow D, precede C with its ‘actual power’ etc. Except
‘it must not be allowed to be exploited,’ D’s ‘it’ refers to ‘religion’
(a) other options are wrong.
introduced by C, B should follow D with its ‘The country has

ww
10. (d) BD is the key. Passage must begin with B as it introduces
‘the system of checks and balances’ telling that nature itself is
their best example. A, C or D cannot begin the passage with
already enough religions,’ it is an appropriate argument with
reference to D’s ‘it must not be allowed to be exploited,’ A
should follow B and conclude the passage telling ‘the educated

w.E
their ‘decision making, however,’ ‘institutions also’ and ‘after
this delicate balance’ respectively. D should follow B with its
‘this delicate balance,’ D’s ‘this’ refers to ‘checks and balances’
and patriotic Indians must rise’ etc. Options (a) and (c): passage
wrongly begins with B instead of C in both the options whereas
B should follow D which in turn should follow C, B’s ‘country

asy
in nature introduced by B, C should follow B as C tells there
is a need of the system of checks and balances in institutions
also. A should follow C and conclude the passage telling that
has already enough religions’ is an appropriate argument
following D’s telling ‘it must not be allowed to be exploited’ and
so on. Option (d): passage wrongly begins with A instead of C

En
group decision making is not a foolproof method etc. Options
(a) and (c): Passage wrongly begins with C in both the options
whereas C should follow D, precede A with its ‘In institutions
whereas A should follow B and conclude the passage, after B
has talked of ‘already enough religions to meddle in politics,’
A’s exhorting ‘the educated and patriotic Indians must rise to
also’ etc. Option (b): Passage correctly begins with B but C
wrongly follows it whereas C should follow D and so on. Except gin the occasion’ is quite appropriate. Except (b), other options are
wrong.
(d) other options are wrong.
11. (c) BD is the key. Passage must begin with B as it introduces
eer
14. (c) BD and AC are the keys. Passage must begin with B as it
introduces the argument of the passage, i.e., giving ‘orders,’
salvaged ‘giant statue of Buddha,’ it cannot begin with A, C,
D with their ‘The operation of last week’, ‘The project’ and
‘It went down’ respectively. They only advance the argument ing
commanding and demanding ‘obedience’ are not good
attributes in an executive. A, C and D cannot begin the passage
with their ‘Suggest or request,’ ‘Be a leader’ and ‘A good one’
introduced by B. D should obviously follow B as D tells when
the ‘giant statue’ of Buddha ‘went down in the lake’, A should
follow D which tells the ‘operation’ (of salvaging the statue) was .ne
respectively. They only tell good or desirable attributes of an
executive after B has introduced the argument. D should
obviously follow B as D tells how does a good executive behave,
‘dramatic’, C should follow A and conclude the passage telling
whose ‘obsession’ the project (of salvaging) was. Option (a):
passage wrongly begins with A instead of B whereas A should
follow D which in turn should follow B after D has told when
t
i.e., ‘he delegates wisely and counsels sanely.’ A should follow
D with its ‘Suggest or request,’ C should follow A and conclude
the passage asking ‘to be a leader and not a boss.’ Option (a):
passage wrongly begins with A instead of B whereas A should
the ‘giant statue’ of Buddha ‘went down in the lake,’ A calls follow D which in turn should follow B, after D has told how
the ‘operation (of salvaging the statue) was dramatic’ and so does a good executive behave or deal, i.e., he ‘delegates wisely’
on. Option (b): passage correctly begins with B but C wrongly etc. A should ask to ‘Suggest or request’ and so on. Option (b):
follows it whereas C should follow A and conclude the passage, passage wrongly begins with D instead of B whereas D should
after A calls the ‘operation’ (of salvaging the statue) ‘dramatic’ follow B after B has introduced the argument, i.e., giving
C tells whose ‘brainchild’ the ‘project’ was. Option (d): passage ‘orders’ or ‘commands’ make a ‘poor executive,’ D’s ‘A good one
correctly begins with B but A wrongly follows B whereas A delegates wisely’ is quite logical and so on. Option (d): passage
should follow D which in turn should follow B, after D has told wrongly begins with C instead of B whereas C should follow A
when the statue ‘went down in the lake’ A calls the ‘operation’ and conclude the passage, after B has introduced the argument,
‘dramatic’ and so on. Except (c) other options are wrong. D advances it telling who is a ‘good’ executive, A advances it
further asking to ‘suggest or request,’ C appropriately concludes
12. (c) CA and AB are the keys. Passage must begin with C which
the passage telling ‘Be a leader’ etc. Except (c), other options are
introduces the problem ‘global warming’, A, B, D cannot begin
wrong.
the passage as they only tell the effects of global warming, A
should obviously follow C as A tells the effect of global warming 15. (d) DB and CA are the keys. Passage must begin with D which
indicated by C as expected. B should follow A as B advances the introduces the topic telling ‘The diseases of the body become
description of the effects, D should follow B and conclude the more pronounced’. A, B, C cannot begin the passage with their

Downloaded From : www.EasyEngineering.net


Downloaded From : www.EasyEngineering.net

Parajumbles  l 303

‘The sickest of such men,’ ‘We find that what,’ and ‘However 22. (a) AEC is the key. Passage must begin with A which tells
the diseases’ respectively. B should follow D because it further what is it to be ‘culturally literate.’ B, C, D or E cannot begin
talks about the diseases of the body. Except (d) other options the passage with their ‘Nor,’ ‘It is,’ ‘cultural literacy constitutes’
are wrong as the link DB is absent in them. and ‘The breadth of that information’ respectively. E should
16. (a) DC is the key. C should follow D with its ‘His writing obviously follow A with ‘The breadth of information’ etc. C
is’ which refers to ‘Is’ai Hsiang’ mentioned by D. Option (b): should follow E talking of ‘culture narrowly understood,’ B
Passage wrongly begins with A whereas A should follow B. should follow C telling ‘culture is not confined,’ ‘to one social
Option (c) and Option (d): Passage begins with D but the link class,’ D should follow B and conclude the passage with ‘cultural
DC is missing. Except (a) other options are wrong. literacy constitutes’ etc. Options (b) and (d): Passage wrongly
begins with D in both the options whereas D should follow
17. (c) AC, BD are the keys. D should follow B with its ‘Such a B and conclude the passage telling what ‘constitutes’ ‘cultural
person ’ which refers to ‘owner’ mentioned by B. Option (a): literacy’ etc. Option (c): Passage correctly begins with A but
Passage wrongly begins with D. Option (b): Passage begins with C wrongly follows A whereas C should follow E telling that
A but the links AC and BD are missing. Option (d): Passage ‘culture’ is ‘narrowly understood.’ Except (a) other options are
wrongly begins with B. Except (c) other options are wrong. wrong.
18. (a) CA and DB are the keys. A should follow C with its ‘His 23. (b) CA and BDE are the keys. Passage must begin with C
influence over’ which refers to ‘Seneca’ mentioned by A. Option which introduces the topic ‘A social cost of theft,’ A, B, D or

ww
(b): Passage wrongly begins with D whereas D should follow A.
Option (c): Passage wrongly begins with A whereas A should
follow C. Option (d): Passage begins with B whereas B should
E only advance the argument with their ‘Both parties,’ ‘The
thief spends time,’ ‘These costs’ and ‘A bank may purchase’
respectively. A should obviously follow C, its ‘both parties’

w.E
follow D. Except (a) other options are wrong.
19. (b) CD is the key. D which further talks about ‘This seed is
called’ should follow C which introduces the subject of ‘Seed’.
referring to ‘the thief and the potential victim’ talked of or
introduced by C. B should follow A with ‘The thief spends’ etc.
D should follow B with its ‘These costs may escalate,’ E should

asy
Option (a) and Option (c): Passage wrongly begins with A
whereas A should follow B. Option (d): Passage wrongly begins
follow D and conclude the passage with ‘A bank may purchase
more and more complicated and sophisticated safes’ etc.
Options (a) and (c): Passage wrongly begins with A whereas A
wrong.
En
with D whereas D should follow C. Except (b) other options are should follow C, precede B with ‘Both parties use capital and
labour’ etc. Option (d): Passage correctly begins with C but B
20. (c) ED is the key. Passage must begin with E which introduces
the topic telling ‘Intelligence can be measured,’ A, B, C or D
cannot begin the passage with their ‘Since then,’ ‘In other words,’ gin wrongly follows it whereas B should follow A, precede D with
‘The thief spends time and money’ etc. Except (b) other options
are wrong.
‘Intelligence is expressed’ and ‘Binet developed’ respectively. D
should follow E with ‘Binet developed the first set of such tests’
eer
24. (b) CE is the key. Passage must begin with C with its ‘however
promising the prospects for growth may be’ as it introduces the
describing the test talked of by E. A should follow E with ‘Since
then’ etc. referring to what has been said in D, C should follow
A telling ‘Intelligence is expressed as intelligence quotient,’ ing
issue of ‘current production constraints.’ A, B, D and E cannot
begin the passage with their ‘Each year,’ ‘Choosing what,’ ‘The
fact’ and ‘There is still’ respectively. E should obviously follow
advancing the argument of E, D and A. B should follow C and
conclude the passage telling ‘intelligence tests give us a norm.’
Options (a) and (d): Passage wrongly begins with C in both the .ne
C which talks of ‘a limit’ to production, D should follow E with
its ‘fact that those limits’ which refers to ‘a limit’ to production
mentioned by E, A should follow D with its ‘Each year we still
options whereas C should follow A, precede B with ‘Intelligence
is expressed’ etc. Option (b): Passage wrongly begins with D
whereas D should follow E, precede A with ‘Binet developed’
etc. Except (c) other options are wrong.
t
have to choose,’ B should follow A and conclude the passage
with its ‘choosing what to produce - a mix of output’ which
refers to A’s ‘we still have to choose some mix of output!’ Option
(a): Passage wrongly begins with A whereas A should follow D
and precede B, telling ‘each year we still have to choose some
21. (c) BC is the key. Passage must begin with B as it initiates mix of output’ and so on. Option (c): Passage wrongly begins
the argument telling ‘Humankind lingers’ in what i.e., ‘Plato’s with D whereas D should follow E which in turn should follow
cave’ or ‘mere images of truth.’ A, C, D and E advance this C, telling ‘those limits may expand in future years’ which refers
argument with their ‘This very,’ ‘But being,’ ‘The inventory’ and to ‘a limit’ to production mentioned by E. Option (d): passage
‘In teaching’ respectively. C should obviously follow B with ‘But wrongly begins with A whereas A has to follow D, telling ‘we
being educated by photographs’ etc. D should follow C with still have to choose some mix of output’ and so on. Except (b)
‘The inventory started in 1839,’ A should follow D talking of other options are wrong.
‘instability of the photographing eye.’ E should follow A and
25. (c) AC is the key. Passage must begin with D as only it introduces
conclude the passage with ‘photograph alter and enlarge our
the argument of ‘command economies,’ ‘ central planning,’ ‘right
notions’ etc. Options (a) and (d): Passage wrongly begins with
mix of output’ etc. A, B, C and E cannot begin the passage with
E in both the options whereas E should follow A and conclude
‘The market mechanism,’ ‘Central planning create,’ ‘As a result’
the passage with ‘In teaching us a new visual code’ etc. Option
and ‘Although’ respectively. A should obviously follow D as A
(b): Passage correctly begins with B but D wrongly follows B
picks up the argument from ‘decentralized market mechanism’
whereas D should follow C, precede A with ‘The inventory mentioned by D, C should follow A as C tells ‘market economy
started in 1839’ etc. Except (c) other options are wrong. may produce lots of frivolous goods,’ B should follow C as B

Downloaded From : www.EasyEngineering.net


Downloaded From : www.EasyEngineering.net

304  l Parajumbles

talks of ‘Central planning’ which takes care of the society’s most ….. people-to-people contact as well’. B must follow A as it is
pressing needs,’ E should follow B and conclude the argument as the correct sequence stating that ‘it is this aspect ….. building
E talks of the ‘worthy’ ‘goals of central planning’ etc. Option (a): up relations’. E would follow B and would also close the passage
passage wrongly begins with A whereas A should follow D, after as it states that ‘it is hoped that these moves ….. channels routed
D has talked of ‘command economies,’ ‘central planning,’ ‘more through third countries’. DCABE is the right sequence.
likely to produce the “right” mix of output than a decentralized
30. (b) ED is the key. The passage must begin with E as it opens
market mechanism,’ A should follow it with its criticism of ‘the
up the topic stating ‘Polyester films intrinsically ….. reject glare
market mechanism.’ Option (b): passage wrongly begins with
and ultra violet rays’. D must follow E as it states that ‘they can
E whereas E should follow B and conclude the passage, after B
be made to reject heat ….. depends on thickness. A must follow
has talked of ‘central planning’ taking care of ‘the society’s most
D as it states that ‘anyone who feels threatened ….. to find them
pressing needs.’ E should follow it talking of ‘worthy’ ‘goals of
central planning.’ Option (d): passage wrongly begins with C useful’. B must follow A as it states that ‘propelled by the rise in
whereas C should follow A which in turn should follow C, after the number …. remain high because of the low costs.’ C must
A has told of ‘market mechanism,’ ‘giving undue weight’ etc. C follow B as well as close the paragraph as it states that ‘films are
should follow it telling ‘a market economy may produce lots of readily available ….. are easy to maintain’. EDABC is the right
frivolous goods’ and so on. Except (c), other options are wrong. sequence.

26. (d); C is an apt opener as it introduces the issue - "we will often 31. (a) CE is the key. The passage must begin with C as it mentions

ww
try to persuade people of what we find beautiful, even though
we do not believe." B gives a reason for such persuasion; thus CB
constitute a mandatory pair. D adds to B by stating "Moreover,
about the topic that ‘India’s handicraft has been ….. India on a
sustained basis’. E must follow C as it states ‘the proper in terms
of product range ….. and the far East’. A, B, D and E cannot

w.E
we are often aware of the contingency of our own judgements'
foundation…" A adds further to the discussion (Nevertheless,
we do think that certain aesthetic …) by providing a concession
start the passage as they are supporting text to the opening
of the passage. D must follow E as it states that ‘hand printed
textiles, shawls, sari goods …. are being targeted’. A must follow
D as it states that ‘the strength of Indian handicrafts ….. high

asy
to D. E further adds to D. Thus, the correct sequence is CBDAE.
27. (b); According to the given options, D, C or E could be the
opener. Sentence C cannot be the opener as it is presenting
export potential’. B must follow A and also close the passage as
it states that ‘the other strength of the industry ….. found across
a contradiction but one does not know what is being
contradicted. EC are a mandatory pair as the antecedent for “
En in India’. CEDAB is the right sequence.
32. (b) BC is the key. The passage must begin with B as it initiates
On the contrary…’in C is E . Option B is the only one that has
EC as a mandatory pair and hence it is the correct option.
28. (d); The paragraph primarily revolves around the idea of
gin the topic by stating ‘According to ‘The Economist’ magazine
….. the 18th centuries.’ C should follow B as it supports the
passage by stating that ‘though it was initially received ….. all
“essentialism” and how it focuses on expressing various
concepts separately through different media so that their eer
over the world’. A, C, E and D cannot begin the passage as they
are supporting the passage. A should follow C as it states that
true value is realized to their full extent. Hence the starting
sentence has to be A. Therefore, options (a), (b) and (c) can be
safely eliminated. Clearly sentence B elaborates further on the ing
‘How coffee really came ….. Chikmangular district’. E should
follow A as it states that ‘the credit for commercialising …..
span of a few centuries’. D should follow E as well as close
thought stated in sentence A by emphasizing that each medium
has different levels of strengths and weaknesses and hence the immensely’. BCAED is the right sequence.
.ne
the passage as it states that ‘the popularity of the brew grew

concepts that have to be expressed through them have to be


obviously different. Therefore, there is a direct AB link. The
sentence following B’s line of thought has to be E as it provides
an example of how one concept suitable for a particular medium
becomes poor in perception if it is expressed through any other
33. (b)
39. (c)
34.
40.
(c)
(d)
35. (a)
41. (b)
36. (d) 37.

t
(b) 38. (c)

42. (a) Statement A ends with the idea of ‘no special creation’.
According to the options, sentence C, D or E follows. Sentence
medium. The EC link is quite prominent because C provides D adds another dimension to the given information by stating
more examples in relation to the idea expressed in E. Finally that ‘it also denied the existence of a Creator’. This point has
sentence D has to be the concluding sentence because it proves not been elaborated anywhere so it cannot follow any other
why essentialism is liked by artists because of the systematic statement. Sentence E mentions ‘preconceptions’ whereas only
method of separating art and media. Hence the correct link at one idea has been mentioned so far. Sentence C elaborates the
the end has to be CD and not DC and so the correct answer is information given in A. ‘The results of chance’ in C can be
option (d) associated with ‘the result of purposeless coincidences’ in B. So,
29. (c) DC is the key. Passage must begin with D as it opens up the CB is a mandatory pair given in (a). The plural ‘preconceptions’
passage stating ‘A battle on the pitch is always more welcome mentioned in E forms a link here and the paragraph ends with
….. current cricketing encounter that entails.’ C must follow D sentence D that provides an additional idea of ‘non-existence of
as it states that ‘there is little else that energises ….. to the entire a creator’. Hence, (a) becomes the correct answer choice.
Indian and Pakistani diaspora the world over’. A, B, C and E
43. (d) According to the given options, sentence B, D or E follows.
cannot open the passage as they do not contain the relevant
The given sentence A mentions ‘extensive plastic surgery’;
topic, moreover they are containing supportive text. A must
sentence B refers to messing up one’s child, this idea does not
follow C as it mentions about ‘while this path breaking event
follow; sentence D might seem like a good choice but a careful

Downloaded From : www.EasyEngineering.net


Downloaded From : www.EasyEngineering.net

Parajumbles  l 305

reading reveals that AE is a mandatory pair. In E, ‘reconstructive B. It is not clear what ‘horrible enough at first sight’ refers to. It
surgery, you might call it’; the ‘it’ refers to the ‘plastic surgery’ can refer to ‘an attempt on a crowned head’ or the fact that ‘it
mentioned in A. The first person used in the last part of E is has entered into the general conception of all chiefs’. Moreover,
continued in B. DC forms a mandatory pair, the ‘but’ used in C C gives a reason why such an outrage might fail (someone
hints that it should follow D. Hence, option (d). giving it a religious manifestation) which fits in perfectly before
44. (d) E will follow the introductory statement A because it talks VI. Therefore the correct sequence is BDAC and option (b) is
about ‘the negotiations ...’ and E tells about the threats to its the correct answer choice
‘progress ...’. CB is the mandatory pair since C tells us as to what 51. (a) A starts the sequence as it follows I in naming the different
Amnesty International says and B elaborates ‘its recent report’. localities that a nation encompasses and on the basis of this, D

45. (d) D follows the first sentence as it voices the general concludes, that the common culture of a nation is only relative. C
perception about gangsters. B would follow D as it continues follows D as it gives a contrasting view by describing a situation
the narrative. C continues the discussion by bringing in the where the relative culture may get dominated by patriotic
French perspective; A continues the discussion about gangsters fervour. B begins another line of thought that gets completed in
in French films. Thus the correct sequence of sentences is statement VI. Also, the ‘this difficulty’ in statement VI refers to
DBCA. the ‘situation’ mentioned in B. The correct sequence is ADCB.
46. (d) D would follow the first sentence; the clue lies in the words Thus, option (a) is the correct answer.

ww
‘these social critics’. B would follow D as it highlights the ‘wit’
mentioned in D. Now the confusion may arise while choosing
between sentence C and A to follow B. If we look at the sixth
52. (a) 1C, AB and B6 form the keys. The passage begins with 1
giving an insight into the topic which is about ‘when we read,
….. repeat his thoughts.’ C must follow 1 as it continues the

w.E
sentence, we would be able to choose C over A as the sixth
sentence talks of ‘reminds the State of its evident failures’, these
words would go with the ‘irrationality of the state’ mentioned
passage stating ‘a book is a machine ….. teacher has outlined
in pencil’. D must follow C as the next suitable link stating that
‘in the same way ….. a means of escape’ A should follow D as it

The correct sequence would be DBCA.


asy
in A. Thus, A would immediately precede the sixth sentence. mentions about ‘Just as man is the tool …. degree of happiness
and wisdom’. B should follow A and also precede 6 as it states
47. (c) The first sentence highlights the problem of cutting the
deficit and national debt. C mentions how getting the funds
from the defence budget are not a good idea and B tells us En that ‘inspite of the sheers of so many ….. with no thoughts’.
This would also allow 6 to close the passage as it leads ‘so it
comes about ….. last forgets how to walk’. CDAB is the correct
why it is so. Thus CB is a mandatory pair. AD is a mandatory
pair with A calling the desire to stop spending on security ‘ill- gin sequence.
53. (d) 1D, BC and C6 form the key. D must follow 1 after it has
advised’ and D giving a reason for it being more so. D falls in
place before the last sentence. Hence, option (c) is the correct
choice. eer
initiated the passage stating ‘one of the earliest ….. in advance’
followed by D which states that ‘when he is twenty ….. until at

48. (d) B follows statement 1 stating the implication of believing


in the ‘nowhere man’ and the ‘cosmic exile’. A comes after B ing
least thirteen.. A would follow D as it mention about ‘we take
the course ….. have a characteristic size’. B would follow A as
it mentions about ‘man is born helpless, ….. has important
as it reasons out why the ‘nowhere man’ is impossible. DC is
a mandatory pair with ‘these concepts’ in C referring to the
‘culture-free concepts’ in D. The last sentence fits in perfectly .ne
psychological consequences’. C should follow B telling, ‘We plan
our lives ….. difficult to live in’ and precede 6 which concludes

after D as it reasons out for the ‘cosmic exile’. Option (d) is the
correct answer.
49. (d) A has to come in the end because the last sentence talks

54. (d) 55. (d) 56. (a) 57. (c) 58. (b) 59. (c)

60. (a) 61. (a) 62. (c) 63. (a) 64. (d) 65. (a)
t
the passage revealing ‘I suppose the main reason ….. are formed
during this period’. Sequence DABC is the appropriate one.

about MSC mentioned in A. D and C have to come before B


because ‘these questions’ mentioned in B refer to the questions 66. (a); D states the position now, as opposed to a 'few years ago'
raised in D and C. Statement I of the question states that mentioned in 1. B makes a comparison with a similar situation
preventing wars was the primary focus of the UN and only if it which A continues with. C asks a question that is answered by
failed in this would it have to resort to coerce warring parties 6. Hence, option (a) is correct.
to stop fighting. Going by this line of thought, D comes before
C as D talks about preventing conflict by dispatching troops to 67. (c); B will follow the opening statement, since it is contradicting
a troubled region while C talks about a situation where a war the ‘claim’ made in statement A. D would follow B. EC is the
or fight has already broken out among opposing parties. Thus, mandatory pair because C is the extension of E which stresses
option (d) is the clear choice on the fact that ‘they were not alone’ and C talks about the other
protests. Hence the correct sequence is ABDEC.
50. (b) B follows 1 giving a reason why terror attempts on a chief
of state are no longer as sensational as they used to be. BD is a 68. (d); D necessarily follows A as it is in continuation of the
mandatory pair since ‘now’ in D serves to contrast an attempt exposition of Takashi Murakami the artist, E takes it forward
on the life of a chief of state with an ‘outrage upon a church’. A forming a pair with B as it defines the ‘divide’ and C is the
follows D since the ‘horrible enough’ in A refers to the outrage concluding statement on Takashi. Hence DEBC is the correct
mentioned in D. Option (c) is incorrect as A cannot come after sequence.

Downloaded From : www.EasyEngineering.net


Downloaded From : www.EasyEngineering.net

306  l Parajumbles

69-73. Use of words like ‘contrastingly’ and ‘these’ in most of 75. (e) E
the sentences imply that some sentences precede these
76. (c) C
sentences. Only Statement which introduces the topic
of discussion is Option (E). So it is the first sentence of 77. (a) F
sequence. Statement 3 extends the topic of books. So 78. (a) A
follows naturally. ‘these two types’ of Statement 7 are
two types discussed in Statement 3. Statement 6 is about 79 (c) 80 (e) 81. (e) 82. (b) 83 (d)
CRMs so the just before it there should be discussion of 84. (a) Sentence F is first ( correct sequence- F,C,A,E,D,B)
NRMs. Last statement is 2 as it finishes the passage on
85. (a) A is the third sentence
summarizing note. So the correct order of sentences is
5374612. 86. (d) sentence B will be the correct answer.

69. (e) 70. (b) 71. (d) 72. (c) 73. (c) 87. (c) E
88. (b) D
74. (d) D
89. (a) 90. (e) 91. (d) 92. (c) 93. (b)

ww
w.E
asy
En
gin
eer
ing
.ne
t

Downloaded From : www.EasyEngineering.net


Downloaded From : www.EasyEngineering.net

17
PTER
CHA

Paragraph Completion

Aw
s the name suggests, our job is to complete the paragraph: one sentence will be omitted form a paragraph, and from the
options given to us, we have to pick which one completes the given paragraph most appropriately and coherently. This sounds
easy, but the passages are picked precisely and options are given specifically to make sure that the comprehension skills of the

ww
students are checked properly. So, you just need to be able to comprehend well what you read. If you are good reader, these questions
should be easy for you.
A paragraph completion question will be made from a paragraph, which would have been randomly picked from an article or

.E
story. One of the sentences would have been deleted and the wrong options will complete the set of four choices given to pick from.
In these questions, it is important to keep in mind that the deleted sentence would have been written by the same author who has

asy
written the rest of the paragraph, so it is essential to understand the subject and purpose of the passage. Knowing how the information
is flowing in the passage, and the kind of tone the author has, is also very important. You can say it is all about maintaining the
continuity of the paragraph with your choice of the option, and making sure that the sentence you pick is in a way, in-line or parallel
to the sentences before/after it.
En
TERMINOLOGY
(1) Scope gin
eer
The sentence you pick to complete the paragraph, has to maintain and be within the scope of the given passage. Scope loosely refers to
the subject of the passage, and what is discussed or talked about there. It basically refers to the limits or the areas in which something

ing
operates, and remains in the context. For example – If the passage is discussing or is based on anthropology, then a sentence which is
on solar energy would be deemed far-fetched or out of context. That is always a great and efficient way to eliminate options, but make
sure that there is no chance of a relation, even an indirect one.
(2) Scale
.ne
Scale directly refers to the limits of the passage, or the subject talked about. For example, if the passage mentions only pollution in the

if the author means to refer to a larger context.


(3) Continuity
t
city, then a sentence referring to the pollution in country, is unlikely to be the right option. But there is still a chance that it could be,

Maintaining continuity means making sure that the flow of information is maintained. The correct sentence would do just that,
i.e. the sentence which will correctly complete the paragraph would never look or sound ‘out of place’, and it will not lead to any
abruptness while reading the passage. The sentence which will appropriately complete the sentence will always either maintain the
thought or take it forward. It would not sway from what has already been mentioned.

(4) Tone
Tone refers to the way in which something is written or said. There could be several different tones, and it is essential to grasp the tone
in which the writer wrote this paragraph. For example, is the author being critical or supportive about the subject? If the passage is
just a plain descriptive one, then you may rule out the sentence which has the critical tone.

TYPES OF QUESTIONS
Basically, there are two types of these questions:
(i) Parenthetical Question: This type has a paragraph with a missing sentence. The sentence is from the middle of the paragraph or
anywhere near that. This normally divides the paragraph in two and leaves the students with two sub-paragraphs that have to be
joined with a fitting sentence.

Downloaded From : www.EasyEngineering.net


Downloaded From : www.EasyEngineering.net

308  l  Paragraph Completion

( ii) Logical Completion of the Paragraph: This is a variation of the parenthetical question type. The only thing here is that the missing
sentence is from the bottom of the paragraph. Simply speaking, it is the last sentence of the paragraph, thereby the name.

HOW TO SOLVE
If you are a good or avid reader, then you can just go with the flow: read the paragraph and pick the option which best suits the given
passage. But, there still exits a way, or approach you can follow in tackling the paragraph completion questions:
(i) Try to get the central meaning, subject or the theme of the given passage after a quick read of it.
(ii) From where the sentence is missing, see how the flow of passage can be maintained. That would go a long way in deciding the
right option.
(iii) If the missing sentence is the last one of the passage, focus on finding how the given passage can be summed up, or summarised.
(iv) After you have selected the option, then a nice way to verify your choice is by giving the passage another quick read, only this
time with the sentence you have chosen. If the continuity and theme of the passage is correctly maintained, then you have
probably chosen the right option.

TIPS
As we have mentioned already, these questions would test your reading and comprehending skills. So, being a good reader will help

ww
you a lot. There are a few tips and tricks which can aid you in finding the right choice.
(i) Underline the keyword, as they would remind you of the subject and gist of the passage while picking the answer.
(ii) You can rule out the option which does not seem to have any relation to the given passage. It might just be there to see if you have

w.E
interpreted the given passage correctly or not.
(iii) The pronoun-antecedent rule can also work very well here.
(iv) The last sentence of the passage (if not omitted) can be very important in selecting the right option, as it often summarises or

asy
concludes the given paragraph. Hence, you can get to know the gist of the paragraph from there.
(v) You can rule out the option which is just repeating what has already been mentioned in the passage. Again, it would be there just
to see if you have understood the passage.

COMMON MISTAKES En
gin
(i) Not reading the sentence which follows the blank or missing sentence: knowing the flow of information, or the direction in which
the paragraph is heading after the missing sentence, is in fact crucial in picking the right answer.
( ii) Picking the answer without reading the entire paragraph: options are always formed to trick you into picking the answer just after

eer
reading the sentence before and after the missing one. But to pick the right answer, you need to know the central idea and the
context of the passage. Hence, a full reading of the paragraph is required.

ing
( iii) Not following consistency: the option you will pick must ensure that the consistency of the passage is maintained and that the
chosen sentence does not lead to any abruptness while reading.

SKILLS REQUIRED
(i) Logical thinking: Being able to link up the thoughts quickly and with precision is very important here. .ne
missing sentence.

THEME BASED PASSAGES


t
( ii) Well read: Being well read gives you a familiarity with the writing skills of different authors and helps in finding out a proper

“Theme Passages” form an important part of most management entrance tests. They cannot be taken lightly by anyone interested in
ensuring a high score in the entrance tests.
Essentials for solving passages
(i) Acute and focused reading habits.
(ii) Focus on varied and multidimensional themes for reading.
(iii) Focus on qualitative reading.
(iv) A well developed vocabulary.
(v) An overall attitude of a ‘GO-GETTER’.
How to go about it? The Process...
(i) Read the passage with intense concentration.
(ii) While reading, by using ellipses (leaving out words) you can get rid of unwanted words, making the passage more concise.
(iii) By compression of words, the ideas also get compressed.
(iv) This exercise brings out just a few emphatic words, may be in a phrase or a sentence, which form the theme.
(v) Now with the theme analysed, you can very easily select the nearest correct option out of the choices given.

Downloaded From : www.EasyEngineering.net


Downloaded From : www.EasyEngineering.net

Paragraph Completion  l 309

BEGINNER TO MASTERY
Example 1.
How does the underlined part of a sentence in the given passage relate to what has been mentioned just before and after it?
“With those words, the passenger opened the coach-door and got in; not at all assisted by his fellow-passengers, who had
expeditiously secreted their watches and purses in their boots, and were now making a general pretence of being asleep.
With no more definite purpose than to escape the hazard of originating any other kind of action.”
Explanation  A probable answer could be: Information conveyed before the underlined sentence: the given passage starts by
mentioning that a passenger entered and he/she was not aided by the other passengers. Antecedent for pronouns who and their is
‘fellow passengers’. The underlined section now explains the state of those fellow passengers, or what they did or what their state was.
The information after this underlined section says what their purpose was behind doing what they did.
Example 2.
Find the relationship the underlined part of the given passage has with the information conveyed before and after it.
The young collector should remember that what is worth doing at all is worth doing well, and that the care bestowed upon his
cabinet is not labour in vain; habits of exactness and precision of arrangement are absolutely necessary if he would make the
best use of the materials which come in his way; and, above all, never let him degenerate into the mere collector: his collection
should be for use, and not merely ornamental.

ww
Explanation  A probable answer could be: The section which come in his way means that he need to make best use of the
material that he gets or get to make use of, i.e. the author is describing or mentioning how a young collector should be, and it does
sound like a suggestion, hence make sure that the verb would be come and not came. The semi colon after in his way indicated that

w.E
two related clauses of a sentence have been separated. It is not compulsory to use it but it sure helps in forming a clear and cohesive
sentence. The antecedent of the pronoun which is materials; whereas the singular pronouns his and him refer to the young collector.
'Above all' is an idiom which means most importantly. So this idiom is introducing something which is most important.
Example 3.
asy
How does the underlined sentence of the given passage act as a concluding sentence?
It must be remembered that the people of Central America are no longer an uneducated and unduly excitable race, except,

En
perhaps, where their personal honour and independence are concerned; they possess an exceedingly clear and precise knowledge
of their prospective or immediate requirements; they have as enlightened leaders among them as ever their powerful Northern

gin
neighbour possessed or possesses: all that they ask, and all that they should be granted, is the freedom to manage their own affairs
in their own way and in their own time. A well-known writer upon Central America, who visited these countries some five-and-

eer
fifty years ago, declared: "Even as it was no one, whatever his prejudices, could fail to perceive the advance in the manners and
customs, and the change in the spirit, of the people of Central America during the ten years of freedom which the Constitution
secured." If that was true then, it is doubly, trebly true to-day, when education and foreign travel have served to open the minds

ing
and broaden the tolerance of these people, who may reasonably be permitted, and even earnestly encouraged, to work out their
own salvation. By free and unrestricted intercourse with the nations of the world this can best be effected, and day by day is

thinking how things may be, see them as they are.


Explanation  .ne
proving the truth of the saying of Dr. Johnson: "The use of travelling is to regulate imagination by reality, and, instead of

A probable answer could be: Author in the given passage is saying that Central American nations have evolved,

t
and improved. The beliefs or how people thought these nations were, is no longer true. Author states that the way in which these na-
tions are looked at and perceived has changed due to education and tourism. Foreign people have visited these countries and actually
seen how these countries are in reality, and this point has been strengthened or referred to by a thought of Dr. Johnson.
Example 4.
It has been great to see Sachin play for so many years. The contribution he has made to the Indian cricket is truly unparalleled,
you only need to look at his records to say that. Indians …..
Give the most appropriate completion of this paragraph/sentence.
Explanation  As we can see, the author is praising Sachin and it seems as if he is a fan of his too. So, the way to complete this
sentence would be to either say what Indians should do for him (which would refer to praising and showing how good he is or has
been) or how Indians think he is (author would probably give a positive opinion, to strengthen his point of view that Indians also
think that). For example – Indians consider him as their role model.
Example 5.
Modern art is one of those things which would bring different opinions: some like it, and some think, well to be polite, they
reckon it is a waste of time and money. I think that it is a mystical art – one that can only be appreciated by the ones who can
understand it. It is special also in the way that you will only be able to realise how good it is once you can see what the painter
had in mind and what he painted – not many can do that. So, we can say..
Complete the concluding sentence of the given paragraph:

Downloaded From : www.EasyEngineering.net


Downloaded From : www.EasyEngineering.net

310  l  Paragraph Completion

Explanation  It seems as though the author does like Modern art, but he also recognises the fact that not many like it. And our
concluding sentence would reflect that. For example – it is a unique form of art, also in terms of how divided the opinions are on it.
DIRECTIONS (Example 6 to 12):  Each of the following questions has some part missing. Choose the one that completes it in the most
appropriate way.
Example 6.
Many people suggest ............... and still others would like to convince people not to buy pirated cassettes.
(a) to bring down audiocassette prices to reduce the incidence of music piracy, others advocate strong legal action against the
offenders.
(b) bringing down audiocassette prices to reduce the incidents of music piracy, others are advocating strong legal action against
offenders.
(c) bringing down audiocassette prices to reduce the incidence of music piracy, others advocate strong legal action against
offenders.
(d) audiocassette prices to be brought down to reduce incidence of music piracy, others advocate that strong legal action must
be taken against offenders.
Explanation  Option (c). According to the parallelism rule, the two actions that we have in the sentence should be in the

ww
same form, either the gerund form or the ‘to … infinitive’ form. Thus, with ‘many people suggest…’, the correct form of the phrase
would be ‘others advocate that…’
Example 7.
w.E
The audiences for crosswords and Sudoku, understandably, overlap greatly, but there are differences, too. A crossword attracts a
more literary person, while Sudoku appeals to a keenly logical mind. Some crossword enthusiasts turn up their noses at Sudoku

asy
because they feel it lacks depth. A good crossword requires vocabulary, knowledge, mental flexibility and sometimes even a sense
of humour to complete. It touches numerous areas of life and provides an “Aha!” or two along the way ...........................
(a) Sudoku, on the other hand, is just a logical exercise, each one similar to the last.

En
(b) Sudoku, incidentally, is growing faster in popularity than crosswords, even among the literati.
(c) Sudoku, on the other hand, can be attempted and enjoyed even by children.
(d) Sudoku, however, is not exciting in any sense of the term.
Explanation  gin
(a); The given paragraph is about two games: Sudoku and Crossword, and what kinds of people like playing

eer
these games. The author seems to be the one who reckons Crossword is a superior or better game.
Looking at the options, we have to pick how would the author conclude what the game of Sudoku is and considering that he

options (b) and (c).


ing
seems biased towards crosswords, the paragraph’s conclusion will reflect that crosswords are a superior game. Hence, we can rule out

Now, option (a) begins with ‘On the other hand’ which is used to introduce a contrastive clause or phrase, and it says that Sudoku

.ne
is just a logical game, and that is it, there are not many facets or aspects of it. It can be inferred that it is a logical game, because the
author already stated in the paragraph that Sudoku is preferred by logical minds. It clearly undervalues Sudoku and puts Crossword
in a better light.
Example 8. t
I am sometimes attacked for imposing ‘rules’. Nothing could be further from the truth. I hate rules. All I do is report on how
consumers react to different stimuli. I may say to a copywriter, “Research shows that commercials with celebrities are below
average in persuading people to buy products. Are you sure you want to use a celebrity?” Call that a rule? Or I may say to an art
director, “Research suggests that if you set the copy in black type on a white background, more people will read it than if you set
it in white type on a black background.”
(a) Guidance based on applied research can hardly qualify as ‘rules’.
(b) Thus, all my so called ‘rules’ are rooted in applied research.
(c) A suggestion perhaps, but scarcely a rule.
(d) Such principles are unavoidable if one wants to be systematic about consumer behaviour.
(e) Fundamentally it is about consumer behaviour - not about celebrities or type settings.
Explanation  (c); The author in the paragraph is saying that he is wrongly accused of being someone who imposes rules. Ac-
cording to him, he just reports the truth and facts. Now, we have to pick a statement which best concludes what the author has to say.
With the examples the author gives, it can be inferred that he just gives suggestions or guidance after doing some analysis. We have
to pick what he would have to say after giving that second example. In the second example, he uses a research to back up his opinion
that and offers some suggestion to the art director. So, the most appropriate option would be (c) because with this the author clearly
mentions that this example can be considered a suggestion, but never a rule. While picking the option, it is important to note that the

Downloaded From : www.EasyEngineering.net


Downloaded From : www.EasyEngineering.net

Paragraph Completion  l 311

author never admitted that he gives suggestions either, he states that ‘all I do is report’ and this is why option (a) would not be deemed
appropriate.
Example 9.
Relations between the factory and the dealer are distant and usually strained as the factory tries to force cars on the dealers
to smooth out production. Relations between the dealer and the customer are equally strained because dealers continuously
adjust prices - make deals - to adjust demand with supply while maximizing profits. This becomes a system marked by a lack of
long-term commitment on either side, which maximizes feelings of mistrust. In order to maximize their bargaining positions,
everyone holds back information - the dealer about the product and the consumer about his true desires ................
(a) As a result, ‘deal making’ becomes rampant, without concern for customer satisfaction.
(b) As a result, inefficiencies creep into the supply chain.
(c) As a result, everyone treats the other as an adversary, rather than as an ally.
(d) As a result, fundamental innovations are becoming scarce in the automobile industry.
(e) As a result, everyone loses in the long run.
Explanation  (e); The given paragraph basically describes how relations are between the factory and the dealer, and between
the dealer and the customer. It says that the relations between factory and dealers are not cordial, because factories try to impose cars

ww
on the dealers or force them to accept cars as the factory tries to clear out the cars they have produced to make sure there is a smooth
manufacturing process. The relationship between dealers and consumers is strained too because the main concern of the dealers is
to maximize the profits, and to ensure that, they vary prices according to demands. So dealers hold back information about the cars

w.E
to make sure they earn good money, and the consumers hold back and the consumers also contribute to this cordial relationship by
not revealing their true wishes. Consumers also try to haggle in order to make sure they get the car at a good price. So, the author
gives three reasons as to why these relations are not friendly: they do not want or strive for long-term commitment, they just want to

asy
make the best out of the present condition for themselves and this leads to mistrust amongst them as they are never keen on building
friendly relations and are just concerned about their selfish purpose. The third reason is that they hide the information which should

En
be shared. Now, the options suggest that we have to decide what happens as a result of all these things, or what all this leads to. Option
(e) is the most appropriate as it states that this would lead to a loss in the long term, because logically, such relations would not last

gin
long and would always be determined by selfishness. Option (a) is wrong, because it does not conclude the paragraph. It only refers
to the relationship between the consumer and the dealer. Option (b) can be ruled out in the same way because it only refers to what
happens due to a strained relationship between the factory and the dealers. Option (d) is wrong, because it would not be appropriate

eer
that everyone considers each other as their enemies. In fact, others are looked at means to satisfy selfish interests.
Example 10.

ing
The ancient Egyptians believed ............... so that when these objects were magically reanimated through the correct rituals, they
would be able to function effectively.

(b) it was essential for things they portray to have had every relevant feature shown as clearly as possible.
(c) it was essential the things they portrayed had every relevant feature shown as clearly as possible. .ne
(a) that it was essential that things they portrayed must have every relevant feature shown as clearly as possible.

(d) that when they portrayed things, it should have every relevant feature shown as clearly as possible.
Explanation  Option (a) is most appropriate one. Option (b) uses the wrong verb tense. Option (c) is awkwardly worded.
Option (d) uses the wrong pronoun (it) for the subject (things).
t
Example 11.
Archaeologists believe that the pieces of red-ware pottery excavated recently near Bhavnagar and ............... shed light on a hitherto
dark 600-year period in the Harappan history of Gujarat.
(a) estimated with a reasonable certainty as being about 3400 years old,
(b) are estimated reasonably certain to be about 3400 years old,
(c) estimated at about 3400 years old with reasonable certainty,
(d) estimated with reasonable certainty to be about 3400 years old.
Explanation  (a); The correct option would use the modifier ‘with reasonable certainty’ correctly. This is done in two op-
tions: (a) and (d). Option (d) has a mistake that it uses a ‘period’ at the end, whence the correct option cannot do so (the missing
part is not a complete sentence in itself and the later part of the sentence depends on this missing one). Thus, option (a) is the cor-
rect one.
Example 12.
Thus the end of knowledge and the closing of the frontier that it symbolises is not a looming crisis at all, but merely one of many
embarrassing fits of hubris in civilization’s long industry. In the end, it will pass away and be forgotten. Ours is not the first
generation to struggle to understand the organizational laws of the frontier, deceive itself that it has succeeded, and go to its grave

Downloaded From : www.EasyEngineering.net


Downloaded From : www.EasyEngineering.net

312  l  Paragraph Completion

having failed. .........................


(a) One would be wise to be humble.
(b) But we might be the first generation to actually reach the frontier.
(c) But we might be the first generation to deal with the crisis.
(d) However, this time the success is not illusory.
Explanation  (a); This is a simple one if you read carefully. It can be seen that the paragraph talks about the end of knowledge
and the closing of the frontier that it symbolises. It says that this is an event that keeps repeating over the course of years. Thus, any
sentence that starts with a ‘but’ or ‘however’ would be contrary to what we need. Thus, all options except option (a) can be negated.
Example 13.
The waterholes of Africa serve as the great meeting point of the prey and the predator. The prey, exhausted after the day’s
hectic grazing seeks the cool water and the predator seeks the prey’s blood. Extremely tense situations develop at these sites and
herds of nervous herbivores can be seen fidgeting and thinking twice before venturing close to the waterholes. Those who dare,
though, reap a rich reward in the form of the thirst quencher. However, it must not be forgotten that death always lurks nearby
in ominous forms.
Select the option which represents the theme of above paragraph.
(a) Africa is a dangerous place to be in.

ww
(b) The waterholes are meant specially for the predators.
(c) Life and death reside simultaneously at the African waterholes.
(d) Herbivores and carnivores are nature’s way of maintaining balance.
Explanation 
w.E (c). The passage reveals the fact that in the waterholes of Africa, the prey and the predator meet each other. Here,
the prey is in search of cold water and the predator seeks the prey's blood. This creates an atmosphere of death for the herds of her-
bivorous animals who are thirsty enough to go close to the waterhole. Option (c) is correct as it best suits the theme of the passage.

asy
Option (a) is incorrect as one cannot derive that conclusion from the passage. Option (b) is incorrect as the waterholes are meant for
the prey and the predator both. Option (d) is incorrect as it is not mentioned anywhere in the passage.
Example 14.

En
Intelligence probably depends on the efficiency of the electrical circuits in the brain, on being able to try out many connections
quickly and select the best. Like memory, intelligence will probably be susceptible to improvement by drugs – the idea of an

gin
“intelligence pill” has been suggested so often that it is becoming more than a joke. We are unlikely ever to produce a drug
for transforming a moron into a genius, but a pill to improve intelligence may well be feasible. Rats clever at threading mazes
apparently have more cholinesterase in their brains than duller rats; so intelligent and dull humans may have significant

eer
differences in the chemical compositions of their brain fluids. If so, the dull ones might be made much brighter with the right
mixture.
Select the option which represents the theme of above paragraph.
(a) Drugs to partially cure poor intelligence in humans are feasible. ing
(b) Intelligence can be tremendously boosted by improving the electrical circuits in the human brain by using the proper drugs.
(c) The experiment on rats shows for sure that human intelligence follows an analogous path.
(d) Drugs to cure poor intelligence are impossible to make but ridiculous attempts have been made. .ne
Explanation 
t
(a). The passage depicts the fact that intelligence of a person depends on the level of efficiency of the electrical
circuits of the brain. It is reasonable enough to improve intelligence by the use of drugs. The difference in the chemical composition
of a dull and intelligent human being is quite obvious. Option (a) is correct as it best suits the theme of the passage. Option (b) is in-
correct as it is not mentioned anywhere in the passage. Option (c) is incorrect as it can’t be inferred directly from the passage. Option
(d) is incorrect as the passage does not mention anywhere that the drug to cure poor intelligence is impossible to make.
Example 15.
Despite the multiple risks venture capitalists take, they have to kowtow to a punishing tax-regime. What irks the VC community
is that while mutual funds are totally exempted, VCFs are required to pay maximum marginal tax. Following an outcry, it has
been decided that VCs would be allowed to set off losses in one invested company against profit in another, as an income tax
sop. Faced with difficulties in raising funds, VCFs are also demanding that pension funds, insurance companies and mutual
funds be allowed to invest about five percent of their corpus in VCFs with a proven track record, on the lines of the US and other
developed countries.
Select the option which represents the theme of above paragraph.
(a) VCFs are roughing it out with mutual funds and insurance companies to survive.
(b) VCFs are having a hard time with adverse government policies.
(c) VCFs are dying in the harsh industrial climate in the country.
(d) VCFs with their professional attitude can sustain it.
Explanation  (b). The passage discusses the issue that VCFs are having a hard time with adverse government policies. Option
(b) is correct as it best suits the theme of the passage. Options (a) and (c) are incorrect as they have not been mentioned anywhere in
the passage. Option (d) is incorrect as it inappropriate in the given context.

Downloaded From : www.EasyEngineering.net


Downloaded From : www.EasyEngineering.net

Paragraph Completion  l 313

Practice Exercise
LEVEL-I
DIRECTIONS (Qs. 1-30):  In each of the following questions a (c) This needs to be corrected.
short passage is given with one of the lines in the passage missing (d) EIA is short of experience and therefore such a negative
and represented by a blank. Select the best from the given options effect.
to make the passage complete and coherent. 3. The International Monetary Fund (IMF), the World Bank
1. There are many industries where India has an advantage and the International Trade Organisation were conceived
because of relatively lower costs of all forms of manpower- at the Breton Woods Conference in July,1944 as institutions
whether it is professional or factory labour. However, while to strengthen international economic cooperation and to
this can give initial advantage, it should not be taken for an help create a more stable and prosperous global economy.
enduring advantage due to the following reasons. One, as While the IMF and the World Bank come into existence
products become more sophisticated, labour as cost factor
and started functioning from 1946, the International Trade

ww
becomes less and less important. Two, the differences in
costs are narrowed down through higher level of automation.
There, in processes that require large number of cheap
Organization could not be set up. Instead, the General
Agreement on Tariffs and Trade (GATT) was set up in 1947.
Through successive round of negotiations, the GATT got

w.E
labour, the industry is bound to shift its operation along.
The line of the ever-declining scale of poorer countries. So
a poorer country than India can eventually overtake us with
yet cheaper labour. Therefore, when one has established an
transformed into what has come to be known as the World
Trade Organization (WTO) that started functioning from
January 1, 1995. The various institutions have set up to govern

asy
export market on the basis of cheaper manpower…………..
(a) One has to be vigilant to make sure that one builds up
international economic relations. While all the institutions
work in close coordination with each other……………..

this temporary advantage.


En
other advantages to compensate for the inevitable loss of

(b) One has to be vigilant to make sure that this advantage


(a) Each of the institutions is independent
(b) Each of the these institutions works with different focus
in different direction
should not be given away
(c) One needs not be vigilant as there is no competition in gin(c) Each of these institutions has its own specific area of
responsibilities
near future
(d) There is need of caution to see the variations in labour
charges of other countries eer
(d) Each of these institutions has major role to play in each
other’s work
2. Standards and standardization, quality systems, certification
and inspections, measurement systems, testing laboratories,
their accreditation and calibration service, production and
ing
4. ADB finances principally specific projects in the region. It
may make loans to or invest in the projects concerned. It
may also guarantee loans granted to the projects. Most of
supply of standard reference materials etc, are all important
building blocks. Quality control through the agency of the .ne
the loans granted are hard loans or tired loans. However,
loans form special funds set aside by the ADB up to 10 per
cent of its paid-up capital are granted under soft loan term
Export Inspection Agency leaves much to be desired. It is
often alleged that EIA is actually playing a retrograde role,
although inadvertently. ………………..
(I) The list of items subject to compulsory export inspection
t
for which purpose is has set up a separate window known as
the Asian Development Fund (ADF).Soft loans are normally
granted to projects of high development priority requiring
longer periods of repayment with lower rates of interest.
needs be reviewed and shortened. A trimmer EIA list
ADB normally finances foreign exchange cost of the project
essential for a modicum of efficiency.
and the loan in repayable in the currency in which it is
(II) EIA should use international agencies to train people
made. India has been eligible for assistance both under the
and update the equipment available for those limited ADB and its soft loan window, ADF……………... However,
items.(III)The quality development process need be it has been getting large assistance under the ADB.
professionalised by making use of quality development (a) But India does not need any assistance from ADB.
skills and managerial methods available around the (b) But India is not a member of ADB
world. Overall, per-export inspection needs to be greatly (c) But India is not interested in ADB aids.
simplified, both in the interests of speedier clearance (d) But India has stayed away from the ADB
and less harassment for the exporters as well as better 5. In the planned economy of India, foreign capital has been
administration. assigned a significant role, although it has been changing
(a) EIA is to be scrapped and new council should be made over time. In the earlier phase of planning, foreign capital
with following improvements. was looked upon as a means to supplement domestic
(b) This is happening in absence from a guiding international investment. Many concession and incentives were given to
foreign investors. Later on, however, the emphasis shifted
agency which can suggest a number of measures.
to encouraging technological collaboration between

Downloaded From : www.EasyEngineering.net


Downloaded From : www.EasyEngineering.net

314  l  Paragraph Completion

India entrepreneurs and foreign entrepreneurs. In more 8. A budget is a statement containing a forecast of revenues
recent times, efforts are on to invite free flow of foreign and expenditures for a period of time, usually a year. It is
capital………………….. a comprehensive plan of action designed to achieve the
(a) It would be instructive in this background to examine policy objectives set by the Government for the coming
the Government’s policy towards foreign capital. year. A budget is plan and a budget document is reflection
(b) It would be instructive in this background to examine or what Government expects to do in future. While any
the World Bank’s policy towards foreign capital in India plan need not be a budget, a budget has to be necessarily a
(c) Let us keep our fingers crossed and look for the next pan. It shows detailed allocation to resources and proposed
parliamentary session for debate on the issue taxation or other measures for their realization. A budget
(d) Issue of Foreign capital is fragile and can be discussed
is, however, not a balanced sheet (exhibiting total assets
only with relevant statistical figures in hand
and liabilities) of the Government on a particular date. Is
6. FDI may actually be harmful to the recipient country if the
a financial blueprint for action and is, therefore, of great
economy is highly protected and foreign investment takes
place behind high tariff walls. This type of investment is advantage to Government departments, legislatures and
generally referred to as the tariff-jumping’ variety of foreign citizens. The budget of government expresses its total
investment, whose primary objective is to take advantage of activity in figures…………………………..
the protected markets in the host country. The longer the (a) A budget reflects what the Government is doing or

ww
Government shields its home market with tariffs……………
and more acute will be the conflict between him and the
domestic entrepreneur. In view of this, an appropriate policy
intends to do.
(b) A budget is a legal document
(c) A budget is a promise of Government to its people

w.E
framework must respond to two conflicting objective: the
need to liberalises rules governing such investment in view
of the growing integration of the world economy, and the
(d) A budget is a guideline for State Governments
9. For a federal country like India, the budget of the
Government of India is the most important instrument

asy
need to ensure that such investment has positive effects on
the country’s economy and does not lead to negative welfare
effects.
for implementing various economic and social objectives.
The budgets of state government affect local activities.

(b) The more the foreign money to come in India En


(a) The more the foreign countries to apply pressure on India
The Government of India budget influences the whole
economy. The latter tries to bring about growth with
social justice through its budget; it influences regional,
(c) The more the foreigner will come to exploit that
protected market
(d) The more the foreigner to protest against that government
gin functional and overall distribution of income and wealth
through its expenditure (transfer) payments, investments
7. The Indian constitution provides for demarcation of
functional responsibilities and finances between the Centre eer
and tax policies. The provisions of grants and loans to State
governments and Union Territories and to the private sector
and various subsidies (such as for export promotion, food
and the States. The provision of public services has been
largely entrusted to the States. These mainly relate to law
and order, public health, sanitation, water supply and
ing
grains distribution, etc.) are some of the elements of Central
government budget policy for promoting growth and
agriculture. The States have to concurrently take certain
functions in areas such as education, infrastructure. Their .ne
income distribution. …………………. Its significance lies
in its ability to promote the various objectives of a modern
share in combined expenditure (Centre and States) on social
services is about 85 per cent, while in the case of economic
services; it is about 60 per cent. Thus, the States have the
primary responsibility to undertake tasks pertaining to
t
state which has assumed the role of a welfare state and of a
catalytic agent for promoting growth with social justice.
(a) A budget in modern times should, therefore not be
judged sound or otherwise merely on the basis of its
developing social and economic infrastructure. However, ‘deficit’ or ‘surplus’ or ‘balanced’ position
their ability to undertake such development functions (b) A budget is therefore not only instrument of
is critically determined by their financial position. The implementing the economic and social objectives but it
growing importance of state finances in the macro-economy is also about growth with social justice
is evident from the fact……………... The size of overall (c) A budget is a reflection of success or failure of a
development expenditures of the states has always been government
higher than that of the Centre and the difference has got (d) A budget in modern times should ,therefore not judged
widened rather significantly in the 1990s. on its face value
(a) That the States has overrun their planned expenditures 10. After the East Asia crisis, the World Bank conducted a study
and lacking freedom of further development
on the underlying reasons for the crisis. It was found that
(b) That the States has reached at the peak of their finances
at least a major part of the fundamental responsibility was
and overtaken Centres in revenues
on banks, which had understated their non performing
(c) That Centre borrow money from the states for its
expenditures on educational and social welfare programmes accounts by as much as 47%. Since this was a study and
(d) That the total expenditures of State governments have not an investigation…………….. Nevertheless, the Basel
even undertaken those of the Centre Committee on supervision did take cognizance, and issued

Downloaded From : www.EasyEngineering.net


Downloaded From : www.EasyEngineering.net

Paragraph Completion  l 315

circulars and directives not only on supervision, but also on risk appetite .Capital allocation (about how much) would be
Internal Functional Management. It will be remembered based on such strategies………..
by those interested that Basel committee had also acted (a) Most banks are yet to conceptualize the same in their
expeditiously after the Barring Bank’s failure, to separate processes
treasury and lending operations from the decision making (b) Most banks have already integrated it in their
processes. Bank failures are nothing new in the world, functioning; it is working over the years satisfactorily.
although we in India have been insulated from such traumas (c) Which would in long run prove to be the growth
for more than two decades. impeding
(a) The findings were not taken note of (d) Of risk aggregation which is really a new concept to
(b) The findings were not taken seriously Indian banks
(c) The findings were not legally binding on any one 14. However, it is possible that the non-resident entity may
(d) The fallout from this revelation was only taken note of have a business connection with the resident Indian entity.
11. Whether the Government is right in bailing out a private In such a case, the resident Indian entity could be treated
sector bank is an issue that is decided more than by the
as Permanent Establishment of the nonresident entity.
long term social security policy of the Government, than
……………During the last decade or so, India has seen
by economic reasons alone……………….. Nevertheless,
a steady growth of outsourcing of business processes by
in a situation of scarcity of resources, bailing out somebody

ww
means the denial of resources to others. The irony of it is that
in performing its duties of proper governance to the larger
non residents or foreign companies to IT-enabled entities
in India. Such entities are either branches or associated
enterprises of the foreign enterprise or an independent

w.E
society through the process of bailing out, Government
excuses the lack of corporate governance in banks.
(a) Economists world over learnt it hard way during the
Great depression
India enterprise. The nonresident entity or foreign company
will be liable to tax in India only if the IT-enabled BPO unit
in India constitutes its Permanent Establishment.

schools asy
(b) This is elementary principle of economics taught in (a) The tax treatment of the Permanent Establishment in
such a case is under consideration
(b) How would the profit would be shared is not decided
liquidity and credit ratings issues
En
(c) Reasons are not limited to these two but extend to debts,

(d) It was unexpected and came like a bolt from the blue
yet?
(c) A lengthy and cumber some process requiring a lot of
12. But no depreciation is allowed on Live Stock i.e., Horses.
Although the horses are in the nature of fixed assets in gin application of mind and revenue principles is ahead for
the tax department of India
the hands of the owner, no depreciation is allowed under
Income Tax Act. Instead when the animal dies or becomes
permanently useless the entire value of the horse can be eer
(d) A new trend is seen in last decade.
15. The Finance Commission is entrusted with periodic review
and resolution of Central- State fiscal problems. It was
written off as revenue loss in the year in which it dies or
becomes permanently useless. When the gross income ing
the clear intention of the father of the India Constitution
that all matters pertaining to normal Central-State
exceeds the total expenditure, it results in net profit which
will be taxable at usual rates of tax applicable to the person.
………………….. Although the live stock is in the nature of
.ne
financial adjustment should be scrutinized by the Finance
commission………………. An incidental and by no means
insignificant advantage of the appointment of a Finance
fixed assets of the owners buy them, maintain them, train
them, and participate in races and Sell them or send them
away to studs when they are useless.
t
Commission has generally been to rekindle interest in issues
pertaining to financial relations between the Centre and the
States and to promote an enlightened national debate on the
several facets of India’s federal fiscal set-up. The role of the
(a) But when the gross income is less than the expenditure,
then results in loss Indian Finance Commission is unique in many ways. It is
one of few commission provided in the constitution.
(b) But when the gross income is higher than the
(a) which was given a pre-eminent role in the resolution of
expenditure, then results in loss
problems in fiscal federalism
(c) But when the gross income is equal to expenditure then
(b) Which was constituted with the vision of a modern
result is loss
India with modern facilities
(d) But when the gross income is there loss is the result
(c) Which was a dream of Father of Nation also
13. Aggregation of risks is somewhat quite new to banks in (d) Which was to be unique in its ways and a constitutional
India. While some banks have started thinking in that line body
by trying to put integrated limits framework and integrated 16. The art of medicine is the art of healing, not just treating,
risk policies as well as using CBS solutions for technological and not even just curing. Yet it is only when the art and
integration, the effort required is beyond such requirement. science join hands that healing is best accomplished. The
Risk aggregation would mean aggregating the individual author then adds ……………., remember that the practice
risk measures to decide most appropriate assets class that of medicine is an art, not a trade, a calling, not a business,
would contain the risk to the desired level dictated by the a calling in which your heart will be exercised equally with

Downloaded From : www.EasyEngineering.net


Downloaded From : www.EasyEngineering.net

316  l  Paragraph Completion

your head. This book is rare work of the art of medicine, that will not hit the roads, its women friendly features have
from a very rare practitioner of the science of medicine. been incorporated in some of the other cars.
(a) “ mankind depends on science as equally on the art” (a) The first to be designed by experts
(b) “ for the mercy’s sake let us have little less science and a (b) The first to be launched by company
little more art” (c) The first to be the concept car
(c) “ let us consider science at par with art” (d) The first to be designed by all women team
(d) “ let us forget what is art and what is science” 20. The growing importance of Union excise amongst the shared
17. The thirteen Finance commissions cover a span of more taxes and the ascendancy of population as the principal basis
than 60 years during which many conditions have changed. of distribution are the two salient features of tax-sharing
Correspondingly, the approach of the later commissions determined by the Finance Commissions. The finance
may be expected to be different in several respects from the Commission is called upon to determine the State that
earlier ones Nevertheless, it is possible to discern certain would be in need of grant-assistance of the quinqennium
common elements in the thinking of the successive Finance under reference and the amount of such assistance in
Commissions. each case. The first Finance Commission laid down some
Which have come to evolve gradually what may be called important principles governing the determination of grants
‘the Indian Finance Commission’s approach to federal -in-aid for States. …………………
finance........ According to this approach, States’ share of (a) These principles have been generally, endorsed by all the

ww
Central taxes is not allocated strictly on the basis of need.
These problems and shortcomings come later in light but
subsequent Commissions.
(b) These principles have been discarded by subsequent

w.E
have caused what was not accounted at that time.
(a) And it happened to be in that way
(b) There are several inadequacies in the approach of the
Finance Commissions
commissions
(c) These principles were taken from the Finance
commissions of the other countries
(d) These principles were derivations from the elementary

commissions asy
(c) Evolution is a long process and it is a same story for formulas of text books
21. The biggest attraction of the public sector is that, for women


(d) But they differ completely from them and each time
new approach was looked for
18. The Parihar is, for all practical purposes, a functional,En with the same qualifications and skills it almost always pays
better than does private industry. For men the differences
are much less pronounced………………….. Figures are
fully fitted out submarine. After this brief ceremony, the
submarine is to be towed out for the first time across the naval gin hard to come by, but in rich countries women typically hold
30-40% of senior managerial posts in central government.
dockyard and moored in an enclosed pier called Site Bravo
…………..Over the next few months, it will commence a
series of harbor trials .the primary system, a nuclear reactor, eer
Hours and conditions too are usually more congenital and
maternity arrangements more generous .So with better pay,
conditions and promotion prospects, it is no wonder that
generates the heat which drives the secondary system, a
steam turbine which spins the submarine’s propeller, are to ing
the public sector is the employer of the choice for so many
women.
be tested separately. First, the steam turbine is to be jump
started with shore based supply. The next significant step
will be starting up the submarine’s nuclear reactor where
to senior jobs
.ne
(a) The public sector is also more likely to promote women

(b) The public sector provide safe working environment for


Zirconium rods in the core of the submarine’s pressurized
water reactor will be slowly raised.
(a) It is the advent of new technology in India
(b) It has entered in chain reaction chamber
the women.
(c) Women are paid more than men in public sector
t
(d) Public sector is better pay master than private banks for
women
(c) It is like coming out from maternity ward to nursery 22. The fiscal position of the Indian Governments - both Centre
and States -has been under stress since the mid -1980s. The
(d) It is unprecedented step to start such a sequence of
stress stems from the inadequacy of receipts in meeting
processes
the growing expenditure requirements. Reflecting the
19. For all those women who perpetually complained about
fiscal stress, the expenditure for development activities,
how all cars are designed for men, company is out with a car which are directly related to growth, has suffered. On other
especially suited for them. The Your Concept Car………… is hand expenditure on non developmental purposes, largely
a dream come true. It has a keyless entry, additional storage committed, has witnessed a steady rise. …… in favor
space, a lower hood and the back seat screen going all the of developmental expenditure in order to enable higher
way till the rear end so that you know exactly where the car growth. That the state of finances of States is in disarray is
ends. Add to this a parking aid for parallel parking and their beyond dispute. The state finances have not been properly
most advanced technology, Ergo vision that scans the body managed not only by the states but also by the planning
at the dealership, stores the data and every time you hop commission and the central Government, which include
into the car, automatically adjusts the height of the seat, the economists who do not see states as autonomous responsible
steering wheel, the distance between them and everything organizations.
else to your specifications. Even though it is a concept car

Downloaded From : www.EasyEngineering.net


Downloaded From : www.EasyEngineering.net

Paragraph Completion  l 317

(a) The crucial issue, therefore, is to bring about of goods. ………….” that even in the Germany of 1923,
improvement in the finances with a view to restructuring after prices had soared hundreds of billions of times, high
expenditure officials and millions of Germans were blaming the whole
(b) The crucial issue, therefore, is to analyse the finances thing on a general “shortage of goods”-at the very moment
with a view to see what can be done to expenditure when foreigners were coming in and buying German goods
(c) Hence, it can be said that management of finances is with gold or their own currencies at prices lower than those
important vis-à-vis management of expenditures of equivalent goods at home.
(d) Therefore, Governments have to mend their way and (a) Yet so stubborn is the fallacy that inflation is caused by a
balance the finances and the expenditures “shortage of goods
23. Under taxation is at the roots of the Indian fiscal problems. (b) Yet people believe on such fallacy to un imagined level
The available evidence shows that the tax -GDP ratio in (c) Yet so wide is acceptance
India is lower than the level it should have for its per capita (d) Yet so timely and abrupt is response to fallacy
GDP by at least 2.5 per-cent. It is, therefore important to
26. The cure for inflation, like most cures, consists chiefly in
focus reform efforts to increase the tax ratio. Of course, this
removal of the cause. The cause of inflation is the increase
does not mean that strategy to increase the tax ratio lies in
of money and credit. The cure is to stop increasing money
increasing the tax rates. The strategy is to reiterate that tax
and credit………………... It is as simple as that. Although
administration is tax policy. All exemptions will not go.
simple in principle; this cure often involves complex and

ww
Politically, it is not possible. The world over, there was a time
when we thought that equity in tax policy meant reducing
the incomes of the rich. But today’s tax philosophy is that
disagreeable decisions on detail. Let us begin with the Federal
budget. It is next to impossible to avoid inflation with a
continuing heavy deficit. That deficit is almost certain to be

w.E
equity in tax policy is increasing the incomes of the poor.
The incomes of the poor can not be increased by reducing
those of the rich……….
(a) As they are the central point of any economy their
financed by inflationary means-i.e., by directly or indirectly
printing more money. Huge government expenditures are
not in themselves inflationary-provided they are made
importance is preemptory
asy
(b) As they have the real remote control in their hands and
poor can not see that.
wholly out of tax receipts, or out of borrowing paid for
wholly out of real savings. But the difficulties in either of
these methods of payment, once expenditures have passed
(c) As they have the capital for investment and give
employment to the poor. En a certain point, are so great that there is almost inevitably a
resort to the printing press.
(d) As they have lobbying power to decide the fate of the
poor. gin (a) The cure for inflation, in brief, is to stop inflating
(b) The cure for inflation ,in brief ,is to think positively
(c) The cure for inflation is planning small things with little
24. ………….Enterprises world wide are therefore, now
putting in place an integrated framework for risk
management, which is proactive, systematic and covers eer
thoughts
(d) The cure of inflation lies in inflation itself
the entire organization. Banks in India are also moving
from the individual silo system to an enterprise -wide risk
management system. This is placing greater demands on the ing
27. The India Union has had more than 55 years of experience
with fiscal federalism operating within the framework
of the parliamentary democracy and planned economic
risk management skills in banks and has brought to the fore
the need for capacity building. While the first mile-stone .ne
development. A comprehensive review of fiscal federalism
in independent India is, therefore, overdue. There have been
would be risk integration across the entity, banks would do
well to aggregate risk across the group both in the specific
risk areas as also across the risks.
(a) Banks are most risk prone of all the financial institutions.
(b) Banks were managing each risk independently, in
federalism……………… Now that single party has been t
feeble protests in form to time about the sprawling powers
of the Central government eroding the foundations of fiscal

dislodged from power in some of the States and parties of


different hues and colors are holding office, a candid and
isolation, which is no longer inadequate comprehensive review of all the aspects of Centre-State
(c) It is about risk level at which an enterprise is operating relations and the working of fiscal federalism in particular
to have or not have risk management system is important and necessary.
(d) Risk management in India is lagging for banks in
(a) But these voices were curbed by all parties.
comparison with other parts of world
(b) But democracy does not allow a mechanism to redress
25. One of the most stubborn fallacies about inflation is the
assumption that it is caused, not by an increase in the this problem
quantity of money, but by a “shortage of goods.” It is true (c) But fiscal federalism is all about ignoring and moving
that a rise in prices (which, as we have seen, should not be on with protests
identified with inflation) can be caused either by an increase (d) But these voices were drowned by the overwhelming
in the quantity of money or by a shortage of goods-or partly influence of the same party at the centre and the States
by both. Wheat, for example, may rise in price either because 28. Do firms need banks, or can they make do with stock
there is an increase in the supply of money or a failure of the markets? Do firms need stock markets, or can they make
wheat crop. But we seldom find, even in conditions of total do with banks? Alexander Gerschenkron long ago argued
war, a general rise of prices caused by a general shortage that economically “backward” countries could not trust
decentralised capital markets to provide their largest firms

Downloaded From : www.EasyEngineering.net


Downloaded From : www.EasyEngineering.net

318  l  Paragraph Completion

sufficient funds……………... More recently, finance (a) And it is going to achieve number one status sooner
theorists have reasoned from agency theory and the (b) It is eying for number one spot in world economy riding
economics of information to much the same result. And on its recent technological developments
the transition in Eastern Europe has given the issue a (c) Japanese Economy is going through recession and is
programmatic touch: what should scholars tell the new bound to slip to lower stands in world economy
finance ministers to do about banks and stock markets? (d) Only USA and China have a higher GNP.
(a) Instead, they needed banks 30. The prospect of renewed war between India and China is,
(b) Instead, they needed centralization mechanisms only for now, something that disturbs the sleep only of virulent
(c) Hence banks and Stock markets are not needed nationalists in the Chinese press and retired colonels in
(d) Therefore Banks score over Stock Markets for them Indian think-tanks. Optimists prefer to hail the $60 billion
29. The Japanese economy is one of the third largest in the world in trade the two are expected to do with each other this
……………….. The Japanese currency is the Yen. Japan’s year. But the 20th century taught the world that blatantly
main export goods are cars, electronic devices and computers. foreseeable conflicts of interest can become increasingly
Most important trade partners are China and the USA, foreseeable wars with unforeseeably dreadful consequences.
followed by South Korea, Taiwan, Hong Kong, Singapore, Relying on prosperity and more democracy in China to

ww
Thailand and Germany. Imports: Japan has a surplus in
its export/import balance. The most important import
sort things out thus seems unwise. Two things need to be
done. First, the slow progress towards a border settlement

w.E
goods are raw materials such as oil, foodstuffs and wood.
Major supplier is China, followed by the USA, Australia,
Saudi Arabia, South Korea, Indonesia and the United
needs to resume. The main onus here is on China. It has
the territory it really wants and has maintained its claim to
Arunachal Pradesh only as a bargaining chip. It has, after all,

asy
Arab Emirates. Industries: Manufacturing, construction,
distribution, real estate, services, and communication are
solved intractable boundary quarrels with Russia, Mongolia,
Myanmar and Vietnam …………………………..

En
Japan’s major industries today. Agriculture makes up only
about two percent of the GNP. Most important agricultural
(a) Surely it cannot be so difficult to treat with India?
(b) Surely it will be more difficult with India?
product is rice. Resources of raw materials are very limited
and the mining industry rather small.
gin (c) Can it solve dispute with India with its non democratic
values?
(d) With India intentions are not clear.

LEVEL-II
eer
DIRECTIONS (Qs. 1-18):  Each question is a logical sequence ing
2. The size and age of our cosmos are beyond ordinary human
of statements with a missing link, the location of which is shown
parenthetically [(....)]. From the four options available choose the .ne
understanding. Lost somewhere between immensity and
eternity is our tiny planetary home. (....) And yet our species
one that best fits into the sequence.
1. Making a mistake is always a miserable experience, and this
one surely was more sickening than most. As I rethought
the case I wondered why I had cut the antibiotic course
t
are young, curious and brave and shows much promise. In
the last few millennia we have made the most astonishing
and unexpected discoveries about the cosmos and our place
within it, explorations that are exhilarating to consider.
short because, in retrospect, it seemed quite foolhardy. (....) (a) Humans have always been ignorant about the intricacies
And because of this, I didn’t want to inflict more pain. The of the cosmos.
potentially distorted judgment of a physician caring for a (b) There has not been much need to delve into the details
loved one is well-worked territory. The traditional argument of universe so far.
is that doctors may underestimate the severity of illnesses (c) In a cosmic perspective most human concerns seem
because they are unable to accept grim information about insignificant, even petty.
loved ones. Hippocrates, in fact, cautions against treating (d) The mankind has been analogously tiny.
one’s own family. 3. Investors, managers and workers are alike thrown into
(a) I tried to zero down the reason, maybe I thought that conflict and confusion. Strange alliances spring up. New
the patient has already had enough pain. forms of judo are invented. (....) Today in addition, they hire
(b) And then I realised the source of the problem: I had
investment bankers, lawyers and tax experts – purveyors
made the mistake because I liked the patient.
of specialised knowledge – hoping to become part of a
(c) I could figure out that I wanted to be a friendly doctor,
restructuring deal rather than its victim. Managers seeking
taking up the persona of Santa Clause.
to head off a takeover, or to buy their own firm, along
(d) The trauma was scaring the patient, and that was causing
with investors seeking to profit from such upheavals, are
him psychological problem in getting well soon.

Downloaded From : www.EasyEngineering.net


Downloaded From : www.EasyEngineering.net

Paragraph Completion  l 319

increasingly dependant on timely, pinpointed information. measures to revive the dying art practices.
Knowledge is a key weapon in the power struggles that 6. Gone are the days when this vast country – Canada is the
accompany the emergence of the super-symbolic economy. second largest country in the world after Russia – was
(a) In the past, labour unions exerted power by striking or written off as large tracts of ice, more known for its hockey
threatening to do so. and tourism than for its business potential. There was a
(b) Previously, labour unions were smug and complacent. time when this lowly populated country ( Canada has just
(c) The employees in past companies just filed petitions and 10% of US’s population ) attracted large numbers of blue-
made silent complaints. collar migrant workers. Somewhere along the way, however,
(d) In old times fighting duals was the popular way of things changed. (....) Besides an abundance of skilled labour,
sorting out problems between two individuals. market place credentials and proximity to the U.S, the costs
4. Slavery existed in America and the vested interests of of doing business in Canada are very low and to top it all,
Southern colonies defended this social evil with all its it is a safe and clean place. Not surprisingly, the nation is
might. America witnessed the horrors of a civil war. On the emerging as a hot investment destination and a viable trade
side of the slaves was the American President–Abraham partner for many companies and countries.
Lincoln–the man with a vision and with a courage to give (a) Canada has over the years grown into a prospective
his vision a practical shape. (....) Lincoln died but with his nation capable of competing with the best in the world.
The transparent and unbiased manner in which its

ww
death, he proved for all times to come, that slavery is an evil
and equality of man is the only means for ensuring peace
and prosperity in the world.
government governs, has given it a credibility which
prompts other nations to develop all round relations

w.E
(a) Slaves in America were oppressed. They were wanting
to get freed but did not have suitable leadership to guide
them.
(b) Lincoln emancipated the slaves and eventually fell a
with it.
(b) Canada with beautiful landscapes and a rich flora and
fauna can very easily claim itself to be the best place
to invest in. Academically too, it has, over the years,

asy
victim to the bullet of an assassin. Indeed a glorious
sacrifice, truly a ‘messiah’ of the American slaves.
offered the best courses and which have received due
acceptance all over.
(c) The American President displayed immense courage
and rescued the slaves from the bondages of the
Southern colonies. En (c) Today, Canada is the only G7 country with a surplus
and boasts of the fastest growing GDP. Its becoming a
preferred destination for knowledge based entrepreneurs
(d) The Southern colonies tried desperately to staff off
Lincoln’s resistance but to no vain. It made them all the gin and has some exciting tech clusters.
(d) Over the years the image of Canada has changed from
more infuriated.
5. In this time of advanced electronic viewing, puppetry is still
enthralling audiences. Puppetry is no longer a marginalised eer
that of a country just big in size to now a country big in
business ventures and entrepreneurship. This is just the
beginning as the Canadians are ecstatic and vowed to
folk art that amuses children or tells royal Rajasthani tales
through petite figures bobbing around in tune to the tugging ing
more of every thing.
7. It would be not be an overstatement to say that the consumer
strings of their manipulators. It is now an assimilated
ingenuity combining real actors, modern dancers, digital
lighting and a variety of gloved, hand-manipulated, string .ne
products industry is on an eternal search for new ways to
drive top-line growth. There are a plethora of challenges that
companies need to grapple with. Maturing brands, choosy
and mammoth puppets to present stories of the here and
now. (....) Caricature puppets are now used to convey social
messages or to get away with biting political satires. India
now has more than 50 modern puppet groups, which work
t
customers, the consolidation of retail space, continued
pressure from retailers for control of product marketing,
limited access to customers, and the push to discover
the next best selling brand are just a few. (....) Also, many
on the theorem of fusing diverse arts into one performance. consumer products companies have deployed a pastiche
(a) Puppetry, like the other art forms, has lost its charm and of homegrown, best-of-breed and legacy systems. The
attraction as the latest electronic gadgets are diverting result of such an amalgam of platforms is that fragmented
the attention of the viewer to more high tech past times. customer and customer information reaches the wrong
(b) Poems, educational messages, wildlife themes even team members at the wrong time, leading to frustrated,
black comedies are choreographed with provocative unsatisfied customers and unresponsive consumers.
music, resulting in spectacular acts which has served as (a) The consumer has come of age. His demands are to be
a revival to the almost obsolete art . taken seriously and not to be ignored. The awareness
(c) India’s rich cultural heritage has been the mainstay of about the quality, quantity, and most critically, about
Indian tourism. Not only the ancient monuments but cost has made him call the shots.
also the various cultural practices have lured tourists, (b) The consumer forum has instilled a sense of security in
even braving the sheer heat. the consumer which has made his approach tactful and
(d) The government should be more keen to take necessary challenging. He is more confident now as he knows that
measures in order to maintain the historical images there is someone to back him up.
depicting our rich background and also take sufficient (c) The businessman, especially the retailer is in for a tough

Downloaded From : www.EasyEngineering.net


Downloaded From : www.EasyEngineering.net

320  l  Paragraph Completion

time. Gone are the days when he was not worried about bars in the backdrop of mountains. Rajasthan is changing
customer satisfaction, but now with the advent of severe for the better. After years of neglect, faulty town planning,
competitiveness, he has to dish out his best. inferior infrastructure, violation of the basic principles of
(d) Not to forget mature markets, globalisation, the architecture in not maintaining even plinth levels, and a
management of key accounts, declining margins, stereotyped society with dull evenings, there is positive
reduced revenue, and the challenge of working together transformation that strikes someone visiting the state after a
with retailers to deliver value. gap.
8. The current economic boom seems distinctly unkind to (a) Indian cultural heritage is internationally acclaimed and
workers. At a time when most segments of the Indian make foreigners flock in lure of a beautiful hierarchy.
economy are shining bright, one large area of darkness Paradoxical is the view that a lot many historical and
is employment. The decade-long downsizing in the cultural settlements are in a ruinous state. Of late, the
manufacturing sector and the much desired shedding of administration has realised their ignorance and have put
some public-sector jobs has created the impression that in their resources for renovating these great memoirs of
India has entered an era of jobless growth. If true, it could the past era.
take the sheen off all other economic achievements. (....) (b) It is not just the government’s odd realisation that
According to one of India’s largest recruitment firm, the heritage has to be protected, there is also a growing
National Employment bureau, with its 939 employment feeling even among corporate houses and individuals

ww
exchanges across the country, placed 1.61 lakh individuals in
2002. That was 10,000 less than the placements done by the
bureau in 2002. In contrast, private recruitment companies
that it pays to evolve modern comforts around heritage.
(c) It’s indeed a pity that we Indians have grossly ignored
the exquisite elegance of history which was intentionally

w.E
collectively found jobs for 3.5 lakh individuals in 2003 and
expect their placements to go up by 25% in 2004.
(a) Thankfully this is not the reality. Sure, industry and
done for leaving behind their impression of great skill
and craftsmanship. The bulk of the monuments depict
their richness of taste and the urge to leave behind

asy
government have not been creating new jobs, but
services and agriculture (which together account for
75% of the economy) are generating employment. It is
depictions of their great civilisations.
(d) Revival of the lost and almost extinct periods of our
glorious history has indeed been an encouraging effort
tracking jobs in these two sectors.
En
just that there is not any credible and regular method of
by our government. The relics, almost on the verge of
extinction into anonymity, are being restored by the
(b) The economic revolution has brought about a radical
change in the service sector as claimed by the analysts
as well as the rulers. However, the masses have a gin Archaeological Survey of India. Also along with, they
are being pepped up in order to attract tourists.
10. What on earth would Indian cricket have done with a player
contrasting theory which very strongly denies all the
great claims made by the opportunists. The common
man continues to strive under the blazing sun and yet is
eer
like Lala Amarnath today? (....) But be glad he played in
wilder days. Amarnath was the enfant terrible of the earlier
not rewarded to the extent he deserves.
(c) Employment in India has been a curse for the citizens ing
days of Indian cricket, an attacking batsman with a temper
to match. Amarnath was no less entertaining off the field .
He had women proposing to him after he scored the first
as the overpopulation has outclassed the proportion of
available jobs. Also an issue undermining the pathetic
condition is the literacy rate which is alarmingly .ne
Indian test century. He was an expansive observer of modern
cricket, a player’s player and a man who knew a good quote
disproportionate to the requisite qualification for
suitable placement.
(d) The Indian Government has earnestly tried to generate
jobs as per the qualifications in varied fields and skills.
t
well before sound bytes were merely the scratching of statics.
(a) The yeomen of Indian cricket have gone into oblivion.
They did the country proud at a time when things were
crude. The cricketing gears were too amateurish and
However, the system implementing these employment their wages too were meagre. Lala Amarnath survived
opportunities get contaminated by the evils of all the difficulties but to what avail?
corruption and other malpractices hovering around like (b) Lala Amarnath is India’s reflection of the greatness of
deadly parasites. these players of yester years. But, unfortunately they did
9. Rajasthan is building its present on its past. And this creativity not get the rewards that our present day cricketers are
goes far beyond the success stories of converting palaces and getting. The players of today are better equipped and
havelis into hotels. (....) Express highways, combined with financially very soundly placed. So, how do we show our
improvement in state highways and link roads to religious gratitude to these great ‘Gentlemen ‘ of cricket.
places, are making people and the bureaucracy understand (c) Either trampled on his spirit or, equally bad, lumped
that the state can be modernised while maintaining its him with a ‘brand image’ and had him endorse fizzy
USP–heritage. That monuments can be beautified and that drinks with a cheesy smile.
foreigners do not come here looking for decay and dirt. (d) Lala Amarnath, like many other veterans, have been
A senior bureaucrat who returned recently from Europe meted out a very shabby and disgraceful farewell when
said tourists can be made to extend their stay by making compared to what our present day players get. They have
night life interesting and suggested running cafeteria and got just a meagre against the laurels they had brought
for their country.

Downloaded From : www.EasyEngineering.net


Downloaded From : www.EasyEngineering.net

Paragraph Completion  l 321

11. The tea plantation industry provides direct employment the mining companies pay the international community for
to more than a million workers in the country. Next to the the privilege of mining? The mining companies care more
Indian Railways, the tea industry is the largest employer of about avoiding big losses than they do about making big
organised labour force being drawn from tribal and socially gains. For them, deep-seabed mining is a major investment.
weaker sections of the society. (....) As the tea plantation They want to reduce the risk. The international community,
areas in India were in the initial stages, sparsely populated, on the other hand, is concerned with revenue. If some
the workers in the industry had to be recruited from company is going to make a lot of money out of “the
considerable distances. In north India, a majority came from common heritage of mankind,” the rest of the world wants
Bihar and Orissa and in the south from the plain districts. a generous share. In this difference lies the potential for a
Most of them belong to the scheduled caste or the scheduled bargain advantageous to both sides. ( ... ). Exploiting the
tribe. difference in aversion to risk, the proposed treaty provides
(a) In a country like India, employment has always been for charging the companies low rates until they recover their
a major problem . Tea is consumed heavily in our belt investment and much higher rates thereafter, when their
and as such its consumption is very high, which in turn risk is low.
induces a process by which the demands can be met (a) Risk can be traded for revenue.
easily. The industry calls for workers to join and help (b) Risk, obViously needs to be taken care of more than

ww
them increase their yield.
(b) Thanks largely to the tea industry in India which has
given a tremendous boost to our exports. Also equally
revenue.
(c) Revenue, if earned in high figures, can override the
magnitude of risk.

w.E
grateful are we to the workers who have sweated it out
endlessly . But have we generously rewarded them for
their efforts?
(c) The tea industry ranks second and pools in substantially
(d) Obviously, no solution is feasible and the deal eventually
falls through.
14. Reason and be open to reason. What makes the negotiation
a joint search is that, however much you may have prepared

asy
in the country’s economy. We take pride at this
achievements. But have we ever thanked those countless
various objective criteria, you come to the table with an open
mind. ( ... ). A police union might, for example, insist upon a

En
heads who toiled endlessly to make this possible or we
are still discriminating amongst the male and the female
worker?
raise of a certain amount and then justify their position with
arguments about what police in other cities make. This use
of standards usually only digs people even deeper into their
(d) About half the labour force consists of women workers.
Mostly women are engaged in plucking and they enjoy gin position.
(a) In negotiating this way, people use time more efficiently,
equal wages.
12. ( ....). Rather, in healthy economies, many successful and
prosperous corporations coexist with intensely competitive eer
talking about possible standards and solutions.
(b) In most negotiations, people use precedents and other
objective standards simply as arguments in support of a
markets in a state of vigorous and creative tension with
one another, each contributing to economic progress but ing
position.
(c) Independent standards are even more important to
in different ways. Companies create new value for society
by continuously creating innovative products and services
and by finding better ways to make and offer existing ones; .ne
efficiency when more parties are involved.
(d) People should make use of precedents and other
objective standards in their arguments to support their
markets, on the other hand, relentlessly force the same
companies to surrender, over time, most of this value to
others. In this symbiotic coexistence, companies and markets
interact jointly to drive the process of creative destruction
position and establish their righteousness.
t
15. If pushing back does not work, what does? How can you
prevent the cycle of action and reaction? Do not push back.
When they assert their positions, do not reject them. When
that Joseph Schumpeter, the Austrian economist, showed to they attack your ideas, don’t defend them. When they attack
be the engine that powers economic progress in capitalist you, don’t counterattack. ( ... ). Instead of pushing back,
societies. sidestep their attack and deflect it against the problem.
(a) Most companies utilise markets to appropriate value for As in the Oriental martial arts of judo and jujitsu, avoid
themselves at the cost of social welfare. pitting your strength against theirs directly; instead, use
(b) Most companies intensify competition in markets to your skill to step aside and turn their strength to your ends.
contribute to the economy through self-interest. Rather than resisting their force, channel it into exploring
(c) Most companies do not usurp markets to appropriate interests, inventing options for mutual gain, and searching
value for themselves at the cost of social welfare. for independent standards.
(d) Most companies strive towards creation of unhealthy (a) React in your own way, by being proactive.
environments to intensify competition and usurp the (b) Break the vicIous cycle by refusing to react.
market. (c) Counter the incursion head on.
13. One last kind of difference, which you may capitalise on, is (d) Do not deflect it against the problem.
aversion to risk. Take, for example, the issue of deep-seabed 16. Organisational changes and moving a cultural baseline
mining in the Law of the Sea negotiations. How much should is a difficult challenge. It takes respect, intensity, and

Downloaded From : www.EasyEngineering.net


Downloaded From : www.EasyEngineering.net

322  l  Paragraph Completion

commitment to stay the course. Some days will be tougher DIRECTIONS (Qs. 19 to 41) :  A paragraph is given below
than others, due to human errors and lapses. Great leaders from which one of the lines has been deleted. From the given
will not be put off by others; they will have the courage to options, choose the one that completes the paragraph in the most
stand up and establish rules. Their goal will be to go to a appropriate way.
higher ground, and they will constantly use their own What is bad writing? One definition of bad writing is that it
actions as a model for others to follow, reflecting consistency only entertains but does not edify. It does not raise ‘important
and stability. ( .... ). Establishing a sense of urgency, yet being questions’ regarding political or social ills but, instead, runs away
able to spend time with those who want to move forward from them.
and share a vision for the future, is important. They will not 19. Put simply, bad writing reinforces popular attitudes even if
check the daily emotional temperature of their environment, it’s at the cost of harming a cultural fabric.
as they are in it for the longer haul. Bad writing’s goals are self-serving. ...................................
(a) When tough decisions are required, wise leaders are (a) It aims to make money for its publisher and author and
patient with themselves. win popularity for its writer without caring two hoots
(b) You cannot succeed through intellectual arrogance. about the greater common good or making the reader
(c) They will emulate through precedents and smother think beyond the text.
individual enthusiasm and the desire to contribute to an (b) Good writing - especially fiction – on the other hand -
organisation. is as man-made and culturally-constructed method of

ww
(d) They will create a culture, which fosters enervated
approaches.
enquiry used to analyse and judge one’s views.
(c) However, no text - and, by extension, an author - is
universally or eternally good or bad.

w.E
17. Originally mutual funds were heralded as a way for the
little guy to get a piece of the market. Instead of spending
all your free time buried in the financial pages of the Wall
Street Journal, all you have to do is buy a mutual fund and
(d) Thus, if good writing is to prevail, ‘good writers’ must
slug it out in the open with ‘bad writers’.
20. Policy failures come in many shapes and sizes. But they

asy
you’d be set on your way to financial freedom. (....). Mutual
funds are an excellent idea in theory, but, in reality, they
are almost invariably accentuated by the choices that have
to be made in periods of financial shortage. Yesterday
the coalition government found itself confronted by two
haven’t always delivered. Not all mutual funds are created

En
equal, and investing in mutual isn’t as easy as throwing your
money at the first salesperson who solicits your business.
of them, one in policing policy and the other in higher
education. The failures are radically different in many
(a) As simple as that!
(b) Bingo! You’ve become an investor. gin details. But they have this in common: they are both serious
failures, and they were both foreseeable. They could both
have been mitigated by clearer policymaking at an earlier st
(c) This causes a positive feedback cycle to begin; the more
you earn, the more you invest and so on.
(d) As you might have guessed, it’s not that easy. eer
age..........................................
(a) On the policing cuts, it is the opinion that Britain has
18. The first known civilisation developed in three river
valley systems in Eurasia; the Tigris-Euphrates Valley ing
more police officers than it needs.
(b) Instead each has been allowed to fester and become
more difficult to solve politically.
(Mesopotamia), the Nile Valley (Egypt); and the Indus
Valley. (....). Each society experienced a long transition
during which techniques of maintaining large-scale societies providing some distraction. .ne
(c) Downing Street must be thanking Prince Andrew for

were developed. Remains of temples and palaces reflect the


emergency of priestly and political managerial classes. These
societies utilised technologies of irrigation to manage water
resources and skills in metallurgy made new materials like
(d) Police numbers have increased without proper regard to
social need.
21. It seems Prime Minister Manmohan Singh’s top scientific
adviser CNR Rao and his band of merry men - three reputed
t
bronze available. scientists from top scientific institutions of the country - have
(a) But due to limited technical know-how and increasing been cherry picking some interesting material from the work
competition between peers, they remained nomadic for of other scientists and passing them off as their own. This is
long. not the first time, people - reputed (and talented) people that
(b) Changing environmental conditions, population is - have been caught doing such things. Often people - the
pressures, and the evolution of available technologies kind-hearted ones - say imitation is actually the sincerest
are possible reasons for the emergency of civilised form of flattery. .....................................................
societies. (a) But the truth is, not many people find it amusing when
(c) Surprisingly, the social in each civilisation consisted for they find that their work has been picked up by someone
four classes; maybe it was the impact of interaction of else without giving them any credit.
other civilisation with the Indus valley civilisation.
(b) To cut a long and not-a-plagiarised-story short, the
(d) The so-called modern age tools like the battery and
Indian scientists have apologised to Advanced Materials,
magnetic motors have been found, which indicate that
a prestigious journal, for “reproduction of text from
these civilisation were far more developed that what we
an article” that appeared in another journal, Applied
may think.
Physics Letters, in 2010.

Downloaded From : www.EasyEngineering.net


Downloaded From : www.EasyEngineering.net

Paragraph Completion  l 323

(c) In other words, the scientists forgot to issue a ‘footnote’ the shelves for an audience that no longer bothers to walk
that the four contentious lines were picked up from through the stacks……………
another source. (a) This has put both the internet search engine and the
(d) But the thing about plagiarism is that it is considered so publishers on an unwarranted collision course
if the offenders are caught. (b) Since the case might drag on for years to the detriment
22. ……………………..and the proponents of market reforms of both the parties, an out of court settlement is well
have no plans for those who do not have the resources and advised
income to buy even two meals a day. The signals are clear (c) Isn’t it rather difficult, or even in fructuous to protect
that those who cannot pay for their food have no right to copyright on published text in this internet age?
survive. These poorer sections of society are reduced to (d) Perhaps both the parties ought to try and appreciate the
mere victims, beneficiaries, clients and recipients. In this other’s viewpoint, as well as legitimate apprehensions,
dichotomous relationship, the state is seen as the ‘dole giver’ but with the overall goal of the public good in mind.
and the people the ‘dole receiver’. It must be recognized that 25. Relations between the factory and the dealer are distant
irrespective of market-governed politics, people remain and usually strained as the factory tries to force cars on the
bound to survival, livelihood and identity issues. dealers to smooth out production. Relations between the
(a) The governments have ceased to govern. dealer and the customer are equally strained because dealers
continuously adjust prices - make deals - to adjust demand

ww
(b) When the market is allowed to govern, the government
becomes powerless to effect any radical social changes.
(c) Elections have failed to make democracy distributive
with supply while maximizing profits. This becomes a system
marked by a lack of long-term commitment on either side,

w.E
and justice oriented.
(d) It is about market reforms and absence of plans for the
poorest of the poor.
23. To succeed in today’s crowded marketplace where most of
which maximize feelings of mistrust. In order to maximize
their bargaining positions, everyone holds back information
- the dealer about the product and the consumer about his
true desires…………………

asy
the products and advertising look exactly the same, a small
business owner must stand out, shouting above the din with
(a) As a result, ‘deal making’ becomes rampant, without
concern for customer satisfaction.

En
a message so clear and compelling that prospects stop and
take notice. It’s a matter of business survival. Unfortunately,
most entrepreneurs quickly retreat to the supposed security
(b) As a result, inefficiencies creep into the supply chain.
(c) As a result, everyone treats the other as an adversary,
rather than as an ally
of sameness, soon to be lost in a sea of anonymity and a tidal
wave of frustration. In effect, albeit at a subconscious level, gin (d) As a result, fundamental innovations are becoming
scarce in the automobile industry.
they are saying, “I don’t want to be different”.
In back room offices and store fronts everywhere, salespeople
are telling business owners they should do this or that kind eer
26. The tax system of India encourages borrowing by granting
its taxpayers tax relief for interest paid on loans. The system
also discourages saving by taxing any interest earned on
of ad because it worked so great for their competitor. The
owners nod and sign on. It’s already proven to be a winner, ing
savings. Nevertheless, it is clear that India’s tax system does
not consistently favor borrowing over saving, for if it did,
right? WRONG……....
(a) To make your advertising work, follow the principle if
your competition is doing it, don’t. .ne
there would be no.........................
(a) tax relief in India for those portions of a taxpayer’s
income, if any, that are set aside to increase that
(b) Following your competitor is a sure recipe for disaster.
(c) Win the battle without a fight.
(d) It will fill people with a sense of déjà vu.
24. Google, the internet powerhouse, seeks to organise the entire
taxpayer’s total savings
t
(b) tax relief in India for the processing fees that taxpayers
pay to lending institutions when obtaining certain kinds
of loans
world’s information. The company has told publishers it will (c) tax relief in India for interest that taxpayers are charged
delay until November its work on copyrighted texts and will on the unpaid balance in credit card accounts
not scan any items that the copyright owner does not want (d) taxes due in India on the cash value of gifts received
included. The Assn. of American Publishers was outraged by taxpayers from banks trying to encourage people to
by this offer, saying Google is trying to turn copyright open savings accounts
law inside out. Google should have to ask permission to 27. ……………….. Unemployment typically continues to rise
copy a book for its database, they say, it shouldn’t be up to even after GDP starts to increase, so pain for workers is far
publishers to object. Google argues that it is making a fair from over. Already 9.5% of the workforce is unemployed, and
use of the books. The dispute could easily wind up in the all of country’s metropolitan areas reported unemployment
courts. Building a guide to the contents of books is hardly rates of at least 10% in June. More jobless will probably
the same as making bootlegged copies or plagiarizing. It’s a mean less shopping and a slower recovery. The latest
monumental and costly task, and publishers have given no consumer-confidence numbers show that people are jittery.
reason to believe they can do it for themselves. Unless their The quarterly GDP report also makes it clear that consumer
works are as well integrated with the Net as other forms of spending, which rose slightly in the first quarter, dropped
information and entertainment, they may be left waiting on again in the second, by 1.2%. The good news, therefore, was

Downloaded From : www.EasyEngineering.net


Downloaded From : www.EasyEngineering.net

324  l  Paragraph Completion

more a result of government stimulus than evidence of a consider. Utilities such as gas, water and especially electricity
real, sustainable recovery in private demand. are expensive, and phone rates are high. For international
(a) A greater worry is the bleeding in country’s labor calls, consider internet phones, callback services and other
market. offers for the expat community.
(b) The Finance Department has revised its estimates of just (a) Consequently; even tiny apartments in the city center
how bad 2008 really was. are very expensive
(c) Figures released by Commerce Department confirmed (b) This land is unauthorized land usurped by Land Mafia
what most had been expected in city
(d) New GDP figures suggest some hope for country’s (c) A new bubble - Reality bubble similar to housing bubble
economy. But the pain is far from over. of West is waiting to burst
28. Everything in New Delhi is extreme. It is a city of the (d) City is breathing on its seams
incredibly rich and the miserably poor. For the rich, there 31. Now digest the main historical event of this week: China
are expensive private schools and hospitals, concert halls has officially become the world’s second-biggest economy,
and theatres - although fewer of those than formerly - overtaking Japan. In the West this has prompted concerns
and restaurants. The poor are on a hiding to nowhere: all about China overtaking the United States sooner than
public facilities, schools and hospitals and housing, are previously thought. But stand back a little farther, apply a

ww
deteriorating. But then there are some world known social
workers in it. There are thousands of heroin addicts, and an
equal number of carriers of the Aids virus. …………….This
more Asian perspective………… These two Asian giants,
which until 1800 used to make up half the world economy,
are not, like Japan and Germany, mere nation states. In

w.E
is Delhi for you.
(a) But then there are thousands of people who are social
workers
(b) But then some of rarest examples of humanity come
terms of size and population, each is a continent-and for all
the glittering growth rates, a poor one.
(a) China’s longer-term contest is with that other recovering
from this city
asy
(c) But then there is a faith in the City
economic behemoth: India
(b) China’s longer term contest with USA is going to end in
near future
also.
En
(d) But then there are people addict to charity and altruism

29. Computers are used in banks for a variety of reasons. They


(c) China is now focusing on Asian Market space more
than other markets
help bank personnel operate more efficiently and effectively.
Computers are used to track certain transactions and they gin (d) China is going in tandem with other Asian economy -
India
32. North India lies in the Indo-Gangetic plain. Towards the
help process other customer information as well. Without
computers, it would be very hard for a bank to offer good
customer service day in and day out. Computers help a bank eer
North is the Himalayas, which separates the country from
Central Asia. The Vindhya ranges separate the North from
save time and money, and can be used as an aid to generate
profits. In nutshell they have become indispensible part of ing
the South. South India is situated in the Peninsular Deccan
Plateau. This region has the Arabian Sea in the west, Bay of
Bengal in the east and Indian Ocean in the south. When
the banks. Bank personnel become so helpless in absence
of their machine that nervousness is evident on their
faces while technical glitch renders the computer systems .ne
talking about the racial differences, the North Indians are
termed as Aryans, and the South Indians as Dravidians. In
dysfunctional for a small period of time. ……… once again
queues starts to move and crowd starts to thin with fingers
starting to work on keyboards; nervousness has made way
for confidence of knowing everything at just a click of
t
physique as well, there are many differences between the
people of the South and North. The North Indians are taller,
and more strongly built than the South Indians. The South
Indians are a bit darker than the North Indians…………..
mouse. Salwar Kamiz is the widely used dress by North Indian
(a) Crowd starts to swell and nervousness starts to turn in women. On the other hand, women in the South wear saris.
fear of failure. While men in the North wear Salwar, the men in South prefer
(b) A sense of relief spreads as the problem gets rectified. dhotis. Another difference that can be seen between North
(c) There is a condition of traffic jam in bank and everything and South India is their food. When compared to the North
comes to a halt. Indian food, the South Indian food is spicier. The South
(d) It is not their fault and they are excused for it. Indians use more tamarind and coconut when compared to
30. Some of the world’s most expensive land can be found in North Indians. The North Indians use more milk products
central Mumbai………. However, housing costs are distinctly when compared to the people of South India. When talking
lower in Mumbai’s suburbs, surrounding prefectures and in about the culture, there is vast difference between North and
other regions and cities of India. Additional commuting South India. One can come across differences in their music
costs are often more than compensated by the savings on the (Northern Hindustani and Southern Carnatic), dance forms
rent, especially as many companies pay part or all of their and folks.
employees’ commuting expenses. If you prefer to live close (a) A big difference is in their style of living
to city centers, rented houses are an inexpensive option to (b) A big difference is in their fashion statements

Downloaded From : www.EasyEngineering.net


Downloaded From : www.EasyEngineering.net

Paragraph Completion  l 325

(c) A big difference can be seen in their dressing styles 35. In finance, the term ‘yield’ describes the amount in cash
(d) A big difference arises in their taste for clothes based on that returns to the owners of a security. Normally it does
their earnings not include the price variations, at the difference of the
33. An open economy is an economy in which there are total return. Yield applies to various stated rates of return
economic activities between domestic community and on stocks (common and preferred, and convertible),
outside, e.g., people, including businesses, can trade in fixed income instruments (bonds, notes, bills, strips,
goods and services with other people and businesses in the zero coupon), and some other investment type insurance
international community, and flow of funds as investment products (e.g. annuities)……... It can be calculated as a
across the border. Trade can be in the form of managerial ratio or as an internal rate of return (IRR). It may be used
to state the owner’s total return, or just a portion of income,
exchange, technology transfers, all kinds of goods and
or exceed the income. It may be used for production output
services. Although, there are certain exceptions that cannot
in other industries,Because of these differences, the yields
be exchanged, like, railway services of a country cannot be
from different uses should never be compared as if they
traded with another. To avail this service, a country has to were equal.
produce its own. This contrasts with a closed economy in (a) The term is a misnomer
which international trade and finance cannot take place. (b) The term is understood differently by different people
The act of selling goods or services to a foreign country is (c) The term has no definite meaning

ww
called exporting. The act of buying goods or services from
a foreign country is called importing…………. There are
a number of advantages for citizens of a country with an
(d) The term is used in different situations to mean different
things
36. Market trends are fluctuated on the demographics and

w.E
open economy. One primary advantage is that the citizen
consumers have a much larger variety of goods and services
from which to choose. Additionally, consumers have an
technology. In a macro economical view, the current state
of consumer trust in spending will vary the circulation of
currency. In a micro economical view, demographics within

asy
opportunity to invest their savings outside of the country.
In an open economy, a country’s spending in any given year
a market will change the advancement of businesses and
companies. With the introduction of the internet, consumers
have access to different vendors as well as substitute products
can spend more money than it produces by borrowing from
En
need not to equal its output of goods and services. A country

abroad, or it can spend less than it produces and lend the


and services changing the direction of which a market will
go. Despite that, it is believed that market trends follow one
difference to foreigners. There is no closed economy in
today’s world. gin direction over a matter of time, there are many different
factors that can change this idea. Technology s-curves as
is explained in the book The Innovator’s Dilemma. It states
(a) Together exporting and importing are collectively called
trade
eer
that technology will start slow then increase in users once
better understood, eventually leveling off once another
(b) Exporting and Importing are exclusive classes of trade
(c) Both are independent of each other and do not constitute
the term ‘trade’. ing
technology replaces it………………….
(a) This proves that change in the market is actually
consistent
(d) Together they make GDP of a country
(e) Together they are indicators of influence of a country on
(b) Change is inevitable
.ne
(c) Fluctuations with these changes do not last long
world map
34. A no budget film is a produced film made with very little,
or no money. Young directors starting out in filmmaking
commonly use this method because there are few other
(d) Be prepared for the change
t
37. The financial year 2007-08 witnessed a slew of acquisitions
across diverse sectors of the economy in India…………...
Of all sectors, steel was the most dominant in terms of
options available to them at that point. All the actors and stake sales as deals valuing $ 3.862 billion took place in
technicians are employed without remuneration, and the Q1 of 2007-08 by the Indian companies in the global
films are largely non-profit.,…………….. or uses a very arena. Energy ranked second, with automotive and auto
minimum “crew” of volunteers to assist him/her on such components close on its heels. In the domestic segment,
projects where no money or financing is available, not iron ore, aviation and steel were the most prolific in terms
including the cost of film. No-budget films are made every of mergers and acquisitions. With Indian corporate houses
day with video tapes and consumer cameras. showing sustained growth over the last decade, many have
(a) Usually the director works alone on such films shown an interest in growing globally by choosing to acquire
(b) These films are huge employers in entertainment or merge with other companies outside India. One such
example would be the acquisition of Britain’s Corus by Tata
industry
an Indian conglomerate by way of a leveraged buy-out. The
(c) A team of experts two or three in number starts without
Tata’s also acquired Jaguar and Land Rover in a significant
any significant infrastructure
cross border transaction. Whereas both transactions
(d) Director is the camera-man, light-man, script writer, involved the acquisition of assets in a foreign jurisdiction,
screen player and sometimes even act himself in the both transactions were also governed by Indian domestic
film law.

Downloaded From : www.EasyEngineering.net


Downloaded From : www.EasyEngineering.net

326  l  Paragraph Completion

(a) These acquisitions are purely Indian but covering risk category subject to a maximum ceiling fixed by RBI. In
diverse sectors ranging from automobiles to Steel the normal course, prudential exposure norms apply to all
(b) Acquisitions are like a big shark swelling a small fish loans and investments overseas including loans to sovereign
without noticeable fight put up by the latter entities………………….. Adequacy of the bank’s policy
(c) Unlike in the past, such activity was not limited to on identification, measurement and control of country
acquisitions within India or of Indian companies risk is assessed during onsite inspection by host country
(d) Globalization has brought this trend to India and Indian representatives. It is also monitored through a quarterly
companies are no longer isolated from its effects return on country-wise counter party exposure.
38. ………………. But there is, in fact, a vast difference. Merger (a) The overseas branches are governed by the host country
generally refers to a circumstance in which the assets and regulations also
liabilities of a company (merging company) are vested in (b) Host country is speculative for the success of these branches
another company (the merged company). The merging (c) Banks and host countries conduct joint audits in
entity loses its identity and its shareholders become branches in that country
shareholders of the merged company. On the other hand, (d) Host country regulations do not bind on these overseas
an amalgamation is an arrangement, whereby the assets branches
and liabilities of two or more companies (amalgamating 41. The prospect of renewed war between India and China is,
companies) become vested in another company (the for now, something that disturbs the sleep only of virulent

ww
amalgamated company). The amalgamating companies all
lose their identity and emerge as the amalgamated company;
though in certain transaction structures the amalgamated
nationalists in the Chinese press and retired colonels in
Indian think-tanks. Optimists prefer to hail the $60 billion

w.E
company may or may not be one of the original companies.
(a) Very often, the two expressions “merger” and
“amalgamation” are used synonymously
in trade the two are expected to do with each other this
year. But the 20th century taught the world that blatantly
foreseeable conflicts of interest can become increasingly
foreseeable wars with unforeseeably dreadful consequences.

interchangeably but up to a limitasy


(b) ‘Merger’ and ‘amalgamation’ are terms that can be used Relying on prosperity and more democracy in China to
sort things out thus seems unwise. Two things need to be

other
(d) ‘Merger’ and ‘amalgamation’ are not synonymous toEn
(c) ‘Merger’ and ‘amalgamation’ are synonymous to each done. First, the slow progress towards a border settlement
needs to resume. The main onus here is on China. It has
the territory it really wants and has maintained its claim to
each other
39. Being one of the central banks which was involved in the gin Arunachal Pradesh only as a bargaining chip. It has, after all,
solved intractable boundary quarrels with Russia, Mongolia,
exercise of drawing up the Core Principles, the Reserve Bank
of India had assessed its own position with respect to these
Principles in 1998. The assessment had shown that most of eer
Myanmar and Vietnam………………….
(a) Surely it cannot be so difficult to treat with India?
(b) Surely it will be more difficult with India?
the Core Principles were already enshrined in our existing
legislation or current regulations. Gaps had been identified ing
(c) Can it solve dispute with India with its non democratic
values?
between existing practice and principle mainly in the areas
of risk management in banks, inter-agency cooperation with
other domestic/international regulators and consolidated
(d) With India intentions are not clear.

.ne
DIRECTIONS (Qs. 42-45):  In each of the following questions,
four options are provided following a paragraph. Select the option
supervision. Internal working groups were set up to suggest
measures to bridge these gaps and their recommendations
have been accepted by the Board for Financial Supervision
and are now in the process of being implemented. Given
best representing the theme of the paragraph.
t
42. No one has ever conclusively proven or disproven the truth
of spontaneous human combustion, but most scientists
say that there are more likely explanations for the charred
the spread and reach of the Indian banking system, with remains. Many of the so-called victims of spontaneous
over 60,000 branches of more than 100 banks…….. human combustion were smokers who were later discovered
However, the Reserve Bank of India is committed to the full to have died by falling asleep with a lit cigarette, cigar or pipe.
implementation of the Core Principles. The Bank also serves A number of them were believed to have been under the
on the Core Principles Liaison Group of the BCBS, which influence of alcohol or to have suffered from a movement-
has been formed “to promote the timely and complete restricting disease that prevented them from moving quickly
implementation of these principles worldwide”. enough to escape the fire. Another possibility is that some of
(a) implementation is a challenge for the supervisors the fires and strange states of the victims’ bodies were the
(b) implementation is impossible result of a criminal act and subsequent cover-up.
(c) implementation should be done on trail basis (A) Nothing like spontaneous human combustion exists.
(d) Implementation is a problem for supervisors This is a myth propagated by those who want to hide
40. Indian banks having overseas operations are required to the real cause behind a burnt person. Mostly criminal
lay down internal guidelines on country risk management activity is the real cause.
and fix limits based on risk rating of the country. Limits (B) Although not explicitly rejected, the theory of spontaneous
should also be fixed for a group of countries in a particular human combustion is weakened by disclosure of other

Downloaded From : www.EasyEngineering.net


Downloaded From : www.EasyEngineering.net

Paragraph Completion  l 327

causes of burning of people, who were supposed to be lives. People who find they are unable to control important
spontaneously combusted. events may suffer from learned helplessness and depression.
(C) Every novel theory is attacked initially and spontaneous However, researchers also have found some important
human combustion is no exception. There may be some exceptions to this rule. People who find they cannot directly
cases where the cause of burning has been discovered to control important outcomes sometimes rely on secondary
be something else, but this does not rule out the theory. control, such as focussing their attention on what they can
(D) Spontaneous human combustion is an enlarged and control or by placing their trust in others.
amplified form of the chemical oxidation that happens (a) Dependability is not having faith in one’s own self. It
all the time inside the body. Scientists are evading this also makes the individual lethargic and sometimes, just
straightforward explanation. not performing because deep in the back of his mind, he
(a) A (b) B has conditioned himself to the idea that there are others
(c) C (d) D who would do the job for him.
43. The first thing to note about mindfulness is that it is not (b) Sometimes people experience increased anxiety when
particularly hard at all to be mindful. It does not take a given control over a situation or choose to relinquish
really strenuous effort to make yourself become mindful control to another person. This may reflect their
and more present. The effort is very small. The problem incapability at handling tough situations but then may
is to remember to do it! We forget it all the time. It is not also reflect their views of getting the thing done by more

ww
hard, but we just do not remember to do it. The ideal way to
become more mindful is simply to make the slight effort to
competent persons.
(c) It is indeed appreciable if a person handles difficult

w.E
be more mindful at all times.
(A) Being conscientious is strenuous and taxing; which
makes people to stay elusive.
(B) ‘Easy things are difficult to complete.’ The memory bank
situations with calm and grace. Without getting bogged
down, he should rather accept it as a challenge and work
out the suitable means of tackling such trying situations.
(d) Life is a struggle and those who bravely withstand the

asy
goes bankrupt and you may become dull.
(C) To make oneself mindful is not as difficult as one thinks
stormy conditions with gusto are the ones who are
definitely going to emerge victorious.]

do it.
En
it to be. It is difficult unless and until one remembers to

(D) Sharpness of mind is a transient feature and so is the


DIRECTIONS (Qs. 46-50): In each of the following questions a
short passage is given with one of the lines in the passage missing
and represented by a blank. Select the best out of the five answer
involved factor of memory.
(a) A (b) B gin
choices given, to make the passage complete and coherent.
[IBPS PO 2011]
(c) C (d) D
44. Hearing is our second most useful sense when it comes
to learning. Many excellent pianists respond to questions
46.
eer
Women’s rights around the world are an important indicator
to understand global well-being. A major global women’s
about their artistry with “oh, I learned to play by ear”. They
‘hear’ the melody in their mind as they play. This helps them ing
rights treaty was ratified by the majority of the world’s
nations a few decades ago. ...... These ranges from the
cultural, political to the economic. For example, women
to remember the thousands of notes that make up a lengthy
piece. Similarly the shriek of a fire alarm immediately
reminds the volunteer fireman that he is needed at the .ne
often work more than men, yet are paid less; gender
discrimination affects girls and women throughout their
firehouse. Such auditory elaboration makes it easier to
remember.
Once you have cultivated the practice of trying to hear more
as well as to see more, you will broaden your path towards
t
lifetime; and women and girls are often the ones that suffer
the most poverty. Many may think that women’s rights are
only an issue in countries where religion is law. Or even
worse, some may think this is no longer an issue at all. But
reading the report about the United Nation’s Women’s Treaty
an improved memory, which at times can prove to be a vital
and how an increasing number of countries are lodging
factor.
reservations will show otherwise. Gender equality furthers
(A) Hearing is a reflexive sense, becoming more inept if
the cause of child survival and development for all of society,
learned through auditory elaboration.
so the importance of women’s rights and gender equality
(B) Hearing and seeing together sharpens the power of
should not be underestimated.
retention, which at times can be an important factor.
(a) This treaty tackled and solved a number of issues related
(C) An ordinary individual is more sensitive towards music
to women.
as he learns to play by ear only.
(b) Why is it then, that moment still face a number of
(D) When hearing and seeing are focused together, the
problems on the domestic front?
power of retention gets confused, which becomes a
(c) Thus, the woman today is ten times more empowered
restricting factor.
as compared to a woman say about a decade ago.
(a) A (b) B
(d) Women’s activists across nations have implored the
(c) C (d) D
respective governments to take this seriously.
45. Researchers have identified a number of benefits that come
(e) Yet, despite many successes in empowering women,
from perceiving that we are in control of the events in our
numerous issues still exist in all areas of life.

Downloaded From : www.EasyEngineering.net


Downloaded From : www.EasyEngineering.net

328  l  Paragraph Completion

47. Research has shown that air pollutants from fossil fuel use the year 2010-11 saw notebook volumes rise, and for the
make clouds reflect more of the sun’s rays back into space. first time a million plus notebooks were sold in India in
This leads to an effect known as global dimming whereby a single quarter. The market has grown nearly four times
less heat and energy reaches the earth. ............ However, it is for notebooks. The demand is driven by all sectors and
believed that global dimming caused the droughts in certain a very buoyant consumer market, which prefers mobile
parts of the world where millions died, because the northern computers. Entry-level notebook prices have dropped below
the Rs. 25,000 mark; this has helped break the ice with new
hemisphere oceans were not warm enough to allow rain
customers. This drop in notebook prices has been helped
formation. Global dimming is also hiding the true power
by the drop in the prices of the building blocks that make
of global warming. By cleaning up global dimming-causing a notebook. It’s simple. With notebook volumes growing,
pollutants without talking greenhouse gas emissions, rapid the prices of the components are also bound to come
warming has been observed, and various human health and down. .........
ecological disasters have resulted, as witnessed during the (a) All this has resulted in a noticeable change in a number
European heat wave in 2003, which saw thousands of people of large government tenders for notebooks; which were
die. traditionally for desktops.
(a) This though, does not bring any relief in the problems (b) Because of this the government still prefers desktops
associated with climate change. to notebooks and has passed tenders for the same.

ww
(b) This phenomenon thus is part of the climate change
problem.
(c) Scientists thus believe that this phenomenon goes hand
(c) Thereby making them more expensive.
(d) Thus the forecast for the coming year states that
desktops will be the preferred technology choice only

w.E
in hand with global warming
(d) At first, it sounds like an ironic savior to climate change
problems
for consumers who cannot afford the exorbitantly
priced notebook.
(e) Thus notebooks will become obsolete after a decade or

48.
change is definitely here asy
(e) The answer to all our problems with respect to climate

Poverty is the state for the majority of the world’s people and
so.
50. Next to China, India is the most populated country in
the world. ...... Particularly, rush to technical and higher

En
nations. Why is this? .......... Have they been lazy, made poor
decisions, and been solely responsible for their own plight?
education has increased as the scope for arts and science
has become lesser and lesser due to lack of reforms and up
What about their governments? Have they pursued policies
that actually harm successful development? Such causes gin gradation in the course structure and materials according
to the developments of the world. Also, qualification in
higher education gives added advantage to face successfully
of poverty and inequality are no doubt real. But deeper
and more global causes of poverty are often less discussed.
Behind the increasing interconnectedness promised by eer
competition in the job market.
(a) Keeping this in mind, the government has provided
globalization are global decisions, policies, and practices.
These are typically influenced, driven, or formulated by the ing
concessions in the admission fees for the arts and
science streams in the country.
(b) Naturally there is too much rush and competition in
rich and powerful. These can be leaders of rich countries
or other global actors such as multinational corporations,
institutions, and influential people. In the face of such
every field.
.ne
(c) Despite this the rush to higher education is lesser.
enormous external influence, the governments of poor
nations and their people are often powerless. As a result,
in the global context, a few get wealthy while the majority
struggles.
with the knowledge expansion around the world.
t
(d) This population increase, though, has not kept pace

(e) In the next decade it will become the most populous.


DIRECTIONS (Qs. 51-55): Which of the phrases (a), (b), (c) and
(a) Is it enough to blame poor people for their own (d) given below each statement should be placed in the blank space
predicament? provided so as to make a meaningful and grammatically correct
(b) What is the government doing about it? sentence? If none of the sentences is appropriate, mark (e) i.e., ‘None
(c) Are the wealthy ones in the nation even aware of this? of the above’ as the answer. [IBPS PO 2013]
(d) The government has already taken measures to 51. Overlooking the fact that water scarcity intensifies during
eradicate the same. summer, …………
(e) The huge gap between the rich and the poor in the (a) the government issued guidelines to all builders to limit
nation is now narrowing their consumption to acceptable limits
49. Analysts and industry pundits forecast that notebook (b) provision for rainwater harvesting has been made to
market, which has been growing faster than the desktop aid irrigation in drought prone area
market for the past three years, is expected to overtake the (c) the water table did not improve even after receiving
desktop market by the year 2011-12. A fall in prices, large normal monsoon in the current year
deals from governments and institutions, and demand from (d) many residential areas continue to use swimming pools,
consumers and sectors such as education are expected to wasting large quantities water
help the notebook numbers. According to research agencies, (e) None of the above

Downloaded From : www.EasyEngineering.net


Downloaded From : www.EasyEngineering.net

Paragraph Completion  l 329

52. Refuting the rationale behind frequent agitations for 54. He has lost most of his life’s earning in the stock market but
formation of separate states, a recent report ………… .....................
(a) proved that such agitations result in loss of governmental (a) He still seems to be leading his life luxuriously and
property extravagantly
(b) indicated that the formation of small states does not (b) he could not save enough to repay his enormous debts
necessarily improve the economy (c) stock market is not a safe option to invest money unless
(c) suggested that only large scale agitations have been done with caution
effective in bringing out desired change in the past
(d) experts have been suggesting to avoid investments in
(d) recommended dividing large states into smaller ones
stock market because of its unpredictable nature
to improve governance
(e) None of the above (e) None of the above
53. Achieving equality for women is not only a laudable goal, 55. ……………..or else they would not keep electing him year
.............. after year.
(a) political reforms are also neglected preventing women (a) The party leader gave a strong message to the mayor
from entering legislatures and positions of power for improving his political style
(b) the problem is also deep rooted in the society and (b) Owing to numerous scandals against the mayor, he was
supported by it told to resign from the post immediately

ww
(c) their empowerment is purposefully hampered by
people with vested interests in all sections of the society
(d) it is also equally difficult to achieve and maintain for a
(c) The mayor threatened the residents against filing a
complaint against him
(d) The residents must really be impressed with the political

w.E
long term
(e) None of the above
style of their mayor
(e) None of the above

asy
En
gin
eer
ing
.ne
t

Downloaded From : www.EasyEngineering.net


Downloaded From : www.EasyEngineering.net

330  l  Paragraph Completion

Hints & Solutions


LEVEL-I blank space so option (d) which encompasses this sense of
part of passage is correct filler. Option (a) is opposite of what is
1. (a) Passage is about the cheap labour and its advantage while being said in the passage. Option (b) and (c) discusses revenues
competition from the other countries with even cheaper labour and borrowing of centre’s which is not mentioned or intended
is there. Part before the blank space discusses the same thing in the passage so these are incorrect.
like overtaking from these cheaper countries. Next part should 8. (a) Only option (a) captures the theme of passage in single
have something (as sentence starts with ‘therefore’) that make sentence. Hence it is the finishing sentence.
India vigilant about the fact and option (a) is having that part 9. (a) Blank part should take some common perception of budget
additionally it contains the information how India should build as the passage wants to indicate that the budget is not this….
on the advantage it had of cheap labor as discussed in major but it is that…… and option (a) in best captures the essence of
part of the passage hence this is best sentence to finish the this logic.
passage. 10. (d) As a study report is not a legal document it is not binding

ww
From the other options, (b) is second best and can be an option
in absence of (a). Option (c) and (d) are not logical.
2. (c) Option (a) is wrong as scrapping of the EIA is not discussed
on anyone. Sentence represents study in a lighter vein so it must
not be taken seriously but as the study is done there must be
some purpose of it and results at least is taken note of. This logic

w.E
in any part of passage. Rather improvement is discussed in
subsequent part after the blank space. Option (b) is totally out
of place as it does not go in with theme of passage. Only option 11.
brings to the option (d) which is correct part to complete the
incomplete sentence of passage.
(a) Only option (a) seems to be logical for the context. As

asy
which is brief and accurate for the blank space and connects
two parts of passage is (c). As this is a general statement and
subsequent part of passage shows how it was a problem to bail
out one and to deny other the same resources. Option (d) is out
of place.

En
can be inserted in with out affecting the sense in which passage
is flowing. Option (d) is also not right for its negative approach.
3. (c) It has been already given that all the institutes work in
12. (a) It is only logical option other options do not follow any
logic. If income is less than expenditure is less than income than
coordination so option (d) is redundant and repeating same
thing in different manner. Sentence indicates that though these gin
13.
there is definitely a loss.
(a) As the passage says that risk aggregation is new at the
institutions work in coordination these are having different
responsibilities to shoulder. But again there are two options
which are nearly saying same thing -option (b) and option
eer
beginning of passage it can be inferred that most banks are yet
to conceptualise it in their processes.
Hence option (a) is correct. Option (c) is farfetched conclusion.
(c). But in option (b) ‘different direction’ is not right. If they
are working in different direction then what is the need of ing
Option (d) is repetition of what is being stated in passage earlier.
So it can not be the answer.
coordinating. Option (a) is not correct for the context.
4. (d) Statement after the blank space contradicts what is said
14.

.ne
(a) Passage is about non-resident Indian entity and its tax
deduction. Other options are not in the context of passage as
they talk about the things which are not given or can be inferred
in the option (a). So it is not right. Membership is not being
discussed here so option (b) is also not right. Reason of
option (a) goes with option (d) also. Only option which comes
appropriate with the passage content and position of blank
15.
from the passage.
t
(a) Before the blank space, centre-state problems are discussed
and after the blank advantage of finance commission is given in
space is option (d). this regard. So option (a) in right approach has shown finance
5. (a) Option (b) is not right as Issue is related to the India not commission in connection with both the problems and it’s
the world, so role of World Bank looks irrelevant in this context. own advantage. Option (c) is out of context. Option (d) is also
Discussion in parliament is too farfetched in option (c). Option mentioned in passage in later parts.
(a) rightly indicates that Indian Government’s policy for the 16. (b) Option (b) is right as it has connection with what is said in
matter discussed in passage would be instructive. Option (d) is the passage after the blank space. Mercy in this option connects
out of context for its deviation from the topic. well with not a trade, not a business etc.
6. (c) Market is being discussed here not the countries. Other
17. (b) Last part of passage is about shortcomings given in
options are inappropriately distant from the context. mentioned through the statement of option (b). New approach
7. (d) Passage is a comparison of Centre and State Government’s in option (d) can fit but leaves the passage stranded and two
plans, finances and expenditures. In this particular portion of parts before the blank and after the blank can not be joined
passage which have the blank part, it is mentioned that State through this option.
finances are increasing at rapid rate and now they have more 18. (c) The submarine is newly developed and introduced in
liberty for spending on expenditures meanwhile a comparison ceremony is now going for trails. It is like coming out from
is also done with centre’s expenditures in part following the maternity ward to nursery.

Downloaded From : www.EasyEngineering.net


Downloaded From : www.EasyEngineering.net

Paragraph Completion  l 331

19. (d) As this car is specially designed for the women it can be and China are referred to in whole passage. So they can be the
said that car is designed by all women team. This is best option other two. Option (a) is hypothetical in nature and needs some
which emphasizes the point of passage. explanation to follow after it. Option (b) also needs some base.
20. (c) First finance commission laid down some of the principles This is the case with option(c) and option (e).
which were followed by subsequent commissions is a positive 30. (a) Last sentence shows that China has solved border dispute
statement which is the tone of passage. Passage presents the with a number of countries, while the passage is mainly about
finance commission in positive light. Option (d) is doubtful the relationship of India and China. This indicates that finishing
and can not be reached from the passage. statement should also be about improvement in relationships
21. (a) In this part of passage a comparison of women with men of India and China. This is given in all options. But option
in public or private sector is given with balance tilting to side (a) poses a valid question as passage shows that relationships
of women. When preference is given to women in a particular between two countries are improving.
sector then promotion to senior position is also imminent. From the other options (d) is not correct as positive
Hence this option is best one to choose. improvements are illustrated in passage. Markets of option (e)
22. (a) Passage portion is not about the management of finances but are not discussed here in passage. Option (b) is opposite to the
the improvement in finances and restructuring of expenditures. spirit of the passage.
Mending the ways of government is a too strong statement and

ww
does not apply here.
23. (c) Only option (c) is logical and sensible as rich can not be
LEVEL-II

w.E
centre point of any economy. They do not have the remote
control either.
24. (b) Certain words in other options make them doubtful -
1. (b) Here the doctor is talking about a reason that is seemingly
foolish and is intended to protect the patient from suffering
further pain. Further sentences indicate that the doctor has
an affectionate feeling towards the patient. Thus option (b) is

asy
‘most’ in option (a); ‘Risk level’ in option (c). Comparison with
other parts of world is not intended here as given in option (d).
25. (a) All options can fit. But the best one is (a) because
the best fit. None of the other options covers these two points
together. So they do not form the answer. Hence, option (b) is

En
It emphasises the point said initially in the passage with word
‘stubborn’.
the correct option.
2. (c) The author talks about the triviality of planet ‘earth’ in
It relates with German issue coherently.
It has detail definition of fallacy which is a requirement when
one thing is being discussed again after a long gap in passage.
ginperspective of universe. So following this thought and applying
this idea on human beings, options (c) and (d) form candidate
answers. But if one notices the sentence following the blank,
26. (a) Next sentence after the blank space suggests that blank
should have something simple about the inflation. This simple eer
option (c) is seen to be most appropriate. Options (a) and (b)
are out of context. Hence, option (c) is the correct option.
thing should be logical and sensible. Option (a) is simple and
goes well with the flow of passage. Option (d) is not right as ing
3. (a) Option (a) best fits in as the missing link. Option (b) is
incorrect as the sentence following the missing link is ‘Today
cure can not lie in cause itself.
27. (d) Option (b) is doubtful as passage does not mention such a
thing in any of its part. Option (a) talks about all parties but the .ne
in addition, they hire investment bankers....’. The words ‘in
addition’ show that earlier also, some efforts have been made
by labour unions. This sense of effort is not conveyed by option
next statement after the blank space is contradicting it. Option
(c) is negative and for fiscal federalism passage discusses its
plight not its shortcoming or of democracy as given in option
t
(b). Option (c) is incorrect as the words ‘just filed’ does not go
with ‘Today in addition’ in the following sentence. Option (d) is
irrelevant in the given context. Hence, option (a) is the correct
option.
(b). Only option that can fill the blank for going in with the
content of passage is option (d). This sentence and next sentence 4. (b) Option (b) is the most logical and clear option as the missing
both are a combination and coherent in flow. link, fitting well from the language point of view and following
28. (a) Other options cannot be inserted in the blank space as the ‘pre’ and ‘post’ statements. Preceding statements mention
they are not logical and in sync with passage. Option (b) can be about ‘Slavery in America and the vested interests of Southern
right if ‘only’ is removed from it. Use of ‘only’ makes it incorrect. countries ….. American President Abraham Lincoln handing
Banks and Stock markets are decentralization agents and hence the vision with courage’. Subsequent to the missing link, is the
are needed as per passage in this way option(c) is against statement, ‘Lincoln died but with his death ….. ensuring peace
the passage. Then option (d) unnecessarily will introduce and prosperity in the world’. Thus analysing the text, option
comparison between banks and stock markets while passage (b) most fittingly expresses and parenthetically gives a logical
presents both in as a pair. sense to the passage as it states that ‘Lincoln emancipated the
29. (d) This question is to be solved by reading whole passage slaves ….. ‘a messiah’ of the American slaves’. Options (a), (c)
as no immediate connections can be derived with sentence and (d) cannot be included as the correct option as they are not
preceding the blank space and one following it. If Japan is third establishing the link between ‘Lincoln’s assassination and slaves’
then which one are two other - it is answered in option (d). USA freedom’.

Downloaded From : www.EasyEngineering.net


Downloaded From : www.EasyEngineering.net

332  l  Paragraph Completion

5. (b) Option (b) is the most befitting statement logically as the Option (c) also does not reflect effectively the context suitable
missing link following the statement : ‘In this time of advanced for the missing link, hence not considered. Option (d) is also
….. audiences’, ‘puppetry ….. manipulators’. ‘It is now an ruled being inadequate in representation.
assimilated ….. here and now’ and preceding the statements 10. (c) Option (c) is the most logical statement as the missing link
‘caricature puppets ….. political satires’, ‘India now into one in the passage as ‘pre’ and ‘post’ parenthetically it fits in snugly.
performance’. The option expresses that things of importance The previous statement says ‘What on earth …. Lala Amarnath
like poems, educational messages, wildlife themes, etc. are today?’ While the post parenthesis contents mention about ‘But
choreographed with provocative music resulting in spectacular be glad he played in wilder ….. temper to match’., ‘Amarnath
arts. None of the other options fit suitably as they are either was no less ….. first Indian test century’., ‘He was an expansive
incomplete in their representation or they are wayward. Option ….. merely the scratching of statics’. Analysis of the content
(b) is the most logical option as the missing link. of option (c) reflects ‘either trampled on his spirit on equally
6. (c) Option (c) is the most befitting statement logically as the bad, lumped him with a ‘brand image’ ….. cheesy smile’ which
missing link following the statement ‘gone are the days ….. appears appropriate to fit in as the missing link. Option (a) is
business potential’. ‘There was time ….. migrant workers’. not considered as it portrays an incomplete representation and
‘Somewhere ….. things changed’., ‘not surprisingly ….. for hence not considered.
many companies and countries.’ The option expresses things 11. (d) Option (d) is the most logical statement as the missing link

ww
of importance like Canada being the only G7 country with a
surplus, has fastest growing GDP and has some exciting tech
clusters besides many more. Options (a), (b) and (d) are not
in the passage as ‘pre’ and ‘post’ parenthetically it fits in simply.
The pretext statements state : ‘The tea plantation ….. country’.
‘Next to ….. sections of the society, while the post parenthesis

w.E
perfect fits as they are either incomplete in their representation
or wayward. Option (c) is the most logical fit for the missing
link.
statements mention about ‘As the tea plantation ….. distances’.,
‘In north India ..... plain districts’, etc. An analysis of .... the
contents of option (d) reflects ‘About half the labour face …..

asy
7. (d) Option (d) is the most befitting statement logically as the
missing link following the statements : ‘an overstatement to say
that the consumer products industry is on an eternal search for
they enjoy equal wages,’ which contextually fits as the missing
link. Option (a) and option (c) cannot be considered as the

En
new ways to drive top-line growth’, ‘there are a plethora …..
limited access to customers ….. are just a few’ and preceding 12.
appropriate fit as they are inadequate in their representation
while option (b) is not considered as its contents are wayward.
(c) 13. (a) 14. (b) 15. (b) 16. (a)
the statements : ‘many legacy systems’, ‘the result of such an
amalgam ….. unresponsive consumers’. The option expresses gin
17.
19.
(d) 18. (b)
(a) The theme of the paragraph revolves around ‘bad writing’.
quite necessary things i.e. involvement of mature markets,
globalisation, challenges of working together, etc. Each of
option (a), (b) and (c) are taking up the issue some way or eer
The author aims to define ‘bad writing’. Nothing in the text
suggests that the author aims to compare good writing with bad
writing. Options (b) and (d) talk about ‘good writing’ which is
the other but none of them is as logical for the missing link as
option (d) is. ing
a shift from the theme and hence can be eliminated. Option (a)
explains the ‘self-serving’ goals mentions in the last sentence of
8. (a) Option (a) is the most logical statement as the missing link
in the passage because it aptly links to the ‘pre’ and ‘post’ textual
content. The statements : ‘The current boom ….. employment’,
20.
.ne
the paragraph and hence seems the best fit.
(b) Option (b) is the answer as it refers to both the policing and

‘The decade-long down sizing in the manufacturing ….. an


era of jobless growth’, etc. reflect the pre-parenthesis reflects
‘According to one ….. in 2002-03’, ‘In contrast, private
recruitment companies ….. placements to go up by 25% in
21.

22.
higher education policies.

presented in the last line of the paragraph. t


(a) Option (a) is clearly correct as it continues with the contrast

(b) As per the passage in starting portion it has been given that
market oriented people do not have plans for poor people or in
2004’. Option (b) does not fit as the missing link because its other words no social agenda which is definitely the priority of
logical representation is incorrect. Option (c) also cannot be any government. But if power of government is transferred to
considered as it gives a misrepresentation which misleads the others (market forces in this case) then there would not be any
passage. Option (d) is inadequately covered and hence not radical social change.
considered as the appropriate missing link. From the other options, (c) discusses elections but there is
9. (b) Option (b) is the most logical statement as the missing nothing related to elections in the passage.
link in the passage because it aptly fits in to the ‘pre’ and ‘post’ Option (d) is contradicting what is said in the statement
text of the passage. The statements : ‘Rajasthan is building ….. following the blank space.
into hotels’ followed as ‘post-parenthesis’ the statements which
23. Option (b) vehemently represents the sense of passage;
read as ‘Express highways ….. heritage’, ‘That monuments …..
additionally it contains in it a message that author wants to
backdrop of mountains’., etc. Option (b) as the missing link also
convey through the passage. One more point supports this
reflects about ‘governments odd realisation that heritage has to
option is that it fits well with the capitalized word (which is
be protected ….. around heritage’ truly justifies the context
of course used to emphasize) WRONG. Hence strong message
to the missing link. Option (a) does not correspond to the
should follow it.
contextual theme and hence not considered as the missing link.

Downloaded From : www.EasyEngineering.net


Downloaded From : www.EasyEngineering.net

Paragraph Completion  l 333

24. (d) This option is right as it does not judge or present an opinion climate or earning based choice or fashion. Rather it is just the
rather it comes up with a possibility with right approach i.e. difference on general basis.
positive and covers legal aspect of matter as well. Other options 33. (a) This question is more a checking of logic and sense than
are more or less judgments or opinions that do not qualify as an understanding of the passage. Only export and import do
finishing /closing statement. not make GDP of a country. They are not the only indicators of
25. (d) All other options are rephrasing of all ready discussed influence of a country on world map. Option(c) is not logical
facts of passage. But option (d) shows it. It can be inferred that and no sense prevails through it.
holding back information would lead to lack of innovations 34. (a) Option (b) is not correct as it goes against the passage.
in auto industry, these innovations can be in supply chain Option (e) is illogical and does not fit as there is no reason why
for dealers, new car models with improved performance at skilled people are not available. Option (d) seems silly on basis
chap pricing for customers/end users and increased profit for of its impossibility. Experts are not discussed in passage. Only
company. option which fits well in the context is director working alone
26. (a) Understand it in this way - on such films of low budget.
Although savings is discouraged but borrowings are also not 35. (b) See the part of passage after the blank space, this gives a
encouraged for a longer (persistent) period. number of meaning of term ‘yield’ taken in different fields. This
This question is a difficult one. Best method to solve these type is same thing which is stated in the passage. Option (d) is links

ww
of questions is to use elimination method.
Option(c) is out of context talking about credit cards so can be
rejected.
the term with situations which is not correct. Option (a) and (c)
are false.
36. (a) This is best option which expresses the fact illustrated in

w.E
Option (d) involves banks and gifts for saving account which
seems to be giving triviality to the matter of paragraph.
Option (a) is best answer for the blank space as it supports only
passage that the market is ever changing. Other options are
generalized statements with no special linkage to passage.
37. (a) Let us examine the validity of each and every option one by
what is given in passage.
asy
27. (d) Passage shall start with a hope in the GDP figures as in next
one.
Only Indian is not right overseas acquisitions are also discussed
in passage.

En
sentence that hope fades away. See the use of word ‘even after’
this indicates that incident detailed here should have happened
Acquisitions are not hostile always.
This can be verified with the subsequent content of the passage.
before it. Another thing that goes in favor of option (d) is that it
shows the pain of unemployment and its sustenance which is a
major issue highlighted in paragraph. gin Globalization is not given in passage.
This option is totally out of context.
38. (a) In option (b) ‘limit’ is doubtful.
28. (d) ‘also’ of this statement connects well with the preceding
sentence. Passage presents Delhi in two different shades. And eer
Option (c) and Option (d) are incorrect and cannot be defined
from the passage.
this statement complements the negative statement just before
the blank space. Same example is given earlier in passage - on
one hand, social amenities deteriorating while on other hand
ing
Option (a) is only option which in combination with next
statement is sensible.
39. (a) Implementation can be a challenge for such a wide
world known social workers are also working in Delhi. In same
sense while there are people with addiction for wrong things, .ne
banking system of India but it cannot be impossible or problem
(supervisors are there to implement). It is foolish to implement
there are also people with addiction to right things.
29. (b) Passage can be divided in two parts. One before the blank
space and another after the blank space. First part says that
bank people are nervous as computer systems are not working.
on trail basis on a vast scale.
t
40. (a) A little reasoning is required to solve the question. If a
foreign bank is operating in a country then it shall be governed
by the regulations of that country along with its original county.
Second part says that queues get thinner. It means problem Then only that country will allow it to operate. On this logic
mentioned in first part gets rectified and effect of it is seen in option (d) can be rejected. Little extension of this logic discards
second part. the possibility of audit in country of origin. Hence rule out (c).
30. (a) This option connects well with the opening statement. Option (b) is inappropriate as passage does not support it.
Second option can not be reached through passage. 41. (a) Last sentence shows that China has solved border dispute
Third option also needs some mentioning in the passage. with a number of countries, while the passage is mainly about
In absence of this linkage it does not qualify for the correct the relationship of India and China. This indicates that finishing
answer. Option (d) is irrelevant. statement should also be about improvement in relationships
31. (a) ‘these two Asian giants’ hints reference of some Asian of India and China. This is given in all options. But option
country in previous line. On this basis Option (b) is wrong. (a) poses a valid question as passage shows that relationships
Option(c) is about only the China. Fight is between Option (d) between two countries are improving.
and Option (a); latter scores over former as word ‘behemoth’ From the other options (d) is not correct as positive
complements ‘giants’ of subsequent sentence. improvements are illustrated in passage. Markets of option (e)
32. (c) After the blank space two or three sentences are about the are not discussed here in passage. Option (b) is opposite to the
dressing styles of North and South Indians. It is not about the spirit of the passage.

Downloaded From : www.EasyEngineering.net


Downloaded From : www.EasyEngineering.net

334  l  Paragraph Completion

42. (b) The paragraph clearly says that although explicitly the effects of these should be discussed. Option (b) can be rejected
spontaneous human combustion theory has neither been as for solving the problems treaty was ratified. (C) is a
accepted nor rejected, but most of the time other causes for judgmental statement and should be avoided when no certain
burning have been discovered, such as smoking, inability to information is available. (D) can fit well before the ratification
flee or criminal activity. Option (b) exactly goes with this idea, of the treaty but not after it. There should be a continuation
hence forms the answer. Options (a) and (d) are totally irrelevant in passage. Option (E) is incorrect for the logic that first basic
and have to be rejected. Option (c) is also to be ruled out as it rights will be achieved and then only empowering can be
gives points like, ‘Every novel theory is attacked initially’ that is mentioned. This option is one step further and not about the
not mentioned in the passage. Hence, option (b) is the correct women’s right.
option. 47. (c) Option (B) & (D) can be rejected as climate change
43. (c) Option (c) best encapsulates the essence of the text. Option is mentioned nowhere in the passage. After the blank global
(a) is incorrect as it contradicts the given passage. Option (b) warming is mentioned and before it global dimming. So blank
is incorrect as it is irrelevant in the given context. Option (d) is can work as a connector for both of these. Option (C) includes
incorrect as the passage does not mention about the sharpness both. Hence this is the best option. (E) is like deciding in haste
of mind being a temporary feature. Hence option (c) is the without understanding the intent of passage.
correct option.
48. (a) Before blank space a question is posed. And after the

ww
44. (b) Option (b) best encapsulates the essence of the text. Option
(a) is incorrect as the passage nowhere mentions about the
sense of hearing as a reflexive sense. Option (c) is incorrect as
blank space reasons which can be associated with poor people
themselves is discussed so the blank space should have a
special mention of poor people whether in form of question or

w.E
it is irrelevant in the given context. Option (d) is incorrect as it
contradicts the passage, which states that the process of hearing
and seeing together improve one’s memory. Hence option (b) is
a simple statement (B) & (D) won’t fit well here so they can be
ruled out.

the correct option.

asy
45. (b) Option (b) is the best option representing the theme of
the passage which enlists about ‘Researchers having identified
Rich is not even introduced in passage till this point, therefore
option (E) can also be rejected. Additionally Option (A) gives
a logical and sensible flow to the passage.
a number of benefits ….. which we control the events on our
En
own’. The passage further says that ‘people unable to control
49. (a) Only option which is positive for notebooks and is in
accordance with their prices coming down is (A). Option (B)
important events may suffer ….. and may hand over the control
to others in whom they can place their trust’. Option (a) does ginis against what is being said in the last line of the passage. (C)
is completely illogical and hard to come by as it talks about
the prices of components coming down but prices note books
not represent the theme of the passage as it is incomplete in
its summing up. Option (c) handles the first aspect but then
trails off, leaving the theme wanting. Option (d) also does not eer
soring high. (E) is also in different direction and is not related
to spirit of passage. Option (D) is about desktops not note
represent the entire theme, hence not considered.
46. (a) For attempting this type of questions it is important to
books.
ing
50. (b) Sentence before the blank space is about the population
correlate what is being stated before the blank space and what
is stated after the blank.
While it is also important to see in which direction the passage
.ne
and after it about the rush in for college education Option (B)
is related to both the statements (before and after the blank)
(E) is repetition of what is said in opening statement. (A) is
will go if a particular choice is selected.
In this question, women’s rights treaty being ratified by a
number of countries is discussed so in the following sentence
t
like sudden intrusion of government’s angle in passage with no
relation to statements preceding and following the blank space.
52. (d) 53. (b) 54. (e) 55. (a)

Downloaded From : www.EasyEngineering.net


Downloaded From : www.EasyEngineering.net

18
PTER
CHA

Reading
Comprehension

INTRODUCTION

ww
As the name implies, this is all about reading and comprehending textual passages. Textual passages of varying lengths are normally
presented to the test-taker and questions about the passage are put up at the end. The length and difficulty level of the passage depends
on the examination. For example, exams that test the reading speed have longer passages than the exams that test the comprehension

w.E
or understanding.
Reading Comprehension (RC) tests can focus on the following skills:
(a) Intellectual Capacity: It is the ability of a person to gauge the meaning of the sentences, the paragraphs and the passage as a
whole. More often than not, this is the skill that is most tested in examinations.

asy
(b) Reading Speed: The speed with which one reads a given text is considered to be the reading speed and is measured in Words
Per Minute (WPM). The higher this is for someone, the better it is, but not at the expense of comprehension.
(c) Vocabulary: It was once said that “The extent of a man’s vocabulary is the range of his understanding.” Although one is not

En
expected to be a know-all and be the living counterpart of a dictionary, still it helps to have an extensive vocabulary. There are
certain examinations that have provided a list of words to master before even trying to take these tests.
TERMINOLOGY
(i)
gin
WPM (Words Per Minute): This is how the reading speed is measured for a person. There are studies to show what is a good

eer
WPM count for a specific age and type of reading. But reading speed is dependent on the use for the text. This means that if
the text is to be used for comprehension, it might require about 200-400 WPM reading speed, whereas if the text is about a

(ii)
to 800 WPM.
ing
dispersed set of information and routine writing, suited for skimming, it might require a higher WPM count of the order: 400

Skimming: Read superficially or examine hastily; this style of reading is best used with long winded passages that contain
repetitive information or redundant paragraphs that can be analyzed with a hurried eye.
.ne
Jargons: Specialized technical terminology characteristic of a particular subject, or a characteristic language of a particular
(iii)
group. Jargons increase the difficulty level of a passage, especially if the context of the passage is new for the reader, e.g. a
medical passage containing medical terminology would be difficult to read, much less understand, for an accountant.
COMMON MISTAKES t
(i) Skimming: Normally, skimming is an activity that we do not indulge in a lot when we live our normal lives. We study books,
newspapers, pamphlets, advertisements, website articles, etc. All of this is active reading and it is required that we understand
the meaning of what is written. But at work, there is a lot of paper work that requires fast reading: white papers, research
articles, long memorandum, company issuances, etc. These are routinely written and hence have a lot of information that is
not absolutely necessary. It is mentioned only to cover all aspects of the item in question. To read a lot of these and still take
time out to work is essential and therefore skimming becomes an important weapon in the arsenal of a business executive.
Skimming might be important examinations, lately, the focus has been on comprehension. Most often, students tend to
skim an article and after reading the questions realize that they cannot answer because they have not really understood
the important parts of the passage. We would warn students against skimming. The articles that appear on the exams are
moderately sized and the questions are detailed. It is best to not read fast, but to take time and comprehend.
(ii) Time Allocation: Mostly, students fall in the trap of not allocating a time limit for the reading comprehension questions.
Maybe the passage is just too much to their liking and they forget that they actually have to answer questions after reading
it, or maybe the passage is created for skimming and the students are aiming for comprehension. There are many other
examples in which students do not manage time, but the RC questions are the most repeated ones. Remember, in other test is
a time bound exercise. You cannot waste an inordinate amount of time on one type of question, otherwise you wouldn’t have
sufficient time to solve other types of questions. In order that one does not commit this mistake, it’s best that a mental time
limit be maintained for the group of questions.

Downloaded From : www.EasyEngineering.net


Downloaded From : www.EasyEngineering.net

336  l  Reading Comprehension

(iii) Faulty Comprehension: Sometimes, we think that we understand what is written only to be shown later that we don’t. On a
test, this could mean lost marks. This can happen because of a lot of reasons like loss of focus, unfamiliarity with the subject
matter, or simply lack of sleep. One technique that is normally used to overcome this problem is “Thinking like the writer”.
When you think like the writer, you try to understand why she/he wrote that particular line, or paragraph or passage, what
was the intention behind using that particular adjective/adverb, why was this tone of voice utilized, etc. Having that kind of a
mindset could be the best thing you carry to a test center.

TYPES OF READING COMPREHENSION QUESTIONS


The major types of reading comprehension questions include:
(i) Inferential Questions: These are some of the most frequently asked questions. Here the examiners ask you to make inferences
from what you have just read in the passage. These could be tough, if you haven’t properly understood the text.
(ii) Tone of the author: These questions test you on your ability of picking up the author’s style, whether he/she is approving/
disapproving, supportive/unsupportive of the idea in question. The options can confuse you, so it’s best to do two things: First,
step in the author’s shoes, and try and understand why she/he is writing like this, thereby you can understand her/his reasons.
Second, look up the following words in the dictionary and hope that the questions that come in the exam use only these:
Acclamatory Disdainful Optimistic

ww
Acerbic
Adulatory
Aggressive
Disparaging
Dogmatic
Emotional
Partisan
Patronising
Peremptory

w.E
Angry
Apathetic
Encouraging
Evocative
Pessimistic
Positive
Apologetic
Bellicose
Gloomy
Harsh asy Prejudiced
Provocative
Belligerent
Biased
Humanistic
Humorous
En
Ridiculous
Romantic
Biting
Buoyant
Hypocritical
Incendiary
Sanguine
Sarcastic gin
Chauvinistic
Cheerful
Inciting
Indifferent
Sardonic
Satirical eer
Colored
Commiserating
Indignant
Inspiring
Scathing
Scornful ing
Complimentary
Condescending
Introspective
Laudatory
Searing
Speculative
.ne
Contemplative
Contemptuous
Critical
Mocking
Motivating
Narrative
Supercilious
Sympathetic
Technical
t
Cutting Negative Trenchant
Derisive Neutral Vicious
Descriptive Obsequious Vitriolic
Didactic Opinionated Vituperative
These are all different adjectives used to describe the tones of the author of the passage. These tones can be divided into
positive, negative and neutral types. We leave it up to you to do some hard work and classify them accordingly.
(iii) Title of the passage: The questions that ask you to identify a proper title for the passage are somewhat tricky as they might
refer to the overarching theme or they might take a zingy turn and be a twist on the author’s style, thereby making the real title
look different from the passage. To take a gander at this kind of a question, it is best that the reader let go of presumptions, if
any, related to the text. The writing, if followed judiciously, would definitely lead you to an appropriate title. Take it easy, and
do not negate any option just because it seems too small or too long winded to be of your average title size.
(iv) Summary of the passage: When you are faced with this question on a test, remember that a summary is a quick recap
of the major points in the passage. This means that it would contain all the important ideas and not have any redundant/
unimportant ones. Also, the summary cannot be simply pointing to one part of the passage, instead it should cover the whole
article.

Downloaded From : www.EasyEngineering.net


Downloaded From : www.EasyEngineering.net

Reading Comprehension  l 337

(v) Theme of the passage: This is the underlying idea of the passage. In any article/passage, there is a running theme that is
prevalent throughout the passage. If the reader has understood the author’s point of view, she/he should have no trouble
identifying the theme. An important mistake to avoid here is that like the summary, the theme also cannot be referring to
only part of the passage.
(vi) Finding out the importance of a line or word in the passage: On almost all tests, there are questions that ask you to identify
some or the other lines/words that may have a bearing on the passage. Here, you must first find that instance where that line/
word has been used in the passage, and then gauge its meaning from the reading. This type of questions is relatively easier,
because if someone has really read the passage well, he/she would know where the line/word has been used, and what its
meaning is.

HOW TO SOLVE
In this section, where we highlight the RC solving strategies, we want to make clear that we do not endorse any one strategy and the
student is free to change them as he/she deems fit. Mostly, every student creates his/her own style. Choosing a strategy is essential
because a habit helps in saving time on the test. Even then, it is up to the student, how he/she wants to take this up: The Pencil: One
of the most successful strategies of understanding reading comprehension is the one where you use the pencil to underline/highlight/
circle the important pieces of text. A lot of us would have got this advice from our guardians/teachers about using the pencil while

ww
reading. It helps in highlighting the important parts as well as in mentally reiterating the text. The Read Once: In this style of solving
the passages, the student reads the passage only once and then moves on to reading and consequently solving the questions.
The passage is not read more than once. This is done to keep oneself attentive. When you form a habit that you are not going to

w.E
read anything again, then you are forced to be extremely tenacious at understanding and remembering the passage. In fact, this kind
of a habit can even help you gain a ‘tencaious memory’. A tenacious memory is one that is good at remembering. Thus, this kind of
a disciplined working strategy could be beneficial for your life too. Questions Before Passage: In this strategy, the test taker reads the

asy
questions first before reading the passage. This way, he/she knows the questions, can keep them in mind while perusing the passage
and hence be more focused at reading and remembering only the important stuff. This is a great way to solve passages, but then again

En
is dependent on the type of the passage. If the passage is lengthy and the questions are more of a referential type (find the meaning of
the word/line, etc.) then this is an ideal way. Otherwise, if the passage is more of an abstruse type and/or the questions are inferential,
then using this style could backfire because then you would not be able to understand the whole passage in its entirety and probably

gin
be unable to answer even one question correctly. This strategy can be modified to form another popular one, which is the Questions
Between Passage strategy. As you can possibly figure out from the name, in this style of solving RC questions, the test taker reads the

eer
passage and tries to answer the questions in a linear fashion, i.e. as and when the answer occurs in the passage. This is a very risky
strategy as more often than not, the questions on a passage do not occur linearly. There are a lot of other question solving styles. We

ing
leave it to the students to discuss among themselves and pick up the better parts of each one.

SKILLS REQUIRED IN RC
Most important skills required in RC include:
(a) Focus .ne
To understand the importance of focus while reading, take a look at what Samuel Taylor Coleridge had to say:
Readers may be divided into four classes:
(1) Sponges, who absorb all that they read and return it in nearly the same state, only a little dirtied.
t
(2) Sand-glasses-who retain nothing and are content to get through a book for the sake of getting through the time.
(3) Strain-bags-who retain merely the dregs of what they read.
(4) Mogul diamonds-equally rare and valuable, who profit by what they read, and enable others to profit by it also.
It is always beneficial to be someone who understands the written word. Also, it is better to read one passage and solve all its
questions correctly, than reading all passages and marking the incorrect answers.
(b) Sound thought process
“You will get little or nothing from the printed page if you bring it nothing but your eye.” — Walter B. Pitkin
When someone mentions reading, it is a given that understanding the text would go hand in hand. But for some lost souls, this is
not true. It is only by a force of will that some people can understand what they are reading. They can go on reading for a long time
without grasping anything. For people like these, the problem lies in the fact that either they have no focus, or they cannot process in
their minds what is being fed to them. If you want to make the most out of your aptitude tests, you would better do this: solve a lot
of logical reasoning questions, or read/analyze a lot of logical thoughts. Here is one way to do this: Sit with a small quotation and try
and understand it in depth. Good quotations from renowned authors/speakers are pithy and very poignant. If you can start making
sense of these quotations, then you might start understanding a lot of logical constructs.

Downloaded From : www.EasyEngineering.net


Downloaded From : www.EasyEngineering.net

338  l  Reading Comprehension

(c) Reading Speed


A great reading speed would be about 700-800 WPM. Even higher speeds are possible, and everyone is welcome to challenge himself/
herself. But for the sake of the aptitude tests, even a speed of around 400-500 WPM for the tough passages would be brilliant. Whether
to go with speed or with precision, the choice will always remain yours. But this must be said, neither is to be sacrificed completely for
the sake of the other. The aptitude tests are a measure of ability of all types and hence you must look a balanced personality. Woody
Allen had something great to say about all the speed reading courses: “I took a speed reading course and read "War and Peace" in
twenty minutes. It involves Russia.”
(d) Vocabulary Mastery
“A man with a scant vocabulary will almost certainly be a weak thinker. The richer and more copious one's vocabulary and the
greater one's awareness of fine distinctions and subtle nuances of meaning, the more fertile and precise is likely to be one's thinking.
Knowledge of things and knowledge of the words for them grow together. If you do not know the words, you can hardly know the
thing.” — Henry Hazlitt
These words by Hazlitt say everything that we need to say about the importance of a balanced vocabulary for solving reading
comprehension questions. A good vocabulary is your shining torch that helps you to wade your way through the difficult passages.
(e) Diverse Reading

ww
All the passages on a test are normally from different sources/genres. If you have a familiarity with them, it would always benefit you.
Whenever one starts reading a passage, there is a lag time that is required to get to the original speed. This lag is because of the change
between the two question types and also because of the unfamiliarity with the text. This time is required for the mind to acclimatize

w.E
to the new reading. Reduce this time and you might gain a few minutes on the test.

asy
En
gin
eer
ing
.ne
t

Downloaded From : www.EasyEngineering.net


Downloaded From : www.EasyEngineering.net

Reading Comprehension  l 339

Practice Exercise
LEVEL-I
DIRECTIONS (Qs. 1-230): Read the following passages and 2. For realising the goal of universal education, the
answer the questions that follow. education system in the country must be made identical
to that of developed countries.
Passage-1 Which of the statements given above is/are correct ?
Now India’s children have a right-to receive at least eight years (a) 1 only (b) 2 only
of education, the gnawing question is whether it will remain on (c) Both 1 and 2 (d) Neither 1 nor 2
paper or become a reality. One hardly needs a reminder that this 4. Which one of the following statements conveys the key
right is different from the others enshrined in the Constitution, message of the passage ?
that the beneficiary – a six year old child cannot demand it, (a) India has declared that education is compulsory for its
nor can she or he fight a legal battle when the right is denied children.

ww
or violated. In all cases, it is the adult society which must act on
behalf of the child. In another peculiarity, where a child’s right
to education is denied, no compensation offered later can be
(b) Adult society is not keen on implementing the Right to
Education.
(c) The Right to Education, particularly of a girl child, needs

w.E
adequate or relevant. This is so because childhood does not last. If
a legal battle fought on behalf of a child is eventually won, it may
be of little use to the boy or girl because the opportunity missed
to be safeguarded.
(d) The system of education should address the issue of
Right to Education.

asy
at school during childhood cannot serve the same purpose later
in life. This may be painfully true for girls because our society
permits them only a short childhood, if at all. The Right to
5. Which one of the following statements conveys the inference
of the passage ?
(a) The society has a tenacious prejudice against the
Education (RTE) has become law at a point in India’s history
En
when the ghastly practice of female infanticide has resurfaced in
intellectual potential of girls.
(b) Adults cannot be relied upon to fight on behalf of
the form of foeticide. This is “symptomatic of a deeper turmoil”
in society which is compounding the traditional obstacles to the gin children for their Right to Education.
(c) The legal fight to get education for children is often
education of girls. Tenacious prejudice against the intellectual
potential of girls runs across our cultural diversity and the system
of education has not been able to address it. eer
protracted and prohibitive.
(d) There is no sufficient substitute for education received in
childhood.
1. With reference to the passage, consider the following
statements : ing Passage-2
For achieving inclusive growth there is a critical need to rethink
1. When children are denied education, adult society does
not act on behalf of them. .ne
the role of the State. The early debate among economists about
the size of the Government can be misleading. The need of
2. Right to Education as a law cannot be enforced in the
country.
Which of the statements given above is/are correct?
(a) 1 only (b) 2 only
t
the hour is to have an enabling Government. India is too large
and complex a nation for the State to be able to deliver all that
is needed. Asking the Government to produce all the essential
goods, create all the necessary jobs, and keep a curb on the prices
(c) Both 1 and 2 (d) Neither 1 nor 2 of all goods is to lead to a large cumbersome bureaucracy and
2. According to the passage, what could be the traditional widespread corruption.
obstacles to the education of girls ? The aim must be to stay with the objective of inclusive growth
1. Inability of parents to fight a legal battle when the Right that was laid down by the founding fathers of the nation and also
to Education is denied to their children. to take a more modern view of what the State can realistically
2. The traditional way of thinking about girls’ role in deliver.
society. This is what leads to the idea of an enabling State, that is, a
3. The prejudice against the intellectual potential of girls. Government that does not try to directly deliver to the citizens
4. Improper system of education. everything that they need. Instead, it (1) creates an enabling
Select the correct answer from the codes given below : ethos for the market so that individual enterprise can flourish
(a) 1 and 2 only (b) 2, 3 and 4 only and citizens can, for the most part, provide for the needs of one
(c) 1, 3 and 4 only (d) 1, 2, 3 and 4 another, and (2) steps in to help those who do not manage to do
3. On the basis of the passage, consider the following statements : well for themselves, for there will always be individuals, no matter
1. Right to Education is a legal right and not a fundamental what the system, who need support and help. Hence we need a
right. Government that, when it comes to the market, sets effective,

Downloaded From : www.EasyEngineering.net


Downloaded From : www.EasyEngineering.net

340  l  Reading Comprehension

incentive-compatible rules and remains on the sidelines with oppressed social groups get politically mobilised and demand
minimal interference, and at the same time, plays an important their rights. The upsurge of the peasants and tribals, the
role in directly helping the poor by ensuring that they get basic movements for regional autonomy and self-determination, the
education and health services and receive adequate nutrition and environmental movements, and the women’s movements in the
food. developing countries are signs of emergence of creative society
6. According to the passage : in contemporary times. The forms of social movements and
1. The objective of inclusive growth was laid down by the their intensity may vary from country to country and place to
founding fathers of the nation. place within a country. But the very presence of movements for
2. Need of the hour is to have an enabling Government. social transformation in various spheres of a society indicates the
3. The Government should engage in maximum emergence of a creative society in a country.
interference in market processes. 11. What does the author imply by “creative society” ?
4. There is a need to change the size of the Government. 1. A society where diverse art forms and literary writings
Which of the statements given above are correct ? seek incentive.
(a) 1 and 2 only (b) 2 and 3 only 2. A society where social inequalities are accepted as the
(c) 1 and 4 only (d) 1, 2, 3 and 4 norm.
7. According to the passage, the strategy of inclusive growth 3. A society where a large number of contradictions are

ww
can be effected by focusing on
(a) Meeting all the needs of every citizen in the country.
(b) Increasing the regulations over the manufacturing
recognised.
4. A society where the exploited and the oppressed groups

w.E
sector.
(c) Controlling the distribution of manufactured goods.
(d) Delivery of the basic services to the deprived sections of
grow conscious of their human rights and upliftment.
Select the correct answer using the codes given below :
(a) 1, 2 and 3 (b) 4 only
the society.
8. What constitutes an enabling Government? asy (c) 3 and 4 (d) 2 and 4
12. What according to the passage are the manifestations of
social movements ?
1. A large bureaucracy.
2. Implementation of welfare programmes through
representatives. En 1. Aggressiveness and being incendiary.
2. Instigation by external forces.
3. Creating an ethos that helps individual enterprise.
4. Providing resources to those who are underprivileged. gin 3. Quest for social equality and individual freedom.
4. Urge for granting privileges and self-respect to
5. Offering direct help to the poor regarding basic services.
Select the correct answer from the codes given below :
eer
disparaged sections of the society.
Select the correct answer using the codes given below :
(a) 1 and 3 only (b) 2 and 4 only
(a) 1, 2 and 3 only
(c) 3, 4 and 5 only
(b) 4 and 5 only
(d) 1, 2, 3, 4 and 5
9. Why is the State unable to deliver “all that is needed”? ing
(c) 3 and 4 only (d) 1, 2, 3 and 4
13. With reference to the passage, consider the following
1. It does not have sufficient bureaucracy.
2. It does not promote inclusive growth.
statements :
.ne
1. To be a creative society, it is essential to have a variety of
Select the correct answer from the codes given below :
(a) 1 only
(c) Both 1 and 2
(b) 2 only
(d) Neither 1 nor 2
10. What is the essential message being conveyed by the author
social movements.

contradictions and conflicts.


Which of the statements given above is/are correct ?
t
2. To be a creative society, it is imperative to have potential

of the passage? (a) 1 only (b) 2 only


(a) The objectives of inclusive growth laid down by the (c) Both 1 and 2 (d) Neither 1 nor 2
founding fathers of the nation should be remembered. 14. With reference to the passage, consider the following
(b) The Government needs to make available more schools statements:
and health services. 1. The need of the hour is to have an enabling government.
(c) The Government needs to establish markets and 2. The need of the hour is to have a changing government.
industries to meet the needs of the poor strata of the Which of the statements given above is/are correct ?
society. (a) 1 only (b) 2 only
(d) There is a need to rethink the role of the State in achieving (c) Both 1 and 2 (d) Neither 1 nor 2
inclusive growth.
Passage - 4
Passage- 3 A country under foreign domination seeks escape from the
The concept of ‘creative society’ refers to a phase of development present in dreams of a vanished age, and finds consolation in
of a society in which a large number of potential contradictions visions of past greatness. That is a foolish and dangerous pastime
become articulate and active. This is most evident when in which many of us indulge. An equally questionable practice for

Downloaded From : www.EasyEngineering.net


Downloaded From : www.EasyEngineering.net

Reading Comprehension  l 341

us in India is to imagine that we are still spiritually great though also an example of a keystone species. Its preferred prey is the
we have come down in the world in other respects. Spiritual or mussel Mytilus californianus. In the absence of sea stars, these
any other greatness cannot be founded on lack of freedom and mussels crowd out other competitors in a broad belt of the
opportunity, or on starvation and misery. Many western writers intertidal zone. By consuming mussels, sea star creates bare
have encouraged that notion that Indians are other-worldly. I spaces that are taken over by a variety of other species.
suppose the poor and unfortunate in every country become to A study at the University of Washington demonstrated the
some extent other-worldly, unless they become revolutionaries, influence of Pisaster on species richness by removing sea stars
for this world is evidently not meant for them. So also subject from selected parts of the intertidal zone repeatedly over a
peoples. period of five years. Two major changes occurred in the areas
As a man grows to maturity he is not entirely engrossed in, or from which sea stars were removed. First, the lower edge of
satisfied with, the external objective world. He seeks also some the mussel bed extended farther down into the intertidal zone,
inner meaning, some psychological and physical satisfaction.
showing that sea stars are able to eliminate mussels completely
So also with peoples and civilizations as they mature and grow
where they are covered with water most of the time. Second, and
adult. Every civilization and every people exhibit these parallel
streams of an external life and an internal life. Where they meet more dramatically, 28 species of animals and algae disappeared
or keep close to each other, there is an equilibrium and stability. from the sea star removal zone. Eventually only Mytilus, the

ww
When they diverge conflict arises and the crises that torture the
mind and spirit.
15. The passage mentions that “this world is evidently not meant
dominant competitor, occupied the entire substratum. Through
its effect on competitive relationships, predation by Pisaster
largely determines which species live in these rocky intertidal

w.E
for them”. It refers to people who
1. seek freedom from foreign domination.
2. live in starvation and misery.
ecosystems.
19. What is the crux of the passage ?
(a) Sea star has a preferred prey.
3. become revolutionaries.

(a) 1 and 2 asy


Which of the statements given above is/are correct ?
(b) 2 only
(b) A preferred prey determines the survival of a keystone
species.
(c) Keystone species ensures species diversity.
(c) 2 and 3 (d) 3 only
16. Consider the following assumptions :
En (d) Sea star is the only keystone species on the Pacific coast
of North America.
1. A country under foreign domination cannot indulge in
spiritual pursuit.
2. Poverty is an impediment in the spiritual pursuit.

gin
20. With reference to the passage, consider the following
statements :
3. Subject peoples may become other worldly.
With reference to the passage, which of the above
eer
1. Mussels are generally the dominant species in intertidal
ecosystems.
2. The survival of sea stars is generally determined by the
assumptions is/are valid ?
(a) 1 and 2
(c) 2 and 3
(b) 2 only
(d) 3 only ing
abundance of mussels.
Which of the statements given above is/are correct ?
17. The passage thematically centres on
(a) the state of mind of oppressed people
(b) starvation and misery
(a) 1 only
(c) Both 1 and 2
.ne
(b) 2 only
(d) Neither 1 nor 2
21. Which of the following is/are implied by the passage?
(c) the growth of civilization
(d) body, mind and spirit of people in general
18. According to the passage, the torture of the mind and spirit
is caused
their evolution.
t
1. Mussels are always hard competitors for sea stars.
2. Sea stars of the Pacific coast have reached the climax of

3. Sea stars constitute an important component in the


(a) by the impact of foreign domination. energy flow in intertidal ecosystem.
(b) by the desire to escape from foreign domination and Which of the statements given above is/are correct?
find consolation in visions of past greatness. (a) 1 and 2 (b) 2 only
(c) due to lack of equilibrium between an external life and (c) 1 and 3 (d) 3 only
an internal life. 22. Consider the following assumptions:
(d) due to one’s inability to be either revolutionary or other 1. The food chains/food web in an ecosystem are influenced
worldly. by keystone species.
2. The presence of keystone species is a specific
Passage- 5 characteristic of aquatic ecosystems.
A species that exerts an influence out of proportion to its 3. If the keystone species is completely removed from an
abundance in an ecosystem is called a keystone species. The ecosystem, it will lead to the collapse of the ecosystem.
keystone species may influence both the species richness of With reference to the passage, which of the above
communities and the flow of energy and materials through assumptions is/are valid ?
ecosystems. The sea star Pisaster ochraceus, which lives in rocky (a) 1 only (b) 2 and 3 only
intertidal ecosystems on the Pacific coast of North America, is (c) 1 and 3 only (d) 1, 2 and 3

Downloaded From : www.EasyEngineering.net


Downloaded From : www.EasyEngineering.net

342  l  Reading Comprehension

Passage- 6 Passage- 7
Ecosystems provide people with a variety of goods and services; A moral act must be our own act; must spring from our own will.
food, clean water, clean air, flood control, soil stabilization, If we act mechanically, there is no moral content in our act. Such
pollination, climate regulation, spiritual fulfilment and aesthetic action would be moral, if we think it proper to act like a machine
enjoyment, to name just a few. Most of these benefits either are and do so. For, in doing so, we use our discrimination. We should
irreplaceable or the technology necessary to replace them is bear in mind the distinction between acting mechanically and
acting intentionally. It may be moral of a king to pardon a culprit.
prohibitively expensive. For example, potable fresh water can be
But the messenger carrying out the order of pardon plays only
provided by desalinating sea-water, but only at great cost.
a mechanical part in the king’s moral act. But if the messenger
The rapidly expanding human population has greatly modified were to carry out the king’s order considering it to be his duty,
the Earth’s ecosystems to meet their increased requirements of his action would be a moral one. How can a man understand
some of the goods and services, particularly food, fresh water, morality who does not use his own intelligence and power of
timber, fibre and fuel. These modifications have contributed thought, but lets himself be swept along like a log of wood by
substantially to human well being and economic development. a current ? Sometimes a man defies convention and acts on his
The benefits have not been equally distributed. Some people have own with a view to absolute good.
actually been harmed by these changes. Moreover, short-term 26. Which of the following statements best describe/describes

ww
increases in some ecosystem goods and services have come at the
cost of the long-term degradation of others. For example, efforts
to increase the production of food and fibre have decreased
the thought of the writer ?
1. A moral act calls for using our discretion.
2. Man should react to a situation immediately


w.E
the ability of some ecosystems to provide clean water, regulate
flooding and support biodiversity.
23. With reference to the passage, consider the following
3. Man must do his duty.
4. Man should be able to defy convention in order to be
moral.
statements.
asy
Expanding human population has an adverse effect on :
Select the correct answer from the codes given below :
(a) 1 only
(c) 2 and 3
(b) 1 and 3
(d) 1 and 4
1. Spiritual fulfilment
2. Aesthetic enjoyment
En 27. Which of the following statements is the nearest definition
of moral action, according to the writer ?
3. Potable fresh water
4. Production of food and fibre
5. Biodiversity gin (a) It is a mechanical action based on official orders from
superiors.

Which of the statements given above are correct ?


(a) 1, 2 and 3 only (b) 2, 4 and 5 only eer
(b) It is an action based on our sense of discretion.
(c) It is a clever action based on the clarity of purpose.
(d) It is a religious action based on understanding.
(c) 3 and 5 only (d) 1, 2, 3, 4 and 5
24. The passage mentions that “some people have actually been
harmed by these changes”. What does it imply ?
ing
28. The passage contains a statement “lets himself be swept
along like a log of wood by a current.” Among the following

1. The rapid expansion of population has adversely affected


some people: .ne
statements, which is/are nearest in meaning to this ?
1. A person does not use his own reason.
2. He is susceptible to influence/pressure.
2. Sufficient efforts have not been made to increase the
production of food and fibre.
3. In the short term some people may be harmed, but in
the long term everyone will benefit from modifications
3. He cannot withstand difficulties/ challenges.
4. He is like a log of wood. t
Select the correct answer using the codes given below :
(a) 1 only (b) 1 and 2
in the Earth’s ecosystems. (c) 2 and 3 (d) 1 and 4
Which of the statements given above is/are correct?
(a) 1 only Passage- 8
(b) 2 Education, without a doubt, has an important functional,
(c) 1 and 3 instrumental and utilitarian dimension. This is revealed when
(d) None of the statements given above one asks questions such as ‘what is the purpose of education?’.
25. With reference to the passage, consider the following The answers, too often, are ‘to acquire qualifications for
employment/upward mobility’, ‘wider/higher (in terms of
statements:
income) opportunities’, and ‘to meet the needs for trained
1. It is imperative to modify the Earth’s ecosystems for the
human power in diverse fields for national development’. But in
well being of mankind.
its deepest sense education is not instrumentalist. That is to say,
2. Technology can never replace all the goods and services
it is not to be justified outside of itself because it leads to the
provided by ecosystems.
acquisition of formal skills or of certain desired psychological –
Which of the statements given above is/are correct?
social attributes. It must be respected in itself. Education is thus
(a) 1 only (b) 2 only
not a commodity to be acquired or possessed and then used, but
(c) Both 1 and 2 (d) Neither 1 nor 2

Downloaded From : www.EasyEngineering.net


Downloaded From : www.EasyEngineering.net

Reading Comprehension  l 343

a process of inestimable importance to individuals and society, and engineering majors-would begin by saying that they could
although it can and does have enormous use value. Education not possibly know anything about such a quantity. Of course,
then, is a process of expansion and conversion, not in the sense of some solutions would be to simply do a count of every piano tuner
converting or turning students into doctors or engineers, but the perhaps by looking up advertisements, checking with a licensing
widening and turning out of the mind—the creation, sustenance agency of some sort, and so on. But Fermi was trying to teach his
and development of self-critical awareness and independence of students how to solve problems where the ability to confirm the
thought. It is an inner process of moral-intellectual development. results would not be so easy. He wanted them to figure out that
29. What do you understand by the ‘instrumentalist’ view of they knew something about the quantity in question.
education? 32. Suppose you apply the same logic as Fermi applied to
(a) Education is functional and utilitarian in its purposes. confetti, which of the following statements would be the
(b) Education is meant to fulfil human needs. most appropriate?
(c) The purpose of education is to train the human intellect. (a) You can calculate the minimum pressure inside the
(d) Education is meant to achieve moral development. cooker by calculating the maximum distance travelled
30. According to the passage, education must be respected in by any of its parts after it explodes.
itself because (b) You can calculate the average potency of a fire cracker
(a) it helps to acquire qualifications for employment by calculating the distance covered by one of its bigger
(b) it helps in upward mobility and acquiring social status

ww
(c) it is an inner process of moral and intellectual
development
fragments.
(c) You can easily find out the average policy of an
earthquake by measuring the length of a crack it makes

w.E
(d) All the (a), (b) and (c) given above are correct in this
context.
31. Education is a process in which
(a) students are converted into trained professionals.
on the surface of the earth.
(d) You can calculate the exact volume of water stored in a
tank by measuring the distance covered by the stream of

(c) asy
(b) opportunities for higher income are generated.
individuals develop self-critical awareness and

water coming out of the tap fixed on the lower corner of
the tank.
(e) All the above conclusions can be drawn.
independence of thought.
(d) qualifications for upward mobility are acquired.
En 33 . Quick estimate, as per Fermi, is most useful in:
(a) In finding an approximate that is more useful than

Passage- 9 gin existing values.


(b) In finding out the exact minimum value of an estimate.
(c) In finding out the exact maximum value of an estimate.
An example of scientist who could measure without instruments
is Enrico Fermi (1901-1954), a physicist who won the Nobel Prize
eer
(d) In finding out the range of values of an estimate.
(e) In finding out the average value of an estimate.
in physics in 1938. He had a Well-developed knack for intuitive,
even casual-sounding measurements. One renowned example of
his measurement skills was demonstrated at the first detonation ing
34 . Given below are some statements that attempt to capture the
central idea of the passage:
1. It is useful to estimate; even when the exact answer is
of the atom bomb, the Trinity Test site, on July 16, 1945, where
he was one of the atomic scientists observing from base camp. known.
.ne
2. It is possible to estimate any physical quantity.
While final adjustments were being made to instruments used to
measure the yield of the blast, Fermi was making confetti out of
a page of notebook paper. As the wind from the initial blast wave
began to blow through the camp, he slowly dribbled the confetti
launched car that can be sold in a city
4. Fermi was a genius.
t
3. It is possible to estimate the number of units of a newly

into the air, observing how far back it was scattered by the blast
(taking the farthest scattered pieces as being the peak of the Which of the following statements (s) best captures the
pressure wave), Fermi concluded that the yield must be greater central idea?
than 10 kilotons. This would have been news, since other initial (a) 1, 2 and 4 (b) 2, 3 and 4
observers of the blast did not know that lower limit. Alter much (c) 2 and 3 (d) 2 only
analysis of the instrument readings, the final yield estimate was (e) 1, 2 and 3
determined to be 18.6 kilotons. Like Eratosthenes7 Fermi was 35 . Read the statements given below:
aware of a rule relating one simple observation-the scattering of 1. Atomic bomb detonation was a result of Fermi’s Nobel
confetti in the Wind-to a quantity he wanted to measure. Prize contribution
The value of quick estimates was something Fermi was familiar 2. Fermi’s students respected him as a scientist
with throughout his career. He was famous for teaching his 3. Yield of atomic bomb can only be measured in Kilotons
students skills at approximation of fanciful-sounding quantities Which of the following statement(s) can be inferred from
that, at first glance, they might presume they knew nothing the passage?
about. The best-known example of such a “Fermi question” was (a) 1,2 (b) 2, 3
Fermi asking his students to estimate the number of piano tuners (c) 1, 3 (d) 2 only
in Chicago, when no one knows the answer. His students-science (e) None of the three statements is correct

Downloaded From : www.EasyEngineering.net


Downloaded From : www.EasyEngineering.net

344  l  Reading Comprehension

Passage- 10 The paying of attention to one's audience is called “rhetoric,”


a word that I later exercise hard. One uses rhetoric, of course,
Unseasonableness is a tendency to do socially permissible things to warn of a fire in a theatre or to arouse the xenophobia of the
at the wrong time. The unseasonable man is the sort of person electorate. This sort of yelling is the vulgar meaning of the word,
who comes to confide in you when you are busy. He serenades his like the president's “heated rhetoric” in a press conference or the
beloved when she is ill. He asks a man who has just lost money by “mere rhetoric” to which our enemies stoop. Since the Greek
paying a bill for a friend to pay a bill for him. He invites a friend flame was lit, though, the word has been used also in a broader
to go for a ride just after the friend has finished a long car trip. He and more amiable sense, to mean the study of all the ways of
is eager to offer services which are not wanted but which cannot accomplishing things with language inciting a mob to lynch the
be politely refused. If he is present at an arbitration, he stirs up accused, to be sure, but also persuading readers of a novel that
dissension between the two parties, who were really anxious to its characters breathe, or bringing scholars to accept the better
agree. Such is the unseasonable man. argument and reject the worse.
36 . He tends to The question is whether the scholar-who usually fancies
(a) entertain women. himself an announcer of “results” or a stater of “conclusions” free
(b) be a successful arbitrator when dissenting parties are of rhetoric -speaks rhetorically. Does he try to persuade? It would
anxious to agree. seem so. Language, I just said, is not a solitary accomplishment.
(c) be helpful when solicited. The scholar doesn’t speak into the void, or to himself. He speaks

ww
(d) tell a long story to people who have heard it many times
before.
37. The unseasonable man tends to
to a community of voices. He desires to be heeded, praised,
published, imitated, honored, en-nobeled. These are the desires.

w.E
(a) bring a higher bidder to a salesman who has just closed
a deal.
(b) disclose confidential information to others.
The devices of language are the means.
Rhetoric is the proportioning of means to desires in speech.
Rhetoric is an economics of language, the study of how scarce
means are allocated to the insatiable desires of people to be heard.
(d) sleep late and rise early.
asy
(c) sing the praises of the bride when he goes to a wedding. It seems on the face of it, a reasonable hypothesis that economists
are like other people in being talkers, who desire listeners that
Passage- 11
If translated into English, most of the ways economists talk En they go to the library or the laboratory as much as when they
go to the office on the polls. The purpose here is to see if this is
true, and to see if it is useful to study the rhetoric of economic
among themselves would sound plausible enough to poets,
journalists, businesspeople, and other thoughtful though gin
scholarship.
The subject is scholarship. It is not the economy, or the
noneconomical folk. Like serious talk anywhere-among boat
designers and baseball fans, say -the talk is hard to follow when
one has not made a habit of listening to it for a while. The culture eer
adequacy of economic theory as a description of the economy,
or even mainly the economist’s role in the economy. The subject
is the conversation economists have among themselves, for
of the conversation makes the words arcane. But the people in the
unfamiliar conversation are not Martians. Underneath it all (the ing
purposes of persuading each other that the interest elasticity
of demand for investment is zero or that the money supply is
economist's favorite phrase) conversational habits are similar.
Economics uses mathematical models and statistical tests and
market arguments, all of which look alien to the literary eye.
controlled by the Federal Reserve.

.ne
Unfortunately, though, the conclusions are of more than
academic interest. The conversations of classicists or of
But looked at closely they are not so alien. They may be seen as
figures of speech-metaphors, analogies, and appeals to authority.
Figures of speech are not mere frills. They think for us.
Someone who thinks of a market as an “invisible hand” and
t
astronomers rarely affect the lives of other people. Those of
economists do so on a large scale. A well known joke describes
a May Day parade through Red Square with the usual mass of
soldiers, guided missiles, rocket launchers. At last come rank
the organization of work as a “production function” and his upon rank of people in gray business suits. A bystander asks,
coefficients as being “significant,” as an economist does, is giving “Who are those?” “Aha!” comes the reply, “those are economists:
the language a lot of responsibility. It seems a good idea to look
you have no idea what damage they can do!” Their conversations,
hard at his language.
do it.
If the economic conversation were found to depend a lot on its
38. According to the passage, which of the following is the
verbal forms, this would not mean that economics would be not
best set of reasons for which one needs to "look hard" at an
a science, or just a matter of opinion, or some sort of confidence
game. Good poets, though not scientists, are serious thinkers economist's language?
about symbols; good historians, though not scientists, are serious (a) Economists accomplish a great deal through their
thinkers about data. Good scientists also use language. What is language.
more (though it remains to be shown) they use the cunning of (b) Economics is an opinion-based subject.
language, without particularly meaning to. The language used (c) Economics has a great impact on other's lives.
is a social object, and using language is a social act. It requires (d) Economics damaging.
cunning (or, if you prefer, consideration), attention to the other (a) a and b (b) c and d
minds present when one speaks. (c) a and c (d) b and d

Downloaded From : www.EasyEngineering.net


Downloaded From : www.EasyEngineering.net

Reading Comprehension  l 345

39. In the light of the definition of rhetoric given in the passage, man what he seems to the astronomer, a tiny lump of carbon and
which of the following will have the least element of rhetoric? water impotently crawling on a small and unimportant planet?
(a) An election speech. Or is he what he appears to Hamlet? Is he perhaps both at once?
(b) An advertisement jingle. Is there a way of living that is noble and another that is base, or
(c) Dialogues of a play. are all ways of living merely futile? If there is a way of living that is
(d) Commands given by army officers. noble, in what does it consist, and how shall we achieve it? Must
40. As used in the passage, which of the following is the closest the good be eternal in order to deserve to be valued, or is it worth
meaning to the statement "The culture of the conversation seeking even if the universe is inexorably moving towards death?
makes the words arcane"? Is there such a thing as wisdom, or is what seems such merely the
(a) Economists belong to a different culture. ultimate refinement of folly? To such questions no answer can be
(b) Only mathematicians can understand economists. found in the laboratory.
(c) Economists tend to use terms unfamiliar to the lay Theologies have professed to give answers, all to definite;
person, but depend on familiar linguistic forms. but their definiteness causes “modern minds to view them with
(d) Economists use similes and adjectives in their analysis. suspicion.
41. As used in the passage, which of the following is the closest The studying of these questions, if not the answering of them,
alternative to the word "arcane"? is the business of philosophy. Why, then, you may ask, waste time

ww
(a) Mysterious
(c) Covert
(b) Secret
(d) Perfidious
42. Based on your understanding of the passage, which of the
on such insoluble problems? To this one may answer as a historian,
or as an individual facing the terror of cosmic loneliness.
The answer of the historian, in so far as I am capable of

w.E
following conclusions would you agree with?
(a) The geocentric and the heliocentric views of the solar
system are equally tenable.
giving it, will appear in the course of this work. Ever since men
became capable of free speculation, their actions in innumerable
important respects, have depended upon their theories as to the
(b) The
rhetoric.
(c) Both views use rhetoric to persuade. asy
heliocentric view is superior because of better world and human life, as to what is good and what is evil. This is
as true in the present day as at any former time. To understand

(d) Scientists should not use rhetoric.

Passage- 12 En an age or a nation, we must understand its philosophy, and to


understand its philosophy we must ourselves be in some degree
philosophers.

The conceptions of life and the world which we call ‘philosophical’ ginThere is here a reciprocal causation: the circumstances of men’s
lives do much to determine their philosophy, but, conversely,
are a product of two factors: one, inherited religious and ethical
conceptions; the other, the sort of investigation which may be
called ‘scientific’, using this word in its broadest sense. Individual eer
their philosophy does much to determine their circumstances.
There is also, however, a more personal answer. Science tells
us what we can know, but what we can know is little, and if we
philosophers have differed widely in regard to the proportions in
which these two factors entered into their systems, but it is the
presence of both, in some degree, that characterizes philosophy.
ing
forget how much we cannot know we may become insensitive
to many things of very great importance. Theology, on the other
Philosophy' is a word which has been used in many ways, some
wider, some narrower. I propose to use it in a very wide sense, .ne
band, induces a dogmatic belief that we have knowledge, where
in fact we have ignorance, and by doing so generates a kind of
which I will now try to explain.
Philosophy, as I shall understand the word, is something
intermediate between theology and science. Like theology,
it consists of speculations on matters as to which definite
t
impertinent insolence towards the universe. Uncertainty, in the
presence of vivid hopes and fears, is painful, but must be endured
if we wish to live without the support of comforting fairy tales.
It is not good either to forget the questions that philosophy asks,
knowledge has, so far, been unascertainable; but like science, or to persuade ourselves that we have found indubitable answers
it appeals to human reason rather than to authority, whether to them. To teach how to live without certainty, and yet without
that of tradition or that of revelation. All definite knowledge- being paralyzed by hesitation, is perhaps the chief thing that
so I should contend-belongs to science; all dogma as to what philosophy, in our age, can still do for those who study it.
surpasses definite knowledge belongs to theology. But between 43. The purpose of philosophy is to
theology and science there is a ‘No man's Land’, exposed to attack (a) reduce uncertainty and chaos.
from both sides; this ‘No Man's Land’ is philosophy. Almost all (b) help us to cope with uncertainty and ambiguity.
the questions of most interest to speculative minds are such as (c) help us to find explanations for uncertainty.
science cannot answer, and the confident answers of theologians (d) reduce the terror of cosmic loneliness.
no longer seem so convincing as they did in former centuries. Is 44. Based on this passage what can be concluded about the
the world divided into mind and matter, and if so, what is mind relation between philosophy and science?
and what is matter? Is mind subject to matter, or is it possessed of (a) The two are antagonistic.
independent powers? Has the universe any unity or purpose? Is it (b) The two are complementary.
evolving towards some goal? Are there really laws of nature, or do (c) There is no relation between the two
we believe in them only because of our innate love of power? Is (d) Philosophy derives from science.

Downloaded From : www.EasyEngineering.net


Downloaded From : www.EasyEngineering.net

346  l  Reading Comprehension

45. From reading the passage, what can be concluded about the 48. The difference between “the good life” and “a good life” is:
profession of the author? He is most likely not to be a (a) The life that one would like to live and the life which is of
(a) historian (b) philosopher a high standard.
(c) scientist (d) theologian (b) The life of the rich and the life which is for the rich.
46. According to the author, which of the following statements (c) A life of opportunity and a life of opulence.
about the nature of the universe must be definitely true? (d) A life which is fulfilling and a life which is enriching.
(a) The universe has unity. (e) The life of the aristocracy and the life of the plebeians.
(b) The universe has a purpose. 49. In the passage, Hermann Hesse symbolizes:
(c) The universe is evolving towards a goal. (a) The classical philosopher.
(d) None of the above. (b) The happy teacher.
(c) The quintessential drifter.
Passage- 13 (d) The new age philosopher.
WHEN I WAS 21, I was trying to decide whether to become a (e) The atypical philosopher.
doctor or a philosophy professor. My older brother, whose advice
I usually followed, asked me why I wanted to study philosophy. I Passage- 14
was evasive. Finally I admitted that a lot of the books I loved had A TED talk (the acronym stands for Technology, Entertainment,
been written by philosophers and philosophy professors. Plus,

ww
one of my favorite books at the time, a book I’d read and reread
since I was a teenager, was Hermann Hesse’s Magister Ludi: The
Glass Bead Game, which unabashedly romanticized the life of
and Design) is one of the routes to academic stardom that didn’t
exist a decade ago. (The 30th-anniversary celebration aside,
curators only began posting fame-making free online videos in

w.E
the professor.
“Be practical. Books are dangerous things,” my brother warned
me. “Just because it’s on paper, you think it’s true. Moneylove was
2006.) Although TED plays an inordinate role in setting the tone
for how ideas are conveyed—not only because of the reach of
its videos but also through spinoffs like regional "TEDx" events
and the TED Radio Hour, one of the few places nonpolicy

asy
one of the most damaging books I ever read. Not to mention
How to Win Friends & Influence People.” (I should probably
mention that my brother is a very successful luxury jeweler,
intellectuals get substantial on-air time—it’s just one of a
number of platforms that are changing the ecology of academic

who continues to love money and, as Dale Carnegie instructs,


En
to “make the other person feel important—and do it sincerely.”)
celebrity. These include similar ideas-in-nuggets conclaves, such
as the Aspen Ideas Festival and PopTech, along with huge online
courses and—yes, still—blogs. These new, or at least newish,
This wasn’t what I wanted to hear, so I called my dad, at that time a
broke New Age guru and sex therapist living in Jupiter, Florida— gin
forms are upending traditional hierarchies of academic visibility
and helping to change which ideas gain purchase in the public
not exactly the oracle of Delphi, and not someone whose advice
I usually followed. “Every doctor I know is miserable, son,” he
told me. “They work all the time and complain about insurance
discourse.

eer
In a famous essay, "The Unbearable Ugliness of Volvos," first
published in the early 90s, the literary scholar Stanley Fish wrote
companies.” (Not much has changed since 1988.) “Be a professor.
You’ll never be rich, but you’ll be doing what you love: reading ing
that "the flourishing of the lecture circuit has brought with it new
sources of extra income ... [and] an ever-growing list of stages
and writing. You get summers off. It’s a good life.”
Note that my father didn’t say the good life, which is how a
philosophically minded adviser might have put it to me—except .ne
on which to showcase one’s talents, and geometric increase in
the availability of the commodities for which academics yearn,
attention, applause, fame, and ultimately, adulation of a kind
that philosophy in America in the 1980s and ’90s seemed to be
losing its way in dry, scholastic debates about the most lifeless
of topics (what is the meaning of and?). But he told me what I
wanted to hear, and a quarter century later, philosophy is making
t
usually reserved for the icons of popular culture." Fish was
Exhibit A among professors taking advantage of such trends,
and his trailblazing as a lit-crit celebrity inspired the dapper,
globe-trotting lit-theory operator Morris Zapp, a character in
David Lodge’s academic satire Small World. But the world Fish
the kind of comeback that leaves a Hermann Hesse groupie glad
was describing, where no one could live-tweet the lectures, let
to have headed for graduate school and ended up with tenure.
alone post the talks for worldwide distribution, now seems sepia-
Amid hand-wringing about the decline of the humanities, the
toned.
philosopher (and novelist) Rebecca Newberger Goldstein can
"If David Lodge’s Morris Zapp were alive and kicking today,"
write a book like Plato at the Googleplex: Why Philosophy
observes John Holbo, an associate professor of philosophy at
Won’t Go Away, confident that she’ll find readers eager to turn to
the National University of Singapore, and blogger at Crooked
philosophers for help in thinking about the meaning of life and
Timber and the Valve, "he’d be giving a TED talk, not an MLA
how best to live it.
talk. Which is to say: He wouldn’t be doing Theory. He probably
47. The writer of the passage is a philosopher because:
wouldn’t be in an English department."
(a) He loved writing.
50. The passage is mainly about:
(b) He liked philosophy.
(a) Technology, Entertainment, and Design
(c) His father advised him.
(b) Turning over the conventional.
(d) He couldn’t get into medical school.
(c) Gaining popular adulation.
(e) His brother guided him.
(d) Changing presentations.

Downloaded From : www.EasyEngineering.net


Downloaded From : www.EasyEngineering.net

Reading Comprehension  l 347

(e) Worldwide metamorphosis. evokes surprise, emotion, and passion in others, but remains
51. The phrase “sepia-toned” implies: entirely insensible himself, producing an effect to which he alone
(a) The end of an era. remains immune. D’Aurevilly's celebration of the dandy at times
(b) The way things were. borders on idolatry: for d’Aurevilly, dandies are “those miniature
(c) The brown pigment. Gods, who always try to create surprise by remaining impassive.”
(d) The time bound nature of things. 53. The word “dandyism” includes all except which one of the
(e) The decadence of ideas. following?
52. Which of the following cannot be inferred from the passage? (a) Victuals (b) Walking sticks
(a) TED is the future. (c) Nicknames (d) Clothes
(b) Theory can no longer be counted on. (d) Passion
(c) Philosophy is best understood through demos. 54. As far as the modern day ‘selfies’ go, the author is mainly
(d) TED is irreplaceable. concerned with:
(e) Teachers are interested in experimenting with new (a) The obsession with one’s self.
techniques. (b) The inclusion in the dictionary.
(c) The fixation with the novelty.
Passage- 15 (d) The narcissistic overtones.

ww
When selfie was crowned the Word of 2013 by the Oxford
Dictionaries, the media reaction ranged from apocalyptic to
cautiously optimistic. For the Calgary Herald’s Andrew Cohen,
(e) The entire thought process.
55. The usage of the word “apocalyptic” in the passage can be
best substituted by which of the following?

w.E
“selfie culture” represents the “critical mass” of selfish entitlement;
for Navneet Alang in the Globe and Mail, selfies are inextricable
from the need for self-expression, a “reminder of what it means
(a) Revelatory
(c) Indifferent
(e) Inauspicious
(b) Prophetic
(d) Critical

asy
to be human.” For the Guardian’s Jonathan Freedland, the selfie is
both: at once “the ultimate emblem of the age of narcissism” and
a function of the “timeless human need to connect.”
56. Which of these is the best description of the author’s tone in
the passage?
(a) Matter of fact. (b) Derisive.
With a few exceptions, commentators tended to converge on
En
one point: the selfie, and the unencumbered act of self-creation
(c) Casual.
(e) Humorous.
(d) Offensive.

it represents, is unmistakably of our time, shorthand for a whole


host of cultural tropes wedded to the era of the smartphone. As
Jennifer O’Connell, writing for the Irish Times, puts it: “It’s hard
gin Passage- 16
to think of a more appropriate—or more depressing—symbol of
the kind of society we have become. We are living in an age of eer
It was for the children's own good, he was told - but he suspected
the English preferred having other people's children in the
household because they could feed them less and work them
narcissism, an age in which only our best, most attractive, most
carefully constructed selves are presented to the world.”
But our obsession with the power of self-creation—and
harder.
ing
His remarks shine a light on a system of child-rearing that

its symbiotic relationship with the technology that makes it


possible—is hardly new. Even the “selfie artist” is hardly a .ne
operated across northern Europe in the medieval and early
modern period. Many parents of all classes sent their children
away from home to work as servants or apprentices - only a
creation of 2013. Its genesis isn’t in the iPhone, but in the painted
portrait: not among the Twitterati, but among the silk-waist
coated dandies of nineteenth-century Paris.
It may seem like a stretch to mention selfie artists like Kim
t
small minority went into the church or to university. They were
not quite so young as the Venetian author suggests, though.
According to Barbara Hanawalt at Ohio State University, the
aristocracy did occasionally dispatch their offspring at the age
Kardashian or James Franco in the same breath as, for example, of seven, but most parents waved goodbye to them at about 14.
the French writer Jules-Amédée Barbey d’Aurevilly, but today’s Model letters and diaries in medieval schoolbooks indicate
self-creators owe more to d’Aurevilly's view of the power of that leaving home was traumatic. "For all that was to me a
public image than you might think. For d’Aurevilly and his ilk— pleasure when I was a child, from three years old to 10… while
recently celebrated in coffee-table book I Am Dandy, which I was under my father and mother's keeping, be turned now
profiles “modern-day” dandies from across the globe, dandyism to torments and pain," complains one boy in a letter given to
was about more than mere sartorial elegance. It was a way of pupils to translate into Latin. Illiterate servants had no means of
consciously existing in the world. communicating with their parents, and the difficulties of travel
And d’Aurevilly existed more consciously than most. His meant that even if children were only sent 20 miles (32 km) away
clothing was as legendary as his writing. He famously kept a they could feel completely isolated.
collection of bejeweled walking sticks in his front parlor and So why did this seemingly cruel system evolve? For the poor,
informed journalists that his favorite was to be referred to as there was an obvious financial incentive to rid the household
“ma femme.” His 1844 hagiography of Beau Brummel, a dandy of a mouth to feed. But parents did believe they were helping
of another age, doubles as a manifesto: in his eyes, the true dandy their children by sending them away, and the better off would
save up to buy an apprenticeship. These typically lasted seven

Downloaded From : www.EasyEngineering.net


Downloaded From : www.EasyEngineering.net

348  l  Reading Comprehension

years, but they could go on for a decade. The longer the term, suspect that even if I did, it would be unlikely to be published."
the cheaper it was - a sign that the Venetian visitor was correct to But like many such requests, it began to disappear beneath a
conclude that adolescents were a useful source of cheap labour pile of other emails. It was only several weeks later that Sokal came
for their masters. In 1350, the Black Death had reduced Europe's across it again and realised that on this occasion he could help
population by roughly half, so hired labour was expensive. The because it was in a field he knew something about: mathematics
drop in the population, on the other hand, meant that food was and physics.
cheap - so live-in labour made sense. Losada had derived his mathematical model from a system
"There was a sense that your parents can teach you certain of differential equations known as the Lorenz equations, after
things, but you can learn other things and different things and Edward Lorenz, a pioneer of chaos theory.
more things if you get experience of being trained by someone "The Lorenz equation Losada used was from fluid dynamics,"
else," says Jeremy Goldberg from the University of York. says Sokal, "which is not the field that I'm specialised in, but it's
57. The passage conveys a sense of: elementary enough that any mathematician or physicist knows
(a) Trauma (b) Unease enough. In 10 seconds I could see it was total bullshit. Nick had
(c) Nostalgia (d) Heartache written a very long critique and basically it was absolutely right.
(e) Dismay There were some points where he didn't quite get the math right
58. According to the passage, live-in labour was preferred but essentially Nick had seen everything that was wrong with the

ww
because:
(a) It was easy to control them.
(b) It was helpful in providing the basic amenities,
Losada and Fredrickson paper."
60. What was the underlying motive behind Sokal’s actions?
(a) Sokal wanted to establish himself.

w.E
(c) Part time labour was unreliable.
(d) It saved cost of labour.
(e) It made distribution of labour easier.
(b) Sokal wanted to ridicule the theorists.
(c) Sokal wanted to propagate mathematical models.
(d) Sokal wanted to expose the established critical theorists.

processes?
(a) Teaching
asy
59. The passage is a commentary on which of the following

(b) Parenting
(e) Sokal wanted to develop his theory on quantum gravity.
61. The word ‘pastiche’ in the passage, can be best replaced by:
(a) Lampoon (b) Satire
(c) Socializing
(e) Refoming
(d) Policing
En (c) Melodrama
(e) Burlesque
(d) Style

Passage- 17 gin
62. Which of the following is the most appropriate title for the
passage?
Back in 1996, Alan Sokal wrote a paper called Transgressing
the Boundaries: Towards a Transformative Hermeneutics of
Quantum Gravity and submitted it to an academic cultural studies eer
(a) Bad science vs. good science.
(b) The old theorists are dead.
(c) The battle of wits.
journal called Social Text, which promptly published the article.
As the title suggested, the paper was dense with impenetrable ing
(d) The man who loved mathematics.
(e) The man who debunked mathematics.
theory. Among other things, it disparaged the scientific method
and western intellectual hegemony and claimed that quantum
gravity could only be understood through its political context.
Passage- 18
.ne
A few years ago a 17-year-old Australian boy was diagnosed with
The paper, as Sokal quickly admitted, was a hoax, a deliberate
pastiche of the sorts of nonsensical postmodern appropriations
of maths and physics at which French critical theorists
t
“climate change delusion” after he tried to stop drinking water
and was found compulsively checking for leaking taps. He had
learned about the climate-change-amplified droughts afflicting
his country and thought that his water consumption would
particularly excelled – among them Jacques Derrida, Jacques
Lacan, Gilles Deleuze and Julia Kristeva. A major intellectual deplete supplies and lead to millions of deaths. Anxiety about
controversy ensued in which postmodernists stood accused of the problem also stimulated a major depressive disorder. Not all
pseudo-science, absurd cultural relativism and the concealing psychological consequences of climate change and environmental
of ignorance and innumeracy behind obscurantist prose. In ruin are so immediately identifiable. We’re entering an era in
response Sokal was derided as a pedant, a literalist and a cultural which widespread knowledge of environmental catastrophe and
imperialist. even possible societal collapse is replacing the specter of nuclear
Armageddon and increasingly becoming part of everyday
Despite the counterattacks, Sokal gained a reputation as
experience. That knowledge of a degraded, jeopardized world is
a formidable enemy of bad science. As such he was regularly
playing out in our psyches in ways that we’re only beginning to
approached by people who believed they had uncovered an
come to grips with—and in ways that our mental health systems
intellectual imposture, be it in architecture, history or musicology.
are not prepared to cope with.
"I don't think I'm a crank," Brown had said in his email to Sokal.
The psychologists Thomas J. Doherty and Susan Clayton
"I am just this grad student with no qualifications or credentials, outlined the various psychological impacts of climate change
starting out in the field. I don't know how to express this kind in a paper published in American Psychologist. They identified
of idea especially coherently in academic written form, and I several categories of affliction: direct impacts of stress and loss

Downloaded From : www.EasyEngineering.net


Downloaded From : www.EasyEngineering.net

Reading Comprehension  l 349

that people experience from extreme weather events such as the period of the previous financial year. Moreover, the increase in
flooding of New Orleans or New York City; indirect impacts— the monthly indices of manufacturing production relative to the
“threats to emotional well-being based on observation of impacts corresponding month of the previous year was negative in 7 of
and concern or uncertainty about future risks”; and psychosocial the first 11 months of financial 2013-14 and nil in one.
impacts—chronic effects on people and communities from Not surprisingly, the quarterly figures indicate that
excessive heat, drought, migrations, and climate-related conflicts. manufacturing growth has been negative in the first and third
Climate change hits us on multiple fronts. Just last month quarters of 2013-14. One definition of a recession adopted by
scientists of the Intergovernmental Panel on Climate Change many analysts requires the rate of growth to be negative for two
reported that unpredictable rainfall patterns were beginning or more consecutive quarters. Others adopt a broader definition
to curtail world food supplies by reducing yields of wheat and and use the term to refer to a decline in industrial sales and
maize (corn in the United States); such reports probably will only production over many months. That does seem to be the case in
get worse over time, stimulating even more anxiety among the
India in recent times.
public.
But the real cause for worry is the medium term trend in
Because of the scale of the problems, the US mental health
industrial production. The quarter-on-quarter growth rates of the
system is unprepared to handle the burgeoning widespread
IIP, which were ruling high before 2008-09 collapsed that year,
psychological stresses of climate change, elaborates another
report, sponsored by the National Wildlife Federation. Our influenced clearly by the spill-over effects of the financial crisis

ww
mental health systems are not ready to treat chronic problems
related to climate change; nor are our systems of first-responders,
and recession in the developed countries. There are two ways in
which the global crisis could have affected Indian industry.
First, through the direct effect on exports of the global

w.E
who must work in the contexts of more frequent and increasingly
severe climate-change-intensified disasters such as Hurricane
Sandy. And the stakes are high.
63. The primary purpose of the author is to:
contraction. And second, through the reduction in credit-financed
domestic demand, because of the liquidity squeeze precipitated
by the exodus of foreign investors needing to take their capital

asy
(a) Highlight the young boy’s mental state.
(b) Find solutions to climate related events.
back to meet commitments at home. Given the overwhelming
influence of domestic demand on industrial growth in India, the
(c) Impress the myriad effects of climate change.
(d) The risks of climate change.
(e) Mental health breakdowns. En second was likely to have been more important than the first.
However, Indian industry experienced a smart recovery in
2009-10, winning the country accolades from domestic and
64. Which of the following is true according to the passage?
(a) The author believes that the public is doomed due to gin
external analysts for its ‘resilience’.
66. Which of the following is the main highlight of the passage?
lack of knowledge.
(b) The author believes that climate change is the biggest
threat in our lives.
eer
(a) The status of foreign investment.
(b) India’s organized sector.
(c) The extraordinary recession.
(c) The author believes that serious psychological distress is
looming on us. ing
(d) India’s economic resilience.
(e) The collapse of industrial production.
(d) The author believes that hurricane occurrence is going
to intensify.
following reasons?
(a) The free fall of the rupee. .ne
67. The global crisis had a critical effect due to which of the

(e) The author believes that societal collapse is going to be a


major challenge.
65. From the passage, which of the following cannot be inferred?
(a) That the phenomenon of climate change is here to stay.
(b) The growth rates of the IIP.
(c) The international financial crisis.
(d) The high domestic growth.
(e) The remittance by the foreign investors.
t
(b) That we need to work seriously on the repercussions of 68. According to the passage the drop in the industrial
climate change. manufacturing sector’s growth can be ascertained by which
(c) That mental health of all individuals is bound to get of the following facts?
affected by climate change. (a) Increase in the indices.
(d) That global warming is a serious threat for all of us. (b) Increase in the number of industries.
(e) That children under eighteen are more susceptible to the (c) Decease in the indices.
effects of climate change. (d) Decease in the number of industries.
(e) Increase in foreign investment.
Passage- 19
India’s organised industrial sector, especially its manufacturing
Passage- 20
segment, seems mired in a recession. The provisional indices of At the end of the 19th century, Mohandas Gandhi was a young
industrial production (IIP) for February 2014 recently released by lawyer living in Durban, South Africa. He left his house in Beach
the CSO suggest that over the first 11 months (April-February) of Grove every morning for an office on Mercury Lane, where he
financial year 2013-14, the overall IIP fell by 0.1 per cent and that spent much of the day helping his fellow Indian immigrants
for manufacturing by 0.7 per cent, relative to the corresponding navigate the onerous colonial bureaucracy. He kept meticulous

Downloaded From : www.EasyEngineering.net


Downloaded From : www.EasyEngineering.net

350  l  Reading Comprehension

records, including a logbook of correspondence — from an Passage- 21


English missionary and local planters, and a series of letters
exchanged with the Protector of Indian Immigrants about the The folks at IDC have confirmed the worst kept secret in the
treatment of indentured laborers. In January of 1897, and again technology industry: Not many folks are looking to buy new
a few months later, he heard from another lawyer who was, like personal computers. PC sales for 2013 will fall to 314 million
him, a Gujarati who had studied in England and then struggled units. That 10% decline below 2012 levels is the worst ever.
to establish a practice in Bombay. The contents of these letters And before anyone blames the economy, keep in mind the
are unknown. In a remarkable new biography, “Gandhi Before post-recession third quarter of 2011 was the best ever for PC
India,” Ramachandra Guha gingerly speculates about what they shipments, with 96 million. With apologies to Taylor Swift, we
might have been. Expressions of support for Gandhi’s nascent will never, ever, ever get back to that level again. None of this is
activism? Or perhaps “explorations of interest in a possible career
good news for Microsoft , HP, or Dell, of course. But IDC does
in South Africa”? Guha wisely stops there. What is not in doubt
is the name in Gandhi’s logbook — “M. A. Jinnah,” Muhammad believe that PC sales will “remain just above 300 million” through
Ali Jinnah, who would become the founder of Pakistan. “All 2017, which offers a lot of selling opportunities. Of course, those
we now know is that, a full 50 years before partition and the are predicated on the forecasts being right. And given that IDC
independence of India and Pakistan, the respective ‘Fathers’ of has been wildly optimistic about the prospects of the PC for a
those nations were in correspondence.” decade, is there any reason to believe these forecasts won’t also

ww
Guha’s description of this encounter is evidence of his strength
as a historian. He mines primary sources — in this case, records
of Gandhi’s law practice from the archives in Pietermaritzburg,
overshoot the mark? Sadly, no.
Ever since the global economy turned bad in 2008, there have
been concerns the unabated growth of spending on personal

w.E
the capital of Natal, and the logbook dug out of a filing cupboard
in the Sabarmati ashram in Ahmedabad, India — to establish
that Gandhi and Jinnah were in contact a decade earlier than
computers would be affected. “As the economic crisis continues
to evolve – constraining GDP, consumer and commercial
confidence, capital and credit availability, and creating other

asy
previously documented. And he writes vividly enough to
compete with that bête noire of all Gandhi biographies, Richard
Attenborough’s 1982 film “Gandhi.” (In the movie, they meet
challenges – demand for personal computers is expected to slow
quickly,” reported Xbit Laboratories in December of that year,

En
at a garden party in India, where a skeptical Jinnah adjusts his
monocle while the turbaned Mahatma smiles beatifically.) Guha
citing IDC’s forecasts. Still, as the accompanying chart shows,
IDC continued to believe any downturn would be temporary. As
reminds us of everything these two legendary opposites had in
common — language, education and the desperate striving of
the ambitious immigrant. “Gandhi Before India” is full of such gin
the data shows, the firm forecast shipments of PCs would rise
from the low 300 million range at the onset of the recession to
444 million by 2012. Each year after it would lower the forecast.
revelations, each one a delight for the reader.
Early on, Guha spells out how his technique differs from those
of previous biographers: He uses the records of contemporaries
eer
But even by August of 2012, it would still remain too optimistic
about PC shipments.
to complement and sometimes challenge Gandhi’s own account.
Gandhi was prolific — the first 12 volumes of his collected works ing
Not content to make the mistake once, that same prediction
last summer would call for about 77 million more PCs shipped
run to 5,000 pages — but, Guha explains, “This reliance on
Gandhi’s words can often narrow the historical landscape against
which his life and work were enacted.” .ne
in 2013 than will actually materialize. This despite the fact that
IDC keeps slashing its own long-range guesstimates. As you
can see, every year the predictions come down, with IDC now
69. The book seeks to mainly explore:
(a) The Indian struggle.
(b) The British struggle.
(c) The founding of Pakistan.
t
believing demand for PCs will more or less stabilize over the
coming 5 years. The problem with taking all that seriously is that
IDC was forecasting up to 180 million units higher just about
a year ago. It’s pretty reasonable to argue the global economy is
(d) The founding of India. in better shape than it was then, with Europe slowly stabilizing.
(e) The India that might have been. U.S. unemployment slowly falling, and Japan pointed northward
70. The passage is:
under Abenomics. So if the economy isn’t killing the PC, what
(a) A passage from a book.
is? The simple answer is that people are finding tablets perfectly
(b) A page from a memoir.
(c) An excerpt from a book review. reasonable alternatives for many tasks and if we look back at
(d) An ode to Gandhi. “peak PC” in 2011, the rise of the iPad and its competitors will
(e) A commentary on Indo-Pak relations. rightly get a lot of the credit for ending the rise of the computer.
71. According to the passage, Guha feels his technique scores 72. All except which one of the following is not a reason for the
over other because PC’s decline?
(a) He uses contemporary data for validation. (a) The tablet is not a good alternative.
(b) He challenges Gandhi’s account. (b) The economy is in recession.
(c) He discards old data. (c) The iPad is doing a lot of business.
(d) He makes use of historical data. (d) The unemployment rate is static.
(e) He has studied the pre-Gandhi era documents in detail. (e) There are no viable alternatives.

Downloaded From : www.EasyEngineering.net


Downloaded From : www.EasyEngineering.net

Reading Comprehension  l 351

73. According to the passage IDC cannot be taken seriously language, either its meaning is changed or it is meaningless.
because: 75. What is the main idea of the passage?
(a) The IDC is very subjective. (a) To differentiate between traditional and modern
(b) The IDC is not serious. idioms.
(c) The IDC is pro-PCs. (b) To trace the history of idioms.
(d) The IDC is pro- Japanese. (c) Discuss the background of various idioms.
(e) The IDC is unpredictable. (d) To provide the meanings of various idioms.
74. The phrase “pointed northward under Abenomics,” means (e) Discuss the various layers of idioms.
which of the following? 76. According to the passage, what does the term “fossilized”
(a) Abenomics is functional in north Japan. convey apropos idioms?
(b) Abenomics is a regional phenomenon. (a) It means an ancient expression.
(c) Abenomics has been marked by success. (b) It means a colloquial expression.
(d) Abenomics is good for PCs. (c) It conveys a ring of nostalgia.
(e) Abenomics is a futile effort. (d) It conveys a sense of passage of rituals.
(e) It makes an effort to ring in the past.
Passage- 22 77. An appropriate title for the passage would be:
(a) The evolution of idioms.

ww
Many idiomatic expressions, in their original use were not
figurative but had literal meaning.
For instance: ‘spill the beans,’ meaning to let out a secret
(b) Idioms as we know them.
(c) The value of idioms.

w.E
probably originates in a physical spilling of beans which are
either being eaten or measured out. The point is that the spiller
certainly does not want to lose any beans. Let the cat out of the
(d) The way we learn idioms.
(e) Fun with idioms.

Passage- 23
asy
bag : has a meaning similar to the former, but the secret revealed
in this case will likely cause some problems. A cat was sometimes
put in bags to keep it under control or to pretend that it was a
The experience of reading (and re-reading) Class is akin
to wiping goggles one didn’t know were fogged. Fussell’s

En
more saleable animal, such as a pig or a rabbit. So, to let the cat
out of the bag suggests either that the ruse is revealed or that the
methodology settles into the brain like a virus; one soon cannot
stop nanocategorizing one’s world. A quarter century later,
situation is out of control. Break a leg: meaning good luck in a
performance/presentation etc. This common idiom comes from
superstition. It was thought that there were gremlins or sprites, gin
most of Fussell’s categories live on—if with some fiscal damage.
Fussell’s topmost denizens were “out of sight” in hilltop manses
at the end of long, curving driveways. The billionaires in Michael
little fairy-like creatures, backstage in theaters who would do
exactly the opposite of whatever they were told. To say break a
leg was to ensure the sprites would not in fact do the performers
eer
Tolkin’s hilariously mordant The Return of the Player are even
farther out, prow-jousting at sea in their satellite-technology-

any damage.
In linguistics, idioms are usually presumed to be figures of ing
equipped yachts. Indeed, this novel is such a teeth-gnashingly
precise class almanac, that Tolkin should surely replace Tom
Wolfe as our modern-day high-society-anxiety chronicler (at
speech contradicting the principle of compositionality. This
principle states that the meaning of a whole should be constructed
from the meanings of the parts that make up the whole. In other
.ne
least of the West Coast variety). Tolkin is particularly hard on his
people, wealthy Los Angeles Jews, a variation on the American

words, one should be in a position to understand the whole if one


understands the meanings of each of the parts that make up the
whole. The following example is widely employed to illustrate the
point: Fred kicked the bucket.
t
upper class with their conspicuously consuming Hebraism. At a
bar mitzvah at a Reform synagogue that shares a driveway with
Milken High (named deftly not for Michael but for the brother):
Torahs dressed in embroidered covers and silver breastplates
stood on the branches of a sculpted tree behind a sheer curtain,
Understood compositionally, Fred has literally kicked an
like expensive boots in a winter window display.
actual, physical bucket. The much more likely idiomatic reading,
however, is non-compositional: Fred is understood to have died. In attendance is a “fiesta” of rich Jews: the trim skeptical men
Arriving at the idiomatic reading from the literal reading is and their two categories of wives, all of them brilliantly educated,
unlikely for most speakers. What this means is that the idiomatic some of them successful professionals themselves, others still
reading is, rather, stored as a single lexical item that is now largely drifting on the messy alibi supplied by their genuinely screwed-
independent of the literal reading. up relationship with their genuinely screwed-up mothers,
In phraseology, idioms are defined as a sub-type of phraeme, but all of them, pediatric endocrinologists, failed Tibetan
the meaning of which is not the regular sum of the meanings of
wool importers, soccer moms and private school committee
its component parts. John Saeed defines an idiom as collocated
words that became affixed to each other until metamorphosing volunteers, recognizing each other’s clan by a signal from within
into a fossilised term. This collocation of words redefines each an unfakable right for their chaotic anxieties and complaints to
component word in the word-group and becomes an idiomatic take up space around them.
expression. Idioms usually do not translate well; in some cases, This isn’t to say that Hollywood Jews’ counterparts, Upper-
when an idiom is translated directly word-for-word into another Class Gentiles, are dead. Their ethos (or at least the ethos of those

Downloaded From : www.EasyEngineering.net


Downloaded From : www.EasyEngineering.net

352  l  Reading Comprehension

who aspire to Upper-Class Gentilehood) is lovingly enshrined, the foot’s metatarsal phalangeal joint becomes problematic
for instance, in Vanity Fair, with its wide-eyed revelations from when running,” explains Lieberman. Our hominid ancestors,
the dusty alcoves of Kennedy history and obsessive detailing of Australopithecus, of which Lucy is the most famous specimen,
the summerings, winterings, and fallings of obscure Eurotrash. had significantly longer toes than humans. “Lucy could have
(Though how I devour like stale-but-still-tasty Mon Cheri walked just fine with her long toes,” says Lieberman. “But if she
candies Dominick Dunne’s dispatches about, oh, “Arch Viscount wanted to run a marathon, or even a half-marathon, she’d have
Fernando of Capri’s 80th birthday party—he’s a Scorpio!” had trouble.”
featuring murky snaps out from which inevitably loom, like The March study is the first attempt to assess the ER hypothesis
death and taxes, Barry Diller and the shiny gorgon head of Diane using an experimental approach, but the idea that humans have
von Furstenberg. a marathoning past first surfaced more than two decades ago,
78. he phrase “prow-jousting” refers to: when David Carrier, a runner and grad student in the lab of
(a) A technology war (b) Low sea fares evolutionary biologist Dennis Bramble, convinced his mentor
(c) Yacht fighting. (d) Gadget war that running ability might explain a number of unique human
(e) A friendly bout. features. Over the years, Bramble’s team at the University of Utah
79. Which of the following cannot be inferred from the passage?
and Lieberman’s team at Harvard have amassed a small ream
(a) America has a British-style class system.

ww
(b) The book is a precise class almanac.
(c) The book is a modern-day high-society-anxiety chronicler.
(d) Americans are deeply class conscious.
of physiological and morphological evidence that they believe
points to a distance-running legacy. In 2004 the groups co-
published a list of 26 such markers on the human body, including

w.E
(e) The Jews in America are most wealthy.
80. All except which one of the following is a characteristic of
the rich Jews as mentioned in the passage?
short toes, a hefty gluteus maximus and Achilles tendon, springy
tendon-loaded legs, and the little-known nuchal ligament that
stabilizes the head when it’s in rapid motion.
(a) The men are smart and cynical.
(b) The men and women lack taste. asy
(c) The wives are brilliantly well-read.
The paper earned the cover of Nature and generated quite
a stir within bio/anthro circles. But it did nothing to answer a

(d) Some wives are professionals themselves.


(e) Some are Tibetan wool importers. En fundamental question: What good would endurance running
have been to primitive man? On an evolutionary battleground
— where the struggle is to eat or be eaten — speed, and not
81. The author means which of the following when he says,
“Tolkin is particularly hard on his people”? gin
endurance, should be the prized trait. If a tiger in high gear could
outpace Homo erectus within 10 seconds and a deer in 20, being
(a) Tolkin depicts them in a bad light.
(b) Tolkin projects them as greedy people.
(c) Tolkin sees them as hollow people. eer
able to run at a modest pace for hours at a time does not seem
like an evolutionary advantage.

(d) Tolkin finds them typical and unbearable.


(e) Tolkin wants them to rise above the rot. ing
82. Which is the thematic highlight of this passage?
(a) The importance of toes in the evolution of mankind.
(b) Evolution and inheritance of endurance running.
Passage- 24 (c) Mankind’s race to fitness.
.ne
(d) The link between toes and evolution.
A handful of scientists think that ultra-marathoners use their
bodies just as our hominid forbears once did for distance
running, a theory known as the endurance running hypothesis
(ER). ER proponents believe that being able to run for extended
(e) The missing link in the evolution process.
t
83. Which of the following would have not been true if the toe
length in humans had remained long?
(a) We would have not been able to run for long distances.
lengths of time is an adapted trait, most likely for obtaining food,
and was the catalyst that forced Homo erectus to evolve from its (b) We would have remained ape-like.
apelike ancestors. Our toes, for instance, are shorter and stubbier (c) We would have not been able to stand on our two feet.
than those of nearly all other primates, including chimpanzees, a (d) We would have been speed runners.
trait that has long been attributed to our committed bipedalism. (e) We would have evolved much faster.
But a study published in the March 1 issue of the Journal of 84. It has been claimed in the passage that “Lucy could have
Experimental Biology, by anthropologists Daniel Lieberman walked fine with her long toes but …she’d have trouble”.
and Campbell Rolian, provides evidence that short toes make According to the passage, which of the following seem(s)
human feet exquisitely suited to substantial amounts of running.
appropriate reason(s) for such a claim?
In tests where 15 subjects ran and walked on pressure-sensitive
A. Long toes act on joints.
treadmills, Lieberman and Rolian found that toe length had
no effect on walking. Yet when the subjects were running, an B. It creates a force on the foot.
increase in toe length of just 20 percent doubled the amount of C. It generates a stress on the mind.
mechanical work, meaning that the longer-toed subjects required (a) A only (b) B only
more metabolic energy, and each footfall produced more shock. (c) C only (d) A & B
“If you have very long toes, the moment of force acting on (e) B & C

Downloaded From : www.EasyEngineering.net


Downloaded From : www.EasyEngineering.net

Reading Comprehension  l 353

Passage - 25 87. Which of the following best summarizes the passage?


(a) Language is innate in human beings.
The most prominent opposition with which Derrida's earlier
work is concerned is that between speech and writing. According (b) Language is based on pure science.
to Derrida, thinkers as different as Plato, Rousseau, Saussure, and (c) Language is better than speech.
Levi-Strauss, have all denigrated the written word and valorised (d) Language is neither better nor worse.
speech, by contrast, as some type of pure conduit of meaning. (e) Language exists to represent speech.
Their argument is that while spoken words are the symbols of 88. Which of these is the best description of the author’s tone in
mental experience, written words are the symbols of that already the passage?
existing symbol. As representations of speech, they are doubly (a) aggravated (b) analytical
derivative and doubly far from a unity w e attempts to illustrate (c) supercilious (d) diagnostic
that the structure of writing and grammatology are more (e) precise
important and even 'older' than the supposedly pure structure of
presence-to-self that is characterised as typical of speech. Passage- 26
For example, in an entire chapter of his Course in General
Linguistics, Ferdinand de Saussure tries to restrict the science of A spectre is haunting the world, just as Karl Marx and
linguistics to the phonetic and audible word only. In the course Friedrich Engels wrote in the Communist Manifesto of 1848.
of his inquiry, Saussure goes as far as to argue that "language and This time, however, it is not the spectre of communism but

ww
writing are two distinct systems of signs: the second exists for
the sole purpose of representing the first". Language, Saussure
insists, has an oral tradition that is independent of writing,
that of neoliberalism. Just as Marx and Engels reported of ‘a
holy alliance to exorcise this spectre,’ there is once again an
alliance, whether holy or unholy, that has formed to chase the

w.E
and it is this independence that makes a pure science of speech
possible. Derrida vehemently disagrees with this hierarchy and
instead argues that all that can be claimed of writing - e.g. that
it is derivative and merely refers to other signs - is equally true
ghost of neoliberalism from the world stage. In any case, it is
a curious alliance that has committed to fighting neoliberalism:
Religious leaders and artists, environmental activists and

asy
of speech. But as well as criticising such a position for certain
unjustifiable presuppositions, including the idea that we are self-
identical with ourselves in 'hearing' ourselves think, Derrida also
globalisation critics, politicians of the left and the right as well
as trade unionists, commentators and academics. They all share
a passion to unmask neoliberalism as an inhuman, anti-social,

En
makes explicit the manner in which such a hierarchy is rendered
untenable from within Saussure's own text.
and potentially misanthropic ideology or as a cynical exercise
by strangely anonymous forces that wish to exploit the world to
Most famously, Saussure is the proponent of the thesis that is
commonly referred to as "the arbitrariness of the sign", and this
asserts, to simplify matters considerably, that the signifier bears no gin
their own advantage.
The members of this colourful alliance against neoliberalism
are as united in their opposition to neoliberalism as they are
necessary relationship to that which is signified. Saussure derives
numerous consequences from this position, but as Derrida
points out, this notion of arbitrariness and of "unmotivated eer
diverse. This suggests that neoliberalism cannot be too clearly
defined as a concept. Rather, it is a broad umbrella under which
institutions" of signs, would seem to deny the possibility of any
natural attachment. After all, if the sign is arbitrary and eschews
any foundational reference to reality, it would seem that a certain
ing
very different groups with various points of view can meet. In
the church of anti-neoliberalism, there is a place for anyone who
believes that neoliberalism stands in the way of reaching his or
type of sign (i.e. the spoken) could not be more natural than
another (i.e. the written). However, it is precisely this idea of a .ne
her political goals. This may also explain the lack of any clear and
coherent definition of neoliberalism among its dissenters.
natural attachment that Saussure relies upon to argue for our
"natural bond" with sound, and his suggestion that sounds are
more intimately related to our thoughts than the written word
hence runs counter to his fundamental principle regarding the
t
Yet the most curious characteristic of neoliberalism is the
fact that these days hardly anyone self-identifies as a neoliberal.
In former times, ideological debates were fought between, say,
conservatives and socialists, collectivists and individualists.
While there may not have been any other agreement between
arbitrariness of the sign. these opposing groups, at least they would have agreed about
85. According to the passage, which of the following does not their respective identities. A socialist would not have felt offended
follow Derrida’s thoughts on the written word? by a conservative calling him a socialist and vice versa.
(a) The presence-to- self structure is not more important In present-day debates around neoliberalism, on the other
than speech. hand, most accused of holding ‘neoliberal’ views would not
(b) The structure of writing is older than speech. accept being called ‘neoliberal.’ Either they would insist on
(c) Grammatology is more important than language. being something else (whether it is ‘liberal,’ ‘classical liberal,’ or
(d) The deprecation of the written word. ‘libertarian’), or they would simply claim to be misunderstood
(e) The denunciation of grammatology. by their opponents. In any case, scarcely anybody wants to be
86. In the passage Saussure defines language as: a ‘neoliberal’ any more. For example, in an online survey of
(a) A pure science. the readers of Andrew Norton’s blog, out of more than 1,200
(b) A derivative of writing. participants not a single person self-identified with the term,
(c) Being independent of writing. while ‘classical liberal,’ ‘conservative,’ and ‘libertarian’ were strong
(d) Dependent on writing. responses. These are strange debates indeed when the enemy you
(e) A restricted science. are fighting claims he does not exist.

Downloaded From : www.EasyEngineering.net


Downloaded From : www.EasyEngineering.net

354  l  Reading Comprehension

89. According to the passage, the spectre, is being chased by all me a great deal about my reading habits, and about how a text
except which one of the following: reveals itself differently as the reading context changes. Along the
(a) Religious leaders and artists. way, I also began to make some predictions about winners and
(b) Environmental activists and globalisation critics. losers in the evolution of books.
(c) Communists and socialists. Little Dorrit was an accidental choice, but I could hardly
(d) Politicians and trade unionists. have done better. Its length, multiple story lines, 19th-century
(e) Commentators and academics. allusions, and teeming cast of characters helped me to test the
90. Why, according to the passage, is there an association functionality of different formats. Beyond the artifice of my
against the phenomenon of neoliberalism? reading experiment, though, please don't think that technology
(a) Because they cannot face it alone. compromised my ability to appreciate this beloved novel, written
(b) Because it is easy to collaborate against it. in 1857 at the height of Dickens's power and popularity. Just the
(c) Because they resent its spread. opposite.
(d) Because they fear the misuse of the world. I started with the paperback, reading in bed. "Thirty years ago,
(e) Because they are apprehensive of the unknown forces. Marseilles lay burning in the sun. ..." As soon as I opened the
91. The author implies: book, there I was, encountering my name and my own marginal
(a) That neoliberalism defies conceptualization. notations — "Sunshine that illuminates or blinds?" — from

ww
(b) That neoliberalism is vague.
(c) That neoliberalism has not been accepted.
decades ago. That and the $2.45 price marked on the back made
me more than a little nostalgic about my graduate-school days,
when I first fell in love with the Victorian novel. In a book about

92. w.E
(d) That neoliberalism has not been understood by people.
(e) That neoliberalism lacks ideologies.
The word ‘libertarian’ used in the passage can be best
how the present is haunted by the past, I was confronting my
old self through the medium of the physical book, still in great
condition, still fitting perfectly in my hands. How dare we think
substituted with –
(a) neoliberal
(c) conservative asy
(b) libertine- immoral
(d) avant-garde
that anything could replace it?
I've been dreading this, but let me get my prediction out now:
The iPhone is a Kindle killer. Regardless of format, Little Dorrit
(e) liberal- favouring individual liberty

Passage- 27 En seized me no less forcefully today in its indictment of society’s


ability to destroy through greed and crushing self-interest.

Hardcover or paperback? Until recently those were our reading gin


93. What is the main idea of the passage?
(a) That reading aids help sell books.
options. As with everything else, whether it's ice cream or
television, things are much more complicated now. We are way
beyond vanilla and chocolate, way beyond the corner bookstore eer
(b) New formats are conducive to reading.
(c) Formats do not induce reading.
(d) New formats are convenience driven.
and neighborhood library and into a multiplicity of forms
and platforms and technologies and interfaces that could be ing
(e) Books conquer formats.
94. It can be inferred that the speaker puts the onus of good
dispiriting if you are inclined to worry about the death of the
book.
Do I love books or do I love reading? When my book group
reading onto:
(a) the writer
(c) the listener .ne
(b) the reader
(d) the context
picked Little Dorrit, I found myself asking that question. Good
old paid-by-the-word Dickens: I figured that it would take me
months to finish nearly 1,000 pages. My reading would take
(e) the reading aid
95. The author’s tone in the passage is:
(a) light-hearted
(c) pragmatic
(b) sentimental
(d) tentative
t
place on the New York City subway, in cars and planes, on (e) inquisitive.
business trips and vacation, and (my all-time favorite) in bed at
the end of the day. Passage- 28
I went automatically to my old Penguin paperback, standing
The public has become all too aware of the term "bubble" to
ready on the shelf. Never mind its familiar and friendly orange describe an asset that is irrationally and artificially overvalued
spine — I hesitated. Maybe it would make sense to read the book and cannot be sustained. The dot-com bubble burst by 2000. More
on the Kindle that my husband bought me last year. Then again, recently the overextended housing market collapsed, helping to
for my daily Manhattan life, I love audiobooks, the best choice trigger a credit meltdown. The stock market has declined more
for crowded public transportation and a wonderful companion than 30 percent in the past year, as companies once considered
for walking. And now that I use an iPhone, I have been surprised flagship investments have withered in value.
by the ease of reading its crisp, bright screen. Is it possible that higher education might be the next bubble to
I decided to read Little Dorrit four ways: paperback, burst? Some early warnings suggest that it could be.
audiobook, Kindle, and iPhone. With tuitions, fees, and room and board at dozens of colleges
It was often maddening to keep finding and losing my place as now reaching $50,000 a year, the ability to sustain private
I switched from format to format. But as an experiment, it taught higher education for all but the very well-heeled is questionable.

Downloaded From : www.EasyEngineering.net


Downloaded From : www.EasyEngineering.net

Reading Comprehension  l 355

According to the National Center for Public Policy and Higher 98. Which of the following cannot be inferred from the passage?
Education, over the past 25 years, average college tuition and (a) Average college tuition and fees have risen astronomically.
fees have risen by 440 percent — more than four times the rate (b) The public is of the opinion that higher education might
of inflation and almost twice the rate of medical care. Patrick be overpriced and under-delivering.
M. Callan, the center's president, has warned that low-income (c) Applications to public institutions have risen drastically.
students will find college unaffordable. (d) The number of enrolments in higher education will
Meanwhile, the middle class, which has paid for higher decline.
education in the past mainly by taking out loans, may now be (e) In an effort to secure students some institutions are
precluded from doing so as the private student-loan market has reviewing their tuition fees.
all but dried up. In addition, endowment cushions that allowed 99. This passage is most likely an extract from which of the
colleges to engage in steep tuition discounting are gone. Declines following?
in housing valuations are making it difficult for families to rely on (a) An article in a journal on economics.
home-equity loans for college financing. Even when the equity is (b) A book on Higher Education.
there, parents are reluctant to further leverage themselves into a (c) A dissertation on the theory of Education.
future where job security is uncertain. (d) An essay on of the current state of Higher Education.
(e) A newspaper article on the state of higher education.

ww
Consumers who have questioned whether it is worth spending
$1,000 a square foot for a home are now asking whether it is
worth spending $1,000 a week to send their kids to college. There
Passage- 29
The monster Caliban, according to his master, Prospero, was “a

w.E
is a growing sense among the public that higher education might
be overpriced and under-delivering.
In such a climate, it is not surprising that applications to some
devil, a pure devil, on whose nature nurture can never stick”. Yet
only a few decades before Shakespeare wrote The Tempest, St
Ignatius Loyola had founded the Jesuit order, with its famous

asy
community colleges and other public institutions have risen by as
much as 40 percent. Those institutions, particularly community
colleges, will become a more-attractive option for a larger swath
maxim: “Give me the child until he is 7, and I will show you the
man.”
This ancient debate over the relative contributions of

En
of the college bound. Taking the first two years of college while
living at home has been an attractive option since the 1920s, but
inheritance and experience to the human condition has never
been more charged than in the genetic age. On one side stood
it is now poised to grow significantly.
With a drift toward higher enrollments in public institutions, gin
those who sought and saw genetic explanations for human
psychology; on the other, those who believed it to be moulded
all but the most competitive highly endowed private colleges are
beginning to wonder if their enrollments may start to evaporate.
Could it get worse for colleges in the coming years? The numbers
eer
by culture. There was little common ground. Sarah Blaffer Hrdy,
an evolutionary psychologist, has even joked that perhaps we are
genetically programmed to set nature against nurture.
of college-aged students in the "baby-boom echo," which crested
with this year's high-school senior class, will decline over the ing
Since the middle of the last century the nurture camp has been
dominant. Just as molecular biology began to unravel the secrets
next decade. Certain Great Plains and Northeastern states may
lose 10 percent of the 12th-graders eligible for college. Vermont .ne
of DNA, genetics and evolution were relegated to psychological
bit-players by a new orthodoxy, which held that biology has
forged a human mind of almost limitless malleability. It was the
is expected to lose 20 percent by 2020.
96. It can be inferred that the author mentions incidents
regarding the term “bubble” in order to:
(a) describe an asset that is irrationally and artificially
doctrine of the blank slate.
t
The idea, usually traced to the 17th-century philosopher John
Locke, grew popular in the Enlightenment, fitting the mood of
challenge to the supposedly innate authority of monarchy and
overvalued and cannot be sustained. aristocracy. It was a statement of individual freedom, which
(b) depict the public awareness about overrated and became strongly associated with the political Left. Though many
untenable qualities of a product. early socialists were enthusiasts for eugenics, later generations
(c) expose the decline in the stock markets. grew suspicious of genetics, particularly after it was abused to
(d) caution us about the extraordinary trends in higher justify oppression of disadvantaged racial and social groups,
education. most brutally in Nazi Germany. Liberal opinion turned
(e) portray that higher education is going out of reach of the against the concept of a biological human nature, which was
common man. increasingly seen as a tool with which male and bourgeois elites
97. Which of the following title best fits the passage? could rationalise hegemony. The movement was driven by the
(a) Higher Education common man’s angst. social sciences. From psychology came Sigmund Freud's notion
(b) Higher Education the Next Bubble to Burst. that attitudes and mental health are explained by childhood
(c) Higher Education going out of reach. experience. The behaviourism of B.F.Skinner added the claim
(d) Higher Education- a need for reform. that human beings could be conditioned by training, much as
(e) Higher Education in doldrums. Ivan Pavlov's celebrated dogs salivated at the sound of a bell. From

Downloaded From : www.EasyEngineering.net


Downloaded From : www.EasyEngineering.net

356  l  Reading Comprehension

anthropology came the research of Franz Boas and Margaret it must be cleaned out to eliminate all the old plants, including
Mead, whose comparative studies of different societies suggested all the roots. A tiller will soften and aerate the soil; it will help
that traditions could steer human behaviour in a multitude of loosen any roots missed. After the first tilling, add fertilizer, top
directions. Mead's purported discovery of free love among soil, mulch and vermiculite and then till the garden plot again
Samoan women was influential because - though founded on to mix everything. It is just like mixing a cake; it must be mixed
poor data - it challenged prevailing sexual mores. Karl Marx's thoroughly. When tilling is complete, gardeners should take a
political and economic theories saw human nature as something rake and smooth the top of the soil.
to be reshaped and directed to facilitate revolution. And Selecting and planting the seeds is the next step in the
postmodernism contributed the mantra that even knowledge gardening process. It is imperative to make sure seeds are selected
and truth are socially constructed and relative. for the area of the country where the garden is located. Seeds that
100. It can be inferred that Prospero’s comment on Caliban “a grow well in the mountains may not grow as well near the ocean.
devil, a pure devil, on whose nature nurture can never stick” One piece of advice is to subscribe to a gardening magazine or
meant: purchase a good gardening book. The information obtained
(a) That Caliban was naturally evil. from either source will help prevent the disappointment of no
(b) That Caliban was evil and nurturing had no effect on harvest. Most seeds only need to be planted one fourth inch deep.
him. The seeds should be sprinkled on your garden and raked to get


ww
(c) That devils are cruel people and nurture is beyond them.
(d) That nurture is effective on a certain type of people.
(e) That nature works through nature.
them below the soil. After planting the seeds, the entire garden
needs a thorough watering; water every day for a week to speed
up seed germination.

w.E
101. According to the passage, what is meant by the sentence
“genetically programmed to set nature against nurture?”
(a) That in humans, genetics overrides nurture.
104. Which of the following inferences can you make about the
garden plot being tilled at least two times?
(a) Old plant roots can make your harvest more plentiful.

asy
(b) That genetic features are difficult to change, but
environmental features are easy to change.
(c) That humans are what they become.
(b) Plant roots can go downward deeper into the earth.
(c) Loose, soft soil absorbs water less than tight, hard soil.
(d) Seeds are self resilient and can seek new ways of nurture.
(d) That nurture is temporary and nature is permanent.
(e) That the mind is susceptible to nurture. En (e) Tilling to remove old debris will not strengthen your
plants.
102. It can be inferred that socialists turned away from heredity
because:
(a) It was used as a tool of oppression.
gin
105. What is the main purpose of using lawn timbers when you
plant a fall garden?
(a) They help control weeds
(b) It was associated with the Leftists.
(c) It helped to secure authority. eer
(b) They make the garden more beautiful.
(c) They help ventilate the soil.
(d) They work as manure.
(d) It aided oppression.

103.
(e) It justified human nature.
The expression “blank slate” alludes to which of the
ing
(e) They increase the usage.
106. The passage does not concern itself with which of the
following?
(a) That the mind was free from all influences.
following?
(a) How to prepare a fall garden
(b) Selecting and planting the seed .ne
(b) That the mind held all the answers.
(c) That the mind was extremely flexible.
(d) That the mind was restless.
(e) That the mind was extremely intractable.
(c) Comparing gardening to baking a cake
(d) Drawbacks of fall gardens
(e) Fall gardens and their timelessness
t
Passage- 30 Passage- 31
The reason why Taoism and Zen present, at first sight, such a
Garden design is the first step in having a productive fall garden.
puzzle to the Western mind is that we have taken a restricted view
One design that works well is to mark off a ten by sixteen-foot
of human knowledge. For Westerners, almost all knowledge is
rectangle. Divide the rectangle into no more than five or six plots. what a Taoist would call conventional knowledge, because we do
The plots should be small enough so gardeners can reach to the not feel that we really know anything unless we can represent it to
middle for easy weeding and picking. A trench should be dug ourselves in words, or in some other system of conventional signs
about three inches deep and wide around each plot to make sure such as the notations of mathematics or music. Such knowledge
rain will not drown the plants. One tip to make gardening more is called conventional because it is a matter of social agreement
pleasant is to use lawn timbers (eight feet in length) to form a as to the codes of communication.
border around the outside of the lot. Using lawn timbers is an When we turn to ancient Chinese society, we find two
excellent idea because they help control weeds and are very "philosophical" traditions playing complementary parts--
inexpensive; seconds sell for about a dollar each. Confucianism and Taoism. Generally speaking, the former
Soil preparation is also an important step to having an concerns itself with the linguistic, ethical, legal, and ritual
abundant harvest. If the garden space was used in the summer, conventions which provide the society with its system of

Downloaded From : www.EasyEngineering.net


Downloaded From : www.EasyEngineering.net

Reading Comprehension  l 357

communication. Confucianism, in other words, preoccupies itself 109. A study of the given passage suggests which of the following?
with conventional knowledge, and under its auspices children (a) Taoism is not a formal religion or philosophy, nor is it a
are brought up so that their originally wayward and whimsical type of coherent science; it is rather a “way of liberation”,
natures are made to fit the Procrustean bed of the social order. similar to Yoga
The individual defines himself and his place in society in terms (b) The problem of understanding Taoism is simply one
of the Confucian formulae. of mastering ideas different from our own, say, as the
Taoism on the other hand, is generally a pursuit of older men, theories of Kant differ from those of Descartes
and especially of men who are retiring from active life in the (c) In Eastern philosophy, Taoism is primarily a literary
community. Their retirement from society is a kind of outward tradition, which is characterizes as being "rational" and
symbol of an inward liberation from the bounds of conventional "structured"
patterns of thought and conduct. For Taoism concerns itself (d) All of the above
with unconventional knowledge, with the understanding of life (e) None of the above
directly, instead of in the abstract, linear terms of representational
thinking. Passage- 32
Confucianism presides, then over the socially necessary
Religious behaviour has been investigated by a wide range of
task of forcing the original spontaneity of life into the rigid
disciplines: Anthropologists deal with the comparative study of
rules of convention-a task which involves not only conflict and

ww
pain, but also the loss of that peculiar naturalness and un-self-
consciousness for which little children are so much loved, and
which is sometimes regained by saints and sages. The function of
primitive religions, examining prayer, ritual, the rites of passage,
etc. Sociologists have investigated the institutional aspects
of religious behaviour, such as the role of the priestly class in

w.E
Taoism is to undo the inevitable damage of this discipline, and
not only to restore but also to develop the original spontaneity,
which is termed tzu-jan or "self-so-ness." For the spontaneity of a
society. Ever since William James, psychologists of religion have
studied the varieties of religious experience, such as mysticism,
ecstasy, talking in tongues, exorcism, etc. Similarly, biologists

asy
child is still childish, like everything else about him. His education
fosters his rigidity but not his spontaneity. In certain natures, the
conflict between social convention and repressed spontaneity is
have postulated a role for religious beliefs and practices in the
evolutionary process and their possible adaptive/survival value.
They have asked, does religiosity have a genetic or environmental

En
so violent that it manifests itself in crime, insanity, and neurosis,
which are the prices we pay for the otherwise undoubted benefits
basis? Others have focused on the neurological correlates of
religious piety, and still others have attempted to test the efficacy
of order.
107. The author of the passage is not likely to agree with which of
the following inferences about Taoism? gin
of prayer.
One can deal with religion in contemporary or historical
contexts. A great deal of attention has been devoted to the
(a) Taoism tries to bring into play the mind’s innate and
spontaneous intelligence by using it without forcing it.
(b) Taoism will remain incomprehensible as long as the
eer
historical analysis of religious claims, especially since the great
classical religions are based on ancient documents (the Old
conscious intellect is tries to clutch the world in its net
of abstractions and insists that life be fitted to its rigid ing
and New Testaments and the Koran), as are some of the newer
religions (such as the nineteenth-century Book of Mormon).
These texts allege that certain miraculous and revelatory events
categories.
(c) Taoism works by holistically reducing the mind to a state
of receptive vacuity, so that it can ‘unlearn’ conventional .ne
have occurred in the past and these warrant religious belief today;
and it is often claimed that belief in them is based upon faith.
learning.
(d) The formal categories of modern Western thought are
incapable of containing and articulating the teachings
of Taoist thought.
t
Paranormal claims are similar to religious claims-both purport
to be exceptions to natural laws. Skeptics have asked: Did D.D.
Home float out of a window and levitate over a street in London
in the late nineteenth century? Did the Fox sisters and Eusapia
(e) Child education fosters rigidity and not spontaneity. Palladino possess the ability to communicate with the dead?
108. Which of the following statements is validated by the And they have sought to provide naturalistic interpretations
passage? for reports of bizarre events. No doubt it is easier to examine
(a) The loss of spontaneity which happens through contemporaneous claims where the record is still available rather
Confucianism can be somewhat restored by reverting to than ancient ones where the record may be fragmentary. Yet
Taoism
in principle at least, the religious investigator is similar to the
(b) Western musicians rely much more on fixed tonal and
rhythmic intervals than do Oriental musicians paranormal investigator, attempting to ascertain the accuracy
(c) Westerners do not go into the depth of concepts that of the historical record. We use similar methods of inquiry to
require much more than a dependence on ‘peripheral examine prosaic historical questions, such as: Did Washington
vision’. cross the Delaware, or Thomas Jefferson sire the children of Sally
(d) In Chinese thought, insanity and crime are a result of Heming? The same goes for religious claims: Did the Red Sea
the imagination and naturalness being repressed by part before the fleeing Hebrews, was there a Great Flood and a
institutional structures. Noah's Ark? I don't see how or why we should declare that these
(e) The Chinese are poles apart from the Westerners. historical religious claims are immune to scientific investigation.

Downloaded From : www.EasyEngineering.net


Downloaded From : www.EasyEngineering.net

358  l  Reading Comprehension

‘Some have argued that religious phenomena-matters of faith-are the past 20 years, India still endures high rates of poverty that
entirely beyond the ken of science; but this surely is false because Asia, by and large, has long left behind. Surprisingly few Indian
the scientific investigation of religion has already made great politicians, officials, or press folk show much interest in getting
strides and there is a vast literature now available.’ to grips with economic questions. Even now, with growth sliding
Religious belief systems are deeply ingrained in human to a worrying 5% or so, public debate on economic matters is
history, culture, and social institutions that predate science, and limited to a narrow field.
thus it is often difficult, if not impossible, to insist upon using Lots of possible explanations exist. Discussing economic ideas
the standards of objective skeptical inquiry retrospectively. This is quite different from applying them. India’s stifling bureaucracy,
is especially the case since to believe in a religion is more than or its demanding politics, or perhaps its troubled universities,
a question of cognitive assent, for religion has its roots in ethnic have driven bright economists abroad. And perhaps the country’s
or national identity; and to question the empirical or rational brightest economists are simply rubbish at communicating sharp
grounds for religious belief is to shake at the very foundations of ideas to the policymakers, activists, media types, business leaders
the social order. and members of the public who could make use of them.
Too often economists are preoccupied with the intellectual
110. The passage deals mainly with:
skill of setting out and solving problems, scribbling equations
(a) Refuting the claims of religion by recourse to scientific
and debating the finer points of theory. But actually doing
theories
something useful then requires describing and explaining it in a

ww
(b) Proving that every phenomenon is nature can be
explained by scientific theories
(c) Establishing that the claims of religion can be examined
way that others can understand. Basing their work in empirical
data might also help politicians—for example—see directly

w.E
through scientific objectivity
(d) Establishing that there are naturalistic explanations for
the so-called bizarre events
how economic insights can be useful when governing. So the
launch of a new Indian economics blog, called Ideas for India,
looks welcome. Unveiled last week in Delhi by Abhijit Banerjee,
a famous economics professor at the Massachusetts Institute of

111. It can be inferred that-
asy
(e) Systems and their effects on humans.

(a) The science should be construed in such a way that it


Technology (who called himself a “cheerleader” for the project),
along with Jairam Ramesh, an Indian cabinet minister, the blog

only applies to experimental laboratory work.


En
(b) Scepticism should be committed only to "methodological
gives economists (and other academics) in India and beyond
a chance to write for non-expert readers. Its first articles give
a sense of what is on offer: an analysis of corruption within
naturalism" and not scientific naturalism while
examining religion.
(c) Science should bring in the tools of logical analysis,
gin
India’s rural job-creation scheme; an economic explanation for
why caste identity persists; an essay on the allocation of land

historical research, and rational investigation while


examining religion. eer
within India. For the general reader on India this could become
a treasure trove: each day a clutch of experts tell you, in around
1,000 words, the essence of their current research, without


(d) We lack the resources and expertise to focus on the
entire range of scientific questions about religion.
(e) Science is the best way out of the religious conundrum.
ing
footnotes or squiggly equations but with suggestions for further
reading at the end.

112. A possible proposition of the last paragraph is:


(a) The sacredness of religion has firmly put it beyond the .ne
Similar sites exist in other parts of the world, most notably
Vox in Europe (where you’ll notice more than a few Indian
economists as contributors). Credit for the new Indian goes, in
sphere of the scientific and the rational.
(b) It is unfeasible for people to let go off long-ingrained
religious beliefs.
(c) In order to protect social order, it is essential that
t
part, to the International Growth Centre, based at the London
School of Economics in partnership with Oxford University.
The contributors, by and large, are Indians, or people of Indian
origin based in America and elsewhere. Their challenge is to
scientific enquiry give way to religious beliefs. engage their countrymen. If they succeed, the rewards could be
(d) To question the tenets of orthodox religion is likely to magnificent.
raise the public charge of blasphemy and heresy. 113. ‘Here is a paradox’. Which of the following is the most likely
(e) Science and religion must coexist or perish. paradox that the author is referring to?
(a) India has a very high rate of poverty even after economic
Passage- 33 expansion.
HERE is a paradox. India churns out lots of brain boxes, (b) Despite India producing brilliant economists, its
including clever economists who thrive at home and abroad. economy is far behind some Asian economies.
Almost anywhere in the world you can drop into a bookshop, (c) In spite of very good universities in India, good
an international development institution, a university or a economics students go abroad.
prospering company and likely as not be confronted by bright (d) Good economists are good only in theory but not in
Indians offering sharp analyses of how best to fight poverty, practice.
create wealth and promote innovation. Yet the performance of 114. Consider the following assumptions:
the Indian economy itself, over the decades, fell well behind 1. Economists should explain their theories in a way that
(most of) the rest of Asia. Despite even the rapid expansion of others can comprehend.

Downloaded From : www.EasyEngineering.net


Downloaded From : www.EasyEngineering.net

Reading Comprehension  l 359

2. Brilliant economists go abroad as living standards are mathematics—the “finitary attitude” as he characterizes this—
much better there. as either constructive or intuitive. The latter is... the specifically
3. Indian economists are best at discussing economic finitistic element [that] requires in addition that the objects
issues abroad. and facts considered should be given in concrete mathematical
With reference to the passage which of the above is/ are intuition.”
NOT valid? So Gödel noted how we can go beyond the intuitive property.
(a) 1 and 2 only (b) 1 and 3 only He retained the constructive property though. He even retained
(c) 2 only (d) 1, 2 and 3 an infinitary property. Functions on a denumerable set were
115. The bright Indian economists are unable to help in solving mobilized. Finite, per se, is of some use only for us. We require
its economic problems because countability and not just finiteness. For Gödel there is“plausibility
(a) They have no interest in solving the problem. that all things conceivable by us are denumerable.”
(b) The Government does not take suggestions from them 117. Which of the following Kant theories was Gödel to a certain
in this matter. extent in agreement with?
(c) Most of them live abroad. (a) Time exists in itself.
(d) They are not able to communicate their views to the (b) Time is an individual characteristic.
Government and other agencies. (c) Time can be understood only in relation to something
116. According to the passage, which of the following suggests a else.

ww
possible reason for politicians being unable to govern and
solve problems in India?
(d) Every happening is pre destined.
118. Which of the following statements is NOT supported in the

w.E
(a) Politicians do not base their governing decisions on
available economic data
(b) Politicians do not communicate with economists.
(c) Politicians are not students of economics.
passage?
(a) Kant’s theories were not altogether false.
(b) Feuerman disagreed with author’s who regarded Gödel’s
theories as incoherent.
(d) None of the above

Passage - 34
asy (c) Mathematics in which evidence depends on intuition is
finite.

Gödel took a somewhat distanced baseline view of Kant:


En (d) Kant supported the theory of Relativity.
119. The passage is mainly about :
(a) Gödel’s assertions on mathematics and time.
“a general feature of Kant’s assertions that if literally understood,
are false, but in a broader sense contain deeper truths.” Faced
with the views of some authors that Gödel’s views were “unstable,” gin (b) Kant’s assertions on theory of Relativity.
(c) Differences in assertions made by Gödel and Kant.
“unsettled,” “vacillating” and even incoherent, Feuerman,
nonetheless, argues for a level of coherence ultimately to be
found in Gödel’s views. Gödel recognized that for Kant, time is
eer
(d) The theory of Relativity.
120. Which of the following theories was supported by Kant?
(a) Time is a multidimensional temporal ordering of the
“not ‘something existing in itself ’ (i.e., a separate entity besides
the objects in it) nor ‘a characteristic or ordering inherent in the
events.
ing
(b) Time is relative to the perceiving subject or its ‘sensibility’.
objects’ but only a characteristic inherent in the relation of the
objects to something else.” With reference to relativity theory,
the idea that time “as its most essential characteristic...consists
.ne
(c) In finite mathematics evidence rests on what is intuitive.
(d) Time consists of a one-dimensional system of points, in
which every happening in the world has a definite place.
[in the traditional view] of a one-dimensional system of points,
isomorphic with a straight line, in which every happening
in the world has a definite place.” Instead, for Kant, time as “a
one-dimensional temporal ordering of the events” is the case,
Passage - 35
We have inherited the tradition of secrecy about the budget
from Britain where also the system has been strongly attacked
t
and is “relative to the perceiving subject or more precisely its by eminent economists and political scientists including Peter
‘sensibility’.” And again: for Kant, “that temporal properties Jay. Sir Richard Clarke, who was the originating genius of
represent certain relations of the things to the perceiving subject nearly every important development in the British budgeting
appears from many passages in his writings.” techniques during the last two decades, has spoken out about
Gödel addresses how finiteness, associated for example with the abuse of budget secrecy: “The problems of long-term tax
inductive proof, is intuitive: “finite mathematics is defined as the policy should surely be debated openly with the facts on the
mathematics in which evidence rests on what is intuitive.” And table. In my opinion, all governments should have just the same
(emphasis in original): we cannot acquire knowledge intuitively duty to publish their expenditure policy. Indeed, this obligation
by passing stepwise from smaller to larger ordinals; we can only to publish taxation policy is really essential for the control of
gain knowledge abstractly by means of notions of higher type. public expenditure in order to get realistic taxation implications.”
Gödel continues: This, as far as the objects are concerned, Realising that democracy flourishes best on the principles of
means that they must be finite space-time configurations of open government, more and more democracies are having an
elements whose nature is irrelevant except for equality or open public debate on budget proposals before introducing the
difference. (In contrast to this, the objects in intuitionist Logic appropriate Bill in the legislature. In the United States the budget
is conveyed in a message by the President to the Congress, which
are meaningful propositions and proofs.) Gödel refers to finitary

Downloaded From : www.EasyEngineering.net


Downloaded From : www.EasyEngineering.net

360  l  Reading Comprehension

comes well in advance of the — date when the Bill is introduced 126. Sir Richard Clarke seems to deserve the credit for
in the Congress. In Finland the Parliament and the people are (a) transformation in the British budgetary techniques.
already discussing in June the tentative budget proposals which (b) maintenance of secrecy of the British budget.
are to be introduced in the Finnish Parliament in September. Every (c) detection of abuse of transparency in budget.
budget contains a cartload of figures in black and white - but the (d) bringing down the tax load on British people.
dark figures represent the myriad lights and shades of India’s (e) None of these
life, the contrasting tones of poverty and wealth, and of bread 127. From the contents of the passage, it can be inferred that the
so dear and flesh and blood so cheap, the deep tints of adventure
author is
and enterprise and man’s ageless struggle for a brighter morning.
(a) authoritarian in his approach.
The Union budget should not be an annual scourge but a part
of presentation of annual accounts of a partnership between the (b) a democratic person.
Government and the people. That partnership would work much (c) unaware of India’s recent economic developments.
better when the nonsensical secrecy is replaced by openness (d) a conservative person.
and public consultations, resulting in fair laws and the people’s (e) None of these
acceptance of their moral duty to pay. 128. Which of the following statement(s) is/are definitely False
121. How do the British economists and political scientists react in the context of the passage?
to budget secrecy? They are A Transparency helps unscrupulous elements to resort to

ww
(a) in favour of having a mix of secrecy and openness.
(b) indifferent to the budgeting techniques and taxation
policies.
corrupt practices.
B Open approach of Government is a sign of healthy

w.E
(c) very critical about maintenance of budget secrecy.
(d) advocates of not disclosing in advance the budget
contents.
democracy.
C People’s acceptance of their moral duties can best be
achieved through openness and public consultations.
(a) Only A (b) Only B
(e) None of these
asy
122. The author thinks that openness in budget is essential as it
(c) Only C
(e) B and C
(d) A and B

leads to
(a) prevention of tax implications
En
(b) people’s reluctance to accept their moral duties
129. For making the budget realistic, the Government should
(a) refrain from making public the proposed provisions

(c) exaggerated revelation of the strengths and weaknesses


of economy gin before finalisation.
(b) discuss it secretly within themselves.
(c) encourage the public to send in their suggestions.
(d) making our country on par with Finland
(e) None of these
123. The author seems to be in favour of
eer
(d) consult the public, defend their own plans and accept
public suggestions.
(a) maintaining secrecy of budget
(b) judicious blend of secrecy and openness ing
(e) None of these
DIRECTIONS (Qs. 130-132) : Choose the word which is most
(c) transparency in budget proposals
(d) replacement of public constitution by secrecy in the passage.
130. SCOURGE .ne
nearly the SAME in meaning to the word printed in bold as used

(e) None of these


124. The secrecy of the budget is maintained by all of the
following countries except
A Finland
(a) ritual
(c) whip
(e) remedy
(b) presentation
(d) t
compromise

B India 131. MYRIAD


C United States (a) adequate (b) functional
(a) Only A (b) Only B (c) incompatible (d) abundant
(c) Only C (d) A and C (e) excellent
(e) B and C 132. DUTY
125. Which of the following statements is definitely TRUE in (a) obligation (b) imposition
the context of the passage? (c) tax-liability (d) function
(a) The British Government has been religiously main (e) job
taining budget secrecy. DIRECTIONS (Qs. 133-135) : Choose the word/phrase which
(b) Budget secrecy is likely to lead to corrupt practices. is most OPPOSITE in meaning to the word printed in bold as
(c) Consulting unjustifiable taxes with public helps make used in the passage.
them accept those taxes. 133. FLOURISHES
(d) There should be no control on public expenditure in (a) disappears (b) degenerates
democratic condition.
(c) vanishes (d) blooms
(e) None of these
(e) opens

Downloaded From : www.EasyEngineering.net


Downloaded From : www.EasyEngineering.net

Reading Comprehension  l 361

134. DEBATED intent on inflicting the maximum amount of pain on the victim.”
(a) questioned severely 136. In the context of the passage, the culprit’s act of emptying
(b) opposed strongly a test tube containing some fluid can be classified as
(c) accepted unconditionally (a) a terrorist attack
(d) discussed frankly (b) an epidemic of a dreaded disease
(e) implemented forcibly (c) a natural calamity
135. IMPORTANT (d) panic created by an imaginary event
(a) major (b) uncountable (e) None of these
(c) significant (d) unscheduled 137. In what way would the new terrorism be different from that
(e) trivial of the earlier years?
A More dangerous and less baffling
Passage - 36 B More hazardous for victims
In the second week of August 1998, just a few days after the C Less complicated for terrorists
incidents of bombing the US embassies in Nairobi and Dar-es- (a) A and C only
Salaam, a high-powered, brain-storming session was held near (b) B and C only
Washington D.C., to discuss various aspects of terrorism. The (c) A and B only

ww
meeting was attended by ten of America’s leading experts in
various fields such as germ and chemical warfare, public health,
disease control and also by the doctors and the law-enforcing
(d) All the three
(e) None of these
138. What was the immediate provocation for the meeting held

w.E
officers. Being asked to describe the horror of possible bio-attack,
one of the experts narrated the following gloomy scenario.
A culprit in a crowded business centre or in a busy shopping
in August 1998?
(a) The insistence of America’s leaders
(b) The horrors of possible bio-attacks

asy
mall of a town empties a test tube containing some fluid, which
in turn creates an unseen cloud of germ of a dreaded disease like
anthrax capable of inflicting a horrible death within 5 days on
(c) A culprit’s heinous act of spreading germs
(d) People’s lack of trust in the government
(e) None of these

have mild influenza which may recede after a day or two. Then
En
any one who inhales it. At first 500, or so victims feel that they 139. What could be the probable consequences of bio-attacks,
as mentioned in the passage?
the symptoms return again and their lungs start filling with fluid.
They rush to local hospitals for treatment, but the panic-stricken
people may find that the medicare services run quickly out of drugs gin A Several deaths
B Political turmoil
C Social unrest
due to excessive demand. But no one would be able to realise that
a terrorist attack has occurred. One cannot deny the possibility eer
(a) A only
(c) C only
(b) B only
(d) A and B only
that the germ involved would be of contagious variety capable of
causing an epidemic. The meeting concluded that such attacks,
apart from causing immediate human tragedy, would have dire ing
(e) All the three
140. The author’s purpose of writing the above passage seems
to explain
long-term effects on the political and social fabric of a country by
way of ending people’s trust on the competence of the government.
.ne
(a) the methods of containing terrorism
(b) the socio-political turmoil in African countries
The experts also said that the bombs used in Kenya and Tanzania
were of the old-fashioned variety and involved quantities of high
explosives, but new terrorism will prove to be more deadly and
probably more elusive than hijacking an aeroplane or a gelignite
(d) reasons for killing innocent people
t
(c) the deadly strategies adopted by modern terrorists

(e) the salient features of terrorism of yester years


141. According to the author of the passage, the root cause of
of previous decades. According to Bruce Hoffman, an American terrorism is
specialist on political violence, old terrorism generally had a A Religious fanatism
specific manifesto - to overthrow a colonial power or the capitalist B Socio-political changes in countries
system and so on. These terrorists were not shy about planting
C The enormous population growth
a bomb or hijacking an aircraft and they set some limit to their
(a) A only (b) B only
brutality. Killing so many innocent people might turn their natural
(c) C only (d) A and B only
supporters off. Political terrorists want a lot of people watching
(e) All the three
but not a lot of people dead. “Old terrorism sought to change the
world while the new sort is often practised by those who believe 142. The phrase “such attacks”, as mentioned in the last sentence
that the world has gone beyond redemption”, he added. of the second paragraph, refers to
Hoffman says, “New terrorism has no long-term agenda but is (a) the onslaught of an epidemic as a natural calamity
ruthless in its short-term intentions. It is often just a cacophonous (b) bio-attack on political people in the government
cry of protest or an outburst of religious intolerance or a (c) attack aimed at damaging the reputation of the
protest against the West in general and the US in particular. Its government
perpetrators may be religious fanatics or diehard opponents of a (d) bio-attack manoeuvred by unscrupulous elements
government and see no reason to show restraint. They are simply (e) None of these

Downloaded From : www.EasyEngineering.net


Downloaded From : www.EasyEngineering.net

362  l  Reading Comprehension

143. The sole objective of the old terrorism, according to thought. Later, he was married to a beautiful princess. She was
Hoffman, was to called Yashodhara. They had a son and named him Rahul. The
(a) plant bombs to kill innocent people king hoped that Siddharth would become a great ruler.
(b) remove colonial power or capitalist system One day Siddharth was driving through the street in his chariot.
(c) make people realise the incompetence of the He saw an old man and then a sick man. The oldman could hardly
government walk. The sick man groaned in pain. Then he saw some people
(d) give a setback to socio-political order carrying a dead body, others were wailing and weeping at the loss
(e) None of these of a dear one. Siddharth was very upset to see so much suffering
144. Which of the following statements is true about new and unhappiness. He was shocked. Then he saw an entirely
terrorism? different sight. A man in yellow robes was walking along the
(a) Its immediate objectives are quite tragic. street. There was no trace of sadness on his radiant face, instead
(b) It has farsighted goals to achieve. it shone with peace. He was a monk who had given up the world
(c) It can differentiate between the innocent people and to escape the misery of life.
the guilty. Siddharth wanted to find out why there was so much suffering
(d) It is free from any political ideology. in the world. He wanted to find out how men could be free of
(e) It advocates people in changing the socio-political misery. He could find neither peace nor happiness in the life he

ww
order
DIRECTIONS (Qs. 145-147): Choose the word which is most
OPPOSITE in meaning of the word printed in bold as used in
was leading at the palace. One night Siddharth left his home,
his wife and his little son. He went into the forest. He wanted to
search for a way out of suffering and sorrow for all mankind. He
the passage
145. gloomy
w.E
(a) discouraging (b) disgusting
meditated and got enlightenment. He became Lord Buddha, the
enlightened one. Truth was revealed to him and he learnt all the
secrets of life and the world.
(c) bright
(e) versatile
146. cacophonous
(d) tragic

asy He found out that the world was full of sorrow and unhappiness.
The reason for it was greed and selfishness. To be free from
(a) loud
(c) sonorous
(b)
(d) harsh
melodious
En suffering, we must be free from greed and desire. Desire is the
root cause of all human suffering. He advocated the Middle Path
(e) distant
147. intolerance
(a) forbearance (b) pesmissibeness
gin
and asked his followers to avoid the two extremes.
[IBPS Clerk 2012]
151. Which of the following sentences is true according to the
(c) adaptability
(e) faithfulness
(c) acceptance passage?
eer
(a) Lord Budha was born in Lumbini.
DIRECTIONS (Qs. 148-150): Choose the word which is most
nearly the SAME in meaning of the word printed in bold as used
in the passage.
ing
(b) The little boy was very happy with fine clothes, good
food and toys.
(c) Siddharth divorced his first wife.
148. perpetrators
(a) opponents (b) followers were very unhappy. .ne
(d) Siddharth and Yashodhara had no children and so they

(c) sympathisers
(e) manoeuvrers
149. elusive
(a) harmful
(d) leaders

(b) fatal
(e) None of these
152. What was Siddharth interested in as a child ?
t
(a) He was interested in making new friends as he did not
have any siblings.
(c) destructive (d) baffling (b) His interests were largely in studying and reading books.
(e) obstructing (c) He was interested in the best of clothes and good toys.
150. inflicting (d) In spending time alone in deep thought.
(a) elevating (b) imposing (e) In spending time with nature.
(c) alleviating (d) reflecting 153. Which of the following can be inferred about Siddharth ?
(e) soothing (A) Siddharth was different from other princes his age.
(B) Siddharth was a spoilt child.
Passage - 37 (C) Siddharth was lonely because he did not have
Lumbini is a beautiful place in the southern part of Nepal. About a mother.
2,500 years ago, a baby boy was born to the king and queen. The (a) Only (A) (b) Only (B)
baby was named Siddharth. His mother, died when he was five days (c) Only (C) (d) Only (B) and (C)
old. The baby boy grew into a handsome prince. His father tried (e) All (A), (B) and (C)
to keep him happy. The little prince had everything he needed — 154. What did the King wish for his son, Siddharth ?
fine clothes, the best food and good toys. But he was not interested (a) He wanted Siddharth to become a great ruler.
in them. He wanted to be alone and was always found in deep (b) He wished that Siddharth would not marry Yashodhara.

Downloaded From : www.EasyEngineering.net


Downloaded From : www.EasyEngineering.net

Reading Comprehension  l 363

(c) He wished that Siddharth’s son would take over his 162. Hardly
kingdom. (a) Easily (b) Barely
(d) He wished that Siddharth would behave like the other (c) Suddenly (d) Carefully
princes. (e) Readily
(e) He wished that Siddharth would find the answer to all 163. Radiant
the suffering in the world. (l) Full (b) Happy
155. What incident changed Siddharth’s life forever ? (c) Burning (d) Sober
(a) His mother’s death. (e) Sickly
(b) The incident where he saw a monk free from the misery DIRECTIONS (164 - 165) : Choose the word which is most
of life. OPPOSITE in meaning to the word printed in bold as used in the
(c) The birth of his son passage.
(d) His marriage to Yashodhara.
164. Revealed to
(e) The time he met Lord Buddha.
(a) Hidden from (b) Exposed to
156. Why did Siddharth leave his home? (c) Presented to (d) Manifested from
(l) He wanted to run away from his wife and son (e) Obtained from
(b) He did not want to become heir to his father’s throne 165. Handsome

ww
(c) He was in search of a bigger kingdom
(d) He wanted to see the world
(e) He wanted to search for a way out of suffering and
(a) Dear
(c) Precious
(b) Ugly
(d) Spoilt

157. w.E
spend time in meditation.
According to Siddharth what was the reason for suffering
and unhappiness in the world?
(e) Attractive

Passage - 38
(a) The increased number of deaths
(b) The advocacy of the Middle Path asy A majority of Indians prefer to use the internet for accessing
banking and other financial services than shopping online, shows
a new survey.
(c) Human greed and selfishness

En
(d) People were unaware of the benefits of meditation.
Almost 57% of Indian respondents using the internet prefer to
bank online and use other financial services due to hassle-free

158.
(e) None of these
What according to passage is the root cause for all human
suffering? gin
access and time saving feature of online banking according to
the survey.
Checking information on products and services online comes
(A) Desire
(C) Meditation
(B) Happiness
eer
a close second at 53% while 50% shop for products online. The
fourth on the list-around 42% of respondents in India surfed
(a) Only (A)
(c) Only (B)
(e) All (A). (B) and (C)
(b) Only (B) and (C)
(d) Only (C)
ing
online to look for jobs, the survey said.
Online banking has made things much easier for the people and
it saves a lot of time.
159. Why was Siddharth called the enlightened one ?
(a) He was smarter than all the princes of his age. .ne
It has eliminated the problems associated with traditional way
of banking where one had to stand in a queue and fill up several
(b) He was the King’s son.
(c) Truth was revealed to him through meditation.
(d) He had a great ability to resist temptation
(e) He always wanted to be left alone
to protect customers against cybercrime. t
forms. Most of the banks in India have introduced customer-
friendly online banking facility with advanced security features

The easy registration process for net banking has improved


customers’ access to several banking products increased customer
160. Why did Lord Buddha advocate the Middle Path ?
loyalty, facilitated money transfer to any bank across India and
(a) He believed that it was the only way to eliminate
has helped banks-attract new customers. The Indian results closely
poverty.
track the global trends as well conducted among 19216 people
(b) He believed that it was the only way of obtaining true from 24 countries, the survey showed that banking and keeping
happiness. track of finances and searching for jobs are the main tasks of
(c) He was a staunch believer of living an extreme life. internet users around the globe.
(d) He did not have a happy life being a prince. Overall, 60% of people surveyed used the web to check their bank
(e) He was not a risk-taker. account and other financial assets in the past 90 days, making it
DIRECTIONS (161 -163) : Choose the word which is most nearly the most popular use of the internet globality, shopping was not
the SAME in meaning as the word printed in bold as used in the too far behind at 48%, the survey showed and 41% went online
passage. in search of a job in terms of country preferences, almost 90% of
161. Root respondents in Sweden use e-banking.
(a) Secondary (b) Common Online banking has also caught on in a big way in nations like
(c) Burning (d) True France, Canada, Australia, Poland, South Africa and Belgium,
(e) Main the survey showed. The Germans and British come on top for

Downloaded From : www.EasyEngineering.net


Downloaded From : www.EasyEngineering.net

364  l  Reading Comprehension

using online shopping with 74% of respondents in both countries 173. Which of the following is/are true in the context of the
having bought something online in the past three months. They passage?
are followed by 68% of respondents in Sweden. 65% in US and A. More than 50% Indians using internet prefer online
62% in South Korea. [IBPS Clerk 2013] banking.
166. If the given sentences were to be arranged in their order of B. Many banks in India have introduced facilities to suit
their popularity (from most popular to least popular), which the needs of customers (customer-friendly)
one of the following would represent the correct sequences C. More people shop online in South Korea as compared
as given in the passage? to those in Sweden.
A. Use internet to gain information about products and (a) Only B (b) B and C both
services. (c) A and B both (d) A and C both
B. Use internet to search for jobs. (e) Only C
C. Use internet for online banking. 174. Which of the following is most nearly the same in meaning
(a) B,A, C (b) C, B, A as the word ‘eliminated’ as used in the passage?
(c) C, A, B (d) A. B, C
(a) taken (b) introduced
(e) A, C, B
(c) begun (d) removed
167. Which of the following is not true in the context of the
(e) cancelled
passage?

ww
(a) Internet users across the globe are mainly interested in
looking for jobs and keeping track of finances
(b) Germany and Great Britain are the top countries where
175. According to the passage, globally, the most popular use of
internet is
(a) looking for a job

w.E
online shopping is quite popular
(c) Many people in Sweden use e-banking to maintain
their finances
(b) checking bank accounts and maintaining financial
assets
(c) finding out information about various products

asy
(d) A majority of Indians prefer shopping online as
compared to other online activities
(e) All the given statements are true
(d) shopping online
(e) blogging and tweeting every update in one’s personal
life

more customers due to En


168. According to the passage, banks are successful in attracting
Passage - 39
A. better training to sales staff
B. opening more branches at various locations
C. easy registration process for net banking gin
Freedom has assuredly given us a new status and new opportunities.
But it also implies that we should discard selfishness, laziness and
all narrowness of outlook. Our freadom suggests toil and creation
(a) Only B
(c) A and B
(b) B and C
(d) A and C eer
of new values for old ones. We should so discipline ourselves as
to be able to discharge our new responsibilities satisfactorily. If
(e) Only C
169. How many of the Indians using the internet shop online?
(a) Between 40% and 50% (b) One-fourth of them ing
there is any one thing that needs to be stressed more, than any
other in the new set-up, it is that we should put into action our
full, capacity, each one of us in productive effort - each one of us in
(c) One-third of them
(e) All of them
(d) Half of them
.ne
his own sphere, however humble. Work, unceasing work, should
now be our watch-word. Work is wealth, and service is happiness.
170. Which of the following is most nearly the same in meaning
as the word hassle-free used in the passage?
(a) unskilled
(c) Subtle
(e) Notable
(b) Not annoying
(d) Unsaddle
will gradually disappear. t
Nothing else is. The greatest crime in India today is idleness. If
we root out idleness, all our difficulties, including even conflicts,
[SSC CHSL, 2013]
176. Anyone can free himself from the clutches of difficulties, if he
(a) eliminates narrow outlook
171. Based on the passage, what can be said about the internet (b) fulfils his responsibilities
in a nutshell? (c) discards idleness
(a) It has increased the number of cybercrimes (d) discharges his obligations
(b) It is useful only for the rich 177. What has freedom undeniably offered to the citizens of India
(c) It has been hyped for no reason
?
(d) It is more popular for online shopping than anything (a) New opportunities
else (b) New outlook
(e) It has made lives easier than before (c) New responsibilities
172. Which of the following can be appropriate title for the (d) New values
passage? 178. One thing needs to be stressed more than anything else in
(a) The growing utility of the internet this new set-up. It is that people should
(b) Internet and its drawbacks (a) discard narrowness of outlook
(c) The traditional versus modern ways of shopping (b) discipline themselves suitably
(d) Use of the internet in different countries (c) work to their full capacity
(e) Internet-The Curse (d) substitute old values with new ones

Downloaded From : www.EasyEngineering.net


Downloaded From : www.EasyEngineering.net

Reading Comprehension  l 365

179. work should be the motto of our citizens. Passage - 41


(a) Resourceful (b) Incessant
(c) Productive (d) Ingenious Violence has played a great part in the would’s history. It is today
180. Nothing else can give us joy except playing an equally important part and probable it will continue
(a) service (b) idleness to do so for a considerable time. It is impossibe to ignore the
(c) wealth (d) freedom importance of violence in the past and present. To do so is to ignore
life. Yet violence is undoubtedly bad and brings an unending trail
Passage - 40 of evil consequences with it. And worse even than violence are
It is not luck but labour that makes good luck, says an the motives of hatred, cruelty, revenge and pun-ishment which
American author, is ever waiting for something to turn up; labour very often accompany violence. Indeed, violence is bad, not
with keen eyes and strong will power turns up something. Luck intrinsically, but because of these motives that go with it. There
lies in bed and wishes the postman would bring him news of a
can be violence without these motives; there can be violence
legacy, labour turns out at six and with busy pen and ringing
hammer lays the foundation of competence. Luck whines, labour for a good object as well as for an evil object. But it is extremely
watches, luck relies upon chance, .labour upon character. Luck difficult to separate violence from these motives, and therefore,
slips downwards to self-indul-gence ; labour strides upwards and it is desirable to avoid as far as possible.
aspires to independence. The conviction, therefore, is extending In avoiding it. however someone can not accept a negative attitude

ww
that diligence is the mother of good luck. In other words, a man’s
success in life will be proportionate to his offorts, to his industry,
to his attention to small things. [SSC Sub Insp. 2013]
of sub-mitting to bad and far greater evils. Submission to violence
or the acceptance of an unjust regime based on violence is against
the spirit of non-violence. The non-violent method, in order to

w.E
181. Which one of the following words in the passage indicates
that the writer does not ultimately reject the element of luck?
(a) ‘Luck whines’
justify itself, must be dynamic and capable of changing such a
regime of social order. [SSC Sub Insp. 2013]
186. The word ‘dynamic’ in the concluding line of the passage

asy
(b) ‘Diligence is the mother of good luck’
(c) Luck ..... wishes the postman would bring him news’.
(d) Luck ..... .is ever waiting.’
means:
(a) active

182. Which pair of words means the same thing?


En (b) energetic
(c) capable of change and progress
(a) Labour and industry
(b) Industry and legacy
(c) Diligence and legacy
gin (d) all of the above
187. Which of the following statements is incorrect?
(d) Legacy and labour
183. Which one of the following statements sums up the meaning eer
(a) Only violence can be used against violence.
(b) Violence is not inherenty ill
of the passage?
(a) Luck waits and complains without working while labour ing
(c) Violence is a historically accepted fact.
(d) Violence can not be ignored.
achieves success although it complains.
(b) Luck is self indulgent, but labour is selfless.
(a) practice .ne
188. ‘Violence without these motives’ is possible only in :
(b) reality
(c) Luck often ends in defeat but labour produces luck.
(d) Luck waits without exertion, but labour exerts without
waiting.
184. ______Labour turns out at six and with busy pen and ringing
(c) dream (d) theory
t
189. ‘Indeed, violence is bad, not intrinsically, but because of these
motives that go with it’.
hammer lays the foundation of competence. What does this
This suggests:
statement mean?
(a) Labour lays the foundation of the building. (a) Violence is basically good.
(b) The writer and the labourer are the true eyes of the (b) Violence is bad only when it is associated with certain
society. motives.
(c) There is no worker who works so hard as the labourer (c) Violence is bad because the people who exercise it are
who begins his day at six in the morning. bad.
(d) Hardwork of all kinds makes people different. (d) Violence is basically bad.
185. Which one of the statements is true about the passage? 190. Non-violence, according to the writer, means:
(a) Success depends only on hardluck. (a) violence without the evil motives.
(b) Expectation of good luck always meets with (b) giving in to the tyranny of the powerful
disappoinment.
(c) accepting violence as a fact of life.
(c) Success is exactly proportionate to hard work.
(d) none of the above.
(d) Luck is neccessary for success.

Downloaded From : www.EasyEngineering.net


Downloaded From : www.EasyEngineering.net

366  l  Reading Comprehension

aspect of maturing is developing the ability to take increasing


Passage - 42 responsibility of our own lives to become increasingly self-
Street theatre in India is a well established ancient art form. directed. The third reason is that many of the new developments in
Despite the proliferation of modern means of entertainment and education put a heavy responsibility on the learners to take a good
communication, street theatre continues to flourish in India. deal of initiative in their own learning. Too meet tthe challenges
Street theatre as a channel of communication has for centuries in today’s instructive environment, self-directed learning is most
been propagating reforms by highlighting social, economic essential. [SSC Sub Insp. 2014]
and political issues present in the society. Unlike in the olden 196. In self-directed learning, an individual
days, its performance is no longer restricted to villages or small (a) Takes initiative with or without the help of others
localities of the city. Today small groups of performers including (b) Is passive and waits for directions
students, would stage performances to mobilize public opinion (c) Is helpless and dependent
or to help create or raise awareness over a particular issue of (d) Takes initiative, without an objective
public importance. Themes on substance abuse, AIDS awareness, 197. There is need for self-directed learning because
and domestic violence are some of the areas highlighted by (a) it is less challenging
contemporary street theatre troupe. Unlike in regular drama street (b) it helps people to learn more things and learn better
drama employ very little props and images. The human body (c) it is a more cost-effective method
becomes the main tool in which choreography, mime, dialogues, (d) it is a modern method of learning

ww
songs and slogans are extensively used.
Street theatre is one of the most intimate media. Its appeal is to
the emotions leading to quick psychological impact on audiences.
198. Which word best describes self-directed learning ?
(a) Active learning (b) Passive learning
(c) Compulsory learning (d) Repulsive learning

w.E
By being local and live they also are able to establish not only
direct contact with the audience but by being cost-effective and
flexible they are popular among all age groups.
199. The modern environment according to the author is
(a) Restrictive
(c) Less developed
(b) Instructive
(d) Impracticable

asy
[SSC Sub Insp. 2014]
191. Modern means of entertainment and communication
200. The synonym of the word “diagnosing” is
(a) Searching
(c) Identifying
(b) Examining
(d) Complying
_______ street theatre.
(a) does affect
(c) helps popularis
(b) does not affect
(d) helps establish En Passage - 44
192. In the olden days street theatre _______ to villages or small
localities of the city. gin
Long ago in Mongolia there lived an emperor who feared growing
old. One day, he saw an old man in the street. Upset at being
(a) was restricted
(c) was opened
(b) was not restricted
(d) was entertained
193. Street theatre usually _______ with issues of public eer
reminded that someday, he too, would age, he ordered all the old
people to leave his land.
One day, a violent storm swept the kingdom. Nothing was safe
importance.
(a) is distanced (b) is performed ing
from its fury. It roared into the palace and blew away the emperor’s
belongings, including his priceless golden pitcher. When the
storm ended, the emperor ordered that the pitcher be found and
(c) deals (d) does not deal
194. Street theatre is _______ to stage.
(a) nothing (b) costly
brought back to him.
.ne
People went in search of the pitcher. They saw it in a lake nearby.
(c) reasonable

(a) intimate
(c) mystical
(d) affordable
195. Street theatre creates an/a _______ impact on audiences.
(b) emotional
(d) physical
glittering and just below the water’s surface. t
But no matter who tried, no one could get a grip on the pitcher.
All they got was a handful of water. Yet it could be plainly seen,

[SSC Multi tasking 2013]

201. The people saw the golden pitcher


Passage - 43 (a) in a river nearby
Self directed learning, in its broadest meaning, describes (b) in a lake nearby
a proess in which individuals take the initiative with or without (c) in a pit nearby
the help of others, in diagnosing their learning needs formulating (d) inside the palace
learning goals, identifying resources for learning, choosing 202. The emperor’s orders were that all the
and implementing learning strategies and evaluating learning (a) children should leave his land
outcomes. Thus it is important to attain new knowledge easily (b) old men should leave his land
and skillfully for the rest of his or her life. (c) old men should live in his land
What is the need for self directed learning? One reason is that (d) young men should stay in his land
there is convincing evidence that people, who take the initiative 203. What did the people who went to bring the pitcher get?
in learning, learn more things and learn better than people who (a) Nothing at all
sit at the feet of teachers passively waiting to be taught. The (c) A handful of water
second reason is that self-directed learning is more in tune with (c) A handful of air
our natural processes of psychological development; an essential (d) The pitcher’s handle

Downloaded From : www.EasyEngineering.net


Downloaded From : www.EasyEngineering.net

Reading Comprehension  l 367

204. The emperor feared cook. Come, eat your lunch.” She served the chapattis, vegetable
(a) getting old (b) getting young and kheer to all of them. The four bullies were stunned when they
(c) getting weak (d) getting ill saw the rabbit sitting in the corner. They told Jairam, “We will
205. The emperor was upset to see the old man because buy your rabbit.” Jairam pretended to think and said, “It is very
(a) it reminded him of his grandfather precious to me.” The four bullies immediately offered him a higher
(b) it reminded him that he might fall ill price. Jairam showed some reluctance. The moment he agreed to
(c) it reminded him that he would grow old too. sell the rabbit they paid him the money and left with the rabbit
(d) it reminded him that he had to colour his hair. immediately. The four bullies decided to test the rabbit’s abilities.
They had been blackmailing a landlord for money. So they told
Passage - 45 the rabbit, “Go and tell the landlord to bring us the money within
Sheela and Jairam were a poor old couple. Their only possession ten minutes.” The rabbit hopped off. They waited for an hour but
was one cow. Once, Jairam fell very sick. Soon all their money was the landlord did not come with the money. They marched to his
used up in buying medicines and they realised that they would house and yelled, “Give us the money and our rabbit.” The landlord
have to sell their cow in order to bear the rest of the expenses. had been waiting for a chance to teach these bullies a lesson. He
Sheela decided to go to the market and sell the cow. She set off,
ordered his strongest bodyguard to give them a good thrashing.
leading the cow by a rope. One the way, she met four young men.
Bleeding and bruised they went back to Jairam’s house and said,
They were the local bullies who enjoyed teasing and tormenting

ww
old people. When they saw old Sheela with her cow, they decided
to play a trick. On of them sneaked up behind her, untied the cow
and tied a goat in its place. Sheela had been walking immersed in
“You fooled us. Return our money at once.” Jairam simply smiled
and said, “The money has disappeared! There is magic in the air.”

w.E
thought, worried about Jairam. Suddenly she heard a goat bleat
behind her. She turned around and was surprised to see that her
cow had vanished. She was leading a goat to the market. The four
[SBI Clerk 2012]
206. Why did Sheela and Jairam decide to sell the cow ?
(a) They had spent all their money on Jairam’s sickness and

asy
bullies came up to her and said, “There is magic in the air these
days. See, it turned your cow into a goat. “Poor Sheela walked on
with the goat. After a little while, the boys untied the goat and tied
needed more money
(b) Sheela wanted to buy a rooster and they needed money
for that

En
a rooster in its place. The rooster crowed and Sheela was surprised
again. The goat had turned into a rooster in its place. The rooster
(c) They were fed up of the four bullies and wanted to teach
them a lesson
crowed and Sheela was surprised again. The goat had turned into a
rooster! The four bullies shouted, “Magic in the air, Grandma.” She
resumed walking with the rooster in tow. After a few minutes the gin (d) They wanted to invite the four bullies for lunch and
needed money for buying the ingredients
boys crept up again, untied the rooster and tied a log of wood in its
place. A few moments later Sheela realised that she was dragging
a log of wood with a rope. Again, the bullies shouted, “Magic in eer
(e) They owned money to the landlord
207. Why was Jairam reluctant to sell the rabbit?
(a) He only pretended to be reluctant in order to fool the
the air, Grandma.” After a few minutes, the boys untied the log of
wood too and ran away with it. When Sheela finally reached the ing
bullies
(b) He was fond of the little rabbit and did not want to sell
market, she found that she had nothing but a rope in her hand. She
came back home dejected as she had lost the cow. She told Jairam
the whole story. He immediately understood what had happened.
it to the bullies
.ne
(c) He wanted to gift the rabbits to the landlord
(d) He knew that the bullies would not take good care of
“Make chapatti, vegetable and kheer for lunch tomorrow,” he said.
“Cook for atleast four people. I will come home with some guests.
As soon as they come you must say, ‘I cooked what the rabbit told
me. Come, eat your lunch.’ Leave everything else to me,” Jairam
the rabbit
t
(e) He wanted to sell the rabbit in the market and get a
better price for it
reassured her. The next morning Jairam went and borrowed two 208. Which of the following may be an appropriate title to the
identical rabbits from a friend. He left one at home, tied the other passage?
one with a string and started walking towards the market with it. (a) Magic in the air (b)  Never steal a cow
On the way he too met the four bullies. ‘Hey Grandfather !’ they (c) The strongest bodyguard (d)  The smart bullies
yelled, “your wife’s cow vanished yesterday. Where are you taking (e) The obedient rabbit
this rabbit now ?” Jairam sighed sadly and said, “This rabbit is like 209. What did Sheela do after reaching the market empty handed?
my son. It obeys everything I say. But now I am sick and we need (a) Determined to earn money, she managed to sell the
money, so I am going to sell it in the market.” The four bullies were rope that she was left with
surprised when they heard this. “Does it really understand what (b) She returned home dejected and narrated the whole
you say, Grandfather ?” they asked. Jairam replied, “Of course it incident to her husband
does. Here, watch me.” Jairam turned towards the rabbit and said, (c) She bought two rabbits and devised a plan to get back
“Hop home and tell Sheela to make chapattis, vegetable and kheer at the bullies
for four people.” Then he untied the string and let the rabbit hop (d) She went to the landlord and complained about the
away. He turned towards the four boys and said, “Come home bullies
and have lunch with me.” When they reached his house his wife (e) She went home and cooked lunch for her husband and
welcomed them and said, “I cooked what the rabbit asked me to herself

Downloaded From : www.EasyEngineering.net


Downloaded From : www.EasyEngineering.net

368  l  Reading Comprehension

210. What did the four bullies tie to Sheela’s rope immediately 217. Possession :
after untrying the cow? (a) Control (b) Power
(a) A log of wood
(c) Custody (d) Keeping
(b) A rooster
(c) A rabbit (e) Belonging
(d) A goat 218. Vanished :
(e) Not mentioned in the passage (a) Gone missing (b) Was found
211. Why were the four bullies surprised when they were talking (c) Was killed (d) Was left behind
to Jairam? (e) Had exchanged
(a) They knew that Jairam was sick and had not expected
to see him DIRECTIONS (Qs. 219-220) : Choose the word which is
(b) They had not expected Jairam to invite them for lunch most opposite in meaning to the word printed in bold as
(c) They could not believe that the rabbit could understand used in the passage. [SBI Clerk 2012]
and obey Jairam 219. AGREED :
(d) They knew that Jairam did not own a rabbit and were (a) Decided (b) Arranged
surprised to see him with one
(c) Accepted (d) Declined
(e) They could not believe that Jairam knew the rabbit’s

ww
language
212. What did the four bullies see when they reached Jairam’s
house?
(e) Fixed
220. DEJECTED :
(a) Happy (b) Hurried

w.E
(a) They saw that two rabbits were sitting in the corner
(b) They saw that Sheela had not prepared any lunch
(c) They saw the cow they had stolen from Sheela on the
(c) Crestfallen
(e) Angrily
(d) Slowly

previous day

asy
(d) They saw the log of wood they had tied to Sheela’s rope
(e) They saw that the same rabbit was sitting in the corner
Passage - 46
Long ago, a Brahmin called Haridatta lived in a little
village. He was a farmer but the piece of land he cultivated
they occurred in the passage.
En
213. Arrange the following incidents in a chronological order as
provided him with so little to survive on that he was very
poor.
A. Jairam and Sheela needed money
B. The four bullies sent the rabbit to the landlord
C. Sheela cooked chapattis, vegetable and kheer gin
One day, unable to stand the heat of the summer sun, he
went to rest for a while under a big tree on his land. Before

D. The four bullies tied a log of wood to the rope


(a) ADBC (b) ACDB eer
he could stretch out on the ground, he saw a huge black
cobra slithering out of an anthill nearby. The snake then
spread his hood and swayed gracefully from side to side.
(c) ADCB
(e) DACB
(d) DABC
ing
Haridatta was astonished to see this and he thoughts.
“This cobra must really be the god of this land. I have
214. Which of the following is true according to the passage ?
(a) The four boys liked to help old people
(b) Jairam asked Sheela to cook lunch for atleast four people .ne
never seen or worshipped him before, which is probably
why I am not able to get anything from the land. From
this day onwards, I will worship him”. He hurried back to
(c) The rabbit could understand Jairam
(d) Jairam could not trick the four boys
(e) None is true
his home at once and returned with a glass full of milk.

“O ruler of the land. I did not know you were living in


t
He poured it into a bowl and turning to the anthill said,

this anthill. That is why I have not paid my tribute to you.


215. What did the landlord do when the four bullies went to his
Please accept my apologies for this omission and accept
house and yelled ?
this humble offering.” He then placed the bowl of milk at
(a) He got scared and immediately gave them the money
the entrance of the anthill and left the place.
(b) He ran away and hid in the market The next day when the Brahmin arrived to work on his
(c) He complained to Jairam about this land before the sun rose, he found a gold coin in the bowl
(d) He ordered his strongest bodyguard to thrash them he had left at the anthill. He was very happy indeed and
(e) He offered them lunch in order to calm them down from that day on, he made it a practice to offer the cobra
milk in a bowl each day. The next morning he would collect
DIRECTIONS (Qs. 216-218) : Choose the word/group of words
a gold coin and leave.
which is most similar in the meaning to the word/group of words
One day Haridatta had to go to a neighbouring village
printed in bold as used in the passage.
on business. He asked his son to go to the anthill as usual
216. Thrashing : and leave a bowl of milk for the cobra. The son did as he
(a) Garbage (b) Beating was told, but when he went to the same spot the next day
(c) Shouting (d) Warning and collected the gold coin he thought. “This anthill must
(e) Rejection be full of gold. If I kill the cobra. I can collect all the gold

Downloaded From : www.EasyEngineering.net


Downloaded From : www.EasyEngineering.net

Reading Comprehension  l 369

in an instant, instead of having to waste my time coming hero (d) Haridatta And His Son
every day.” He then struck the cobra with a big stick. The cobra (e) The Mourning Father
deftly dodged the blow but bit Haridatta’s son with his poisonous 225. Why did Haridatta’s son try to kill the cobra?
fangs. The boy soon died. When Haridatta returned to his village (a) The cobra had angered him through his behaviour.
the next day, he heard how his son had met his death. He realised (b) The cobra was known to be poisonous and had killed
at once that his son’s greed would probably have caused him to several men in the village by bitting them.
attack the cobra.
(c) He believed that the anthill in which the cobra lived
The Brahmin went to the anthill the day after his son’s cremation
contained all the gold coins.
and offered milk to the cobra as usual. This time, the cobra did
(d) He thought the cobra would bite him if he went near
not even come out of his hole. Instead, he called out to Haridatta.
“You have come here for gold, forgetting that you have just lost a it.
precious son and that you are in mourning. The reason for this is (e) He did not like the fact that his father was wasting his
pure greed. From today, there is no meaning to our relationship. I time by feeding the cobra milk every day.
am going to give you a diamond as a final gift. But please don’t ever DIRECTIONS (Qs. 226-228): Choose the word / group of words
come back again.” He slithered away as the Brahmin watched. which is most similar in meaning to the word / group of words
[SBI Clerk 2012] printed in bold as used in the passage.
221. Which of the following is TRUE according to the story?

ww
(a) Haridatta had asked his son to give milk to the cobra
because he himself was tired of doing it.
226. HUMBLE
(a) rich
(c) meek
(b) waste
(d) modest

w.E
(b) The cobra gave Haridatta a diamond every day.
(c) Haridatta’s son was greedy
(d) Haridatta had no idea as to why the cobra had killed
his son.
(e) low
227. ASTONISHED
(a) petrified (b) upset
(e) All are true
asy
222. What was the cobra’s final gift to the Brahmin?
(c) dazed
(e) surprised
228. STRETCH OUT
(d) fearful

(a) Venom
(c) A bite
(e) None of these
(b) A diamond
(d) A gold coin
En (a) lie down
(c) fall
(b) exercise
(d) elongate
223. What did Haridatta think on seeing the cobra for the first
time? gin
(e) extend out
DIRECTIONS (Qs. 229-230): Choose the word / group of words
(a) The cobra was divine and he would get gold coins from
it if worshipped.
(b) The cobra was the god of his land and had to be eer
which is most opposite in meaning in the word / group of words
printed in bold as used in the passage.
worshipped.
(c) He was fearful of the cobra and run away from the spot
229. DEFTLY
(a) nimbly
ing
(c) skilfully
(b) artistically
(d) shallowly
(d) The cobra was responsible for his poverty.
(e) None of these
224. Which of the following is possibly the most appropriate title
(e) sluggishly
230. IN AN INSTANT .ne
for the story?
(a) The Cobra and the Brahmin
(b) The Foolish Brahmin
(c) The Brave Cobra
(a) in lieu of
(c) at once
(e) speedily
(b) quickly
(d) one each day t

Downloaded From : www.EasyEngineering.net


Downloaded From : www.EasyEngineering.net

370  l  Reading Comprehension

LEVEL-II
DIRECTIONS (Qs. 1-337): Read the following passages and (c) Some individuals in any given population show
answer the questions that follow. resistance after the application of pesticides.
(d) None of the statements (a), (b) and (c) given above is
Passage- 1 correct.
2. With reference to the passage, consider the following statements:
Chemical pesticides lose their role in sustainable agriculture if 1. Use of chemical pesticides has become imperative in all
the pests evolve resistance. The evolution of pesticide resistance the poor countries of the world.
is simply natural selection in action. It is almost certain to occur 2. Chemical pesticides should not have any role in
when vast numbers of a genetically variable population are killed. sustainable agriculture.
One or a few individuals may be unusually resistant (perhaps 3. One pest can develop resistance to many pesticides.
because they possess an enzyme that can detoxify the pesticide). Which of the statements given above is/are correct?
If the pesticide is applied repeatedly, each successive generation of (a) 1 and 2 only (b) 3 only
the pest will contain a larger proportion of resistant individuals. (c) 1 and 3 only (d) 1, 2 and 3
Pests typically have a high intrinsic rate of reproduction, and so 3. Though the problems associated with the use of chemical
pesticides is known for a long time, their widespread use has

ww
a few individuals in one generation may give rise to hundreds or
thousands in the next, and resistance spreads very rapidly in a
population.
not waned. Why?
(a) Alternatives to chemical pesticides do not exist at all.
(b) New pesticides are not invented at all.

w.E
This problem was often ignored in the past, even though the
first case of DDT (dichlorodiphenyltrichloroethane) resistance
was reported early as 1946. There is exponential increase in the
(c) Pesticides are biodegradable.
(d) None of the statements (a), (b) and (c) given above is
correct.

asy
numbers of invertebrates that have evolved resistance and in
the number of pesticides against which resistance has evolved.
Resistance has been recorded in every family of arthropod pests
4. How do pesticides act as agents for the selection of resistant
individuals in any pest population?
1. It is possible that in a pest population the individuals

En
(including dipterans such as mosquitoes and house flies, as well
as beetles, moths, wasps, fleas, lice and mites) as well as in weeds
will behave differently due to their genetic makeup.
2. Pests do possess the ability to detoxify the pesticides.
3. Evolution of pesticide resistance is equally distributed in
and plant pathogens. Take the Alabama leaf-worm, a moth pest
of cotton, as an example. It has developed resistance in one or
more regions of the world to aldrin, DDT, dieldrin, endrin,
gin pest population.
Which of the statements given above is/are correct?

lindane and toxaphene.


If chemical pesticides brought nothing but, problems, — if 5. eer
(a) 1 only
(c) 3 only
(b) 1 and 2 only
(d) 1, 2 and 3
Why is the use of chemical pesticides generally justified by
their use was intrinsically and acutely unsustainable — then
they would already have fallen out of widespread use. This has ing
giving the examples of poor and developing countries?
1. Development countries can afford to do away with use
of pesticides by adapting to organic farming, but it is
not happened. Instead, their rate of production has increased
rapidly. The ratio of cost to benefit for the individual agricultural
producer has remained in favour of pesticide use. In the USA, chemical pesticides. .ne
imperative for poor and developing countries to use

insecticides have been estimated to benefit the agricultural


products to the tune of around $5 for every $1 spent.
Moreover, in many poorer countries, the prospect of imminent
mass starvation, or of an epidemic disease, are so frightening
food problem. t
2. In poor and developing countries, the pesticide addresses
the problem of epidemic diseases of crops and eases the

3. The social and health costs of pesticide use are generally


ignored in poor and developing countries.
that the social and health costs of using pesticides have to be Which of the statements given above is/are correct?
ignored. In general the use of pesticides is justified by objective (a) 1 only (b) 1 and 2 only
measures such as ‘lives saved’, ‘economic efficiency of food (c) 2 only (d) 1, 2 and 3
production’ and ‘total food produced’. In these very fundamental 6. What does the passage imply?
senses, their use may be described as sustainable. In practice, (a) Alternative options to chemical pesticides should be
sustainability depends on continually developing new pesticides promoted.
that keep at least one step ahead of the pests – pesticides that are (b) Too much use of chemicals is not good for the ecosystem.
less persistent, biodegradable and more accurately targeted all (c) There is no scope for the improvement of pesticides and
the pests. making their use sustainable.
1. “The evolution of pesticide resistance is natural selection in (d) Both the statements (a) and (b) above are correct.
action.” What does it actually imply?
(a) It is very natural for many organisms to have pesticide
Passage- 2
resistance. Today’s developing economices use much less energy per capita
(b) Pesticide resistance among organisms is a universal than developed countries such as the United States did at similar
phenomenon. incomes, showing the potential for lower-carbon growth.

Downloaded From : www.EasyEngineering.net


Downloaded From : www.EasyEngineering.net

Reading Comprehension  l 371

Adaptation and mitigation need to be integrated into a climate- 9. Which of the following conditions is/are necessary for
smart development strategy that increases resilience, reduces the sustainable economic growth?
threat of further global warming, and improves development 1. Spreading of economic prosperity more.
outcomes. Adaptation and mitigation measures can advance 2. Popularising/spreading of adaptive technologies widely.
development, and prosperity can raise incomes and foster 3. Investing on research in adaptation and mitigation
better institutions. A healthier population living in better-built technologies.
houses and with access to bank loans and social security is better Select the correct answer using the codes given below:
equipped to deal with a changing climate and its consequences. (a) 1 only (b) 2 and 3 only
Advancing robust, resilient development policies that promote (c) 1 and 3 only (d) 1, 2 and 3
adaptation is needed today because changes in the climate, 10. Which of the following inferences can be made from the
already begun, will increase even in the short term. passage?
The spread of economic prosperity has always been intertwined 1. Rainfed crops should not be cultivated in irrigated areas.
with adaptation to changing ecological conditions. But as growth 2. Farming under water-deficient areas should not be a
has altered the environment and as environmental change part of development strategy.
has accelerated, sustaining growth and adaptability demands Select the correct answer using the codes given below:
greater capacity to understand our environment, generate new (a) 1 only (b) 2 only
adaptive technologies and practices, and diffuse them widely.
(c) Both 1 and 2 (d) Neither 1 nor 2

ww
As economic historians have explained, much of humankind’s
creative potential has been directed at adapting to the changing
world. But adaptation cannot cope with all the impacts related
to climate change, especially as larger changes unfold in the long
11. Consider the following assumptions :
1. Sustainable economic growth demands the use of
creative potential of man.
term.
w.E
Countries cannot grow out of harm’s way fast enough to match the
changing climate. And some growth strategies, whether driven
2. Intensive agriculture can lead to ecological backlash.
3. Spread of economic prosperity can adversely affect the
ecology and environment.

asy
by the government or the market, can also add to vulnerability
— particularly if they overexploit natural resources. Under the
Soviet development plan, irrigated cotton cultivation expanded
With reference to the passage, which of the above
assumptions is/are valid?
(a) 1 only (b) 2 and 3 only

En
in water-stressed Central Asia and led to the near disappearance
of the Aral Sea, threatening the livelihoods of fishermen, herders
(c) 1 and 3 only (d) 1, 2 and 3
12. Which one of the following statements constitutes the
and farmers. And clearing mangroves — the natural coastal
buffers against storm surges — to make way for intensive farming
or housing development , increases the physical vulnerability of gin central theme of this passage?
(a) Countries with greater economic prosperity are better
equipped to deal with the consequences of climate
coastal settlements, whether in Guinea or in Louisiana.
7. Which of the following conditions of growth can add to
eer
change.
(b) Adaptation and mitigation should be integrated with
development strategies.
vulnerability?
1. When the growth occurs due to excessive exploitation of
mineral resources and forests. ing
(c) Rapid economic growth should not be pursued by both
developed and developing economies.
2. When the growth brings about a change in humankind’s
creative potential.
3. When the growth is envisaged only for providing houses .ne
(d) Some countries resort to overexploitation of natural
resources for the sake of rapid development.

and social security to the people.


4. When the growth occurs due to emphasis on farming
only.
Select the correct answer using the codes given below:
Passage- 3
t
Invasions of exotic species into new geographic areas sometimes
occur naturally and without human agency. However, human
actions have increased this trickle to a flood. Human-caused
(a) 1 only (b) 2, 3 and 4 only introductions may occur either accidentally as a consequence
(c) 1 and 4 only (d) 1, 2, 3 and 4 of human transport, or intentionally but illegally to serve
8. What does low-carbon growth imply in the present context? some private purpose or legitimately to procure some hoped-
1. More emphasis on the use of renewable sources of for public benefit by bringing a pest under control, producing
energy. new agricultural products or providing novel recreational
2. Less emphasis on manufacturing sector and more opportunities. Many introduced species are assimilated into
emphasis on agriculture sector. communities without much obvious effect. However, some have
3. Switching over from monoculture practices to mixed been responsible for dramatic changes to native species and
farming. natural communities. For example, the accidental introduction
4. Less demand for goods and services. of the brown tree snake Boiga irregularis into Guam, an island
in the Pacific, has through nest predation reduced 10 endemic
Select the correct answer using the codes given below:
forest bird species to the point of extinction.
(a) 1 only
(b) 2, 3 and 4 only One of the major reasons for the world’s great biodiversity is
(c) 1 and 4 only the occurrence of centers of endemism so that similar habitats
(d) None of the above implies low-carbon growth in different parts of the world are occupied by different groups

Downloaded From : www.EasyEngineering.net


Downloaded From : www.EasyEngineering.net

372  l  Reading Comprehension

of species that happen to have evolved there. If every species 17. What can be the impact of invasion of exotic species on an
naturally had access to everywhere on the globe, we might ecosystem?
expect a relatively small number of successful species to become 1. Erosion of endemic species.
dominant in each biome. The extent to which this homogenization 2. Change in the species composition of the community of
can happen naturally is restricted by the limited powers of the ecosystem.
dispersal of most species in the face of the physical barriers Select the correct answer using the codes given below:
that exist to dispersal. By virtue of the transport opportunities (a) 1 only (b) 2 only
offered by humans, these barriers have been breached by an ever- (c) Both 1 and 2 (d) Neither 1 nor 2
increasing number of exotic species. The effects of introductions
have been to convert a hugely diverse range of local community Passage- 4
compositions into something much more homogeneous. Most champions of democracy have been rather reticent in
It would be wrong, however, to conclude that introducing species suggesting that democracy would itself promote development
to a region will inevitably cause a decline in species richness there. and enhancement of social welfare–they have tended to see
For example, there are numerous species of plants, invertebrates them as good but distinctly separate and largely independent
and vertebrates found in continental Europe but absent from the goals. The detractors of democracy, on the other hand, seemed
British Isles (many because they have so far failed to recolonize to have been quite willing to express their diagnosis of what they
see as serious tensions between democracy and development.

ww
after the last glaciations). Their introduction would be likely to
augment British biodiversity. The significant detrimental effect
noted above arises where aggressive species provide a novel
The theorists of the practical split — ‘‘Make up your mind : do
you want democracy, or instead, do you want development ?’’
— often came, at least to start with, from East Asian countries,

w.E
challenge to endemic biotas ill-equipped to deal with them.
13. With reference to the passage, which of the following
statements is correct?
and their voice grew in influence as several of these countries
were immensely successful — through the 1970s and 1980s and
even later — in promoting economic growth without pursuing

asy
(a) Introduction of exotic species into new geographical
areas always leads to reduced biodiversity.
(b) Exotic species introduced by man into new areas have
democracy.
To deal with these issues we have to pay particular attention
to both the content of what can be called development and to
always greatly altered the native ecosystems.
(c) Man is the only reason to convert a hugely diverse En the interpretation of democracy (in particular to the respective
roles of voting and of public reasoning). The assessment of
range of local community compositions into more
homogeneous ones.
(d) None of the statements (a), (b) and (c) is correct in this gin
development cannot be divorced from the lives that people can
lead and the real freedom that they enjoy. Development can
scarcely be seen merely in terms of enhancement of inanimate
context.
14. Why does man introduce exotic species into new eer
objects of convenience, such as a rise in the GNP (or in personal
incomes), or industrialization – important as they may be as
geographical areas?
1. To breed exotic species with local varieties.
2. To increase agricultural productivity. ing
means to the real ends. Their value must depend on what they
do to the lives and freedom of the people involved, which must
be central to the idea of development.
3. For beautification and landscaping.
Which of the above statements is/are correct? .ne
If development is understood in a broader way, with a
focus on human lives, then it becomes immediately clear that
(a) 1 only
(c) 1 and 3 only
(b) 2 and 3 only
(d) 1, 2 and 3
15. How is homogenization prevented under natural conditions?
(a) Evolution of groups of species specific to local habitats.
the relation between development and democracy has to be

t
seen partly in terms of their constitutive connection, rather than
only through their external links. Even though the question
has often been asked whether political freedom is ‘‘conducive
(b) Presence of oceans and mountain ranges. to development’’, we must not miss the crucial recognition
that political liberties and democratic rights are among the
(c) Strong adaptation of groups of species to local physical
“constituent components” of development. Their relevance for
and climatic conditions.
development does not have to be established indirectly through
(d) All the statements (a), (b) and (c) given above are correct
their contribution to be growth of GNP.
in this context. 18. According to the passage, why is a serious tension perceived
16. How have the human beings influenced the biodiversity? between democracy and development by the detractors of
1. By smuggling live organisms. democracy?
2. By building highways. (a) Democracy and development are distinct and separate
3. By making ecosystems sensitive so that new species are goals.
not allowed. (b) Economic growth can be promoted successfully without
4. By ensuring that new species do not have major impact pursuing a democratic system of governance.
on local species. (c) Non-democratic regimes deliver economic growth
Which of the statements given above are correct? faster and far more successfully than democratic ones.
(a) 1 and 2 (b) 2 and 3 (d) All the statements (a), (b) and (c) given above are correct
(c) 1 and 3 (d) 2 and 4 in this context.

Downloaded From : www.EasyEngineering.net


Downloaded From : www.EasyEngineering.net

Reading Comprehension  l 373

19. According to the passage, what should be the ultimate 2. Foreign companies establish joint ventures with
assessment/aim/view of development? domestic companies.
(a) Rise in the per capita income and industrial growth 3. Affiliates in a particular market/sector lose their
rates. independence as their parent companies overseas merge.
(b) Improvement in the Human Development Index and 4. Foreign companies lower the cost of their products as
GNP. compared to that of products of domestic companies.
(c) Rise in the savings and consumption trends.
Which of the statements given above are correct?
(d) Extent of real freedom that citizens enjoy.
20. What does a ‘‘constitutive’’ connection between democracy (a) 1 and 2 only (b) 2 and 3 only
and development imply? (c) 1, 2 and 3 only (d) 1, 2, 3 and 4
(a) The relation between them has to be seen through 23. What is the inference from this passage?
external links. (a) Foreign investors and multinational companies always
(b) Political and civil rights only can lead to economic dominate the domestic market.
development. (b) It is not in the best interests of the domestic economy to
(c) Political liberties and democratic rights are essential allow mergers of companies.
elements of development. (c) With competition law, it is easy to ensure a level playing
(d) None of the statements (a), (b) and (c) given above is field between domestic and foreign firms.

ww
correct in this context.

Passage- 5
(d) For countries with open economy, Foreign Direct
Investment is essential for growth.

w.E
The need for Competition Law becomes more evident when
foreign direct investment (FDI) is liberalized. The impact of
FDI is not always pro-competitive. Very often FDI takes the
Passage- 6
The poor especially in market economies, need the strength

asy
form of a foreign corporation acquiring a domestic enterprise
or establishing a joint venture with one. By making such
an acquisition the foreign investor may substantially lessen
that collectivities offer for creating more economic, social
and political space for themselves, for enhancing their socio-
economic well-being and voice, and as a protection against free
competition and gain a dominant position in the relevant
En
market, thus charging higher prices. Another scenario is where
market individualism. It has been argued that a group approach
to farming, especially in the form of bottom up agricultural
the affiliates of two separate multinational companies (MNCs)
have been established in competition with one another in a gin
production collectivities, offers substantial scope for poverty
alleviation and empowering the poor as well as enhancing
particular developing economy, following the liberalization of
FDI. Subsequently, the parent companies overseas merge. With
the affiliates no longer remaining independent, competition in eer
agricultural productivity. To realize this potential, however,
the groups would need to be voluntary in nature, small in size,

the host country may be virtually eliminated and the prices of


the products may be artificially inflated. Most of these adverse ing
participative in decision making and equitable in work sharing
and benefit distribution. There are many notable examples of
such collectivities to be found in varied contexts, such as in the
consequences of mergers and acquisitions by MNCs can be
avoided if an effective competition law is in place. Also, an
.ne
transition economies. All of them bear witness to the possibility
of successful cooperation under given conditions. And although
economy that has implemented an effective competition law is in
a better position to attract FDI than one that has not. This is not
just because most MNCs are expected to be accustomed to the
operation of such a law in their home countries and know how to
t
the gender impact of the family cooperatives in the transition
economies are uncertain, the Indian examples of women-only
groups farming offer considerable potential for benefiting
women.
deal with such concerns but also that MNCs expect competition
authorities to ensure a level playing field between domestic and 24. Agricultural collectivities such as group based farming can
foreign firms. provide the rural poor
21. With reference to the passage, consider the following 1. empowerment.
statements: 2. increased agricultural productivity.
1. It is desirable that the impact of Foreign Direct 3. safeguard against exploitative markets.
Investment should be pro-competitive. 4. surplus production of agricultural commodities.
2. The entry of foreign investors invariably leads to the Select the correct answer using the codes given below:
inflated prices in domestic markets.
(a) 1, 2, 3 and 4 (b) 1, 2 and 3 only
Which of the statements given above is/are correct?
(c) 2 and 4 only (d) 1, 3 and 4 only
(a) 1 only (b) 2 only
25. What does the author imply by ‘‘gender impact’’?
(c) Both 1 and 2 (d) Neither 1 nor 2
(a) Women are doubtful participants in cooperatives.
22. According to the passage, how does a foreign investor
dominate the relevant domestic market? (b) Family cooperatives may not include women.
1. Multinational companies get accustomed to domestic (c) Women benefiting from group farming.
laws. (d) Women’s role in transition economies is highly restrictive.

Downloaded From : www.EasyEngineering.net


Downloaded From : www.EasyEngineering.net

374  l  Reading Comprehension

26. Consider the following assumptions: (c) Social change through transfer of power from upper
1. It is imperative for transition economies to have caste elites to subaltern groups.
agricultural collectivities. (d) All the statements (a), (b) and (c) given above are correct
2. Agricultural productivity can be increased by group in this context.
approach to farming.
With reference to the above passage, which of these Passage- 8
assumptions is/are valid? Asked what a business is, the typical businessman is likely to
(a) 1 only (b) 2 only answer, “An organisation to make a profit.” The typical economist
(c) Both 1 and 2 (d) Neither 1 nor 2 is likely to give the same answer. This answer is not only false, it
is irrelevant.
Passage- 7 The prevailing economic theory of the mission of business
enterprise and behaviour, the maximization of profit which
In a typical Western liberal context, deepening of democracy
is simply a complicated way of phrasing the old saw of buying
invariably leads to consolidation of ‘liberal values’. In the Indian cheap and selling dear — may adequately explain how Richard
context, democratization is translated into greater involvement Sears operated. But it cannot explain how Sears, Roebuck or any
of people not as ‘individuals’ which is a staple to liberal discourse, other business enterprise operates, or how it should operate.
but as communities or groups. Individuals are getting involved in The concept of profit maximization is. in fact, meaningless. The

ww
the public sphere not as ‘atomized’ individuals but as members
of primordial communities drawn on religious or caste identity.
Community-identity seems to be the governing force. It is not
danger in the concept of profit maximization is that it makes
profitability appear a myth.
Profit and profitability are, however, crucial for society even

w.E
therefore surprising that the so-called peripheral groups continue
to maintain their identities with reference to the social groups
(caste, religion or sect) to which they belong while getting involved
more than for the individual business. Yet profitability is not
the purpose of, but a limiting factor on business enterprise and
business activity. Profit is not the explanation, cause, or rationale

asy
in the political processes despite the fact that their political
goals remain more or less identical. By helping to articulate the
of business behaviour and business decisions, but rather the
test of their validity. If archangels instead of businessmen sat
in directors’ chairs, they would still have to be concerned with

En
political voice of the marginalized, democracy in India has led to
‘a loosening of social strictures’ and empowered the peripherals
profitability, despite their total lack of personal interest in making
profits.
to be confident of their ability to improve the socio-economic
conditions in which they are placed. This is a significant political
process that had led to a silent revolution through a meaningful ginThe root of the confusion is the mistaken belief that the motive
of a person — the so called profit motive of the businessman is an
explanation of his behaviour or his guide to right action. Whether
transfer of power from the upper caste elites to various subaltern
groups within the democratic framework of public governance. eer
there is such a thing as a profit motive at all is highly doubtful.
The idea was invented by the classical economists to explain the
27 . According to the passage, what does ‘‘deepening of
democracy’’ mean in the Western context?
(a) Consolidation of group and class identities.
ing
economic reality that their theory of static equilibrium could
not explain. There has never been any evidence for the existence
of the profit motive, and we have é long since found the true
(b) Democratization translated as greater involvement of
people. .ne
explanation of the phenomena of economic change and fig
growth which the profit motive was first put forth to explain.
(c) Democratization as greater involvement of ‘atomized’
individuals in the public sphere.
(d) None of the statements (a), (b) and (c) given above is
correct in this context.
t
It is irrelevant for an understanding of business behaviour,
profit, and profitability, whether there is a profit motive or not.
That Jim Smith is in business to make a profit concerns only him
and the Recording Angel. It does not tell us what Jim 5 Smith
does and how he performs. We do not learn anything about the
28. Greater democratization in India has not necessarily led to work of a prospector hunting for uranium in the Nevada desert
(a) the dilution of caste and communal identities in the by being told that he is trying to make his fortune. We do not learn
public sphere. anything about the work of a heart specialist by being told that he
(b) irrelevance of community identity as a governing force is trying to make a livelihood, or even that he is trying to benefit
in Indian politics. humanity. The profit motive and its offspring maximisation of
(c) marginalization of elite groups in society. profits are just as irrelevant to the function of a business, the
purpose of a business, and the job of managing a business.
(d) relative unimportance of hereditary identities over class
In fact, the concept is worse than irrelevant: it does harm. It is
identities.
a major cause of the misunderstanding of the nature of profit in
29. What is the ‘‘silent revolution’’ that has occurred in the
our society and of the deep – seated hostility to profit, which are
Indian democratic process?
among the most dangerous diseases of an industrial society. It is
(a) Irrelevance of caste and class hierarchies in political
largely responsible for the worst mistakes of public policy — in this
processes.
country as well as in Western Europe — which are squarely based
(b) Loosening of social strictures in voting behaviour and
on the failure to understand the nature, function, and purpose
patterns.
of business enterprise. And it is in large part responsible for the

Downloaded From : www.EasyEngineering.net


Downloaded From : www.EasyEngineering.net

Reading Comprehension  l 375

prevailing belief that there is an inherent contradiction between not l knew the answer. If I was putting on airs, like someone who
profit and a company’s ability to make a social contribution. even in ordinary clothes, adds a‘gaudy piece of jewellery, it’s also
Actually, a company can make a social contribution only if it is true that I admired my teacher and was desperate to cooperate.
highly profitable. Another thing I was happy to discover at school was the
To know what a business is, we have to start with its purpose. teacher’s ‘authority’. At home, in the crowded and disordered
Its purpose must lie outside of the business itself. In fact, it must Pamuk Apartments, things were never so clear; at our crowded
lie in society since business enterprise is an organ of society. table, everyone talked at the same time. Our domestic routines,
There is only one valid definition of business purpose: to create our love for one another, our conversations, meals and radio
a customer. hours; these ‘were never debated — they just happened. My
Markets are not created by God, nature, or economic forces father held little obvious authority at home, and he was often
but by businesspeople. The want a business satisfies may have absent. He never scolded my brother or me, never even raised
been felt by the customer before he or she was offered the means his eyebrows in disapproval. In later years, he would introduce
of satisfying it. Like food in a famine, it may have dominated us to his friends as ‘my two younger brothers’, and we felt he had
the customer’s life and filled all his waking moments, but it earned the right to say so. My mother was the only authority I
remained a potential want until the action of business people recognised at home. But she was hardly a distant or alien tyrant:
converted it into effective demand. Only then is there a customer her power came from my desire to be loved by her. And so – I
and a market. The want may have been unfelt by the potential was fascinated by the power my teacher wielded over her twenty-

ww
customer; no one knew that he wanted a Xerox machine or a
computer until these became available. There may have been
no want at all until business action created it — by innovation,
five pupils.
Perhaps I identified my teacher with my mother, for I had an
insatiable desire for her approval. ‘Join your arms together like

w.E
by credit. by advertising, or by salesmanship. In every case, it is
business action that creates the customer.
30. The author of this passage is of the opinion that profits and
this and sit down quietly,’ she would say, and I would press my
arms against my chest and sit patiently all through the lesson.
But gradually the novelty wore off; soon it was no longer exciting
profitability are:
asy
(a) The purpose of setting up a business
(b) The sole goal and responsibility of a businessman
to have every answer or solve an arithmetic problem ahead of
everyone else or earn the highest mark; time began to flow with
painful slowness, or stop flowing altogether.
(c) The test of validity of business existence
(d) The guiding factor for a businessman‘s actions and
En Turning away from the fat, half-witted girl who was writing
on the blackboard, who gave everyone — teachers, school
decisions
31 . This passage highlights that the theory of profit maximisation
and profit motive gin
caretakers and her classmates — the same vapid, trusting smile,
my eyes would float to the window, to the upper branches of
the chestnut tree that I could just see rising up between the
(a) Is largely responsible for the worst mistakes in public
policy
eer
apartment buildings. A crow would land on a branch. Because
I was viewing it from below, I could see the little cloud floating
behind it — as it moved, it kept changing shape: first a fox’s nose,
(b) Is a synchronised goal with a company’s ability to make
a social contribution
(c) Is the main purpose and job of managing a business ing
then a head, then a dog. I didn’t want it to stop looking like a dog,
but as it continued its journey it changed into one of the four-
(d) Was an idea not invented by classical economists
32. As stated in this passage, the purpose of a business is to
(a) Make profits (b) Increase wants .ne
legged silver sugar bowls from my grandmother’s always—locked
display case, and I’d long to be at home. Once I’d conjured up the
reassuring silence of the shadows of home, my father would step
(c) Create customers

(a) Want
(c) Demand
(d) Manage demand
33. According to the author of this passage, what comes first?
(b) Market
(d) Customer
t
out from them, as if from a dream, and off we’d go on a family
outing to the Bosphorus. Just then, a window in the apartment
building opposite would , open, a maid would shake her duster
and gaze absentmindedly at the street that I could not see from
where I was sitting. What was going down there? I’d wonder. I’d
Passage- 9 hear a horse cart rolling over the cobblestones, and a rasping
The first thing I learned at school was that some people are idiots; voice would cry out ‘Eskiciiiiiii! The maid would watch the junk
the second thing I learned was that some are even worse. I was still dealer make his way down the street before pulling her head back
too young to grasp that people of breeding were meant to affect inside and shutting the window behind her, but then, right next
innocence of this fundamental distinction. and that the same to that window, moving as fast as the first cloud but going in the
courtesy applied to any disparity that might rise out of religious. opposite direction, I’d see a second cloud. But now my attention
racial, sexual class, financial and (latterly) cultural difference. So was called back to the classroom, and seeing all the other raised
in my innocence I would raise my hand every time the teacher hands, I would eagerly raise my hand too: long before I worked
asked a question, just to make it clear I knew the answer. out from my classrnates’ responses what the teacher had asked
After some months of this, the teacher and my classmates us, I was foggily confident I had the answer.
must have been vaguely aware I was a good student, but still I It was exciting, though sometimes painful, to get to know my
felt the compulsion to raise my hand. By now the teacher seldom classmates as individuals, and to find out how different they were
called on me, preferring to give other children a chance to speak, from me. There was that sad boy who, whenever he was asked to
too. Still my hand shot up without my even willing it, whether or read out loud in Turkish class, would skip every other line; the

Downloaded From : www.EasyEngineering.net


Downloaded From : www.EasyEngineering.net

376  l  Reading Comprehension

poor boy’s mistake was as involuntary as the laughter it would


elicit from the class. In first grade, there was a girl who kept her Passage- 10
red hair in a ponytail, who sat next to me for a time. Although Not many people saw it coming. It had seemed that the time for
her bag was a slovenly jumble of half-eaten apples, simits, sesame Kaun Banega Crorepati had come and gone. This column argued
seeds, pencils and hair bands, it always smelled of dried lavender as much a few years ago, when Shah Rukh Khan took over the
around her, and that attracted me; I was also drawn to her for reigns of the show. He did well enough, but it still seemed that the
speaking so openly about the little taboos of daily life, and if I time for the genteel game of knowledge had passed. There was too
didn’t see her at the weekend, I missed her, though there was much blood in reality television, and KBC simply did not have
another girl so tiny and delicate that I was utterly entranced by enough platelets for it. It had no backbiting intrigue, it lacked a
her as well. Why did that boy keep on telling lies even knowing cast of almost-losers and missed the low-life loquaciousness of
no one was going to believe him‘? How could that girl be so other reality shows, and nothing ever needed to be beeped out on
indiscreet about the goings-on in her house? And could this other it, a sure touch that it was out of touch with the times.
girl be shedding real tears as she read that poem about Atatiirk?’ And yet, not only is KBC back, but it is back in a very real
Just as I was in the habit of looking at the fronts of cars and seeing sense not just as a TV show that gets good ratings, but as an
noses, so too did I like to scrutinise my classmates, looking for idea that connects with something deep and real in our lives.
the creatures they resembled. The boy with the pointed nose was What makes this particularly interesting is that not very much
a fox and the big one next to him was, as everyone said, a bear, has changed in the show. Its focus has shifted to smaller towns

ww
and the one with the thick hair was a hedgehog... I remember
a Jewish girl called Mari telling us all about Passover — there
andan ‘aadmi’ more ‘aam’, and the prize money has gone up over
the years, but these are minor adjustments, not major departures.
The format is pretty much the same and the return of Amitabh

w.E
were days when no one in her grandmother’s house was allowed
to touch the light switches. Another girl reported that one
evening, when she was in’her room, she turned around so fast she
glimpsed the shadow of an angel — a fearsome story that stayed
Bachchan restores to the show both the gravitas and the empathy
that has been its hallmark.
Perhaps KBC works because it reconciles many competing

asy
with me. There was a girl with very long legs who wore very long
socks and always looked as if she was about to cry; her father was
ideas for us. For a show that bestows undreamt of wealth on
people who win. and does so with reasonable regularity, KBC
manages somehow to rise above the money it throws 1 around.
a government minister and when he died in a plane crash from
which Prime Minister Menederes emerged without a scratch, I
was sure she’d been crying because she had known in advance En By locating money squarely in the context of small dreams,
family and community, KBC shows us a face of money that is
what was going to happen. Lots of children had problems with
their teeth; a few wore braces. On the top floor of the building gin
ennobling. The money of KBC is treated not as a jackpot but as
a ‘vardaan’, a gift from divinity that comes for one’s persistent
effort, a prize for the penance called ordinary life. The images
that housed the lycée dormitory and the sports hall, just next
to the infirmary, there was rumoured to be a dentist, and when
teachers got angry they would often threaten to send naughty eer
that surround the winners are not big cars and fancy brands but
houses made ‘pukka’ and IAS dreams pursued. The winners have
children there. For lesser infractions pupils were made to stand
in the corner between the blackboard and the door with their ing
been remarkable ambassadors for the show, focusing not what
the money buys them but what it enables them to work at in
the future. Money speaks in the language of responsibility, not
backs to the class, sometimes one leg, but because we were all
so curious to see how long someone could stand on one leg, the
lessons suffered, so this particular punishment was rare. .ne
indulgence and steeps a larger collective in its pleasing warmth.
The format of the show ensures that we see people as they
34. The synonym for the term ‘vapid’ is
(a) Lively
(c) Lacklustre
(b) Original
(d) Spicy
35. Who is the least talked about character in this passage?
t
are, rather than the usual sight of raw innocents losing their
transparent naiveté in a haze of hair dye and exfoliation. On other
reality shows, fame and money are insistent in transforming
those that they favour and what they tell us is that success must
(a) Mother (b) Classmates put distance between destination and sources. between who we
(c) Grandmother (d) Teacher are and what we must become. On it iw the innocence that is
36. Which among the following cannot be concluded from this spoken to and as an audience it is this quahty we respond to.
passage? When a Sushi] Kumar descnbes hfe and attributes his success to
(a) The author was a good student but sometimes felt bored his_wife, who in turn is quick to shyly shrug off the credit, we
in class see, for once, something that smacks of the real on a reality show.
(b) The author got along fairly well with his classmates As the reality show evolved, it found reality too boring and
(c) The author came from a very authoritarian home vapid. It was so much for fun to manufacture it by making
environment people act in unpleasant ways. and say unsavoury things to each
(d) The author had an imaginative mind other. Now, no reality show can really bring us reality; any act
37. What did the teachers do when they get angry? of representation and framing creates its own version of reality
(a) Sent the students to the infirmary in many different ways I by aestheticizing it. emotionalising
(b) Denied them a chance to answer questions moments, dramatising revelations, withholding information
(c) Made them join their hands togetlier and sit quietly selectively, or by imbuing some moments with significance,
(d) Threatened to send them to the dentist. while ignoring others and even KBC uses these techniques. The
difference is that it uses these to drive us towards the central

Downloaded From : www.EasyEngineering.net


Downloaded From : www.EasyEngineering.net

Reading Comprehension  l 377

premise of the show rather than see those as individual ‘masala’ Passage- 11
elements. In a world where television is racked by anxiety about
itself, and where every new season is an exercise in renewed Babur’s head was throbbing with the persistent ache that dogged
desperation, KBC stands apart by continuing to tell a human him during the monsoon. The warm rain had been falling for
story about dreams and their fulfilment and doing so without three days now but the still. heavy air held no promise of relief ‘
trying too hard. The rains would go on for weeks, even months. Lying back against
There is no question that KBC rests on the persona of silken bolsters in his bedchamber in the Agra fort, he tried to
Amitabh Bachchan for he reconciles for us the idea of fame and imagine the chill, thin rains of Ferghana blowing in over the
humility, of achievement and empathy in the way he treats the jagged summit of Mount Beshtor and failed. The punkah above
participants. He has a special ability to look into the ordinary his head hardly disturbed the air. It was hard even to remember
and find something special and the humility to be awed by it. what it was like not to feel hot. There was little pleasure just now
He is simultaneously. The Amitabh Bachchan, the wax God even in visiting his garden the sodden flowers, soggy ground and
who we touch and squeal when we find out that it is real and
overflowing water channels only depressed him.
a fellow sympathizer and co-traveller on the journey called life.
Babur got up and tried to concentrate on writing an entry in
As a a carrier of life-altering destiny, he underplays his role to
perfection, acknowledging the enormity of that winning means his diary but the words wouldn’t come and he pushed his jewel-
for the participant while revealing the wisdom that knows that studded inkwell impatiently aside. Maybe he would go to the

ww
it is only money. Under his steerage money is no longer cold
acquisitive urgency but warm with unfolding possibility.
KBC shows us, close-up and in slow motion, the act of a
women’s apartments. Hewould ask Maha_m to sing. Sonhetimes
she acoompanied herself on the round-bellied, slender-necked
lute that had once belonged to Esan Dawlat. Maham lacked her

w.E
miracle colliding with a dream. In doing so, it tells us that
money can change things for the better, when it finds the right
home. By appliying good fortune to good intention. It keeps the
grandmother’s but the lute still made a sweet sound in her hands.
Or he might play a game of ehess with Humayun. His son had
a shrewd, subtle mind — but so, he prided himself, did he and he

asy
miracle alive, well after the movement of impact. As the winners
no doubt find out, one can never have enough money, and that
relative scale makes everyone a relative pauper. In the final
could usually beat him. It amused him to see Humayun’s startled
look as he claimed victory with the traditidnal cry shah mat
— ‘check-mate’, ‘the king is at a loss’. Later, they would discuss

En
analysis, Kaun Banega Crorepati reveals both the nobility and the
eventual poverty of money, no matter if it comes in eight figures.
38. According to the author’s opinion a few years before writing
Babur’s plans to launch a campaign when the rains eased against
the rulers of Bengal. In their steamy jungles in the Ganges delta,
this article, which of the following appeared to be in store for
KBC? gin
they thought they could defy Moghul authority and deny Babur’s
overlordship.
‘Send for my son Humayun and fetch my chessmen,’ Babur
i. The show’s time was over
ii. The show was too refined to compete with other reality
shows eer
ordered a servant. Trying to shake off his lethargy he got up and
went to a casement projecting over the riverbank to watch the
iii. Shah Rukh Khan as the show host would take it to new
heights
iv. The show’s viciousness was leading it, to its end
ing
swollen, muddy waters of the Jumna rushing by. A farmer was
leading his bony bullocks along the oozing bank.
Hearing footsteps Babur turned, expecting to see his son, but
(a) i only
(c) ii and iii
(b) i and ii only
(d) i and iv .ne
it was only the white-tunicked servant. ‘Majesty, your son begs
your forgiveness but he is unwell and cannot leave his chamber.’
39. Unlike most reality shows, KBC has gained viewership on
television by
(a) Using glamorous participants on the show
(b) Getting participants to say unpleasant things about the
What is the matter with him?’
‘I do not know, Majesty.
t
Humayun was never ill. Perhaps he, too, was suffering from
the torpor that came with the monsoon, sapping the energy and
truth of life spirit of even the most vigorous.
(c) Making major adjustments to its format time and again ‘I will go to him.’ Babur wrapped a yellow silk robe around
(d) Connecting with the depth and reality of lives of people himself and thrust his feet into pointed kidskin slippers. Then
40. According to the author, KBC presents the prize money as he hurried from his apartments to Humayun’s on the opposite
(a) A means for indulgence side of a galleried courtyard, where water was not shooting as it
(b) A jackpot should, in sparkling arcs from the lotus-shaped marble basins of
(c) A reward for relentless work the fountains but pouring over the inundated rims.
(d) A reason for changing the real person Humayun was lying on his bed, arms thrown back, eyes
41. In what context does the author use the phrase “a relative closed, forehead beaded with sweat, shivering. When he heard
pauper”? his father’s voice he opened his eyes but they were bloodshot, the
(a) No one can ever have enough money pupils dilated. Babur could hear his heavy wheezing breathing.
(b) Money can change who we are Every scratchy intake of air seemed an effort which hurt him.
(c) Money is cold and has materialistic importance ‘When did this illness begin?’
(d) Money can change things for better only if it finds the Early this morning, Father.’
right home ‘Why wasn’t I told?’ Babur looked angrily at his son’s
attendants. ‘Send for my hakim immediately!’ Then he dipped his

Downloaded From : www.EasyEngineering.net


Downloaded From : www.EasyEngineering.net

378  l  Reading Comprehension

own silk handkerchief into some water and wiped Humayun’s Passage- 12
brow. The sweat returned at once — in fact, it was almost
running down his face and he seemed to be shivering even “All raw sugar comes to us this way. You see, it is about the color
more violently now and his teeth had begun to chatter. of maple or brown sugar, but it is not nearly so pure, for it has a
‘Majesty, the hakim is here.’ great deal of dirt mixed with it when we first get it.”
Abdul-Malik went immediately to Humayun’s bedside, “Where does it come form?” inquired Bob.
laid a hand on his forehead, pulled back his eyelids and felt “Largely from the plantations of Cuba and Porto Rico.
his pulse. Then, with increasing concern, he pulled open Toward the end of the year we also get raw sugar from Java,
Humayun’s robe and, bending, turned his neatly turbaned and by the time this is refined and ready for the market the new
head to listen to Humayun’s heart. crop from the West Indies comes along. In addition to this we
‘What is wrong with him?’ get consignments from the Philippine Islands, the Hawaiian
Abdul-Malik paused. ‘It is hard to say, Majesty. I need to Islands, South America, Formosa, and Egypt. I suppose it is quite
examine him further.’ unnecessary to tell you young men anything of how the cane is
Whatever you require you only have to say...’ grown; of course you know all that.”
‘I will send for my assistants. If I may be frank, it would be “I don’t believe we do, except in a general way,” Bob admitted
best if you were to leave the chamber, Majesty. I will report honestly. “I am ashamed to be so green about a thing at which
to you when l have examined the prince thoroughly – but it Dad has been working for years. I don’t know why I never asked

ww
looks serious, perhaps even grave. His pulse and heartbeat
are weak and rapid.’ Without waiting for Babur’s reply, Abdul-
Malik turned back to his patient. Babur hesitated and, after
about it before. I guess I never was interested. I simply took it for
granted.”
“That’s the way with most of us,” was the superintendent’s

w.E
a glance at his son’s waxen trembling face, the room. As
attendants closed the doors behind him he found that he, tob,
was trembling.
kindly answer. “We accept many things in the world without
actually knowing much about them, and it is not until something
brings our ignorance before us that we take the pains to focus our

asy
A chill closed round his heart. So many times he had
feared for Humayun. At Panipat he could have fallen beneath
the feet of one of Sultan Ibrahim’s war elephants. At Khanua
attention and learn about them. So do not be ashamed that you
do not know about sugar raising; I didn’t when I was your age.
Suppose, then, I give you a little idea of what happens before this
he had never thought that Humayun — so healthy and strong
En
he might have been felled by the slash of a Rajput sword. But

— might succumb to sickness. How could he face life without


raw sugar can come to us.”
“I wish you would,” exclaimed both boys in a breath.
his beloved eldest son? Hindustan and all its riches would be
worthless if Humayun died. He would never have come to gin“Probably in your school geographies you have seen pictures
of sugar-cane and know that it is a tall perennial not unlike
our Indian corn in appearance; it has broad, flat leaves that
this sweltering, festering land with its endless hot rains and
whining, bloodsucking mosquitoes if he had known this
would be the price. eer
sometimes measure as many as three feet in length, and often the
stalk itself is twenty feet high. This stalk is jointed like a bamboo
42. Babur was feeling depressed because
(a) The rulers of Bengal were defying Moghul authority ing
pole, the joints being about three inches apart near the roots and
increasing in distance the higher one gets from the ground.”
“How do they plant it?” Bob asked.
(b) He could not usually beat Humayun at chess
(c) He did not like the warm rains and the heaviness of
monsoon air .ne
“It can be planted from seed, but this method takes much
time and patience; the usual way is to plant it from cuttings, or
(d) Maham could not play the lute as well as her
grandmother.
43. Which among the following things did Babur not
consider doing to relieve himself of depression?
t
slips. The first growth from these cuttings is called plant cane;
after these are taken off the roots send out ratoons or shoots
from which the crop of one or two years, and sometimes longer,
is taken. If the soil is not rich and moist replanting is more
frequently necessary and in places like Louisiana, where there
(a) Go to the women’s apartments is annual frost, planting must be done each year. When the cane
(b) Visit his garden is ripe it is cut and brought from the field to a central sugar mill,
(c) Play a game of chess with Humayun where heavy iron rollers crush from it all the juice. This liquid
(d) Listen to Maham sing drips through into troughs from which it is carried to evaporators
44. What was it that Babur currently feared for Humayun? where the water portion of the sap is eliminated and the juice left;
(a) Humayun could fall beneath the feet of war elephants you would surprised if you were to see this liquid. It looks like
(b) Humayun could be felled by the slash of a sword nothing so much as the soapy, bluish-gray dish-water that is left
(c) Humayun may not be treated properly by the Hakim in the pan after the dishes have been washed.”
(d) Humayun might succumb to sickness “A tempting picture!” Van exclaimed.
45. According to this passage, which of the following has not “I know it. Sugar isn’t very attractive during its process of
been used to describe Humayun? preparation,” agreed Mr. Hennessey. “The sweet liquid left after
(a) Shrewd and subtle minded the water has been extracted is then poured into vacuum pans
(b) Healthy and strong bodied to be boiled until the crystals form in it, after which it is put into
(c) Neatly turbaned head whirling machines, called centrifugal machines that separate the
(d) Father’s beloved dry sugar from the syrup with which it is mixed. This syrup is

Downloaded From : www.EasyEngineering.net


Downloaded From : www.EasyEngineering.net

Reading Comprehension  l 379

later boiled into molasses. The sugar is then dried and packed make choices based on expectations of how alternative options
in these burlap sacks such as you see here, or in hogsheads, and will perform (i.e., expected utility). Expectations about the quality
shipped to refineries to be cleansed and whitened.” of a product also play a central role in subsequent satisfaction.
“Isn’t any of the sugar refined in the places where it grows?” These expectations may be based on a number of factors
queried Bob. including the quality of a typical brand in a category, advertised
“Practically none. Large refining plants are too expensive to be quality, and disconfirmation sensitivity. Recent evidence
erected everywhere; it therefore seems better that they should be suggests that consumers, who are more disconfirmation sensitive
built in our large cities, where the shipping facilities are good not (i.e., consumers who are more satisfied when products perform
only for receiving sugar in its raw state but for distributing it after better than expected or more dissatisfied when products perform
it has been refined and is ready for sale. Here, too, machinery worse than expected) have lower expectations. However, there is
can more easily be bought and the business handled with less little research concerning the role of culture-specific variables in
difficulty.” expectation formation, particularly how they relate to the impact
46. Which one of the following is not a essential condition for of disconfirmation Sensitivity on consumer
setting up sugar refining plants? 49. “Future existences” in the paragraph can refer to’.
(a) Facilities for transportation of machinery 1. Human life, 5 years afterwards
(b) Facilities for import of raw material 2. Next birth in human form
(c) Facilities for transportation of finished products

ww
(d) Proximity to the raw material sources
47. Which of the following is the correct sequence of sugar
3. Next birth in any embodiment
Which of the following statement(s) is correct?
(a) 1,2 (b) 2, 3

w.E
preparation process?
(a) Cutting ® Crushing ® Evaporation ® Boiling ® Whirling.
(b) Boiling ® Crushing ® Evaporation ® Whirling ® Cutting.
(c) Cutting ® Boiling ® Evaporation ® Crushing ® Whirling.
(c) 1, 3
(e) None of the three
(d) 2 only

50. Consider the following assertion and conclusion: Assertion:


The meaning of karma in the above passage (refer to first

asy
(d) Whirling ® Crushing ® Boiling ® Evaporation ® Cutting.
48. Which of the following statements, as per the paragraph, is
two lines of the paragraph in italics.)
Conclusion: Belief that long term consequences are
incorrect?

En
(a) Sugar in its raw form is brownish in colour due to the
presence of dirt
important.
Now read the following statements carefully.
(b) After evaporation, cane juice looks bluish-gray in colour
(c) Molasses is obtained as a bye-product from the process gin 1. The conclusion will always follow the assertion.
2. The conclusion may follow the assertion.
3. The conclusion may follow the assertion only if an
of sugar production
(d) Cane plantation and sugar production process is widely
and equally spread across the countries. eer
individual lives long enough.
4. The conclusion cannot follow the assertion.

Passage- 13 (a) 1 only ing


Which of the following statement(s) is correct?
(b) 1 and 2 only
“Whatever actions are done by an individual in different
embodiments, [s]he reaps the fruit of those actions in those very
bodies or embodiments (in future existences)”.
(c) 2 only
(e) 4 only
.ne
(d) 3 only

51. Which of the following statements, if true, would contradict


A belief in karma entails, among other things, a focus on
long run consequences, i.e., a long term orientation. Such an
orientation implies that people who believe in karma may be more
honest with themselves in general and in setting expectations in
paragraph? t
the first of the three premises mentioned in the first

(a) Higher satisfaction leads to lower expectation.


(b) Lower expectation leads to long term consequences
particular-a hypothesis we examine here. This research is based (c) Satisfaction depends on achievement and not on
on three simple premises. First, because lower expectations often expectation
lead to greater satisfaction, individuals in general, and especially
(d) Karma affects our immediate feelings
those who are sensitive to the gap between performance and
(e) Lower expectation would lead to lower efforts
expectations, have the incentive to and actually do “strategically”
52. Read the following statements carefully:
lower their expectations. Second, individuals with a long term
orientation are likely to be less inclined to lower expectations 1. Temporary feelings and law of karma are independent
in the hope of temporarily feeling better. Third, long term 2. As per theory of karma, temporary feelings would not
orientation and the tendency to lower expectations are at least lower the expectation.
partially driven by cultural factors. In India, belief in karma, 3. Temporary feelings and law of karma are contradictory.
with its emphasis on a longer term orientation, will therefore Which of the following combination of statements is
to some extent counter-act the tendency to lower expectations. consistent with the second premise?
The empirical results support our logic; those who believe (a) 1 only (b) 1 and 2 only
more strongly in karma are less influenced by disconfirmation (c) 1 and 3 only (d) 3 only
sensitivity and therefore have higher expectations. Consumers (e) 1, 2 and 3 only

Downloaded From : www.EasyEngineering.net


Downloaded From : www.EasyEngineering.net

380  l  Reading Comprehension

53. A manager went out to have dinner in a restaurant and found implications concerning the acceptance or rejection of verified
the food to be good. When asked to provide feedback on the claims. Falsifying claims sometimes give us a good reason for
quality of food, the manager rated the quality as “excellent”. rejecting a scientific belief, namely when the claims are accepted.
Which of the following can be concluded from this? But verifying claims, even when accepted, give us no good and
(a) The manager does not believe in karma appropriate reason for accepting any scientific belief, because any
(b) The manager definitely has disconfirmation sensitivity. such reason would have to be inductive to be appropriate and
(c) It is not possible to comment on the disconfirmation there are no good inductive reasons.
sensitivity of the manager. 54. According to Popper, the statement “Scientific beliefs are
(d) The manager does not have disconfirmation sensitivity. universal in character” implies that
(e) None of the above. (a) Positive instances of scientific belief imply that it is
universal in character.
Passage- 14 (b) There are equal numbers of negative and positive
Popper claimed, scientific beliefs are universal in character, and instances of a universal scientific belief.
have to be so if they are to serve us in explanation and prediction. (c) If there are negative and positive instances of a scientific
For the universality of a scientific belief implies that, no matter belief then it cannot be universal.
how many instances we have found positive, there will always (d) We can only deduce that a scientific belief is false but

ww
be an indefinite number of unexamined instances which may or
may not also be positive. We have no good reason for supposing
that any of these unexamined instances will be positive, or will be
cannot induce that it is true.
(e) We can only induce that a scientific belief is false but
cannot induce that it is true.

w.E
negative, so we must refrain from drawing any conclusions. On
the other hand, a single negative instance is sufficient to prove that
the belief is false, for such an instance is logically incompatible
55. The statement, “this ‘asymmetry’ between verification and
falsification is not nearly as pronounced as Popper declared
it to be”, implies that

asy
with the universal truth of the belief. Provided, therefore, that
the instance is accepted as negative we must conclude that the
scientific belief is false. In short, we can sometimes deduce that a
(a) Falsification is better than verification in universal
acceptance beliefs.
(b) Verification is better than falsification in universal

En
universal scientific belief is false but we can never induce that a
universal scientific belief is true.
acceptance of scientific beliefs.
(c) Both falsification and verification together can result in
It is sometimes argued that this ‘asymmetry’ between
verification and falsification is not nearly as pronounced as Popper gin universal acceptance of scientific beliefs.
(d) Capability of falsification in accepting of scientific
beliefs is not better than that of verification in rejection
declared it to be. Thus, there is no inconsistency in holding that
a universal scientific belief is false despite any number of positive
instances; and there is no inconsistency either in holding that a eer
of scientific beliefs.
(e) Capability of falsification in rejection of scientific
beliefs is not always better than that of verification in
universal scientific belief is true despite the evidence of a negative
instance. For the belief that an instance is negative is itself a ing
acceptance of scientific beliefs.
56. With which of the following statements, would the author
scientific belief and may be falsified by experimental evidence
which we accept and which is inconsistent with it. When, for
agree most?

.ne
(a) Verification is better than falsification in establishing the
claims.
example, we draw a right-angled triangle on the surface of a
sphere using parts of three great circles for its sides, and discover
that for this triangle Pythagoras’ Theorem does not hold, we may
decide that this apparently negative instance is not really negative
(b) Falsification and verification are equally good in
establishing the claims.
(c) Verification and falsification are equally bad in
establishing the claims. A
t
because it is not a genuine instance at all. Triangles drawn on the (d) Falsification is better than verification in disproving the
surfaces of spheres are not the sort of triangles which fall within claims.
the scope of Pythagoras’ Theorem. Falsification, that is to say, (e) Verification is better than falsification in disproving the
is no more capable of yielding conclusive rejections of scientific claims.
belief than verification is of yielding conclusive acceptances 57. Which of the following would be the most appropriate
of scientific beliefs. The asymmetry between falsification and conclusion?
verification, therefore, has less logical significance than Popper (a) Falsification gives us an appropriate reason for rejecting
supposed. a scientific belief.
We should, though, resist this reasoning. Falsifications may (b) Falsification gives us all the reasons for accepting a
not be conclusive, for the acceptances on which rejections are claim.
based are always provisional acceptances. But, nevertheless, (c) Verification gives us a reason for rejecting a claim.
it remains the case that, in falsification, if we accept falsifying (d) Verification gives us an appropriate reason for accepting
claims then, to remain consistent, we must reject falsified claims. a scientific belief.
On the other hand, although verifications are also not conclusive, (e) Verification gives us an appropriate reason for rejecting
our acceptance or rejection of verifying instances has no a scientific belief.

Downloaded From : www.EasyEngineering.net


Downloaded From : www.EasyEngineering.net

Reading Comprehension  l 381

Passage- 15 economists who look at markets as if they were machines need


to be taken with a grain Of Salt Some - for example the school of
Soros, we must note, has never been a champion of free market Austrian economists - have made exactly that point against the
capitalism. He has followed for nearly all his public life the political neo-classical.
ideas of the late Sir Karl Popper who laid out a rather jumbled case Soros draws a mistaken inference: if one defense of the market
for what he dubbed “the open society” in his The Open Society is flawed, the market lacks defense. This is wrong. If it is true
and Its Enemies (1953). Such a society is what we ordinarily call that from A we can infer B, it does not prove that B can only be
the pragmatic system in which politicians get involved in people’s inferred from A; C or Z, too, might be a reason for B.
lives but without any heavy theoretical machinery to guide them, 58. As per the paragraph, author believes that
simply as the ad hoc parental authorities who are believed to be (a) Free market capitalism can be explained using neo-
needed to keep us all on the straight and narrow. Popper was classical economics.
at one time a Marxist socialist but became disillusioned with (b) Neo-classical economics does not address the idea of
that idea because he came to believe that systematic ideas do free-market system.
not work in any area of human concern. The Popperian open (c) Free market capitalism and open society are not different
society Soros promotes is characterized by a very general policy from each other.
of having no firm principles, not even those needed for it to (d) Free market capitalism and laissez-faire are not different
have some constancy and integrity. This makes the open society from each other.

ww
a rather wobbly idea, since even what Popper himself regarded
as central to all human thinking, critical rationalism, may be
(e) Technical underpinning of neo-classical economics can
address the idea of laissez-faire.
59. As per the paragraph, which of the following is true?

w.E
undermined by the openness of the open society since its main
target is negative: avoid dogmatic thinking, and avoid anything
that even comes close to a set of unbreachable principles. No, the
open society is open to anything at all, at least for experimental
(a) Economic benefits of open society and laissez-faire are
same.
(b) Soros’ open society means no interference from the

asy
purposes. No holds are barred, which, if you think about it,
undermines even that very idea and becomes unworkable.
government.
(c) Free market capitalism means no interference from the
government.
Accordingly, in a society Soros regards suited to human
community living, the state can manipulate many aspects of
human life, including, of course, the economic behavior of En (d) Laws of economics are not capable of explaining the
human nature completely.
individuals and firms. It can control the money supply, impose
wage and price controls, dabble in demand or supply-side gin (e) Laws of economics capture the human nature completely
as most of the human endeavors are economic in nature.
60. According to the author,
economics, and do nearly everything a central planning board
might - provided it does not settle into any one policy firmly,
unbendingly. That is the gist of Soros’s Popperian politics.
eer
(a) George Soros believes in regulated economies.
(b) George Soros does not believe in government

Soros’ distrusts capitalism in particular, because of the


alleged inadequacy of neoclassical economics, the technical ing
intervention in state policies.
(c) George Soros believes in state intervention provided it
does not remain static.
economic underpinnings of capitalist thinking offered up in
many university economics departments. He, like many others
.ne
(d) George Soros believes that laissez-faire economics
perform better than free-market economies.
outside and even inside the economics discipline, finds the arid
reductionism of this social science false to the facts, and rightly
so. But the defense of capitalist free markets does not rest on this
position.
perform better than controlled economies.
t
(e) George Soros believes that free-market economies

61. According to the author which of the following statement


could be true about critical rationalism.
Neo-classical thinking depends in large part on the (a) Ideas of critical rationalism underpin the foundation of
18th- and 19th-century belief that human society operates neo-classical economics.
according to laws, not unlike those that govern the physical (b) Ideas of critical rationalism underpin the foundation of
universe. Most of social science embraced that faith, so laissez-faire.
economics isn’t unusual in its loyalty to classical mechanics. Nor (c) Ideas of critical rationalism underpin the foundation of
do all economists take the deterministic lawfulness of economic open society.
science literally - some understand that the laws begin to operate (d) Ideas of critical rationalism underpin the foundation of
only once people embark upon economic pursuits. Outside their Newtonian physics.
commercial ventures, people can follow different principles and (e) None of the above.
priorities, even if it is undeniable that most of their endeavors 62. The word deterministic (used in forth line of fifth paragraph),
have economic features. Yet, it would be foolish >to construe in the above passage refers to:
religion or romance or even scientific inquiry as solely explicable (a) An effect can only be caused by a single event.
by reference to the laws of economics. (b) An effect may be produced by many causes.
In his criticism of neo-classical economic science, then, George (c) An effect cannot be produced by a cause.
Soros has a point: the discipline is too dependent on Newtonian (d) Cause(s) of an effect can always be known.
physics as the model of science. As a result, the predictions of (e) Economics does not follow cause and effect relationship.

Downloaded From : www.EasyEngineering.net


Downloaded From : www.EasyEngineering.net

382  l  Reading Comprehension

Passage- 16 (c) Loans taken by the company from banks and financial
institutions.
Of the several features of the Toyota Production System that have
(d) Ability of workers to evolve solutions to problems.
been widely studied, most important is the mode of governance
(e) Skill and charisma of the top leadership.
of the shop-floor at Toyota. Work and inter-relations between
workers are highly scripted in extremely detailed ‘operating 64. Which of the following can be best defended as a pre-condition
procedures’ that have to be followed rigidly, without any for the Toyota type of production system to work ?
deviation at Toyota. Despite such rule-bound rigidity, however, (a) Existence of workers’ union to protect worker’s rights.
Toyota does not become a ‘command-control system’. It is able to (b) Existence of powerful management to create unique
retain the character of a learning organization. strategies.
In fact, many observers characterize it as a community of (c) Cordial worker-management relations to have idustrial
scientists carrying out several small experiments simultaneously. peace.
The design of the operating procedure is the key. Every principle (d) High management involvement towards problems
must find an expression in the operating procedure-that is how it identified by workers.
has an effect in the domain of action. Workers on the shop-floor, (e) Management’s faith in workers’ abilities to solve
often in teams, design the ‘operating procedure’ jointly with the problems in a rigorous manner.
supervisor through a series of hypothesis that are proposed and 65. Based on the above passage, which of the following

ww
validated or refuted through experiments in action. The rigid and
detailed ‘operating procedure’ specificaion throws up problems
of the very minute kind: while its resolution leads to a reframing
statements is best justified ?
(a) Workers have significant control rights over the design
of work rules that allow worker skills and ingenuity to

w.E
of the procedure and specifications. This inter-temporal change
(or flexibility) of the specification (or operating procedure) is
done at the lowest level of the organization; i.e., closest to the
continuously search for novel micro-solutions using
information that often sticks to the local micro-context
of the work.
site of action.

asy
One implication of this arrangement is that system design
can no longer be rationally optimal and stadardized across the
(b) Managers have significant control rights over the design
of work rules that allow worker skills and ingenuity to

En
organization. It is quite common to find different work norms
in contiguous assembly lines, because each might have faced a
continuously search for novel micro-solutions around
micro-information that often sticks to the local micro-
context of the work.
different set of problems and devised different counter-measures
to tackle it. Design of the coordinating process that essentially
imposes the discipline that is required in large scale complex gin(c) Work rules enable the workers to report problems faced
at the shop-floor to specialised personnel who set up
manufacturing systems is therefore, customized to variations
in man-machine context of the site of action. It evolves through
numerous points of negotiation throughout the organization. It
eer
experiments to replicate the conditions. This allows the
specialists to come up with solutions that are rigorously

implies then that the higer levels of the hierarchy do not exercise
the power of the fiat in setting work rules, for such work rules are ing
tested in experimental conditions.
(d) Toyota as an organization has extensive networks with
different specialists who are subject matter experts in
no longer a standard set across the whole organization.
It might be interesting to go through the basic Toyota
.ne
different fields. These networks allow problems to be
resolved in the most advanced manner, enabling Toyota
philosophy that underlies its system designing practices. The
notion of the ideal production system in Toyota embraces the
following –’the ability to deliver just-in-time (or on demand) a
customer order in the exact specification demanded in a batch
to beat the competition.
t
(e) Toyota’s products are extensively tested by customers
in simulated conditions before they are released to the
market. This extensive testing is done by workers who
size of one (and hence, an infinite proliferation of variants,
models and specifications), defect-free, without wastage of double up as a community of scientists experimenting
material, labour, energy or motion in a safe and (physically to develop the most advanced product.
and emotionally) fulfilling production environment’. It did not 66. What could be the best defence of the “different work norms
embrace the concept of a standardized product that can be cheap in contiguous assembly lines’?
(a) Without such variations allowed, rights of manager to
by giving up variations. Preserving consumption variety was seen,
design work-rules would have made very little sense
in fact, as one mode of serving society. It is interesting to note
making the company similar to Ford.
that the articulation of the Toyota philosophy was made around
(b) Prosecribing standardised work norms would prevent
roughly the same time that the Fordist system was establishing
Toyota from benefiting from workers’ problem solving
itself in the US automotive industry.
ability in resolving different kinds of problems that
63. What can be best defended as the asset which Toyota model emerge, thus making it difficult to attain the Toyota
of production leverages to give the vast range of models in a philosophy.
defect-free fashion? (c) If similarities were imposed, rights of workers to
(a) Large scale complex manufacturing systems. experiment with work-rules would have made very little
(b) Intellectual capital of the company’s management. sense.

Downloaded From : www.EasyEngineering.net


Downloaded From : www.EasyEngineering.net

Reading Comprehension  l 383

(d) Standardisation of work-rules is only justified when the I. The source of motivation for both examples is same
investments in plants are huge and experimenting with II. Individuals may commit actions for reasons beyond
the work-rules would be detrimental to the efficiency of duty
the plants. Since, Toyota’s plants typically involved low III. Both examples illustrate the concept of moral worth
investments, it could tolerate non-standards work-rules. (a) Option I only (b) Option II only
(e) With standardisation of processes, right of the workers (c) Option I & II (d) Option III only
in design of work-rules made sense. Since, Toyota’s (e) Option II & III
manufacturing processes were non-standardised, the 68. Which of the following inferences would be against the ideas
different work norms did not make sense. in the passage?
I. Kantian ethics considers the moral worth of an
Passage- 17 inclination on the basis of its consequence.
One key element of Kantian ethics is the idea that the moral II. Actions motivated by the inclination of an individual
worth of any action relies entirely on the motivation of the lacks moral worth.
agent: human behaviour cannot be said good or bad in light III. Elements of moral obligation reduces the moral of a
of the consequences it generates, but only with regards to what duty, which has some worth in itself.
moved the agent to act in that particular way. Kant introduces (a) Option I only (b) Option I & II

ww
the key concept of duty to clarify the rational underpinning of
his moral theory, by analysing different types of motivation. First
of all individuals commit actions that are really undertaken for
(c) Option II only
(e) Option II & III
Passage- 18
(d) Option III only

w.E
the sake of duty itself, which is , done because the agent thinks
they are the right thing to do. No consideration of purpose of the
action matters, but only whether the action respects a universal
The driving force of evolution, according to the emerging
new theory, is not to be found in the chance events of random
mutations but in life’s inherent tendency to create novelty, in

asy
moral law. Another form of action (motivation) originates from
immediate inclination: Everyone has some inclinations, such
as to preserve one’s life, or to preserve honour. These are also
the spontaneous emergence of increasing complexity and order.
Once this fundamental new insight has been understood, we can

En
duties that have worth in their own sake. But acting according
to the maxim that these inclinations might suggests - such as
then ask: What are the avenues in which evolution’s creativity
expresses itself ?
The answer to this question comes not only from molecular
taking care of one’s own health - lacks for Kant true moral worth.
For example, a charitable person who donates some goods to
poor people might do it following her inclination to help the
gin
biology but also, and even more importantly, from microbiology,
from the study of the planetary web of the myriads of micro-
organisms that were the only forms of life during the first two
others - that is, because she enjoys helping the others. Kant
does not consider it as moral motivation, even if the action is eer
billion years of evolution. During those two billion years, bacteria
continually transformed the Earth’s surface and atmosphere
in conformity with duty. The person acting from duty would in
fact donate to the other because she recognizes that helping the
others is her moral obligation. Final type of motivation suggested
ing
and, in so doing, invented all of life’s essential biotechnologies,
including fermentation, photosynthesis, nitrogen fixation,
respiration, and rotary devices for rapid motion.
by Kant include actions that can be done in conformity with duty,
yet are not done from duty, but rather as a mean to some further .ne
During the past three decades, extensive research in microbiology
has revealed three major avenues of evolution. The first, but least
end. In order to illustrate this type of motivation, Kant provides
the following example. A shopkeeper who does not overcharge
the inexperienced customer and treats all customers in the same
way certainly is doing the right thing - that is, acts in conformity
neo-Darwinian theory. Gene mutation is caused by a chance
t
important, is the random mutation of genes, the centrepiece of

error in the self-replication of DNA, when the two chains of


the DNA’s double helix separate and each of them serves as a
template for the construction of a new complementary chain.
with duty - but we cannot say for sure that he is acting in this It has been estimated that those chance errors occur at a rate of
way because he is moved by the basic principles of honesty: “it is about one per several hundred million cells in each generation.
his advantage that requires it”. Moreover, we cannot say that he is This frequency does not seem to be sufficient to explain the
moved by an immediate inclination toward his customers, since evolution of the great diversity of life forms, given the well-
he gives no preference to one with respect to another. Therefore, known fact that most mutations are harmful, and only very few
concludes Kant, “his action was done neither from duty nor from result in useful variations.
immediate inclination but merely of purposes of self - interest” In the case of bacteria the situation is different, because bacterium
67. Consider the following examples: divides so rapidly. Fast bacteria can divide about every twenty
(i) Red Cross volunteer who donates blood every year to minutes, so that in principle several billion individual bacteria
thank an anonymous donor who saved the life of his can be generated from a single cell in less than a day. Because of
mother some time back this enormous rate of reproduction, a single successful bacterial
(ii) A voluntary organization which conducts regular mutant can spread rapidly through its environment, and mutation
blood donation camps to improve its legitimacy As per is indeed an important evolutionary avenue for bacteria.
the passage, correct statement(s) related to the above However, bacteria have developed a second avenue of
examples would be: evolutionary creativity that is vastly more effective than random

Downloaded From : www.EasyEngineering.net


Downloaded From : www.EasyEngineering.net

384  l  Reading Comprehension

mutation. They freely pass hereditary traits from one to another 70. Which three processes are responsible for evolution:
in a global exchange network of incredible power and efficiency. (a) Random mutation; Rapid division of genes in bacteria;
Here is how Lynn Margulis and Dorion Sagan describe it: Genes exchange in bacteria
Over the past fifty years or so, scientists have observed that (b) Random exchange of genes in bacteria; Speedy
[bacteria] routinely and rapidly transfer different bits of genetic multiplication of bacteria; Creative mutation
material to other individuals. Each bacterium at any given time (c) DNA self replication; Autopoieses; Gene pool theory
(d) Chance separation of double helix; Autopoiesis; Random
has the use of accessory genes, visiting from sometimes very
selection
different strains, which perform functions that its own DNA
71. Regarding diseases caused by bacteria and virus and their
may not cover. Some of the genetic bits are recombined with the eradication by medical science which conclusion is valid ?
cell’s native genes; others are passed on again. As a result of this (a) Medical science generally remains ahead of bacteria and
ability, all the world’s bacteria essentially have access to a single virus
gene pool and hence to the adaptive mechanisms of the entire (b) Bacteria and virus are generally ahead of medical
bacterial kingdom. science
This global trading of genes, technically known as DNA (c) Bacteria and virus are not only ahead, but manage to
recombination, must rank as one of the most astonishing undo somethings that medical science have achieved
(d) Bacteria and virus, and medical science are equal

ww
discoveries of modern biology. ‘If the genetic properties of the
microcosm were applied to larger creatures, we would have
a science-fiction world,’ write Margulis and Sagan, ‘in which
72. Which statement is true regarding the work that bacteria do
for the cause of humanity:

w.E
green plants could share genes for photosynthesis with nearby
mushrooms, or where people could exude perfumes; or grow
ivory by picking up genes from a rose or a walrus.’
(a) Bacteria invented many essential biotechnologies that
sustain life
(b) Bacteria challenge human beings to innovate
(c) Bacteria can give important lessons to human beings

asy
The speed with which drug resistance spreads among
bacterial communities is dramatic proof that the efficiency
about sharing and communicating
(d) All of the above work are important for the cause of
of their communications network is vastly superior to that of
adaptation through mutations. Bacteria are able to adapt to
environmental changes in a few years where larger organisms En humanity
73. Which philosophical paradigm does the model of creativity
in evolution as described in the passage derives from:
would need thousands of years of evolutionary adaptation. Thus
microbiology teaches us the sobering lesson that technologies gin(a) Holistic world view
(b) Descartes, Darwin, Newton
like genetic engineering and a global communications network,
which we consider to be advanced achievements of our modern eer
(c) Ecological framework
(d) Deep Ecology
74. What are the reasons given in the passage against the theory
civilization, have been used by the planetary web of bacteria for
billions of years to regulate life on Earth.
The constant trading of genes among bacteria results in an
ing
of “random mutation”, with respect to explaining evolution ?
(a) Random mutation is a slow process

amazing variety of genetic structures besides their main strand


of DNA. These include the formation of viruses, which are not
organism
.ne
(b) Most of the times random mutation is harmful for the

(c) Random mutation is not possible in smaller organisms


full autopoietic systems but consist merely of a stretch of DNA or
RNA in a protein coating. In fact, Canadian bacteriologist Sorin
Sonea has argued that bacteria, strictly speaking, should not be
classified into species, since all of their strains can potentially
(d) (a) and (b) are correct
t
75. Which principle described in the passage can become the
basis of science fiction:
(a) DNA recombination
share hereditary traits and, typically, change up to fifteen percent (b) DNA recombination among large organism
of their genetic material on a daily basis. ‘A bacterium is not a (c) DNA recombination among very small organism
unicellular organism,’ writes Sonea, ‘it is an incomplete cell (d) Autopoietic system
belonging to different chimeras according to circumstances. In Passage - 19
other words, all bacteria are part of a single microcosmic web of
One of the criteria by which we judge the vitality of a style
life’.
of painting is its ability to renew itself-its responsiveness to
69. If all human beings started behaving like bacteria, which
the changing nature and quality of experience, the degree
of the following would be the most desired outcome by all of conceptual and formal innovation that it exhibits. By this
humanity: criterion, it would appear that the practice of abstractionism
(a) Creativity and innovation will increase has failed to engage creatively with the radical change in human
(b) Greater unity in diversity experience in recent decades.
(c) Population increase It has, seemingly, been unwilling to re-invent itself in relation to
(d) We shall become identical to each other and be free of the systems of artistic expression and viewers’ expectations that
conflict have developed under the impact of the mass media.

Downloaded From : www.EasyEngineering.net


Downloaded From : www.EasyEngineering.net

Reading Comprehension  l 385

The judgment that abstractionism has slipped into ‘inertia gear’ privileging the new-minted geometric symbol as an autonomous
is gaining endorsement, not only among discerning viewers and sign of the desire for infinity.
practitioners of other art forms, but also among abstract painters Against this backdrop, we can identify three major abstractionist
themselves. Like their companions elsewhere in the world, idioms in Indian art. The first develops from a love of the earth,
abstractionists in India are asking themselves an overwhelming and assumes the form of a celebration of the self ’s dissolution
question today: Does abstractionism have a future? The major in the cosmic panorama; the landscape is no longer a realistic
crisis that abstractionist face is that of revitalising their picture transcription of the scene, but is transformed into a visionary
surface; few have improvised any solutions beyond the ones occasion for contemplating the cycles of decay and regeneration.
that were exhausted by the 1970s. Like all revolutions, whether The second idiom phrases its departures from symbolic and
in politics or in art, abstractionism must now confront its archetypal devices as invitations to heightened planes of
moment of truth: having begun life as a new and radical pictorial awareness. Abstractionism begins with the establishment
approach to experience, it has become an entrenched orthodoxy or dissolution of the motif, which can be drawn from diverse
itself. Indeed, when viewed against a historical situation in which sources, including the hieroglyphic tablet, the Sufi meditation
a variety of subversive, interactive and richly hybrid forms are dance or the Tantric diagram. The third idiom is based on
available to the art practitioner, abstractionism assumes the the lyric play of forms guided by gesture or allied with formal
remote and defiant air of an aristocracy that has outlived its age: improvisations like the assemblage. Here, sometimes, the line
trammelled by formulaic conventions yet buttressed by a rhetoric dividing abstract image from patterned design or quasi-random

ww
of sacred mystery, it seems condemned to being the last citadel of
the self-regarding ‘fine art’ tradition, the last hurrah of painting
for painting’s sake.
expressive marking may blur. The flux of forms can also be
regimented through the poetics of pure colour arrangements,
vectordiagrammatic spaces and gestural design.

w.E
The situation is further complicated in India by the circumstances
in which an indigenous abstractionism came into prominence
here during the 1960s. From the beginning it was propelled by
In this genealogy, some pure lines of descent follow their logic
to the inevitable point of extinction, others engage in cross
-fertilization, and yet others undergo mutation to maintain their

asy
the dialectic between two motives, one revolutionary and the
other conservative-it was inaugurated as an act of emancipation
from the dogmas of the nascent Indian nation state, when art
energy. However, this genealogical survey demonstrates the
wave at its crests, those points where the metaphysical and the
painterly have been fused in images of abiding potency, ideas

an instrument for the celebration of the republic’s hopes and


En
was officially viewed as an indulgence at worst, and at best, as sensuously ordained rather than fabricated programmatically to
a concept. It is equally possible to enumerate the thoughts where
aspirations.
Having rejected these dogmas, the pioneering abstractionists
also went on to reject the various figurative styles associated gin
the two principles do not come together, thus arriving at a very
different account. Uncharitable as it may sound, the history of
Indian abstractionism records a series of attempts to avoid the
with the Shantiniketan circle and others. In such a situation,
abstractionism was a revolutionary move. It led art towards the
eer
risks of abstraction by resorting to an overt and near-generic
symbolism, which many Indian abstractionists embrace when
exploration of the subconscious mind, the spiritual quest and
the possible expansion of consciousness. Indian painting entered
into a phase of selfinquiry, a meditative inner space where ing
they find themselves bereft of the imaginative energy to negotiate
the union of metaphysics and painterliness.
Such symbolism falls into a dual trap: it succumbs to the pompous
cosmic symbols and non-representational images ruled. Often,
the transition from figurative idioms to abstractionist ones took
place within the same artist. .ne
vacuity of pure metaphysics when the burden of intention is
passed off as justification; or then it is desiccated by the arid

At the same time, Indian abstractionists have rarely committed


themselves wholeheartedly to a non-representational idiom. They
have been preoccupied with the fundamentally metaphysical
project of aspiring to the mystical-holy without altogether
t
formalism of pure painterliness, with delight in the measure
of chance or pattern guiding the execution of a painting. The
ensuing conflict of purpose stalls the progress of abstractionism
in an impasse. The remarkable Indian abstractionists are precisely
those who have overcome this and addressed themselves to the
renouncing the symbolic. This has been sustained by a hereditary basic elements of their art with a decisive sense of independence
reluctance to give up the murti, the inviolable iconic form, which from prior models. In their recent work, we see the logic of Indian
explains why abstractionism is marked by the conservative abstractionism pushed almost to the furthest it can be taken.
tendency to operate with images from the sacred repertoire of the Beyond such artists stands a lost generation of abstractionists
past. Abstractionism thus entered India as a double-edged device whose work invokes a wistful, delicate beauty but stops there.
in a complex cultural transaction. Ideologically, it served as an Abstractionism is not a universal language; it is an art that
internationalist legitimisation of the emerging revolutionary points up the loss of a shared language of signs in society. And
local trends. However, on entry, it was conscripted to serve local yet, it affirms the possibility of its recovery through the effort of
artistic preoccupationsa survey of indigenous abstractionism will awareness. While its rhetoric has always emphasised a call for new
show that its most obvious points of affinity with European and forms of attention, abstractionist practice has tended to fall into a
American abstract art were with the more mystically oriented complacent pride in its own incomprehensibility; a complacency
of the major sources of abstractionist philosophy and practice, fatal in an ethos where vibrant new idioms compete for the
for instance the Kandinsky-Klee school. There have been no viewers’ attention. Indian abstractionists ought to really return
takers for Malevich’s Suprematism, which militantly rejected to basics, to reformulate and replenish their understanding of
both the artistic forms of the past and the world of appearances, the nature of the relationship between the painted image and the

Downloaded From : www.EasyEngineering.net


Downloaded From : www.EasyEngineering.net

386  l  Reading Comprehension

world around it. But will they abandon their favourite conceptual 82. According to the author, the attraction of the Kandinsky-
habits and formal conventions, if this becomes necessary?  Klee school for Indian abstractionist can be explained by
76. Which one of the following is not stated by the author as a which one of the following?
reason for abstractionism losing its vitality? (a) The conservative tendency to aspire to the mystical
(a) Abstractionism has failed to reorient itself in the context without a complete renunciation of the symbolic.
of changing human experience. (b) The discomfort of Indian abstractionists with Malevich’s
(b) Abstractionism has not considered the developments in Suprematism.
artistic expression that have taken place in recent times. (c) The easy identification of obvious points of affinity
(c) Abstractionism has not followed the path taken by all with European and American abstract art, of which the
revolutions, whether in politics or art. Kandinsky-Klee school is an example.
(d) The impact of mass media on viewers’ expectations has (d) The double-edged nature of abstractionism which
not been assessed, and responded to, by abstractionism. enabled identification with mystically-oriented schools.
77. Which one of the following, according to the author, is the 83. Which one of the following, according to the author, is the
role that abstractionism plays in a society? most important reason for the stalling of abstractionism’s
(a) It provides an idiom that can be understood by most progress in an impasse?
members in a society. (a) Some artists have followed their abstractionist logic to
(b) It highlights the absence of a shared language of the point of extinction.

ww
meaningful symbols which can be recreated through
greater awareness.
(c) It highlights the contradictory artistic trends of
(b) Some artists have allowed chance or pattern to dominate
the execution of their paintings.
(c) Many artists have avoided the trap of a near-generic and

w.E
revolution and conservatism that any society needs to
move forward.
(d) It helps abstractionists invoke the wistful, delicate beauty
that may exist in society.
an open symbolism.
(d) Many artists have found it difficult to fuse the twin
principles of the metaphysical and the painterly.

asy
78. According to the author, which one of the following
characterises the crisis faced by abstractionism?
Passage- 20
At the heart of the enormous boom in wine consumption that
(a) Abstractionists appear to be unable to transcend the
solutions tried out earlier.
En
(b) Abstractionism has allowed itself to be confined by set
has taken place in the English-speaking world over the last two
decades or so is a fascinating, happy paradox. In the days when
wine was exclusively the preserve of a narrow cultural elite,
forms and practices.
(c) Abstractionists have been unable to use the multiplicity gin
bought either at auctions or from gentleman wine merchants in
wing collars and bow-ties, to be stored in rambling cellars and
of forms now becoming available to an artist.
(d) All of the above.
79. According to the author, the introduction of abstractionism eer
decanted to order by one’s butler, the ordinary drinker didn’t
get a look-in. Wine was considered a highly technical subject,
in which anybody without the necessary ability could only fall
was revolutionary because it
(a) celebrated the hopes and aspirations of a newly ing
flat on his or her face in embarrassment. It wasn’t just that you
needed a refined aesthetic sensibility for the stuff if it wasn’t to
be hopelessly wasted on you. It required an intimate knowledge
independent nation.
(b) provided a new direction to Indian art, towards self-
inquiry and non-representational images. .ne
of what came from where, and what it was supposed to taste like.
Those were times, however, when wine appreciation essentially
(c) managed to obtain internationalist support for the
abstractionist agenda.
(d) was emancipation from the dogmas of the nascent
nation state.
t
meant a familiarity with the great French classics, with perhaps
a smattering of other wines-like sherry and port. That was
what the wine trade dealt in. These days, wine is bought daily
in supermarkets and high-street chains to be consumed that
80. Which one of the following is not part of the author’s evening, hardly anybody has a cellar to store it in and most don’t
characterisation of the conservative trend in Indian even possess a decanter. Above all, the wines of literally dozens
abstractionism? of countries are available on our market. When a supermarket
(a) An exploration of the subconscious mind. offers its customers a couple of fruity little numbers from Brazil,
(b) A lack of full commitment to non-representational we scarcely raise an eyebrow.
symbols. It seems, in other words, that the commercial jungle that wine
(c) An adherence to the symbolic while aspiring to the has now become has not in the slightest deterred people from
mystical. plunging adventurously into the thickets in order to taste and see.
(d) Usage of the images of gods or similar symbols. Consumers are no longer intimidated by the thought of needing
81. Given the author’s delineation to the three abstractionist to know their Pouilly-Fumé from their Pouilly-Fuissé, just at the
idioms in Indian art, the third idiom can be best distinguished very moment when there is more to know than ever before.
from the other two idioms through its The reason for this new mood of confidence is not hard to find. It
(a) depiction of nature’s cyclical renewal. is on every wine label from Australia, New Zealand, South Africa
(b) use of non-representational images. and the United States : the name of the grape from which the
(c) emphasis on arrangement of forms. wine is made. At one time that might have sounded like a fairly
(d) limited reliance on original models. technical approach in itself. Why should native English-speakers

Downloaded From : www.EasyEngineering.net


Downloaded From : www.EasyEngineering.net

Reading Comprehension  l 387

know what Cabernet Sauvignon or Chardonnay were? The support for Dr. Renaud’s findings about the effect of tannins?
answer lies in the popularity that wines made from those grape (a) A survey showed that film celebrities based in France
varieties now enjoy. Consumers effectively recognize them as have a low incidence of coronary heart disease.
brand names, and have acquired a basic lexicon of wine that can (b) Measurements carried out in southern France showed
serve them even when confronted with those Brazilian upstarts. red wine drinkers had significantly higher levels of
In the wine heartlands of France, they are scared to death of that coronary heart incidence than white wine drinkers did.
trend–not because they think their wine isn’t as good as the best (c) Data showed a positive association between sales of red
from California or South Australia (what French winemaker wine and incidence of coronary heart disease.
will ever admit that?) but because they don’t traditionally call (d) Long-term surveys in southern France showed that the
their wines Cabernet Sauvignon or Chardonnay. They call them incidence of coronary heart disease was significantly
Château Ducru-Beaucaillou or Corton-Charlemagne, and they lower in red wine drinkers than in those who did not
aren’t about to change. Some areas, in the middle of southern drink red wine.
France, have now produced a generation of growers using the 88. Which one of the following CANNOT be reasonably
varietal names on their labels and are tempting consumers back attributed to the labelling strategy followed by wine
to French wine. It will be an uphill struggle, but there is probably producers in Englishspeaking countries?
no other way if France is to avoid simply becoming a specialty (a) Consumers buy wines on the basis of their familiarity
source of old-fashioned wines for old-fashioned connoisseurs. with a grape variety’s name.

ww
Wine consumption was also given a significant boost in the early
1990s by the work of Dr. Serge Renaud, who has spent many
years investigating the reasons for the uncannily low incidence
(b) Even ordinary customers now have more access to
technical knowledge about wine.
(c) Consumers are able to appreciate better quality wines.

w.E
of coronary heart disease in the south of France. One of his major
findings is that the fat-derived cholesterol that builds up in the
arteries and can eventually lead to heart trouble, can be dispersed
(d) Some non-English speaking countries like Brazil
indicate grape variety names on their labels.

Passage- 21
asy
by the tannins in wine. Tannin is derived from the skins of grapes,
and is therefore present in higher levels in red wines, because
they have to be infused with their skins to attain the red colour.
The controversy over genetically-modified food continues
unabated in the West. Genetic modification (GM) is the science
That news caused a huge upsurge in red wine consumption in
the United States.
En by which the genetic material of a plant is altered, perhaps to
make it more resistant to pests or killer weeds, or to enhance
its nutritional value. Many food biotechnologists claim that GM
It has not been accorded the prominence it deserves in the
UK, largely because the medical profession still sees all alcohol
as a menace to health, and is constantly calling for it to be gin
will be a major contribution of science to mankind in the 21st
century. On the other hand, large numbers of opponents, mainly
made prohibitively expensive. Certainly, the manufacturers of
anticoagulant drugs might have something to lose if we all got
the message that we would do just as well by our hearts by taking
eer
in Europe, claim that the benefits of GM are a myth propagated
by multinational corporations to increase their profits, that they
pose a health hazard, and have therefore called for governments
half a bottle of red wine every day !
84. The tone that the author uses while asking “What French ing
to ban the sale of genetically modified food.
The anti-GM campaign has been quite effective in Europe, with
several European Union member countries imposing a virtual
winemaker will ever admit that?” is best described as
(a) caustic.
(c) critical.
(b) satirical.
(d) hypocritical. .ne
ban for five years over genetically-modified food imports. Since
the genetically-modified food industry is particularly strong in
85. What according to the author should the French do to avoid
becoming a producer of merely old-fashioned wines?
(a) Follow the labelling strategy of the English-speaking
countries.
another chapter in the US-Europe skirmishes which have
t
the United States of America, the controversy also constitutes

become particularly acerbic after the US invasion of Iraq. To a


large extent, the GM controversy has been ignored in the Indian
media, although Indian biotechnologists have been quite active
(b) Give their wines English names.
in GM research. Several groups of Indian biotechnologists have
(c) Introduce fruity wines as Brazil has done.
been working on various issues connected with crops grown in
(d) Produce the wines that have become popular in the
India.
English-speaking world.
One concrete achievement which has recently figured in the news
86. The development which has created fear among winemakers is that of a team led by the former vice-chancellor of Jawaharlal
in the wine heartlands of France is the
Nehru University, Asis Datta-it has successfully added an extra
(a) tendency not to name wines after the grape varieties that
gene to potatoes to enhance the protein content of the tuber by at
are used in the wines.
least 30 percent. Not surprisingly, the new potato has been called
(b) ‘education’ that consumers have derived from wine
the protato. The protato is now in its third year of field trials. It is
labels from English-speaking countries.
quite likely that the GM controversy will soon hit the headlines
(c) new generation of local winegrowers who use labels that
in India since a spokesperson of the Indian Central government
show names of grape varieties.
has recently announced that the government may use the protato
(d) ability of consumers to understand a wine’s qualities
in its midday meal programme for schools as early as next year.
when confronted with “Brazilian upstarts”.
Why should “scientific progress”, with huge potential benefits
87. Which one of the following, if true, would provide most
to the poor and malnourished, be so controversial? The anti-

Downloaded From : www.EasyEngineering.net


Downloaded From : www.EasyEngineering.net

388  l  Reading Comprehension

GM lobby contends that pernicious propaganda has vastly (c) Germany and France
exaggerated the benefits of GM and completely evaded the costs (d) Australia and New Zealand.
which will have to be incurred if the genetically modified food 91. Genetic modification makes plants more resistant to killer
industry is allowed to grow unchecked. In particular, they allude weeds. However, this can lead to environmental damage by
to different types of costs. (a) wiping out competing varieties of plants which now fall
This group contends that the most important potential cost is prey to killer weeds.
that the widespread distribution and growth of genetically- (b) forcing application of stronger herbicides to kill weeds
modified food will enable the corporate world (alias the which have become resistant to weak herbicides.
multinational corporations-MNCs) to completely capture the (c) forcing application of stronger herbicides to keep the
food chain. A “small” group of biotech companies will patent the competing plants weed-free.
transferred genes as well as the technology associated with them. (d) not allowing growth of any weeds, thus reducing soil
They will then buy up the competing seed merchants and seed fertility. 138.According to the passage, biotechnology
breeding, centres, thereby controlling the production of food at research
every possible level. Independent farmers, big and small, will be 92. According to the passage, biotechnology research
completely wiped out of the food industry. At best, they will be (a) is of utility only for high value food items.
reduced to the status of being subcontractors. (b) is funded only by multinational corporations.
This line of argument goes on to claim that the control of the food (c) allows multinational corporations to control the food

ww
chain will be disastrous for the poor since the MNCs, guided by
the profit motive, will only focus on the high-value food items
basket of the poor.
(d) addresses the concerns of rich and poor countries.

w.E
demanded by the affluent. Thus, in the long run, the production
of basic staples which constitute the food basket of the poor will
taper off. However, this vastly overestimates the power of the
MNCs. Even if the research promoted by them does focus on
93. Which of the following about the Indian media’s coverage of
scientific research does the passage seem to suggest?
(a) Indian media generally covers a subject of scientific
importance when its mass application is likely.

asy
the high-value food items, much of biotechnology research is
also funded by governments in both developing and developed
(b) Indian media’s coverage of scientific research is generally
dependent on MNCs interests.

En
countries. Indeed, the protato is a by-product of this type of
research. If the protato passes the field trials, there is no reason
(c) Indian media, in partnership with the government, is
actively involved in publicizing the results of scientific
research.
to believe that it cannot be marketed in the global potato market.
And this type of success story can be repeated with other basic
food items. gin(d) Indian media only highlights scientific research which is
funded by the government.
The second type of cost associated with the genetically modified
food industry is environmental damage. The most common type eer Passage- 22
Social life is an outflow and meeting of personality, which
of “genetic engineering” involves gene modification in plants
designed to make them resistant to applications of weed-killers.
This then enables farmers to use massive dosages of weed-killers ing
means that its end is the meeting of character, temperament
and sensibility in which our thoughts and feelings and sense
so as to destroy or wipe out all competing varieties of plants in
their fields. .ne
perceptions are brought into play at their lightest and yet keenest.
This aspect, to my thinking, is realized as much in large parties
composed of casual acquaintances or even strangers, as in
However, some weeds through genetically modified pollen
contamination may acquire resistance to a variety of weed-
killers. The only way to destroy these weeds is through the use of
ever-stronger herbicides, which are poisonous and linger on in
t
intimate meetings of old friends. I am not one of those superior
persons who hold cocktail parties in contempt, looking upon
them as barren or at best as very tryingly kaleidoscopic places
the environment.  for gathering, because of the strangers one has to meet in them,
89. The author doubts the anti-GM lobby’s contention that which is no argument, for even our most intimate friends must
MNC control of the food chain will be disastrous for the at one time have been strangers to us. These large gatherings will
poor because be only what we make of them if not anything better, they can be
(a) MNCs will focus on high-value food items. as good places to collect new friends from as the slave-markets of
(b) MNCs are driven by the motive of profit maximization. Istanbul were for beautiful slaves or New Market for race horses.
(c) MNCs are not the only group of actors in genetically But they do offer more immediate enjoyment. For one thing,
modified food research. in them one can see the external expression of social life in
(d) economic development will help the poor buy MNC- appearance and behaviour at its widest and most varied where
produced food. one can admire beauty of body or air, hear voices remarkable
90. Using the clues in the passage, which of the following either for sweetness or refinement, look on elegance of clothes or
countries would you expect to be in the forefront of the anti- deportment. What is more, these parties are schools for training
GM campaign? in sociability, for in them we have to treat strangers as friends. So,
(a) USA and Spain. in them we see social sympathy in widest commonalty spread,
(b) India and Iraq. or at least should. We show an atrophy of the natural human

Downloaded From : www.EasyEngineering.net


Downloaded From : www.EasyEngineering.net

Reading Comprehension  l 389

instinct of getting pleasure and happiness out of other human 95. In this passage the author is essentially
beings if we cannot treat strangers as friends for the moment. (a) showing how shallow our social life is.
And I would go further and paraphrase Pater to say that not to be (b) poking fun at the lower middle class people who howl at
able to discriminate every moment some passionate attitude in better off people.
(c) lamenting the drying up of our real social life.
those about us, even when we meet them casually, is on this short
(d) criticizing the upper class for lavish showy parties.
day of frost and sun which our life is, to sleep before evening.
96. The author’s conception of ‘social life’ requires that
So, it will be seen that my conception of social life is modest, for it (a) people attend large gatherings.
makes no demands on what we have, though it does make some (b) people possess qualities like wonder and interest.
on what we are. Interest, wonder, sympathy and love, the first (c) people do not spend too much time in the company of
two leading to the last two, are the psychological prerequisites for intimate friends.
social life; and the need for the first two must not be underrated. (d) large parties consist of casual acquaintances and intimate
We cannot make the most even of our intimate social life unless friends.
we are able to make strangers of our oldest friends everyday by 97. The word ‘discriminate’ in the last sentence of the third
discovering unknown areas in their personality, and transform paragraph means
them into new friends. In sum, social life is a function of vitality. (a) recognise. (b) count.
(c) distinguish. (d) analyse.

ww
It is tragic, however, to observe that it is these very natural springs
of social life which are drying up among us. It is becoming more
and more difficult to come across fellow feeling for human
98. What is the author trying to show through the two incidents
in the paragraph beginning, “Their most innocent exhibition
of sociability …’’?

w.E
beings as such in our society-and in all its strata. In the poor
middle class, in the course of all my life, I have hardly seen any
social life properly so-called. Not only has the grinding routine
(a) The crowds in poor Calcutta suburbs can turn violent
without any provocation.
(b) Although poor, the people of poor Calcutta suburbs

asy
of making a living killed all desire for it in them, it has also
generated a standing mood of peevish hostility to other human
have a rich social life.
(c) It is risky for rich people to move around in poor
suburbs.

En
beings. Increasing economic distress in recent years has infinitely
worsened this state of affairs, and has also brought a sinister
addition-class hatred. This has become the greatest collective
(d) Achieving a high degree of sociability does not stop the
poor from hating the rich.
emotional enjoyment of the poor middle class, and indeed they
feel most social when they form a pack, and snarl or howl at gin Passage- 23
people who are better off than they.
Their most innocent exhibition of sociability is seen when they eer
Modern science, exclusive of geometry, is a comparatively recent
creation and can be said to have originated with Galileo and
Newton. Galileo was the first scientist to recognize clearly that the
spill out from their intolerable homes into the streets and bazaars.
I was astonished to see the milling crowds in the poor suburbs of
Calcutta. But even there a group of flippant young loafers would
ing
only way to further our understanding of the physical world was
to resort to experiment. However obvious Galileo’s contention
put on a conspiratorial look if they saw a man in good clothes
passing by them either on foot or in a car. I had borrowed a car .ne
may appear in the light of our present knowledge, it remains a
fact that the Greeks, in spite of their proficiency in geometry,
never seem to have realized the importance of experiment.
from a relative to visit a friend in one of these suburbs, and he
became very anxious when I had not returned before dusk. Acid
and bombs, he said, were thrown at cars almost every evening in
that area. I was amazed. But I also know as a fact that my brother
t
To a certain extent this may be attributed to the crudeness of
their instruments of measurement. Still, an excuse of this sort
can scarcely be put forward when the elementary nature of
Galileo’s experiments and observations is recalled. Watching a
was blackmailed to pay five rupees on a trumped up charge when lamp oscillate in the cathedral of Pisa, dropping bodies from
passing in a car through one such locality. the leaning tower of Pisa, rolling balls down inclined planes,
The situation is differently inhuman, but not a whit more human, noticing the magnifying effect of water in a spherical glass vase,
among the well-to-do. Kindliness for fellow-human beings such was the nature of Galileo’s experiments and observations.
As can be seen, they might just as well have been performed by
has been smothered in them, taken as a class, by the arrogance
the Greeks. At any rate, it was thanks to such experiments that
of worldly position, which among the Bengalis who show this
Galileo discovered the fundamental law of dynamics, according
snobbery is often only a third-class position. to which the acceleration imparted to a body is proportional to
94. The word ‘they’ in the first sentence of the third paragraph the force acting upon it.
refers to The next advance was due to Newton, the greatest scientist of all
(a) Large parties consisting of casual acquaintances and time if account be taken of his joint contributions to mathematics
strangers. and physics. As a physicist, he was of course an ardent adherent
(b) Intimate meetings of old friends. of the empirical method, but his greatest title to fame lies in
(c) New friends. another direction. Prior to Newton, mathematics, chiefly in the
(d) Both (a) & (b). form of geometry, had been studied as a fine art without any

Downloaded From : www.EasyEngineering.net


Downloaded From : www.EasyEngineering.net

390  l  Reading Comprehension

view to its physical applications other than in very trivial cases. 99. According to the author, why did the Greeks NOT conduct
But with Newton all the resources of mathematics were turned experiments to understand the physical world?
(a) Apparently they did not think it necessary to experiment.
to advantage in the solution of physical problems. Henceforth
(b) They focused exclusively on geometry.
mathematics appeared as an instrument of discovery, the most
(c) Their instruments of measurement were very crude.
powerful one known to man, multiplying the power of thought
(d) The Greeks considered the application of geometry to
just as in the mechanical domain the lever multiplied our physical the physical world more important.
action. It is this application of mathematics to the solution of 100. The statement “Nature thus exhibited rationality and was
physical problems, this combination of two separate fields of not mere blind chaos and uncertainty” suggests that
investigation, which constitutes the essential characteristic (a) problems that had baffied scientists like Archimedes
of the Newtonian method. Thus problems of physics were were not really problems.
metamorphosed into problems of mathematics. (b) only a small group of natural phenomena was chaotic.
But in Newton’s day the mathematical instrument was still in a (c) physical phenomena conformed to mathematical laws.
very backward state of development. In this field again Newton (d) natural phenomena were evolving towards a less chaotic
showed the mark of genius by inventing the integral calculus. future.
As a result of this remarkable discovery, problems, which would 101. Newton may be considered one of the greatest scientists of

ww
have baffled Archimedes, were solved with ease. We know that
in Newton’s hands this new departure in scientific method led
to the discovery of the law of gravitation. But here again the real
all time because he
(a) discovered the law of gravitation.
(b) married physics with mathematics.

w.E
significance of Newton’s achievement lay not so much in the
exact quantitative formulation of the law of attraction, as in his
having established the presence of law and order at least in one
(c) invented integral calculus.
(d) started the use of the empirical method in science.
102. Which of the following statements about modern science
best captures the theme of the passage?

asy
important realm of nature, namely, in the motions of heavenly
bodies. Nature thus exhibited rationality and was not mere blind
chaos and uncertainty. To be sure, Newton’s investigations had
(a) Modern science rests firmly on the platform built by the
Greeks.

been concerned with but a small group of natural phenomena,


but it appeared unlikely that this mathematical law and order En (b) We need to go back to the method of enquiry used by
the Greeks to better understand the laws of dynamics.

should turn out .to be restricted to certain special phenomena,


and the feeling was general that all the physical processes of gin (c) Disciplines like Mathematics and Physics function best
when integrated into one.
(d) New knowledge about natural phenomena builds on
nature would prove to be unfolding themselves according to
rigorous mathematical laws.
eer
existing knowledge.
103. The significant implication of Einstein’s special principle of
When Einstein, in 1905, published his celebrated paper on
the electrodynamics of moving bodies, he remarked that the
difficulties, which surrounded the equations of electrodynamics, ing
relativity is that
(a) absolute velocity was meaningless in the realm of
mechanics.
together with the negative experiments of Michelson and others,
would be obviated if we extended the validity of the Newtonian .ne
(b) Newton’s principle of relativity needs to be modified.
(c) there are limits to which experimentation can be used to
principle of the relativity of Galilean motion, which applied
solely to mechanical phenomena, so as to include all manner
of phenomena: electrodynamics, optical etc. When extended in
this way the Newtonian principle of relativity became Einstein’s
understand some physical phenomena.

t
(d) it is meaningless to try to understand the distinction
between velocity and accelerated or rotational motion.

Passage- 24
special principle of relativity. Its significance lay in its assertion
that absolute Galilean motion or absolute velocity must ever As you set out for Ithaka hope the journey is a long one, full of
adventure, full of discovery.
escape all experimental detection. Henceforth absolute velocity
Laistrygonians and Cyclops, angry Poseidon – don’t be afraid of
should be conceived of as physically meaningless, not only in
them: you’ll never find things like that on your way as long as you
the particular realm of mechanics, as in Newton’s day, but in the
keep your thoughts raised high, as long as a rare excitement stirs
entire realm of physical phenomena.
your spirit and your body.
Einstein’s special principle, by adding increased emphasis to this Laistrygonians and Cyclops, wild Poseidon – you won’t encounter
relativity of velocity, making absolute velocity metaphysically them unless you bring them along inside your soul, unless your
meaningless, created a still more profound distinction between soul sets them up in front of you.
velocity and accelerated or rotational motion. This latter type of Hope the voyage is a long one, may there be many a summer
motion remained absolute and real as before. It is most important morning when, with what pleasure, what joy, you come into
to understand this point and to realize that Einstein’s special harbours seen for the first time: may you stop at Phoenician
principle is merely an extension of the validity of the classical trading stations to buy fine things, mother of pearl and coral,
Newtonian principle to all classes of phenomena. (CAT 2003 C) amber and ebony, sensual perfume of every kind – as many

Downloaded From : www.EasyEngineering.net


Downloaded From : www.EasyEngineering.net

Reading Comprehension  l 391

sensual perfumes as you can; and may you visit many Egyptian other in the middle class, people like me are Straddlers, at home
cities to gather stores of knowledge from their scholars. in neither world, living a limbo life.
Keep Ithaka always in your mind. What drove me to leave what I knew? Born blue– collar, I still
Arriving there is what you are destined for. never felt completely at home among the tough guys and anti-
But do not hurry the journey at all. intellectual crowd of my neighbourhood in deepest Brooklyn.
Better if it lasts for years, so you are old by the time you reach the I never did completely fit in among the preppies and suburban
island, wealthy with all you have gained on the way, not expecting royalty of Columbia, either. It’s like that for Straddlers. It was
lthaka to make you rich. Ithaka gave you the marvellous journey, not so smooth jumping from Italian old–world style to US
without her you would not have set out. She has nothing left to professional in a single generation. Others who were the first in
give you now. their families to go to college, will tell you the same thing : the
And if you find her poor, Ithaka won’t have fooled you. academy can render you unrecognisable to the very people who
Wise as you will have become, so full of experience, you will have launched you into the world. The ideas and values absorbed in
understood by then what these Ithakas mean. 
college challenge the mom–and–pop orthodoxy that passed for
104. Which of the following best reflects the central theme of this
truth for 18 years. Limbo folk may eschew polyester blends for
poem?
sea–isle cotton, prefer Brie to Kraft slices. They marry outside the
(a) If you don’t have high expectations, you will not be

ww
disappointed.
(b) Don’t rush to your goal; the journey is what enriches
you.
neighbourhood and raise their kids differently. They might not
be in church on Sunday.
When they pick careers (not jobs), it’s often a kind of work their


w.E
(c) The longer the journey the greater the experiences you
gather.
(d) You cannot reach Ithaka without visiting Egyptian ports.
parents never heard of or can’t understand. But for the white–
collar kids of blue– collar parents, the office is not necessarily
a sanctuary. In Corporate America, where the rules are based

is the most comprehensive reason for it? asy


105. The poet recommends a long journey. Which of the following

(a) You can gain knowledge as well as sensual experience.


on notions foreign to working–class people, a Straddler can get
lost. Social class counts at the office, even though nobody likes
to admit it. Ultimately, corporate norms are based on middle–
(b) You can visit new cities and harbours.
(c) You can experience the full range of sensuality. En class values, business types say. From an early age, middle–class
people learn how to get along, using diplomacy, nuance, and
(d) You can buy a variety of fine things.
106. In the poem, Ithaka is a symbol of gin
politics to grab what they need. It is as though they are following
a set of rules laid out in a manual that blue–collar families never
have the chance to read.

(a) the divine mother.
(c) the path to wisdom.
(b) your inner self.
(d) life’s distant goal.
107. What does the poet mean by ‘Laistrygonians’ and ‘Cyclops’? eer
People born into the middle class to parents with college degrees
have lived lives filled with what French sociologist Pierre Bourdieu
(a) Creatures which, along with Poseidon, one finds during
a journey. ing
calls ‘cultural capital’. Growing up in an educated environment,
they learn about Picasso and Mozart, stock portfolios and crème
brulee. In a home with cultural capital, there are networks:
(b) Mythological characters that one should not be afraid of.
(c) Intra-personal obstacles that hinder one’s journey.
(d) Problems that one has to face to derive the most from
.ne
someone always has an aunt or golfing buddy with the inside
track for an internship or some entry–level job. Dinner–table
one’s journey.
108. Which of the following best reflects the tone of the poem?
(a) Prescribing. (b) Exhorting.
t
talk could involve what happened that day to mom and dad at
the law firm, the doctor’s office, or the executive suite. Middle–
class kids can grow up with a sense of entitlement that will carry
them through their lives. This ‘belongingness’ is not just related
(c) Pleading. (d) Consoling.
to having material means, it also has to do with learning and
Passage- 25 possessing confidence in your place in the world. Such early
While I was in class at Columbia, struggling with the esoterica access and direct exposure to culture in the home is the more,
du jour, my father was on a bricklayer’s scaffold not far up the organic, ‘legitimate’ means of appropriating cultural capital,
street, working on a campus building. Once we met up on the Bourdieu tells us. Those of us possessing ‘ill–gotten Culture’ can
subway going home – he was with his tools, I with my books. My learn it, but never as well. Something is always a little off about
father wasn’t interested in Thucydides, and I wasn’t up on arches. us, like an engine with imprecise timing. There’s a greater match
between middle–class lives and the institutions in which the
My dad has built lots of places in New York City he can’t get
middle class works and operates–universities or corporations.
into : colleges, condos, office towers. He made his living on the
Children of the middle and upper classes have been speaking
outside. Once the walls were up, a place took on a different feel
the language of the bosses and supervisors forever. Blue–collar
for him, as though he wasn’t welcome anymore. Related by blood,
kids are taught by their parents and communities to work hard
we’re separated by class, my father and I. Being the white–collar to achieve, and that merit is rewarded. But no blue–collar parent
child of a blue– collar parent means being the hinge on the door knows whether such things are true in the middle–class world.
between two ways of life. With one foot in the working–class, the Many professionals born to the working– class report feeling

Downloaded From : www.EasyEngineering.net


Downloaded From : www.EasyEngineering.net

392  l  Reading Comprehension

out of place and outmanoeuvred in the office. Soon enough,


materials and the genius of Whittle and von Ohain to recognize
Straddlers learn that straight talk won’t always cut. Resolving
the advantages that a gas turbine offered over a piston engine,
conflicts head–on and speaking your mind doesn’t always work,
including speeds in excess of 350 miles per hour. The progress
no matter how educated the Straddler is.
from the first flights of liquid propellant rocket and jetpropelled
In the working–class, people perform jobs in which they
aircraft in 1939 to the first faster–than–sound (supersonic)
are closely supervised and are required to follow orders and
manned airplane (the Bell X–1) in 1947 happened in less than a
instructions.
decade. This then led very rapidly to a series of supersonic fighters
That, in turn, affects how they socialise their children. Children of
and bombers, the first of which became operational in the 1950s.
the working–class are brought up in a home in which conformity,
World War II technology foundations and emerging Cold War
obedience and intolerance for back talk are the norm–the same
imperatives then led us into space with the launch of Sputnik in
characteristics that make a good factory worker. 
1957 and the placing of the first man on the moon only 12 years
109. When Straddlers enter white collar jobs, they get lost because
later—a mere 24 years after the end of World War II.
(a) they are thrown into an alien value system.
Now, a hypersonic flight can take you anywhere in the planet
(b) their families have not read the rules in corporate
in less than four hours. British Royal Air Force and Royal Navy,
manuals.
and the air forces of several other countries are going to use a
(c) they have no one to guide them through the corporate
single–engine cousin to the F/A–22 called the F–35 Joint Strike
maze.

ww
(d) they miss the ‘mom and pop orthodoxy’.
110. What does the author’s statement, “My father wasn’t interested
Fighter. These planes exhibit stealthy angles and coatings that
make it difficult for radar to detect them, among aviation’s most
cutting–edge advances in design. The V–22, known as tilt–rotor,

w.E
in Thucydides, and I wasn’t up on arches”, illustrate?
(a) Organic cultural capital.
(b) Professional arrogance and social distance.
(c) Evolving social transformation.
part helicopter, part airplane, takes off vertically, then tilts its
engine forward for winged flight. It provides speed, three times
the payload, five times the range of the helicopters it’s meant
(d) Breakdown of family relationships.
asy
111. Which of the following statements about Straddlers does the
to replace. The new fighter, F/A–22 Raptor, with more than a
million parts, shows a perfect amalgamation of stealth, speed,
avionics and agility.
passage NOT support explicitly?
(a) Their food preferences may not match those of their
parents. En It seems conventional forms, like the Predator and Global Hawk
are passe, the stealthier unmanned aerial vehicles (VA Vs) are in.
(b) They may not keep up some central religious practices of
their parents. gin
They are shaped like kites, bats and boomerang, all but invisible to
the enemy radar and able to remain over hostile territory without
any fear of getting grilled if shot down. Will the UAVs take away
(c) They are at home neither in the middle class nor in the
working–class.
(d) Their political ideologies may differ from those of their eer
pilots’ jobs permanently? Can a computer–operated machine
take a smarter and faster decision in a war–like situation? The
parents.
112. According to the passage, which of the following statements ing
new free–flight concept will probably supplement the existing
air traffic control system by computers on each plane to map the
altitude, route, weather and other planes; and a decade from now,
about ‘cultural capital’ is NOT true?
(a) It socializes children early into the norms of middle class
institutions.
there will be no use of radar any more.
.ne
How much bigger can the airplanes get? In the ‘50s they got
(b) It helps them learn the language of universities and
corporations.
(c) It creates a sense of enlightenment in middle– class
children
t
speed, in the ‘80s they became stealthy. Now, they are getting
smarter thanks to computer automation. The change is quite
huge: from the four–seater to the A380 airplane. It seems we
are now trading speed for size as we build a new Super–jumbo
(d) It develops bright kids into Straddlers. jet, the 555 seater A380, which will fly at almost the same speed
113. According to the passage, the patterns of socialization of of the Boeing 707, introduced half a century ago, but with an
working–class children make them most suited for jobs that improved capacity, range, greater fuel economy. A few years
require down the line will come the truly larger model, to be known as
(a) diplomacy 747X. In the beginning of 2005, the A380, the world’s first fully
(b) compliance with orders. double– decked superjumbo passenger jet, weighing 1.2 million
(c) enterprise and initiative. pounds, may carry a load of about 840 passengers.
(d) high risk taking. Barring the early phase, civil aviation has always lagged behind
the military technologies (of jet engines, lightweight composite
Passage- 26 materials etc.). There are two fundamental factors behind the
The invention of the gas turbine by Frank Whittle in England and decline in commercial aeronautics in comparison to military
Hans von Ohain in Germany in 1939 signalled the beginning of aeronautics.
jet transport. There is no collective vision of our future such as the one that
Although the French engineer Lorin had visualized the concept drove us in the past. There is also a need for a more aggressive pool
of jet propulsion more than 25 years earlier, it took improved of airplane design talents to maintain an industry that continues

Downloaded From : www.EasyEngineering.net


Downloaded From : www.EasyEngineering.net

Reading Comprehension  l 393

to find a multibillion dollar–a–year market for its product. Passage- 27


Can the history of aviation technology tell us something
about the future of aeronautics? Have we reached a final state Fifty feet away three male lions lay by the road. They didn’t
appear to have a hair on their heads. Noting the color of their
in our evolution to a mature technology in aeronautics? Are
the challenges of coming out with the ‘better, cheaper, faster’ noses (leonine noses darken as they age, from pink to black),
designs somehow inferior to those that are suited for ‘faster, Craig estimated that they were six years old—young adults. “This
is wonderful!” he said, after staring at them for several moments.
higher, further’? Safety should improve greatly as a result of
“This is what we came to see. They really are maneless.” Craig, a
the forthcoming improvements in airframes, engines, and
professor at the University of Minnesota, is arguably the leading
avionics. Sixty years from now, aircraft will recover on their
expert on the majestic Serengeti lion, whose head is mantled in
own if the pilot loses control. Satellites are the key not only to
long, thick hair. He and Peyton West, a doctoral student who has
GPS (global positioning system) navigation but also to in–flight been working with him in Tanzania, had never seen the Tsavo
communications, uplinked weather, and even in– flight e–mail. lions that live some 200 miles east of the Serengeti. The scientists
Although there is some debate about what type of engines will had partly suspected that the maneless males were adolescents
power future airplanes—lightweight turbines, turbocharged mistaken for adults by amateur observers. Now they knew better.
diesels, or both—there is little debate about how these power The Tsavo research expedition was mostly Peyton’s show. She had
spent several years in Tanzania, compiling the data she needed

ww
plants will be controlled. Pilots of the future can look forward to
more and better on–board safety equipment.
114. According to the first paragraph of the passage, which of the
to answer a question that ought to have been answered long ago:
Why do lions have manes? It’s the only cat, wild or domestic,
that displays such ornamentation. In Tsavo she was attacking the

w.E
following statements is NOT false?
(a) Frank Whittle and Hans von Ohain were the first to
conceive of jet propulsion.
(b) Supersonic fighter planes were first used in the Second
riddle from the opposite angle. Why do its lions not have manes?
(Some “maneless” lions in Tsavo East do have partial manes, but
they rarely attain the regal glory of the Serengeti lions’.) Does
World War.
asy
(c) No man had traveled faster than sound until the 1950s.
environmental adaptation account for the trait? Are the lions
of Tsavo, as some people believe, a distinct subspecies of their
Serengeti cousins?
has been remarkably fast.
En
(d) The exploitation of jet propulsion for supersonic aviation
The Serengeti lions have been under continuous observation for
more than 35 years, beginning with George Schaller’s pioneering
115. What is the fourth paragraph of the passage, starting, “How
much bigger... .”, about?
(a) Stealth, speed, avionics, and agility of new aircraft. gin
work in the 1960s. But the lions in Tsavo, Kenya’s oldest and largest
protected ecosystem, have hardly been studied. Consequently,
legends have grown up around them. Not only do they look
(b) The way aircraft size has been growing.
(c) Use of computer automation in aircraft. eer
different, according to the myths, they behave differently,
displaying greater cunning and aggressiveness. “Remember too,”
(d) Super–jumbo jets that can take more than 500
passengers.
116. What is the most noteworthy difference between V –22 and
of ferocity.”
ing
Kenya: The Rough Guide warns, “Tsavo’s lions have a reputation

Their fearsome image became well-known in 1898, when two


a standard airplane?
(a) It can take off vertically. (b) It has winged flight. .ne
males stalled construction of what is now Kenya Railways by
allegedly killing and eating 135 Indian and African laborers.
(c) It has excellent payload. (d) Its range is very high.
117. Why might radars not be used a decade from now?
(a) Stealth technology will advance so much that it is
pointless to use radar to detect aircraft.
t
A British Army officer in charge of building a railroad bridge
over the Tsavo River, Lt. Col. J. H. Patterson, spent nine months
pursuing the pair before he brought them to bay and killed them.
Stuffed and mounted, they now glare at visitors to the Field
(b) UAVs can remain over hostile territory without any Museum in Chicago. Patterson’s account of the leonine reign of
danger of being detected. terror, The Man-Eaters of Tsavo, was an international best-seller
when published in 1907. Still in print, the book has made Tsavo’s
(c) Computers on board may enable aircraft to manage safe
lions notorious. That annoys some scientists. “People don’t want
navigation on their own. to give up on mythology,” Dennis King told me one day. The
(d) It is not feasible to increase the range of radars. zoologist has been working in Tsavo off and on for four years. “I
118. According to the author, commercial aeronautics, in contrast am so sick of this man-eater business. Patterson made a helluva
to military aeronautics, has declined because, among other lot of money off that story, but Tsavo’s lions are no more likely to
things, turn man-eater than lions from elsewhere.”
(a) speed and technology barriers are more easily overcome But tales of their savagery and wiliness don’t all come from
in military aeronautics. sensationalist authors looking to make a buck. Tsavo lions are
(b) the collective vision of the past continues to drive civil generally larger than lions elsewhere, enabling them to take down
and commercial aeronautics. the predominant prey animal in Tsavo, the Cape buffalo, one of
(c) though the industry has a huge market, it has not the strongest, most aggressive animals of Earth. The buffalo don’t
attracted the right kind of aircraft designers. give up easily: They often kill or severely injure an attacking lion,
(d) there is a shortage of materials, like light weight and a wounded lion might be more likely to turn to cattle and
composites, used in commercial aeronautics. humans for food.

Downloaded From : www.EasyEngineering.net


Downloaded From : www.EasyEngineering.net

394  l  Reading Comprehension

And other prey is less abundant in Tsavo than in other traditional Passage- 28
lion haunts. A hungry lion is more likely to attack humans. Safari
guides and Kenya Wildlife Service rangers tell of lions attacking A game of strategy, as currently conceived in game theory, is a
Land Rovers, raiding camps, stalking tourists. Tsavo is a tough situation in which two or more “players” make choices among
neighborhood, they say, and it breeds tougher lions. available alternatives (moves). The totality of choices determines
But are they really tougher? And if so, is there any connection the outcomes of the game, and it is assumed that the rank order
between their manelessness and their ferocity? An intriguing of preferences for the outcomes is different for different players.
hypothesis was advanced two years ago by Gnoske and Peterhans: Thus the “interests” of the players are generally in conflict.
Tsavo lions may be similar to the unmanned cave lions of the Whether these interests are diametrically opposed or only
Pleistocene. The Serengeti variety is among the most evolved partially opposed depends on the type of game.
of the species—the latest model, so to speak—while certain Psychologically, most interesting situations arise when the
morphological differences in Tsavo lions (bigger bodies, smaller interests of the players are partly coincident and partly opposed,
skulls, and maybe even lack of a mane) suggest that they are because then one can postulate not only a conflict among the
closer to the primitive ancestor of all lions. Craig and Peyton had players but also inner conflicts within the players. Each is torn
serious doubts about this idea, but admitted that Tsavo lions pose between a tendency to cooperate, so as to promote the common
a mystery to science.  interests, and a tendency to compete, so as to enhance his own
119. The book Man-Eaters of Tsavo annoys some scientists individual interests.

ww
because
(a) it revealed that Tsavo lions are ferocious.
Internal conflicts are always psychologically interesting. What
we vaguely call “interesting” psychology is in very great measure
the psychology of inner conflict. Inner conflict is also held to be

w.E
(b) Patterson made a helluva lot of money from the book by
sensationalism.
(c) it perpetuated the bad name Tsavo lions had.
(d) it narrated how two male Tsavo lions were killed.
an important component of serious literature as distinguished
from less serious genres. The classical tragedy, as well as the
serious novel, reveals the inner conflict of central figures. The

asy
120. According to the passage, which of the following has NOT
contributed to the popular image of Tsavo lions as savage
superficial adventure story, on the other hand, depicts only
external conflict; that is, the threats to the person with whom
the reader (or viewer) identifies stem in these stories exclusively
creatures?

En
(a) Tsavo lions have been observed to bring down one of
the strongest and most aggressive animals—the Cape
from external obstacles and from the adversaries who create
them. On the most primitive level this sort of external conflict is
buffalo.
(b) In contrast to the situation in traditional lion haunts, gin
psychologically empty. In the fisticuffs between the protagonists
of good and evil, no psychological problems are involved or, at
any rate, none are depicted in juvenile representations of conflict.
scarcity of non-buffalo prey in the Tsavo makes the
Tsavo lions more aggressive.
(c) The Tsavo lion is considered to be less evolved than the eer
The detective story, the “adult” analogue of a juvenile
adventure tale, has at times been described as a glorification of
Serengeti variety.
(d) Tsavo lions have been observed to attack vehicles as well
as humans.
ing
intellectualized conflict. However, a great deal of the interest
in the plots of these stories is sustained by withholding the
unraveling of a solution to a problem. The effort of solving the
121. The sentence which concludes the first paragraph, “Now
they knew better”, implies that: .ne
problem is in itself not a conflict if the adversary (the unknown
criminal) remains passive, like Nature, whose secrets the scientist
supposedly unravels by deduction. If the adversary actively puts
(a) The two scientists were struck by wonder on seeing
maneless lions for the first time.
(b) Though Craig was an expert on the Serengeti lion, now
he also knew about the Tsavo lions.
t
obstacles in the detective’s path toward the solution, there is
genuine conflict. But the conflict is psychologically interesting
only to the extent that it contains irrational components such
as a tactical error on the criminal’s part or the detective’s insight
(c) Earlier, Craig and West thought that amateur observers into some psychological quirk of the criminal or something of
had been mistaken. this sort. Conflict conducted in a perfectly rational manner is
(d) Craig was now able to confirm that darkening of the psychologically no more interesting than a standard Western.
noses as lions aged applied to Tsavo lions as well. For example, Tic-tac-toe, played perfectly by both players, is
122. Which of the following, if true, would weaken the hypothesis completely devoid of psychological interest.
advanced by Gnoske and Peterhans most? Chess may be psychologically interesting but only to the extent
(a) Craig and Peyton develop even more serious doubts that it is played not quite rationally. Played completely rationally,
about the idea that Tsavo lions are primitive. chess would not be different from Tic-tac-toe.
(b) The maneless Tsavo East lions are shown to be closer to In short, a pure conflict of interest (what is called a zero-sum
the cave lions. game) although it offers a wealth of interesting conceptual
(c) Pleistocene cave lions are shown to be far less violent problems, is not interesting psychologically, except to the extent
than believed. that its conduct departs from rational norms.  (CAT 2005)
(d) The morphological variations in body and skull size 123. According to the passage, internal conflicts are
between the cave and Tsavo lions are found to be psychologically more interesting than external conflicts
insignificant. because

Downloaded From : www.EasyEngineering.net


Downloaded From : www.EasyEngineering.net

Reading Comprehension  l 395

(a) internal conflicts, rather than external conflicts, rocketed to more than $65 a barrel, more than double its level
form an important component of serious literature as 18 months ago. The accepted wisdom is that we shouldn’t worry
distinguished from less serious genres. our little heads about that, because the incentives are there for
(b) only juveniles or very few “adults” actually experience business to build new production and refining capacity, which
external conflict, while internal conflict is more widely will effortlessly bring demand and supply back into balance and
prevalent in society. bring crude prices back to S25 a barrel. As Tommy Cooper used
(c) in situations of internal conflict, individuals experience a to say, ‘just like that’.
dilemma in resolving their own preferences for different Then there is the result of the French referendum on the
outcomes. European Constitution, seen as thick-headed luddites railing
(d) there are no threats to the reader (or viewer) in case of vainly against the modern world. What the French needed to
external conflicts. realise, the argument went, was that there was no alternative to
124. Which, according to the author, would qualify as interesting
the reforms that would make the country more flexible, more
psychology?
competitive, more dynamic. Just the sort of reforms that allowed
(a) A statistician’s dilemma over choosing the best method
Gate Gourmet to sack hundreds of its staff at Heathrow after the
to solve an optimisation problem.
sort of ultimatum that used to be handed out by Victorian mill
(b) A chess player’s predicament over adopting a defensive
strategy against an aggressive opponent. owners. An alternative way of looking at the French non” is that

ww
(c) A mountaineer’s choice of the best path to Mt. Everest
from the base camp.
our neighbours translate “flexibility” as “you’re fired”.
Finally, take a squinl at the United States. Just like Britain a
century ago, a period of unquestioned superiority is drawing to

w.E
(d) A finance manager’s quandary over the best way of
raising money from the market.
125. According to the passage, which of the following options
about me application of game theory to a conflict-of-interest
a close.
China is still a long way from matching America’s wealth, but it
is growing at a stupendous rate and economic strength brings
situation is true?
asy
(a) Assuming that the rank order of preferences for options
geopolitical clout. Already, there is evidence of a new scramble
for Africa as Washington and Beijing compete for oil stocks.
Moreover, beneath the surface of the US economy, all is not
is different for different players.

En
(b) Accepting that the interests of different players are often
in conflict.
well. Growth looks healthy enough, but the competition from
China and elsewhere has meant the world’s biggest economy
(c) Not assuming that the interests are in complete
disagreement. gin
now imports far more than it exports. The US is living beyond its
means, but in this time of studied complacency a current account
deficit worth 6 percent of gross domestic product is seen as a sign
(d) All of the above.
126. The problem solving process of a scientist is different from
that of a detective because eer
of strength, not weakness.
In this new Edwardian summer, comfort is taken from the fact
(a) scientists study inanimate objects, while detectives deal
with living criminals or law offenders. ing
that dearer oil has not had the savage inflationary consequences
of 1973-74, when a fourfold increase in the cost of crude brought
an abrupt end to a postwar boom that had gone on uninterrupted
(b) scientists study known objects, while detectives have to
deal with unknown criminals or law offenders.
(c) scientists study phenomena that are not actively altered, .ne
for a quarter of a century. True, the cost of living has been affected
by higher transport costs, but we are talking of inflation at 2.3 per
while detectives deal with phenomena that have been
deliberately influenced to mislead.
(d) scientists study psychologically interesting phenomena,
while detectives deal with “adult” analogues of juvenile
t
cent and not 27 per cent. Yet the idea that higher oil prices are of
little consequence is fanciful. If people are paying more to fill up
their cars it leaves them with less to spend on everything else, but
there is a reluctance to consume less. In the 1970s unions were
adventure tales. strong and able to negotiate large, compensatory pay deals that
served to intensify inflationary pressure. In 2005, that avenue is
Passage- 29 pretty much closed off, but the abolition of all the controls on
credit that existed in the 1970s means that households are invited
Crinoline and croquet are out. As yet, no political activists have
to borrow more rather than consume less. The knock-on effects
thrown themselves in front of the royal horse on Derby Day. Even
of higher oil prices are thus felt in different ways -through high
so, some historians can spot the parallels. It is a time of rapid
levels of indebtedness, in inflated asset prices, and in balance of
technological change. It is a period when the dominance of the payments deficits.
world’s superpower is coming under threat. It is an epoch when There are those who point out, rightly, that modern industrial
prosperity masks underlying economic strain. And, crucially, capitalism has proved mightily resilient these past 250 years, and
it is a time when policy-makers are confident that all is for the that a sign of the enduring strength of the system has been the
best in the best of all possible worlds. Welcome to the Edwardian way it apparently shrugged off everything -a stock market crash,
Summer of the second age of globalisation. 9/11, rising oil prices -that have been thrown at it in the half
Spare a moment to take stock of what’s been happening in decade since the millennium. Even so, there are at least three
the past few months. Let’s start with the oil price, which has reasons for concern. First, we have been here before. In terms

Downloaded From : www.EasyEngineering.net


Downloaded From : www.EasyEngineering.net

396  l  Reading Comprehension

of political economy, the first era of globalisation mirrored our 130. What can be inferred about the author’s view when he states,
own. There was a belief in unfettered capital flows, in free trade, ‘As Tommy Cooper used to say “just like that’”?
and in the power of the market. It was a time of massive income (a) Industry has incentive to build new production and
inequality and unprecedented migration. Eventually, though, refining capacity and therefore oil prices would reduce.
there was a backlash, manifested in a struggle between free (b) There would be a correction in the price levels of oil
traders and protectionists, and in rising labour militancy. once new production capacity is added.
Second, the world is traditionally at its most fragile at times (c) The decline in oil prices is likely to be short-term in
when the global balance of power is in flux. By the end of the nature.
nineteenth century, Britain’s role as the hegemonic power was (d) It is not necessary that oil prices would go down to
being challenged by the rise of the United States, Germany, and earlier levels.
Japan while the Ottoman and Hapsburg empires were clearly in
rapid decline, Looking ahead from 2005, it is clear that over the
Passage- 30
next two or three decades, both China and India -which together While complex in the extreme, Derrida’s work has proven to be
account for half the world’s population -will flex their muscles. a particularly influential approach to the analysis of the ways in
Finally, there is the question of what rising oil prices tell us. The which language structures our understanding of ourselves and
the world we inhabit, an approach he termed deconstruction.
emergence of China and India means global demand for crude
In its simplest formulation, deconstruction can be taken to

ww
is likely to remain high at a time when experts say production is
about to top out. If supply constraints start to bite, any declines in
the price are likely to be short-term cyclical affairs punctuating a
refer to a methodological strategy which seeks to uncover
layers of hidden meaning in a text that have been denied or
suppressed. The term ‘text’, in this respect, does not refer simply

w.E
long upward trend.
127. By the expression ‘Edwardian Summer’, the author refers to
a period in which there is
to a written form of communication, however. Rather, texts
are something we all produce and reproduce constantly in our
everyday social relations, be they spoken, written or embedded
(a) unparalleled luxury and opulence.
asy
(b) a sense of complacency among people because of all-
round prosperity.
in the construction of material artifacts. At the heart of Derrida’s
deconstructive approach is his critique of what he perceives to
be the totalitarian impulse of the Enlightenment pursuit to bring
(c) a culmination of all-round economic prosperity.
(d) an imminent danger lurking behind economic En all that exists in the world under the domain of a representative
language, a pursuit he refers to as logocentrism. Logocentrism
prosperity.
128. What, according to the author, has resulted in a widespread gin
is the search for a rational language that is able to know and
represent the world and all its aspects perfectly and accurately.
Its totalitarian dimension, for Derrida at least, lies primarily in
belief in the resilience of modern capitalism?
(a) Growth in the economies of Western countries despite
shocks in the form of increase in levels of indebtedness eer
its tendency to marginalize or dismiss all that does not neatly
comply with its particular linguistic representations, a tendency
and inflated asset prices.
(b) Increase in the prosperity of Western countries and ing
that, throughout history, has all too frequently been manifested
in the form of authoritarian institutions. Thus logocentrism has,
in its search for the truth of absolute representation, subsumed
China despite rising oil prices.
(c) Continued growth of Western economies despite a rise
in terrorism, an increase in oil prices and other similar .ne
difference and oppressed that which it designates as its alien
‘other’. For Derrida, western civilization has been built upon such
shocks.
(d) The success of continued reforms aimed at making
Western economies more dynamic, competitive and
efficient.
the name of reason and progress.
t
a systematic assault on alien cultures and ways of life, typically in

In response to logocentrism, deconstruction posits the idea


that the mechanism by which this process of marginalization
and the ordering of truth occurs is through establishing systems
129. Which of the following best represents the key argument of binary opposition. Oppositional linguistic dualisms, such as
made by the author? rational/ irrational, culture/nature and good/bad are not, however,
(a) The rise in oil prices, the flux in the global balance construed as equal partners as they are in, say, the semiological
of power and historical precedents should make us structuralism of Saussure. Rather, they exist, for Derrida, in a
question our belief that the global economic prosperity series of hierarchical relationships with the first term normally
would continue. occupying a superior position. Derrida defines the relationship
(b) The belief that modern industrial capitalism is highly between such oppositional terms using the neologism differance.
resilient and capable of overcoming shocks will be belied This refers to the realization that in any statement, oppositional
soon. terms differ from each other (for instance, the difference between
(c) Widespread prosperity leads to neglect of early signs of rationality and irrationality is constructed through oppositional
underlying economic weakness, manifested in higher oil usage), and at the same time, a hierarchical relationship is
prices and a flux in the global balance of power. maintained by the deference of one term to the other (in the
(d) A crisis is imminent in the West given the growth of positing of rationality over irrationality, for instance). It is this
countries like China and India and the increase in oil latter point which is perhaps the key to understanding Derrida’s
prices. approach to deconstruction. For the fact that at any given time

Downloaded From : www.EasyEngineering.net


Downloaded From : www.EasyEngineering.net

Reading Comprehension  l 397

one term must defer to its oppositional ‘other’, means that the two 134. Derrida rejects the idea of ‘definitive authority of the subject’
terms are constantly in a state of interdependence. The presence because
of one is dependent upon the absence or ‘absent-presence’ (a) interpretation of the text may not make the unseen
of the ‘other’, such as in the case of good and evil, whereby to visible
understand the nature of one, we must constantly relate it to the (b) the meaning of the text is based on binary opposites.
absent term in order to grasp its meaning. That is, to do good, (c) the implicit power relationship is often ignored.
we must understand that our act is not evil for without that (d) any act of interpretation must refer to what the author
comparison the term becomes meaningless. intends.
Put simply, deconstruction represents an attempt to
demonstrate the absent-presence of this oppositional ‘other’, to Passage- 31
show that what we say or write is in itself not expressive simply of
Our propensity to look out for regularities, and to impose
what is present, but also of what is absent. Thus, deconstruction
laws upon nature, leads to the psychological phenomenon of
seeks to reveal the interdependence of apparently dichotomous
dogmatic thinking or, more generally, dogmatic behaviour:
terms and their meanings relative to their textual context;
we expect regularities everywhere and attempt to find them
that is, within the linguistic power relations which structure
even where there are none; events which do not yield to these
dichotomous terms hierarchically. In Derrida’s own words, a
attempts we are inclined to treat as a kind of ‘background noise’;
deconstructive reading “must always aim at a certain relationship,

ww
unperceived by the writer, between what he commands and what
he does not command of the patterns of a language that he uses. .
. .[It] attempts to make the not-seen accessible to sight.”
and we stick to our expectations even when they are inadequate
and we ought to accept defeat. This dogmatism is to some extent
necessary. It is demanded by a situation which can only be dealt

w.E
Meaning, then, is never fixed or stable, whatever the intention
of the author of a text. For Derrida, language is a system of
relations that are dynamic, in that all meanings we ascribe to the
with by forcing our conjectures upon the world. Moreover, this
dogmatism allows us to approach a good theory in stages, by way
of approximations: if we accept defeat too easily, we may prevent

asy
world are dependent not only on what we believe to be present but
also on what is absent. Thus, any act of interpretation must refer
not only to what the author of a text intends, but also to what is
ourselves from finding that we were very nearly right.
It is clear that this dogmatic attitude, which makes us stick
to our first impressions, is indicative of a strong belief; while

En
absent from his or her intention. This insight leads, once again, to
Derrida’s further rejection of the idea of the definitive authority
a critical attitude, which is ready to modify its tenets, which
admits doubt and demands tests, is indicative of a weaker belief.
of the intentional agent or subject. The subject is decentred; it is
conceived as the outcome of relations of differance. As author
of its own biography, the subject thus becomes the ideological gin
Now according to Hume’s theory, and to the popular theory,
the strength of a belief should be a product of repetition; thus
it should always grow with experience, and always be greater in
fiction of modernity and its logocentric philosophy, one that
depends upon the formation of hierarchical dualisms, which eer
less primitive persons. But dogmatic thinking, an uncontrolled
wish to impose regularities, a manifest pleasure in rites and in
repress and deny the presence of the absent ‘other’. No meaning
can, therefore, ever be definitive, but is merely an outcome of a
particular interpretation.  ing
repetition as such, is characteristic of primitives and children;
and increasing experience and maturity sometimes create an
attitude of caution and criticism rather than of dogmatism.
131. According to the passage, Derrida believes that
(a) Reality can be construed only through the use of rational
analysis. .ne
My logical criticism of Hume’s psychological theory, and the
considerations connected with it, may seem a little removed

(b) Language limits our construction of reality.


(c) A universal language will facilitate a common
understanding of reality.
(d) We need to uncover the hidden meaning in a system of
t
from the field of the philosophy of science. But the distinction
between dogmatic and critical thinking, or the dogmatic and the
critical attitude, brings us right back to our central problem. For
the dogmatic attitude is clearly related to the tendency to verify
our laws and schemata by seeking to apply them and to confirm
relations expressed by language. them, even to the point of neglecting refutations, whereas the
132. To Derrida, ‘logocentrism’ does not imply critical attitude is one of readiness to change them – to test them;
(a) A totalitarian impulse. to refute them; to falsify them, if possible. This suggests that we
(b) A domain of representative language. may identify the critical attitude with the scientific attitude, and
(c) Interdependence of the meanings of dichotomous the dogmatic attitude with the one which we have described as
terms. pseudo¬scientific. It further suggests that genetically speaking
(d) A strategy that seeks to suppress hidden meanings in a the pseudo-scientific attitude is more primitive than, and prior
text. to, the scientific attitude: that it is a pre-scientific attitude. And
133. According to the passage, Derrida believes that the system of this primitivity or priority also has its logical aspect. For the
binary opposition critical attitude is not so much opposed to the dogmatic attitude
(a) represents a prioritization or hierarchy. as superimposed upon it: criticism must be directed-against
(b) reconciles contradictions and dualities. existing and influential beliefs in need of critical revision – in
(c) weakens the process of marginalization and ordering of other words, dogmatic beliefs. A critical attitude needs for its raw
truth. material, as it were, theories or beliefs which are held more or less
(d) deconstructs reality. dogmatically. Thus, science must begin with myths, and with the

Downloaded From : www.EasyEngineering.net


Downloaded From : www.EasyEngineering.net

398  l  Reading Comprehension

criticism of myths; neither with the collection of observations, (a) A critical attitude implies endless questioning, and,
nor with the invention of experiments, but with the critical therefore, it cannot lead to strong beliefs.
discussion of myths, and of magical techniques and practices. (b) A critical attitude, by definition, is centred on an analysis
The scientific tradition is distinguished from the pre-scientific of anomalies and “noise”.
tradition in having two layers. (c) A critical attitude leads to questioning everything, and
Like the latter, it passes on its theories; but it also passes on in the process generates “noise” without any conviction.
a critical attitude towards them. The theories are passed on, not (d) A critical attitude is antithetical to conviction, which is
as dogmas, but rather with the challenge to discuss them and required for strong beliefs.
improve upon them. (e) A critical attitude leads to questioning and to tentative
The critical attitude, the tradition of free discussion of theories hypotheses.
with the aim of discovering their weak spots so that they may be 139. According to the passage, which of the following statements
improved upon, is the attitude of reasonableness, of rationality. best describes the difference between science and pseudo-
From the point of view here developed, all laws, all theories, science?
remain essentially tentative, or conjectural, or hypothetical, even (a) Scientific theories or hypothesis are tentatively true
when we feel unable to doubt them any longer. Before a theory whereas pseudo-sciences are always true.
has been refuted we can never know in what way it may have to (b) Scientific laws and theories are permanent and
immutable whereas pseudo-sciences are contingent on

ww
be modified. 
135. In the context of science, according to the passage, the
interaction of dogmatic beliefs and critical attitude can be
the prevalent mode of thinking in a society.
(c) Science always allows the possibility of rejecting a theory

w.E
best described as:
(a) A duel between two warriors in which one has to die.
(b) The effect of a chisel on a marble stone while making a
or hypothesis, whereas pseudo-sciences seek to validate
their ideas or theories.
(d) Science focuses on anomalies and exceptions so that
fundamental truths can be uncovered, whereas pseudo-
sculpture.

asy
(c) The feedshare (natural gas) in fertilizer industry being
transformed into fertilizers.
sciences focus mainly on general truths.
(e) Science progresses by collection of observations or


(d) A predator killing its prey.
(e) The effect of fertilizers on a sapling.
En by experimentation, whereas pseudo-sciences do not
worry about observations and experiments.
136. According to the passage, the role of a dogmatic attitude or
dogmatic behaviour in the development of science is
(a) critical and important, as, without it, initial hypotheses gin Passage- 32
My aim is to present a conception of justice which generalizes
or conjectures can never be made.
(b) positive, as conjectures arising out of our dogmatic eer
and carries to a higher level of abstraction the familiar theory
of the social contract. In order to do this we are not to think of
attitude become science.
(c) negative, as it leads to pseudo-science.
(d) neutral, as the development of science is essentially ing
the original contract as one to enter a particular society or to
set up a particular form of government. Rather, the idea is that
the principles of justice for the basic structure of society are the
because of our critical attitude.
(e) inferior to critical attitude, as a critical attitude leads to .ne
object of the original agreement. They are the principles that free
and rational persons concerned to further their own interests
the attitude of reasonableness and rationality.
137. Dogmatic behaviour, in this passage, has been associated
with primitives and children. Which of the following best
describes the reason why the author compares primitives
t
would accept in an initial position of equality. These principles
are to regulate all further agreements; they specify the kinds of
social cooperation that can be entered into and the forms of
government that can be established. This way of regarding the
with children? principles of justice, I shall call justice as fairness. Thus, we are
(a) Primitives are people who are not educated, and hence to imagine that those who engage in social cooperation choose
can be compared with children, who have not yet been
together, in one joint act, the principles which are to assign
through school.
basic rights and duties and to determine the division of social
(b) Prim itives are people who, though not modern, are as
innocent as children. benefits. Just as each person must decide by rational reflection
(c) Primitives are people without a critical attitude, just as what constitutes his good, that is, the system of ends which it
children are. is rational for him to pursue, so a group of persons must decide
(d) Primitives are people in the early stages of human once and for all what is to count among them as just and unjust.
evolution; similarly, children are in the early stages of The choice which rational men would make in this hypothetical
their lives. situation of equal liberty determines the principles of justice.
(e) Primitives are people who are not civilized enough, just In ‘justice as fairness’, the original position is not an actual
as children are not. historical state of affairs. It is understood as a purely hypothetical
138. Which of the following statements best supports the situation characterized so as to lead lo a certain conception of
argument in the passage that a critical attitude leads to a justice. Among the essential features of this situation is that no
weaker belief than a dogmatic attitude does? one knows his place in society, his class position or social status,

Downloaded From : www.EasyEngineering.net


Downloaded From : www.EasyEngineering.net

Reading Comprehension  l 399

nor anyone know his fortune in the distribution of natural assets (b) A hypothetical situation in which every individual is
and abilities, his intelligence, strength, and the like. I shall even equal and no individual enjoys any privilege based on
assume that the parties do not know their conceptions of the the existing positions and powers.
good or their special psychological propensities. The principles (c) A hypothetical situation to ensure fairness of agreements
of justice are chosen behind a veil of ignorance. This ensures that among individuals in society.
no one is advantaged or disadvantaged in the choice of principles (d) An imagined situation in which principles of justice
by the outcome of natural chance or the contingency of social would have to be fair.
(e) An imagined situation in which fairness is the objective
circumstances. Since all are similarly situated and no one is able to
of the principles of justice to ensure that no individual
design principles to favor his particular condition, the principles
enjoys any privilege based on the existing positions and
of justice are the result of a fair agreement or bargain. Justice
powers.
as fairness begins with one of the most general of all choices 142. Which of the following best illustrates the situation that is
which persons might make together, namely, with the choice of equivalent to choosing ‘the principles of justice’ behind a
the first principles of a conception of justice which is to regulate ‘veil of ignorance’?
all subsequent criticism and reform of institutions. Then, having (a) The principles of justice are chosen by businessmen,
chosen a conception of justice, we can suppose that they are to who are marooned on an uninhabited island after a
choose a constitution and a legislature to enact laws, and so on, shipwreck, but have some possibility of returning.

ww
all in accordance with the principles of justice initially agreed
upon. Our social situation is just if it is such that by this sequence
of hypothetical agreements we would have contracted into the
(b) The principles of justice are chosen by a group of school
children whose capabilities are yet to develop.
(c) The principles of justice are chosen by businessmen,

w.E
general system of rules which defines it. Moreover, assuming
that the original position does determine a set of principles, it
will then be true that whenever social institutions satisfy these
who are marooned on an uninhabited island after a
shipwreck and have no possibility of returning.
(d) The principles of justice are chosen assuming that such

asy
principles, those engaged in them can say to one another that
they are cooperating on terms to which they would agree if they
were free and equal persons whose relations with respect to one
principles will govern the lives of the rule makers only
in their next birth if the rule makers agree that they will
be born again.
another were fair. They could all view their arrangements as
meeting the stipulations which they would acknowledge in an En (e) The principles of justice are chosen by potential
immigrants who are unaware of the resources necessary
initial situation that embodies widely accepted and reasonable
constraints on the choice of principles.
The general recognition of this fact would provide the basis
gin to succeed in a foreign country.
143. Why, according to the passage, do principles of justice need

for a public acceptance of the corresponding principles of justice.


No society can, of course, be a scheme of cooperation which men eer
to be based on an original agreement?
(a) Social institutions and laws can be considered fair only
if they conform to principles of justice.
enter voluntarily in a literal sense; each person finds himself
placed at birth in some particular position in some particular
society, and the nature of this position materially affects his ing
(b) Social institutions and laws can be fair only if they
are consistent with the principles of justice as initially
agreed upon.
life prospects. Yet a society satisfying the principles of justice
as fairness comes as close as a society can to being a voluntary .ne
(c) Social institutions and laws need to be fair in order to be
just.
scheme, for it meets the principles which free and equal persons
would assent to under circumstances that are fair. 
140. A just society, as conceptualized in the passage, can be best
described as
t
(d) Social institutions and laws evolve fairly only if they
are consistent with the principles of justice as initially
agreed upon.
(e) Social institutions and laws conform to the principles of
(a) A Utopia in which everyone is equal and no one enjoys justice as initially agreed upon.
any privilege based on their existing positions and 144. Which of the following situations best represents the idea of
powers. justice as fairness, as argued in the passage?
(b) A hypothetical society in which people agree upon (a) All individuals are paid equally for the work they do.
principles of justice which are fair. (b) Everyone is assigned some work for his or her livelihood.
(c) A society in which principles of justice are not based on (c) All acts of theft are penalized equally.
the existing positions and powers of the individuals.* (d) All children are provided free education in similar
(d) A society in which principles of justice are fair to all. schools.
(e) A hypothetical society in which principles of justice are (e) All individuals are provided a fixed sum of money to
not based on the existing positions and powers of the take care of their health.
individuals.
141. The original agreement or original position in the passage Passage- 33
has been used by the author as
(a) A hypothetical situation conceived to derive principles Human Biology does nothing to structure human society. Age
of justice which are not influenced by position, status may enfeeble us all, but cultures vary considerably in the prestige
and condition of individuals in the society.* and power they accord to the elderly. Giving birth is a necessary

Downloaded From : www.EasyEngineering.net


Downloaded From : www.EasyEngineering.net

400  l  Reading Comprehension

condition for being a mother, but it is not sufficient. We expect (e) Reciprocal roles determine normative human behavior
mothers to behave in maternal ways and to display appropriately in society.
maternal sentiments. We prescribe a clutch of norms or rules that 146. Which of the following would have been true if biological
govern the role of a mother. That the social role is independent linkages structured human society?
of the biological base can be demonstrated by going back three (a) The role of mother would have been defined through her
sentences. Giving birth is certainly not sufficient to be a mother reciprocal relationship with her children.
but, as adoption and fostering show, it is not even necessary! The (b) We would not have been offended by the father playing
fine detail of what is expected of a mother or a father or a dutiful his role ‘tongue in cheek’.
son differs from culture to culture, but everywhere behaviour (c) Women would have adopted and fostered children
is coordinated by the reciprocal nature of roles. Husbands and rather than giving birth to them.
wives, parents and children, employers and employees, waiters (d) Even if warlords were physically weaker than their
and customers, teachers and pupils, warlords and followers; each followers, they would still dominate them.
makes sense only in its relation to the other. The term ‘role’ is an (e) Waiters would have stronger motivation to serve their
appropriate one, because the metaphor of an actor in a play neatly customers.
expresses the rule-governed nature or scripted nature of much of 147. It has been claimed in the passage that “some roles are more
social life and the sense that society is a joint production. Social absorbing than others”. According to passage, which of the

ww
life occurs only because people play their parts (and that is as
true for war and conflicts as for peace and love) and those parts
make sense only in the context of the overall show. The drama
following seem(s) appropriate reason(s) for such a claim?
A. Some roles carry great expectations from the society
preventing manifestation of the true self.

w.E
metaphor also reminds us of the artistic licence available to the
players. We can play a part straight or, as the following from J.P.
Sartre conveys, we can ham it up.
B. Society ascribes so much importance to some roles that
the conception of self may get aligned with the roles
being performed.

asy
Let us consider this waiter in the cafe. His movement is quick
and forward, a little too precise, a little too rapid. He comes
towards the patrons with a step a little too quick. He bends
C. Some roles require development of skill and expertise
leaving little time for manifestation of self.
(a) A only (b) B only

En
forward a little too eagerly; his voice, his eyes express an interest
a little too solicitous for the order of the customer. Finally there
(c) C only
(e) B & C
(d) A & B

he returns, trying to imitate in his walk the inflexible stiffness


of some kind of automaton while carrying his tray with the
recklessness of a tightrope-walker....All his behaviour seems to us gin Passage- 34
Every civilized society lives and thrives on a silent but profound
a game....But what is he playing? We need not watch long before
we can explain it: he is playing at being a waiter in a cafe. The
American sociologist Erving Goffman built an influential body
eer
agreement as to what is to be accepted as the valid mould of
experience. Civilization is a complex system of dams, dykes, and

of social analysis on elaborations of the metaphor of social life as


drama. Perhaps his most telling point was that it is only through ing
canals warding off, directing, and articulating the influx of the
surrounding fluid element; a fertile fenland, elaborately drained
and protected from the high tides of chaotic, unexercised, and
acting out a part that we express character. It is not enough to be
evil or virtuous; we have to be seen to be evil or virtuous. There
is distinction between the roles we play and some underlying .ne
inarticulate experience. In such a culture, stable and sure of itself
within the frontiers of ‘naturalized’ experience, the arts wield
self. Here we might note that some roles are more absorbing
than others. We would not be surprised by the waitress who plays
the part in such a way as to signal to us that she is much more
than her occupation. We would be surprised and offended by
t
their creative power not so much in width as in depth. They do
not create new experience, but deepen and purify the old. Their
works do not differ from one another like a new horizon from a
new horizon, but like a madonna from a madonna.
The periods of art which are most vigorous in creative passion
the father who played his part ‘tongue in cheek’. Some roles are
seem to occur when the established pattern of experience loosens
broader and more far-reaching than others. Describing someone
its rigidity without as yet losing its force. Such a period was the
as a clergyman or faith healer would say far more about that
Renaissance, and Shakespeare its poetic consummation. Then it
person than describing someone as a bus driver. 
was as though the discipline of the old order gave depth to the
145. What is the thematic highlight of this passage?
excitement of the breaking away, the depth of job and tragedy, of
(a) In the absence of strong biological linkages, reciprocal
incomparable conquests and irredeemable losses.
roles provide the mechanism for coordinating human
Adventurers of experience set out as though in lifeboats to
behaviour.
rescue and bring back to the shore treasures of knowing and
(b) In the absence of reciprocal roles, biological linkages feeling which the old order had left floating on the high seas. The
provide the mechanism for coordinating human works of the early Renaissance and the poetry of Shakespeare
behaviour. vibrate with the compassion for live experience in danger of dying
(c) Human behaviour is independent of biological linkages from exposure and neglect. In this compassion was the creative
and reciprocal roles. genius of the age. Yet, it was a genius of courage, not of desperate
(d) Human behaviour depends on biological linkages and audacity. For, however elusively, it still knew of harbours and
reciprocal roles. anchors, of homes to which to return, and of barns in which to

Downloaded From : www.EasyEngineering.net


Downloaded From : www.EasyEngineering.net

Reading Comprehension  l 401

store the harvest. The exploring spirit of art was in the depths of instrumental applications. These are the community’s paradigms,
its consciousness still aware of a scheme of things into which to revealed in its textbooks, lectures, and laboratory exercises. By
fit its exploits and creations. But the more this scheme of things studying them and by practicing with them, the members of the
loses its stability, the more boundless and uncharted appears the corresponding community learn their trade. The historian, of
ocean of potential exploration. In the blank confusion of infinite course, will discover in addition a penumbral area occupied by
potentialities flotsam of significance gets attached to jetsam of achievements whose status is still in doubt, but the core of solved
experience; for everything is sea, everything is at sea -.... The sea problems and techniques will usually be clear. Despite occasional
is all about us; The sea is the land’s edge also, the granite Into ambiguities, the paradigms of a mature scientific community can
which it reaches, the beaches where it tosses Its hints of earlier be determined with relative ease.
and other creation ... -and Rilke tells a story in which, as in T.S. That demands a second step and one of a somewhat different
Eliot’s poem, it is again the sea and the distance of ‘other creation’ kind. When undertaking it, the historian must compare the
that becomes the image of the poet’s reality. community’s paradigms with each other and with its current
A rowing boat sets out on a difficult passage. The oarsmen research reports. In doing so, his object is to discover what isolable
labour in exact rhythm. There is no sign yet of the destination. elements, explicit or implicit, the members of that community may
Suddenly a man, seemingly idle, breaks out into song. And if the have abstracted from their more global paradigms and deploy it
labour of the oarsmen meaninglessly defeats the real resistance as rules in their research. Anyone who has attempted to describe
of the real waves, it is the idle single who magically conquers the

ww
despair of apparent aimlessness.
While the people next to him try to come to grips with the
element that is next to them, his voice seems to bind the boat to
or analyze the evolution of a particular scientific tradition will
necessarily have sought accepted principles and rules of this sort.
Almost certainly, he will have met with at least partial success.

w.E
the farthest distance so that the farthest distance draws it towards
itself. ‘I don’t know why and how,’ is Rilke’s conclusion, ‘but
suddenly I understood the situation of the poet, his place and
But, if his experience has been at all like my own, he will have
found the search for rules both more difficult and less satisfying
than the search for paradigms. Some of the generalizations he
employs to describe the community’s shared beliefs will present

asy
function in this age. It does not matter if one denies him every
place -except this one. There one must tolerate him.’
148. In the passage, the expression “like a madonna from a
more problems. Others, however, will seem a shade too strong.
Phrased in just that way, or in any other way he can imagine, they

madonna” alludes to
(a) The difference arising as a consequence of artisticEn would almost certainly have been rejected by some members
of the group he studies. Nevertheless, if the coherence of the
research tradition is to be understood in terms of rules, some
license.
(b) The difference between two artistic interpretations. gin
specification of common ground in the corresponding area is
needed. As a result, the search for a body of rules competent to
(c) The difference between ‘life’ and ‘interpretation of life’.
(d) The difference between ‘width’ and ‘depth’ of creative
power. eer
constitute a given normal research tradition becomes a source
of continual and deep frustration. Recognizing that frustration,
however, makes it possible to diagnose its source. Scientists can
(e) The difference between the legendary character and the
modern day singer.
149. The sea and ‘other creation’ leads Rilke to
ing
agree that a Newton, Lavoisier, Maxwell, or Einstein has produced
an apparently permanent solution to a group of outstanding

(a) Define the place of the poet in his culture.


(b) Reflect on the role of the oarsman and the singer. .ne
problems and still disagree, sometimes without being aware of
it, about the particular abstract characteristics that make those
solutions permanent. They can, that is, agree in their identification

150.
(c) Muse on artistic labour and its aim lessens.
(d) Understand the elements that one has to deal with.
(e) Delve into natural experience and real waves.
According to the passage, the term “adventurers of
t
of a paradigm without agreeing on, or even attempting to produce,
a full interpretation or rationalization of it.
Lack of a standard interpretation or of an agreed reduction to
rules will not prevent a paradigm from guiding research. Normal
experience” refers to science can be determined in part by the direct inspection of
(a) Poets and artists who are driven by courage. paradigms, a process that is often aided by but does not depend
(b) Poets and artists who create their own genre. upon the formulation of rules and assumption. Indeed, the
(c) Poets and artists of the Renaissance. existence of a paradigm need not even imply that any full set of
(d) Poets and artists who revitalize and enrich the past for rules exists. 
us. 151. What is the author attempting to illustrate through this
(e) Poets and artists who delve in flotsam and jetsam in sea. passage?
(a) Relationships between rules, paradigms, and normal
Passage- 35 science
To discover the relation between rules, paradigms, and normal (b) How a historian would isolate a particular ‘loci of
science, consider first how the historian isolates the particular commitment’
loci of commitment that have been described as accepted rules. (c) How a set of shared beliefs evolve in to a paradigm.
Close historical investigation of a given specialty at a given time (d) Ways of understanding a scientific tradition
discloses a set of recurrent and quasi-standard illustrations (e) The frustrations of attempting to define a paradigm of a
of various theories in their conceptual, observational, and tradition

Downloaded From : www.EasyEngineering.net


Downloaded From : www.EasyEngineering.net

402  l  Reading Comprehension

152. The term ‘loci of commitment’ as used in the passage would literature of France—from the writings of Georges Bataille,
most likely correspond with which of the following? Jean Genet, and Jean-Paul Sartre to the bleak emptiness of the
(a) Loyalty between a group of scientists in a research nouveau roman.
laboratory Of course, there were great artists who tried to rescue beauty
(b) Loyalty between groups of scientists across research from the perceived disruption of modern society—as T. S.
laboratories Eliot tried to recompose, in Four Quartets, the fragments he
(c) Loyalty to a certain paradigm of scientific inquiry had grieved over in The Waste Land. And there were others,
(d) Loyalty to global patterns of scientific inquiry particularly in America, who refused to see the sordid and
(e) Loyalty to evolving trends of scientific inquiry the transgressive as the truth of the modern world. For artists
153. The author of this passage is likely to agree with which of the like Hopper, Samuel Barber, and Wallace Stevens, ostentatious
following? transgression was mere sentimentality, a cheap way to stimulate
(a) Paradigms almost entirely define a scientific tradition. an audience, and a betrayal of the sacred task of art, which is to
(b) A group of scientists investigating a phenomenon would magnify life as it is and to reveal its beauty—as Stevens reveals
benefit by defining a set of rules. the beauty of “An Ordinary Evening in New Haven” and Barber
(c) Acceptance by the giants of a tradition is a sine qua non that of Knoxville: Summer of 1915. But somehow those great life-
for a paradigm to emerge. affirmers lost their position at the forefront of modern culture.
(d) Choice of isolation mechanism determines the types of So far as the critics and the wider culture were concerned, the

ww
paradigm that may emerge from a tradition.
(e) Paradigms are a general representation of rules and
beliefs of a scientific tradition.
pursuit of beauty was at the margins of the artistic enterprise.
Qualities like disruptiveness and immorality, which previously
signified aesthetic failure, became marks of success; while the

w.E Passage- 36
At any time between 1750 and 1930, if you had asked an educated
pursuit of beauty became a retreat from the real task of artistic
creation. This process has been so normalized as to become a
critical orthodoxy, prompting the philosopher Arthur Danto to
argue recently that beauty is both deceptive as a goal and in some

asy
person to describe the goal of poetry, art, or music, “beauty”
would have been the answer. And if you had asked what the point
of that was, you would have learned that beauty is a value, as
way antipathetic to the mission of modern art. Art has acquired
another status and another social role.
important in its way as truth and goodness, and indeed hardly
distinguishable from them. Philosophers of the Enlightenment En 154. Which one of the following is the main idea of the passage?
(a) Beauty’s rescue from modernity.
(b) The subversion of art.
saw beauty as a way in which lasting moral and spiritual values
acquire sensuous form. And no Romantic painter, musician, or
writer would have denied that beauty was the final purpose of
gin (c) The post modernist’s view of art.
(d) The violation of beauty.
his art.
At some time during the aftermath of modernism, beauty eer
(e) The social role of art.
155. According to the information in the passage, each of the
following is an accurate statement about the old theory of
ceased to receive those tributes. Art increasingly aimed to disturb,
subvert, or transgress moral certainties, and it was not beauty
but originality—however achieved and at whatever moral cost— ing
beauty EXCEPT:
(a) The goal of poetry art and music was beauty.
that won the prizes. Indeed, there arose a widespread suspicion
of beauty as next in line to kitsch—something too sweet and
(b) Beauty was equated with value.

.ne
(c) Beauty was equivalent to truth and goodness.
(d) Art relied on shocks administered to our failing faith in
inoffensive for the serious modern artist to pursue. In a seminal
essay—“Avant-Garde and Kitsch,” published in Partisan Review
in 1939—critic Clement Greenberg starkly contrasted the avant-
garde of his day with the figurative painting that competed with
it, dismissing the latter (not just Norman Rockwell, but greats like

human nature.
(e) Beauty was the final purpose of art. t
156. The author’s attitude as it is revealed in the language used is:
(a) Dejection. (b) Reasoning.
Edward Hopper) as derivative and without lasting significance. (c) Approval. (d) Condemnation
The avant-garde, for Greenberg, promoted the disturbing and the (e) Investigative.
provocative over the soothing and the decorative, and that was 157. A suitable title for the above passage is
why we should admire it. (a) In exploration of beauty.
The value of abstract art, Greenberg claimed, lay not in beauty (b) The ruin of pure art.
but in expression. This emphasis on expression was a legacy of the (c) The desecration of beauty.
Romantic movement; but now it was joined by the conviction that (d) The role of pure beauty.
the artist is outside bourgeois society, defined in opposition to it, (e) The debacle of beauty.
so that artistic self-expression is at the same time a transgression
of ordinary moral norms. We find this posture overtly adopted in Passage- 37
the art of Austria and Germany between the wars—for example, The return of capitalism is haunting the world. Over the past
in the paintings and drawings of Georg Grosz, in Alban Berg’s six months, politicians, businessmen and pundits have been
opera Lulu (a loving portrait of a woman whose only discernible convinced that we are in the midst of a crisis of capitalism that
goal is moral chaos), and in the seedy novels of Heinrich Mann. will require a massive transformation and years of pain to fix.
And the cult of transgression is a leading theme of the postwar Nothing will ever be the same again. “Another ideological god

Downloaded From : www.EasyEngineering.net


Downloaded From : www.EasyEngineering.net

Reading Comprehension  l 403

has failed,” the dean of financial commentators, Martin Wolf, (c) talk about the failure of ideology.
wrote in the Financial Times. Companies will “fundamentally (d) showcase the failure of capitalism.
reset” the way they work, said the CEO of General Electric, Jeffrey (e) discuss the possibilities of containing the damage.
Immelt. “Capitalism will be different,” said Treasury Secretary 159. It can be inferred that the economic system:
Timothy Geithner. (a) Is back on its feet.
No economic system ever remains unchanged, of course, and (b) Has received a great boost.
certainly not after a deep financial collapse and a broad global (c) Has stabilized to a great extent.
recession. But over the past few months, even though we’ve (d) Has been controlled because of measures taken by the
had an imperfect stimulus package, nationalized no banks and various states.
undergone no grand reinvention of capitalism, the sense of (e) Has seen large scale changes.
panic seems to be easing. Perhaps this is a mirage—or perhaps 160. It can be inferred that this passage is an extract from
the measures taken by states around the world, chiefly the U.S. (a) an article in a magazine on capitalism
government, have restored normalcy. Every expert has a critique (b) a speech by an economist on the role played by capitalism
of specific policies, but over time we might see that faced with the across history
decision to underreact or overreact, most governments chose the (c) a review of a book which deals with economics
latter. That choice might produce new problems in due course—a (d) a book on the recession.
(e) an article in a magazine on economics
breakdown.
ww
topic for another essay—but it appears to have averted a systemic

There is still a long road ahead. There will be many more


161. Why, according to the author, will it be difficult to punish the
erring American bankers?
(a) Because stability will bring in forgiveness.

w.E
bankruptcies. Banks will have to slowly earn their way out of
their problems or die. Consumers will save more before they
start spending again. Mountains of debt will have to be reduced.
American capitalism is being rebalanced, reregulated and thus
(b) Because pain cannot bring change.
(c) Because solutions are more important.
(d) Because financial systems are complex.

asy
restored. In doing so it will have to face up to long-neglected
problems, if this is to lead to a true recovery, not just a brief
(e) Because markets do not believe in morality.

Passage- 38
reprieve.

En
Many experts are convinced that the situation cannot improve
yet because their own sweeping solutions to the problem have
Over the last several days, two pieces attacking the realist
approach to Russia were published in prominent media outlets
not been implemented. Most of us want to see more punishment
inflicted, particularly on America’s bankers. Deep down we all gin
in the United States and Russia. One, co-authored by Lev
Gudkov of the Levada Center, Igor Klyamkin, vice president of
have a Puritan belief that unless they suffer a good dose of pain,
they will not truly repent. In fact, there has been much pain,
especially in the financial industry, where tens of thousands of eer
the Liberal Mission Foundation, Georgy Satarov, president of
the Russian NGO the Indem Foundation and Lilia Shevtsova,
a senior associate at the Carnegie Moscow Center was featured
jobs, at all levels, have been lost. But fundamentally, markets are
not about morality. They are large, complex systems, and if things ing
on the editorial page of the Washington Post. The other, by
Andrei Piontkovsky, a visiting fellow at the Hudson Institute, was
released in the Moscow Times.
get stable enough, they move on.
Consider our track record over the past 20 years, starting
with the stock-market crash of 1987, when on Oct. 19 the Dow .ne
I read these pieces concerning the moves to improve
relations between America and Russia with a profound feeling
Jones lost 23 percent, the largest one-day loss in its history. The
legendary economist John Kenneth Galbraith wrote that he just
hoped that the coming recession wouldn’t prove as painful as the
Great Depression. It turned out to be a blip on the way to an even
recommendations of the Hart-Hagel Commission or statesment
of depression. This is not just because there is something bizarre
and twisted about pro-Western Russian liberals attacking the

such as Henry Kissinger and James Baker. It is also because


their criticism serves as a mouthpiece for the agendas of the
bigger, longer boom. Then there was the 1997 East Asian crisis,
most bitterly anti-Russian and geopolitically aggressive liberal
during the depths of which Paul Krugman wrote in a Fortune
interventionists and neocons who help maintain tensions
cover essay, “Never in the course of economic events—not even
between Russia and the West—and actually between the United
in the early years of the Depression—has so large a part of the States and the rest of the world.
world economy experienced so devastating a fall from grace.” And these tensions are extremely damaging to any hopes of
He went on to argue that if Asian countries did not adopt his the long-term liberalization and Westernization of Russia which
radical strategy—currency controls—”we could be looking these liberals want to further. Do Piontkovsky, Shevtsova and
at?.?.?.?the kind of slump that 60 years ago devastated societies, the others seriously think that the U.S.-Russian rivalry in the
destabilized governments, and eventually led to war.” Only one Caucasus, and the war over South Ossetia which resulted, helped
Asian country instituted currency controls, and partial ones at the cause of liberalism in Russia? Do they ever actually talk to
that. All rebounded within two years. any ordinary Russians, one wonders? Or do their duties briefing
158. The author mentions the details about the current crisis in Americans simply leave them no time for this?
order to: My depression is also because Russia does in fact desperately
(a) detail his acute sense of observation. need a strong liberal movement which can influence the
(b) describe the reason for the crisis. state in a positive direction. Thus figures like Igor Yurgens, a

Downloaded From : www.EasyEngineering.net


Downloaded From : www.EasyEngineering.net

404  l  Reading Comprehension

leading businessman and adviser to President Medvedev, are 165. Why does the author use the axiom about people living in
playing an extremely valuable role in resisting moves to further glass houses?
authoritarianism, centralization and nationalization in response (a) To draw similarities between the current and the past
to the economic crisis. They could do much better if they had situation.
bigger support within the population at large. (b) To highlight the American benevolence toward the two
Tragically however, many Russian liberals in the 1990s— Russian writers.
through the policies they supported and the arrogant contempt (c) To present the malaise existing in the Russian system.
they showed towards the mass of their fellow Russians—made
(d) To challenge the allegations made by the writers.
liberals unelectable for a generation or more across most of
(e) To discredit the theory presented by the writers.
Russia; and to judge by these and other writings of liberals like
the ones under discussion, they have learnt absolutely nothing Passage- 39
from this experience. They think that they form some kind of
opposition to the present Russian establishment. In fact, they are In 1802 Georg W.F. Hegel wrote an impassioned treatise on faith
such an asset to Putin in terms of boosting public hostility to and reason, articulating the major philosophical conflict of the
Russian liberalism that if they hadn’t already existed, Putin might day. Among European intellectual circles, the Enlightenment
have been tempted to invent them. credo, which celebrated the “sovereignty of reason,” had recently
Two aspects of their approach are especially noteworthy. The triumphed. From that standpoint, human intellect was a self-

ww
first is the profoundly illiberal—even McCarthyite—way in which
Piontkovsky tries to disqualify views with which he disagrees by
suggesting that they are motivated purely by personal financial
sufficient measure of the true, the just, and the good. The outlook’s
real target, of course, was religion, which the philosophes viewed
as the last redoubt of delusion and superstition. Theological

w.E
gain, rather than conviction. Where, one wonders, would this
leave all those Russian liberals, and U.S. think tanks, which took
money from Mikhail Khodorkovsky and other Russian oligarchs
claims, they held, could only lead mankind astray. Once the last
ramparts of unreason were breached — our mental Bastilles, as it
were — sovereign reason would take command and, presumably,

asy
in the past? Where would it leave those U.S. officials linked to
leading U.S. private financial companies whose shares benefited
so magnificently from the plundering of Russia in the 1990s?
human perfection would not be long in coming.
Soon legions of skeptics and naysayers emerged to cast doubt
on the Enlightenment’s presumptuous self-conceit. By making
Where, indeed, does it leave Russians—like two of the writers
under discussion—who draw their salaries from U.S. think En the lowly human intellect the measure of all truth, weren’t the
philosophes arbitrarily isolating humanity from the possibility
tanks? Actually, I do believe that most are motivated by sincere
conviction—but all the same; they would do well to remember
the old adage about people who live in glass houses.
gin
of attaining a higher order of truth? Who would really want
to inhabit a totally enlightened universe, denuded of mystery,
plurality, and sublimity? What if ultimate reality weren’t
162. Why does the author regard the attack on Russia as
unfortunate? eer
attainable by the prosaic methods of cognition or secular reason?
What if, instead, the Absolute had more to do with the faculties
(a) There is something odd about it.
(b) It fuels anti Russia agenda.
(c) It adds to the tension with the West.
ing
of the imagination, intuition, or the unfathomable mysteries of
the human unconscious?
A cursory glance at the major cultural divide of our day
(d) It affects all the Russians deeply.
(e) It detracts from hopes of Westernization. .ne
suggests that, in many respects, we haven’t gotten much beyond
the landmark dispute between faith and reason that separated
163. What is the author’s opinion about criticism as a hurdle in
the way of liberalism?
(a) It is an exaggeration since liberal movements do not
require outside support.
t
the leading lights in Hegel’s time. For with the notable exception
of Western Europe, on nearly every continent, religion seems to
have found its second wind. And it would be difficult to deny that
this global revival of spirituality has occurred in pointed reaction
(b) It is unreasonable as censure brings in despondency in to the broken promises of enlightened modernity. Nineteenth-
the masses. century utopians like Charles Fourier speculated that, once
(c) It is false as the legitimacy of the leaders is doubtful. industrial society was perfected, rivers and lakes would pulsate
(d) It is a historical fallacy since many states have been with lemonade, public fountains would overflow with salmon,
liberated through the efforts of their leaders. men would learn to fly, and wild beasts would do our hunting.
(e) It is inaccurate as criticism is a healthy requirement for Instead, as we confront on a daily basis the dislocations of
survival. Western modernity — teeming cities, urban blight, industrially
164. What, according to the author, does the line “Putin might scarred landscapes, massive pollution, and climate change
have been tempted to invent them”, mean? of eschatological proportions — it seems as though Oswald
Spengler’s Decline of the West was more clairvoyant than
(a) That Putin would frame new laws for the liberals.
Fourier’s odes to universal harmony.
(b) That Putin has been ultimately helped by the liberals.
Prominent secularization theorists like Peter L. Berger who,
(c) That Putin has seen through the liberals’ game.
as recently as the 1960s, openly conceded religion’s demise, are
(d) That the liberals are unable to decipher the damage
having to radically alter their forecasts. They have had to invent
caused by them. new concepts and categories to describe the phenomenon of
(e) That the liberals are cutting their own feet. religion’s unexpected global resurgence. The philosopher Jurgen

Downloaded From : www.EasyEngineering.net


Downloaded From : www.EasyEngineering.net

Reading Comprehension  l 405

Habermas now felicitously refers to the advent of a “postsecular 169. What, according to the passage, was the chief argument in
society” to characterize religiosity’s remarkable staying power. In Hegel’s time?
recent works such as Between Naturalism and Religion (2008), (a) That reason should be held supreme.
he questions whether modern societies possess the moral (b) That reason would help in perfecting mankind.
resources to persevere without relying on their religious roots (c) That faith and reason could co-exist side by side.
— the Judeo-Christian basis of secular ethics, for example. And (d) That religion was unimportant.
Berger himself, who was once secularization theory’s most vocal (e) That religion helped the development of intellect.
proponent, has expressed his change of heart in a book title, The
Desecularization of the World (1999). Passage -40
Today academe is rife with discussions of “political theology,” Humans communicate with one another using a dazzling array
a term popularized during the 1920s by the German jurist of languages, each differing from the next in innumerable ways.
Carl Schmitt. Schmitt meant by it that all modern political Do the languages we speak shape the way we see the world, the
concepts — sovereignty, natural rights, the social contract — are way we think, and the way we live our lives? Do people who
secularized versions of theological concepts. He sought to call speak different languages think differently simply because they
into question the legitimacy of the modern age, which in his speak different languages? Does learning new languages change
view fed parasitically off of a nobler theological past. Along the the way you think? Do polyglots think differently when speaking

ww
same lines, two weighty anthologies edited by the Johns Hopkins
philosopher Hent de Vries have stressed the centrality of political
theology for comprehending the impasse of the political present,
different languages?
These questions touch on nearly all of the major controversies

w.E
defined in terms of the sordid triumph of neoliberalism and
globalization.
The resurgence of political theology suggests that the
in the study of mind. They have engaged scores of philosophers,
anthropologists, linguists, and psychologists, and they have
important implications for politics, law, and religion. Yet despite
nearly constant attention and debate, very little empirical work

asy
promises of secular modernity have played themselves out and
been found to be severely wanting. Formerly, Marxism provided
was done on these questions until recently. For a long time, the
idea that language might shape thought was considered at best

En
a framework for radical social criticism. But with Communism’s
demise, the discourse of critique has seemingly been deprived
untestable and more often simply wrong. Research in labs at
Stanford University and at MIT has helped reopen this question.
of an immanent, secular basis. This is one key reason behind
the revival of scholarly interest in political theology, which
gin
Language is a uniquely human gift, central to our experience of
being human. Appreciating its role in constructing our mental
lives brings us one step closer to understanding the very nature
employs a messianic or salvific idiom to expose the failings of a
predominantly “secular age.”
166. According to the passage, all, except which of the following
of humanity.
eer
I often start my undergraduate lectures by asking students
were the doubts expressed by the cynics?
(a) Religion was secluded and this could lead to unrest. ing
the following question: which cognitive faculty would you most
hate to lose? Most of them pick the sense of sight; a few pick
hearing. Once in a while, a wisecracking student might pick her
(b) Humanity was in danger of getting isolated.
(c) Complete enlightenment was undesirable.
.ne
sense of humor or her fashion sense. Almost never do any of
them spontaneously say that the faculty they’d most hate to lose
(d) Reason could fall short in delivering the ultimate reality.
(e) Unconditional surrender could throw up further
challenges.
167. What is the irony mentioned by the author in the passage
t
is language. Yet if you lose (or are born without) your sight or
hearing, you can still have a wonderfully rich social existence.
You can have friends, you can get an education, you can hold a
job, you can start a family. But what would your life be like if you
had never learned a language? Could you still have friends, get an
regarding “enlightened modernity”?
education, hold a job, start a family? Language is so fundamental
(a) Religion has got a second life around the globe.
to our experience, so deeply a part of being human, that it’s hard
(b) The religious revival is the response to the broken to imagine life without it. But are languages merely tools for
promises. expressing our thoughts, or do they actually shape our thoughts?
(c) The absurdity of the Nineteenth-century predictions. Clearly, languages require different things of their speakers.
(d) The truth in modern works. Does this mean that the speakers think differently about
(e) The utopian model of modernity. the world? Do English, Indonesian, Russian, and Turkish
168. According to the passage, the turnabout in the theory of speakers end up attending to, partitioning, and remembering
reason can be attributed to: their experiences differently just because they speak different
(a) religion’s extraordinary staying power. languages? For some scholars, the answer to these questions has
(b) religion’s amazing influence. been an obvious yes. Just look at the way people talk, they might
(c) a lack of an alternative plan. say. Certainly, speakers of different languages must attend to and
(d) the demise of communism. encode strikingly different aspects of the world just so they can
(e) a weak secular system. use their language properly.

Downloaded From : www.EasyEngineering.net


Downloaded From : www.EasyEngineering.net

406  l  Reading Comprehension

170. The author’s attitude can be best described as: his or her own self-interest for the sake of others. However,
(a) Reproachful (b) Informative understanding properly the character of such attachments should
(c) Admonitory (d) Investigative take into account not only the act of self-transcendence common
(e) Cautious to all of these attachments, but also the different objects of those
171. According to the passage, why is language essential to our attachments. Thus, it may be more helpful to distinguish the love
experience? for one’s paramour or children from the ‘love’ for one’s nation
(a) It forges friendships. (b) It helps get educated. by understanding patriotism as signifying attachments of loyalty
(c) It gets you a job. (d) It helps start a family. to a territorial community. There are often different aspects to
(e) It helps form our thoughts. the patriotic attachments that one forms to one’s nation, as a
172. In the passage, the research on languages has proved all but consequence of the different factors involved in.
which of the following? Nationalism is the historical formation of a particular nation.
(a) People who speak different languages think differently. One may, for example, be loyal to one’s nation because of its
(b) Grammar is common to all languages. laws, or its customs, or its religion. There are usually many and
(c) Linguistic expressions encode partial information. differing, even conflicting, views of the nation that correspond
(d) Language is a uniquely human gift. to these different factors. However, inescapable is the fact that
(e) Language brings us closer to humanity. the individual often shows a preference for his or her fellow
173. The approach contained in this passage can be best compared nationals. This preference need not take the form of a prejudice
with:
ww
(a) The study of the existence of varied regions and food
habits.
against, or hatred of, those who are not members of one’s nation.
Patriotism need not deny varying and different pursuits by the
members of the nation. It need not reject differing conceptions

w.E
(b) The research on the existence of animal noises.
(c) The reason for the existence of fashions.
(d) The cause for the existence of climates.
of the nation held by members of the nation, as nationalism often
does. Indeed, in so far as patriotism implies a commitment to the
well-being of one’s country, it provides the basis for working out
(e) The origin of various seasons.

Passage- 41 asy the differences, involving reasonable compromise, between the


individual members of the nation and their differing conceptions
of what the nation should be out of a concern for promoting that
Positive psychology has been extremely successful on a number
of fronts including over 1000 publications, numerous special En well-being. The process of working out these differences through
compromise is politics. The concern for the well-being of the
issues, numerous handbooks, etc. As well, hundreds of millions
of dollars have been secured to support research. Seligman has
promoted positive psychology in a number of areas including gin
nation that includes the willingness to compromise is central
to the civility between the members of the nation that makes
politics possible.
psychotherapy, youth development, occupational and workplace
psychology, neuroscience, coaching, educational curricula, eer
174. The author believes that
(a) One’s love for oneself determines the level of love for
health, and a major initiative involving the American Army.
Seligman sees his efforts trying to teach positive traits and
resilience in the American Army as a critical testing ground, ing
one’s children and nation.
(b) The love for one’s nation depends on the love for one’s
children.
if successful, these programs will then be implemented “in the
civilian population.” Part of Seligman’s agenda is that most of
the traits associated with positive psychology can be taught in .ne
(c) The term love describes one’s love for the nation.
(d) Patriotism arises out of a feeling of hatred for other
schools and delivered while teaching any subject matter by using
“embedding techniques.”
The preoccupation with vitality involves establishing different
kinds of limits or boundaries to respectively different kinds of

nations.

t
(e) The love for one’s children arises due to familiarity.
175. According to the author what is the genesis of politics ?
(a) The concern for the well being of the nation.
(b) Patriotism because it involves the working out of
relations of vitality. Humans draw a distinction between their own differences involving compromises between members
children and those of another. One usually does not love another’s of a nation to promote its well being.
children as if they were one’s own. And one does not usually love (c) Nationalism because it is the historical formation of a
another nation as if it were one’s own. Such a limitation on the nation.
recognition of, and the love for, what is understood to be one’s (d) An individual shows a preference for his/her fellow
own is a consequence of the preoccupation with the continuation nationals.
of the self, both its biological and cultural components. The love (e) There are often different aspects to the patriotic
that one has for one’s nation is designated by the term ‘patriotism’. attachments that one forms of one’s nation.
The widely used term ‘love’ as an expression of the attachments 176. What does the term ‘love’ indicate?
that the individual has to his or her nation is not altogether (a) an attachment for one’s children.
satisfactory because we also employ the same term to describe (b) an attachment for one’s nation.
the attachments one has to one’s paramour, children, friends, (c) an attachment for God.
and god. Indeed, some individuals have genuinely loved all of (d) an attachment to one’s friends.
humanity. What such a wide use of the term indicates is that, in (e) setting aside or transcending one’s self interest for the
each of these instances, the individual puts aside, or ‘transcends’, sake of others.

Downloaded From : www.EasyEngineering.net


Downloaded From : www.EasyEngineering.net

Reading Comprehension  l 407

177. Which of the following statements from the passage if true shadows of things on a screen in front of them. Plato says, that
would support Seligman’s concept of positive psychology? these shadows were the only real things.
(a) Humans draw a distinction between their own children Now suppose that one of the prisoners was to be freed and
and those of another. emerged blinking into the light. In time he would come to see
(b) One does not usually love another nation as if it were real objects in the world, and understand that what he had
one’s own. seen before were no more than shadows. But if he were then to
(c) Some individuals have genuinely loved all of humanity. return to the cave to try to persuade his fellows of their mistake,
(d) One may, for example, be loyal to one’s nation because of they would be unlikely to believe him. This, Plato thinks, is the
its laws, or its customs, or its religion. position of the philosopher: he has genuine knowledge while
(e) The concern for the well-being of the nation includes those around him have only distorted opinions, but because the
the willingness to compromise which is central to the path to philosophical knowledge is long and hard, very few are
civility between the members of the nation. willing to take it.
But was Plato justified in drawing such a sharp contrast
Passage- 42 between philosophical knowledge and common opinion? My
There is no better way to understand what political philosophy conception of political philosophy does not involve endowing
is and why we need it than by looking at Lorenzetti’s magnificent philosophers with a special kind of knowledge not available to

ww
mural. We can define political philosophy as an investigation
into the nature, causes, and effects of good and bad government,
and our picture not only encapsulates this quest, but expresses
other human beings. Instead they think and reason in much the
same way as everyone else, but they do so more critically and
more systematically. They take less for granted.. It is easiest to

w.E
in striking visual form ideas that stand at the very heart of the
subject. Good and bad government profoundly affects the quality
of human lives. Lorenzetti shows us how the rule of justice and
the other virtues allows ordinary people to work, trade, hunt,
explain this by an example. Suppose we were to ask a politician
what his goals were; what aims or values the political community
he belongs to should be trying to achieve. If he belonged to a

asy
dance, and generally do all those things that enrich human
existence, while on the other side of the picture, tyranny breeds
contemporary Western society, he would probably come up
with a fairly predictable list: law and order, individual liberty,

En
poverty and death. So it really makes a difference to our lives
whether we are governed well or badly. We cannot turn our back
economic growth, full employment, and one or two others. How
might a political philosopher respond to this? Well, first of all she
would turn the spotlight on the goals themselves and ask which
on politics, retreat into private life, and Political Philosophy and
imagine that the way we are governed will not have profound
effects on our personal happiness. gin
of them were really ultimate goals.
178. When is the need for political philosophy, the maximum?
Nobody can tell in advance whether any given work of
political thought will have the effect of Hobbes’s Leviathan or eer
(a) When political parties draw up their election manifestos.
(b) When politicians need to make statements that are
Rousseau’s Social Contract, or to take a later example, Marx
and Engels’s The Communist Manifesto. It depends entirely on
whether the underlying shift in thinking that the philosopher ing
politically correct.
(c) When political parties introspect after losses in elections.
(d) When new political challenges cannot be dealt with
proposes corresponds to political and social change in such a
way that the new ideas can become the commonplaces of the
conventional means.
.ne
179. According to the passage it can be inferred that
following generations. Other works of political philosophy
have enjoyed a limited success and then disappeared virtually
without trace. But the need for political philosophy is always
there, especially perhaps at moments when we face new political
t
(a) Political philosophers have specialized knowledge.
(b) Our political beliefs are based on the original thoughts
of political philosophers although not necessarily as told
to us directly by them.
challenges that we cannot deal with using the conventional
(c) Political philosophers spread their knowledge directly
wisdom of the day. At these moments we need to dig deeper, to
probe the basis of our political beliefs, and it is here that we may to the people.
turn to political philosophy, not perhaps at source, but as filtered (d) The impact of a philosophers thinking can be predicted
through pamphlets, magazines, newspapers and the like – every beforehand.
successful political philosopher has relied on media-friendly 180. Which of the following statements about political
disciples to put his or her ideas into circulation. philosophers is not validated by the passage?
But even if political philosophy answers to a genuine need, (a) Political philosophers rely on media to spread their
are its own credentials genuine? Political philosophy claims philosophy amongst people.
that it can bring to us a kind of truth about politics, something (b) Political Philosophers do not have special knowledge as
different from the opinions that guide us from day to day. This compared to other people.
claim was presented most dramatically by Plato, often regarded (c) Political philosophers think the same way as common
as the father of the subject, through the allegory of the cave in people.
the Republic. Plato likens ordinary people to prisoners who have (d) Successful political philosophers’ thoughts guide future
been chained in a cave in such a way that they can only see the generations.

Downloaded From : www.EasyEngineering.net


Downloaded From : www.EasyEngineering.net

408  l  Reading Comprehension

181. “Governance has a profound influence on our happiness” that our computers aren’t fast enough. The idea is not that some
Which of the following statements in the passage contradicts computational or representational limitation prevents you from
this assumption? having a definite probability. Give an agent access to exactly
(1) The rule of justice allows ordinary people to work, trade, your evidence relevant to the toothpaste claim, or, say, the claim
hunt, and dance. that there is a God. Give her all the computers, representational
(2) Tyranny breeds poverty and death. tools, brain upgrades, etc. that you like. Still it seems as though
(3) There is no better way to understand what political the agent would go wrong to have any very precise degree of
philosophy is and why do we need to look at Lorenzetti’s belief in the relevant claim. According to Scott Sturgeon: When
magnificent mural. evidence is essentially sharp, it warrants a sharp or exact attitude;
(4) None of the above when evidence is essentially fuzzy—as it is most of the time—it
(a) 4 only (b) 1 and 2 only warrants at best a fuzzy attitude.
(c) 3 only (d) 1, 2 and 3 182. According to the author which of the following is the most
likely reason for not arriving at an exact probability?
Passage- 43 (a) The size of the sample is too large.
(b) The computers are not adequately equipped to calculate
Sometimes one’s evidence for a proposition is sharp. For
the exact probability.
example, you’ve tossed a biased coin thousands of times. 83%
(c) The quality of evidence that is being assessed.

ww
of the tosses landed heads, and no pattern has appeared even
though you’ve done a battery of statistical tests. Then it is clear
that your confidence that the next toss will land heads should be
(d) The degree of belief that one has in the sample.
183. Which of the following statements is not supported in the
passage?

w.E
very close to 83%. Sometimes one’s evidence for a proposition is
sparse but with a clear upshot. For example: You have very little
evidence as to whether the number of humans born in 1984 was
(a) Non specific evidence reduces our belief in the
probability of any happening.
(b) Reliance on machines increases our belief in the

in this claim.
asy
even. But it is clear that you should be very near to 50% confident

But sometimes one’s evidence for a proposition is sparse and


probability of any happening.
(c) The contents of a sample increase our belief in the
probability of any happening.

En
unspecific. For example: A stranger approaches you on the street
and starts pulling out objects from a bag. The first three objects
(d) A symmetrical sample increases our belief in the
probability of any happening.
he pulls out are a regular-sized tube of toothpaste, a live jellyfish,
and a travel-sized tube of toothpaste. To what degree should you
believe that the next object he pulls out will be another tube of gin
184. In the passage, the author was concerned with which of the
following?
(a) Arriving at ways to calculate Probability through real
toothpaste? The answer is not clear. The contents of the bag are
clearly bizarre. You have no theory of “what insane people on the
eer
life examples.
(b) Proving the theory of Keynes about the scientific
street are likely to carry in their bags,” nor have you encountered
any particularly relevant statistics about this. The situation doesn’t
have any obvious symmetry, so principles of indifference seem to ing
calculation of Probability.
(c) Showing the importance of computers in arriving at the
correct Probability.
be of no help. Should your probability be 54%? 91%? 18%?
It is very natural in such cases to say: You shouldn’t have any
.ne
(d) Validating the fact that certainty about Probability
depends on nature of evidence.
very precise degree of confidence in the claim that the next object
will be toothpaste. It is very natural to say: Your degree of belief
should be indeterminate or vague or interval-valued. On this
way of thinking, an appropriate response to this evidence would
185. Why according to the author is the range a better

t
representative of probability as compared to specific
numbers?
(a) Because the degree of belief in most cases cannot be
be a degree of confidence represented not by a single number, but precise.
rather by a range of numbers. The idea is that your probability (b) Because most situations follow a symmetrical pattern.
that the next object is toothpaste should not equal 54%, 91%, (c) There is available scientific data to prove that a range is
18%, or any other particular number. Instead it should span a better representative of probability.
an interval of values, such as 10%, 80%.The toothpaste-in-the- (d) It is easier to calculate range using advanced computers
bag example is artificial, but many realistic examples have been and scientific methods.
proposed. What is your confidence that “there will be a nuclear
Passage- 44
attack on an American city this century”? What is your state of
opinion concerning “the price of copper and the rate of interest As the New Deal wound down its ambitious domestic program,
the Roosevelt administration began looking outward again.
twenty years hence, or the obsolescence of a new invention, or
Secretary of State Cordell Hull, like Keynes, had for decades
the position of private wealth owners in the social system in 40 believed that an open world economy would tend towards peace
years”? and prosperity. ‘‘Unhampered trade,’’ Hull said, ‘‘dovetailed
It is tempting to agree with J. M. Keynes that “About these with peace.’’ To this end he worked to secure trade agreements
matters there is no scientific basis on which to form any calculable including the Anglo-American Trade Agreement of 1938, which
probability whatever” and to think that the problem isn’t just contributed to the idea that international cooperation might

Downloaded From : www.EasyEngineering.net


Downloaded From : www.EasyEngineering.net

Reading Comprehension  l 409

restore the global economy of the era before World War I. Near 187. Which of the following is the most likely reason for the
the end of World War II, ideas like Hull’s approached fruition. acceptance of White’s plan at the Bretton Woods conference?
In June 1944, John Maynard Keynes went to the United (a) White’s plan advocated a contributory scheme that
States to represent Britain at the Bretton Woods conference. would be politically acceptable.
On paper Keynes was rated as only one of 730 delegates from (b) This would allow free trade between countries.
forty-four countries convening to establish rules for the postwar (c) America had more decision making powers at the
economy. But in person Keynes played the role of protagonist at conference as compared to other countries.
the conference. In 1941, before the United States had entered the (d) The Social Security Scheme was very successful in
war, he had drafted a plan to supply part of what the Versailles America.
Treaty left out—a system to ensure the smooth operation of 188. What does the first line of the passage indicate about the
the world’s finance and commerce, to ‘‘prevent the piling up of Roosevelt administration’s economic policy?
credit and debit balances without limit’’—after all, the cardinal (1) It was very successful.
rule for the postwar economy would be to avoid reproducing (2) It was not successful.
the prewar economy. Keynes’s plan for an International Clearing (3) It was casual.
Union would allot governments credit, based on their share of
(4) It was directionless.
world trade and allow them to draw that credit, denominated in
(5) It did not encourage free trade.
a notional banking currency, the Bancor, as needed to keep their
(a) Option 1 only. (b) Option 2 and 3.

ww
economies stable.
Morally alongside Keynes stood his antagonist, the American
representative Harry Dexter White. White had his own plan
(c) Option 4 and 5. (d) Option 2 and 5.
189. According to the passage, which of the following would best
indicate the state of the prewar economy?

w.E
to solve the same problem by slightly different solutions:
governments would still borrow, but from a contributory fund
rather than from a pool of Bancors. White’s plan stood in much
(a) It was a closed world economy.
(b) Governments contributed to a contributory fund and
then borrowed from it based on their requirement.

asy
the same relation to Keynes’s idea as the New Deal stood in
relation to European welfare states. Under British programs to
address poverty and disability, as drawn up in the Beveridge
(c) Each country had unlimited credit limits and could
draw at will.

En
plan of 1942, citizens received benefits from the state as a matter
of right. Nobody got pensions as a matter of right under Social
Security—retirees drew benefits because they had contributed.
(d) Governments enjoyed credit based on their world trade.

Passage- 45
The American delegation rejected Keynes’s plan and insisted on
White’s for much the same reason the Roosevelt administration gin
It is hard to understand British politics without some
understanding of the structure of British society. The way society
had insisted on a contributory basis for Social Security: a
contributory scheme would limit claims and satisfy Congress.
Thus White’s plan became the major basis for the International eer
is organized and the divisions within it reflect past political
decisions and are the source of many political pressures and
conflicts. Many commentators argue that the social structure
Monetary Fund (IMF) as agreed at Bretton Woods, and Congress
placed further restrictions on IMF to prevent unconditional
withdrawals from the fund.
ing
of Britain has changed rapidly in the last 25 years. The size of
the manual working class has declined, class is less important
IMF had a twin, the International Bank for Reconstruction
and Development, better known as the World Bank. Where IMF .ne
in politics, most women now go out to work and Britain, it is
said, has become a multicultural society. Yet for Marsh when
analyzing the nature of British society, it continues to be the
was opposed to allow countries to weather the vicissitudes of free
economies, the World Bank was supposed to lend money for the
repair of war damage and for long-poor countries to enter the
club of modern nations. It stood in relation to the world’s less-
t
happy hour for men with money, knowledge and power. What
he means by this is that those with resources seem to be the ones
who are continually blessed with even greater opportunities and
developed regions rather as PWA, TVA, and WPA stood to the wealth. This chapter will examine the nature of the British social
American South and West. Also rather like those New Deal relief structure and examine the cleavages in terms of class, ethnicity
agencies, the World Bank labored under the limits of prevailing and gender. What accounts for inequality, how great is inequality
economic opinion, which limited also the bank’s capital: its first and what are its implications for politics?
loan to France committed a full third of its available resources Much of the existing evidence supports the claim above by
186. Which of the following statements can be inferred from the Marsh. In British society, middle class, white, men are the ones
passage? who do best in terms of education, health care and life chances.
(a) The Global economy after World War II was in better There are three possible explanations for their success. First,
shape than it being before World War I. they could be lucky. However, luck is a weak explanation for
(b) Keynes believed that trade restrictions can help boost such a systematic and constant level of success. What could
economies.
explain white, middle class, men having more luck than anyone
(c) Keynes advocated the idea of an International Fund
else? Second, they could be more able than others. They may be
that would allow countries to draw up to the share of a
more intelligent, have more drive or be more competitive. But
country’s contribution to world trade
(d) In America, social security schemes were linked to it seems strange that all the attributes of intelligence, drive or
contributions from members. competition reside in white, middle class, men but it could be

Downloaded From : www.EasyEngineering.net


Downloaded From : www.EasyEngineering.net

410  l  Reading Comprehension

(a) Holding men responsible for white middle class male


true that our notion of what is drive or intelligence is one defined
dominance in British Society.
by middle class men. Third, the way Society is organized—the
(b) Holding women responsible for white middle class male
social structure—may privilege certain groups. In other words,
dominance in British Society.
society is organized in such a way that middle class men are more
(c) Holding Political history to be responsible for white
likely to succeed. middle class male dominance in British Society.
The second explanation is based on the idea of agency and (d) Holding women, white middle class men and ethnic
explanations of inequality are phrased in terms of choices and minorities to be responsible for white middle class male
actions. From this perspective, women and ethnic minorities will dominance in British Society.
start to reduce levels of inequality once they have equal education 193. Which of the following statements is not validated in this
opportunities and then choose to compete with men. The third passage?
explanation is a structural explanation. In other words, inequality
(a) In Britain greater happiness is not directly correlated to
is a result of the ways in which institutions are organized. They
greater prosperity.
can protect or promote certain interests or groups. Consequently,
(b) Middle class white men are more prosperous because
the end of inequality will depend on significant organizational
there is no competition from women for them.
change.
(c) Political decisions of the past have had an influence on
Britain has changed over the past 60 years. Most British
British society.

ww
people now enjoy greater prosperity and opportunities than
in the past, so that poverty today is a relative, rather than an
absolute, concept. Many parts of the economy have experienced
(d) Some ethnic groups in the middle class do well and are
prosperous in British society.

w.E
strong growth relative to other European countries since 2001,
although economic forecasts in 2005 reported a slowdown in
national performance before an upturn in 2006. But opinion
Passage- 46
In 1940 the English writer and socialist, George Orwell, wrote:

asy
polls suggest that greater prosperity has not brought greater
happiness for many Britons. Consumerism, multi-ethnic growth,
feminism and an expanded role for women (particularly in a
‘England is the most class ridden country under the sun. It is a
land of snobbery and privilege, ruled largely by the ‘old and the
silly’ (Orwell 1970: 87). However, by the 1950s politicians were
mobile workforce), greater individual freedom and more (if
En
not complete) tolerance for alternative lifestyles (such as the
talking about Britain being a classless society and in 1959, after
Labour lost three elections in a row, the question was raised of
increased acceptance of gays), technological advances and new
economic policies have helped to transform Britain, sometimes
for the better, sometimes for the worse. But continuing structural gin
whether Labour could ever win again because of the decline in
the size of the working class. By the 1980s Robertson was arguing
that class was no longer important in British politics. Likewise, the
and social problems, as well as very varying life-chances and
opportunities for the people, warn against undue complacency.
190. Which of the following if true would weaken Marsh’s
eer
sociologist Peter Saunders argues that Britain is an open society
where it is possible for people from working class backgrounds
to move up into middle class occupations relatively easily and
argument that middle class men are the most successful in
Britain? ing
failures in mobility are the consequence of ability rather than
class structures. The British Election Survey concluded that class
(a) The number of students entering British Universities
from the working class outnumbered students from the
middle class. .ne
was no longer a significant determinant of the vote. It is clear that
by 2001 the majority of people worked in white collar or what
some people would see as middle class occupations.
(b) The British Parliament had fewer representatives from
the minority and underprivileged groups.
(c) Children of only the educated and rich, continued
higher studies.
t
Mines have been replaced by call centres and steel factories
have turned into leisure facilities. Sheffield, the steel city of
Britain’s industrial dominance now produces as much steel as
at its height of production but with a fraction of the workforce.
(d) Companies in Britain reserved certain number of jobs The key employers in Sheffield are universities, hospitals and
only for women. government offices. People have shifted from manual to non
191. According to the author, which of these could lessen the manual occupations. With these changes in occupations it
white middle class male dominance in British Society? seems that some of the other Markers of class have declined.
(1) If luck favours women more than men. The significant differences in dress, accent, and education that
(2) If women were more educated than men. existed between the middle class and the working class in the
(3) If women were as educated as men and wanted to excel 1950s and 1960s seemed to have disappeared. By watching a film
as much as their male counterparts at work. from the 1950s or listening to a BBC presenter even up to the
(4) If women were offered more jobs in the organizational 1960s and 1970s, it is apparent how class distinctions in accents
structure of institutions. have softened. Even things like travel have changed dramatically.
(5) If white middle class men were less intelligent. In the 1960s and 1970s foreign and air travel was unavailable to
(a) Option 1. (b) Option 2 and 3. the majority of people. Now nearly everyone has been on a plane.
(c) Option 3 and 4. (d) Option 2 and 5. Does this mean that class has disappeared? Before writing off
192. Which of the following can be inferred as the author’s class as a significant factor in politics, it is worth considering a
attitude in the passage? number of factors:

Downloaded From : www.EasyEngineering.net


Downloaded From : www.EasyEngineering.net

Reading Comprehension  l 411

(1) The shift from manual to non-manual work does not Passage- 47
mean that the working class has become middle class.
Whilst people may now be working in call centres rather In the Khrushchev era, the Soviet regime’s divided attitude to
than mines, it does not mean that they have changed questions of poverty and wealth became positively schizophrenic.
class. The work in call centres is highly routinised, the On the one hand, the regime remained hostile to market activity, to
workplace highly structured with limited autonomy for undue self-enrichment, and to unearned income. The prevailing
workers, and pay rates and promotion opportunities are anti-market egalitarian ethos was also, more positively, reflected
low. The non-manual category hides class distinctions in new welfare measures such as a comprehensive pensions law
based on occupation, autonomy at work, work for urban people that was passed in 1956.
conditions, pay and life chances. A further landmark in social policy was a mass housing
(2) Whilst some of the markers of class may have lessened, campaign that saw the construction of more than 35 million
others are crucially important. For instance, health and new flats between 1955 and 1970 and established the separate
life expectancy are very closely related to class in Britain. dwelling as the right – though not yet always the reality – of every
Those on lower incomes are more likely to be overweight, Soviet family. On the other hand, the Soviet system was trapped
unhealthy and to die earlier. Indeed the inequalities between in the story it insisted on telling of ever increasing prosperity.
the poor and rich have become greater in terms of health. As Among the consequences was a tendency to enter undignified
a report on health inequalities said: ‘Since 1980, although

ww
health and expectations of life have generally improved,
the social gradients of many indicators of health have
deteriorated or at best remained unchanged’. In other words
and futile competition with the West over living standards. Hi-
tech and diet-conscious America was much less interested than
Soviet Russia in claiming supremacy in production of animal fats

w.E
people are living longer but the difference between the life
expectancy of the poorest and the richest has become greater.
This reflects the general change in the nature of economic
(even if supremacy was what it continued to enjoy).
After Khrushchev, however, the rhetoric was rather more
subdued. For the last 20 years of its existence, the Soviet Union
inequality.

asy
194. In the first paragraph of the passage, the writer establishes
that …..
achieved modest but tangible advances on the consumer front.
Above all, it maintained its social contract by keeping prices low
for the basics. CIA figures (perhaps the best available) estimated

1970s.
En
(a) Government offices replaced factories in Britain by the a steady growth in food consumption between 1964 and
1973. Between 1946 and 1990, Soviet annual output increased
(b) Life expectancy of the working class has increased.
(c) By the beginning of the twenty first century most of the
working class in Britain had converted to the middle gin
consistently, apart from 1963 and 1979 (which were years of bad
harvests). Living standards were extremely modest by Western
European standards, but until the late 1980s there was no
class.
(d) By the beginning of the twenty first century, Britain had
become a classless Society.
eer
significant downturn, and the Soviet population could enjoy a
long period of unprecedented stability. Per capita consumption
195. Which of the following statements is NOT supported in the
passage? ing
grew at an average annual rate of 3.5% between 1951 and 1980.
The increase in discretionary income was also reflected in
levels of savings, which rose from an average of 157 rubles per
(a) In Britain failure to move from one class to another
depended on the ability of a person.
(b) Although people moved from a lower class to a higher .ne
account in 1960 to 1,189 rubles in 1980. Another reason that
savings increased, however, was that there was very little on
class, the economic inequality increased.
(c) Class does not change with the type of work.
(d) None of the above.
196. Which of the following would most likely be the writer’s
t
which money could be spent. Shortage vitiated the economic
upturn that Soviet people enjoyed from the 1960s to the 1980s.
This was still a population starved of consumer goods. In 1976,
there were only 223 television sets per 1,000 people in the
message to the reader? USSR (compared to 571 per 1,000 in the USA). But television
(a) George Orwell was wrong in saying that England was production had actually been an investment priority of the Soviet
the most class ridden country. regime. On other consumer fronts it performed even less well.
(b) One’s class is not necessarily represented by one’s If the Americans had nearly 100 million cars by this time, the
occupation. Soviet Union could count only 5 million in personal use. In 1970,
(c) The results of elections depend on class. two-fifths of the average household budget was spent on food.
(d) Quality of health depends on class. 198. “After Khrushchev, however, the rhetoric was rather more
197. Economic inequality in British Society is best reflected by subdued” The word rhetoric most likely refers to which one
which of the following Statements?
of the following?
(a) the difference between the life expectancy of the poorest
(a) The Pension Law for Urban people.
and the richest has become greater.
(b) The mass housing campaign launched by the Soviet
(b) Mines have been replaced by call centers and steel
factories have turned into leisure facilities. regime.
(c) by 2001 the majority of people worked in white collar or (c) The Soviets’ harping on the increasing prosperity in
what some people would see as middle class occupations. their country.
(d) Now nearly everyone has been on a plane. (d) The rise in consumer goods in the Soviet Union.

Downloaded From : www.EasyEngineering.net


Downloaded From : www.EasyEngineering.net

412  l  Reading Comprehension

growth rates. In addition, growth spillovers and exogenous shocks


199. Which of the following statements is supported in the
could be transmitted through changes in factor productivity and
passage?
(a) Khrushchev supported a free market economy. business confidence from neighboring countries.
(b) The production of animal fats was greater in Russia as Studies have indicated that when a major economy coexists
compared to Americ(a) side by side with smaller countries, spillover effects of the major
economy’s growth on the smaller economies is often high. Given
(c) The rate of savings did not increase because of an
India’s rapid growth and the size of its economy relative to those
increase in income.
of its South Asian neighbors, a strong positive spillover effect
(d) Between 1950 and 1970 every Russian did not have a flat
could manifest in higher growth for countries in the region.
of his own.
India’s strong performance as a supply chain for the service
200. What according to the author was the most likely reason for
sector of advanced economy markets is well researched; but there
people’s living standard not improving in spite of increase in
is little or no research into how India’s growth affects the growth
their income?
of its immediate neighbors.
(a) People liked to spend more on food than consumer
products. The results indicate that overall, India’s growth has good
(b) Non-availability of sufficient consumer goods. explanatory power for growth in SAC after 1995, controlling
(c) People spent most of their income on housing. for other sources of growth endogenous to the countries such

ww
(d) People had to save for their old age expenditure.
201. Which of the following statements substantiate the view that
in the last twenty years of its existence after Khrushchev, the
as human capital, the size of the government, inflation, and
other trading partners’ growth. The growth spillovers from India
are likely to be transmitted through a combination of different

w.E
Soviet Union laid stress on Socialism?
(a) The prices of essential commodities were maintained at
low levels.
channels. The Indian economy was until the early 1990s a closed
economy with low international trade, high tariff and non tariff
barriers, and a tightly controlled capital account.

priority. asy
(b) Television production had actually been an investment

(c) Construction of more than 35 million new flats between


The focus of economic policy was on self sufficiency. Though
deregulation began in the 1980s, the stage for India’s integration

1955 and 1970.


(d) There was a fall in the level of self-enrichment andEn into the world economy was set in 1991 when major reforms
were initiated after a severe balance of payments crisis. The
reforms included a unified exchange rate system, lowering of
unearned income.

Passage- 48 gin
tariffs across the board, and substantial reductions in regulation
of, and restrictions on, trade. The results of the liberalization
Since the mid-1990s, the size of the Indian economy has
increased nearly twenty-fold in U.S. dollar terms. Real GDP eer
were a large increase in exports, imports, and foreign direct
investment (FDI), with external debt declining and foreign
growth averaged 5 percent annually, poverty reduction has been
significant, and living conditions have improved in a variety of ing
reserves recording strong improvements.
202. Which of these would best reflect the main objective of the
author in the above passage?
dimensions—such as life expectancy, infant mortality and years
of schooling. At the same time, India’s share in global exports
has more than doubled. In an interdependent world, changes .ne
(a) To show the growth story of the Indian economy.
(b) To analyze the impact of one country’s economic
in economic performance in one country could manifest in
countries with which it is highly integrated as positive spillovers
or adverse exogenous shocks.
Various studies have identified trade, financial flows, human
performance on other countries.

neighbouring countries. t
(c) To analyze the impact of India’s economic growth on its

(d) To focus on the factors through which one country’s


economic performance affects other countries.
capital and terms of trade as key channels through which this 203. Which of the following is supported in the passage?
takes place. Demand for imports, inputs and final products (a) The economic growth of India’s neighbours is
affect partner countries’ supply of exports and real incomes independent of India.
through the trade channel. Increased demand for imports from (b) The import of other countries from India has
partner countries impacts positively domestic production, while considerably reduced.
higher partner countries’ exports could worsen current account (c) Demand for goods in one country does not affect the
dynamics, and lower the competitiveness of domestically exports of other partner countries.
produced competing goods, which could feedback into (d) India’s neighbours should benefit from India’s economic
investment and consumption decisions. growth.
Integrated stock markets and cross border financial services 204. According to the author, which of the following could
often provide a wider pool for the intermediation of financial contribute to the growth of interlinked economies?
resources and greater depth to financial markets, and could also (1) Decrease in import requirement of one country.
be sources of contagion. Access to education and health services (2) Increase in exports from partner countries.
in more developed economies could contribute to human capital (3) Access to health and education of advanced economies.
development and skills accumulation, which feed into higher (4) Cross border financial services.

Downloaded From : www.EasyEngineering.net


Downloaded From : www.EasyEngineering.net

Reading Comprehension  l 413

(5) Increase in business confidence transmitted from compromise, between Moscow and Washington. By 1948, the
interdependent economies. occupation divisions had instead hardened. In the north, a pro-
(a) 5 only (b) 1 and 3 Soviet regime under the leadership of the former anti- Japanese
(c) 3, 4 and 5 (d) 2 and 4 fighter Kim Il-Sung took on all the trappings of an independent
205. According to the passage, which of these assumptions could regime. So, too, did its counterpart in the south: a pro- American
most likely affect the investment and consumption decisions regime headed by the virulently anti-communist Syngman Rhee,
of a country? a Korean nationalist of long standing. Each side regularly rattled
(a) Change in patterns of exports and imports of partner sabres at the other; neither North nor South Koreans could
countries. accept a permanent division of their homeland.
(b) Access to education and health services in more In 1948, the Truman administration, seeking to extricate itself
developed countries. gracefully from its Korean commitment, began withdrawing US
(c) Deregulation of the economy. military forces from the peninsula. American defense planners
(d) Changes in economic performance of other countries. believed not only that US military personnel had become
overextended worldwide, necessitating this pullback, but that
Passage- 49 Korea, in fact, possessed minimal strategic worth. The North
The impact of the Korean War on the Cold War is difficult Korean invasion two years later brought a different calculus to
the fore. Although it might have lacked great intrinsic strategic

ww
to overstate. Not only did the Korean fighting lead to an
intensification and geographical expansion of the Cold War, it
threatened a wider conflict between the United States and the
value, Korea stood as a potent symbol, especially in view of
America’s role as midwife and protector of the Seoul regime.

w.E
communist powers, and foster increased East–West hostility, but
it also spurred a huge increase in American defense spending
and, more broadly, a militarization and globalization of American
Further, the North Korean attack, sanctioned and backed by the
Soviet Union and China, threatened America’s credibility as a
regional and global power every bit as much as it threatened the
survival of the South Korean government.

asy
Foreign policy. Beyond Asia, the conflict in Korea also hastened
the strengthening of NATO, the arming of Germany, and the
stationing of US troops on European soil. ‘It was the Korean
To Truman, Acheson, and other senior decision-makers,
the stakes at risk in Korea appeared enormous. Consequently,
without any dissenting voices being raised, the president quickly

En
War and not World War II that made the United States a world
military-political power’, diplomat Charles Bohlen has argued. authorized US military intervention. ‘If the United Nations
yields to the force of aggression’, Truman declared publicly on
With uncommon unanimity, scholars have affirmed that
judgement, identifying the Korean War as a key turning point
in the international history of the postwar era. America’s ‘real gin
30 November, ‘no nation will be safe or secure. If aggression is
successful in Korea, we can expect it to spread throughout Asia
and Europe to this hemisphere. We are fighting in Korea for our
commitment to contain Communism everywhere originated in
the events surrounding the Korean War’, contends John Lewis
Gaddis. Warren I. Cohen calls it ‘a war that would alter the nature
eer
own national security and survival.
206. “The Korean War is a key turning point in the international
of the Soviet-American confrontation, change it from a systemic
political competition into an ideologically driven, militarized ing
history of the postwar era.” Which of the following statements
in the passage supports this view?
(a) It also spurred a huge increase in American defense
contest that would threaten the very survival of the globe’. Yet,
as Cohen also notes, ‘that a civil war in Korea would provide
the critical turning point in the postwar Soviet-American
spending.
.ne
(b) The Korean fighting led to an intensification and

relationship, and raise the possibility of world war, seems, in


retrospect, nothing short of bizarre’.
Certainly, in the aftermath of World War II, few places
appeared less likely to emerge as a focal point of Great Power
geographical expansion of the Cold War.

NATO. t
(c) The conflict in Korea also hastened the strengthening of

(d) ‘A war that would alter the nature of the Soviet-American


confrontation, change it from a systemic political
competition. Occupied and ruled by Japan as a colony ever competition into an ideologically driven, militarized
since 1910, Korea factored into wartime councils merely as yet contest that threatened the very survival of the globe.
another minor and obscure territory whose future disposition 207. According to the author which amongst the following was
fell on the Allies’ already overburdened shoulders. At the the reason for the falling through of the proposal to establish
Potsdam Conference, the Americans and Soviets agreed to share an independent unified Korea?
occupation responsibilities there by temporarily dividing the (a) Korea was a minor and obscure territory.
country at the 38th parallel; they also agreed to work towards the (b) The Allies were overburdened.
establishment of an independent, unified Korea at the earliest (c) There was an escalation in Cold War between U.S.A and
practicable time. Russia.
In December 1945, at a foreign ministers’ meeting in Moscow, (d) North Korea and South Korea did not agree to such a
the Soviets accepted a US proposal for the establishment of a proposal.
joint Soviet-American commission to prepare for the election 208. What according to the author was the impact of the Korean
of a provisional Korean government as a first step toward full War on the Cold War?
independence. But that plan soon fell victim to larger Cold War (a) It made the United States of America into a world
tensions that militated against any meaningful cooperation, or military political power.

Downloaded From : www.EasyEngineering.net


Downloaded From : www.EasyEngineering.net

414  l  Reading Comprehension

(b) It decreased East West hostility. view of their country by insisting that it should have only one
(c) It increased America’s spending on armament. religion, Hinduism; while others think that there should be
(d) It led to a greater hostility and enveloped more Capitalist freedom of religion such that Muslims, Sikhs, and Christians are
and Communist countries. rightly members of the nation.
209. Which of the following statements is not validated in this Distinctive of nationalism is the belief that the nation
passage? is the only goal worthy of pursuit – an assertion that often
(a) World War II resulted in the United States emerging as leads to the belief that the nation demands unquestioned and
a world military political Power. uncompromising loyalty. When such a belief about the nation
(b) The Korean War acted as an obstacle in the spread of becomes predominant, it can threaten individual liberty.
communism. Moreover, nationalism often asserts that other nations are
(c) At a certain point in time it was felt that a civil war in implacable enemies to one’s own nation; it injects hatred of what
Korea could escalate into a world war. is perceived to be foreign, whether another nation, an immigrant,
(d) America sent its troops back to Korea to protect its or a person who may practice another religion or speak a different
credibility as a regional and global power. language. Of course, one need not view one’s own nation and its
relation to other nations in such a manner.
Passage- 50 210. The author is most likely to believe that disputes in India
arise today due to

ww
It must be acknowledged that human beings exhibit another
tendency, when they engage in activities in which it seems not
to matter who were their parents, where they where born, or
(a) strong religious sentiments
(b) A strong sense of nationalism
(c) Strong religious sentiments about one’s own religion as

w.E
what language they speak. These activities, rather than asserting
divisions within humanity, bring people together. For example,
scientists are concerned with understanding the physical facts
opposed to different religions of others.
(d) Intolerance towards immigrants.
211. According to the passage which of the following statements

asy
of the universe, such as the nature of light. Light itself is not
English, French, or German; and there is no English, French,
or German scientific method. There is only science. To speak of
is true?
(a) Significance of the nation is the focus of the passage.
(b) Uncompromising loyalty towards the nation can

En
a supposedly racial or national scientific method, as when the
Nazis insisted that there was an ‘Aryan science’, is to betray the
threaten individual liberty.
(c) One always views other nations as being hostile to one’s
character of science by introducing considerations that have
no place in understanding the physical aspects of the universe.
Other notable examples of activities and their corresponding gin own nation.
(d) Nationalism is a synonym for nation.
212. What according to you would be a suitable title for the
conceptions that bring humans together are the monotheistic
religions and commerce. Furthermore, throughout history,
passage?
eer
(a) Nationalism
empires, such as the Roman and Ottoman, have sought to unify
their peoples as a political alternative to nations. Thus, while an
individual often understands himself or herself as a member of ing
(b) Nation and nationalism
(c) Consequences of Nationalism
(d) Liberty and nationalism
a particular nation, one may also recognize oneself as a part of
humanity.
.ne
213. Which of the following activities does not result in separating
people from each other?
If a proper examination of the question ‘what is a nation?’
requires consideration of the tendency of humans to assert
distinctions, then it must also take into account those activities
that unify humanity. To fail to do so will only result in a
(a) activities in which parental lineage matters.
(b) activities in which the place of birth matters.
(c) activities based on linguistic lines. t
(d) activities in which neither of the above mentioned
misapprehension of the significance of the nation in human factors matter.
affairs; and it is precisely an inquiry into that significance that
is the focus of this book. We are concerned, above all, with the Passage- 51
question ‘what does the existence of nations tell us about human No 16th-century audience would have recognized the term
beings?’ But what is a nation, and what is nationalism? Many ‘Renaissance’. The Italian word rinascita (‘rebirth’) was used in
wrongly use the term ‘nationalism’ as a synonym for ‘nation’. the 16th century to refer to the revival of classical culture. But the
Nationalism refers to a set of beliefs about the nation. Any specific French word ‘Renaissance’ was not used as a descriptive
particular nation will contain differing views about its character; historical phrase until the middle of the 19th century. The first
thus, for any nation there will be different and competing beliefs person to use the term was the French historian Jules Michelet,
about it that often manifest themselves as political differences. a French nationalist deeply committed to the egalitarian
Some may view their nation as standing for individual liberty, principles of the French Revolution. Between 1833 and 1862
while others may be willing to sacrifice that liberty for security. Michelet worked on his greatest project, the multi–volume
Some may welcome immigrants, and support policies that make History of France. He was a progressive republican, vociferous
it easy for them to become citizens; while others may be hostile to in his condemnation of both the aristocracy and the church. In
immigration. To take another example, consider disputes today 1855 he published his seventh volume of the History, entitled La
in India. Some members of that nation have a narrow, intolerant Renaissance. For him the Renaissance meant: . . . the discovery

Downloaded From : www.EasyEngineering.net


Downloaded From : www.EasyEngineering.net

Reading Comprehension  l 415

of the world and the discovery of man. The sixteenth century (a) The Church, during the Renaissance became more
. . . went from Columbus to Copernicus, from Copernicus to powerful than what it was during the middle Ages.
Galileo, from the discovery of the earth to that of the heavens. (b) Classical culture rediscovered itself during Renaissance.
Man rediscovered himself. (c) Egalitarianism replaced political tyranny during
The scientific discoveries of explorers and thinkers like Renaissance.
Columbus, Copernicus, and Galileo went hand in hand with more (d) The concept of individuality and an understanding of
philosophical definitions of individuality that Michelet identified human values and spirit characterized Renaissance.
in the writings of Rabelais, Montaigne, and Shakespeare. This 217. Which of the following statements is not validated in the
new spirit was contrasted with what Michelet viewed as the passage?
(1) The aristocracy came in for praise from Jules Michelet.
‘bizarre and monstrous’ quality of the Middle Ages. To him the
(2) During Michelet’s own time there was freedom and
Renaissance represented a progressive, democratic condition
dignity for every person in France.
that celebrated the great virtues he valued – Reason, Truth, Art, (3) Michelet linked Renaissance to the middle ages.
and Beauty. According to Michelet, the Renaissance ‘recognized (4) Michelet used the word Renaissance to describe a
itself as identical at heart with the modern age’. Michelet was the historical phase.
first thinker to define the Renaissance as a decisive historical (a) Statements 1 and 2 only (b) Statements 1, 2 and 3
period in European culture that represented a crucial break with (c) Statement 4 only (d) Statement 3 only

ww
the Middle Ages, and which created a modern understanding
of humanity and its place in the world. He also promoted the Passage- 52

w.E
Renaissance as representing a certain spirit or attitude, as much
as referring to a specific historical period. Michelet’s Renaissance
does not happen in Italy in the 14th and 15th centuries. Instead,
In a reversal of the norm elsewhere, in India policy makers and
economists have become optimists while bosses do the worrying.
The country’s Central Bank has predicted that the country’s

asy
his Renaissance takes place in the 16th century. As a French
nationalist, Michelet was eager to claim the Renaissance as a
French phenomenon. As a republican he also rejected what he
economy is likely to grow at a double digit rate during the next
20-30 years. India has the capability with its vast labour and lauded
entrepreneurial spirit. But the private sector which is supposed to

En
saw as 14th-century Italy’s admiration for church and political
tyranny as deeply undemocratic, and hence not part of the spirit
do the heavy lifting that turns India from the world’s tenth largest
economy to its third largest by 2030 has become fed up. Business
of the Renaissance.
Michelet’s story of the Renaissance was shaped decisively gin
people often carp about India’s problems but their irritation this
time has a nervous edge. In the first quarter of 2011, GDP grew at
an annual rate of 7.8 percent; in 2005-07 it managed 9-10 percent.
by his own 19th-century circumstances. In fact, the values of
Michelet’s Renaissance sound strikingly close to those of his
cherished French Revolution: espousing the values of freedom,
eer
The economy may be slowing naturally as the low interest rates
and public spending that got India through the global crisis are
belatedly withdrawn. At the same time the surge in inflation caused
reason, and democracy, rejecting political and religious tyranny,
and enshrining the spirit of freedom and the dignity of ‘man’. ing
by exorbitant food prices has spread more widely, casting doubt
over whether India can grow at 8-10 percent in the medium term
Disappointed in the failure of these values in his own time,
Michelet went in search of a historical moment where the values
without overheating.
.ne
In India, as in many fast growing nations, the confidence to invest
depends on the conviction that the long term trajectory is intact
of liberty and egalitarianism triumphed and promised a modern
world free of tyranny.
214. What is the assumption on which the author writes the
opening sentence?
t
and it is that which is in doubt. Big Indian firms too sometimes
seem happier to invest abroad than at home, in deals that are often
hailed as symbols of the country’s growing clout but sometimes
speak to its weaknesses – purchases of natural resources that India
(a) There was no Renaissance in the sixteenth century.
(b) The people of the sixteenth century did not understand has in abundance but struggles to get out of the ground. In fact a
Renaissance. further dip in investment could be self fulfilling: if fewer roads,
(c) The word Renaissance was not coined in the sixteenth ports and factories are built, this will hurt both short term growth
century. figures and reduce the economy’s long term capacity.
(d) Sixteenth century people did not discuss art and culture. There is a view that because a fair amount of growth is assured
215. Which one of these arguments is not offered by Michelet to the government need not try very hard. The liberalization reforms
counter the belief that Renaissance started in Italy? that began in 1991 freed markets for products and gave rise
(a) Italy supported political tyranny. to vibrant competition, at the same time what economists call
(b) Italy supported religion as advocated by the Church. factor markets, those for basic inputs like land, power, labour
(c) Italian philosophy was deeply undemocratic. etc remain unreformed and largely under state control, which
(d) Renaissance represented human attitude and not just a creates difficulties. Clearances today can take three to four years
historical period. and many employers are keen to replace workers with machines
216. According to Michelet, “The Renaissance represented a despite an abundance of labor force. This can be attributed to labor
break from the Middle Ages” Which one of the following laws which are inimical to employee creation and an education
best substantiates the result of this break? system that means finding quality manpower a major problem.

Downloaded From : www.EasyEngineering.net


Downloaded From : www.EasyEngineering.net

416  l  Reading Comprehension

In fact the Planning Commission concluded that even achieving 223. Which of the following is most similar in meaning to the
9 percent growth will need marked policy action in unreformed word CLOUT given in bold as used in the passage?
sectors. Twenty years age it was said that yardstick against which (a) Strike
India should be measured was its potential and it is clear that there (b) Standing
remains much to do. (c) Force
218. Which of the following can be said about the Indian economy (d) Launch
at present? (e) Achieve
(a) It can comfortably achieve double digit growth rare at 224. Which of the following is most opposite in meaning to the
present. word MARKED given in bold as used in the passage?
(b) High food prices have led to overheating of the (a) Decreased
economy. (b) Ignored
(c) Citizens are affluent owing to laxity in regulation. (c) Clear
(d) Private sector confidence in India’s growth potential is (d) Assessed
high. (e) Imperceptible
(e) Unreformed sectors are a drag on economic growth. 225. What measures do experts suggest be taken to ensure
219. Why are employers reluctant to hire Indian labour force? targeted economic growth?

ww
(1) India’s labour force is overqualified for the employment
opportunities available.
(2) High attrition rate among employees stemming from
(a) Loweing of interest rates to help industries hit by
recession.
(b) Prolonged financial support for basic input industries.

w.E
their entrepreneurial spirit.
(3) Labour laws are not conducive to generating
emploment.
(a) Only (3)
(c) Incentives to Indian companies to invest in infrastucture.
(d) Formulation of policies and their implementation in
factor markets
(e) Stringent implementation of licensing system.
(b) All (1), (2) and (3)
(c) Only (1) and (3) asy Passage- 53
(d) Only (1) and (2)
(e) None of these
En
220. What is the state of India’s basic input sectors at present?
In many countries, a combustible mixture of authoritarianism,
unemployment and youth has given rise to disaffection with

(a) These sectors attract Foreign Direct Investment because


of their vast potential. gin
strongmen rulers which has in turn spill over into uprising.
Young people in these countries are far better educated than
their parents were. In 1990 the average Egyptian had 4.4 years
(b) These sectors are lagging as projects are usually awarded
to foreign companies.
(c) These sectors are stagnating and badly in need of eer
of schooling; by 2010 the figure had risen to 7.1 years. Could it
be that education, by making people less willing to put up with

reforms.
(d) These sectors are well regulated as these are governed ing
restrictions on freedom and more willing to question authority,
promotes democratization. Ideas about the links between
education, Income and democracy are at the heart of what social
by the State.
(e) None of these
221. What is the author’s main objective in writing the passage? .ne
scientists have long studied. Since then plenty of economists and
political scientists have looked for statistical evidence of a causal

(a) Showcasing the potential of India’s growth potential to


entice foreign investors .
(b) Exhorting India to implement measures to live up to
its potential.
t
link between education and democratization. Many have pointed
to the strong correlation that exists between levels of education
and measures like the pluralism of party politics and the existence
of civil liberties. The patterns are similar when income and
democracy are considered. There are outliers, of course – until
(c) Recommending India’s model of development to other recently, many Arab countries managed to combine energy-based
developing countries wealth and decent education with undemocratic political systems.
(d) Berating the private sector for not bidding for But some deduce from the overall picture that as China and other
infrastructure development projects. authoritarian states get more educated and richer, their people
(e) Criticising the measures taken by India during the will agitate for greater political freedom, culminating in a shift to
global economic crisis. a more democratic form of government.
222. What impact has the GDP growth of 7.8 percent had? This apparently reasonable intuition is shakier than it seems. Critics
(1) Indian Industry is anxious about India’s economic of the hypothesis point out that correlation is hardly causation.
growth. The general trend over the past half century may have been
(2) India has achieved status as the world’s third largest towards rising living standards, a wider spread of basic education
economy at present. and more democracy, but it is entirely possible that this is being
(3) Foreign investment in India has drastically increased. by another variable. Even if the correlation were not spurious, it
(a) Only (1) (b) All (1), (2) and (3) would be difficult to know which way causation ran. Does more
(c) Only (1) and (3) (d) Only (1) and (2) education lead to greater democracy? Or are more democratic
(e) None of these countries better at educating their citizens? A recent NBER paper

Downloaded From : www.EasyEngineering.net


Downloaded From : www.EasyEngineering.net

Reading Comprehension  l 417

compared a group of Kenyan girls in 69 primary school whose 230. What according to the author has led to uprisings in
students were randomly selected to receive a scholarship with authoritarian countries?
similar students in schools which received no such financial aid. (a) Lack of access to education.
Previous studies has shown that the scholarship programme (b) Vast numbers of uneducated and unemployable youth.
led to higher test scores and increased the likelihood that girls (c) Frustration with the existing system of governance.
enrolled in secondary school. Overall, it significantly increased (d) Unavailability of natural energy resources like coal and
the amount of education obtained. For the new study the authors oil.
tried to see how the extra schooling had affected the political and (e) Government’s overambitious plans for development.
social attitudes of the women in question. Findings suggested that 231. Which of the following is/are true about China in the context
education may make people more interested in improving their of the passage?
own lives but they may not necessarily see democracy as the way (1) China’s citizens are in favor of a more representative
to do it. Even in established democracies, more education does not form of government.
always mean either more active political participation or greater (2) China has made huge strides in infrastructure
faith in democracy. Poorer and less educated people often vote in developments.
larger numbers than their more educated compatriots, who often (3) China is in the midst of a political revolution.
express disdain for the messiness of democracy, yearning for the (a) None (b) Only (1)
kind of government that would deal strongly with the corrupt (c) Only (1) and (3) (d) Only (2)

ww
and build highways, railway lines and bridges at a dizzying pace
of authoritarian China.
226 Which of the following most aptly describes the central
(e) All (1), (2) and (3)
232. What does the phrase “messiness of democracy” convey in

w.E
theme of the passage?
(a) Democratic nations are richer and have a better track
record of educating their citizens.
the context of the passage?
(a) Democratic nations are chaotic on account of individual
freedoms.
(b) Most democratic countries frequently have violent

asy
(b) Education does not necessarily lead to greater enthusiasm
for a democratic form of government
(c) Educated societies with autocratic form of government
revolts among their citizens.
(c) The divide between the poor and educated is growing
enjoy a better quality of life than democracies.
En
(d) Citizens can fulfill their personal aspirations only under
wider in democracies.
(d) High levels of pollution on account of frantic pace of
infrastructure development.
a democratic form of government.
(e) Democracy makes citizens more intolerant as it does
not restrict personal freedoms gin (e) Resigned acceptance of intrinsic corruption in the
education system.
227 Which of the following is most similar in meaning to the
word PROMOTES given in bold as used in the passage? eer Passage- 54
(a) Up grades
(c) Recommends
(e) Publicizes
(b) Prefers
(d) Advocates
ing
When times are hard, doomsayers are aplenty. The problem
is that if you listen to them too carefully, you tend to overlook the
most obvious signs of change. 2011 was a bad year. Can 2012 be
228. What conclusion can be drawn from the statistics cited about
Egypt’s education system?
(a) Job prospects have been on the rise in Egypt in recent .ne
any worse? Doomsday forecasts are the easiest to make these days.
So let’s try a contrarian’s forecast instead.
times.
(b) Authoritarian leaders have played a vital role in
reforming Egypt’s education system.
(c) Egypt has one of the youngest and best educated
t
Let’s start with the global economy. We have seen a steady
flow of good news from the US. The employment situation seems
to be improving rapidly and consumer sentiment, reflected in
retail expenditures on discretionary items like electronics and
demographies in the world. clothes, has picked up. If these trends sustain, the US might post
(d) Egypt is likely to be successful vibrant democracy. better growth numbers for 2012 than the 1.5-1.8 percent being
(e) There has been a rise in education levels in Egypt in forecast currently.
recent times. Japan is likely to pull out of a recession in 2012 as post-
229. In the context of the passage which of the following earthquake reconstruction efforts gather momentum and the
characterize (s) democracies? fiscal stimulus announced in 2011 begins to pay off. The consensus
(1) Active participation of majority of educated citizens in estimate for growth in Japan is a respectable 2 per cent for 2012.
electoral process. The “hard-landing” scenario for China remains and will
(2) Fast paced economic growth and accountability of those remain a myth. Growth might decelerate further from the 9 per
in power. cent that it expected to clock in 2011 but is unlikely to drop below
(3) Better standards of living and access to higher 8-8.5 percent in 2012.
education. Europe is certainly in a spot of trouble. It is perhaps already
(a) All (1), (2) and (3) (b) Only (2) and (3) in recession and for 2012 it is likely to post mildly negative growth.
(c) Only (3) (d) Only (1) and (2) The risk of implosion has dwindled over the last few months
(e) None of these -peripheral economies like Greece, Italy and Spain have new

Downloaded From : www.EasyEngineering.net


Downloaded From : www.EasyEngineering.net

418  l  Reading Comprehension

governments in place and have made progress towards genuine 233. Which of the following is NOT TRUE according to the
economic reform. passage ?
Even with some of these positive factors in place, we have (a) China’s economic growth may decline in the year 2012
to accept the fact that global growth in 2012 will be tepid. But as compared to the year 2011
there is a flipside to this. Softer growth means lower demand for (b) The European economy is not doing very well
commodities and this is likely to drive a correction in commodity (c) Greece is on the verge of bringing about economic
prices. Lower commodity inflation will enable emerging market reforms
central banks to reverse their monetary stance. China, for instance, (d) In the year 2012, Japan may post a positive growth and
has already reversed its stance and has pared its reserve ratio thus pull out of recession
twice. The RBI also seems poised for a reversal in its rate cycle as (e) All are true
headline inflation seems well on its way to its target of 7 per cent 234. Which of the following will possibly be a result of softer
for March 2012. growth estimated for the year 2012 ?
That said, oil might be an exception to the general trend (A) Prices of oil will not increase.
in commodities. Rising geopolitical tensions, particularly the (B) Credit availability would be lesser.
continuing face-off between Iran and the US, might lead to a spurt (C) Commodity inflation would be lesser.
in prices. It might make sense for our oil companies to hedge this (a) Only (B) (b) Only (A) and (B)
risk instead of buying oil in the spot market. (c) Only (A) and (C) (d) Only (C)

ww
As inflation fears abate and emerging market central banks
begin to cut rates, two things could happen Lower commodity
inflation would mean lower interest rates and better credit
(e) All (A), (B) and (C)
235. Which of the following can be said about the present status
ol the US economy ?

w.E
availability. This could set a floor to growth and slowly reverse
the business cycle within these economies. Second, as the fear of
untamed, runaway inflation in these economies abates, the global
investor’s comfort levels with their markets will increase.
(a) There is not much improvement in the economic
scenario of the country from the year 2011
(b) The growth in the economy of the country, in the year

asy
Which of the emerging markets will outperform and who
will get left behind? In an environment in which global growth
2012, would definitely be lesser than 1.8 percent
(c) The expenditure on clothes and electronic commodities,
by consumers, is lesser than that in the year 2011

domestic consumption dynamic should lead; those dependent


En
is likely to be weak, economies like India that have a powerful

on exports should, prima facie, fall behind. Specifically for India,


(d) There is a chance that in 2012 the economy would do
better than what has been forecast
(e) The pace of change in the employment scenario of the
a fall in the exchange rate could not have come at a better time. It
will help Indian exporters gain market share even it global trade gin country is very slow.
236. Which of the following is possibly the most appropriate title
remains depressed. More importantly, it could lead to massive
import substitution that favours domestic producers.
Let’s now focus on India and start with a caveat. It is eer
for the passage ?
(a) The Economic Disorder
(b) Indian Economy Versus The European Economy
important not to confuse a short-run cyclical dip with a permanent
de-rating of its long-term structural potential. The arithmetic
is simple. Our growth rate can be in the range of 7-10 per cent ing
(c) Global Trade
(d) The Current Economic Scenario
(e) Characteristics of The Indian Economy
depending on policy action. Ten per cent if we get everything right,
7 per cent if we get it all wrong. Which policies and reforms are .ne
237. According to the author, which of the following would
characterize Indian growth scenario in 2012 ?
critical to taking us to our 10 per cent potential ? In judging this,
let’s again be careful. Lets not go by the laundry list of reforms
that FIIs like to wave: increase in foreign equity limits in foreign
shareholding, greater voting rights tor institutional shareholders
global trade scenario.
t
(A) Domestic producers will take a hit because of depressed

(B) On account of its high domestic consumption, India


will lead.
in banks, FDI in retail, etc. These can have an impact only at (C) Indian exporters will have a hard time in gaining market
the margin. We need not bend over backwards to appease the share.
FIIs through these reforms - they will invest in our markets (a) Only (B) (b) Only (A) and (B)
when momentum picks up and will be the first to exit when the (c) Only (B) and (C) (d) Only (A)
momentum flags, reforms or not. (e) All (A), (B) and (C)
The reforms that we need are the ones that can actually raise 238. Why does the author not recommend taking up the reforms
out. Sustainable long-term growth rate. These have to come in areas suggested by FII’s ?
like better targeting of subsidies, making projects in infrastructure (a) These will bring about only minor growth
viable so that they draw capital, raising the productivity of (b) The reforms suggested will have no effect on the
agriculture, improving healthcare and education, bringing the economy ol our country, whereas will benefit the FII’s
parallel economy under the tax net, implementing fundamental significantly
reforms in taxation like GST and the direct tax code and finally (c) The previous such recommendations had backfired
easing the myriad rules and regulations that make doing business (d) These reforms will be the sole reason for our country’s
in India such a nightmare. A number of these things do not require economic downfall
new legislation and can be done through executive order. (e) The reforms suggested by them are not to be trusted as
[IBPS PO 2012] they will not bring about any positive growth in India

Downloaded From : www.EasyEngineering.net


Downloaded From : www.EasyEngineering.net

Reading Comprehension  l 419

239. Which of the following is TRUE as per the scenario presented Passage- 55
in the passage?
(a) The highest growth rate that India can expect is 7 The great fear in Asia a short while ago was that the region would
percent suffer through the wealth destruction already taking place in
(b) The fall in the exchange rate will prove beneficial to the U.S as a result of the financial crisis. Stock markets tumbled
India as exports plunged and economic growth deteriorated. Lofty
(c) Increased FDI in retail as suggested by Flls would property prices in China and elsewhere looked set to bust as credit
benefit India tremendously tightened and buyers evaporated. But with surprising speed, fear
(d) The reforms suggested by the author require new in Asia swung back to greed as the region shows signs of recovery
legislation in India and property and stock prices are soaring in many parts of Asia.
(e) None is true Why should this sharp Asian turnaround be greeted with
240. According to the author, which ot the following reform/s is/ skepticism? Higher asset prices mean households feel wealthier
are needed to ensure long term growth in India?
and better able to spend, which could further fuel the region’s
(A) Improving healthcare and educational facilities.
(B) Bringing about reforms in taxation. nascent rebound. But just as easily, Asia could soon find itself
(C) Improving agricultural productivity. saddled with overheated markets similar to the U.S. housing
(a) Only (B) (b) Only (A) and (B) market. In short, the world has not changed, it has just moved

ww
(c) Only (B) and (C)
(e) All (A), (B) and (C)
(d) Only (A) placed.
The incipient bubble is being created by government policy. In

w.E
DIRECTIONS (Qs. 241-244) : Choose the word/group of words
which is most similar in meaning to the word/group of words
printed in bold as used in the passage.
response to the global credit crunch of 2008. Policy makers in
Asi< slashed interest rates and flooded financial sectors with cash
in frantic attempts to keep loans flowing and economies growing.

241. DRAW asy [IBPS PO 2012] Thesf steps were logical for central bankers striving to reverse a
deepening economic crisis. But there is evidence that there is
(a) entice
(c) decoy
(e) persuade
(b) push
(d) attract
En too much easy money around. It’s winding up in stocks and real
estate, pushing prices up too far and too fast for the undenying

242. CLOCK
(a) watch (b) achieve gin
economic fundamentals. Much of the concern is focused on China
where government stimulus efforts have been large and effective,
Money ir China has been especially easy to find. Aggregate new
(c) time
(e) regulate
243. ABATE
(d) second

eer
bank lending surged 201% in first half of 2009 from the same
period a yea earlier, to nearly 51.1 turn on. Exuberance over a
(a) rise
(c) hurl
(b) gear
(d) lessen ing
quick recovery which was given a boost by China’s surprisingly
strong 7.9% GDI growth in the second quarter has buoyed investor
(e) retreat
244. EMERGING
(a) raising (b) developing .ne
sentiment not just for stocks but also for real estate.
Former U.S. Federal Reserve Chairman Alan Greenspan argued
(c) noticeable
(e) uproaring
(d) conspicuous

DIRECTIONS (Qs. 245-247) : Choose the word/group of words


t
that bubbles could only be recognised in hand sight. But investor
who have been well schooled in the dangers of bubbles over the
past decade are increasingly wary that prices have risen too far
ant that the slightest bit of negative, economic news could knock
which is most opposite in meaning to the word/group of words
markets for a loop. These fears are compounded by the possibility
printed in bold as used in the passage.
that Asia’s central bankers will begin taking stops to shut off the
[IBPS PO 2012] money. Rumours that Beijing was on the verge of tightening cred
245. MYRIAD led to Shanghai stocks plunging 5%. Yet many economists believe
(a) trivial (b) difficult that, there is close to a zero possibility that the Chines government
(c) few (d) effortless will do anything this year that constitutes tightening. And without
(e) countless a major shift in thinking, the easy-money condition will stay in
246. TEPID place. In a global economy that has produced more dramatic ups
(a) moderate (b) high and downs than anyone thought possible over the pas two years.
(c) warm (d) irregular Asia may be neading for another disheartening plunge.
(e) little [IBPS PO 2013]
247. MYTH
(a) reality (b) belief 248. To which of the following has the author attributed the 2008
(c) contrast (d) idealism Asian financial crisis?
(e) falsehood A. Reluctance or Asian governments to taper off the
economic stimulus.

Downloaded From : www.EasyEngineering.net


Downloaded From : www.EasyEngineering.net

420  l  Reading Comprehension

B. Greed of Asian investors causing them to trad stocks (b) Disputing financial theories about how recessions can
of American companies at high prices. be predicted and avoided
C. Inflated real estate prices in Asian countries. (c) Warning Asian countries about the dangers of favouring
(a) None (b) Only (A) fast growth and profits over sound economic-principles
(c) Only (C) (d) (A) and (B) (d) Extolling China’s incredible growth and urging other
(e) Only (B) countries to emulate it
249. What does the author want to convey through the phrase (e) Advising governments about the changes in policy to
“The world has not changed it has just moved places”? strengthen economic fundamentals
(a) At present countries are more dependent on Asian 255. Why does the author doubt the current resurgence of Asian
economies than on the US economy economics?
(b) Economies have become interlinked on account of (a) Their economies are too heavily reliant on the American
globalisation economy which is yet to recover
(c) Asian governments are implementing the same (b) Central banks have slashed interest rates too abruptly
economic reforms as developed countries which is likely to cause stock markets to crash
(d) All economies are susceptible to recession because of
(c) With their prevailing economic conditions they are at
the state of the US economy
risk for a financial crisis
(e) None of the above
(d) Their GDP has not grown significantly during the last
250.
ww
Which of the following can be said about the Chinese
government’s efforts to revive the economy?
(a) These were largely unsuccessful as only the housing
financial year
(e) None of the above

w.E
market improved
(b) The governments only concern was to boost investor
confidence in stocks
(c) These efforts were ineffectual as the economy recovered
Passage- 56
Delays of several months in National Rural Employment
Guarantee Scheme (NREGS) wage payments and work sites where

asy
owing to the US market stabilising
(d) These were appropriate and accomplished the goal of
labourers have lost all hope of being paid at all have become the
norm in many states. How are workers who exist on the margins
economic revival

En
(e) They blindly imitated the economic reforms adopted
by the US
of subsistence supposed to feed their families? Under the scheme,
workers must be paid within 1 5 days, failing which they are
entitled, to compensation under the Payment of Wages Act - upto
251. Why do experts predict that Asian policymakers will not
withdraw fiscal stimulus? gin
3000 per aggrieved worker. In reality, compensation is received
in only a few isolated instances. It is often argued by officials that
A. The US economy is not likely to recover for a long time.
B. Stock markets are yet to regain their former levels.
C. Fear of revolt by greedy citizens. eer
the main reason for the delay is the inability of banks and post
offices to handle mass payments of NREGS wages. Though there
is a grain of truth in this, as a diagnosis it is misleading. The ‘jam’
(a) None of these
(c) (A) and (C)
(e) (B) and (C)
(b) Only (C)
(d) Only (B) ing
in the banking system has been the result of the hasty switch to
bank payments imposed by the Central Government against the

252. What do the statistics about loans given by Chinese banks


in 2009 indicate? .ne
recommendation of the Central Employment Guarantee Council
which advocated a gradual transition starting with villages
relatively close to the nearest bank. However, delays are not
(a) There was hardly any demand for loans in 2008
(b) The Chinese government has borrowed funds from the
US
(c) China will take longer than the US to recover from the
t
confined solely to the banking system. Operational hurdles include
implementing agencies taking more than fifteen days to issue
payment orders, viewing of work measurement as a cumbersome
process resulting in procrastination by the engineering staff and
economic crisis non maintenance of muster rolls and job card etc. But behind
(d) The GDP of China was below expectations these delays lies a deeper and deliberate ‘backlash’ against the
(e) None of the above NREGS. With bank payments making it much harder to embezzle
253. Why has investor confidence in the Chinese stock market NREGS funds, the programme is seen as a headache by many
been restored? government functionaries the workload has remained without
A. Existing property prices which are stable and affordable. the “inducements”. Slowing down wage payments is a convenient
B. The government has decided to tighten credit. way of sabotaging the scheme because workers will desert NREGS
C. Healthy growth of the economy indicated by GDP worksites.
figures.
The common sense solution advocated by the government is to
(a) Only (C) (b) (A) and (B)
adopt the business correspondent model. Where in bank agents
(c) All (A), (B) and (C) (d) Only (B)
will go to villages to make cash payments and duly record them
(e) None of these
on handheld, electronic devices. This solution is based on the
254. What is the author’s main objective in writing the passage?
wrong diagnosis that distance separating villages from banks is
(a) Illustrating that Asian economies are financially more
the main issue. In order to accelerate payments, clear timelines
sound than those of developed countries
for every step of the payment process should be incorporated into

Downloaded From : www.EasyEngineering.net


Downloaded From : www.EasyEngineering.net

Reading Comprehension  l 421

the system as Programme Officers often have no data on delays (e) Engineers efforts to wreck the NREGS because of low
and cannot exert due pressure to remedy the situation. Workers wages
are both clueless and powerless with no provision for them to 261. Which of the following is NOT true in the context of the
air their grievances and seek redress. In drought affected areas passage?
the system of piece rate work can be dispensed with where work A. Workers are reluctant to open bank accounts as
measurement is not completed within a week and wages may be branches are not conveniently located.
paid on the basis of attendance. Buffer funds can be provided to B. Local officials often delay wage payments in drought
gram panchayats and post offices to avoid bottlenecks in the flow prone areas to benefit workers.
of funds. Partial advances could also be considered provided C. The Government has not implemented ever)’
wage payments are meticulously tracked. But failure to recognise recommendation of the Central Employment
problems and unwillingness to remedy them will remain major Guarantee Council.
threats to the NREGS. [IBPS PO 2013] (a) Only (B) (b) (A) and (B)
256. What impact have late wage payments had on NREGS (c) (B) and (C) (d) (A) and (C)
workers? (e) All of these
(a) They cannot obtain employment till their dues are 262. Which of the following can be considered a deficiency in
cleared the NREGS?
(b) They have benefited from the compensation awarded (a) Lack of co-ordination among Programme Officers

ww
to them
(c) They have been unable to provide for their families
(d) They have been ostracised by their families who depend
(b) Local officials are unaware of correct operational
procedures
(c) Workers have no means of obtaining redressal for

w.E
on them for sustenance
(e) None of the above
257. Which of the following factors has not been responsible for
untimely wage payments
(d) Disbursing wages through banks instead of readily
accessible post offices
untimely payment of NREGS wages?
asy
(a) Communication delays between agencies implementing
the scheme
(e) The Central Employment Guarantee Council is
reluctant to award compensation to workers
(b) Improper record keeping
(c) Behind schedule release of payments by banks En Passage-57
(d) Drought conditions prevalent in the country
(e) Delays in work measurement
258. What has the outcome of disbursing NREGS wages through gin
It may be quite a while before climatologists are able to predict
rainfall in the American Midwest by measuring snow-fall in the
banks been?
(a) Theft of funds by administration officials responsible eer
Himalayas. But there is one prediction which they can confidently
make now, and that is that the earth’s ice cover — from the polar
for the scheme has reduced
(b) Increased work load for local government officials
(c) Protests by workers who have to travel long distances ing
ice caps to the Himalayas — is thawing at an alarming rate. So
much so that over 50 per cent of the planet’s mountain glacier
mass could be history by the turn of the next century. According
to the nearest bank to claim their wages
(d) Time consuming formalities have to be completed by
workers .ne
to the latest findings of the US-based environmental thinktank,
Worldwatch institute, the Arctic Sea ice has shrunk by nearly 40
(e) None of the above
259. According to the passage, which of the following has/have
been the consequence (s) of delayed wage payments?
A. Compensation to victimised workers has amounted to
t
per cent in the last 25 years, even as Antarctica’s extensive ice fields
and glaciers have been badly ‘bleeding’ at their edges. This would
indicate that the earth has entered a period of climatic change that
is likely to cause widespread environmental, economic and social
crores. disruption over the next century if emissions of heat-trapping
B. Banks will no longer be entrusted with remitting wages. gases are not reduced. As a result of global warming, average
C. Regulations to ensure punctual wage payments have planetwide temperatures have been going up steadily.
come into force. If the levels of carbon dioxide — the bad boy of global
(a) None of these (b) Only (A) warming — in the atmosphere are allowed to increase at the
(c) (A) and (C) (d) (A) and (B) present rate, more heat will be trapped in the planetary cocoon,
(e) (B) and (C) raising global temperatures to scorching highs. The polar ice caps
260. To which of the following has the author attributed the delay will melt and the resultant rise in sea levels will be catastrophic
in wage payments? for low-lying island-states and countries with large coastal
(a) Embezzlement of funds by corrupt bank staff populations, such as Marshall Islands and Bangladesh. Regional
(b) Lack of monitoring by the Central Employment flooding will threaten water supplies and dramatically alter the
Guarantee Council habitats of many flora and fauna. This is particularly bad news for
(c) An attempt to derai1 the NREGS by vested interests such regions as northern India, home to half of the total Indian
(d) Overworked bank staff deliberately delay payments to populace who depend wholly on the glacier-fed rivers for their
protest against extra work drinking water and irrigation needs.

Downloaded From : www.EasyEngineering.net


Downloaded From : www.EasyEngineering.net

422  l  Reading Comprehension

With the Himalayan ice caps melting like ice-cream on a (d) Next century will damage the glaciers heavily.
hot summer day, these snow-fed rivers will first swell and then (e) Glacier will be a subject of interest in the next century.
run dry, triggering off devastating floods, followed by a desolating 269. What efforts need to be taken to stabilise the climate?
drought. People used to think there was time to sort out problems (a) Use of carbon should be checked and alternatives for
related to climate change, but no longer. The chilling prospect of an it should be looked for to drive our industries and
imminent global glacial melt calls for immediate damage control transport.
exercises to stabilise the climate. A good way to begin, perhaps,
(b) Mountains and glaciers should be preserved at any cost
will be to overhaul the energy and transportation systems which
(c) The Himalayas should be given special protection as
drive the world’s fossil fuel economy and, instead develop low-
they shape the climate.
carbon energy systems based on electronic technologies.
(d) People should not reside near natural flora and fauna.
[SBI PO 2011]
263. Prediction of rainfall in the American Midwest depends on (e) None of these
(a) the rainfall in the region in previous years. 270. If the polar ice cap melts and the sea level rises
(b) the climatic conditions in the Himalayan region. (a) it will bring more rain.
(c) the condition of glaciers. (b) it will increase the amount of water in the world.
(d) the changing patterns of the season. (c) it will submerge low coastal areas, thus destroying life

ww
(e) None of these
264. Which of the following will be the consequence(s) if there
is a rapid decline in the ice cover of earth?
and property.
(d) hilly areas will come under water.
(e) excess water will generate diseases and epidemic.

w.E
(I) It will bring ecological disaster.
(II) It will have negative effect on the economy.
(III) It will affect the normal life mainly in the advanced
271. Give a suitable title to the passage.
(a) Glacier as a source of water
(b) Glacier causing floods
nations.

asy
(IV) It will snatch the dreamland of our poets.
(a) Only I and II (b) Only III and IV
(c) Global warming and human survival
(d) Glacier and its importance
(c) Only III
(e) None of these
(d) Only I
En (e) Save water.
DIRECTIONS (Qs.272-274) : Choose the word which is same in
265. What is the prime cause behind the shrinking of ice fields?
(a) human activity taking place at these places gin
meaning as the word given in bold as used in the passage.
[SBI PO 2011]
(b) scientific experiments being done in these area
(c) rising temperature due to pollution in atmosphere
(d) drying up of rivers which are snow-fed
272. THAWING
eer
(a) diminishing (b) receding
(e) None of these
266. What measure is imminent for saving our mountain glaciers (e) breaking
273. SCORCHING
ing
(c) evaporating (d) melting

and ice fields?


(a) launching a worldwide campaign to save them
(b) checking the emission of carbon dioxide into the
(a) extreme
(c) uneven
(b) mild
(d) odd .ne
atmosphere
(c) Making people aware of our environment
(d) depending less on the environment for livelihood
(e) None of these
(e) rapid
274. IMMINENT
(a) future (b) impending
t
(c) supposed (d) thought
267. Which of the following is false in the context of the passage? (e) surmise
(a) Worldwatch Institute is related to America.
(b) Ice cover of the earth is receding at a fast pace. DIRECTIONS (Qs. 275-277) : Choose the word which is opposite
(c) Carbon dioxide is the major contributor to global in meaning of the word given in bold as used in the passage.
warming. [SBI PO 2011]
(d) Snow-fed rivers will always have water in it. 275. SHRUNK
(e) None of these (a) developed (b) emerged
268. Over 50 per cent of the planet’s mountain glacier mass could (c) built (d) widened
be history by the turn of the next century means (e) multiplied
(a) In the next century mountain glacier will be taught as 276. CATASTROPHIC
a history subject.
(a) fortunate (b) yielding
(b) As the next century starts, over half of the mountain
(c) contributing (d) ushering
glaciers will have become extinct.
(e) jovial
(c) After the next century less than half of the planet’s
mountain glacier will be remaining.

Downloaded From : www.EasyEngineering.net


Downloaded From : www.EasyEngineering.net

Reading Comprehension  l 423

277. STEADILY 278. Which of the following facts is true as per the passage?
(a) gradually (b) systematically (a) People from rural areas have high perceived value of
(c) slowly (d) simply banking services.
(e) inconstantly (b) Cost is not a valid criterion for technological package
selection for financial inclusion initiatives
Passage- 58 (c) The inclusion segment is a singular impoverished,
undifferentiated mass
Financial Inclusion (FI) is an emerging priority for banks that
(d) The branch timings of banks generally do not coincide
have nowhere else to go to achieve business growth. The viability
with the off-work hours of the labour class in urban
of FI business is under question, because while banks and their
deliver partners continue to make investments, they haven’t seen markets
commensurate returns. In markets like India, most programs are (e) All the given statements are true
focused on customer on-boarding, an expensive process which 279. According to the passage, for which of the following reasons
people often find difficult to afford, involving issuance of smart do the delivery partners fail to serve their bank’s goal to
cards to the customers. However, large scale customer acquisition expand in the unbanked markets?
hasn’t translated into large scale business, with many accounts (A) They do not have adequate client base to sell their
lying dormant and therefore yielding no return on the bank’s financial products

ww
investment. For the same reason, Business Correspondent Agents
who constitute the primary channel for financial inclusion are
unable to pursue their activity as a full-time job. One major reason
(B) They do not have adequate knowledge and skills to
explain anything beyond basic financial products to
the customers.

w.E
for this state of events is that the customer on-boarding process
is often delayed after the submission of documents (required to
validate the details of the concerned applicant) by the applicant and
might take as long as two weeks. By this time the initial enthusiasm
(C) They do not have the skills to operate advanced
technological aids that are a prerequisite to tap the
unbanked market.
(a) Only (B) (b) Only (C)

asy
of applicants fades away. Moreover, the delivery partners don’t have
the knowledge and skill to propose anything other than the most
(c) All (A), (B) and (C) (d) Only (A)
280. According to the passage, for which of the following reasons

En
basic financial products to the customer and hence do not serve
their banks’ goal of expanding the offering in unbanked markets.
Contrary to popular perception, the inclusion segment is not a
is the viability of financial inclusion under question?
(a) The banks always prefer the cheapest package (to cut
cost) while making a choice of technology to be used
singular impoverished, undifferentiated mass and it is important
to navigate its diversity to identify the right target customers gin (b) The Business Correspondent Agents are highly
demotivated to pursue their activity as a full-time job
for various programs. Rural markets do have their share of rich
people who do not use banking services simply because they are
incovenient to access or have low perceived value. At the same eer
(c) The investments made by banks and its delivery
partners are not yielding equal amount of returns
(d) The banks do not have adequate number of delivery
time, urban markets, despite a high branch density, have multitude
of low wage earners outside the financial net. Moreover, the branch
timings of banks rarely coincide with the off-work hours of the
ing
partners required to tap the unbanked market
(e) The banks do not have adequate manpower to explore

labour class.
Creating affordability is crucial in tapping the unbanked market.
programs
.ne
the diversity of right target customers for various

281. In the passage, the author has specified which of the following
No doubt pricing is a tool, but banks also need to be innovative
in right-sizing their proposition to convince customers that
they can derive big value even from small amounts. One way of
doing this is to show the target audience that a bank account is
t
characteristics of the customer on-boarding process?
(a) It involves collection of documents from the applicants
in order to validate their details.
(b) It involves issuance of smart cards to the customers
actually a lifestyle enabler, a convenient and safe means to send (c) It suffers from latency as it takes a long time after
money - to - family or make a variety of purchases. Once banks
submission of documents by the customer.
succeed in hooking customers with this value proposition they
(d) It is an expensive process which people find difficult to
must sustain their interest by introducing a simple and intuitive
afford
user application, unbiquitous access over mobile and other touch
(e) All of the given characteristics have been specified
points, and adopting a banking mechanism which is not only
282. What did the author try to highlight in the passage?
secure but also reassuring to the customer. Technology is the most
important element of financial inclusion strategy and an enabler of (A) The ailing condition of financial inclusion business at
all other. The choice of technology is therefore a crucial decision, present
which could make or mar the agenda. Of the various selection (B) Strategies that may help bank to expand in the
criteria, cost is perhaps the most important. This certainly does unbanked market
not mean buying the cheapest package, but rather choosing that (C) Role of government in modifying the existing financial
solution which by scaling transactions to huge volumes reduces inclusion policies
per unit operating cost. An optimal mix of these strategies would (a) Both (A) and (B) (b) All (A), (B) and (C)
no doubt offer an innovative means of expansion in the unbanked (c) Only (C) (d) Only (A)
market. [SBI PO 2013] (e) Only (B)

Downloaded From : www.EasyEngineering.net


Downloaded From : www.EasyEngineering.net

424  l  Reading Comprehension

283. According to the passage, which of the following ways may But what about smaller financial businesses? They might not have
help banks to sustain the interest of their customers after such well developed strategies to protect confidential data.
hooking them? Crucially, within younger employee group, 55% of the people share
(A) Adoption of a banking mechanism which is not only an expectation that they should be allowed to use their own devices
secure but reassuring to the customers in the workplace or for work purposes. With this expectation
(B) Increasing the number of delivery partners in rural comes the very real risk that employees may consider contravening
market company policy banning the use of own devices. The threats posed
(C) Introduction of a simple and intuitive user application by this level of subversion cannot be overstated. The survey casts
(a) Only (A) (b) Only (C) doubt on the idea of BYOD leading to greater productivity by
(c) Only (B) (d) All, (A), (B) and (C) revealing the real reason people want to use their own devices.
(e) Both (A) and (C) Only 26% of people in this age group cite efficiency as the reason
they want to use their own devices, while 63% admit that the
DIRECTIONS (Qs.284-285): Choose the word which is most main reason is so they have access to their favourite applications.
similarly in meaning to the word printed in bold as used in the But with personal applications so close to hand, the risks to the
passage. [SBI PO 2013] business must surely include distraction and time wasting. To
284. Multitude support this assumption 46% of people polled acknowledged time
(a) Impoverished (b) Handful wasting as the greatest threat to the organization, while 42% citing

ww
(c) Acknowledged
(e) Solitude
285. Ubiquitous
(d) Plenty greater exposure to theft or loss of confidential data. Clearly, from
a user perspective there is great deal of contradiction surrounding
BYOD and there exists an undercurrent of selfishness where users

w.E
(a) Quintessential
(c) Omnipresent
(e) Abnormal
(b) Popular
(d) Simplified
except to use their own devices, but mostly for personal interest.
They recognize the risks to the organization but are adamant that
those risks are worth taking. [SBI PO 2013]

asy
DIRECTIONS (Qs.286-287): Choose the word which is most
similarly in meaning to the word printed in bold as used in the
passage. [SBI PO 2013]
288. According to the passage, for which of the following reasons
did Fortinet conduct the survey on a group of graduate
employees in their early twenties?
286. Dormant
(a) Emaciated (b) Pertinent En (A) As this group represents the future decision makers
(B) As this group represents the first generation who
entered the workforce with a better understanding of
(c) Cornered
(e) Active
287. Delayed
(d) Rejected
gin sophisticated gadgets
(C) As this group represents the first generation to enter
(a) Perturbed
(c) Expedited
(b) Popularised
(d) Stabilised eer
the workplace expecting that they can use their own
devices for work purpose
(a) All (A), (B) and (C) (b) Only (C)
(e) Repressed

Passage- 59 ing
(c) Both (A) and (C)
(e) Only (B)
(d) Only (A)

The evolution of Bring Your Own Device (BYOD) trend has


been as profound as it has been rapid. It represents the more .ne
289. Which of the following is not true about BYOD?
(a) BYOD enables employees to access enterprise network
from anywhere and anytime
visible sign that the boundaries between personal life and work
life are blurring. The 9 am - 5 pm model of working solely from
office has become archaic and increasingly people are working
extended hours from a range of locations. At the vary heart of
t
(b) Due to evolution of BYOD trend the 9am - 5pm model
of working solely from office has become outdated
(c) Recent research has confirmed that BYOD boosts
organisational productivity
this evolution is the ability to access enterprise networks from (d) The concept of cloud computing facilates the BYOD
anywhere and anytime. The concept of cloud computing serves trend
effectively to extend the office out of office. The much heralded 290. According to the passage, why would the decision to embrace
benefit of BYOD is greater productivity. However, recent research BYOD prove dangerous to smaller financial businesses?
has suggested that this is the greatest myth of BYOD and the reality (a) Their employees have poor knowledge about their
is that BYOD in practice poses new challenges that may outweigh devices which in turn pose a threat to the confidential
the benefits. A worldwide survey commissioned by Fortinet chose data of the organisation
to look at attitudes towards BYOD and security from the users’s (b) Their employees are more vulnerable to misplacement
point of view instead of the IT managers. Specifically the survey of devices
was conducted in 15 territories on a group of graduate employees (c) They may lack mature IT strategies and policies
in their early twenties because they represent the first generation required to protect confidential data
to enter the workplace with an expectation of own devide use. (d) They cannot afford to deal with damage liability issues
Moreover, they also represent tomorrow’s influences and decision of employee-owned devices
makers. The survey findings reveal that for financial organizations, (e) Their employees have a tendency to change jobs
the decision to embrace BYOD is extremely dangerous. Larger frequently
organizations will have mature IT strategies and policies in place.

Downloaded From : www.EasyEngineering.net


Downloaded From : www.EasyEngineering.net

Reading Comprehension  l 425

291. According to the passage, the expectation of younger 297. Subversion


employees that they should be allowed to use their own (a) Compliance (b) Sanity
devices in the workplace, entails which of the following risks? (c) Popularity (d) Destabilisation
(A) Younger employees may deliberately transfer (e) Clarity
confidential data of their companies to rivals if they are
not allowed to use their own devices for work purpose Passage- 60
(B) Younger employees may strongly feel like leaving the Stuck with the development dilemma? Stay away from
company if it presents usage of own device and join management courses. Seriously, one of the biggest complaints that
some other company that does not have such stringent organisations have about management courses is that they fail to
policies impact the participants’ on-the-job behaviour. Some management
(C) Younger employees may consider flouting company trainers stress the need for follow-up and reinforcement on the
policy prohibiting usage of their own devices in the job. Some go so far as briefing the participants’ managers on what
workplace or for work purposes behaviour they should be reinforcing back on the job. Other
(a) Only (C) (b) Only (B) include a follow-up training day to review the progress of the
(c) Both (A) and (C) (d) Only (A) participants. None of this is really going far enough.
(e) All (A), (B) and (C) The real problem is that course promoters view development
292. According to the findings of the survey conducted by

ww
Fortinet, why do majority of employees prefer using their
own devices for work purpose?
as something which primarily, takes place in a classroom. A
course is an event and events are, by definition limited in time.
When you talk about follow-up after a course, it is seen as a nice

w.E
(a) As they often find that the devices provided by the
company lack quality
(b) As they have access to their favourite applications while
working
idea, but not as an essential part of the participants’ development
programme. Any rational, empowered individual should be able
to take what has been learnt in a course and transfer it to the work

asy
(c) As majority of them believe that output doubles when
they use their own devices for work purpose
place - or so the argument goes. Another negative aspect of the
course mindset is that, primarily, development is thought to be
about skill-acquisition.
(d) As handling data from their own devices reinforces
their sense of responsiblity
En
293. What is/are the author’s main objective(s) in writing the
So, it is felt that the distinction between taking the course
and behaving differently in the work place parallels the distinction

passage?
(A) To break the myth that BYOD promotes employee gin
between skill-acquisition and skill-application. But can such a
sharp distinction be maintained? Skills are really acquired only
in the context of applying them on the job, finding them effective
efficiency and organisational productivity
(B) To suggest ways to control subversion across levels of
corporate chain of command eer
and, therefore, reinforcing them.
The problem with courses is that they are events, while
development is an on-going process which, involves, within a
(C) To throw light upon the fact that employees even after
knowing the risks involved, prefer to use their own ing
complex environment, continual interaction, regular feedback
and adjustment. As we tend to equate development with a one-off
devices for work purpose mostly for personal benefits
(a) Both (A) and (C)
(c) Only (C)
(b) All (A), (B) and (C)
(d) Only (A) .ne
event, it is difficult to get seriously motivated about the follow-
up. Anyone paying for a course tends to look at follow-up as an
unnecessary and rather costly frill. [SSC CGL 2012]
(e) Only (B)
DIRECTIONS (Qs.294-295): Choose the word which is most
similar in meaning to the word printed in bold as used in the
298. What is the passage about?
(a) personal management
(b) development dilemma
(c) management courses
t
passage. [SBI PO 2013]
294. Heralded (d) course promotors’ attitude
(a) Suspected (b) Publicised 299. Which of the following statements is false?
(c) Dragged (d) Objective (a) Some management trainers stress the need for follow-
(e) Benefit up and reinforcement on the job
295. Outweigh (b) Some suggest a follow-up training day to review the
(a) Control (b) Venture progress of the participants
(c) Perish (d) Determine (c) Some go to the extent of briefing the participants’
(e) Surpass managers on what behaviour they should be reinforcing
back on the job
DIRECTIONS (Qs.296-297): Choose the word which is most (d) The real problem is that course promoters view
opposite in meaning to the word printed in bold as used in the development as something which does not take place
passage [SBI PO 2013] during a course.
296. Embrace 300. The writer’s attitude, as reflected in the passage, is
(a) Contradict (b) Disobey (a) critical (b) ironic
(c) Curtail (d) Reject (c) sympathetic (d) philosophical
(e) Obscure

Downloaded From : www.EasyEngineering.net


Downloaded From : www.EasyEngineering.net

426  l  Reading Comprehension

301. The course promoters’ attitude is 307. What does society mean to a Gandhian?
(a) self-righteous (b) indifferent (a) a sum of individuals
(c) easy-going (d) unprogressive (b) an organic entity
302. The word ‘mindset’ here means (c) a regime of living regulated by discipline from within
(a) a determined mind (d) a disciplined social community
(b) a (fixed) attitude of mind
(c) an open mind Passage- 62
(d) mindful The stunning Baltimore Oriole is a common summer
visitor to eastern and mid western deciduous woodlands,
Passage- 61 neighbourhoods, and gardens. Baltimore Orioles winter in the
One may look at life, events, society, history, in another way. tropics. About 7 inches in length, the male Baltimore Oriole has a
A way which might, at a stretch, be described as the Gandhian way, black head, throat, back and wings. Its breast, stomach, and rump
though it may be from times before Mahatma Gandhi came on are bright orange. It also has an orange patch on the top of each
the scene. The Gandhian reaction to all grim poverty, squalor and wing and white wing bars. The tail is mostly black with orange
degradation of the human being would approximate to effort at fringes. The female is dull orange throughout.
self-change and self-improvement, to a regime of living regulated Baltimore Orioles range throughout the eastern and mid

ww
by discipline from within. To change society, the individual must
first change himself. In this way of looking at life and society, words
too begin to mean differently. Revolution, for instance, is a term
western United States, and can be found as far west as the Dakotas.
At the western edge of their range, Baltimore Orioles may breed
with the Bullock’s Oriole (They were once considered the same

w.E
frequently used, but not always in the sense it has been in the
lexicon of the militant. So also with words like peace and struggle.
Even society may mean differently, being some kind of organic
entity for the militant, and more or less a sum of individuals for
species under the name Northern Oriole).
Baltimore Orioles build unusual pouch like nests that hang
down from branches. They usually nest high in the trees, but often

asy
the Gandhian. There is yet another way, which might, for want of
a better description, be called the mystic. The mystic’s perspective
come down to lower heights, flashing bright orange and black
feathers to delighted observers Active and acrobatic by nature,
Baltimore Orioles may even feed upside down at time.
dynamism of the reformer, whether he be militant or Gandhian.
The mystic measures the terror of not knowing the remorseless En
measures these concerns that transcend political ambition and the Baltimore Orioles eat insects and berries. They can easily be
attracted to gardens by nailing orange wedges to tree branches.
march of time; he seeks to know what was before birth, what
comes after death? The continuous presence of death, of the gin
Baltirmore Orioles are also known to feed at hummingbird feeders
and sapsucker wells. [SSC CGL 2013]
308. The other name of Baltimore Oriole was _________ .
consciousness of death, sets his priorities and values: militants
and Gandhians, kings and prophets, must leave all that they have
built; all that they have unbuilt and depart when messengers of eer
(a) Bullock’s Oriole
(c) Northern Oriole
(b) Baltimore’s Oriole
(d) Southern Oriole
the buffalo-riding Yama come out of the shadows. Water will to
water, dust to dust. Think of impermanence. Everything passes.
[SSC CGL 2012]
ing
309. The nest of the Baltimore Oriole _________ .
(a) is in a tree cavity
(b) stands upon a branch of a tree
303. The Gandhian reaction of poverty is
(a) a total war on poverty
(c) hangs from a branch of a tree
.ne
(d) is usually low in the branches
(b) self-discipline
(c) self-abnegation
(d) a regulated distribution of wealth
304. According to Gandianism, the individual who wants to
Oriole ?
(a) The size of a half-scale
(b) A little more than a half-scale
t
310. Which of the following is the closest in size to a Baltimore

change society (c) A little less than a half-scale


(a) should destroy the existing society (d) A foot ruler
(b) must re-form society 311. The Baltimore Oriole spend the winters in the _______ .
(c) must change himself (a) Dakotas (b) Carolinas
(d) may change society without changing himself (c) Tropics (d) Deserts
305. Who, according to the passage, finds new meaning for words 312. What is the colour of the female Baltimore Oriole ?
like revolutions, peace and struggle? (a) Bright Orange (b) Light Orange
(a) A Gandhian who believes in non-violent revolution (c) Dull Orange (d) White
(b) A militant 313. Which of the following does not attract the Baltimore Oriole?
(c) A mystic
(a) Oranges (b) Hummingbird feeders
(d) A Gandhian who disciplines himself from within
(c) Sapsueker wells (d) Sunflower seeds
306. The expression ‘water will to water, dust to dust’ means
314. The Baltimore Oriole can be found as far west as
(a) water and dust can mix well
(a) North and South Dakota(b) The Carolinas
(b) man will become water after death
(c) California (d) Baltimore
(c) man will one day die and become dust
(d) man will become dust and water after death

Downloaded From : www.EasyEngineering.net


Downloaded From : www.EasyEngineering.net

Reading Comprehension  l 427

315. Which of the following is not true about the Baltimore 321. The writer found it difficult to keep to the path because of
Oriole? (a) the darkness and narrowness of the path.
(a) They feed upside down sometimes. (b) poor visibility and grassy track.
(b) They may breed with the Bullock’s Oriole. (c) the darkness and his slow pace.
(c) The Baltimore Oriole is uncommon in the U.S. (d) poor visibility and dew on grass.
(d) The Baltimore Oriole has a black throat. 322. When he settled himself on the fork of the tree the writer
316. Where would I probably not find a Baltimore Oriole ? ___________
(a) High in the trees (a) had a sound sleep.
(b) In gardens and neighbourhoods
(b) was disturbed by noises of animals.
(c) Deciduous woodlands
(c) was too afraid to sleep.
(d) The Sahara desert
317. Which of these colours is not found on a Baltimore Oriole? (d) tried to sleep but without much success.
(a) Purple (b) Orange
(c) White (d) Black
Passage- 64
PASSAGE - II (Q. NOS. 196-200)
Passage- 63 It is sad that in country after country, progress should

ww
As I stepped out of the train I felt unusually solitary since
I was the only passenger to alight. I was accustomed to arriving
become synonymous with an assault on nature. We who are a part
of nature and dependent on her for every need, speak constantly
about ‘exploiting’ nature. When the highest mountain in the

w.E
in the summer, when holiday-makers throng coastal resorts and
this was my first visit when the season was over. My destination
was a little village which was eight miles by road. It took only a
world was climbed in 1953, Jawaharlal Nehru objected to the
phrase conquest of Everest’ which he thought was arrogant. Is it
surprising that this lack of consideration and the constant need to

asy
few minutes for me to come to the foot of the cliff path. When
I reached the top I had left all signs of habitation behind me. I
was surprised to notice that the sky was already a flame with the
prove one’s superiority should be projected on to our treatment of
our fellowmen? I remember Edward Thompson, a British writer

En
sunset. It seemed to be getting dark amazingly quickly. I was at
a loss to account for the exceptionally early end of daylight since
and a good friend of India, once telling Mr. Gandhi that wildlife
was fast disappearing. Remarked Mr. Gandhi: ‘It is decreasing in
the jungles but it is increasing in the towns’
I did not think I had walked unduly slowly. Then I recollected
that on previous visits I had walked in high summer and how it
was October. gin On the one hand the rich look askance at our continuing
poverty; on the other they warn us against their own methods. We
All at once it was night. The track was grassy and even in
daylight sghowed up hardly at all. I was terrified of hurtling over
the edge of the cliff to the rocks below. I felt my feet squelching
eer
do not wish to impoverish the environment any further and yet
we cannot forget the grim poverty of large numbers of people. Are

and sticking in something soggy. Then I bumped into a little clump


of trees that loomed up in front of me. I climbed up the nearest ing
not poverty and need the great polluters? For instance, unless we
are in a position to provide employment and purchasing power for
the daily necessities of the tribal people and those who live in and
trunk and managed to find a tolerabley comfortable fork to sit
on. The waiting was spent by my attempts to identify the little
stirrings and noises of animal life that I could hear. I grew colder
.ne
around our jungles, we cannot prevent them from combing the
forest for food and livelihood, from poaching and from despoiling
and colder and managed to sleep only in uneasy fitful starts. At
last when the moon came up I was on my way again.
[SSC CGL 2014]
the vegetation.
t
[SSC CGL 2014]
323. At the beginning of the passage, the writer expresses her
opinion that in many countries progress is synonymous with
318. The writer felt unusually solitary because
(a) development.
(a) he was feeling very lonely without his family.
(b) utmost care for nature.
(b) he was missing the company of other holiday-makers.
(c) his destination was a little village eight miles away. (c) a balanced treatment of nature.
(d) there was no one to meet him. (d) utmost cruelty to nature.
319. “I left all signs of habitation behind me.” This means that he 324. In the passage the term ‘exploiting’ nature suggests
(a) came to a place where there were very few houses. (a) regretfulness.
(b) was in front of a large collection of cottages. (b) sarcasm.
(c) had come very far from places where people lived. (c) destructive urge of man.
(d) had just passed a remote village. (d) greed of man.
320. I became darker than the writer expected because 325. Nehru objected to the phrase ‘conquest of Everest’ since
(a) the nights are shorter in autumn than in summer. (a) it carries a war-like connotation.
(b) the nights are longer in October than mid summer. (b) it sounds pompous and boastful.
(c) the train arrived later than usual. (c) it depicts Everest as a victim.
(d) he had walked unduly slowly. (d) Everest is unconquerable.

Downloaded From : www.EasyEngineering.net


Downloaded From : www.EasyEngineering.net

428  l  Reading Comprehension

326. Gandhi’s statement ‘It is decreasing in the jungles but it is Yet the weaknesses far out-weigh the strengths. India educates
increasing in the towns.!’ approximately 10 per cent of its young people in higher education
(a) Refers to wild animals’ decrease in the jungle. compared with more than half in the major industrialised
(b) Refers to flora and fauna. countries and 15 per cent in China. Almost all of the world’s
(c) Refers to man’s selfishness. academic systems resemble a pyramid. With a small high quality
(d) Is a satirical comparison of man’s callousness to the tier at the top and a massive sector at the bottom. India has a tiny
animals. top tier. None of its universities occupies a solid position at the top.
327. The writer is of opinion that tribal people can be prevented A few of the best universities have some excellent departments and
from combing forest for food centres, and there is a small number of outstanding undergraduate
(a) to provide employment colleges. The University Grants. Commission’s recent major
(b) to increase purchasing power support of five universities to build on their recognised strength is
(c) by deterring them from poaching and despoiling a step toward recognising a differentiated academic system – and
vegetation fostering excellence. At present, the world-class institutions are
(d) to provide employment and purchasing power for daily mainly limited to the Indian Institutes of Technology (IITs), the
necessities. Indian Institutes of Management (IIMs) and perhaps a few others
such as the All India Institute of Medical Sciences and the Tata
Passage- 65 Institute of Fundamental Research. These institutions, combined,

ww
India is rushing headlong toward economic success and
modernisation, counting on high-tech industries such as
enroll well under 1 per cent of the student population.
India’s colleges and universities, with just a few exceptions, have

w.E
information technology and biotechnology to propel the nation
to prosperity. India’s recent announcement that it would no
longer produce unlicensed inexpensive generic pharmaceuticals
bowed to the realities of the World Trade Organisation while at
become large, under-funded, ungovernable institutions. At
many of them, politics has intruded into campus life, influencing
academic appointments and decisions across levels. Under-
investment in libraries, information technoloy, laboratories,

asy
the same time challenging the domestic drug industry to compete
with the multinational firms. Unfortunately, its weak higher
and classrooms makes it very difficult to provide top-quality
instruction or engage in cutting-edge research.

En
education sector constitutes the Achilles’ Heel of this strategy. Its
systematic disinvestment in higher education in recent years has
yielded neither world-class research nor very many highly trained
The rise in the number of part-time teachers and the freeze on
new full-time appointments in many places have affected morale
in the academic profession. The lack of accountability means
scholars, scientists, or managers to sustain high-tech development.
India’s main competitors especially China but also Singapore, gin
that teaching and research performance is seldom measured. The
system provides few incentives to perform. Bureaucratic inertia
Taiwan, and South Korea are investing in large and differentiated
higher education systems. They are providing access to large
number of students at the bottom of the academic system while eer
hampers change. Student unrest and occasional faculty agitation
disrupt operations. Nevertheless, with a semblance of normality,
faculty administrators are able to provide teaching, coordinate
at the same time building some research-based universities that
are able to compete with the world’s best institutions. The recent ing
examinations, and award degrees.
Even the small top tier of higher education faces serious problems.
London Times Higher Education Supplement ranking of the world’s
top 200 universities included three in China. three in Hong Kong.
Three in South Korea, one in Taiwan, and one in India (an Indian .ne
Many IIT graduates, well trained in technology, have chosen not
to contribute their skills to the burgeoning technology sector in
India. Perhaps half leave the country immediately upon graduation
institute of Technology at number 41- the specific campus was
not specified). These countries are positioning themselves for
leadership in the knowledge-based economies of the coming era.
There was a time when countries could achieve economic success
to pursue advanced study abroad – and most do not return.
A stunning 86 per cent of students in science and technology
fields from India who obtain degrees in the United States do
not return home immediately following their study, another
t
with cheap labour and low-tech manufacturing. Low wages significant group, of about 30 per cent, decides to earn MBAs in
still help, but contemporary large-scale development requires a lndia because local salaries are higher – and are lost to science
sophisticated and at least partly knowledge-based economy. India and technology. A corps of dedicated and able teachers work at
has chosen that path, but will find a major stumbling block in its the IITs and IIMs, but the lure of jobs abroad and in the private
university system. sector make it increasingly difficult to lure the best and brightest
India has significant advantages in the 21st century knowledge to the academic profession.
race. It has a large higher education sector - the third largest in Few in India are thinking creatively about higher education. There
the world in student numbers, after China and the United States. is no field of higher education research. Those in government
It uses English as a primary language of higher education and as well as academic leaders seem content to do the “same old
research. It has a long academic tradition. Academic freedom is thing.” Academic institutions and systems have become large
respected. There are a small number of high quality institutions, and complex. They need good data, careful analysis, and creative
departments, and centres that can form the basis of quality sector ideas. In China, more than two-dozen higher education research
in higher education. The fact that the States, rather than the Central centres, and several government agencies are involved in higher
Government, exercise major responsibility for higher education
education policy.
creates a rather cumbersome structure, but the system allows for
a variety of policies and approaches.

Downloaded From : www.EasyEngineering.net


Downloaded From : www.EasyEngineering.net

Reading Comprehension  l 429

India has survived with an increasingly mediocre higher education 331. In writer’s opinion which of the following reason(s) is/are
system for decades. Now as India strives to compete in a globalised responsible for poor higher education in India?
economy in areas that require highly trained professionals, the I. India’s colleges and universities, with some exceptions,
quality of higher education becomes increasingly important. have become large under funded, ungovernable
institutions.
India cannot build internationally recognised research-oriented
II. Politics has intruded into many compuses that
universities overnight, but the country has the key elements in
influences academic appointments and decisions across
place to begin and sustain the process. India will need to create levels.
a dozen or more universities that can compete internationally III. Under investment in libraries, laboratories, IT and
to fully participate in the new world economy. Without these classrooms hinder cutting edge research.
universities, India is destined to remain a scientific backwater. (a) Only I (b) Both 1 and II
[SBI PO, 2014] (c) Both II and III (d) All I. II and III
328. Which of the following statement(s) is/are correct in the (e) None of these
332. Which of the following statements is not true as per the
context of the given passage ?
given information in the passage ?
I. India has the third largest higher education sector in (a) About fifty per cent of IIT graduates leave India to
the world in student numbers.

ww
II. India is moving rapidly toward economic success and
modernisation through high tech industries such as
pursue advanced study abroad.
(b) About 86 per cent of students in science and technology
fields from India who obtain degrees in US do not

w.E
information technology and biotechnology to make
the nation to prosperity
III. India’s systematic disinvestment in higher education
return home following their study.
(c) In China more than two-dozen higher education
research centres and several government agencies are

asy
in recent years has yielded world class research and
many world class trained scholars, scientists to sustain
involved in higher education policy.
(d) The rise in the number of part-time teachers and the
freeze on new full-time appointments in many places
high-tech development.
(a) Only I (b) Only II
En have boosted morale in academic profession in India.
(e) None of these
(c) Both I and II
(e) All I, II and III
(d) Both I and Ill

329. Which of the following statements in regard to the gin


333. What in your opinion should be an appropriate title of the
given passage ?
(a) Poor state of Higher Education in India
information given in the passage is not true ?
(a) The London Times Higher Education Supplement eer
(b) Politics in India’s Education system
(c) Modernisation of Indian Education System
ranking of the world’s top 200 universities has included
three universities of South Korea. ing
(d) Higher Education Supplement Ranking
(e) None of these
DIRECTIONs (Qs.334-335) : Choose the word/group of words
(b) India has recently announced not to produce
unlicensed inexpensive generic pharmaceuticals that
.ne
which is most similar in meaning to the word/group of words printed
in bold as used in the passage.
will be a challenge for the domestic drug industry to
compete with the multinational firms.
(c) Contemporary large-scale development requires a
sophisticated and at least partly knowledge-based
334. Achi1les’ Heel
(a) weakness
(c) acquiescene
(e) strong heel
(b) strength
(d) vulnerable
t
economy. 335. Sustain
(d) China has the fourth largest higher education sector in (a) suffer (b) maintain
the world. (c) swag (d) swallow
(e) weaken
(e) None of these
DIRECTIONS (Qs.336-337) : Choose the word/ group of words
330. According to the view expressed by the writer in the passage, which is most opposite in meaning to the word / group of words
what is a step toward recognising a differentiated academic printed in bold as used in the passage.
system and fostering excellence ? 336. Cumbersome
(a) The University Grant Commission’s recent major (a) complicated (b) complex
support to five universities to build on their strength. (c) simple (d) bulky
(b) New Education policy of the new government. (e) heavy
(c) Scholarships granted by the Central government for 337. Differentiated
research. (a) Distinguished (b) similar
(c) distinct (d) undistinguished
(d) Government policy to open new world class institutions
(e) distraught
(e) None of these

Downloaded From : www.EasyEngineering.net


Downloaded From : www.EasyEngineering.net

430  l  Reading Comprehension

Hints & Solutions


LEVEL- I 20. (d) Neither 1 nor 2 is correct as the dominant species is the
keystone species and that is sea star. The sea stars do not
1. (a) Statement 1 is correct as passage clearly states ‘‘In all live exclusively on mussels as their removal resulted in the
cases, it is the ... behalf of the child.’’ disappearance of 28 species more.
2. (b) Statement 1 is not correct as not stated by the passage. 21. (c) Only 1 and 3 are correct statements as the ‘sea stars’
Statements 1 and 2 are correct as stated in passage are the keystone species which influences both richness of
‘‘This is ... to the education of girls.’’ Statement 4 is correct communities and flow of energy.
as the faulty education system. 22. (c) Assumption 1 is correct as disappearance of 28 species
3. (a) Statement 1 is correct as stated in the passage ‘‘one
along with mussels. Assumption 3 is also correct according
hardly needs .... denied or violated.’’ Statement 2 is irrelevant
to the passage.
as developed countries have not been mentioned in the
passage. 23. (c) With reference to the passage, only 3 and 5 are adverse
effect as potable fresh water and bio-diversity are drastically
4.

5. ww
(c) Statement (c) is correct as the passage clearly states that ‘‘
This may be painfully ...if at all.’’
(a) Since girl’s education is the key massage, so, prejudice
against the intellectual potential of girls.
24.
affected.
(a) Statement 2 is not correct as the efforts are being taken.
3 is also not correct as the short term gain have resulted into
6.
w.E
(a) Statement 3 is eliminated as the government should side
line with minimal interference. Statement 4 is eliminated as
change in size of government.

25.
long term degradation of other.
(c) Both statements are correct as ecosystem provide
people with variety of goods and benefits, they have to be
7.
asy
(d) Strategy of inclusive growth can be effected by focusing
on delivery of the basic services to the deprived section of
the society. It is clearly stated in the passage ‘‘The aim must
modified considering the population expansion. Secondly
technology will always remain expensive and nature is
above man.

8.
be .... state can realistically deliver.’’
(b) According to given passage only 4 and 5 are that En 26. (d) Statement 1 is correct as the writer talks of ‘moral act
that should be done by our own will’. Statement 4 is also
9.
constitutes.
(b) State does not promote inclusive growth as stated in
2nd para ‘‘ the aim ... realistically deliver.’’ gin correct as the personal thinking and in order to be moral
one can defy convention.
10. (d) Options (a), (b) and (c) cannot be the message. The
only sentence that talks of a message is option (d) which is

27.

28. eer
(b) According to the writer moral action is neither
mechanical nor with, clarity of purpose and religions action.
(b) Only statements 1 and 2 are correct as let himself be
11.
the main idea of the passage.
(c) 1 is eliminated as ‘‘ art form’’ is not mentioned in the
passage. Social inequalities are not accepted. Only 3 and 4
29.
ing
swept away means he does not hold his own ground.
(a) The passage clearly suggests that education is not
instrumentalist in its deepest sense. But the opening sentence
12.
are mentioned.
(b) According to passage Instigation by external forces
(social group get politically .... ) and ‘‘urge for granting .ne
calls it to be functional, instrumental and utilitarian. Thus
the instrumentalist view of education is the functional and

13.
privileges and self respect to disparaged section of the
society’’ are manifestations of social movements.
(c) Statement 1 is correct as stated in the passage ‘‘The
forms of .... in a country’’. Statement 2 is correct as ‘‘phase
30.
utilitarian dimension in its purposes.

t
(c) The second part of the passage clearly states education is
not a commodity but a process of expansion and conversion
of the mind – the moral-intellectual development. Acquiring
of development ..... active’’. qualifications, upward mobility and social status are the
14. (a) basic utility values of education.
15. (a) Only statements 1 and 2 are true as people who turn 31. (c) Again the second part clearly states the answer. a, b &
revolutionary are not ‘other wordly’ unless world shows the d are the utilitarian dimensions of education. But ultimately
contradiction. education leads to self-critical awareness and independence
16. (c) Only assumptions 2 and 3 are valid as for achieving of thought.
anything great one has to be free of worries of basic needs 32. (a) Fermi used method to calculate a approximation
and should be mentally as well as physically free. estimate for the yield from the blast.
17. (a) The passage the matically centres on the condition as \ The correct answer is option (a).
they cannot dream of freedom or aspire for any kind of 33. (d) Quick estimates as per fermi is most useful in finding
opportunity. out the range of values of an estimate.
18. (c) Option (c) is the correct answer as stated in the passage 34. (d) The focus of passage is how minimal information can
‘‘ Every civilization.....life’’ be used for estimation. this makes statement (2) the only
19. (c) Option (c) is correct answer as the author shows that correct answer.
not only mussels are affected but other 28 species also 35. (e) None of the statements can be inferred. Hence the
disappeared. correct answer is option (e).

Downloaded From : www.EasyEngineering.net


Downloaded From : www.EasyEngineering.net

Reading Comprehension  l 431

36. (d); Statements (a), (b) and (c) are all describing acts that 49. (d) The passage talks about the new age groupie and Herman
might be committed by someone who is a congenial person Hesse in one breath, making option (d) correct.
and is sociable. For example, someone who can entertain 50. (b) The passage discusses the new ideas that have come up
women cannot be described as a person who does not and TED is an example of such an idea.
understand the general mood of the party. On the other 51. (a) The phrase harks back to an era where everything was
hand, statement (d) states that an unseasonable man will sepia toned, implying an era long ago.
tell a long story to people who have heard it many times 52. (d) Even though the passage is extremely gung ho about TED,
before. This kind of an act will incite feelings of boredom nowhere is the fact of its being irreplaceable mentioned.
and irritation in the listeners. This is typical behaviour for
53. (a) Victuals means food and Dandyism is associated with
an unsociable person.
fashion, thus, option 1 is correct.
37. (a); It can be seen that option (a) is one that can cause
considerable aggravation to the salesperson. This salesperson 54. (c) The author feels that it is not a novelty, hence is quite
has just sold something to someone and just after this, our disturbed by it being treated as one.
unseasonable man presents to him a client that could have 55. (d) The best word would be option 4, as the others are not
offered him more than his last sales price. This would be in line with the chain of thought.
vexing for anyone and hence can be identified as a doing 56. (b) The author is quite contemptuous of the entire
of an unseasonable man. excitement hence option 2 is correct.
38.
39.
(c)
ww
(d) Rhetoric refers to ‘speaking/writing which is intended
to persuade’. Rhetoric’s different degrees are mentioned in
57.

58.
(e) The write is quite depressed with even the thought of
the existence of such a system, hence option 5 is correct.
(d) It was the time of recession and external labour was very

w.E
the third paragraph, and this is also what the other options
refer to. Though, the commands which the army officers
give are free from any rhetoric traits. 59.
expensive, thus the live labourers could be used to do the
external work too.
(b) The author is unable to understand this harsh parenting
40.
asy
(c) Option (c) is correct because the same has been said
in the opening paragraph -. Economics uses mathematical
models and statistical tests and market arguments, all of
60.
process in practice.
(b) Sokal felt that the theories were more of jargons than
any real theory being stipulated.

41.
which look alien to the literary eye.
En
(a) Arcane is used to define something which is known only
61. (a) The idea was to lampoon the so called established
writers, making option 1 correct.
by a few people; and the author states that the culture of
the language used by economists make the words arcane.
Hence, the most appropriate meaning of the word arcane
gin
62.

63.
(e) The passage is about the person who dared to debunk
mathematics making option 5 correct.
(c) The passage is talking about the all around effect of global

42.
is given by option (a).
(d) One look at the fifth paragraph will reveal that the 64.
warming.
eer
(b) The author is talking about the biggest threat to our lives.

43.
correct answer has to be option (d).
(b) From the final statement of the passage, we can say
that option (b) is the required answer -To teach how to
65.
ing
(c) Though the passage depicts the effect on a few
individuals, nowhere does it state that everyone will get
affected similarly.
live without certainty, and yet without being paralyzed by
hesitation, is perhaps the chief thing that philosophy, in our
66.
recession in India. .ne
(c) The passage discusses the up and down swing of the

44.
age, can still do for those who study it.
(b) Option (b) comes out to be our answer for the given
question. The author states that philosophy has the traits of
theology, but it still cannot be put under these categories, so
67.

68.
by the investors was detrimental.
t
(e) The author mentions that the taking back of the money

(c) The passage mentions that there was a decrease in the


indices.
in a way, science and philosophy complement each other. 69. (e) The passage traces thought of the book writer who
explores this chain of thought.
Hence, options (c) and (d) can be ruled out. Option (a) is
70. (c) The passage is a book review of Gandhi Before India by
wrong, because antagonistic means “indicating opposition”,
Ramachandra Guha.
but from the given passage, we cannot say that about science 71 . (a) Guha, discusses that his method is better because he
and philosophy. uses data and may even reject Gandhiji’s view point in some
45. (c) cases.
46. (d) Option (d) is the required answer, because we cannot 72 . (c) The fact that the iPad is doing good business is one of
infer option (a), (b) or (c) from the information provided the reason’s of the PCs decline.
by the given passage. 73
. (e) The paragraph consistently mentions the flip-flop
47. (b) “But he told me what I wanted to hear, and a quarter attitude of the IDC.
century later, philosophy is making the kind of…”these lines 74 . (c) The paragraph on the Japan economy pointing north, is
speak of the profession which he eventually took up. the one where the economy gaining some stability in Japan
48. (a) The writer mentions that his father wanted him to have is being discussed making option 3 correct.
an average life where he would be happy which was far 75 . (e) The passage discusses the types of idioms and their
removed from his aspirations. variation.

Downloaded From : www.EasyEngineering.net


Downloaded From : www.EasyEngineering.net

432  l  Reading Comprehension

76 . (a) The writer quotes John Saeed who talks about idioms 99 . (a) Option (a) is correct as an article in an economics based
and their being there through ancient times. journal as it takes in the economics of the situation, option
77 . (a) The passage discusses the idioms and how they have (b) is incorrect as it lacks the detailing which a book involves,
evolved, which makes option 1 correct. and option (c) is incorrect as it is not a thesis because of
78 . (c) Prow means the front part of a ship and joust is an the details presented in the concise version. Option (d) is
old-fashioned fight on horseback. Since the location of the incorrect as it does not present the entire philosophy Option
(e) is incorrect as the reporting element of a newspaper
yachts in the sea is the best answer; hence would be option
report is missing.
(c) 100. (e) The statement makes nature a necessary part of nurture,
79 . (a) The passage mentions a certain class of wealthy people this makes option (e) correct.
but nowhere does it specify a system akin to the British, 101. (a) The passage states that in humans genetics is supreme
making 1 the correct answer. All the other points are and nurture has little effect on this aspect.
mentioned in the passage. 102. (c) The example of the Nazi misuse of the genetics theory
80 . (b) Option (b) is the correct answer as all the other options put off most socialists as it became a tool to secure power.
are mentioned as examples of the Jewish way of life, only 103. (c) The passage states that the human mind is extremely
option (b) is not and hence is the correct answer. malleable making option (c) correct.
81 . (d) Tolkin finds them typical and unbearable while 104. (c) The passage states, ‘If the garden space was used in the

ww
conspicuously consuming Hebraism
82 . (b) The passage is mainly concerned with establishing
the theory that endurance running has been inherent in
summer, it must be cleaned out to eliminate all the old
plants, including all the roots. A tiller will soften and aerate
the soil; it will help loosen any roots missed.’ This implies
that if the old roots are left underground, the new roots will

w.E
mankind making option (b) the correct answer. All the
other options are incomplete and hence incorrect.
83 . (e) Options (a)-(d) hold true if the toe length had remained 105.
not be able to take good hold.
(a) The passage states, ‘Using lawn timbers is an excellent idea
because they help control weeds and are very inexpensive;
not support this and is hence not true.
asy
long. Option (e) is the correct answer as the passage does

84 . (d) It has been stated in the passage that long toes put stress
106.
seconds sell for about a dollar each.’
(d) The passage doesn’t discuss the drawbacks of fall gardens,
while it does concern itself with the other options at some
on the joints and the foot which makes options (a) & (b)
correct.
En
85 . (d) According to the passage Derrida is a champion of the 107.
points.
(c) The passage makes it clear that Taoism does not work by
written word making option (d) correct.
86 . (c) Saussure was antithetical to Derrida’s school of thought gin making one ‘unlearn’ conventional learning. On the other
hand, it tries to correct the imbalance that has been brought
about by too much of conventional knowledge. All other
and felt that language was independent of writing.
87 . (c) The main point of discussion in the passage is that
language existed to translate speech into words. 108. eer
inferences are validated by the passage.
(c) The passage says, ‘Now the general tendency of the
88 . (b) The author is methodical in his/ her approach as s/he
analyses the various points of view available on the subject.
89 . (c) The passage makes no mention of them, making it the ing
Western mind is to feel that we do not really understand
what we cannot represent, what we cannot communicate.
For some reason Westerners do not trust and do not fully
correct answer.
90 . (d) The 1st paragraph mentions the collaboration and the
109.
(c) correct.
.ne
use the “peripheral vision” of our minds.’ This makes option

(a) A study of the passage suggests that Taoism, even though


reasons for this feeling.
91 . (d) The last passage elucidates the author’s feelings on the
subject making option 4 the correct answer.
92 . (e) Option (e) is correct, a libertarian is a person who
believes in limited state intervention.
t
it does not stand in direct opposition to structured Western
philosophy and concepts, does form an alternative that seeks
to liberate the individual from the repression of structures
of conventional knowledge and modes of thought.
93 . (e) The last line of the passage conveys the main idea which 110. (c) The passage states, ‘Some have argued that religious
is books in whatever format are known for their substance. phenomena-matters of faith-are entirely beyond the ken
94 . (a) The onus of reading lies mainly on the content, the other of science; but this surely is false because the scientific
aspects are secondary making option 1 correct. investigation of religion has already made great strides and
95 . (c) The author is quite practical and explores all the new there is a vast literature now available.’ This makes it clear
fangled book readers available and comes to a conclusion. that the passage sets out to establish that religion, like any
96 . (d) The term bubble is used to highlight the alarming trends other field of human knowledge, is open to examination by
in higher education which are like the dotcom bubble and science.
will not be able to sustain the artificial rise and may similarly 111. (c) The second paragraph of the passage makes it clear that
burst. the author is in favour of an interdisciplinary approach that
97 . (b) Option (b) is the best answer as it captures the main includes a variety of methods of analysis while investigating
idea of the passage all the others are perfunctory or trite in religious claims.
nature. 112. (d) The passage states, ‘…religion has its roots in ethnic
98 . (e) All the options except (e) are mentioned in the passage, 5 or national identity; and to question the empirical or
can be inferred because the private institutes are beginning rational grounds for religious beliefs is to shake at the very
to wonder about enrolments. foundations of the social order.’ This adequately implies

Downloaded From : www.EasyEngineering.net


Downloaded From : www.EasyEngineering.net

Reading Comprehension  l 433

that a questioning of religious beliefs is likely to create passage, “Basing their work in empirical data might also
tremendous furore and an accusation of blasphemy and help politicians—for example—see directly how economic
heresy towards the questioner of religion. insights can be useful when governing.”
113. (b) Option (a) is incorrect because although the poverty rate 117. (c) Option (a) is not correct because this statement is
in India being high despite economic expansion explaining contrary to Kant’s theory that time is “not ‘something
India’s position but this not the paradox that the author is existing in itself ’. Option (b) is incorrect because Kant did
referring to. Option (c) is incorrect because again there is not believe that time is an individual characteristic. Option
nothing in the passage to suggest that there are very good (d) is not correct because the given theory is again contrary
universities in India. Option (d) is not correct because to Kant’s views. Option (c) is the correct option because
this statement is not really a paradox that the author is the following lines in the passage support this statement,
suggesting in the passage. It is an explanation to the main “Gödel recognized that for Kant, time is only a characteristic
paradox. Option (b) is the correct answer because the inherent in the relation of the objects to something else.”
following lines in the passage show that this is the paradox 118. (d) Option (a) is incorrect because the author says that
that the author is referring to and the others are supporting literally Kant’s assertions may be false but some deeper
statements to this argument, ‘India churns out lots of brain truth is hidden in them and hence all of them are not false.
boxes, including clever economists, Yet the performance of Option (b) is incorrect because according to the passage
the Indian economy itself, over the decades, fell well behind the following lines support the given statement, “Feuerman,
114.
ww
(most of) the rest of Asia.’
(c) Option (a) is incorrect because statement 1 is valid as
the following line in the passage supports it, ‘But actually
nonetheless, argues for a level of coherence ultimately to be
found in Gödel’s views” Option (c) is not the correct option
because the following line in the passage supports this

w.E
doing something useful then requires describing and
explaining it in a way that others can understand.’ Statement
2 is not valid because there is nothing in the passage to
suggest that economists go abroad because of better living
argument, “finite mathematics is defined as the mathematics
in which evidence rests on what is intuitive.” Option (d)
is the correct option because Kant did not agree with the

asy
standards there. Hence statement 1 is valid but statement 2
is not valid and since both statements together are not valid
this is not the right answer. Option (b) is not the correct
Relativity theory which stated that time is one dimensional
whereas Kant believed that time was relative to a subject and
its sensibility.

En
option because statement 1 is valid as already shown and
statement 3 is also valid because the following line supports
119. (a) Option (b) is incorrect because there is only a reference
in the passage as to how Kant differed with the Theory of
Relativity. Option (c) is not correct because again only in the
it, “be confronted by bright Indians offering sharp analyses
of how best to fight poverty, create wealth and promote
innovation.” Hence both statements 1 and 3 are valid and gin first paragraph does the author talk about some difference
in opinion but subsequently there is no mention about it.
the answer b is wrong. Option (d) is incorrect because out
of the given 3 statements all are not valid. As discussed only eer
Option (d) is not the correct option because the passage
does not focus entirely on the theory of Relativity; there is
only a passing mention about it. Option (a) is the correct

115.
statement 2 is not valid. Option (c) is the correct option
because only statement 2 is incorrect and not valid.
(d) Option (a) is incorrect because the passage does not ing
option because the entire passage is describing Gödel’s
views on mathematics, finite and infinite and theories of
time.
suggest that economists have no interest in solving the
problem. Option (b) is not correct because again there
is nothing in the passage to suggest that the government .ne
120. (b) Option (a) is incorrect because according to the passage,
“for Kant, time as “a one-dimensional temporal ordering
wants to keep economists away from solving economic
problems. Option (c) is incorrect because although the
passage does mention that bright economists live abroad
but that does not debar them from helping in solving their
t
of the events” whereas the given statement states that time
is a multi-dimensional ordering of events. Option (c) is
not correct because the given statement is one of Gödel’s
theories as is evidenced by the following line in the passage,
country’s problems. Option (d) is the correct option because “Gödel addresses how finiteness, associated for example
the following line in the passage supports this argument, with inductive proof, is intuitive: “ finite mathematics is
‘perhaps the country’s brightest economists are simply defined as the mathematics in which evidence rests on what
rubbish at communicating sharp ideas to the policymakers, is intuitive.” Option (d) is incorrect because the given theory
activists, media types, business leaders and members of the is against Kant’s theory because in the passage, after the given
public who could make use of them.’ theory, the words instead, for Kant… show that Kant did
116. (a) Option (b) is incorrect because there is nothing in the not agree with the theory. The relevant lines are reproduced,
passage to suggest that politicians and economists do not “idea that time “as its most essential characteristic...consists
communicate. Option (c) is not correct because again there [in the traditional view] of a one-dimensional system of
is nothing to suggest that politicians are not economists points, isomorphic with a straight line, in which every
and this is a reason for poor governance. Option (d) is happening in the world has a definite place.” Instead, for
incorrect because there is a reason amongst the available Kant, time as “a one-…..” Option (b) is the correct option
options. Hence Option (a) is the correct option because the because the following lines in the passage support this
writer does mention that the data analysis of economists statement, “Instead, for Kant, time as “a one-dimensional
if conveyed in simple language may help politicians to temporal ordering of the events” is the case, and is “relative
govern, as is substantiated by the following lines in the to the perceiving subject or more precisely its ‘sensibility’.”

Downloaded From : www.EasyEngineering.net


Downloaded From : www.EasyEngineering.net

434  l  Reading Comprehension

121. (c) Eminent British economists and political scientists 175. (b) The whole passage emphasises on the popularity of
have strongly attacked the tradition of budget secrecy. internet and checking bank accounts and maintaining
122. (e) It leads to the control of public expenditure in order to financial assets is the most popular usage of Internet
set realistic taxation implications. 176 (c) 177 (a) 178 (c) 179 (b)
123. (b) He has presented the example of both, the open budget
system and the secret budget system, practised by 180 (a) 181 (b) 182 (a) 183 (a)
various countries and has looked into all their aspects. 184 (d) 185 (c) 186 (d) 187 (a)
124 (d) 188 (d) 189 (b) 190 (d)
125 (e) 191. (b) Modern means of entertainment and communication
126. (a) Sir Richard Clarke was the originating genius of nearly does not affect street theatre. It is still a popular mode
every important development in the British budgeting of taking up crucial societal issues.
techniques during the last two decades. 192. (a) In the olden days, street theatre was restricted to villages
127 (b) or small localities of the cities.
128. (a) The statement goes against the idea of the passage. 193. (c) Street theatre usually deals with issues of public
129. (d) An open public debate on budget proposals should be importance.
held before introducing the appropriate bill. 194. (c) Street theatre is reasonable to stage as little props and
130 (c) 131 (d) 132 (a) 133 (b) images are used and not huge set up is required.
134 (c)
ww 135 (e)
136. (a) Ascertain the hidden meaning of the sentence : “but
no one would be able to realise that a terrorist attack
195. (b) Street theatre creates an emotional impact on audiences
that leads to quick psychological impact.
196. (a) In self directed learning, an individual takes initiative

w.E
has occurred”. So, undoubtedly the culprit’s act can be
classified as a terrorist attack.
137. (b) “New terrorism has no long-term agenda but its ruthless
with or without the help of others to learn new things.
197. (b) there is need for self-directed learning because it helps
people to learn more things in a better way.

asy
in its short-term intentions”. This statement from the
passage supports (B). While, in the light of passage,
(C) also seems suitable.
198. (a) Self-directed learning is active learning as one does
not sit passively and waits for someone to teach. The
learner actively initiates its own learning process.
138. (e) The immediate provocation for the meeting held in
En
August 1998 has not been given among the options.
199. (b) The modern environment is instructive in nature.
200. (c) Diagnosing means identifying.
It was the incidents of bombing the US embassies in
Nairobi and Dar-es-Salaam.
139. (e) Bio-attack will result in several deaths which will lead gin
201. (b) 202. (b) 203. (b) 204. (a) 205. (c)
206. (a) Sheela and Jairam had to sell their cow because they

140 (c)
to political turmoil, creating social unrest.
eer
needed money for Jairam’s sickness. They were a very poor
old couple.
207. (a) Jairam was pretending to be reluctant to sell the rabbit
141. (a) ‘Religious intolerance’, as cited in the last paragraph,

142 (d)
stands behind terrorism.
143 (d) 144 (a) 145 (c) to him. ing
so that the bullies would realise how precious the rabbit was

146 (b)
150 (b)
147 (a)
151 (a)
148 (e)
152 (d)
149 (d)
153 (a)
.ne
208. (a) ‘Magic in the Air’ has been used repeatedly throughout
the passage. It is the most appropriate title.
209. (b) She was very upset. So she came back home and told
154 (a)
158 (a)
162 (b)
155 (b)
159 (c)
163 (b)
156 (e)
160 (b)
164 (a)
157 (c)
161 (e)
165 (b)
166. (c) Almost 57% of Indian respondents using the internet
prefer to bank online. Checking information on products
her husband the whole story.
t
210. (d) The four bullies first tied a goat to Sheela’s rope.
211. (c) The four bullies were surprised because it was different
to believe that the rabbit could understand obey Jairam.
and services online comes a close second to 53% while 212. (e) The four bullies thought it was the same rabbit sitting
50% shop for products online. The fourth on the list- in a corner.
around 42% of respondents in India surfed online to 213. (c) The first event is of the couple being needy as in (A).
look for jobs. Hence, option (c) is correct choice. Among the various things they tie to the rop is a log of
167. (d) Only 50% Indian shop for products online while wood as in (D). The next given event in terms of the four
statement said majority of Indian which is not true. sentences can be found in (C) where Sheela cooks for the
168. (e) four bullies. Once they buy the rabbit they send it to the
169. (d) 50% Indian using the Internet shop online. landlord to extract money as in (B) and when they reach
170. (b) Hassle free means without problems or bother. his house they get good thrashing.
171. (e) 214. (b) Fouram decides to take revenge on the bullies and trick
172. (a) The whole passage describes the growing usage of them in the same manner as they had tricked his wife. So
internet. Hence, option (a) is right choice. he asks her to cook for them.
173. (c) 215. (d) The landlord was so angry that he had the bullies
thrashed by his strongest bodyguard.
174. (d) Eliminated means removed as used in the passage.

Downloaded From : www.EasyEngineering.net


Downloaded From : www.EasyEngineering.net

Reading Comprehension  l 435

216. (b) Thrashing means to give a sound beating. elimination of maladapted individuals from a population
217. (e) In this case possession means belonging. is natural selection.
5. (c) 1 is not correct as organic farming is not mentioned in
218. (a) Vanished means had got lost.
the passage. Further the passage justifies the cost benefit to
219. (d) In this case agreed means to say yes, so the opposite developed countries like USA.
is declined which means to say no. 2 is correct. Because of this problem it becomes imperative
220. (c) Here dejected means sad or upset which is the same to use pesticides.
as crest fallen. 3 is wrong as the social and health costs have to be ignored
221. (c) Haridatta’s son was greedy because he tried to kill the because of the frightening prospects of the epidemic
cobra to collect all the gold at an instant. diseases.
222 (b) 6. (b) The passage does not talk about any alternative option
223 (b) to chemical pesticides. It talks about a balance being drawn
224 (a) between sustainability and use of pesticides.
225 (c) 7. (a) Only 1 makes sense. According to the 3rd paragraph
226. (d) Humble (adj.) means marked by meekness in behaviour, second line, ‘And some growth ........................... natural
attitude, or spirit; not arrogant or prideful. resources.’ 2, 3 & 4 are irrelevant statements.
Hence, Meek is the most similar in meaning to word

ww‘Humble’ as used in the passage.


227. (e) Astonished (verb) means to fill with sudden wonder or
amazement or surprised.
8. (d) The options provided in the question does not imply
low-carbon growth.
A number of low-carbon growth options exist for reducing

w.E
Hence, surprised is the most similar in meaning to word
‘Astonished’ as used in the passage.
228. (a) Stretch out (noun) means lie down comfortably.
our net greenhouse gas emissions, particularly carbon
dioxide, which could be implemented over different time
horizons. These include: (1) improved efficiency in energy

asy
Hence, lie down is the most similar in meaning to word
‘stretch out’ as used in the passage.
use, especially over the short to medium term, through
technological and behavioral changes; (2) producing
energy which minimizes carbon dioxide emitted, especially
229 (d)
230 (d)
En for new power plants, and realistically over the medium
to long term; and (3) reducing carbon dioxide produced
LEVEL-II
1. (d) Natural selection is a key mechanism of evolution. It gin in non-energy sectors, such as agriculture and forestry,
and industries, such as cement production. In addition
to these, technologies are being developed to capture and
is the gradual, non-random, process by which biological
traits become either more or less common in a population
as a function of differential reproduction of their bearers.
dioxide. eer
permanently store greenhouse gases, especially carbon

Variation exists within all populations of organisms. This


occurs partly because random mutations cause changes in
9.

ing
(b) 1 is not correct as economic prosperity can raise
incomes and foster better institutions but it cannot forster
sustainable economic growth. 2 is correct. Generating
the genome of an individual organism, and these mutations
can be passed to offspring. Throughout the individuals’
lives, their genomes interact with their environments to .ne
adaptive technologies can lead to a sustainable growth as
discussed in para 2. 3 is correct. As investing on research in

2.
cause variations in traits.
(b) 1 is not correct because the passage does not talk about
all the poor countries.
2 is not correct because the passage talks about the role
10.
climate.
t
adaptation will help us in better handling of the changing

(a) 1 is correct because if rainfed crops are grown in


irrigated areas it would lead to overexploitation of natural
resources. 2 is wrong as farming provides employability
of pesticides in sustainable agriculture especially in poor
countries. as well as food resources. The strategy must be not to
3 is correct as the 2nd para clearly illustrates Alabama overexploit and do limited farming.
leaf-worm developing resistance to aldrin, DDT, dieldrin, 11. (a) 1 is correct as the creative potential of man will provide
endrin, lindane and toxaphene. better adaptability to the changing climate.
3. (d) The widespread use of pesticides has not waned because 2 is wrong as the passage does not talk about intensive
the ratio of cost to benefit for the individual agricultural agriculture leading to an economic back lash. It is our
producer has remained in favour of pesticide use. growth –industrial and overexploitation of resources –
4. (a) Natural populations of pests contain, among their which has lead to such changes.
vast numbers of individual members, considerable 3 is wrong as economic prosperity will enable us in better
variation in their genetic material, primarily as the result handling of the environmental changes.
of mutations. When exposed to pesticides, most pests die 12. (b) The central theme of the passage is clear that adaptation
quickly, but some may have mutations that make them and mitigation should be integrated with development
slightly less susceptible. If the exposure to pests is short, strategies. The author is not against development but a
these individuals will survive the treatment. This selective sustainable development is what he is talking about.

Downloaded From : www.EasyEngineering.net


Downloaded From : www.EasyEngineering.net

436  l  Reading Comprehension

13 . (c) (a) There is a single example given of such a case but (d) Same as in (c). Moreover option (4) where foreign
it is not always true. companies lower costs finds no mention in the
(b) This statement is true in some cases but not always paragraph.
so. 23. (c) The message conveyed in the passage is that it is
(c) It is clearly stated that man is the only reason to important to have a competition law in the country to
convert ocal community compositions into more ensure that both domestic and foreign firms have a level
homogeneous ones. playing flied.
(d) The option (c) is correct in this context. 24. (b) This is the first of the passage. Group farming helps
14. (b) (a) To breed exotic species with local varieties is remove poverty, increases agricultural productivity and
obviously not the purpose. secures the individual from exploitation.
(b) Man intentionally introduces exotic species There is no mention of surplus production. So the other
into new geographical areas for agricultural or options are ruled out.
recreational purposes. This is clearly stated in the 25. (d) By “gender impact” the author implies that women do
passage. not enjoy much power in the transition economies.
(c) (1) is ruled out. So this answer is not correct. This is the most appropriate meaning the others come close
(d) Again (1) is ruled out making the answer incorrect. but are not as accurate.
15. (b) (a) This is not the correct answer. 26. (b) (a) There is compulsion on the transition economies to

ww
(b) This is the dominant idea in the whole passage.
The presence of “physical barriers” have prevented
homogenization.
go in for group farming. Therefore, this is not the correct
answer.
(b) The paragraph is all about the benefits of group

w.E
(c) This is not clearly stated in the passage.
(d) This idea of “physical barriers” is the most
important reasons for the others.
approach to farming. This is the right answer.
(c) Both options correct included.
(d) Both cannot be negated.

asy
16. (a) (a) This obviously the correct answer, because it is
the most practical one. It is humanly possible to
27. (c) In the Western context “deepening of democracy”
means the increased participation of the individual in the
democratic process.

En
smuggle live organisms and to build highways.
(b) It is not easy nor practical to make ecosystems So, the other option (a), (b) and (d) are automatically
negated.
sensitive. So this option is ruled out.
(c) This option is also ruled out for the above said
reason. gin
28. (a) According to the passage Democracy in the Indian
context means the increased participation of communities
based on religious, caste or sectarian identities. That is
(d) It is very difficult and quite unnecessary to ensure
new species do not have an impact on local species. eer
why greater democratization in India does not lead to the
dilution of caste and communal identities of the individual.
So this option is also ruled out.
17. (c) Both the options (1) and (2) are correct.
18. (b) The first paragraph in the passage conveys the message

ing
In the context of this explanation options (b), (c) and
(d) are automatically negated.
29. (c) The involvement of communities in the democratic
that the detractors of democracy are quite vocal about
that fact that a number of non-democratic governments .ne
process in India has led to a silent revolution. The upper
castes held power in earlier days. This power is getting
particularly in East Asia have successfully achieved
economic development.
Options (a), (c) and (d) are automatically ruled out.
19. (d) The second and last paragraphs the fact that freedom
groups
t
slowly, silently and surely transferred to the subaltern

In the context of this argument option (a), (b) and (d) are
ruled out.
and liberty are essential components of development. 30. (c) The third paragraph states that “Profit is not …..
20. (c) The “constitutive” connection between democracy and validity.”
development is political freedom and democrative rights. 31. (a) The sixth paragraph mentions that the concept of profit
21. (a) The effect of FDI or Foreign Direct Investment should maximization and profit motive is largely responsible for
be to induce competition because this is ensured in most the worst mistakes of public policy.
countries worldwide. 32 . (c) The last line of seventh paragraph states that, “There
22. (b) (a) The first option of multinational companies getting is only one definition of business purpose: to create a
accustomed to domestic laws is not mentioned at
customer.”
all. So, this is not the correct answer.
33. (a) The last paragraph states that “..it remained a potential
(b) Foreign companies may establish joint ventures
want until the action of business people converted it into
with domestic or companies may get stronger
effective demand. Only then is there a customer and a
as the parent companies merge overseas. Both
market”.
options are mentioned in the paragraph. So, this
is the correct answer. 34. (c) The adjective ‘vapid’ means tasteless or dull. ‘lacklusture’
(c) Since option (1) is included this is not the right which means dull or lacking in liveliness is the correct
answer. synonym.

Downloaded From : www.EasyEngineering.net


Downloaded From : www.EasyEngineering.net

Reading Comprehension  l 437

35. (c) The least talked about character in the passage is the 53. (c) The manager has found out the food to be good
‘Grandmother’. The author mentions her only once when he and excellent. This does not help up to understand the
compares a cloud’s shape to his grandmother’s four-legged disconfirmation senstivity of the manager.
silver sugar bowl. 54. (d) Scientific beliefs are universal in character is stated as a
36. (c) All statements except for (c) can be inferred from the fact. As mentioned in last part of first paragraph that we can
passage. sometimes deduce that a universal scientific belief is false
37. (d) The last paragraph states that on the top floor, next to but we can never induce that a universal scientific belief is
the infirmary, there was supposedly a dentist. Whenever the true.
teachers got angry, they would threaten to send the naughty 55. (e) Second paragraph last part states that falsification is no
more capable of yielding conclusive rejections of scientific
students to this dentist.
belief than verification is of yielding conclusive acceptances
38. (b) The first paragraph states that people thought that
of scientific beliefs.
the time for the genteel game of knowledge, Kaun Banega
56. (c) The last paragraph finds support for option (c).
Crorepati had passed. It lacked the backbiting intrigue and
57. (a) The last paragraph states that falsifying claims
low-life loquaciousness of other reality shows. Thus we can sometimes gives us a good reason for rejecting a scientific
say that both statements (i) and (ii) are correct. belief.
39.

40. ww
(d) the second paragraph mentions, “but as an idea that
connects with something deep and real in our lives.
(c) The third paragraph states that the prize money of KBC
58. (b)
59. (d) Option (d) is in sync with paragraph 4 “......economics
discipline, find the arid reductionism ..... false to the facts

41 .
w.E
is not a jackpot or means of indulgence, but a reward or gift
from the divine for the winner’s persistent efforts.
(a) The last paragraph states that “As the winners no doubt
and sightly so.”
60. (c) Paragraph (c) is in sync with option (c). Hence the
correct answer is option (c).

42 .
asy
find out, one can never have enough money…”.
(c) The first paragraph mentions,” the warm rains had
been falling...no promise for relief ”. The paragraph ends
61. (e) The correct answer is option (e).
62 . (d) The word deterministic as used in the passage is
connected with effects produced by causes. Hence, the
with “…soggy ground and overflowing water channels only
depressed him.” En correct answer is option (b).
63. (d) The passage speaks of the “mode of governance of the
43 . (b) The second paragraph mentions points a and D. the
third paragraph mentions point C.
The first paragraph states that given the monsoon
gin shop-floor at Toyota of which “scripted operating procedures”
is an important factor. Refer Para 2 “operating procedure” is
designed by the workers jointly with the supervisors. Thus,
weather, sodden flowers and soggy ground, Babur would
find little pleasure in visiting his garden. eer
the procedure can be modified according to the problem
encountered. Thus, the design of the “operating procedure”
44. (d) The last paragraph states, “but he had never thought
that Humayun-so healthy and strong- might succumb to ing
becomes very efficient and results in defect-free, 0 waste and
just in time delivery of the products/models.
64. (e) The passage does not speak of workers’ union, unique

45 .
sickness.”
(c) The words ‘neatly turbaned head’ have been used to
describe the Hakim that came to treat Humayun. .ne
strategies or industrial peace so a, b, c can be ruled out.
The passage lays stress on the involvement of workers in
rectifying/modifying the “operation procedure” model
46.

47.
(d) The last para indicates that sugar mills need not be close
to sugarcane growing areas.
(a) The production process is described in the second half
of the passage, and it can be seen to follow the sequence
which is given in (e).
t
65. (a) The answer is choice (a) for the same reason as above.
66. (b) (d) can be ruled out as the passage nowhere mentions
low investment, (d) can be ruled out as it says that Toyota’s
provided in choice (a). manufacturing processes were not standardised.
48. (d) The passage starts with an account of where sugar is (a) is incorrect because para (2) says that the operating staff
grown – predominantly in Cuba and Puerto Rico, and, work jointly with the supervisors to design the operating
additionally in other places. It can be understood that the procedure.
distribution is not equal. Choice (d) is, therefore, incorrect. (c) is ruled out because para 1 says that operating procedures
The other statements are as per latter half the passage. have to be followed rigidly, without any deviation .
49. (d) Statement (1) is not mentioned anywhere in paragraph. 67. (e) The source of motivation for both examples is not same.
“next birth in human form” is in sync with the idea presented. For (i) it is the moral obligation of the anonymous donor,
Hence, the correct answer is option (d). that the volunteer behaves in such a manner. For (ii) it is the
50. (b) Statements (1) & (2) are correct. self-interest of the organization (as explained in the example
51. (c) The first premise is lower expectation = greater of shopkeeper in the passage) which makes it work in this
satisfaction. Option (c) contradict with this premise. manner.
52
. (d) Statement (2) stated in the paragraph itself and Although both the actions are in conformity with their duty.
statements (2) & (3) are contradictory. Hence, the correct So I is not correct. II is correct as clearly explained above.
answer is option (d). In both cases the reasons were beyond duty.

Downloaded From : www.EasyEngineering.net


Downloaded From : www.EasyEngineering.net

438  l  Reading Comprehension

III is correct as both examples illustrate the concept of moral


80. (a); The dual motives behind Indian abstractionism
worth, which relies entirely on the motivation of the agent. mentioned in the third paragraph are:
68. (a) I is clearly against the passage as explained in the first • Revolutionary, and • Conservative
sentence. It is not the consequence but the motivation Option (a) is correct because it refers to the revolutionary
behind the action. motive, which is unlike the remaining options as they are
II is clearly given in the passage, 6th sentence, ‘These are all about the features of the conservative trend.
also .............true moral worth.’ 81. (c); The final statement of the fifth paragraph talks about
III can also be inferred from the passage as is clear from the the recurrent alterations which were induced in the forms,
example of the charitable person, ‘Kant does not ...........her color and arrangement of the third idiom. This means
moral obligation’. that importance was given to the arrangement of forms.
69. (a) Para 8, sentence 2 “If the genetic properties ......... rose This feature distinguishes it from the other two idioms
or a walrus” portrays creativity and innovation on the part mentioned in the same paragraph.
of human beings, owing to behavorial practies similar to 82. (a); The passage’s fourth paragraph mentions that Indian
that of bacteria. abstractionists have been essentially preoccupied with
70. (a) Para 3, sentence 2 “ The first, but ............theory”. para a supernatural project of reaching out to the occult holy
5, sentence 1 “ In the case of bacteria .........so rapidly” and without completely giving up on the symbolic. The

ww
para 6, sentence 2 “ They freely pass ..............power and
efficiency” mention the three processes responsible for
evolution. 83.
Kandinsky-Klee School was more orphically oriented to
the major causes of abstractionist philosophy.
(d); From the sixth paragraph (final sentence), we can say that

w.E
71. (c) Para 9 implies that these micro-organisms, owing to
efficient communications network, become drug resistant
at such a pace that it becomes difficult for medical science
after seeing themselves as people without the imaginative
energy for negotiating the merger of painterliness and
metaphysics, the Indian abstractionists were left feeling
to catch up.
asy
72. (a) Only option (a) is mentioned in the passage in para 2,
84.
embarrassed. This is referred to in option (d) and hence, it
is our answer.
(b); It is quite evident from the passage that the wines
sentence 2 “ During those two billion.....rapid motion.”
73. (a) Since, the passage talks of macro as well as micro
organisms, we get a holistic world view of the model of En from different countries of the world have been compared.
Satirical refers to criticizing people or ideas in a humorous
creativity in evolution .
74 . (d) Para 4 describes both the mentioned properties of gin way, and that is what the author is representing by stating
that even if French wines are not better than American or
random mutation.
75. (a) Para 8 talks of DNA recombination principle, if applied
eer
Australian ones, they would not admit that – common trait
of not accepting that your product is worse. The author did
not outrightly say that French wines are worse, he was just
to macro-organisms , could become the basis of science
fiction.
76. (c); Option c) is correct because the reason it presents due ing
stating that those makers will not admit that even if it was,
hence the remaining options can be ruled out and Satirical
to which the importance of abstractionism was lost, is not
mentioned in the given passage. On the other hand, the 85.
becomes the choice.

.ne
(a); It can be easily inferred from the given passage that the
labeling strategies led to the success and popularity of the
reasons which the other options refer to, can be found in
the opening paragraph.
77. (b); Option (b) is our answer, because it is clearly and
specifically referred to in the final paragraph. We can see
86.
t
wines from English speaking countries. French must adopt
the same to gain the same popularity.
(b); From the fourth paragraph, we can say that option (b) is
the required answer. The author states that-Consumers are
that the first statement of the final paragraph talks about
no longer intimidated by the thought of needing to know
abstractionism by mentioning that it is a kind of art which
their Pouilly-Fumé from their Pouilly-Fuissé, just at the
is directed towards non-existence of a universal societal
very moment when there is more to know than ever before.
language. The same has been conveyed by the option (b)
Consumers are now keen on reading the specifications of
and therefore, it is our answer.
wine, for example – they would like to know the type of
78. (d); The crisis confronted by the abstractionists has been
grapes the wines belongs to, as wines made from some grape
referred to in the second paragraph. The first three options,
varieties are more popular. And this is the trend which the
i.e. (a), (b) and (c) have characterized the crises confronted
author reckons is troubling French winemakers. Hence,
by abstractionism. Option (d) is our answer because it option (b) is the most appropriate answer.
presents a recipe of all the other options; hence, it is the 87. (d); We can easily state from the final paragraph that option
most appropriate one. (d) is the required answer. One of the findings of Dr. Renaud
79. (b); A sort of self-enquiry (i.e. very thoughtful inner reflection -the fat-derived cholesterol that builds up in the arteries and
which are governed by the nonrepresentation images and can eventually lead to heart trouble, can be dispersed by the
cosmic symbol) came as a result of the abstraction and tannins in wine. And, this tannin is found from the grapes’
hence it was deemed as a revolutionary move. Therefore, skin and is present in high amounts in the red wine. Hence,
option (b) is correct.

Downloaded From : www.EasyEngineering.net


Downloaded From : www.EasyEngineering.net

Reading Comprehension  l 439

98 . (d); Author in this passage is saying how the poor middle


option (d) is our answer as it compares the people who
class depict their sociability. Author stated in the previous
drank red wine and who did not; and the comparison was
paragraph they feel most social when they form a pack
based on whether they developed coronary heart disease or
or howl at people who are better off than they. So, author
not. Option (a) is wrong, because we cannot infer that all
mentions these two incidents to strengthen this argument.
the film celebrities in France drink red wine. Option (b) is Hence, option (d) is our answer. Option (a) just states how
wrong because it goes against the findings. Option (c) may the poor Calcutta suburbs are, but it does not explain why
look like our answer, but (d) is more appropriate as it refers author gave this example and similarly, we can rule out
that the study was specifically aimed at finding this, and this option (d) too. Option (b) is wrong, because we cannot
one was a long-term study too. infer that.
88 . (c); Option c) is correct as the given passage has not 99 . (a); The author states -Greeks, in spite of their proficiency
considered quality as one of the factors. in geometry, never seem to have realized the importance
89 . (c); The notion that GM research is promoted only by MNC of experiments. Hence, option (a) is the most appropriate
can be deemed incorrect from the fourth line of the sixth answer. Option (b) is wrong, because we cannot infer that,
paragraph. and anyway, it pales out in front of option (a). Option (c) is
90 . (c) Opening line of the second paragraph – ‘the anti not our answer because that just gives a possible reason for
GM campaign has been quite effective in Europe’ clearly (a). We cannot say this one is an appropriate answer. Option

91 .
ww
indicates that option (c) is correct.
(b); The final three lines of the final paragraph clearly
indicate that option (b) is our required answer -some
(d) is wrong, because it is contrary to what is mentioned in
the passage.
100. (c); Author mentions that Newton formed the law

w.E
weeds through genetically modified pollen contamination
may acquire resistance to a variety of weed-killers -the
only way to destroy these weeds is through the use of ever-
of attraction, the motion of heavenly bodies became
understandable, and it all seemed to make sense. Hence,
option (c) is our answer; whereas, the remaining options

92 . asy
stronger herbicides, which are poisonous and linger on in
the environment. Hence, option (b) is correct.
(d); It is quite evident that the given passage talks about
cannot be inferred from the given passage.
101. (b); It can be found in the second paragraph of the passage.
Author states that Newton is the greatest scientist of all time

that this food has potential benefits to the poor and En


the concerns of both, the rich and the poor. Author states if account be taken of his joint contributions to mathematics
and physics. Thereby, he was also able to relate physics and
mathematics, and form laws. Therefore, option (b) is our
malnourished and whereas, the anti-GM bodies claim that
the MCN as guided by the profit motive, will only focus on
the high-value food items demanded by the affluent. gin answer.
102. (d); We can understand the theme of the passage regarding
93 . (a); Option (a) is the correct option, because it can be
found in the third paragraph of the passage. Author only eer
modern science. This sentence of the passage -It is most
important to understand this point and to realize that
Einstein’s special principle is merely an extension of the
states that the GM controversy has been ignored by India.
Indian biotechnologists are still working with GM and
have not produced something which is given out to the ing
validity of the classical Newtonian principle to all classes of
phenomena. Hence, option (d) is our answer. Options (a)
and (b) are wrong, because Greeks in fact, never realized
citizen – it is still under the experimentation process. From
this information, we can cite option (a) as our choice. The .ne
the importance, they even just considered geometry as a
form of fine art and did not apply their thoughts to the real

94 .
remaining options cannot be inferred from what is given
in the passage.
(a); Option a) is the correct option because it can be found
in the second paragraph of the passage. We can get what
t
and physical world. Option (c) is wrong, because we cannot
infer that from the given passage.
103. (a); Option a) is the correct option because it can be found
in the fourth paragraph of the passage.
they refers to from how this paragraph ends -These large 104. (b); The author in this poem is talking about a journey to
gatherings will be only what we make of them if not anything Ithaka, and how enriching an experience that could be, and
better, they can be as good places to collect new friend. So, how one should enjoy this journey while it lasts. Option (a)
option (a) is our most appropriate answer. is wrong because author is telling about the ways to deal
95 . (c); The author is trying to say from how he begins the fifth with it. Option (c) is wrong because it just refers to one of
paragraph -It is tragic, however, to observe that it is these the things mentioned in the poem. Option (d) cannot be
very natural springs of social life which are drying up among inferred from the passage, and it fails to capture the essence
us. Hence, option (c) is the most appropriate answer. or purpose of the poem.
96 . (b); Option (b) is the correct option because it can be found 105. (a); It is evident that option (a) is our answer. The author
in the second line of the fourth paragraph -interest, wonder, states -Better if it lasts for years, so you are old by the time
sympathy and love, the first two leading to the last two, are you reach the island, wealthy with all you have gained on the
the psychological prerequisites for social life. way. By wealthy, he meant being rich in experience. Option
97 . (c); After reading the later part of the second paragraph, (b) is wrong, because it states what a part of the journey
we can say that although “recognize” cannot be completely is. Option (c) is not appropriate, because it fails to totally
deemed wrong, yet ‘distinguish’ is the most appropriate grasp what the author meant by wealthy. Option (d) is out
answer. of context of the given passage.

Downloaded From : www.EasyEngineering.net


Downloaded From : www.EasyEngineering.net

440  l  Reading Comprehension

106. (d); The author in the given poem talks about this long 115. (b); It is easily seen that option (b) is our answer. Author
journey which would make one gain knowledge as well as states ‘How much bigger can the airplanes get?’ To show
sensual experiences. So, by that, he refers to the distant goals the increment in the number of passengers the airplanes
– the goals which are far away, but you will learn a lot while can carry; the author gives an example: ‘The change is quite
you are in the process of achieving. Option (c) looks a lot huge: from the four–seater to the A380 airplane.’ Hence,
option (b) is the only correct option.
close, but the journey is not all about gaining wisdom, it is
116. (a); Option (a) is correct as it can be found in the second
also about experiencing different and new things. paragraph. While describing V-22, author says that The
107. (c); The author states that one should not be afraid of V–22, known as tilt–rotor, part helicopter, part airplane,
Laistrygonians and Cyclops, wild Poseidon while we are takes off vertically, then tilts its engine forward for winged
in this journey to Ithaka. It refers to the obstacles we face flight. Hence, option (a) is our answer.
117. (c); Option (c) is correct, as it can be found in the third
while we are on our way to distant goals. Options (a) and
paragraph. Radars are used to make sure that no collision
(b) cannot be inferred from the passage, and they also do takes place in the air, but with the advancements in
not comply with what the author is trying to refer to. Option technology, the computer on board which could even take
(d) could look like an answer, but it refers to the problems place of a pilot, would ensure safe passage. Hence, option
(c) is our answer.

ww
faced in general while on the journey. The author states that
these problems we face are all within us and he states that
we should not be afraid.
118. (c); Option (c) is correct, as it can be found in the next to
last paragraph. Author states two reasons for this: There is
no collective vision of our future such as the one that drove

w.E
108. (b); Option (b) is correct, because exhorting refers to urging
on, and that is what the author is doing by trying to inspire
readers about their distant goals.
us in the past. There is also a need for a more aggressive pool
of airplane design talents to maintain an industry. Hence,
option (c) is our answer.

asy
109. (a); Option (a) is the required answer as third paragraph of
the passage mentions the same: the author states ‘But for
119. (c); Option (c) mentions why some of the scientists found
the book framed on Tsavao lions (the man-eater one), as
annoying. As mentioned in the third paragraph, this book

necessarily a sanctuary. In Corporate America, where the


En
the white–collar kids of blue– collar parents, the office is not
spreads false rumours about Tsavo lions being very ferocious
and the ones who pose a serious threat to humans. Option
rules are based on notions foreign to working–class people,
a Straddler can get lost.’ This implies that the value system gin (a) is not totally wrong, but fails to completely capture
what has been mentioned in the passage. The remaining
that they enter into is totally unknown and alien, hence
option (a) is our answer.
110. (b); Thucydides goes back to the Greek historians, and it can
this book.eer
two options do not reveal why scientists were annoyed by

be inferred that the author likes knowing and reading about


the Greek history, but his father did not. His father was ing
120. (c); Option (c) is our answer, as it does not add to Tsavo lions’
well known image of being savage creatures and man-eaters.
This thought of Tsavo lions being not as evolved was just a
probably some construction worker who would have been
involved in making buildings, i.e. the father was keen on .ne
conclusion of the study conducted by Gnoske and Peterhans
by analyzing the traits of these lions. The remaining options
arches, and the author was not. And this represents option
(b).
111. (d); Options (a), (b) and (c) can be stated from the second
eater image of the Tsavo lions. t
can be ruled out as they do reveal what created this’ man-

121. (c); The opening paragraph mentions that Craig and West
paragraph of the passage, whilst option (d) is not mentioned were experts on Serengeti lions and had never seen Tsavo
anywhere in the passage. lions before. Lions are well known for their manes, but these
112. (d); Option (d) does form a part of the given passage, whilst were not present in Tsavo lions. The concluding sentence
the other options do not. Hence, it is our answer. of the first paragraph is formed as the antecedent of option
113. (b); Option (b) is our answer because it has been mentioned (c) by the opening paragraph. Option (c) is hence correct,
in the final paragraph, while what the other options refer as the remaining options can be ruled out with respect to
to, cannot be found in the given passage. The author states- the context of the given passage.
“Children of the working–class are brought up in a home in 122. (c); Gnoske and Peterhans gave an hypothesis that Tsavo
which conformity, obedience and intolerance for back talk lions share similarities with Pleistocene cave lions in terms
are the norm–the same characteristics that make a good of their body structure. They stated that these lions differ
factory worker.” Hence, option (b) is our answer. from Sarengeti lions because Sarengeti ones are the more
114. (d); We can easily see from the first paragraph that Option evolved ones, whereas the Tsavo and Pleistocene cave lions’
(d) is right because what all the other options refer to can physical structure depicts that they are not that evolved
be deemed incorrect. and they are closer to the primitive ancestor of all lions.

Downloaded From : www.EasyEngineering.net


Downloaded From : www.EasyEngineering.net

Reading Comprehension  l 441

Option (d) can be ruled out, because it refers to what this oil prices’. Hence, option (c) is our answer because it captures
hypothesis is based on. Option (a) cannot be inferred from all these mentioned reasons
the given passage. Option (b) is wrong, because it just 129. (c); From the first, second and third paragraph, we can
supports the hypothesis. Option (c) is right because that infer that option (c) is right. The author points at increased
would then differentiate cave lions from the Tsvao lions, oil prices and the shift in global balance of power is what
hence weakening the hypothesis which puts cave and Tsavo is leading to the Edwardian Summer. Option (a) is wrong
lions in the same category of primitive lions. because the author even states that our historical precedents
123. (c); The third paragraph of the given passage mentions have proven that modern capitalism is resilient. Option (b)
that the existence of psychological dilemma is what makes is not our answer, because it states what could happen, but
internal conflicts more interesting than external conflicts. not the basis on which author’s argument is based. Option
The second paragraph states that the one having internal (d) is wrong, because we cannot infer from the passage that
conflict ‘is torn between a tendency to cooperate, so as to crisis is imminent in the West.
promote the common interests, and a tendency to compete, 130. (d); After reading the second paragraph of the given
so as to enhance his own individual interests’ And this is passage, we can state that option (d) is our answer. The
exactly what option (c) refers to. author mentions in the second paragraph that oil prices can

ww
124. (b); Option (b) would qualify as a psychological dilemma
because the author stated in the second paragraph
‘Psychologically, most interesting situations arise when
be brought down with the formation of a new production
and refining capacity, hence bringing down those increased

w.E
the interests of the players are partly coincident and partly
opposed‘. So, adopting a defensive strategy over an aggressive
opponent would come under the partly opposed category.
oil prices is in our hands and can be done. From that, we
can infer that oil prices can be brought down.
131. (d); The author states in the opening paragraph, ‘At the

categories.
asy
None of the remaining options can be kept in either of these

125. (a); Option (a) is correct, because author states in the


heart of Derrida’s deconstructive approach is his critique
of what he perceives to be the totalitarian impulse of the
Enlightenment pursuit to bring all that exists in the world

En
opening paragraph, The totality of choices determines the
outcomes of the game, and it is assumed that the rank order
under the domain of a representative language, a pursuit
he refers to as logocentrism’. Hence, option (d) is the most
of preferences for the outcomes is different for different
players; according to author, this is what leads to the conflict
of interests. gin appropriate one, as he believes that there should be a
language which can represent all the aspects of the world,
126. (c); From the fourth paragraph, we can state that the
‘alteration’ element is what differentiates the scientific and
the detective processes of problem solving. The author states
eer
which as of now, remain hidden.
132. (c); Author states ‘Logocentrism is the search for a rational

“If the adversary actively puts obstacles in the detective’s


path toward the solution, there is a genuine conflict. But the ing
language that is able to know and represent the world and
all its aspects perfectly and accurately.’ So, it implies that
conflict is psychologically interesting only to the extent that
it contains irrational components such as a tactical error
on the criminal’s part or the detective’s insight into some .ne
this language would represent the world and every aspect
it possesses. This also means that it implies a totalitarian
principle. As of now, we do not have such a language, hence
psychological quirk of the criminal or something of this
sort” So, the fact that the adversary is active in terms that
he leaves indications to mislead a detective, is what makes
answer. t
hidden meanings in texts exist. Therefore, option (c) is our

133. (a); Binary opposition refers to a pair of related concepts


this problem solving process psychologically interesting. which are opposite in meaning like, two theoretical
And this is exactly what option (c) refers to. opposites that are strictly set off against each other for
127. (d); From the opening paragraph, we can infer that option d) example: rational/ irrational. These systems, according to
is true. Edwardian Summer is referred to as ‘a period when Derrida, represent ordering of truth, and this is what option
the dominance of the world’s superpower is coming under (a) refers to. Option (a) can be inferred from the second
threat. It is an epoch when prosperity masks underlying paragraph, and hence it is our answer.
economic strain.’ Hence, option (d) is the only appropriate 134. (a); The author states in the last paragraph that Derrida’s
answer. rejection of the definitive authority idea is based on
128. (c); After reading sixth paragraph, we can say that option his decentralization premise that states that in order
(c) is the required answer for the given question. Author to understand the meaning of a text, equal importance
states that modern capitalism has been very resilient, and he should be given to what is written and what is not stated.
gives examples of that by stating ‘that a sign of the enduring This unstated text can stay invisible if the subject is posited
strength of the system has been the way it apparently definitive authority as in the case of a biographical account.
shrugged off everything -a stock market crash, 9/11, rising Thus, Derrida rejects it.

Downloaded From : www.EasyEngineering.net


Downloaded From : www.EasyEngineering.net

442  l  Reading Comprehension

135. (c); Author states that ‘For the critical attitude is not so much 140. (e); The author in the given passage describes a just society in
opposed to the dogmatic attitude as superimposed upon it: which the principles of justice have no connection with the
criticism must be directed-against existing and influential powers and positions of individuals. It promotes equality in
beliefs in need of critical revision – in other words, dogmatic that way. But as the author is just imagining such a society
beliefs. A critical attitude needs for its raw material, as it and how giving his explanation of how a society should be,
was, theories which are held more or less dogmatically.’ So, we would deem this one as a hypothetical society, one which
dogmatic beliefs, which could be considered as myth, serve comes out of the imagination of the author. Option (a) is
as the raw material for critical thinking. Dogmatic beliefs wrong, because enjoying a privilege is different from having
are what critical thinking is done on. Thereby, dogmatic principles based on justice, so this option only catches a
beliefs serve as a raw material (like natural gas) which is part of what the author meant. Option (b) is incorrect,
then converted in scientific theories (fertilizer) via critical because just agreeing on fair principles is not the point, but
thinking. So, option (c) fits perfectly. its actual implantation and practice is. Option (c) is wrong,
136. (a); Option a) is our answer as it can be inferred from the not completely wrong though, because it differs from option
given passage (third and fourth paragraph). We discussed (e) by just the use of the term hypothetical, but we have
described why this society should be considered so. Option

ww
the same in the previous question that critical thinking
(which is done in science) is done on dogmatic beliefs.
Hence, dogmatic behavior and attitude is very important for
(d) is not our answer because it pales out in front of option
(e).

w.E
science because they lead to scientific theories. Therefore,
option (a) is the most appropriate answer.
137. (c); In the given passage, by giving an example of children
141. (a); First and second lines of the third paragraph indicate
that option (a) is correct. We can also use the explanation
presented in the previous question to pick option a) as our

asy
who have dogmatic nature and of primitives, who possess
the same stubborn quality, the author has tried to justify
answer.
142. (d); The ones who make rules are nearer to the notion
presented in the given passage, because they are devoid of
his argument. Hence, option (c) is our answer because it
En
refers to the same. Children and primitives do not possess
thoughts when it comes to what strength or what position,
or what intelligence they would possess.
the ability to think critically, they just have their dogmatic
beliefs. Option (a) takes the passage away from context,
gin
143. (b); Original agreement has been mentioned in the previous
questions. Social institution can only be fair if they stick to
option (b) is wrong, because being modern or innocent
does not link with the passage in any way. And similarly,
the remaining options make no sense given what the author
eer
these principles of justice and do not sway from them, and
this is what option (b) refers to.

is trying to say in the passage.


138. (d); We can understand from the second paragraph that ing
144. (b); Author in the given passage talks about a just society
which stands for justice and equality: everyone is treated the

conviction is the reason behind dogmatic attitude being


considered as a stronger belief; critical attitude refers to .ne
same irrespective of the the power they have, and in terms of
justice as fairness. Option (b) is right, because an individual
must have some way to survive and get his livelihood, and
continued questioning, and hence does not possess any
conviction. Author mentions that critical thinking is not
based on strong beliefs. It in fact based on testing those
t
hence, they all should be employed and given some job
to ensure that. Option (a) is wrong, because it would be
unfair to equally pay two individuals whose degree of work
beliefs, and finding the flaws points. On the other hand, differed. Option (c) is wrong because punishment for theft
dogmatic beliefs possess a much stronger belief set, and criminal acts is not talked about in the passage, so to have
trust in their thoughts because they do not test or try to test it to present what is mentioned in the passage would be
or falsify them, it has a conviction which critical thinking inappropriate. It also fails to capture the purpose of this
lacks. Hence, option (d) is the most appropriate one. Option passage, of how the author imagined this society. Option (d)
(a) pales out in front of option (d) and similarly, option is not our answer, because all children also refers to those
(b) can be ruled out. Option (c) is wrong, because critical children who have rich parents, so to given free education
thinking never had conviction on the thoughts, so saying would be deemed as privilege which the author opposes.
that it lead to noise which did not have conviction is wrong. Option (e) though not totally wrong, but pales out in front
Option e) is wrong because fails to capture what d) did. of option (b).
139. (c); Pseudo-science is based on dogmatic attitude, whereas 145. (e); The author starts off the passage by stating that human
science is based on critical attitude. We have already biology does nothing to structure human society. He gives
discussed how these two differ, hence option (c) is the most an example of mother by stating that even though giving
appropriate answer. birth is essential to become a mother, yet it is not enough

Downloaded From : www.EasyEngineering.net


Downloaded From : www.EasyEngineering.net

Reading Comprehension  l 443

as she has to behave in a way that aligns with the way a using this, the author tries to explain the relation that exists
mother should be. As the author stated, we have ‘rules that between paradigms and normal science. Hence, option (d)
govern the role of a mother’ and what are these norms and is correct. Remaining options do not form a part of the given
rules? According to the author, that varies from culture to paragraph.
culture. But what is constant everywhere is that the behavior 153. (e); We can derive option (e) from the third-last line of the
is coordinated by the reciprocal nature of roles. For example; given passage. We can rule out option (a), as paradigms
everywhere, there is a way in which a waiter and customer actually aid as far defining scientific traditions is concerned.
behave and act. So, not the human biology here, but the Paradigms never define themselves. We can disregard
reciprocal nature of roles defines the behavior, and this option (b), because the final paragraph has author opposing
feature is seen everywhere, irrespective of culture or human the existing rules, and similarly, option (c) can also be ruled
biology. Therefore, option (e) is the most appropriate one. out. Option (d) lacks connectivity with the given passage
146. (b); The author gives an example of how biological linkages and hence can be deemed as inappropriate.
154. (a) The underlying theme of the passage is that beauty has
structure human society by saying that we would have been
been misunderstood by the modernists and needs to be
offended by the father playing his role ‘tongue in cheek’ saved.

ww
because that does not represent how a father should behave,
that does not play with the role of a father. Hence, this
biological linkage creates the way in which one should act,
155. (d) All options except for option (d) are statements about
the old theory, option (d) is not mentioned, and thus is the
correct answer.

w.E
and if these biological linkages did not exist, we will not be
offended. The other options cannot be inferred from the
given passage.
156. (d) Option (a) is correct as the author all through is
lamenting the loss of beauty’s status.
157. (c) The author eulogizes beauty; option (c) best answer.
147.
asy
(b); The author in the passage infers that people behave
in terms of the roles they have, and thereby, forget the self
158. (d) The author is concerned about the return to capitalism;
option (d) is the best answer.
159. (e) The discussion is about the upheavals the economic

En
and try their best to portray the role they have. The author
says that some roles are more far reaching and absorbing
system is undergoing, this has resulted in large scale changes
at all levels.
than others. For example, a bus driver just has to be able
to drive the bus and do it well to be called a bus driver,
gin
160. (e) Option (e) is the best answer as the length and the style
of wrting point to the fact that it is an article, the content
to the fact that it has to be a subject related magazine.
but a clergymen has to make sure that all his actions point
towards the role he has, and do justice to it, and such role
affects the self even more. Option (b) is the only correct one
eer
161. (a) The author talks about the systems rebounding which
will overlook the mistakes of the bankers.

as the author does mean that the self gets aligned with the
roles. ing
162. (d) The passage is mainly concerned with the anti-Russian
propaganda which s/he feels is detrimental to Russia’s
progress.
148. (b); This answer can be deduced from the three lines that
conclude the passage. .ne
163. (b) The author feels that the criticism works against the
cause of liberalism as the masses are unable to delve deeper
149.

150.
(a); Option (a) can be inferred as our answer from the final
paragraph.
(c); This answer can be ascertained from the second and
into the situation.

t
164. (d) The author laments the damage wreaked by the liberals
and feels that they serve Putin’s cause indirectly making
option (d) correct, that they are so effective in their tirade.
third paragraph. 165. (d) The first paragraph alludes to the life style of the writers
151. (a); Opening line of the paragraph tells us what this passage and this is his take on the writers who are maligning other
is about and hence, option (a) is correct, because the author authors.
explains how rules, paradigms and normal science are 166. (a) The sceptics felt that undermining religion could lead
related. Option (b) is not our answer, because it just refers to doubts in the minds of people leading to chaos making
to an example that the author gives for explaining what is option (a) correct.
mentioned in option (a). Option (c) is incorrect as it does 167. (b) The irony is the going back to religion as modernity
could not keep up the tall claims made by it making option
not refer to normal science, which of course is also what
(b) correct.
the author has talked about in the passage in terms of how
168. (e) The secular system was not able to provide the answers
related it is to rules and paradigms. The remaining two mankind was seeking which led to people seeking religion
options also fail to completely capture what this passage is once again.
about. 169. (b) The main argument during Hegel’s time was that reason
152. (d); Loci of commitment is referred to the commitment, was the most important element for mankind making
or abiding by the way scientific inquires are looked at. By option (b) correct.

Downloaded From : www.EasyEngineering.net


Downloaded From : www.EasyEngineering.net

444  l  Reading Comprehension

170. (d) The author is at best analytical as the matter under Option (e) is incorrect because again there is a feeling of
scrutiny requires a detailed scrutiny. compromise which indicates a not fully positive feeling
171. (e) The second paragraph talks of how people who speak and a sense of remorse. Option (c) is the correct answer
different languages may think differently which makes because the statement shows love for all sections of people
option (e) the correct answer. without any biases and this can only be done when one
172. (b) All options except for option (b) are attributable to the thinks positively and above petty considerations which are
research on languages. the core tenets of positive psychology. The following lines
173. (a) The passage deals with the existence of different in the passage further substantiate this positivism, “in each
languages and thinking processes, option (a) is most similar of these instances, the individual puts aside, or ‘transcends’,
as it is region specific, all the other options are generic. his or her own self-interest for the sake of others.”
174. (a) Option (b) is incorrect because nowhere in the passage, 178. (d) Option ‘a’ is incorrect because there is no reference to
does the author link the love for one’s nation with the love election manifestos in the passage. Option ‘b’ is incorrect
for one’s children. Option (c) is incorrect because the because the reference to politicians answering questions
author states that the term love is also used for love for in the passage is only with regard to politicians belonging
one’s paramour, children, friends and God. Option (d) is not to contemporary western society. Option ‘c’ is incorrect
correct because the author states that patriotism signifies because there is no such reference in the passage. Option

ww
attachment of loyalty to one’s territorial community and
not hatred towards other nations. Option (e) is incorrect
because this belief is not supported anywhere in the passage.
‘d’ is the correct option which is supported by the following
statement in the passage, “But the need for political
philosophy is always there, especially perhaps at moments

w.E
Option (a) is the correct option because the author has
supported it with the following line in the passage, “the
love for, what is understood to be one’s own is a consequence 179.
when we face new political challenges that we cannot deal
with using the conventional wisdom of the day.”
(b) Option (a) is not correct because according to author’s

asy
of the preoccupation with the continuation of the self, both
its biological and cultural components.”
conception, political philosophers do not have a special
kind of knowledge that is not available to normal human
175. (b) Option (a) is not correct because according to the

En
author the concern for the well being of the nation makes
politics possible, it is not the genesis of politics. Option
beings, Option (c) is incorrect because the author says that
political philosophers have circulated their knowledge
through media friendly disciples. Option (d) is not the
(c) is incorrect because it is not supported anywhere in
the passage. Again Option (d) is incorrect because an gin correct option because the opening line of the passage,
“Nobody can tell in advance whether any given work of
individual’s preference for his fellow nationals is not linked
to politics anywhere in the passage. Option (e) is not
correct because one’s patriotic attachment to one’s nation
eer
political thought will have the effect of Hobbes’s Leviathan
or Rousseau’s Social Contract, or to take a later example,

is individualistic and does not involve other people. Option


(b) is the correct choice because the following lines support ing
Marx and Engels’s The Communist Manifesto negates the
option. Option (b) is correct because in the passage the
following substantiates the statement, “we may turn to
the choice, “patriotism implies a commitment to the well-
being of one’s country, it provides the basis for working
out the differences, “The process of working out these
.ne
political philosophy, not perhaps at source, but as filtered
through pamphlets, magazines, newspapers and the like”

differences through compromise is politics.”


176. (e) Options a to d are incorrect because they restrict the
feeling of love to only one’s children, or nation or friends
180.

t
(c) Option (a) is incorrect because the statement is validated
by the following lines in the passage, “every successful
political philosopher has relied on media-friendly disciples
to put his or her ideas into circulation.” Option (b) is not
or God which is contrary to the author’s views. Option (e) correct because as per the passage political philosophers are
is the correct option because it supports the author’s view not endowed with special kind of knowledge as compared
that love is an attachment that involves self denial and is to ordinary people. Option (d) is not correct because the
supported by the following lines from the passage, What statement is validated by the following lines in the passage,
such a wide use of the term indicates is that, in each of these “the philosopher proposes corresponds to political and
instances, the individual puts aside, or ‘transcends’, his or social change in such a way that the new ideas can become
her own self-interest for the sake of others. the commonplaces of the following generations.”
177. (c) Option (a) is not correct because if humans differentiate 181. (a) Option (b) is incorrect because both statements 1 and
between their own and other’s children they are displaying 2 support the given assumption. Statement 1 shows that
negative emotions and negative psychology. Option (b) good governance allows people to do their normal activity
is incorrect because again the differentiation indicates without obstacles and this increases their happiness, while
negative psychology. Option (d) is not correct because statement 2 shows that tyranny which is a result of bad
as per this statement the loyalty towards one’s nation and governance leads to poverty and death. Hence governance is
thereby indirectly to its people is conditional on laws, related to people’s happiness. Option (c) is incorrect because
customs and religion and not natural or from within the statement 3 supports the assumption because Lorenzetti’s
heart that would indicate positive thinking or psychology. murals depicted effects of good and bad governance and

Downloaded From : www.EasyEngineering.net


Downloaded From : www.EasyEngineering.net

Reading Comprehension  l 445

situation doesn’t have any obvious symmetry, so principles


they being advised by the author to be seen to understand
of indifference seem to be of no help”. Hence the given
political philosophy, show that governance affected people’s
statement is contrary to what is stated in the passage. Option
happiness. Option (d) is not correct because it includes
(c) is incorrect because according to the passage there is no
statements 1,2 and 3 all of which support the assumption
scientific data available to show patterns of probability as is
as discussed earlier. Option (a) is the correct answer because
evidenced by the following lines, “nor have you encountered
according to statement 4 none of the given statements 1,2
any particularly relevant statistics about this.” Hence this
and 3 contradict the assumption.
cannot be reason enough to justify that range is a better
182. (c) Option (a) is not correct because there is nothing in the
measure of probability. Option (d) is not correct because
passage to suggest that probability depends on the size of
there is nothing in the passage to suggest that advanced
the sample. Option (b) is incorrect because again the author
computers and techniques can help in calculating range
does not refer to inadequate computers as being the reason better than specific numbers. The following lines support
for not arriving at the correct probability. Option (d) is not this view, “give her all the computers, representational tools,
the correct option because there is no connection between brain upgrades, etc. that you like. Still it seems as though
the degree of belief and probability in the passage. Option the agent would go wrong to have any very precise degree
(c) is the correct option because the following lines in the of belief in the relevant claim.” Option (a) is the correct
passage support this view, “ When evidence is essentially option because the author feels that because of numerous

ww
sharp, it warrants a sharp or exact attitude; when evidence
is essentially fuzzy—as it is most of the time—it warrants
at best a fuzzy attitude. ”
factors the degree of belief of the next happening can never
be precise, hence a range of numbers is a better technique
as supported by the following argument, “an appropriate

w.E
183. (b) Option (a) is incorrect because this statement is
supported in the passage as the author believes that when
one’s evidence for a proposition is sparse and unspecific
response to this evidence would be a degree of confidence
represented not by a single number, but rather by a range
of numbers.”

asy
the answer is not clear as to the probability of that event
happening. Option (c) is not correct because this statement
is supported in the passage by the example in the passage of
186. (d) Option (a) is incorrect because according to the passage
economists were trying to bring back the Global economy
to the pre World War I stage thereby indicating that the

En
contents of a bag and difficulty at arriving at the probability
because the contents of the bag are not known thereby
economy after World War II was worse than what it was
prior to World War I. Option (b) is not correct because
reinforcing the logic that contents of a sample increases
our belief in the probability of any happening. Option (d)
is not the correct option because the following lines in the
gin Keynes believed in open market economy as suggested by
the following lines in the passage, “Secretary of State Cordell
Hull, like Keynes, had for decades believed that an open
passage, the situation doesn’t have any obvious symmetries,
so principles of indifference seem to be of no help. Should eer
world economy would tend towards peace and prosperity.
” Option (c) is not the correct option because it was Dexter
your probability be 54%? 91%? 18%? support the logic of the
given statement. Option (b) is the correct option because
according to the author the following lines in the passage, ing
and not Keynes who advocated the idea of a contributory
fund. Option (d) is the correct option because the following
lines support this view, “the Roosevelt administration had
“Give her all the computers, representational tools, brain
upgrades, etc. that you like. Still it seems as though the agent .ne
insisted on a contributory basis for Social Security”.
187. (a) Option (b) is incorrect because there is nothing in the
would go wrong to have any very precise degree of belief
in the relevant claim” do not support the statement that
reliance on machines increases our belief in the probability
of any happening.
t
passage which shows that White’s plan was accepted because
it would allow free trade between countries. Option (c) is
not correct because the passage does not contain anything
that shows that the acceptance of White’s plan was a result
of any country having more decision making powers than
184. (d) Option (a) is incorrect because nowhere in the passage
has the author linked real life examples with ways to arrive at others. Option (d) is not the correct option because we
the correct probability. Option (b) is not the correct option cannot conclude from the passage that the Social Security
because there is only a passing reference to Keynes theory Scheme was very successful in America. Option (a) is the
and the author does not seem to focus the entire passage correct option because the following line in the passage
on it. Option (c) is not correct because there is nothing supports this argument, “a contributory scheme would limit
in the passage to suggest that the author’s prime concern claims and satisfy Congress”.
is to show the importance of computers in arriving at the 188. (d) Option (a) is not correct because the first line says that
correct probability. Option (d) is the correct option because “As the New Deal wound down its ambitious domestic
throughout the passage the author tries to convey through program the Roosevelt administration began looking
examples and otherwise that nature of evidence is the most outward again.” indicating that the policy was not successful
crucial basis on which the correct result of probability and thus had to come to an end. Option (b) is incorrect
depends. because statement 2 says that the policy was not successful
185. (a) Option (b) is not correct because according to the which is supported in the passage but statement 3 says that
passage there is no symmetry in the available evidence the policy was casual and there is nothing in the passage to
as is evident from the following lines in the passage, “The show that the policy was casual or otherwise. Option (c) is

Downloaded From : www.EasyEngineering.net


Downloaded From : www.EasyEngineering.net

446  l  Reading Comprehension

incorrect because there is nothing in the passage to support groups. Consequently the end of inequality will depend
statement 4 that the policy was directionless. on significant organizational change.” From this, we can
Option d is the correct option because according to the conclude that if women had more say in institutions, they
statement 2, the policy was not successful, and was being can counter male dominance.
wound up which is indicated by the first line and as per 192. (d) Option (a) is not the correct option because the author
statement 5 the policy did not support free trade which just feels that this could be one of the parties responsible.
is supported by the following words in the first line, “the Option (b) is again incorrect for the same reason. Option
Roosevelt administration began looking outward again.” (c) is not correct because this is not reflected anywhere
189. (c) Option (a) is not correct because the first paragraph in the passage. Option (d) is the correct option because
suggests that the pre-war economy was an open one. This there is mention of the possibility of all three factors in the
is reflected in the views of Keynes and Hull who, had for passage which shows that the author believes that these are
decades believed that an open world economy would tend the possible reasons for white middle class male dominance
towards peace and prosperity. Again the following line in in British Society.
193. (b) Option (a) is incorrect because the given statement is
the first paragraph, “that international cooperation might
substantiated by the following lines in the passage, “But
restore the global economy of the era before World War
opinion polls suggest that greater prosperity has not
I”, shows that during the prewar days countries were

ww
cooperating and exchanging economic systems. Option
(b) is incorrect because this statement reflects the plan
advocated by Harry Dexter White at a conference held
brought greater happiness for many Britons.” Option (c) is
not correct because as per the passage, “The way society is
organized and the divisions within it reflect past political

w.E
in the post war era. Option (d) is not the correct option
because this again was an idea advocated by Keynes in the
post war era. Option (c) is the correct option because the
decisions.” which validates the given statement. Option
(d) is not the correct option, because as per the passage,
“In British society, middle class, white men are the ones
who do best in terms of education, health care and life

asy
following line, ‘prevent the piling up of credit and debit
balances without limit’—after all, the cardinal rule for
the postwar economy would be to avoid reproducing the
chances.” which shows that all ethnic groups do not prosper
equally and this is the view expressed in the given sentence.
Option (b) is the correct option because of all the three
prewar economy,” indicates that in the pre-war economy
governments were drawing from unlimited credit limits.
En explanations given for prosperity of middle class white men,
the one suggested by the given sentence does not exist in
190. (a) Option (b) is incorrect because if this statement was true
that would indicate and strengthen the belief that only the
men were successful. Option (c) is not correct because again gin the passage.
194. (c) Option (a) is not correct because there is nothing in the
if this was true it would show that since middle class men
were successful only their children could pursue higher eer
first paragraph to suggest that by the 1970s Government
offices had replaced factories in Britain. Option (b) is
incorrect because again there is nothing in the first paragraph
studies. Option (d) is incorrect because if there was need for
reservation for jobs for women it showed that they were not
as successful as men and hence had to be uplifted. Option ing
that establishes this statement. Option (d) is not the correct
option because the writer does not establish that Britain
became a classless society. In the first paragraph it only
(a) is the correct option because if students from working
class background were more than students from middle
.ne
mentions that , “by the 1950s politicians were talking about
Britain being a classless society” Option (c) is the correct
class background it would show that successful men did not
belong only to the middle class and hence would weaken
Marsh’s argument.
191. (c) Option (a) is incorrect because the statement that if
t
option because the following lines in the first paragraph
support this view, “It is clear that by 2001 the majority of
people worked in white collar or what some people would
see as middle class occupations.”
luck favours women more than men, male dominance 195. (d) Option (a) is not the correct option because the
would end is not supported in the passage. The author is following line in the passage supports this statement,
of the view that it could be possible that because of luck “failures in mobility are the consequence of ability rather
men are more successful and then himself says that this than class structures.” Option (b) is incorrect because the
is a weak explanation. Option (b) is not correct because following line, “the difference between the life expectancy
while statement 3 is supported in the passage, statement 2 of the poorest and the richest has become greater” shows
in itself is not conclusive and not the only reason that could that economic inequality has increased in spite of people
end male dominance. Option (d) is not the correct option moving up from the working class to the middle class.
because both statements 2 and 5 are not supported by the Option (c) is not correct because the following line, “The
passage as being the only means to end male dominance. shift from manual to non-manual work does not mean that
Option (c) is the correct option because the following lines the working class has become middle class” supports the
support statement 3, “women and ethnic minorities will given statement. Option (d) is the correct option because
start to reduce levels of inequality once they have equal all the other options are supported in the passage and none
education opportunities and then choose to compete with of the options are supported in the passage.
men.” Statement 5 is also supported by the following 196. (b) Option (a) is not correct because George Orwell’s
line,“They can protect or promote certain interests or opinion is just taken by the writer as a backdrop to show

Downloaded From : www.EasyEngineering.net


Downloaded From : www.EasyEngineering.net

Reading Comprehension  l 447

changes in British class structure over the years. It is not a a flat whereas in reality he may not be owning a flat. This
message. Option (c) is incorrect because politics and class view supports the given statement.
are being discussed in the passage just as being views of 200. (b) Option (a) is not correct because although people did
sociologists and politicians. Option (d) is not the correct spend more on food there is nothing in the passage to
option because as per the passage quality of health depends suggest that their living standard remained poor because
on one’s economic status and not on one’s class and hence of this. Option (c) is incorrect because there is nothing
this statement cannot be a message from the writer. Option in the passage that shows that people spent most of their
(b) is the correct option because throughout the passage income on housing. Option (d) is not the correct option
the writer makes us feel that by changing work one may because according to the author there was a pension law for
not necessarily move from one class to another. There are urban people in Russia and hence people need not have to
other factors like health which also has to be taken into save too much for their old age. Option (b) is the correct
account while determining one’s class. Also people doing option because according to the author, people in Russia
non-manual labour may not necessarily belong to a higher were starved for consumer goods and their saving rate also
class. increased because there were no consumer goods on which
197. (a) Option (b) is not correct because people working in they could spend. As a result their living standard did not
call centres or leisure facilities instead of mines and steel improve.

ww
factories are not an indication of their economic status.
At best it shows the change in working conditions or
environment. Hence it does not have a bearing on economic
201. (a) Option (b) is incorrect because investment in Television
industry could not be a socialist decision since Televisions
are consumer goods which can be afforded by the more

w.E
equality or inequality. Option (c) is incorrect because again
working in middle class occupations does not necessarily
mean that people are of equal or unequal economic status.
Option (d) is not the correct option because this statement
affluent in Society. Option (c) is not correct because
construction activity took place during the Khrushchev
era and not in the post Khrushchev era. Option (d) is not

asy
at best shows that everyone is more prosperous than before
but who is more prosperous and who is less prosperous
the correct option because there is nothing in the passage
to suggest that there was a fall in the level of self enrichment
and unearned income during the era after Khrushchev.
cannot be concluded from this statement. Option (a) is

En
the correct option because as per the passage, in British
Society, health is an indicator of economic status and also
Option (a) is the correct option because the following line
in the passage, “Above all, it maintained its social contract
reflects the difference between the rich and the poor, as is
supported by the following lines in the passage, “Those on gin by keeping prices low for the basics.” shows that to maintain
socialism prices of essential commodities were kept in
check.
lower incomes are more likely to be overweight, unhealthy
and to die earlier. Indeed the inequalities between the poor
and rich have become greater in terms of health.”
202.
eer
(b) Option (a) is incorrect because India’s growth story
is just the backdrop of the passage and not the primary
198. (c) Option (a) is incorrect because the Soviet regime never
made loud claims about the Pension Law and hence there ing
objective. Option (c) is incorrect because this statement
comes towards the end of the passage and is not a part of
the main passage. Option (d) is again not the main objective
was no rhetoric about it. Option (b) is not correct because
again for the same reason as a there was no rhetoric about
the housing campaign. Option (d) is not the correct option .ne
but a part of discussion on the main objective. Option (b)
is the correct option because the main objective of the
because the author talks about rise in consumer goods only
in the post Khrushchev era while the word rhetoric has been
used for the period during Khrushchev’s regime. Option
(c) is the correct option because the following line in the 203.
that are responsible for this.
t
author is to analyze the impact of any country’s economic
performance on its interlinked countries and the factors

(d) Option (a) is incorrect because there is insufficient


passage indicates that Soviets were repeatedly talking about data available to show the impact of India’s growth on its
their prosperity, “the Soviet system was trapped in the story neighbours and hence it is not possible to conclude that
it insisted on telling of ever increasing prosperity.” India’s neighbours are financially independent of India.
199. (d) Option (a) is not correct because the statement is not Option (b) is not correct because according to the passage
supported in the passage according to which “the regime exports from India have increased substantially thereby
remained hostile to market activity,” Option (b) is not the imports of other countries from India should increase.
correct because according to the passage the production Option (c) is incorrect because the following line in the
of animal fats was greater in America which is supported passage negates this argument, “Demand for imports,
by the following statement (even if supremacy was what it inputs and final products affect partner countries’ supply
continued to enjoy) and “it” referring to America. Option of exports”. Option (d) is the correct option because the
(c) is not correct because according to the passage rate following line in the passage supports this argument, “given
of savings increased because of rise in incomes which is India’s rapid growth and the size of its economy relative to
contrary to the given statement and this does not support those of its South Asian neighbors, a strong positive spillover
the given statement. Option (d) is the correct option because effect could manifest in higher growth for countries in the
according to the passage every Russian had a right to own region”

Downloaded From : www.EasyEngineering.net


Downloaded From : www.EasyEngineering.net

448  l  Reading Comprehension

204. (c) Option (a) is incorrect because although statement 5 on armament had nothing to do with the Cold War. Option
is correct but it is not the only correct choice. Option (b) (d) is the correct answer because the following lines in the
is not correct because although statement 3 supports the passage support this argument, “the Korean fighting lead to
argument but statement 1 points towards a negative growth. an intensification and geographical expansion of the Cold
Option (d) is incorrect because again although option 4 is War, threaten a wider conflict between the United States
correct but statement 2 points towards negative growth and and the communist powers.”
hence is incorrect. Option (c) is the correct option because 209. (a) Option (b) is incorrect because it was true that the spread
all three statements 3,4 and 5 show factors that contribute of communism was halted due to the Korean War as is
to positive growth of interlinked economies. evidenced by the following lines in the passage, “America’s
205. (a) Option (b) is not the correct option because according ‘real commitment to contain Communism everywhere
to the author, access to education and health services in originated in the events surrounding the Korean War’
more developed economies could contribute to human Hence the given statement is validated. Option (c) is not
capital development and skills accumulation. He does not correct because the following lines show that there was
relate it with investment and consumption patterns of any speculation that the Korean Civil War could culminate
country. Option (c) is incorrect because in the passage, in a world war, “that a civil war in Korea would provide
deregulation of an economy is mentioned in the Indian the critical turning point in the postwar Soviet-American
context and not linked to investment and consumption

ww
decisions. Option (d) is not correct because changes in
economies of integrated partner countries may affect a
relationship, and raise the possibility of world war, seems,
in retrospect, nothing short of bizarre’. Option (d) is not the
correct option because the given statement is validated by

w.E
country but changes in economics of any unrelated country
may have no impact on a country. Option (a) is the correct
option because the following lines in the passage support
the given assumption, “Increased demand for imports from
the following lines, “the North Korean attack, sanctioned
and backed by the Soviet Union and China, threatened
America’s credibility as a regional and global power every
bit” Option (a) is the correct option because as per the

asy
partner countries impacts positively domestic production,
while higher partner countries’ exports could worsen
passage it was the Korean War and not World War II which
resulted in the United States emerging as a military political

of domestically produced competing goods, which could


feedback into investment and consumption decisions.” En
current account dynamics, and lower the competitiveness power. Hence the given statement is not supported in the
passage.

206. (d) Option (a) is not correct because increased American


defence spending would not change the course of gin
210. (c) Option (a) is not the entire reason for disputes in India.
Option (b) is about nationalism and nowhere in the passage
is nationalism linked to disputes in India. Option (c) is the
international history. Option (b) is incorrect because
more countries being included in the cold war would not
necessarily change the course of history. Option (c) is not eer
correct option as is evident from the following statement in
the passage, “Some members of that nation have a narrow,

the correct option because again it is not a key factor in


altering history. Option (d) is the correct option because ing
intolerant view of their country by insisting that it should
have only one religion, Hinduism; while others think that
there should be freedom of religion such that Muslims,
the change in ideology of Americans and Russians together
with militarization would have a very big impact on the post
war happenings in the world. .ne
Sikhs, and Christians are rightly members of the nation.
Option (d) is again not correct because disputes do not arise

207. (c) Option (a) is incorrect because there is no linking in


the passage of Korea’s size with its unification plan. Option
(b) is incorrect because according to the author in spite
of their being overburdened the Americans and Russians
211.
because of intolerance to immigrants.

t
(b) Option (a) is incorrect because the focus of the passage
is on the significance of nation in human affairs and not
just the nation. Option (c) is incorrect because of the word
always whereas in the passage the author refers to the
agreed to unify Korea at the earliest. Option (d) is not the word ‘often’. Option (d) is not correct because nationalism
correct option because the following lines “neither north refers to a set of beliefs about the nation. Option (b) is the
nor South Koreans could accept a permanent division of correct option as is supported by the following statement
their homeland.” shows that this was not the reason for in the passage, “the nation demands unquestioned and
the unification proposal to fall through. Option (c) is the uncompromising loyalty. When such a belief about the
correct option because the following lines in the passage, nation becomes predominant, it can threaten individual
“But that plan soon fell victim to larger Cold War tensions liberty.”
that militated against any meaningful cooperation, or 212. (b) Option (a) is incorrect because the passage is not only
compromise, between Moscow and Washington.” Also, about nationalism. Option (c) is not correct because the
support the reason for the proposal falling through. entire passage does not refer to consequences of nationalism.
208. (d) Option (a) is not correct because there is no connection Option (d) is not the correct option because liberty and
in the passage between America’s military might and the nationalism are not pre dominant in the entire passage.
Cold War. Option (b) is incorrect because the hostility Option (b) is the correct option because the entire passage
between the East and West increased due to the Korean is based on the central idea of a nation and the consequences
war. Option (c) is not correct because America’s spending of nationalism on it.

Downloaded From : www.EasyEngineering.net


Downloaded From : www.EasyEngineering.net

Reading Comprehension  l 449

213. (d) Option (a) is not correct because according to the historian Jules Michelet”. Option (d) is not the correct
passage, activities in which parental lineage does not matter, option because although statement 3 is not validated in the
result in uniting people. Option (b) is incorrect because passage but it is not the only statement not to be validated
again activities in which place of birth does not matter by the passage. Option (b) is the correct option because all
unites people. Option (c) is also not the correct option three statements 1, 2 and 3 are not validated in the passage.
because according to the passage, activities in which the Statement 1 is not validated because according to the
language spoken does not matter are not divisive in nature. passage, Michelet condemned aristocracy as the following
The following lines in the passage support these arguments;
lines show, “He was a progressive republican, vociferous in
“human beings exhibit another tendency, when they engage
his condemnation of both the aristocracy and the church”.
in activities in which it seems not to matter who were their
Statement 2 is not validated because freedom and dignity
parents, where they were born, or what language they speak.
were denied to people during Michelet’s time as is shown by
These activities, rather than asserting divisions within
humanity, bring people together.” According to these lines the following lines, “espousing the values of freedom, and
Option (d) is the correct option because activities in which enshrining the spirit of freedom and the dignity of ‘man’.
neither parents, place of birth or language spoken matter, Disappointed in the failure of these values in his own time,
nor the activities that do not divide people. Michelet…” Statement 3 is not validated because according

ww
214. (c) Option (a) is not the correct option because people not
recognizing the term Renaissance does not imply that there
was no Renaissance. Option (b) is incorrect because not
to the passage, “Michelet was the first thinker to define the
Renaissance as a decisive historical period in European
culture that represented a crucial break with the Middle

w.E
understanding Renaissance does not necessarily mean that
they would not recognize Renaissance. Option (d) is not the
correct option because people may recognize a thing even
Ages,” which shows that Michelet did not link Renaissance
with the middle ages.
218. (a) Option (c) can be rejected as it is out of the context.

asy
if they do not discuss it. Option (c) is the correct option
because the author is referring to the recognition of the term
Renaissance in the opening sentence and since the term was
Option (d) is also not true as they like to invest abroad than
in India. Option (e) is true in parts. Only option which is in
sync with major portion of passage is (a).

it.
En
not coined until then, people would not be knowing about 219. (a) Other options are not mentioned in the passage.
220. (c) Last part of the passage chiefly describes it. Other options
215. (d) Options a, b, c are incorrect because the following lines
show that these were the arguments offered by Michelet to
support the view that Italy was not part of Renaissance, “ he gin show these sectors in positive light which is against passage
content.
also rejected what he saw as 14th-century Italy’s admiration
for church and political tyranny as deeply undemocratic, eer
221. (b) Option (b) is true in its completeness, encompassing the
whole of the passage. But other options are true in parts only.
and hence not part of the spirit of the Renaissance.” Option
(d) is the correct option because there is nothing in the
passage to show that Michelet used this argument to show ing
Option (e) is false in parts.
222. (c) Both (1) and (3) are directly mentioned in passage.

that Renaissance did not start in Italy.


216. (d) Option (a) is not correct because there is no reference in
Statement 2 is just not true.
223. (c) Meaning of Clout is force.
.ne
the passage to the Church becoming more powerful during
Renaissance. Option (b) is incorrect because according to
the passage, “Italian word rinascita (‘rebirth’) was used in
the 16th century to refer to the revival of classical culture”
is that cannot be distinguished or percepted.
t
224. (e) Marked means distinguished and different. Imperceptible

225. (d) Option (d) can be related to the experts as mentioned in


the passage.
It is not related to Michelet’s idea of Renaissance. Option
226. (d) Option (d) has plenty of illustrations, references and
(c) is incorrect because there is nothing in the passage to
structure to support it in the passage. Other options like (e)
suggest that egalitarianism became pre-dominant during
and (b) are true but not the central theme of the passage.
Renaissance. Option (d) is the correct option because in
Option (a) and (c) are not true.
Michelet’s view it was the human face together with values
such as reason, truth, art and beauty that received the 227. (d) PROMOTES means to advocate a particular cause.
maximum attention during Renaissance. 228. (e) A rise from the 4% level to 7 % says that there is rise in
217. (b) Option (a) is not correct because although statements 1 education in Egypt.
and 2 are not validated by the passage but they are not the 229. (a) All of the options are mentioned in one or other part of
only two statements that are not validated. the passage.
Option c is incorrect because the statement 4 is validated
in the passage as is evident by the following lines, “But 230. (c) First line of the passage is self explanatory.
the specific French word ‘Renaissance’ was not used as a 231. (b) Only option which can be linked with the passage is 1. Their
descriptive historical phrase until the middle of the 19th people will agitate for greater political freedom, culminating
century. The first person to use the term was the French in a shift to a more democratic form of government.

Downloaded From : www.EasyEngineering.net


Downloaded From : www.EasyEngineering.net

450  l  Reading Comprehension

232. (c) Option (a) is wrong as this is not intended from the use 278. (d) In paragraph 2 of the passage, it is mentioned that bank
of this phrase. Last part of passage has this phrase and poor- branch timings rarely coincide with off work hours of labour
rich divide has been discussed their in voting pattern or why class.
rich people even educated do not vote. Option (d) is out of 279. (a) in paragraph 1, the last 5 lines explain this point. There
the context. Option E can also be rejected as corruption is is no mention of the other points in the passage. Hence, 1 is
not the issue here.
the correct answer.
233. (e) All are true. 280. (c) paragraph 1 , line 3rd to 6th explain this aspect.
234. (c) Only (a) and (c). 281. (e) all the points are covered in paragraph 1.
235. (d) There is a chance that in 2012 the economy would be
282. (a) the role of government is nowhere mentioned, the other
better than what has been forecast.
2 points are covered, hence (a) is the correct answer
236. (d) The current Economic Scenario.
283. (e) the passage explains only points A and C, hence (e) is
237. (a) Only (b).
the correct answer.
238. (a) These will bring about only minor growth.
284. ‘(d) multitude means ‘a large number of people or things,
239. (e) None is true.
hence plenty (d) is correct

ww
240. (e) All (a), (b) and (c).
241. (a) The meaning of word Draw (verb) as used in the passage
is : Influence.
285. (c) ubiquitous means-‘present, appearing, or found
everywhere’, hence (c) omnipresent is the correct answer

w.E
Hence, the words Draw and Entice are synonymous.
242. (a) The meaning of clock (noun) as used in the passage is :
286. (e) dormant means inactive, hence opposite would be active
(e)
time keeping device.

asy
Hence, the words Clock and Watch are synonymous.
243. (d) The meaning of Abate (verb) as used in the passage is : to
287. (c) delayed means late, so opposite would be to rush or hurry,
hence expedited (c) is correct
288. (c) as per paragraph 1 (a) and (c) are correct, hence correct
become less.
Hence, the words Abate and Lessen are synonymous. En answer is (c)

244. (b) The meaning of emerging (verb) as used in the passage


is : come into sight. gin
289. (c) paragraph1 says (in line 13-14) that it is a myth... hence
answer is (c)
290. (c) paragraph 1 – last few lines
Hence, the words Emerging and Developing are synonymous.
245. (c) The meaning of Myraid (adjective) as used in the passage eer
291. (a) paragraph 2 , the 4th and 5th line explains this point.

is : numerous.
Hence, the words myraid and few are antonymous.
292. (b)

ing
293. (a) the passage does not talk of point B, hence A and C is
correct (a)
246. (a) The meaning of Tepid (Adjective) as used in the passage
is warm.
.ne
294. (a) means indicate, hence (a) suspected is correct
295. (e) outweigh means to be more significant- hence it means
Hence, the words tepid and moderate are antonymous.
247. (a) The meaning of Myth as used in the passage is : fictional.
248. (c)
252. (e)
249. (e)
253. (a)
250. (d)
254. (c)
251. (a)
255. (c)
surpass

(d)
t
296. (d) embrace means to accept, hence opposite would be reject

256. (e) 257. (d) 258. (b) 259. (a) 297. (a) compliance
260. (c) 261. (b) 262. (c) 263. (e) 298. (c) The passage is about the management courses
264. (a) 299. (d) Look at the sentence : The real problem is that course
265. (c) The ice fields are melting down and shrinking due to
promoters view development as something which primarily,
global warming, which is caused by emission of carbon
takes place in a class room.
dioxide in the atmosphere.
300. (a) Critical
266. (b) It is necessary to check and control the cause of global
warming. 301. (d) Unprogressive
267. (d) 268. (b) 269. (a) 302. (b) a (fixed) attitude of mind
270. (c) It will be catastrophic for low-lying island states and 303. (b) self-discipline
countries with large coastal populations. 304. (c) must change himself
271. (c) 272. (d) 273. (a) 274. (b) 305. (c)
275. (d) 276. (a) 277. (e) 306. (c) Man well one day die and become dust

Downloaded From : www.EasyEngineering.net


Downloaded From : www.EasyEngineering.net

Reading Comprehension  l 451

307. (a) A sum of Individuals 325. (b) Nehru objected to the phrase ‘conquest of Everest’ since
308. (a) it sounds pompous and boastful.
309. (c) 326. (c) Gandhiji’s statement ‘It is decreasing in the jungles but
310. (b) Baltimore Oriole is of 7 inches in length. it is increasing in the towns!’ refers to man’s selfishness.
311. (c) 312. (c) 313. (d) 314. (a) 327. (d) The writer is of opinion that tribal people can be prevented
from combing forest or food to provide employment and
315. (c) 316. (d) 317. (d)
purchasing power for daily necessities.
318. (b) The writer felt unusually solitary because he was missing
328. (c) 329. (d) 330. (a) 331. (d) 332. (d) 333. (a)
the company of other holiday makers.
334. (a) Achilles heel (Noun) = a weak point or fault in somebody’s
319. (c) “I left all signs of habitation behind me”
character which can be attacked by other people.
This means that he had come very far from places where
335. (b) Sustain (Verb) = to make something continue for sometime
people lived.
without becoming less; maintain.
320. (b) It became darker than the writer expected because the
Look at the sentence:
nights are longer in October than midsummer.
She managed to sustain everyone’s interest until the end of
321. (d) The writer found it difficult to keep to the path because her speech.

ww
of the poor visibility and dew on grass.
322. (d) When he settled himself on the fork of the tree the writer
tried to sleep but without much success.
336. (c) Cumbersome (Adjective) = large and heavy; difficult to
carry; bulky; complex; complicated.
Look at the sentence:

w.E
323. (d) At the beginning of the passage the writer expresses her
opinion that in many countries progress is synonymous with
utmost cruelty to nature.
Government should ease the cumbersome legal procedures.
337. (d) Differentiated (Adjective)

‘sarcasm’. asy
324. (b) In the passage the term ‘exploiting’ nature suggests
= distinguished; to be the particular thing; important.
Look at the sentence:
I think grey hair makes you look very differentiated.

En
gin
eer
ing
.ne
t

Downloaded From : www.EasyEngineering.net

You might also like